Exam 1 Practice Questions

¡Supera tus tareas y exámenes ahora con Quizwiz!

The nurse is teaching a patient how to self-administer ipratropium (Atrovent) via a metered dose inhaler (MDI). Which instruction given by the nurse is most appropriate to help the patient learn the proper inhalation technique? "Avoid shaking the inhaler before use." "Breathe out slowly before positioning the inhaler." "Using a spacer should be avoided for this type of medication." "After taking a puff, hold the breath for 30 seconds before exhaling."

"Breathe out slowly before positioning the inhaler." It is important to breathe out slowly before positioning the inhaler. This allows the patient to take a deeper breath while inhaling the medication, thus enhancing the effectiveness of the dose. The inhaler should be shaken well. A spacer may be used. Holding the breath after the inhalation of medication helps keep the medication in the lungs, but 30 seconds will not be possible for a patient with COPD.

A nurse is caring for a client who has peripheral vascular disease (PVD) and ulcers on the toes. Which of the following findings of PVD is a risk factor for ulceration of the extremities? A. Insufficient skin care B. Dehydration C. Immobility D. Impaired circulation

D. Impaired circulation

A nurse is reviewing laboratory results for an adult client who has sickle cell anemia and a history of receiving blood transfusions. For which of the following complications should the nurse monitor? A. Hypokalemia B. Lead poisoning C. Hypercalcemia D. Iron toxicity

D. Iron toxicity RATIONALE: A client who has received several blood transfusions is at risk of hemosiderosis, which is excess storage of iron in the body. Excessive iron can come from overuse of supplements or from receiving frequent blood transfusions -A- hyperkalemia is a risk, not hypo -C- hypocalcemia is a risk because the citrate in the transfused blood bonds with calcium (not hyper)

A patient with consistent dietary intake who loses 1 kg of weight in 1 day has lost _________________ mL of fluid.

1000

The patient with leukemia has acute disseminated intravascular coagulation (DIC) and is bleeding. What diagnostic findings should the nurse expect to find? A. Elevated D-dimers B. Elevated fibrinogen C. Reduced prothrombin time (PT) D. Reduced fibrin degradation products (FDPs)

A The D-dimer is a specific marker for the degree of fibrinolysis and is elevated with DIC. FDP is elevated as the breakdown products from fibrinogen and fibrin are formed. Fibrinogen and platelets are reduced. PT, PTT, aPTT, and thrombin time are all prolonged.

A patient with a recent diagnosis of heart failure has been prescribed furosemide (Lasix) in an effort to physiologically do what for the patient? A Reduce preload. B Decrease afterload. C Increase contractility. D Promote vasodilation.

A Reduce preload. Diuretics such as furosemide are used in the treatment of HF to mobilize edematous fluid, reduce pulmonary venous pressure, and reduce preload. They do not directly influence afterload, contractility, or vessel tone.

A nurse is providing discharge instructions to a client who has a new laryngectomy. The nurse should tell the client to be careful while bathing to prevent which of the following complications? A. Aspiration of water B. Infection of the stoma C. Bleeding around the stoma D. Skin breakdown around the stoma

A. Aspiration of water RATIONALE: The client should use a shower shield over the stoma when bathing or showering to keep water out of the airway

The patient diagnosed with anemia had laboratory tests done. Which results indicate a lack of nutrients needed to produce new red blood cells (RBCs)? (Select all that apply.) A. Increased homocysteine B. Decreased reticulocyte count C. Decreased cobalamin (vitamin B12) D. Increased methylmalonic acid (MMA) E. Elevated erythrocyte sedimentation rate (ESR)

A, C, D Increased homocysteine and MMA along with decreased cobalamin (vitamin B12) indicate cobalamin deficiency, which is a nutrient needed for RBC production. Decreased reticulocytes indicate low bone marrow activity in producing RBCs, not available nutrients. Elevated ESR is related to an increased inflammatory process, not anemia.

When preparing to defibrillate a patient. In which order will the nurse perform the following steps?(Put a comma and a space between each answer choice [A, B, C, D, E].) a. Turn the defibrillator on. b. Deliver the electrical charge. c. Select the appropriate energy level. d. Place the paddles on the patients chest. e. Check the location of other staff and call out all clear.

A, C, D, E, B This order will result in rapid defibrillation without endangering hospital staff.

A patient with cancer is having chemotherapy treatments and has now developed neutropenia. What care should the nurse expect to provide and teach the patient about (select all that apply.)?Select all that apply. A. Strict hand washing B. Daily nasal swabs for culture C. Monitor temperature every hour. D. Daily skin care and oral hygiene E. Encourage eating all foods to increase nutrients. F. Private room with a high-efficiency particulate air (HEPA) filter

A, D, F Strict hand washing and daily skin and oral hygiene must be done with neutropenia, because the patient is predisposed to infection from the normal body flora; other people; and uncooked meats, seafood, and eggs; unwashed fruits and vegetables; and fresh flowers or plants. The private room with HEPA filtration reduces the aerosolized pathogens in the patient's room. Blood cultures and antibiotic treatment are used when the patient has a temperature of 100.4°F or more, but temperature is not monitored every hour.

What are things that you can expect form a blood clot? (Select all that apply) A. Stroke B. Myocardial Infraction C. Nausea D. Pulmonary Embolism E. DVT

A. Stroke B. Myocardial Infraction D. Pulmonary Embolism E. DVT

A patient who has a history of heart failure and chronic obstructive lung disease is admitted with severe dyspnea. Which value would the nurse expect to be elevated if the cause of dyspnea was cardiac related? a. Serum potassium b. Serum homocysteine c. High-density lipoprotein d. B-type natriuretic peptide (BNP)

ANS: D Elevation of BNP indicates the presence of heart failure. Elevations help to distinguish cardiac versus respiratory causes of dyspnea. Elevated potassium, homocysteine, or HDL levels may indicate increased risk for cardiovascular disorders but do not indicate that cardiac disease is present.

The patient has an order for each of the following inhalers. Which one should the nurse offer to the patient at the onset of an asthma attack? Albuterol (Proventil) Salmeterol (Serevent) Beclomethasone (Qvar) Ipratropium bromide (Atrovent)

Albuterol (Proventil) Albuterol is a short-acting bronchodilator that should be given initially when the patient experiences an asthma attack. Salmeterol (Serevent) is a long-acting β2-adrenergic agonist, which is not used for acute asthma attacks. Beclomethasone (Qvar) is a corticosteroid inhaler and not recommended for an acute asthma attack. Ipratropium bromide (Atrovent) is an anticholinergic agent that is less effective than β2-adrenergic agonists. It may be used in an emergency with a patient unable to tolerate short-acting β2-adrenergic agonists (SABAs).

A patient with multiple myeloma becomes confused and lethargic. You would expect that these clinical manifestations may be explained by diagnostic results that indicate A. hyperkalemia. B. hyperuricemia. C. hypercalcemia. D. CNS myeloma.

Answer: C Rationale: Bony degeneration in multiple myeloma causes calcium to be lost from bones, eventually causing hypercalcemia. Hypercalcemia may cause renal, GI, or neurologic manifestations such as polyuria, anorexia, confusion, and ultimately seizures, coma, and cardiac problems.

A 45-year-old man with asthma is brought to the emergency department by automobile. He is short of breath and appears frightened. During the initial nursing assessment, which clinical manifestation might be present as an early manifestation during an exacerbation of asthma? Anxiety Cyanosis Bradycardia Hypercapnia

Anxiety An early manifestation during an asthma attack is anxiety because the patient is acutely aware of the inability to get sufficient air to breathe. He will be hypoxic early on with decreased PaCO2 and increased pH as he is hyperventilating. If cyanosis occurs, it is a later sign. The pulse and blood pressure will be increased.

Before starting a transfusion of packed red blood cells for an older anemic patient, the nurse would arrange for a peer to monitor his or her other assigned patients for how many minutes when the nurse begins the transfusion? A. 5 B. 15 C. 30 D. 60

B As part of standard procedure, the nurse remains with the patient for the first 15 minutes after starting a blood transfusion. Patients who are likely to have a transfusion reaction will more often exhibit signs within the first 15 minutes that the blood is infusing. Monitoring during the transfusion will be every 30 to 60 minutes.

The nurse is reviewing the objective data from the table below of a patient with suspected allergies. Which assessment finding does the nurse know indicates allergies? Tab 1: Physical examination Dry cough, Pale skin Tab 2: Laboratory results Neutrophils: 60%, Eosinophils: 10%, Basophils: 1%, Lymphocytes: 20%, Monocytes: 6% Tab 3: Medications Acetaminophen 1000 mg every 12 hours, Levothyroxine (Synthroid) 125 mcg each day A. Dry cough B. Eosinophil result C. Lymphocyte result D. Acetaminophen use

B Eosinophils are granulocytes that phagocytize antigen-antibody complexes formed during an allergic response. The normal eosinophil count is 2% to 4% of all white blood cells. The dry cough, lymphocyte result, and acetaminophen use do not indicate allergies.

A patient with a diagnosis of hemophilia had a fall down an escalator earlier in the day and is now experiencing bleeding in her left knee joint. What should be the emergency nurse's immediate response to this? A. Immediate transfusion of platelets B. Resting the patient's knee to prevent hemarthroses C. Assistance with intracapsular injection of corticosteroids D. Range-of-motion exercises to prevent thrombus formation

B In patients with hemophilia, joint bleeding requires resting of the joint in order to prevent deformities from hemarthrosis. Clotting factors, not platelets or corticosteroids, are administered. Thrombus formation is not a central concern in a patient with hemophilia.

A patient has anemia related to inadequate intake of essential nutrients. Which intervention would be appropriate for the nurse to include in the plan of care for this patient? A. Plan for 30 minutes of rest before and after every meal. B. Encourage foods high in protein, iron, vitamin C, and folate. C. Teach the patient to select only soft, bland, and nonacidic foods. D. Give the patient a list of medications that inhibit iron absorption.

B Increased intake of protein, iron, folate, and vitamin C provides nutrients needed for maximum iron absorption and hemoglobin production. The other interventions do not address the patient's identified problem of inadequate intake of essential nutrients. Selection of foods that are soft, bland, and nonacidic is appropriate if the patient has oral mucosal irritation. Scheduled rest is an appropriate intervention if the patient has fatigue related to anemia. Providing information about medications that may inhibit iron absorption (e.g. antacids, tetracycline, soft drinks, tea, coffee, calcium, phosphorus, and magnesium salts) is important but does not address the patient's problem of inadequate intake of essential nutrients.

A patient with an acute peptic ulcer and major blood loss requires an immediate transfusion with packed red blood cells. Which task is appropriate for (RN) to delegate to (UAP)? A. Confirm the IV solution is 0.9% saline. B. Obtain the vital signs before the transfusion is initiated. C. Monitor the patient for shortness of breath and back pain. D. Double-check the patient identity and verify the blood product.

B The RN may delegate tasks such as taking vital signs to UAP. Assessments (e.g. monitoring for signs of a blood transfusion reaction [shortness of breath and back pain]) are within the scope of practice of the RN and may not be delegated to UAP. The RN must also assume responsibility for ensuring the correct IV fluids is used with blood products. A licensed nurse must complete verification of the patient's identity and the blood product data.

The nurse receives a physician's order to transfuse fresh frozen plasma to a patient suffering from an acute blood loss. Which procedure is most appropriate for infusing this blood product? A. Hang the fresh frozen plasma with lactated Ringer's solution. B. Fresh frozen plasma must be given within 24 hours after thawing. C. Infuse the fresh frozen plasma at a rate of 50 mL/hr for the duration. D. Hang the fresh frozen plasma as a piggyback to a primary IV solution without KCl.

B The fresh frozen plasma should be administered as rapidly as possible and should be used within 24 hours of thawing to avoid a decrease in factors V and VIII. Fresh frozen plasma is infused using any straight-line infusion set. Any existing IV should be interrupted while the fresh frozen plasma is infused, unless a second IV line has been started for the transfusion.

A patient had a splenectomy for injuries sustained in a motor vehicle accident. Which phenomena are likely to result from the absence of the patient's spleen? (Select all that apply.) A. Impaired fibrinolysis B. Increased platelet levels C. Increased eosinophil levels D. Fatigue and cold intolerance E. Impaired immunologic function

B, E Splenectomy can result in increased platelet levels and impaired immunologic function because of the loss of storage and immunologic functions of the spleen. Fibrinolysis, fatigue, and cold intolerance are less likely to result from the loss of the spleen since coagulation and oxygenation are not primary responsibilities of the spleen.

If a pt has a Magnesium level of 5. What are you going to see? A. V-fib B. Torsades de pointes C. Vertigo D. Decreased reflexes

D. Decreased reflexes

The nurse knows that hemolytic anemia can be caused by which extrinsic factors? A. Trauma or splenic sequestration crisis B. Abnormal hemoglobin or enzyme deficiency C. Macroangiopathic or microangiopathic factors D. Chronic diseases or medications and chemicals

C Macroangiopathic or microangiopathic extrinsic factors lead to acquired hemolytic anemias. Trauma or splenic sequestration crisis can lead to anemia from acute blood loss. Abnormal hemoglobin or enzyme deficiency are intrinsic factors that lead to hereditary hemolytic anemias. Chronic diseases or medications and chemicals can decrease the number of RBC precursors which reduce RBC production.

The nurse collects a nutritional history from a 22-yr-old woman who is planning to conceive a child in the next year. Which foods reported by the woman would indicate that her diet is high in folate and iron? A. Crab, fish, and tuna B. Milk, cheese, and yogurt C. Spinach, beans, and liver D. White rice, potatoes, and pasta

C Normal intake of iron and folic acid is necessary for the development of red blood cells, and normal levels before conception and in early pregnancy are particularly important for normal fetal development. Foods high in both folic acid and iron include liver, red meat, egg yolks, turkey or chicken giblets, beans, lentils, chick peas, soybeans, spinach, and collard greens. In addition, enriched cereals, pasta, and breads are also high in both folic acid and iron (check the labels).

The emergency department nurse is caring for a patient who was just admitted with left anterior chest pain, suggesting possible acute myocardial infarction (MI). Which action will the nurse take first? A. Insert an IV catheter. B. Auscultate heart sounds. C. Administer sublingual nitroglycerin. D. Draw blood for troponin I measurement.

C. Administer sublingual nitroglycerin.

A patient returns after cardiac catheterization. Which nursing care would the registered nurse delegate to the licensed practical nurse? Monitor the electrocardiogram for dysrhythmias Check for bleeding at the catheter insertion site Prepare discharge teaching related to complications Take vital signs and report abnormal values

Check for bleeding at the catheter insertion site The licensed practical nurse can check for bleeding at the puncture sites. If bleeding is identified, it should be reported to the registered nurse. Vital signs should be delegated to the unlicensed assistive personnel. Preparation of discharge teaching and monitoring for dysrhythmias such as S-T elevation would be registered nurse scope of practice.

A patient with leukemia is admitted for severe hypovolemia after prolonged diarrhea has a platelet count of 43,000/µL. It is most important for the nurse to take which action? A. Insert two 18-gauge IV catheters. B. Administer prescribed enoxaparin. C. Monitor the patient's temperature every 2 hours. D. Check stools for presence of frank or occult blood.

D A platelet count below 150,000/µL indicates thrombocytopenia. Prolonged bleeding from trauma or injury does not usually occur until the platelet counts are below 50,000/µL. Bleeding precautions (e.g., check all secretions for frank and occult blood) are indicated for patients with thrombocytopenia. Injections (including IVs) should be avoided; however, when needed for critical fluids and medications, IV access should be provided through the smallest bore devices that are feasible. Enoxaparin, an anticoagulant administered subcutaneously, is contraindicated in patients with thrombocytopenia. Monitoring temperature would be indicated in a patient with leukopenia.

a nurse working in an ED is caring for a client following an acute chest trauma. which of the following findings should indicate to the nurse that the client is possibly experiencing a tension pneumothorax? A.) Collapsed neck veins on the affected side B.) Collapsed neck veins on the unaffected side C.) Tracheal deviation to the affected side D.) Tracheal deviation to the unaffected side

D.) Tracheal deviation to the unaffected side rationale: The nurse should recognize that deviation of the trachea to the unaffected side is a possible indicator that the client is experiencing a tension pneumothorax. A tension pneumothorax results from free air filling the chest cavity, causing the lung to collapse and forcing the trachea to deviate to the unaffected side.

a nurse is planning care for a client who has asthma. which of the following meds should the nurse plan to administer during an acute asthma attack? A.) cromolyn sodium B.) prednisone C.) fluticasone/salmeterol D.) albuterol

D.) albuterol rationale: The nurse should administer albuterol because it acts quickly to produce bronchodilation during an acute asthma attack.

The nurse, who has administered a first dose of oral prednisone to a patient with asthma, writes on the care plan to begin monitoring for which patient parameters? Apical pulse Daily weight Bowel sounds Deep tendon reflexes

Daily weight Corticosteroids such as prednisone can lead to weight gain. For this reason, it is important to monitor the patient's daily weight. The drug should not affect the apical pulse, bowel sounds, or deep tendon reflexes.

The nurse is providing teaching to a patient recovering from a myocardial infarction. How should resumption of sexual activity be discussed? Delegated to the primary care provider Discussed along with other physical activities Avoided because it is embarrassing to the patient Accomplished by providing the patient with written material

Discussed along with other physical activities Although some nurses may not feel comfortable discussing sexual activity with patients, it is a necessary component of patient teaching. It is helpful to consider sex as a physical activity and to discuss or explore feelings in this area when other physical activities are discussed. Although providing the patient with written material is appropriate, it should not replace a verbal dialogue that can address the individual patient's questions and concerns.

A nurse is assessing a client who has pancreatitis. The client's arterial blood gases reveal metabolic acidosis. Which of the following are expected findings? (Select all that apply.) A. Tachycardia B. Hypertension C. Bounding pulses D. Hyperreflexia E. Dysrhythmia F. Tachypnea

E. Dysrhythmia F. Tachypnea Dysrhythmia is an expected finding in a client who has pancreatitis and metabolic acidosis. Tachypnea is an expected finding in a client who has pancreatitis and metabolic acidosis.

A nurse is assessing a client for manifestations of aplastic anemia. Which of the following findings should the nurse expect? A. Plethoric appearance of facial skin B. Glossitis and weight loss D. Jaundice with enlarged liver E. Petechiae and ecchymosis

E. Petechiae and ecchymosis RATIONALE: In aplastic anemia, all 3 major blood components (RBC, WBC, platelets) are reduced/absent, which is known as pancytopenia and manifestations usually develop gradually

The nurse prepares a discharge teaching plan for a 44-yr-old male patient who has recently been diagnosed with coronary artery disease (CAD). Which risk factor should the nurse plan to focus on during the teaching session? Type A personality Elevated serum lipids Family cardiac history Hyperhomocysteinemia

Elevated serum lipids Dyslipidemia is one of the four major modifiable risk factors for CAD. The other major modifiable risk factors are hypertension, tobacco use, and physical inactivity. Research findings related to psychologic states (i.e., type A personality) as a risk factor for coronary artery disease have been inconsistent. Family history is a nonmodifiable risk factor. High homocysteine levels have been linked to an increased risk for CAD.

The nurse is assigned to care for a patient who has anxiety and an exacerbation of asthma. What is the primary reason for the nurse to carefully inspect the chest wall of this patient? Allow time to calm the patient. Observe for signs of diaphoresis. Evaluate the use of intercostal muscles. Monitor the patient for bilateral chest expansion.

Evaluate the use of intercostal muscles. The nurse physically inspects the chest wall to evaluate the use of intercostal (accessory) muscles, which gives an indication of the degree of respiratory distress experienced by the patient. The other options may also occur, but they are not the primary reason for inspecting the chest wall of this patient.

When admitting a patient with a diagnosis of asthma exacerbation, the nurse will assess for what potential triggers (select all that apply)? Exercise Allergies Emotional stress Decreased humidity Upper respiratory infections

Exercise Allergies Emotional stress Upper respiratory infections Although the exact mechanism of asthma is unknown, there are several triggers that may precipitate an attack. These include allergens, exercise, air pollutants, upper respiratory infections, drug and food additives, psychologic factors, and gastroesophageal reflux disease (GERD).

Which position is most appropriate for the nurse to place a patient experiencing an asthma exacerbation? Supine Lithotomy High Fowler's Reverse Trendelenburg

High Fowler's The patient experiencing an asthma attack should be placed in high Fowler's position and may need to lean forward to allow for optimal chest expansion and enlist the aid of gravity during inspiration. The supine, lithotomy, and reverse Trendelenburg positions will not facilitation ventilation.

When planning teaching for the patient with chronic obstructive pulmonary disease (COPD), the nurse understands that what causes the manifestations of the disease? An overproduction of the antiprotease α1-antitrypsin Hyperinflation of alveoli and destruction of alveolar walls Hypertrophy and hyperplasia of goblet cells in the bronchi Collapse and hypoventilation of the terminal respiratory unit

Hyperinflation of alveoli and destruction of alveolar walls In COPD there are structural changes that include hyperinflation of alveoli, destruction of alveolar walls, destruction of alveolar capillary walls, narrowing of small airways, and loss of lung elasticity. An autosomal recessive deficiency of antitrypsin may cause COPD. Not all patients with COPD have excess mucus production by the increased number of goblet cells.

A 68-year-old patient with bronchiectasis has copious thick respiratory secretions. Which intervention should the nurse add to the plan of care for this patient? Use the incentive spirometer for at least 10 breaths every 2 hours. Administer prescribed antibiotics and antitussives on a scheduled basis. Increase intake to at least 12 eight-ounce glasses of fluid every 24 hours. Provide nutritional supplements that are high in protein and carbohydrates.

Increase intake to at least 12 eight-ounce glasses of fluid every 24 hours. Adequate hydration helps to liquefy secretions and thus make it easier to remove them. Unless there are contraindications, the nurse should instruct the patient to drink at least 3 liters of fluid daily. Although nutrition, breathing exercises, and antibiotics may be indicated, these interventions will not liquefy or thin secretions. Antitussives may reduce the urge to cough and clear sputum, increasing congestion. Expectorants may be used to liquefy and facilitate clearing secretions.

The nurse evaluates that a patient is experiencing the expected beneficial effects of ipratropium (Atrovent) after noting which assessment finding? Decreased respiratory rate Increased respiratory rate Increased peak flow readings Decreased sputum production

Increased peak flow readings Ipratropium is a bronchodilator that should result in increased peak expiratory flow rates (PEFRs).

The nurse is preparing the patient for a diagnostic procedure to remove pleural fluid for analysis. The nurse would prepare the patient for which test? a. Thoracentesis b. Bronchoscopy c. Pulmonary angiography d. Sputum culture and sensitivityterm-315

a. Thoracentesis

During an assessment of a 45-year-old patient with asthma, the nurse notes wheezing and dyspnea. The nurse interprets that these symptoms are related to what pathophysiologic change? Laryngospasm Pulmonary edema Narrowing of the airway Overdistention of the alveoli

Narrowing of the airway Narrowing of the airway by persistent but variable inflammation leads to reduced airflow, making it difficult for the patient to breathe and producing the characteristic wheezing. Laryngospasm, pulmonary edema, and overdistention of the alveoli do not produce wheezing.

A 74-yr-old man with a history of prostate cancer and hypertension is admitted to the emergency department with substernal chest pain. Which action will the nurse complete before administering sublingual nitroglycerin? Administer morphine sulfate IV. Auscultate heart and lung sounds. Obtain a 12-lead electrocardiogram (ECG). Assess for coronary artery disease risk factors.

Obtain a 12-lead electrocardiogram (ECG). If a patient has chest pain, the nurse should institute the following measures: (1) administer supplemental oxygen and position the patient in upright position unless contraindicated, (2) assess vital signs, (3) obtain a 12-lead ECG, (4) provide prompt pain relief first with a nitrate followed by an opioid analgesic if needed, and (5) auscultate heart sounds. Obtaining a 12-lead ECG during chest pain aids in the diagnosis.

The nurse is caring for a 48-year-old male patient admitted for exacerbation of chronic obstructive pulmonary disease. The patient develops severe dyspnea at rest, with a change in respiratory rate from 26 breaths/minute to 44 breaths/minute. Which action by the nurse would be the most appropriate? Have the patient perform huff coughing. Perform chest physiotherapy for 5 minutes. Teach the patient to use pursed-lip breathing. Instruct the patient in diaphragmatic breathing.

Teach the patient to use pursed-lip breathing. Pursed-lip breathing (PLB) prolongs exhalation and prevents bronchiolar collapse and air trapping. PLB is simple and easy to teach and learn. It also gives the patient more control over breathing. Evidence from controlled studies does not support the use of diaphragmatic breathing in patients with COPD. Diaphragmatic breathing results in hyperinflation because of increased fatigue and dyspnea and abdominal paradoxical breathing rather than with normal chest wall motion. Chest physiotherapy (percussion and vibration) is used primarily for patients with excessive bronchial secretions who have difficulty clearing them. Huff coughing is a technique that helps patients with COPD to use a forced expiratory technique to clear secretions.

A male patient who has coronary artery disease (CAD) has serum lipid values of low-density lipoprotein (LDL) cholesterol of 98 mg/dL and high-density lipoprotein (HDL) cholesterol of 47 mg/dL. What should the nurse include in patient teaching? Consume a diet low in fats. Reduce total caloric intake. Increase intake of olive oil. The lipid levels are normal.

The lipid levels are normal. For men, the recommended LDL is less than 100 mg/dL, and the recommended level for HDL is greater than 40mg/dL. His normal lipid levels should be included in the patient teaching and encourage him to continue taking care of himself. Assessing his need for teaching related to diet should also be done.

A patient with an acute exacerbation of chronic obstructive pulmonary disease (COPD) needs to receive precise amounts of oxygen. Which equipment should the nurse prepare to use? Oxygen tent Venturi mask Nasal cannula Oxygen-conserving cannula

Venturi mask The Venturi mask delivers precise concentrations of oxygen and should be selected whenever this is a priority concern. The other methods are less precise in terms of amount of oxygen delivered.

Before discharge, the nurse discusses activity levels with a 61-year-old patient with chronic obstructive pulmonary disease (COPD) and pneumonia. Which exercise goal is most appropriate once the patient is fully recovered from this episode of illness? Slightly increase activity over the current level. Swim for 10 min/day, gradually increasing to 30 min/day. Limit exercise to activities of daily living to conserve energy. Walk for 20 min/day, keeping the pulse rate less than 130 beats/min.

Walk for 20 min/day, keeping the pulse rate less than 130 beats/min. The patient will benefit from mild aerobic exercise that does not stress the cardiorespiratory system. The patient should be encouraged to walk for 20 min/day, keeping the pulse rate less than 75% to 80% of maximum heart rate (220 - patient's age).

The nurse is caring for a patient with an acute exacerbation of asthma. Following initial treatment, what finding indicates to the nurse that the patient's respiratory status is improving? Wheezing becomes louder. Cough remains nonproductive. Vesicular breath sounds decrease. Aerosol bronchodilators stimulate coughing.

Wheezing becomes louder. The primary problem during an exacerbation of asthma is narrowing of the airway and subsequent diminished air exchange. As the airways begin to dilate, wheezing gets louder because of better air exchange. Vesicular breath sounds will increase with improved respiratory status. After a severe asthma exacerbation, the cough may be productive and stringy. Coughing after aerosol bronchodilators may indicate a problem with the inhaler or its use.

What is the goal of LDL? a. 100 or less b. 40 or more c. 150 or less d. 200 or less

a. 100 or less

Symptoms of tuberculosis include (select all that apply): a. Low grade fever b. Hemoptysis c. Increased appetite d. Night sweats

a. Low grade fever b. Hemoptysis d. Night sweats

When assessing the patient's integumentary system, which dermatologic manifestation may indicate anemia? a. Pallor b. Jaundice c. Skin tags d. Cyanosis

a. Pallor

T o F. A pt with a long history of CAD may have good collateral circulation to provide the infarction site with blood supply. a. True b. False

a. True

TB is the leading cause of mortality in patients with HIV infection a. True b. False

a. True

Best blood lab value shows effective treatment of pneumonia after IV antibiotics? a. WBC count b. RBC count c. Platelets count d. hemoglobin count

a. WBC count

The parathyroid hormone (PTH) regulates the movement of calcium into & out of: a. bones b. blood c. muscles d. liver

a. bones

Defense mechanisms that help protect the lung from inhaled particles and microorganisms include the (select all that apply) a. cough reflex. b. mucociliary escalator. c. alveolar macrophages. d. reflex bronchoconstriction. e. alveolar capillary membrane.

a. cough reflex. b. mucociliary escalator. c. alveolar macrophages. d. reflex bronchoconstriction.

Significant information from the patient's health history that relates to the hematologic system includes a. jaundice. b. bladder surgery. c. early menopause. d. multiple pregnancies.

a. jaundice

In the patient with supraventricular tachycardia, which assessment indicates decreased cardiac output? a. Hypertension and dyspnea b. Chest pain and palpitations c. Abdominal distention and tachypnea d. Bounding pulses and a systolic murmur

b. Chest pain and palpitations

A nurse is assessing a client who has PVD and a venous ulcer on the right ankle. Which of the following findings should the nurse expect to find on the patient affected extremity a. Absent pedal pulse b. Ankle swelling c. Hair loss d. Skin atrophy

b. Ankle swelling

When teaching a patient about rest pain with PAD, what should the nurse explain as the cause of the pain? a. Vasospasm of cutaneous arteries in the feet. b. Decrease in blood flow to the nerves of the feet. c. Increase in retrograde venous perfusion to the lower legs. d. Constriction in blood flow to leg muscles during exercise.

b. Decrease in blood flow to the nerves of the feet.

A patient with a hemoglobin (Hgb) level of 7.8 g/dL (78 g/L) has cardiac palpitations, a heart rate of 102 bpm, and an increased reticulocyte count. Considering the severity of anemia, what other manifestation would the nurse expect the patient to exhibit? a. Pallor b. Dyspnea c. A smooth tongue d. Sensitivity to cold

b. Dyspnea The patient's hemoglobin (Hgb) level indicates a moderate anemia, and at this severity, additional findings usually include dyspnea and fatigue. Pallor, smooth tongue, and sensitivity to cold usually manifest in severe anemia when the Hgb level is below 6 g/dL (60 g/L).

A nurse is caring for an adult male who is undergoing test for atherosclerosis. Which find should the nurse identify an increased risk for this disorder a. Cholesterol of 195 b. Elevated LDL c. Elevated hdl d. Triglyceride of 135

b. Elevated LDL

A patient with latent TB can still transmit the TB bacteria to others a. True b. False

b. False

f a lymph node is palpated, what is a normal finding? a. Hard, fixed nodes b. Firm, mobile nodes c. Enlarged, tender nodes d. Hard, nontender nodes

b. Firm, mobile nodes

Which function test fits description "Maximum amount of air lungs can contain"? a. FEV1 b. TLC c. Vt d. VC

b. TLC Total lung capacity

What are characteristics of PAD (select all that apply)? a. Pruritus b. Thickened, brittle nails c. Dull ache in calf or thigh d. Decreased peripheral pulses e. Pallor on elevation of the legs f. Ulcers over bony prominences on toes and feet

b. Thickened, brittle nails d. Decreased peripheral pulses e. Pallor on elevation of the legs f. Ulcers over bony prominences on toes and feet PAD is manifested as thick, brittle nails; decreased peripheral pulses; pallor when the legs are elevated; ulcers over bony prominences on the toes and feet; and paresthesia. The other options are characteristic of venous disease and paresthesia could occur with venous thromboembolism (VTE).

Which patient is at greatest risk for developing hypermagnesemia? a. 83-year-old man with lung cancer and hypertension b. 65-year-old woman with hypertension taking β-adrenergic blockers c. 42-year-old woman with systemic lupus erythematosus and renal failure d. 50-year-old man with benign prostatic hyperplasia and a urinary tract infection

c. 42-year-old woman with systemic lupus erythematosus and renal failure

What is the normal pH range of the blood, and what ratio of base to acid does this reflect? a. 7.32 to 7.42; 25 to 2 b. 7.32 to 7.42; 28 to 2 c. 7.35 to 7.45; 20 to 1 d. 7.35 to 7.45; 30 to 1

c. 7.35 to 7.45; 20 to 1 The other answers are incorrect

What is a possible cause for auscultation abnormal finding "absent breath sounds"? a. Lung consolidation with fluid or exudate b. Air trapping c. Atelactasis d. Interstitial edema

c. Atelactasis

The patient has been on the cardiac monitor with these occasional ectopic beats. The nurse recognizes this rhythm as: a. premature atrial contraction polymorphic b. premature ventricular contraction c. monomorphic premature ventricular d. contraction ventricular tachycardia

c. monomorphic premature ventricular

Which definition describes asthma? a. Dilated bronchioles, making it hard to clear secretions b. Disorder causing airway obstruction resulting increased mucus c. Disease with bronchial hyperreactivity with airflow limitation d. Chronic inflammatory response in the airways, primarily caused by smoking

c. Disease with bronchial hyperreactivity with airflow limitation

The client diagnosed with rule-out myocardial infarction is experiencing chest pain while walking to the bathroom. Which action should the nurse implement first? a. Administer sublingual nitroglycerin. b. Obtain a STAT electrocardiogram. c. Have the client sit down immediately. d. Assess the client's vital signs.

c. Have the client sit down immediately. Stopping all activity will decrease the need of the myocardium for oxygen and may help decrease the chest pain and decrease fall risk.

The nurse is caring for a patient with acute decompensated heart failure who is receiving IV dobutamine. Why would this drug be prescribed? (select all that apply) a. It dilates renal blood vessels. b. It will increase the heart rate. c. Heart contractility will improve. d. Dobutamine is a selective β-agonist. e. It increases systemic vascular resistance.

c. Heart contractility will improve. d. Dobutamine is a selective β-agonist.

The typical fluid replacement for the patient with a fluid volume deficit is a. dextran. b. 0.45% saline. c. lactated Ringer's solution. d. 5% dextrose in 0.45% saline.

c. lactated Ringer's solution.

A woman has ham with gravy and green beans cooked with salt pork for dinner. What could happen to the woman's serum osmolality because of this meal?

serum osmolality increases as a large amount of sodium is absorbed

The physician has prescribed salmeterol (Serevent) for a patient with asthma. In reviewing the use of dry powder inhalers (DPIs) with the patient, what instructions should the nurse provide? "Close lips tightly around the mouthpiece and breathe in deeply and quickly." "To administer a DPI, you must use a spacer that holds the medicine so that you can inhale it." "You will know you have correctly used the DPI when you taste or sense the medicine going into your lungs." "Hold the inhaler several inches in front of your mouth and breathe in slowly, holding the medicine as long as possible."

"Close lips tightly around the mouthpiece and breathe in deeply and quickly." The patient should be instructed to tightly close the lips around the mouthpiece and breathe in deeply and quickly to ensure the medicine moves down deeply into the lungs. Dry powder inhalers do not require spacer devices. The patient may not taste or sense the medicine going into the lungs.

The nurse teaches a 33-year-old male patient with asthma how to administer fluticasone (Flovent HFA) by metered-dose inhaler (MDI). Which statement by the patient to the nurse indicates correct understanding of the instructions? "I should not use a spacer device with this inhaler." "I will rinse my mouth each time after I use this inhaler." "I will feel my breathing improve over the next 2 to 3 hours." "I should use this inhaler immediately if I have trouble breathing."

"I will rinse my mouth each time after I use this inhaler." Fluticasone (Flovent HFA) may cause oral candidiasis (thrush). The patient should rinse the mouth with water or mouthwash after use or use a spacer device to prevent oral fungal infections. Fluticasone is an inhaled corticosteroid, and it may take 2 weeks of regular use for effects to be evident. This medication is not recommended for an acute asthma attack.

The nurse instructs a 68-yr-old woman with hypercholesterolemia about natural lipid-lowering therapies. The nurse determines further teaching is necessary if the patient makes which statement? "Omega-3 fatty acids are helpful in reducing triglyceride levels." "I should check with my physician before I start taking any herbal products." "Herbal products do not go through as extensive testing as prescription drugs do." "I will take garlic instead of my prescription medication to reduce my cholesterol."

"I will take garlic instead of my prescription medication to reduce my cholesterol." Current evidence does not support using garlic in the treatment of elevated cholesterol. Strong evidence supports the use of omega-3 fatty acids for reduction of triglyceride levels. Many herbal products are not standardized and effects are not predictable. Patients should consult with their health care provider before starting herbal or natural therapies.

Which statement made by the patient with chronic obstructive pulmonary disease (COPD) indicates a need for further teaching regarding the use of an ipratropium inhaler? "I can rinse my mouth following the two puffs to get rid of the bad taste." "I should wait at least 1 to 2 minutes between each puff of the inhaler." "Because this medication is not fast-acting, I cannot use it in an emergency if my breathing gets worse." "If my breathing gets worse, I should keep taking extra puffs of the inhaler until I can breathe more easily."

"If my breathing gets worse, I should keep taking extra puffs of the inhaler until I can breathe more easily." The patient should not just keep taking extra puffs of the inhaler to make breathing easier. Excessive treatment could trigger paradoxical bronchospasm, which would worsen the patient's respiratory status. Rinsing the mouth after the puffs will eliminate a bad taste. Waiting 1 to 2 minutes between each puff will facilitate the effectiveness of the administration. Ipratropium is not used in an emergency for COPD.

The nurse determines that the patient understood medication instructions about the use of a spacer device when taking inhaled medications after hearing the patient state what as the primary benefit? "I will pay less for medication because it will last longer." "More of the medication will get down into my lungs to help my breathing." "Now I will not need to breathe in as deeply when taking the inhaler medications." "This device will make it so much easier and faster to take my inhaled medications."

"More of the medication will get down into my lungs to help my breathing." A spacer assists more medication to reach the lungs, with less being deposited in the mouth and the back of the throat. It does not affect the cost or increase the speed of using the inhaler.

Although a diagnosis of cystic fibrosis is most often made before age 2, an 18-year-old patient at the student health center with a history of frequent lung and sinus infections has clinical manifestations consistent with undiagnosed cystic fibrosis (CF). Which information would be accurate for the nurse to include when teaching the patient about a scheduled sweat chloride test? "Sweat chloride greater than 60 mmol/L is consistent with a diagnosis of CF." "If sweating occurs after an oral dose of pilocarpine, the test for CP is positive." "The test measures the amount of sodium chloride in your postexercise sweat." "If the sweat chloride test is positive on two occasions, genetic testing will be necessary."

"Sweat chloride greater than 60 mmol/L is consistent with a diagnosis of CF." The diagnostic criteria for CF involve a combination of clinical presentation, sweat chloride testing, and genetic testing to confirm the diagnosis. The sweat chloride test is performed by placing pilocarpine on the skin and carried by a small electric current to stimulate sweat production. This takes about 5 minutes, and the patient feels a slight tingling or warmth. The sweat is collected on filter paper or gauze and then analyzed for sweat chloride concentrations (for about 1 hour). Values above 60 mmol/L for sweat chloride are consistent with the diagnosis of CF. However, a second sweat chloride test is recommended to confirm the diagnosis, unless genetic testing identifies a CF mutation. Genetic testing is used if the results from a sweat chloride test are unclear.

When assessing a patient with possible peripheral artery disease (PAD), the nurse obtains a brachial BP of 147/82 and an ankle pressure of 112/74. The nurse calculates the patients ankle-brachial index (ABI) as ________ (round up to the nearest hundredth).

0.76 The ABI is calculated by dividing the ankle systolic BP by the brachial systolic BP.

Priority Decision: A patient has atrial fibrillation and develops an acute arterial occlusion at the iliac artery bifurcation. What are the 6 Ps of acute arterial occlusion the nurse may assess in this patient that require immediate notification of the HCP?

1. Pain; 2. pallor; 3. pulselessness; 4. paresthesia; 5. paralysis; 6. poikilothermia. The HCP requires immediate notification to begin immediate intervention to prevent tissue necrosis and gangrene. 8. b, d, f. Arterial leg ulcers and/or gangrene of the leg caused by PAD and chronic ischemic rest pain lasting more than 2 weeks characterize critical limb ischemia. Optimal therapy is revascularization via bypass surgery.

A 45-year-old patient with chronic arterial disease has a brachial systolic blood pressure (SBP) of 132 mm Hg and an ankle SBP of 102 mm Hg. The ankle-brachial index is ___________________________________ and indicates ___________________________________ (mild/moderate/severe) arterial disease.

102 ÷ 132 = 0.77; mild

The nurse obtains a blood pressure of 176/83 mm Hg for a patient. What is the patients mean arterial pressure (MAP)?

114 mm Hg MAP = (SBP + 2 DBP)/3 MAP = (176+83+83)/3

The ECG pattern of a patient with a regular HR reveals 20 small squares between each R-R interval. What is the patient's HR?

1500 ÷ 20 = 75 bpm

A patient has a serum Na + of 147 mEq/L (147 mmol/L), blood urea nitrogen (BUN) of 6 mg/dL (2.1 mmol/L), and a blood glucose level of 126 mg/dL (7.0 mmol/L). The patient's effective serum osmolality is ________ mOsm/kg. Is the patient's plasma osmolality normal, increased, or decreased?

2x147 + 6/2.8 + 126/18 = 303.14 The patient's plasma osmolality is increased.

A patient with a regular heart rate (HR) has 4 QRS complexes between every 3-second marker on the ECG paper. Calculate the patient's HR.

4 (beats per 3 seconds) + 4 = 8 beats per 6 seconds × 10 = 80 bpm

When analyzing an electrocardiographic (ECG) rhythm strip of a patient with a regular heart rhythm, the nurse counts 30 small blocks from one R wave to the next. The nurse calculates the patients heart rate as ____.

50 There are 1500 small blocks in a minute, and the nurse will divide 1500 by 30

A man who weighs 90 kg has a total body water content of approximately _________________ L.

54 to 45 L: 90 kg × 60% = 54, 90 kg × 50% = 45

A nitroglycerin drip is ordered for your patient to control his chest pain. the concentration is 100 mg in 250 cc D5W. the order is to begin the infusion at 40 mcg/min. what is the rate you would begin on the infusion pump?

6 mL/hr 100mg X 1000 = 100,000/250= 400mcg 40mcg X60 min = 2,400/400mcg = 6ml/hr

Which patient is most likely to experience anemia related to an increased destruction of red blood cells? A. A 23-year-old African American man who has a diagnosis of sickle cell disease B. A 59-year-old man whose alcoholism has precipitated folic acid deficiency C. A 13-year-old child whose impaired growth and development is attributable to thalassemia D. A 50-year-old woman with a history of "heavy periods" accompanied by anemia

A A result of a sickling episode in sickle cell anemia involves increased hemolysis of the sickled cells. Thalassemias and folic acid deficiencies cause a decrease in erythropoiesis, whereas the anemia related to menstruation is a direct result of blood loss.

A patient will receive a hematopoietic stem cell transplant (HSCT). What is the nurse's priority after the patient receives combination chemotherapy before the transplant? A. Prevent patient infection. B. Avoid abnormal bleeding. C. Give pneumococcal vaccine. D. Provide companionship while isolated.

A After combination chemotherapy for HSCT, the patient's bone marrow is destroyed in preparation to receive the bone marrow graft. Thus the patient is immunosuppressed and is at risk for a life-threatening infection. The priority is preventing infection. Bleeding is not usually a problem. Giving the pneumococcal vaccine at this time should not be done, but should have been done previously. Providing companionship is not the primary role of the nurse, although the patient will need support during the time of isolation.

Results of a patient's most recent blood work indicate an elevated neutrophil level. The nurse recognizes that this diagnostic finding most likely suggests which problem? A. Infection B. Hypoxemia C. Acute thrombotic event D. Risk of hypocoagulation

A An increase in the neutrophil count most commonly occurs in response to infection or inflammation. Hypoxemia and coagulation do not directly affect neutrophil production.

When evaluating a patient's nutritional-metabolic pattern related to hematologic health, what priority assessment should the nurse perform? A. Inspect the skin for petechiae. B. Ask the patient about joint pain. C. Assess for vitamin C deficiency. D. Determine if the patient can perform activities of daily living.

A Any changes in the skin's texture or color should be explored when assessing the patient's nutritional-metabolic pattern related to hematologic health. The presence of petechiae or ecchymotic areas could be indicative of hematologic deficiencies related to poor nutritional intake or related causes. The other options are not specific to the nutritional-metabolic pattern related to hematologic health.

The nurse is caring for a 36-yr-old male patient receiving phenytoin (Dilantin) to treat seizures resulting from his traumatic brain injury as a teenager. It is most important for the nurse to observe for which hematologic adverse effect of this medication? A. Anemia B. Leukemia C. Polycythemia D. Thrombocytosis

A Hematologic adverse effects of phenytoin include anemia, thrombocytopenia, leukopenia, granulocytopenia, agranulocytosis, and pancytopenia.

A patient with thrombocytopenia secondary to sepsis has small, pinpoint deposits of blood visible through the skin on the anterior and posterior chest. The nurse will document this skin abnormality as: A. petechiae B. erythema C. ecchymosis D. telangiectasia

A Petechiae are pinpoint, discrete deposits of blood less than 1 to 2 mm in the extravascular tissues and visible through the skin or mucous membranes. Erythema is redness occurring in patches of variable size and shape. Telangiectasia is visibly dilated, superficial, cutaneous small blood vessels. Ecchymosis is a large, bruise-like lesion caused by a collection of extravascular blood in the dermis and subcutaneous tissue.

The nurse is preparing to perform an assessment for a newly admitted patient with a potential hematologic disorder and petechiae. What does the nurse anticipate finding when assessing this patient? A. Tiny purple spots on the skin B. Large ecchymotic areas on the skin C. Hyperkeratotic papules and plaques D. Small, raised red areas on the soles of the feet

A Petechiae present as tiny purple spots on the skin. Large ecchymotic areas are purpura. Hyperkeratotic papules and plaques characterize actinic keratosis. Small, raised red areas on the soles of the feet signify Osler's nodes.

At a clinic visit, the nurse provides dietary teaching for a 56-year-old woman who was recently hospitalized with an exacerbation of chronic heart failure. The nurse determines that teaching is successful if the patient makes which statement? A "I will limit the amount of milk and cheese in my diet." B "I can add salt when cooking foods but not at the table." C "I will take an extra diuretic pill when I eat a lot of salt." D "I can have unlimited amounts of foods labeled as reduced sodium ."

A "I will limit the amount of milk and cheese in my diet." Milk products should be limited to 2 cups per day for a 2500-mg sodium-restricted diet. Salt should not be added during food preparation or at the table. Diuretics should be taken as prescribed (usually daily) and not based on sodium intake. Foods labeled as reduced sodium contain at least 25% less sodium than regular.

A70-year-old woman with chronic heart failure and atrial fibrillation asks the nurse why warfarin (Coumadin) has been prescribed for her to continue at home. Which response by the nurse is accurate? A "The medication prevents blood clots from forming in your heart." B "The medication dissolves clots that develop in your coronary arteries." C "The medication reduces clotting by decreasing serum potassium levels." D "The medication increases your heart rate so that clots do not form in your heart."

A "The medication prevents blood clots from forming in your heart." Chronic heart failure causes enlargement of the chambers of the heart and an altered electrical pathway, especially in the atria. When numerous sites in the atria fire spontaneously and rapidly, atrial fibrillation occurs. Atrial fibrillation promotes thrombus formation within the atria with an increased risk of stroke and requires treatment with cardioversion, antidysrhythmics, and/or anticoagulants. Warfarin is an anticoagulant that interferes with hepatic synthesis of vitamin K-dependent clotting factors.

Which individuals would the nurse identify as having the highest risk for coronary artery disease (CAD)? A 45-yr-old depressed man with a high-stress job A 60-yr-old man with below normal homocysteine levels A 54-yr-old woman vegetarian with elevated high-density lipoprotein (HDL) levels A 62-yr-old woman who has a sedentary lifestyle and body mass index (BMI) of 23 kg/m2

A 45-yr-old depressed man with a high-stress job The 45-yr-old depressed man with a high-stress job is at the highest risk for CAD. Studies demonstrate that depression and stressful states can contribute to the development of CAD. Elevated HDL levels and low homocysteine levels actually help to prevent CAD. Although a sedentary lifestyle is a risk factor, a BMI of 23 kg/m2 depicts normal weight, and thus the patient with two risk factors is at greatest risk for developing CAD.

Which patient is at greatest risk for sudden cardiac death (SCD)? A 42-yr-old white woman with hypertension and dyslipidemia A 52-yr-old African American man with left ventricular failure A 62-yr-old obese man with diabetes mellitus and high cholesterol A 72-yr-old Native American woman with a family history of heart disease

A 52-yr-old African American man with left ventricular failure Patients with left ventricular dysfunction (ejection fraction <30%) and ventricular dysrhythmias after myocardial infarction are at greatest risk for SCD. Other risk factors for SCD include (1) male gender (especially African American men), (2) family history of premature atherosclerosis, (3) tobacco use, (4) diabetes mellitus, (5) hypercholesterolemia, (6) hypertension, and (7) cardiomyopathy.

A patient admitted with heart failure appears very anxious and complains of shortness of breath. Which nursing actions would be appropriate to alleviate this patient's anxiety (select all that apply)? A Administer ordered morphine sulfate. B Position patient in a semi-Fowler's position. C Position patient on left side with head of bed flat. D Instruct patient on the use of relaxation techniques. E Use a calm, reassuring approach while talking to patient.

A Administer ordered morphine sulfate. B Position patient in a semi-Fowler's position. D Instruct patient on the use of relaxation techniques. E Use a calm, reassuring approach while talking to patient. Morphine sulfate reduces anxiety and may assist in reducing dyspnea. The patient should be positioned in semi-Fowler's position to improve ventilation that will reduce anxiety. Relaxation techniques and a calm reassuring approach will also serve to reduce anxiety.

Despite a high dosage, a male patient who is taking nifedipine (Procardia XL) for antihypertensive therapy continues to have blood pressures over 140/90 mmHg. What should the nurse do next? A Assess his adherence to therapy. B Ask him to make an exercise plan. C Instruct him to use the DASH diet. D Request a prescription for a thiazide diuretic.

A Assess his adherence to therapy. A long-acting calcium-channel blocker such as nifedipine causes vascular smooth muscle relaxation resulting in decreased SVR and arterial BP and related side effects. The patient data the nurse has about this patient is very limited, so the nurse needs to assess his adherence to therapy.

The nurse teaches a patient with hypertension that uncontrolled hypertension may damage organs in the body primarily by which mechanism? A Hypertension promotes atherosclerosis and damage to the walls of the arteries. Hypertension causes direct pressure on organs, Bresulting in necrosis and replacement of cells with scar tissue. C Hypertension causes thickening of the capillary membranes, leading to hypoxia of organ systems. D Hypertension increases blood viscosity, which contributes to intravascular coagulation and tissue necrosis distal to occlusions.

A Hypertension promotes atherosclerosis and damage to the walls of the arteries. Hypertension is a major risk factor for the development of atherosclerosis by mechanisms not yet fully known. However, once atherosclerosis develops, it damages the walls of arteries and reduces circulation to target organs and tissues.

The patient has heart failure (HF) with an ejection fraction of less than 40%. What core measures should the nurse expect to include in the plan of care for this patient (select all that apply)? A Left ventricular function is documented. B Controlling dysrhythmias will eliminate HF. C Prescription for digoxin (Lanoxin) at discharge D Prescription for angiotensin-converting enzyme (ACE) inhibitor at discharge E Education materials about activity, medications, weight monitoring, and what to do if symptoms worsen

A Left ventricular function is documented. D Prescription for angiotensin-converting enzyme (ACE) inhibitor at discharge E Education materials about activity, medications, weight monitoring, and what to do if symptoms worsen The Joint Commission has identified these three core measures for heart failure patients. Although controlling dysrhythmias will improve CO and workload, it will not eliminate HF. Prescribing digoxin for all HF patients is no longer done because there are newer effective drugs and digoxin toxicity occurs easily related to electrolyte levels and the therapeutic range must be maintained.

The nurse is caring for a patient with hypertension who is scheduled to receive a dose of esmolol (Brevibloc). The nurse should withhold the dose and consult the prescribing physician for which vital sign taken just before administration? A Pulse 48 B Respirations 24 C Blood pressure 118/74 D Oxygen saturation 93%

A Pulse 48 Because esmolol is a β1-adrenergic blocking agent, it can cause hypotension and bradycardia as adverse effects. The nurse should withhold the dose and consult with the health care provider for parameters regarding pulse rate limits.

The patient with chronic heart failure is being discharged from the hospital. What information should the nurse emphasize in the patient's discharge teaching to prevent progression of the disease to ADHF? A Take medications as prescribed. B Use oxygen when feeling short of breath. C Only ask the physician's office questions. D Encourage most activity in the morning when rested.

A Take medications as prescribed. The goal for the patient with chronic HF is to avoid exacerbations and hospitalization. Taking the medications as prescribed along with nondrug therapies such as alternating activity with rest will help the patient meet this goal. If the patient needs to use oxygen at home, it will probably be used all the time or with activity to prevent respiratory acidosis. Many HF patients are monitored by a care manager or in a transitional program to assess the patient for medication effectiveness and monitor for patient deterioration and encourage the patient. This nurse manager can be asked questions or can contact the health care provider if there is evidence of worsening HF.

Which test result identifies that a patient with asthma is responding to treatment? An increase in CO2 levels A decreased exhaled nitric oxide A decrease in white blood cell count An increase in serum bicarbonate levels

A decreased exhaled nitric oxide Nitric oxide levels are increased in the breath of people with asthma. A decrease in the exhaled nitric oxide concentration suggests that the treatment may be decreasing the lung inflammation associated with asthma and adherence to treatment. An increase in CO2 levels, decreased white blood cell count, and increased serum bicarbonate levels do not indicate a positive response to treatment in the asthma patient.

A nurse is caring for a client who asks why the provider prescribed a daily aspirin. Which of the following responses should the nurse make? A. "Aspirin reduces the formation of blood clots that could cause a heart attack." B. "Aspirin relieves the pain due to myocardial ischemia." C. "Aspirin dissolves clots that are forming in your coronary arteries." D. "Aspirin relieves headaches that are caused by other medications."

A. "Aspirin reduces the formation of blood clots that could cause a heart attack." Aspirin decreases platelet aggregation that can cause a myocardial infarction

A nurse is providing teaching for a client who has a prescription for a low-sodium diet to manage hypertension. Which of the following statements by the client indicates an understanding of the teaching? A. "I can snack on fresh fruit" B. "I can continue to eat lunchmeat sandwiches" C. "I can have cottage cheese with my meals" D. "Canned soup is a good lunch option"

A. "I can snack on fresh fruit"

A nurse is caring for a client following insertion of a permanent pacemaker. Which of the following client statements indicates a potential complication of the insertion procedure? A. "I can't get rid of these hiccups" B. "I feel dizzy when I stand" C. "My incision site stings" D. "I have a headache"

A. "I can't get rid of these hiccups" RATIONALE: the pacemaker could be stimulating the chest wall/diaphragm, which can occur as a result of a lead wire perforation

A nurse is caring for a client following insertion of a permanent pacemaker. Which of the following client statements indicates a potential complication of the insertion procedure? A. "I can't get rid of these hiccups." b. "I feel dizzy when I stand." c. "My incision site stings." d. "I have a headache."

A. "I can't get rid of these hiccups."

A nurse is providing discharge teaching to a client who has a new permanent pacemaker. Which of the following statements by the client indicates an understanding of the teaching? A. "I should check my heart rate at the same time each day" B. "I don't have to take my antihypertensive medications now that I have a pacemaker" C. "I should keep a pressure dressing over the generator until the incision is healed" D. "I cannot stand in front of our new microwave when it is on"

A. "I should check my heart rate at the same time each day"

A nurse is teaching a client with heart disease about a low-cholesterol diet. Which of the following client statements indicates the teaching was effective? A. "I should remove the skin from poultry before eating it" B. "I will eat seafood once per week" C. "I should use margarine when preparing meals" D. "I can use whole milk in my oatmeal"

A. "I should remove the skin from poultry before eating it"

A nurse is teaching a group of clients about influenza. Which of the following client statements indicates an understanding of the teaching? A. "I should wash my hands after blowing my nose to prevent spreading the virus." B. "I need to avoid drinking fluids if I develop symptoms." C. "I need a flu shot every 2 years because of the different flu strains." D. "I should cover my mouth with my hand when I sneeze."

A. "I should wash my hands after blowing my nose to prevent spreading the virus." Hand hygiene decreases the risk of the client spreading influenza viruses

A nurse is providing information to a client who is scheduled for an exercise electrocardiography test. Which of the following client statements indicates an understanding of the teaching? A. "I will not drink coffee 4hr prior to the test" B. "I can eat a light meal 1hr prior to the test" C. "I can have a cigarette up to 30min prior to the test" D. "I will take my heart medication on the day of the test"

A. "I will not drink coffee 4hr prior to the test"

A nurse is caring for a client who is undergoing treatment for hypertension. Which of the following statements indicates that the client is adhering to the treatment plan? A. "I would never have believed I could get used to enjoying my food without salt" B. "My blood pressure device at home usually shows about 156/98 or so" C. "I make sure I take my blood pressure medicine when I have headaches" D. "My blood pressure pills are very expensive. Could I take a cheaper medication"?

A. "I would never have believed I could get used to enjoying my food without salt"

A nurse is providing discharge teaching for a client who had a bone marrow transplant and has thrombocytopenia. Which of the following statements indicates that the client understands the precautions he must take at home? A. "I'll stick with soft foods for now" B. "My family will be bringing me fresh flowers today" C. "I'll use a new disposable razor each day" D. "I"ll blow my nose more often to avoid nosebleeds"

A. "I'll stick with soft foods for now" RATIONALE: To prevent bleeding from mouth trauma

A client with hypertension asks the nurse to explain how amlodipine besylate (Norvasc) lowers her blood pressure. The nurse gives which explanation to the client? A. "It prevents calcium from entering the smooth muscle, which relaxes the blood vessels to lower heart rate and blood pressure." B. "It is a diuretic that works by removing extra sodium and water from the body through the kidneys, which helps lower blood pressure." C. "It causes the body to produce less angiotensin, which allows blood vessels to relax and open up, therefore reducing blood pressure." D. "It lowers blood pressure by lowering the heart rate and the workload of the heart, and decreases the amount of blood pumped out of the hear

A. "It prevents calcium from entering the smooth muscle, which relaxes the blood vessels to lower heart rate and blood pressure."

A charge nurse is teaching a group of nurses about conditions related to metabolic acidosis. Which of the following statements by a unit nurse indicates the teaching has been effective? A. "Metabolic acidosis can occur due to diabetic ketoacidosis." B. "Metabolic acidosis can occur in a client who has myasthenia gravis." C. "Metabolic acidosis can occur in a client who has asthma." D. "Metabolic acidosis can occur due to cancer."

A. "Metabolic acidosis can occur due to diabetic ketoacidosis." Metabolic acidosis results from an excess production of hydrogen ions, which occurs in diabetic ketoacidosis.

A nurse is providing discharge teaching for a client who has heart failure and is on a fluid restriction of 2,000 mL/day. The client asks the nurse how to determine the appropriate amount of fluids they are allowed. Which of the following statements is an appropriate response by the nurse? A. "Pour the amount of fluid you drink into an empty 2‑liter bottle to keep track of how much you drink." B. "Each glass contains 8 ounces. There are 30 milliliters per ounce, so you can have a total of 8 glasses or cups of fluid each day." C. "This is the same as 2 quarts, or about the same as two pots of coffee." D. "Take sips of water or ice chips so you will not take in too much fluid."

A. "Pour the amount of fluid you drink into an empty 2‑liter bottle to keep track of how much you drink." Pouring the amount of fluid consumed into an empty 2 L bottle provides a visual guide for the client as to the amount consumed and how to plan daily intake.

A nurse is discharging a client who has COPD. The client is concerned about not being able to leave the house due to the need for staying on continuous oxygen. Which of the following responses should the nurse make? A. "There are portable oxygen delivery systems that you can take with you." B. "When you go out, you can remove the oxygen and then reapply it when you get home." C. "You probably will not be able to go out as much as you used to." D. "Home health services will come to you so you will not need to get out."

A. "There are portable oxygen delivery systems that you can take with you." Inform the client that there are portable oxygen systems that can be used to leave the house. This should alleviate the client's anxiety.

A nurse is providing teaching to a client who is 2 days postoperative following a heart transplant. Which of the following statements should the nurse include in the teaching? A. "You might no longer be able to feel chest pain" B. "Your level of activity intolerance will not change" C. "After 6 months, you will no longer need to restrict your sodium intake" D. "You will be able to stop taking immunosuppressants after 12 months"

A. "You might no longer be able to feel chest pain" RATIONALE: This is d/t denervation of the heart

A nurse is providing discharge teaching to a client who had a gastrectomy for stomach cancer. Which of the following information should the nurse include in the teaching? (Select all that apply.) A. "You will need a monthly injection of vitamin B12 for the rest of your life." B. "Using the nasal spray form of vitamin B12 on a daily basis can be an option." C. "An oral supplement of vitamin B12 taken on a daily basis can be an option." D. "You should increase your intake of animal proteins, legumes, and dairy products to increase vitamin B12 in your diet." E. "Add soy milk fortified with vitamin B12 to your diet to decrease the risk of pernicious anemia."

A. "You will need a monthly injection of vitamin B12 for the rest of your life." B. "Using the nasal spray form of vitamin B12 on a daily basis can be an option." The client who had a gastrectomy will require monthly injections of vitamin B12 for the rest of their life due to lack of intrinsic factor being produced by the parietal cells of the stomach. Cyanocobalamin nasal spray used daily is an option for a client who had a gastrectomy.

A nurse is preparing a client for cardiac catheterization. Which of the following pieces of information should the nurse give the client before the procedure? SATA A. "You'll have to lie flat for several hours after the procedure" B. "You'll receive medication to relax you before the procedure" C. "You'll feel a cool sensation after the injection of the dye" D. "You'll have to keep your leg straight after the procedure" E. "You'll have to limit the amount of fluid you drink for the first 24hr"

A. "You'll have to lie flat for several hours after the procedure" B. "You'll receive medication to relax you before the procedure" D. "You'll have to keep your leg straight after the procedure" RATIONALE: Not C- would feel a hot flash

A nurse is caring for a client who is undergoing conservative treatment for DVT. The client asks the nurse what will happen to the clot. Which of the following responses should the nurse make? A. "Your body has a process called fibrinolysis that will eventually dissolve the clot" B. "Your body has a mechanism that will keep the clot stable in its present location" C. "The clot will break into tiny fragments and float harmlessly in your bloodstream D. "Treatment with heparin will dissolve the clot and keep other clots from forming"

A. "Your body has a process called fibrinolysis that will eventually dissolve the clot"

A nurse is providing teaching for a client who is 2 days post-op following a heart transplant. Which of the following statements should the nurse include in the teaching? A. "you may no longer be able to feel chest pain." b. "your level of activity tolerance will not change." c. "after 6 months, you will no longer need to restrict your sodium intake." d. "you will be able to stop taking immunosuppressants after 12 months."

A. "you may no longer be able to feel chest pain." Rationale: Heart transplant clients usually are no longer able to feel chest pain due to the denervation of the heart.

A client with a history of heart failure is due for a morning dose of furosemide. Which serum potassium level, if noted in the client's laboratory report, should be reported before administering the dose of furosemide? A. 3.2 mEq/L (3.2 mmol/L) B. 3.8 mEq/L (3.8 mmol/L) C. 4.2 mEq/L (4.2 mmol/L) D. 4.8 mEq/L (4.8 mmol/L)

A. 3.2 mEq/L (3.2 mmol/L)

By definition, neutropenia occurs when the white blood cell (WBC) count drops below? A. 4000/μL B. 3000/μL C. 2000/μL D. 1000/μL

A. 4000/μL Rationale: A total WBC count of less than 4000/μL (4 × 109/L) defines leukopenia.

A nurse on a medical-surgical unit is caring for 4 clients. Which of the following clients should the nurse monitor for crepitus? A. A client who has a chest tube following a pneumothorax B. A client who has an acute exacerbation of Crohn's disease C. A client who is postoperative following a laparoscopic appendectomy D. A client who is recovering from thyroid storm

A. A client who has a chest tube following a pneumothorax RATIONALE: Crepitus, a crackling sound resulting from air trapped under the skin can be palpated following a pneumothorax

A nurse on a cardiac unit is caring for a group of clients. The nurse should recognize which of the following clients as being at risk for the development of a dysrhythmia? (Select all that apply.) A. A client who has metabolic alkalosis B. A client who has a blood potassium level of 4.3 mEq/L C. A client who has an SaO2 of 96% D. A client who has COPD E. A client who underwent stent placement in a coronary artery

A. A client who has metabolic alkalosis D. A client who has COPD E. A client who underwent stent placement in a coronary artery A client who has an acid‑base imbalance such as metabolic alkalosis is at risk for a dysrhythmia. A client who has lung disease, such as COPD, is at risk for a dysrhythmia. A client who has cardiac disease and underwent a stent placement is at risk for a dysrhythmia.

A nurse is assessing a client who has pulmonary edema related to heart failure. Which of the following findings indicates effective treatment of the client's condition? A. Absence of adventitious breath sounds B. Presence of a nonproductive cough C. Decrease in respiratory rate at rest D. Sao2 86% on room air

A. Absence of adventitious breath sounds Rationale: Adventitious breath sounds occur when there is fluid in the lungs. The absence of adventitious breath sounds indicates that the pulmonary edema is resolving.

A nurse in the ED is assessing a client for closed pneumothorax and significant bruising of the left chest following a MVA. The client reports severe left chest pain on inspiration. The nurse should assess the client for which of the following manifestations of pneumothorax? A. Absence of breath sounds B. Expiratory wheezing C. Inspiratory stridor D. Rhonchi

A. Absence of breath sounds RATIONALE: on the affected side B- asthma C- d/t obstruction D- thick sputum or foreign body will produce rhonchi, which are dry, low pitched snoring like noises produced in the throat

A nurse is rewarming a client following CABG surgery. For which of the following complications of rewarming should the nurse monitor the client? A. Acidosis B. Infection C. Hypertension D. Cardiac tamponade

A. Acidosis RATIONALE: Metabolic acidosis a/w hypoxia can occur if a client is rewarmed too quickly; acidosis develops after the client starts to shiver and increases myocardial oxygen consumption; rewarming of the client after CABG should occur at a rate no faster than 1.8F per hour

A nurse is caring for a client who has an abdominal aortic aneurysm and is scheduled for surgery. The client vital signs are blood pressure 160/90, heart rate 102/min, respirations 22/min, and SpO2 95%. Which of the following actions should the nurse take? A. Administer antihypertensive medication for blood pressure B. Monitor to ensure the client's urinary output is 20mL/hr C. Withhold pain medication to prepare the client for surgery D. Take the client's vital signs every 2hr

A. Administer antihypertensive medication for blood pressure RATIONALE: the BP can cause a sudden rupture of the aneurysm due to pressure on the arterial wall

A nurse is providing discharge instructions to a client who has a new laryngectomy. The nurse should tell the client to be careful while bathing to prevent which of the following complications? A. Aspiration of water B. Infection of the stoma C. Bleeding around the stoma D. Skin breakdown around the stoma

A. Aspiration of water The client should be careful during bathing and showering and should avoid swimming due to the risk of aspiration of water. The client should use a shower shield over the stoma when bathing or showering to keep water out of the airway.

A nurse is caring for a client who has an upper GI bleed and a hematocrit of 24%. Prior to initiating a transfusion of PRBCs, which of the following actions should the nurse take? SATA A. Assess and document the client's vital signs B. Restart the IV with a 22-gauge needle C. Verify with another nurse the blood type and Rh of the PRBCs D. Hang a bag of lactated ringers IV solution E. Change IV tubing to a set that has a filter

A. Assess and document the client's vital signs C. Verify with another nurse the blood type and Rh of the PRBCs E. Change IV tubing to a set that has a filter RATIONALE: NOT B- should be a 20gauge or larger needle; NOT D- 0.9% sodium chloride should be used; lactated ringers is not used because it causes clotting and hemolysis of the blood cells

A charge nurse receives notification of the admission of a client who is coughing frequently and whose sputum is pink, frothy, and copious. The client has a history of night sweats, anorexia and weight loss. Which of the following actions should the nurse take? SATA A. Assign the client to a private room with negative-pressure airflow B. Add contact precautions to the client's plan of care C. Wear a N95 respirator when entering the client's room D. Ensure the clients environment provides 4 exchanges of fresh air per minute E. Institute protective environment precautions as soon as the client arrives on the unit

A. Assign the client to a private room with negative-pressure airflow C. Wear a N95 respirator when entering the client's room RATIONALE: B- AIRBORNE precautions D- 6-12/minute E- this is for immunocompromised patients

A charge nurse receives notification of the admission of a client who is coughing frequently and whose sputum is pink, frothy, and copious. The client has a history of night sweats, anorexia, and weight loss. Which of the following actions should the nurse take? (Select all that apply.) A. Assign the client to a private room with negative-pressure airflow. B. Add contact precautions to the client's plan of care. C. Wear an N95 respirator when entering the client's room. D. Ensure the client's environment provides 4 exchanges of fresh air per minute. E. Institute protective environment precautions as soon as the client arrives on the unit.

A. Assign the client to a private room with negative-pressure airflow. C. Wear an N95 respirator when entering the client's room. This client's history and present status suggest tuberculosis, a communicable infection that mandates a private room with negative-pressure airflow. Airborne precautions will be required, including wearing an N95 respirator when entering the client's room.

A nurse is planning care for a client who has acute myelogenous leukemia and a platelet count of 48,000. Which of the following interventions should the nurse include? A. Avoid IM injections B. Assess the client for ecchymosis once per shift C. Do not allow the client to have visitors D. Encourage daily flossing between teeth

A. Avoid IM injections

The nurse is providing care for a patient with recently diagnosed asthma. Which key points would the nurse be sure to include in the teaching plan for this patient? Select all that apply. A. Avoid potential environmental asthma triggers such as smoke B. Use the inhaler 30 minutes before exercising to prevent bronchospasm C. Wash all bedding in cold water to reduce and destroy dust mites D. Be sure to get at least 8 hours of rest and sleep every night E. Avoid foods prepared with monosodium glutamate F. Keep a symptom and intervention diary to learn specific triggers for your asthma

A. Avoid potential environmental asthma triggers such as smoke B. Use the inhaler 30 minutes before exercising to prevent bronchospasm D. Be sure to get at least 8 hours of rest and sleep every night E. Avoid foods prepared with monosodium glutamate F. Keep a symptom and intervention diary to learn specific triggers for your asthma

A nurse is reviewing the medical record of a client who has heart failure. Which of the following findings should the nurse expect? A. BNP of 200 B. Bradycardia C. Fluid restriction of 3L/day D. 4g sodium diet

A. BNP of 200 RATIONALE: A client who has HF will have an elevated human B-type natriuretic peptide level of >100. Endogenous BNP is released into the clients bloodstream due to decreased cardiac output, a process called natriuresis B- should be tachycardia C- should be 2L/day D- should be 3g/day

A nurse is teaching a client who has iron deficiency anemia. The nurse should encourage the client to increase consumption of which of the following foods? A. Beef liver B. Oranges C. Turnips D. Whole milk

A. Beef liver

A nurse is teaching about a low-cholesterol diet to a client who had a myocardial infarction. Which of the following meal selections by the client indicates an understanding of the teaching? A. Chicken breast and corn on the cob B. Shrimp and rice C. Cheese omelet and turkey bacon D. Liver and onion s

A. Chicken breast and corn on the cob

A nurse is monitoring a group of clients for increased risk for developing pneumonia. Which of the following clients should the nurse expect to be at risk? (Select all that apply.) A. Client who has dysphagia B. Client who has AIDS C. Client who was vaccinated for pneumococcus and influenza 6 months ago D. Client who is postoperative and has received local anesthesia E. Client who has a closed head injury and is receiving mechanical ventilation F. Client who has myasthenia gravis

A. Client who has dysphagia B. Client who has AIDS E. Client who has a closed head injury and is receiving mechanical ventilation F. Client who has myasthenia gravis The client who has difficulty swallowing is at increased risk for pneumonia due to aspiration. The client who has AIDS is immunocompromised, which increases the risk of opportunistic infections, such as pneumonia. Mechanical ventilation is invasive and places the client at risk for ventilator-associated pneumonia. A client who has myasthenia gravis has generalized weakness and can have difficulty clearing airway secretions, which increases the risk of pneumonia.

4 pts assessing for good fluid balance...which client has dehydration? A. Client with temp of 100.2F B. Bp of 115/80 C. Respiratory acidosis D. Heart rate of 97

A. Client with temp of 100.2F

A nurse is caring for a client who had a MI 5 days ago. The client has a sudden onset of SOB and begins coughing frothy, pink sputum. The nurse auscultates loud, bubbly sounds on inspiration. Which of the following adventitious breath sounds should the nurse document? A. Coarse crackles B. Wheezes C. Rhonchi D. Friction rub

A. Coarse crackles RATIONALE: This is due to risk for left sided HF

A nurse in an ED is assessing a client who has bradydysrhythmia. Which of the following findings should the nurse monitor for? A. Confusion B. Friction rub C. Hypertension D. Dry skin

A. Confusion

A nurse is assessing a client in the emergency room who has a bradydysrhythmia. Which of the following findings should the nurse expect? A. Confusion B. Friction Rub C. Hypertension D. Dry Skin

A. Confusion Rationale: Bradydysrhythmia can cause decreased systemic perfusion, which can lead to confusion. Therefore, the nurse should monitor the client's mental status.

A nurse is reviewing the laboratory findings of a client who has protein-calorie malnutrition. Which of the following findings should the nurse expect? A. Decreased albumin B. Elevated hemoglobin C. Elevated lymphocytes D. Decreased cortisol

A. Decreased albumin

A nurse is admitting a client who reports nausea, vomiting, and weakness. The client has dry oral mucous membranes and blood pressure 102/64 mm Hg. Which of the following findings should the nurse identify as manifestations of fluid volume deficit? (Select all that apply.) A. Decreased skin turgor B. Concentrated urine C. Bradycardia D. Low‑grade fever E. Tachypnea

A. Decreased skin turgor B. Concentrated urine D. Low‑grade fever E. Tachypnea Decreased skin turgor is a manifestation present with fluid volume deficit. Skin turgor is decreased due to the lack of fluid within the body and results in dryness of the skin. Concentrated urine is a manifestation present with fluid volume deficit. Urine is concentrated due to lack of fluid in the vascular system, causing a decreased profusion of the kidneys and resulting in an increased urine specific gravity. Tachycardia is a manifestation present with fluid volume deficit due to an attempt to maintain a normal blood pressure. Low‑grade fever is a manifestation present with fluid volume deficit. Low‑grade fever is one of the body's ways to maintain homeostasis to compensate for lack of fluid within the body. Tachypnea is a manifestation present with fluid volume deficit. Increased respirations are the body's way to obtain oxygen due to the lack of fluid volume within the body. NCLEX® Connection: Physiological Adaptati

A nurse is assessing a client who has hyperkalemia. The nurse should identify which of the following conditions as being associated with this electrolyte imbalance? A. Diabetic ketoacidosis B. Heart failure C. Cushing's syndrome D. Thyroidectomy

A. Diabetic ketoacidosis Hyperkalemia, an increase in blood potassium, is a laboratory finding associated with diabetic ketoacidosis.

The emergency department nurse is monitoring a client who received treatment for a severe asthma attack. The nurse determines that the client's respiratory status has worsened if which is noted on assessment? A. Diminished breath sounds B. Wheezing during inhalation C. Wheezing during exhalation D. Wheezing throughout the lung fields

A. Diminished breath sounds

A nurse is caring for a client who smokes cigarettes and has a new diagnosis of emphysema. How should the nurse assist the client with smoking cessation? A. Discuss ways the client can reduce the number of cigarettes smoked per day B. Suggest the client switch from smoking cigarettes to smoking a pipe C. Inform the client that treatment will be ineffective if smoking continues D. Discourage the use of nicotine gum

A. Discuss ways the client can reduce the number of cigarettes smoked per day The nurse should discuss ways the client can reduce the number of cigarettes smoked per day to assist the client in creating a realistic goal to decrease smoking gradually.

A nurse is caring for a client who smokes cigarettes and has a new diagnosis of emphysema. How should the nurse assist the client with smoking cessation? A. Discuss ways the client can reduce the number of cigarettes smoked per day B. Suggest the client switch from smoking cigarettes to smoking a pipe C. Inform the client that treatment will be ineffective if smoking continues D. Discourage the use of nicotine gum

A. Discuss ways the client can reduce the number of cigarettes smoked per day RATIONALE: realistic and gradual

Pt has pancreatitis with ABG metabolic acidosis, what should you expect? (Select all that apply) A. Dizziness B. Dysrhythmias C. Muscle weakness D. Hypoventilation

A. Dizziness B. Dysrhythmias C. Muscle weakness

A nurse in an urgent care clinic is collecting data from a client who reports exposure to anthrax. Which of the following findings is an indication of the prodromal stage of inhalation anthrax? A. Dry cough B. Rhinitis C. Sore throat D. Swollen lymph nodes

A. Dry cough RATIONALE: During the prodromal stage, it is difficult to distinguish the condition from influenza or pneumonia because there is NO sore throat or rhinitis

A nurse is admitting an older adult client who reports a weight gain of 2.3 kg (5 lb) in 48 hr. Which of the following manifestations of fluid volume excess should the nurse expect? (Select all that apply.) A. Dyspnea B. Edema C. Bradycardia D. Hypertension E. Weakness

A. Dyspnea B. Edema D. Hypertension E. Weakness Dyspnea is a manifestation present with fluid volume excess. Dyspnea is due to an excess of fluids within the body and lungs, and the client is struggling to breathe to obtain oxygen. Edema is a manifestation present with fluid volume excess. Weight gain can be a result of edema. Tachycardia and bounding pulses are manifestations related to fluid volume excess. Hypertension is a manifestation related to fluid volume excess. Blood pressure rises as the heart must work harder due to the excess fluid. Weakness is a manifestation present with fluid volume excess. Weakness is due to the excess fluid that is retained, which depletes ener

A nurse is caring for a client following a thoracentesis. Which of the following manifestations should the nurse recognize as risks for complications? (Select all that apply.) A. Dyspnea B. Localized bloody drainage on the dressing C. Fever D. Hypotension E. Report of pain at the puncture site

A. Dyspnea C. Fever D. Hypotension Dyspnea can indicate a pneumothorax or a reaccumulation of fluid. The nurse should notify the provider immediately. Fever can indicate an infection. The nurse should notify the provider immediately. Hypotension can indicate intrathoracic bleeding. The nurse should notify the provider immediately.

A nurse is caring for a client who has dilated cardiomyopathy. Which of the following findings should the nurse expect? A. Dyspnea on exertion B. Tracheal deviation C. Pericardial rub D. Weight loss

A. Dyspnea on exertion Rationale: The nurse should identify dyspnea on exertion as an expected manifestation of dilated cardiomyopathy. Dyspnea on exertion is due to ventricular compromise and reduced cardiac output.

A nurse is caring for a client who has a blood potassium 5.4 mEq/L. The nurse should assess for which of the following manifestations? A. ECG changes B. Constipation C. Polyuria D. Paresthesia

A. ECG changes Assess for ECG changes. Potassium levels can affect the heart and result in arrhythmias.

A nurse is planning care for a client who has COPD and is malnourished. Which of the following recommendations to promote nutritional intake should the nurse include in the plan? A. Eat a high-calorie foods first B. Increase intake of water at meal times C. Perform active ROM exercises before meals D. Keep saltine crackers nearby for snacking

A. Eat a high-calorie foods first

A nurse is planning care for a client who has chronic obstructive pulmonary disease (COPD) and is malnourished. Which of the following recommendations to promote nutritional intake should the nurse include in the plan? A. Eat high-calorie foods first B. Increase intake of water at meal times C. Perform active range-of-motion exercises before meals D. Keep saltine crackers nearby for snacking

A. Eat high-calorie foods first Clients who have COPD often experience early satiety. Therefore, the client should eat calorie-dense foods first.

A nurse is teaching a client who has pernicious anemia. The nurse should encourage the client to increase consumption of which of the following foods? A. Eggs B. Squash C. Kale D. Tofu

A. Eggs RATIONALE: Other foods high in B12 are dairy products, animal protein, shellfish

A nurse is caring for a client who has femoral thrombophlebitis and a prescription for enoxaparin. Which of the following actions should the nurse take? A. Elevate the affected leg B. Place the client on bed rest C. Massage the affected leg D. Administer aspirin for discomfort

A. Elevate the affected leg

A nurse is examining the ECG of a client who has hyperkalemia. Which of the following ECG changes should the nurse expect? A. Elevated ST segments B. Absent P waves C. Depressed ST segments D. Varying PP intervals

A. Elevated ST segments

A nurse is reviewing the laboratory results of a client who has end-stage renal disease and reports fatigue. The clients hemoglobin level is 8g/dL. The nurse should expect a prescription for which of the following medications? A. Erythropoietin B. Erythromycin C. Filgrastim D. Calcitriol

A. Erythropoietin RATIONALE: Normal hemoglobin levels are 12-16; EPO stimulates production of RBCs to treat anemia a/w chronic renal failure -Filgrastim stimulates the production of neutrophils to treat neutropenia

A nurse is preparing a client for a bronchoscopy. Which of the following actions should the nurse take? SATA A. Explain that the client will receive sedation and will not remember the procedure B. Verify that the client understands the purpose and nature of the procedure C. Offer the client sips of clear liquids until 1hr before the test D. Obtain a pre-procedural specimen E. Instruct the client to keep his neck in a neutral position

A. Explain that the client will receive sedation and will not remember the procedure B. Verify that the client understands the purpose and nature of the procedure RATIONALE: C- NPO 6-8hrs prior D- the provider can obtain the specimen during the procedure E- neck will be hyperextended

A nurse is preparing a client for a bronchoscopy. Which of the following actions should the nurse take? (Select all that apply.) A. Explain that the client will receive sedation and will not remember the procedure. B. Verify that the client understands the purpose and nature of the procedure. C. Offer the client sips of clear liquids until 1 hr before the test. D. Obtain a pre-procedural sputum specimen. E. Instruct the client to keep his neck in a neutral position.

A. Explain that the client will receive sedation and will not remember the procedure. B. Verify that the client understands the purpose and nature of the procedure. For a bronchoscopy, clients typically receive premedication with a benzodiazepine or an opioid to ensure sedation and amnesia. The client will have signed a consent form, so the nurse should verify that the provider explained the procedure and that the client understands it.

A nurse is caring for a client who experienced defibrillation. Which of the following should be included in the documentation of this procedure? (Select all that apply.) A. Follow‑up ECG B. Energy settings used C. IV fluid intake D. Urinary output E. Skin condition under electrodes

A. Follow‑up ECG B. Energy settings used E. Skin condition under electrodes The client's ECG rhythm is documented following the procedure. Energy settings used during the procedure are documented The condition of the client's skin where electrodes were placed is documented.

While participating in a community health fair, a nurse is providing information to a client who has a blood pressure of 150/90 during screening. Which of the following actions should the nurse take? A. Give the client a written record of his BP to bring to his provider B. Encourage the client to go to the nearest emergency department C. Instruct the client to follow-up with a provider within 6 months D. Explain to the client that he is not at risk unless he has manifestations of hypertension

A. Give the client a written record of his BP to bring to his provider

Clinical manifestations of pernicious anemia? A. Glossitis B. Enlarging spleen C. Pale skin D. Tachycardia

A. Glossitis Glossitis: A smooth, red tongue Extreme weakness Jaundice: "pale yellow skin

A nurse is assessing a client who has DVT in her left calf. Which of the following manifestations should the nurse expect to find? SATA A. Hardening along the blood vessel B. Absence of peripheral pulse C. Tenderness in the calf D. Cool skin on the leg E. Increased leg circumference

A. Hardening along the blood vessel C. Tenderness in the calf E. Increased leg circumference

A patient with a history of hypertension treated with a diuretic and an angiotensin-converting enzyme (ACE) inhibitor arrives in the emergency department. The patient reports a severe headache and nausea and has a blood pressure (BP) of 238/118 mm Hg. Which question should the nurse ask to follow up on these findings? A. Have you been taking all of your blood pressure medications as directed? B. Did you consume more caffeine than usual today? C. Have there been any recent stressful events in your life? D. Have you recently taken any antihistamine medications?

A. Have you been taking all of your blood pressure medications as directed?

A nurse is caring for a client who has DIC. Which of the following medications should the nurse anticipate administering? A. Heparin B. Vitamin K C. Mefoxin D. Simvastatin

A. Heparin Heparin can be administered to decrease the formation of microclots, which deplete clotting factors.

Which topics will the nurse plan to include in discharge teaching for a patient who has been admitted with heart failure? Select all that apply. A. How to monitor and record daily weight B. Importance of stopping exercise if heart rate increases C. Symptoms of worsening heart failure D. Purpose of chronic antibiotic therapy E. How to read food labels for sodium content F. Date and time for follow-up appointments

A. How to monitor and record daily weight C. Symptoms of worsening heart failure E. How to read food labels for sodium content F. Date and time for follow-up appointments

A nurse is caring for a client who has emphysema and chronic respiratory acidosis. The nurse should monitor the client for which of the following electrolyte imbalances? A. Hyperkalemia B. Hyponatremia C. Hypercalcemia D. Hypomagnesemia

A. Hyperkalemia RATIONALE: High K= Low pH (acidosis)

A nurse is caring for a client who has emphysema and chronic respiratory acidosis. The nurse should monitor the client for which of the following electrolyte imbalances? A. Hyperkalemia B. Hyponatremia C. Hypercalcemia D. Hypomagnesemia

A. Hyperkalemia The nurse should monitor the client for hyperkalemia because chronic respiratory acidosis can result in high potassium levels due to potassium shifting out of the cells into the extracellular fluid.

A nurse is assessing a client who has heart failure and is taking daily furosemide. The client's apical pulse is weak and irregular. The nurse should identify these findings as manifestations of which of the following electrolyte imbalances? A. Hypokalemia B. Hypophosphatemia C. Hypercalcemia D. Hypermagnesemia

A. Hypokalemia RATIONALE: Furosemide (Lasix) causing loss of potassium, sodium, calcium and magnesium. S/S of hypokalemia include shallow respirations, muscle weakness, lethargy, and ectopic heartbeats

A nurse is assessing a client who has heart failure and is taking daily furosemide. The client's apical pulse is weak and irregular. The nurse should identify these findings as manifestations of which of the following electrolyte imbalances? A. Hypokalemia B. Hypophosphatemia C. Hypercalcemia D. Hypermagnesemia

A. Hypokalemia Furosemide can cause the loss of potassium, sodium, calcium, and magnesium. Manifestations of hypokalemia can include shallow respirations, muscle weakness, lethargy, and ectopic heartbeats.

A nurse is preparing to administer an initial dose of prednisone to a client who has COPD. The nurse should monitor for which of the following adverse effects of this medication? (Select all that apply.) A. Hypokalemia B. Tachycardia C. Fluid retention D. Nausea E. Black, tarry stools

A. Hypokalemia C. Fluid retention E. Black, tarry stools Observe for hypokalemia. This is an adverse effect of prednisone Observe for fluid retention. This is an adverse effect of prednisone Monitor for black, tarry stools. This is an adverse effect of prednisone.

A nurse on a medical-surgical unit is assessing a client who recently transferred from the ICU following endotracheal extubation. Which of the following findings should the nurse identify as a possible manifestation of tracheal stenosis and report to the provider? A. Increased coughing B. Diaphragmatic breathing C. Hemoptysis D. Kussmaul respirations

A. Increased coughing

A nurse on a medical-surgical unit is assessing a client who recently transferred from the ICU following endotracheal extubation. Which of the following findings should the nurse identify as a possible manifestation of tracheal stenosis and report to the provider? A. Increased coughing B. Diaphragmatic breathing C. Hemoptysis D. Kussmaul respirations

A. Increased coughing The nurse should identify increased coughing as a manifestation of tracheal stenosis. Other manifestations include an inability to cough up secretions and difficulty talking or breathing.

A nurse is assessing a client who has isotonic dehydration, Which of the following findings should the nurse expect? A. Increased hematocrit level B. Bradycardia C. Distended neck veins D. Decreased urine specific gravity

A. Increased hematocrit level RATIONALE: This is d/t hemoconcentration caused by reduced plasma fluid volume B- would have tachycardia C- would have flat neck veins D- would have increased urine specific gravity

A nurse is caring for a client who is receiving mechanical ventilation and develops acute respiratory distress. Which of the following actions should the nurse take first? A. Initiate bag-valve mask ventilation B. Provide the client with a communication board C. Obtain a blood sample for ABG analysis D. Document the ventilator settings

A. Initiate bag-valve mask ventilation

A nurse is caring for a client who is receiving mechanical ventilation and develops acute respiratory distress. Which of the following actions should the nurse take first? A. Initiate bag-valve-mask ventilation B. Provide the client with a communication board C. Obtain a blood sample for ABG analysis D. Document the ventilator settings

A. Initiate bag-valve-mask ventilation The nurse should apply the ABC priority-setting framework, which emphasizes the basic core of human functioning: having an open airway, being able to breathe in adequate amounts of oxygen, and circulating oxygen to the body's organs via the blood. An alteration in any of these areas can indicate a threat to life and is the nurse's priority concern. When applying the ABC priority-setting framework, airway is always the highest priority because the airway must be clear for oxygen exchange to occur. Breathing is the second-highest priority because adequate ventilatory effort is essential in order for oxygen exchange to occur. Circulation is the third-highest priority because the delivery of oxygen to critical organs only occurs if the heart and blood vessels are capable of efficiently carrying oxygen to them. Therefore, the nurse should first provide ventilations with a bag-valve-mask device.

A nurse is assessing a client who has fluid volume overload for a cardiovascular disorder. Which of the following manifestations should the nurse expect? SATA A. Jugular vein distention B. Moist crackles C. Postural hypotension D. Increased heart rate E. Fever

A. Jugular vein distention B. Moist crackles D. Increased heart rate

A nurse is teaching a client who has iron deficiency anemia. The nurse should encourage the client to increase her consumption of which of the following foods? A. Lentils B. Avocados C. Cabbage D. Broccoli

A. Lentils

If a pt has a calcium level of 12.3, what findings to expect during assessment? A. Lethargy B. Energetic C. Paresthesia D. Chvostek Signs

A. Lethargy

A nurse is planning a presentation about hypertension for a community women's group. Which of the following lifestyle modifications should the nurse include? (Select all that apply.) A. Limited alcohol intake B. Regular exercise program C. Decreased magnesium intake D. Reduced potassium intake E. Smoking cessation

A. Limited alcohol intake B. Regular exercise program E. Smoking cessation Rationale: Limited alcohol intake is correct. Clients who have hypertension should limit alcohol intake. Regular exercise program is correct. Clients who have hypertension should develop a regular exercise program to help reduce blood pressure. Decreased magnesium intake is incorrect. Low magnesium intake is associated with hypertension and is not a lifestyle modification the nurse should include. Reduced potassium intake is incorrect. Low potassium intake is associated with hypertension and is not a lifestyle modification the nurse should include. Tobacco cessation is correct. Clients who have hypertension should have a goal of tobacco cessation because tobacco use exacerbates hypertension.

The patient is admitted with hypercalcemia; polyuria; and pain in the pelvis, spine, and ribs with movement. Which hematologic problem is likely to display these manifestations in the patient? A. Multiple myeloma B. Thrombocytopenia C. Megaloblastic anemia D. Myelodysplastic syndrome

A. Multiple myeloma Multiple myeloma typically manifests with skeletal pain and osteoporosis that may cause hypercalcemia, which can result in polyuria, confusion, or cardiac problems. Serum hyperviscosity syndrome can cause renal, cerebral, or pulmonary damage. Thrombocytopenia, megaloblastic anemia, and myelodysplastic syndrome are not characterized by these manifestations.

A nurse is planning postoperative education for a client who will undergo a radical neck dissection for cancer of the larynx. The nurse should include which of the following topics? SATA A. NPO status B. Alternative methods of communication C. Endotracheal intubation D. Changes in body image E. Swallowing exercises

A. NPO status B. Alternative methods of communication D. Changes in body image E. Swallowing exercises RATIONALE: A- the client will receive fluids & nutrition via enteral tube while healing from the surgery E- contact SLP

The nurse notes that a client's arterial blood gas (ABG) results reveal a pH of 7.50 and a Paco2 of 30 mm Hg (30 mm Hg). The nurse monitors the client for which clinical manifestations associated with these ABG results? Select all that apply. A. Nausea B. Confusion C. Bradypnea D. Tachycardia E. Hyperkalemia F. Lightheadedness

A. Nausea B. Confusion D. Tachycardia F. Lightheadedness

A nurse is teaching a client who has coronary artery disease about the difference between angina pectoris and MI. Which of the following manifestations should the nurse identify as indications of MI? A. Nausea and vomiting B. Diaphoresis and dizziness C. Chest and left arm pain that subsides with rest D. Anxiety and feelings of doom E. Bounding pulse and bradypnea

A. Nausea and vomiting B. Diaphoresis and dizziness D. Anxiety and feelings of doom

A nurse is examining the ECG of a client who is having an acute MI. The nurse should identify that the elevated ST segments on the ECG indicate which of the following alterations? A. Necrosis B. Hypokalemia C. Hypomagnesemia D. Insufficiency

A. Necrosis

A nurse is caring for a client who, upon awakening, was disoriented to person, place, and time. The client reports chills and chest pain that is worse upon inspiration. Which of the following actions is the nursing priority? A. Obtain baseline vital signs and oxygen saturation. B. Obtain a sputum culture. C. Obtain a complete history from the client. D. Provide a pneumococcal vaccine.

A. Obtain baseline vital signs and oxygen saturation. The first action the nurse should take using the nursing process is to assess the client in order to determine the next nursing intervention.

A nurse is completing a medication history for a client who reports using fish oil as a dietary supplement. Which of the following substances in fish oil should the nurse recognize as a health benefit to the client? A. Omega 3 fatty acids B. Antioxidants C. Vitamins A, D, and C D. Beta-carotene

A. Omega 3 fatty acids

A nurse is caring for a client who is scheduled for a thoracentesis. Which of the following supplies should the nurse ensure are in the client's room? (Select all that apply.) A. Oxygen equipment B. Incentive spirometer C. Pulse oximeter D. Sterile dressing E. Suture removal kit

A. Oxygen equipment C. Pulse oximeter D. Sterile dressing Oxygen equipment is necessary to have in the client's room if the client becomes short of breath following the procedure. Pulse oximetry is necessary to monitor oxygen saturation level during the procedure. A sterile dressing is necessary to apply to the puncture site following the procedure.

A nurse is checking paradoxical blood pressure of a client who has a possible cardiac tamponade. In what order should the nurse complete the following steps? A. Palpate the blood pressure and inflate the cuff above the systolic pressure B. Identify the first BP sounds audible on expiration and then on inspiration C. Inspect for jugular venous distention and notify the provider D. Deflate the cuff slowly and listen for the first audible sounds E. Subtract the inspiratory pressure from the expiratory pressure

A. Palpate the blood pressure and inflate the cuff above the systolic pressure D. Deflate the cuff slowly and listen for the first audible sounds B. Identify the first BP sounds audible on expiration and then on inspiration E. Subtract the inspiratory pressure from the expiratory pressure C. Inspect for jugular venous distention and notify the provider

A nurse is caring for an older adult client who had an acute MI. When assessing this client, the nurse should identify that older adults are prone to complications from MI from poor tissue perfusion because of which of the following age related factors? A. Peripheral vascular resistance increases B. The sensitivity of blood pressure-adjusting baroreceptors increases C. Blood is hypercoaguable and clots more quickly D. Cardiac medications are less effective

A. Peripheral vascular resistance increases

A nurse is providing information about tuberculosis to a group of clients at a local community center. Which of the following manifestations should the nurse include? (Select all that apply.) A. Persistent cough B. Weight gain C. Fatigue D. Night sweats E. Purulent sputum

A. Persistent cough C. Fatigue D. Night sweats E. Purulent sputum A persistent cough is a manifestation of tuberculosis. Fatigue is a manifestation of tuberculosis. Night sweats is a manifestation of tuberculosis. Purulent sputum is a manifestation of tuberculosis.

A nurse is caring for a client who has disseminated intravascular coagulation (DIC). Which of the following laboratory values indicates the client's clotting factors are depleted? (Select all that apply.) A. Platelets 100,000/mm3 B. Fibrinogen levels 120 mg/dL C. Fibrin degradation products 4.3 mcg/mL D. D‑dimer 0.03 mcg/mL E. Sedimentation rate 38 mm/hr

A. Platelets 100,000/mm3 B. Fibrinogen levels 120 mg/dL In DIC, platelet levels are decreased, causing clotting factors to become depleted. Clotting times are increased, which raises the risk for fatal hemorrhage In DI C, fibrinogen levels are decreased, causing clotting factors to become depleted. Clotting times are increased, which raises the risk for fatal hemorrhage

A nurse is caring for a client who is scheduled for a thoracentesis. Prior to the procedure, which of the following actions should the nurse take? A. Position the client in an upright position, leaning over the bedside table. B. Explain the procedure. C. Obtain ABGs. D. Administer benzocaine spray.

A. Position the client in an upright position, leaning over the bedside table. Positioning the client in an upright position and bent over the bedside table widens the intercostal space for the provider to access the pleural fluid.

A nurse is caring for a client who is postoperative following vein ligation and stripping for varicose veins. Which of the following actions should the nurse take? A. Position the client supine with his legs elevated when in bed B. Encourage the client to ambulate for 15 minutes every hour while awake for the first 24 hours C. Tell the client to sit with his legs dependent after ambulating D. Instruct the client to wear knee-length socks for 2 weeks after surgery

A. Position the client supine with his legs elevated when in bed RATIONALE: Legs should be elevated above the clients heart to promote venous return via gravity; it will also be important following discharge to periodically position the legs above the heart B- should be 5-10 minutes every hour while awake C- discourage dependent sitting to prevent venous stasis D- wear graduated compression stockings for up to 1 week after surgery

A nurse is caring for a client with heart failure whose telemetry reading displays a flattening T wave. Which of the following laboratory results should the nurse anticipate as the cause of this ECG change? A. Potassium 2.8 mEq/L B. Digoxin level 0.7ng/mL C. Hemoglobin 9.8g/dL D. Calcium 8.0 mg

A. Potassium 2.8 mEq/L RATIONALE: A flattened T wave or the development of U waves is indicative of a low potassium level -Digoxin therapeutic level is 0.5-0.8 -A low hemoglobin manifests as tachycardia on the ECG (normal range 12-18) -A low calcium level can manifest as a prolonged S-T segment and a prolonged QT interval

A nurse is reviewing a client's repeat laboratory results 4 hours after administering fresh frozen plasma (FFP). Which of the following laboratory results should the nurse review? A. Prothrombin time B. WBC count C. Platelet count D. Hematocrit

A. Prothrombin time RATIONALE: FFP's are rick in clotting factors and is administered to treat acute clotting disorders. The desired effect is a decrease in the prothrombin time

A nurse is planning care for a client who has Hgb 7.5 g/dL and Hct 21.5%. Which of the following actions should the nurse include in the plan of care? (Select all that apply.) A. Provide assistance with ambulation. B. Monitor oxygen saturation. C. Weigh the client weekly. D. Obtain stool specimen for occult blood. E. Schedule daily rest periods

A. Provide assistance with ambulation. B. Monitor oxygen saturation. D. Obtain stool specimen for occult blood. E. Schedule daily rest periods Assist the client when ambulating to prevent a fall because the client who has anemia can experience dizziness. Monitor oxygen saturation when the client has anemia due to the decreased Obtain the client's stool to test for occult blood, which can identify a possible cause of anemia caused from gastrointestinal bleeding. Schedule the client to rest throughout the day because the client who has anemia can experience fatigue. Rest periods should be planned to conserve energy.

Which assessment finding would support the presence of a hemostasis abnormality? A. Purpura B. Pruritus C. Weakness D. Pale conjunctiva

A. Purpura Purpura may occur when platelets or clotting factors are decreased and bleeding into the skin occurs. Pruritus is not related to hemostasis but to hematologic cancers (e.g., lymphomas, leukemias) or increased bilirubin. Weakness and pale conjunctiva are not related to hemostasis unless a lot of bleeding leads to anemia with low hemoglobin level.

Which condition can cause an acid base imbalance? (Select all that apply) A. Renal impairment B. High bp C. Vomiting D. Anxiety E. High altitudes

A. Renal impairment C. Vomiting D. Anxiety E. High altitudes

The nurse is caring for a client with several broken ribs. The client is most likely to experience what type of acid-base imbalance? A. Respiratory acidosis from inadequate ventilation B. Respiratory alkalosis from anxiety and hyperventilation C. Metabolic acidosis from calcium loss due to broken bones D. Metabolic alkalosis from taking analgesics containing base products

A. Respiratory acidosis from inadequate ventilation

What are the TB meds to prescribe when TB is active? A. Rifampicin B. Albuterol C. Corticosteroid D. Isoniazid E. Ethambutol

A. Rifampicin D. Isoniazid E. Ethambutol

The nurse is teaching a client with emphysema about positions that help breathing during dyspneic episodes. The nurse instructs the client that which positions alleviate dyspnea? Select all that apply. A. Sitting up and leaning on a table B. Standing and leaning against a wall C. Lying supine with the feet elevated D. Sitting up with elbows resting on knees E. Lying on the back in a low-Fowler's position

A. Sitting up and leaning on a table B. Standing and leaning against a wall D. Sitting up with elbows resting on knees

A nurse is caring for a client who has HF and is experiencing atrial fibrillation. Which of the following findings should the nurse plan to monitor for and report to the provider immediately? A. Slurred speech B. Irregular pulse C. Dependent edema D. Persistent fatigue

A. Slurred speech RATIONALE: This indicates impaired circulation to the brain

A nurse is caring for a client who has heart failure and is experiencing atrial fibrillation. The nurse should plan to monitor for and report which of the following findings to the provider immediately? A. Slurred speech B. Irregular pulse C. Dependent edema D. Persistent fatigue

A. Slurred speech Rationale: The greatest risk to this client is injury from an embolus caused by the pooling of blood that can occur with atrial fibrillation. Slurred speech can indicate inadequate circulation to the brain because of an embolus. Therefore, the nurse should report this finding to the provider immediately.

What are precipitating factors for angina? (Select all that apply) A. Smoking B. Walking C. Eating D. Exertion E. Soak in hot tub

A. Smoking B. Eating D. Exertion E. Soaking in a hot tub

A nurse in a medical-surgical unit is assessing a client. The nurse should identify that which of the following findings is a manifestation of a pulmonary embolism? A. Stabbing chest pain B. Calf tenderness C. Elevated temperature D. Bradycardia

A. Stabbing chest pain RATIONALE: also dyspnea, coughing, hemoptysis, tachypnea, tachycardia, diaphoresis, and feeling of impending doom

A nurse is admitting a client who has a suspected myocardial infarction (MI) and a history of angina. Which of the following findings will help the nurse distinguish stable angina from an MI? A. Stable angina can be relieved with rest and nitroglycerin. B. The pain of an MI resolves in less than 15 min. C. The type of activity that causes an MI can be identified. D. D. Stable angina can occur for longer than 30 min

A. Stable angina can be relieved with rest and nitroglycerin. Stable angina can be relieved by rest and nitroglycerin

A nurse is caring for a client who is receiving a blood transfusion. Which of the following actions should the nurse plan to take if an allergic transfusion reaction is suspected? (Select all that apply.) A. Stop the transfusion. B. Monitor for hypertension. C. Maintain an IV infusion with 0.9% sodium chloride. D. Position the client in an upright position with the feet lower than the heart. E. Administer diphenhydramine.

A. Stop the transfusion. C. Maintain an IV infusion with 0.9% sodium chloride. E. Administer diphenhydramine. Immediately stop the infusion if an allergic transfusion reaction is suspected. Administer 0.9% sodium chloride solution through new IV tubing if an allergic transfusion reaction is suspected. Administer an antihistamine, such as diphenhydramine, if an allergic transfusion reaction is suspected.

A nurse is caring for a client immediately following extubation. Which of the following manifestations indicates that the nurse should call the rapid response team? A. Stridor B. Coughing C. Hoarseness D. Extensive oral secretions

A. Stridor RATIONALE: High pitched sound heard during inspiration= laryngeal edema and can indicate impending airway obstruction

A nurse is caring for a client immediately following extubation. Which of the following manifestations indicates that the nurse should call the rapid response team? A. Stridor B. Coughing C. Hoarseness D. Extensive oral secretions

A. Stridor The nurse should identify that stridor (a high-pitched crowing sound heard during inspiration) is caused by laryngeal edema and can indicate impending airway obstruction. The nurse should call the rapid response team for assistance before the airway becomes completely obstructed.

A nurse on a medical-surgical unit is caring for a client who is postoperative following a hip replacement surgery. The client reports feeling apprehensive and restless. Which of the following findings should the nurse recognize as an indication of pulmonary embolism? A. Sudden onset of dyspnea B. Tracheal deviation C. Bradycardia D. Difficulty swallowing

A. Sudden onset of dyspnea

A nurse is caring for a client who has a demand pacemaker inserted with a set rate of 72/min. Which of the following findings should the nurse expect? A. Telemetry monitoring showing QRS complexes occurring at a rate of 74/min with no pacing spikes B. Premature ventricular complexes at 12/min C. Telemetry monitoring showing pacing spikes with no QRS complexes D. Hiccups

A. Telemetry monitoring showing QRS complexes occurring at a rate of 74/min with no pacing spikes

Which factor results from pernicious anemia? A. The absence of intrinsic factor secreted by the gastric mucosa B. The absence of folic acid secreted by the gastric mucosa C. The absence of extrinsic factor secreted by the gastric mucosa D. Cobalamin intake

A. The absence of intrinsic factor secreted by the gastric mucosa Inadequate production of intrinsic factor also leads to vitamin B12 deficiency.

A nurse is caring for a client who has a blood sodium level 133 mEq/L and blood potassium level 3.4 mEq/L. The nurse should recognize that which of the following treatments can result in these laboratory findings? A. Three tap water enemas B. 0.9% sodium chloride solution IV at 50 mL/hr C. 5% dextrose with 0.45% sodium chloride solution with 20 mEq of K+ IV at 80 mL/hr D. Antibiotic therapy

A. Three tap water enemas Three tap water enemas can result in a decrease in blood sodium and potassium. Tap water is hypotonic, and gastrointestinal losses are isotonic. This creates an imbalance and solute dilution.

A nurse is providing teaching to a client about pulmonary function testing. Which of the following tests measures the volume of air the lungs can hold at the end of maximum inhalation? A. Total lung capacity B. Vital lung capacity C. Functional residual capacity D. Residual volume

A. Total lung capacity Pulmonary function tests are used to examine the effectiveness of the lungs and to identify lung problems. Total lung capacity measures the amount of air the lungs can hold after maximum inhalation.

A nurse is providing teaching to a client about pulmonary function testing. Which of the following tests measures the volume of air the lungs can hold at the end of maximum inhalation? A. Total lung capacity B. Vital lung capacity C. Functional residual capacity D. Residual volume

A. Total lung capacity RATIONALE: B- measures the amount of air the client can exhale after maximum inhalation C- measures the amount of air in the lungs after normal expiration D- Amount of air in the lungs after forced expiration

A nurse is caring for a client following a right pleural thoracentesis. The nurse measures a total of 35mL of purulent drainage. Which of the following findings should the nurse recognize as an indication of a tension pneumothorax? SATA A. Tracheal deviation to the left B. Temperature of 102F C. Absent breath sounds on the right side D. Neck vein distention E. Bradypnea

A. Tracheal deviation to the left C. Absent breath sounds on the right side D. Neck vein distention

A nurse is reviewing the menu selections of a client who has heart failure and anticipates discharge to home the following day. Which of the following lunch choices should the nurse identify as an indication that the client understands his dietary instructions? A. Turkey on whole-wheat bread bread B. Hamburger and french fries C. Frankfurter on a white roll D. Macaroni and cheese

A. Turkey on whole-wheat bread bread

A nurse is providing discharge teaching about improving gas exchange for a client who has emphysema. Which of the following instructions should the nurse include in the teaching? A. Use pursed-lip breathing during periods of dyspnea B. Limit fluid intake to 1500mL/day C. Practice chest breathing each day D. Wear home oxygen to maintain an SaO2 of at least 94%

A. Use pursed-lip breathing during periods of dyspnea

A nurse is providing discharge teaching about improving gas exchange for a client who has emphysema. Which of the following instructions should the nurse include in the teaching? A. Use pursed-lip breathing during periods of dyspnea B. Limit fluid intake to 1,500 mL per day C. Practice chest breathing each day D. Wear home oxygen to maintain an SaO2 of at least 94%

A. Use pursed-lip breathing during periods of dyspnea The nurse should instruct the client about using pursed-lip breathing during periods of dyspnea to slow expiration, increase airway pressure, and facilitate effective gas exchange.

A nurse is monitoring a client for reperfusion following thrombolytic therapy to treat acute myocardial infarction (MI). Which of the following indicators should the nurse identify to confirm reperfusion? A. Ventricular dysrhythmias B. Appearance of Q waves C. Elevated ST segments D. Recurrence of chest pain

A. Ventricular dysrhythmias RATIONALE: Ventricular dysrhythmias following thrombolytic therapy is a sign of reperfusion of the coronary artery

A nurse is teaching a client who heart failure and new prescriptions for furosemide and digoxin. Which of the following information should the nurse include? (Select all that apply). A. Weigh daily, first thing each morning. B. Decrease intake of potassium. C. Expect muscle weakness while taking digoxin. D. Hold digoxin if heart rate is less than 70/min. E. Decrease sodium intake.

A. Weigh daily, first thing each morning. E. Decrease sodium intake. Weighing daily when first getting out of bed will assist the client in tracking fluid loss and gain. Decrease sodium intake to prevent fluid retention, which could worsen heart failure manifestations.

A nurse is assessing a client who has late-stage heart failure and is experiencing fluid volume overload. Which of the following findings should the nurse expect? A. Weight gain of 1kg (2.2lb) in 1 day B. Pitting edema +1 C. Client report of a nocturnal cough D. B-type natriuretic peptide (BNP) level of 100

A. Weight gain of 1kg (2.2lb) in 1 day RATIONALE: Worsening HF

A nurse is providing discharge teaching for a client who has heart failure. The nurse should instruct the client to report which of the following findings immediately to the provider? A. Weight gain of 2 lb. in 24 hr b. Increase of 10 mmhg in systolic BP c. Dyspnea with exertion d. Dizziness when rising quickly

A. Weight gain of 2 lb. in 24 hr Rationale: When using the urgent vs. nonurgent approach to client care, the nurse should determine that the priority finding is a weight gain of 0.5 to 0.9 kg (1.1 to 2 lb) in 1 day. This weight gain is an indication of fluid retention resulting from worsening heart failure. The client should report this finding immediately.

A nurse is providing preoperative teaching to a client who has lung cancer and will undergo a pneumonectomy. Which of the following statements should the nurse include? SATA A. You will have a chest tube in place after surgery B. We'll frequently help you turn, cough, and breathe deeply after surgery C. You will have to remain in bed for about 2 days after the surgery D. We'll give you oxygen to support your breathing if you need it E. You should expect pain for the first few days after surgery

A. You will have a chest tube in place after surgery B. We'll frequently help you turn, cough, and breathe deeply after surgery D. We'll give you oxygen to support your breathing if you need it

In a severely anemic patient, you expect to find A. dyspnea and tachycardia. B. cyanosis and pulmonary edema. C. cardiomegaly and pulmonary fibrosis. D. ventricular dysrhythmias and wheezing.

A. dyspnea and tachycardia. Rationale: Patients with severe anemia (hemoglobin <6 g/dL) exhibit the following cardiovascular and pulmonary manifestations: tachycardia, increased pulse pressure, systolic murmurs, intermittent claudication, angina, heart failure, myocardial infarction; tachypnea, orthopnea, dyspnea at rest.

A nurse is caring for a client who was in a motor‑vehicle accident. The client reports chest pain and difficulty breathing. A chest x‑ray reveals the client has a pneumothorax. Which of the following arterial blood gas findings should the nurse expect? A. pH 7.06 PaO2 86 mm Hg PaCO2 52 mm Hg HCO3 − 24 mEq/L B. pH 7.42 PaO2 100 mm Hg PaCO2 38 mm Hg HCO3 − 23 mEq/L C. pH 6.98 PaO2 100 mm Hg PaCO2 30 mm Hg HCO3 − 18 mEq/L D. pH 7.58 PaO2 96 mm Hg PaCO2 38 mm Hg HCO3 − 29 mEq/L

A. pH 7.06 PaO2 86 mm Hg PaCO2 52 mm Hg HCO3 − 24 mEq/L A pneumothorax can cause alveolar hypoventilation and increased carbon dioxide levels, resulting in a state of respiratory acidosis.

a nurse receives prescriptions from the provider for performing nasopharyngeal suctioning on 4 clients. for which of the following clients should the nurse clarify the provider's prescription? A.) A client who has epistaxis B.) A client who has amyotrophic lateral sclerosis C.) A client who has pneumonia D.) A client who has emphysema

A.) A client who has epistaxis rationale: The nurse should avoid providing nasopharyngeal suctioning for a client who has nasal bleeding because this intervention might cause an increase in bleeding.

a nurse is caring for 4 clients. which of the following clients is at greatest risk for a pulmonary embolism? A.) A client who is 48 hr postoperative following a total hip arthroplasty B.) A client who is 8 hr postoperative following an open surgical appendectomy C.) A client who is 2 hr postoperative following an open reduction external fixation of the right radius D.) A client who is 4 hr postoperative following a laparoscopic cholecystectomy

A.) A client who is 48 hr postoperative following a total hip arthroplasty rationale: The nurse should identify that a client who has undergone a total hip arthroplasty surgery is at greatest risk for a pulmonary embolus because of decreased mobility of the affected extremity and an increased amount of blood clots forming in the veins of the thigh following hip surgery. Deep-vein thromboses are most likely to occur 48 to 72 hr following the arthroplasty. The nurse should intervene to reduce the risk by applying sequential compression devices or antiembolic stockings and by administering anticoagulant medications.

a nurse developing a plan of care for a client who has active TB. which of the following isolation precautions should the nurse include in the plan? A.) Airborne B.) Neutropenic C.) Contact D.) Droplet

A.) Airborne rationale: The nurse should initiate airborne precautions for a client who has tuberculosis because tuberculosis is a respiratory infection that is spread through the air. The client should be placed in a room with negative airflow pressure that is filtered through a high-efficiency particulate air (HEPA) filter. Members of the health care team should not enter the client's room without wearing an N95 respirator mask.

a nurse in an ED is caring for a client who's experiencing a pulmonary embolism. which of the following actions should the nurse take first? A.) Apply supplemental oxygen. B.) Increase the rate of IV fluids. C.) Administer pain medication. D.) Initiate cardiac monitoring.

A.) Apply supplemental oxygen. rationale: When using the airway, breathing, circulation approach to client care, the greatest risk to the client is severe hypoxemia. Therefore, the first action the nurse should take is to apply supplemental oxygen.

a nurse is assessing a client who has lung cancer. which of the following manifestations should the nurse expect? A.) Blood-tinged sputum B.) Decreased tactile fremitus C.) Resonance with percussion D.) Peripheral edema

A.) Blood-tinged sputum rationale: The nurse should expect blood-tinged sputum secondary to bleeding from the tumor.

a nurse is preparing a client for discharge following a bronchoscopy with the use of moderate sedation. the nurse should identify that which of the following assessments if the priority? A.) presence of gag reflex B.) pain level rating using 0 to 10 scale C.) hydration status D.) appearance of the IV insertion site

A.) presence of gag reflex rationale: The greatest risk to the client is aspiration due to a depressed gag reflex. Therefore, the priority assessment by the nurse is to determine the return of the gag reflex.

A nurse is reviewing the ECG rhythm strip of a client who is receiving telemetry. Identify the area of the strip the nurse should examine to observe for atrial depolarization. (You will find "Hot Spots" to select in the artwork below. Select only the hotspot that corresponds to your answer.)

A: P wave Rationale: you observe the p wave for atrial depolarization.......so thats A.

The nurse obtains a blood pressure of 176/82 mm Hg for a patient. What is the patient's mean arterial pressure (MAP)?

ANS: 113 mm Hg MAP = (SBP + 2 DBP)/3 DIF: Cognitive Level: Apply (application)

The nurse conducts a complete physical assessment on a patient admitted with infective endocarditis. Which finding is significant? A. Regurgitant murmur at the mitral valve area B. Point of maximal impulse palpable in fourth intercostal space C. Heart rate of 94 beats/min and capillary refill time of 2 seconds D. Respiratory rate of 18 breaths/min and heart rate of 90 beats/min

ANS: A A regurgitant murmur of the aortic or mitral valves would indicate valvular disease, which is a complication of endocarditis. All the other findings are within normal limits.

While admitting a patient with pericarditis, the nurse will assess for what manifestations of this disorder? A. Pulsus paradoxus B. Prolonged PR intervals C. Widened pulse pressure D. Clubbing of the fingers

ANS: A Pericarditis can lead to cardiac tamponade, an emergency situation. Pulsus paradoxus greater than 10 mm Hg is a sign of cardiac tamponade that should be assessed at least every 4 hours in a patient with pericarditis. Prolonged PR intervals occur with first-degree AV block. Widened pulse pressure occurs with valvular heart disease and increased intracranial pressure. Clubbing of fingers may occur in subacute forms of infective endocarditis and valvular heart disease.

The nurse is teaching a community group about preventing rheumatic fever. What information should the nurse include? A. Prompt recognition and treatment of streptococcal pharyngitis B. Avoiding respiratory infections in children born with heart defects C. Completion of 4 to 6 weeks of antibiotic therapy for infective endocarditis D. Requesting antibiotics before dental surgery for individuals with rheumatoid arthritis

ANS: A The nurse should emphasize the need for prompt and adequate treatment of streptococcal pharyngitis infection, which can lead to the complication of rheumatic fever.

A patient who has myocarditis now has fatigue, weakness, palpitations, and dyspnea at rest. The nurse assesses pulmonary crackles, edema, and weak peripheral pulses. Sinoatrial tachycardia is evident on the cardiac monitor. The Doppler echocardiography shows dilated cardiomyopathy. What should collaborative care accomplish to improve cardiac output and quality of life? A. Decrease preload and afterload. B. Relieve left ventricular outflow obstruction. C. Improve diastolic filling and the underlying disease process. D. Improve ventricular filling by reducing ventricular contractility.

ANS: A The patient has developed dilated cardiomyopathy. To improve cardiac output and quality of life, drug, nutrition, and cardiac rehabilitation will be focused on controlling heart failure by decreasing preload and afterload. Relief of left ventricular outflow obstruction and improving ventricular filling by reducing ventricular contractility is done for hypertrophic cardiomyopathy. There are no specific treatments for restrictive cardiomyopathy, but interventions are aimed at improving diastolic filling and the underlying disease process.

The nurse determines that a patient's pedal pulses are absent. What factor could contribute to this finding? a. Atherosclerosis b. Hyperthyroidism c. Atrial dysrhythmias d. Arteriovenous fistula

ANS: A Atherosclerosis can cause an absent peripheral pulse. The feet would also be cool and may be discolored. Hyperthyroidism causes a bounding pulse. Arteriovenous fistula gives a thrill or vibration to the vessel, although this would not be in the foot. Cardiac dysrhythmias cause an irregular pulse rhythm.

A patient with a history of myocardial infarction is scheduled for a transesophageal echocardiogram to visualize a suspected clot in the left atrium. What information should the nurse include when teaching the patient about this diagnostic study? a. IV sedation may be administered to help the patient relax. b. Food and fluids are restricted for 2 hours before the procedure. c. Ambulation is restricted for up to 6 hours before the procedure. d. Contrast medium is injected into the esophagus to enhance images.

ANS: A IV sedation is administered to help the patient relax and ease the insertion of the tube into the esophagus. Food and fluids are restricted for at least 6 hours before the procedure. Smoking and exercise are restricted for 3 hours before exercise or stress testing but not before TEE. Contrast medium is administered IV to evaluate the direction of blood flow if a septal defect is suspected.

The charge nurse observes a new registered nurse (RN) doing discharge teaching for a patient with hypertension who has a new prescription for enalapril (Vasotec). The charge nurse will need to intervene if the new RN tells the patient to a. increase the dietary intake of high-potassium foods. b. make an appointment with the dietitian for teaching. c. check the blood pressure (BP) at home at least once a day. d. move slowly when moving from lying to sitting to standing.

ANS: A The ACE inhibitors cause retention of potassium by the kidney, so hyperkalemia is a possible adverse effect. The other teaching by the new RN is appropriate for a patient with newly diagnosed hypertension who has just started therapy with enalapril. DIF: Cognitive Level: Apply (application)

The nurse is reviewing the laboratory test results for a patient who has recently been diagnosed with hypertension. Which result is most important to communicate to the health care provider? a. Serum creatinine of 2.8 mg/dL b. Serum potassium of 4.5 mEq/L c. Serum hemoglobin of 14.7 g/dL d. Blood glucose level of 96 mg/dL

ANS: A The elevated serum creatinine indicates renal damage caused by the hypertension. The other laboratory results are normal. DIF: Cognitive Level: Analyze (analysis)

Which nursing action should the nurse take first to assist a patient with newly diagnosed stage 1 hypertension in making needed dietary changes? a. Collect a detailed diet history. b. Provide a list of low-sodium foods. c. Help the patient make an appointment with a dietitian. d. Teach the patient about foods that are high in potassium.

ANS: A The initial nursing action should be assessment of the patient's baseline dietary intake through a thorough diet history. The other actions may be appropriate, but assessment of the patient's baseline should occur first. DIF: Cognitive Level: Analyze (analysis)

The nurse on the intermediate care unit received change-of-shift report on four patients with hypertension. Which patient should the nurse assess first? a. 48-yr-old with a blood pressure of 160/92 mm Hg who reports chest pain b. 52-yr-old with a blood pressure of 198/90 mm Hg who has intermittent claudication c. 50-yr-old with a blood pressure of 190/104 mm Hg who has a creatinine of 1.7 mg/dL d. 43-yr-old with a blood pressure of 172/98 mm Hg whose urine shows microalbuminuria

ANS: A The patient with chest pain may be experiencing acute myocardial infarction and rapid assessment and intervention are needed. The symptoms of the other patients also show target organ damage but are not indicative of acute processes. DIF: Cognitive Level: Analyze (analysis)

When caring for a patient with infective endocarditis, the nurse will assess the patient for which vascular manifestations? (Select all that apply.) A. Osler's nodes B. Janeway's lesions C. Splinter hemorrhages D. Subcutaneous nodules E. Erythema marginatum lesions

ANS: A, B, C Osler's nodes, Janeway's lesions, and splinter hemorrhages are all vascular manifestations of infective endocarditis. Subcutaneous nodules and erythema marginatum lesions occur with rheumatic fever.

What age-related cardiovascular changes should the nurse assess for when providing care to an older adult patient? (Select all that apply.) a. Systolic murmur b. Diminished pedal pulses c. Increased maximal heart rate d. Decreased maximal heart rate e. Increased recovery time from activity

ANS: A, B, D, E Well-documented cardiovascular effects of the aging process include valvular rigidity leading to systolic murmur, arterial stiffening leading to diminished pedal pulses or possible increased blood pressure, and an increased amount of time that is required for recovery from activity. Maximal heart rate tends to decrease rather than increase with age related to cellular aging and fibrosis of the conduction system.

The nurse provides discharge instructions for a 40-yr-old woman newly diagnosed with cardiomyopathy. Which statement indicates that further teaching is necessary? A. "I will avoid lifting heavy objects." B. "I can drink alcohol in moderation." C. "My family will need to take a CPR course." D. "I will reduce stress by learning guided imagery."

ANS: B Patients with cardiomyopathy should avoid alcohol consumption, especially in patients with alcohol-related dilated cardiomyopathy. Avoiding heavy lifting and stress, as well as family members learning CPR, are recommended teaching points.

What should the nurse teach the patient who has had a valve replacement with a biologic valve? A. Long-term anticoagulation therapy B. Antibiotic prophylaxis for dental care C. Exercise plan to increase cardiac tolerance D. β-Adrenergic blockers to control palpitations

ANS: B The patient will need to use antibiotic prophylaxis for dental care to prevent endocarditis. Long-term anticoagulation therapy is not used with biologic valve replacement unless the patient has atrial fibrillation. An exercise plan to increase cardiac tolerance is needed for a patient with heart failure. Taking β-adrenergic blockers to control palpitations is prescribed for mitral valve prolapse, not valve replacement.

What position should the nurse place the patient in to auscultate for signs of acute pericarditis? a. Supine without a pillow b. Sitting and leaning forward c. Left lateral side-lying position d. Head of bed at a 45-degree angle

ANS: B A pericardial friction rub indicates pericarditis. To auscultate a pericardial friction rub, the patient should be sitting and leaning forward. The nurse will hear the pericardial friction rub at the end of expiration.

The nurse has just finished teaching a hypertensive patient about the newly prescribed drug, ramipril (Altace). Which patient statement indicates that more teaching is needed? a. "The medication may not work well if I take aspirin." b. "I can expect some swelling around my lips and face." c. "The doctor may order a blood potassium level occasionally." d. "I will call the doctor if I notice that I have a frequent cough."

ANS: B Angioedema occurring with angiotensin-converting enzyme (ACE) inhibitor therapy is an indication that the ACE inhibitor should be discontinued. The patient should be taught that if any swelling of the face or oral mucosa occurs, the health care provider should be immediately notified because this could be life threatening. The other patient statements indicate that the patient has an accurate understanding of ACE inhibitor therapy. DIF: Cognitive Level: Apply (application)

Which assessment finding for a patient who is receiving IV furosemide (Lasix) to treat stage 2 hypertension is most important to report to the health care provider? a. Blood glucose level of 175 mg/dL b. Serum potassium level of 3.0 mEq/L c. Orthostatic systolic BP decrease of 12 mm Hg d. Most recent blood pressure (BP) reading of 168/94 mm Hg

ANS: B Hypokalemia is a frequent adverse effect of the loop diuretics and can cause life-threatening dysrhythmias. The health care provider should be notified of the potassium level immediately and administration of potassium supplements initiated. The elevated blood glucose and BP also indicate a need for collaborative interventions but will not require action as urgently as the hypokalemia. An orthostatic drop of 12 mm Hg will require intervention only if the patient is symptomatic. DIF: Cognitive Level: Analyze (analysis)

An older patient has been diagnosed with possible white coat hypertension. Which planned action by the nurse best addresses the suspected cause of the hypertension? a. Instruct the patient about the need to decrease stress levels. b. Teach the patient how to self-monitor and record BPs at home. c. Schedule the patient for regular blood pressure (BP) checks in the clinic. d. Inform the patient and caregiver that major dietary changes will be needed.

ANS: B In the phenomenon of "white coat" hypertension, patients have elevated BP readings in a clinical setting and normal readings when BP is measured elsewhere. Having the patient self-monitor BPs at home will provide a reliable indication about whether the patient has hypertension. Regular BP checks in the clinic are likely to be high in a patient with white coat hypertension. There is no evidence that this patient has elevated stress levels or a poor diet, and those factors do not cause white coat hypertension. DIF: Cognitive Level: Apply (application)

Which blood pressure (BP) finding by the nurse indicates that no changes in therapy are needed for a 48-yr-old patient with newly diagnosed hypertension? a. 98/56 mm Hg b. 128/76 mm Hg c. 128/92 mm Hg d. 142/78 mm Hg

ANS: B The 8th Joint National Committee's recommended goal for antihypertensive therapy for a 30- to 59-yr-old patient with hypertension is a BP below 140/90 mm Hg. The BP of 98/56 mm Hg may indicate overtreatment of the hypertension and an increased risk for adverse drug effects. The other two blood pressures indicate a need for modifications in the patient's treatment. DIF: Cognitive Level: Apply (application)

The patient tells the nurse that he does not understand how there can be a blockage in the left anterior descending artery (LAD), but there is damage to the right ventricle. What is the best response by the nurse? a. "One coronary vessel curves around and supplies the entire heart muscle." b. "The LAD supplies blood to the left side of the heart and part of the right ventricle." c. "The right ventricle is supplied during systole primarily by the right coronary artery." d. "It is actually on the right side of the heart, but we call it the left anterior descending vessel."

ANS: B The best response is explaining that the lower portion of the right ventricle receives blood flow from the left anterior descending artery as well as the right coronary artery during diastole.

Which action will the nurse in the hypertension clinic take to obtain an accurate baseline blood pressure (BP) for a new patient? a. Deflate the BP cuff at a rate of 5 to 10 mm Hg per second. b. Have the patient sit in a chair with the feet flat on the floor. c. Assist the patient to the supine position for BP measurements. d. Obtain two BP readings in the dominant arm and average the results.

ANS: B The patient should be seated with the feet flat on the floor. The BP is obtained in both arms, and the results of the two arms are not averaged. The patient does not need to be in the supine position. The cuff should be deflated at 2 to 3 mm Hg per second. DIF: Cognitive Level: Understand (comprehension)

The nurse is admitting a patient who is scheduled to undergo a cardiac catheterization. What allergy information is most important for the nurse to assess and document before this procedure? a. Iron b. Iodine c. Aspirin d. Penicillin

ANS: B The provider will usually use an iodine-based contrast to perform this procedure. Therefore, it is imperative to know whether the patient is allergic to iodine or shellfish. Knowledge of allergies to iron, aspirin, or penicillin will be secondary.

The nurse obtains the following information from a patient newly diagnosed with prehypertension. Which finding is most important to address with the patient? a. Low dietary fiber intake b. No regular physical exercise c. Drinks a beer with dinner every night d. Weight is 5 pounds above ideal weight

ANS: B The recommendations for preventing hypertension include exercising aerobically for 30 minutes most days of the week. A weight that is 5 pounds over the ideal body weight is not a risk factor for hypertension. The Dietary Approaches to Stop Hypertension (DASH) diet is high in fiber, but increasing fiber alone will not prevent hypertension from developing. The patient's alcohol intake is within guidelines and will not increase the hypertension risk. DIF: Cognitive Level: Analyze (analysis)

While auscultating the patient's heart sounds with the bell of the stethoscope, the nurse hears a ventricular gallop. How should the nurse document what is heard? a. Diastolic murmur b. Third heart sound (S3) c. Fourth heart sound (S4) d. Normal heart sounds (S1, S2)

ANS: B The third heart sound is heard closely after the S2 and is known as a ventricular gallop because it is a vibration of the ventricular walls associated with decreased compliance of the ventricles during filling. It occurs with left ventricular failure. Murmurs sound like turbulence between normal heart sounds and are caused by abnormal blood flow through diseased valves. The S4 heart sound is a vibration caused by atrial contraction, precedes the S1, and is known as an atrial gallop. The normal S1 and S2 are heard when the valves close normally.

A patient is being admitted for valve replacement surgery. Which assessment finding is indicative of aortic valve stenosis? a. Pulse deficit b. Systolic murmur c. Distended neck veins d. Splinter hemorrhages

ANS: B The turbulent blood flow across a diseased valve results in a murmur. Aortic stenosis produces a systolic murmur. A pulse deficit indicates a cardiac dysrhythmia, most commonly atrial fibrillation. Right-sided heart failure may cause distended neck veins. Splinter hemorrhages occur in patients with infective endocarditis.

A patient presents to the emergency department reporting chest pain for 3 hours. What component of the blood work is most clearly indicative of a myocardial infarction (MI)? a. CK-MB b. Troponin c. Myoglobin d. C-reactive protein

ANS: B Troponin is the biomarker of choice in the diagnosis of MI, with sensitivity and specificity that exceed those of CK-MB and myoglobin. CRP levels are not used to diagnose acute MI.

The nurse is performing an assessment for a patient with fatigue and shortness of breath. Auscultation reveals a heart murmur. What does this assessment finding indicate? a. Increased viscosity of the patient's blood b. Turbulent blood flow across a heart valve c. Friction between the heart and the myocardium d. A deficit in conductivity impairs normal contractility

ANS: B Turbulent blood flow across the affected valve results in a murmur. A murmur is not a direct result of variances in blood viscosity, conductivity, or friction between the heart and myocardium.

A 72-yr-old man with a history of aortic stenosis is admitted to the emergency department. He reports severe left-sided chest pressure radiating to the jaw. Which medication, if ordered by the health care provider, should the nurse question? A. Aspirin B. Oxygen C. Nitroglycerin D. Morphine sulfate

ANS: C Aspirin, oxygen, nitroglycerin, and morphine sulfate are all used to treat acute chest pain suspected to be caused by myocardial ischemia. However, nitroglycerin should be used cautiously or avoided in patients with aortic stenosis as a significant reduction in blood pressure may occur. Chest pain can worsen because of a decrease in blood pressure.

A 25-yr-old patient with a group A streptococcal pharyngitis does not want to take the antibiotics prescribed. How should the nurse respond? A. "You will not feel well if you do not take the medicine and get over this infection." B. "Once you have been treated for a group A streptococcal infection, you will not get it again." C. "Without treatment, you could get rheumatic fever, which can lead to rheumatic heart disease." D. "You may not want to take the antibiotics for this infection, but you will be sorry if you do not."

ANS: C Rheumatic fever (RF) is not common because of effective use of antibiotics to treat streptococcal infections. Without treatment, RF can occur and lead to rheumatic heart disease, especially in young adults. Saying that the patient will not feel well or that the patient will be sorry if the antibiotics are not taken is threatening to the patient and inappropriate for the nurse to say. Patients may have reoccurring infection of group A streptococcus.

A 56-yr-old patient who has no previous history of hypertension or other health problems suddenly develops a blood pressure (BP) of 198/110 mm Hg. After reconfirming the BP, it is appropriate for the nurse to tell the patient that a. a BP recheck should be scheduled in a few weeks. b. dietary sodium and fat content should be decreased. c. diagnosis, treatment, and ongoing monitoring will be needed. d. there is an immediate danger of a stroke, requiring hospitalization.

ANS: C A sudden increase in BP in a patient older than age 50 years with no previous hypertension history or risk factors indicates that the hypertension may be secondary to some other problem. The BP will need treatment and ongoing monitoring. If the patient has no other risk factors, a stroke in the immediate future is unlikely. There is no indication that dietary salt or fat intake have contributed to this sudden increase in BP, and reducing intake of salt and fat alone will not be adequate to reduce this BP to an acceptable level. DIF: Cognitive Level: Apply (application)

After the nurse teaches the patient with stage 1 hypertension about diet modifications that should be implemented, which diet choice indicates that the teaching has been most effective? a. The patient avoids eating nuts or nut butters. b. The patient restricts intake of chicken and fish. c. The patient drinks low-fat milk with each meal. d. The patient has two cups of coffee in the morning.

ANS: C For the prevention of hypertension, the Dietary Approaches to Stop Hypertension (DASH) recommendations include increasing the intake of calcium-rich foods. Caffeine restriction and decreased protein intake are not included in the recommendations. Nuts are high in beneficial nutrients and 4 to 5 servings weekly are recommended in the DASH diet. DIF: Cognitive Level: Apply (application)

Which information is most important for the nurse to include when teaching a patient with newly diagnosed hypertension? a. Most people are able to control BP through dietary changes. b. Annual BP checks are needed to monitor treatment effectiveness. c. Hypertension is usually asymptomatic until target organ damage occurs. d. Increasing physical activity alone controls blood pressure (BP) for most people.

ANS: C Hypertension is usually asymptomatic until target organ damage has occurred. Lifestyle changes (e.g., physical activity, dietary changes) are used to help manage BP, but drugs are needed for most patients. Home BP monitoring should be taught to the patient and findings checked by the health care provider frequently when starting treatment for hypertension and then every 3 months when stable. DIF: Cognitive Level: Apply (application)

Propranolol (Inderal) is prescribed for a patient diagnosed with hypertension. The nurse should consult with the health care provider before giving this drug when the patient reveals a history of a. daily alcohol use. c. reactive airway disease. b. peptic ulcer disease. d. myocardial infarction (MI).

ANS: C Nonselective -blockers block 1- and 2-adrenergic receptors and can cause bronchospasm, especially in patients with a history of asthma. -Blockers will have no effect on the patient's peptic ulcer disease or alcohol use. -Blocker therapy is recommended after MI. DIF: Cognitive Level: Apply (application)

Which aspect of the heart's action does the QRS complex on the ECG represent? a. Depolarization of the atria b. Repolarization of the ventricles c. Depolarization from atrioventricular (AV) node throughout ventricles d. The length of time it takes for the impulse to travel from the atria to the ventricles

ANS: C The QRS recorded on the ECG represents depolarization from the AV node throughout the ventricles. The P wave represents depolarization of the atria. The T wave represents repolarization of the ventricles. The interval between the PR and QRS represents the length of time it takes for the impulse to travel from the atria to the ventricles.

A patient has just been diagnosed with hypertension and has been started on captopril . Which information is most important to include when teaching the patient about this drug? a. Include high-potassium foods such as bananas in the diet. b. Increase fluid intake if dryness of the mouth is a problem. c. Change position slowly to help prevent dizziness and falls. d. Check blood pressure (BP) in both arms before taking the drug.

ANS: C The angiotensin-converting enzyme (ACE) inhibitors frequently cause orthostatic hypotension, and patients should be taught to change position slowly to allow the vascular system time to compensate for the position change. Increasing fluid intake may counteract the effect of the drug, and the patient is taught to use gum or hard candy to relieve dry mouth. The BP should be taken in the nondominant arm by newly diagnosed patients in the morning, before taking the drug, and in the evening. Because ACE inhibitors cause potassium retention, increased intake of high-potassium foods is inappropriate. DIF: Cognitive Level: Apply (application)

Which action should the nurse implement with auscultation during a patient's cardiovascular assessment? a. Position the patient supine. b. Ask the patient to hold their breath. c. Palpate the radial pulse while auscultating the apical pulse. d. Use the bell of the stethoscope when auscultating S1 and S2.

ANS: C To detect a pulse deficit, simultaneously palpate the radial pulse when auscultating the apical area. The diaphragm is more appropriate than the bell when auscultating S1 and S2. A sitting or side-lying position is most appropriate for cardiac auscultation. It is not necessary to ask the patient to hold their breath during cardiac auscultation.

Which patients are most at risk for developing endocarditis? (Select all that apply.) A. Older woman with histoplasmosis B. Man with reports of chest pain and dyspnea C. Man who is homeless with history of IV drug use D. Patient with end-stage renal disease on peritoneal dialysis E. Adolescent with exertional palpitations and clubbing of fingers F. Female with peripheral intravenous site for medication administration

ANS: C, D Intravenous drug use, especially if reusing or sharing needles are at risk of developing sepsis. In addition, risk for infection is increased in the elderly, homeless, and those with chronic illness. Peritoneal dialysis requires strict sterile technique to prevent peritonitis. Chest pain, shortness of breath, and palpitations may be signs of endocarditis. Clubbing of the fingers indicates long-term hypoxia. Central venous catheters, not peripheral, increase risk to for infective endocarditis. Patients with fungal infections, such as histoplasmosis and candida, are at risk for pericarditis.

A 55-yr-old female patient develops acute pericarditis after a myocardial infarction. Which assessment finding indicates a possible complication? A. Presence of a pericardial friction rub B. Distant and muffled apical heart sounds C. Increased chest pain with deep breathing D. Decreased blood pressure with tachycardia

ANS: D Cardiac tamponade is a serious complication of acute pericarditis. Signs and symptoms include narrowed pulse pressure, tachypnea, tachycardia, a decreased cardiac output, and decreased blood pressure. The other symptoms are consistent with acute pericarditis.

On admission to the emergency department, a patient with cardiomyopathy has an ejection fraction of 10%. On assessment, the nurse notes bilateral crackles and shortness of breath. Which additional assessment finding would most indicate patient decline? A. Increased heart rate B. Increased blood pressure C. Decreased respiratory rate D. Decreased level of consciousness

ANS: D Decreased level of consciousness indicates a lack of perfusion, hypoxia, or both. A patient with an ejection fraction of 10% indicates very low cardiac output. Bilateral crackles and shortness of breath are consistent with decompensating heart failure. The nurse would expect an increase in heart rate, blood pressure, and respiratory rate in response to the low ejection fraction. When blood pressure drops, the nurse would be aware of potential shock.

Which laboratory result for a patient with multifocal premature ventricular contractions (PVCs) is most important for the nurse to communicate to the health care provider? a. Blood glucose 243 mg/dL b. Serum chloride 92 mEq/L c. Serum sodium 134 mEq/L d. Serum potassium 2.9 mEq/L

ANS: D Hypokalemia increases the risk for ventricular dysrhythmias such as PVCs, ventricular tachycardia, and ventricular fibrillation. The health care provider will need to prescribe a potassium infusion to correct this abnormality. Although the other laboratory values also are abnormal, they are not likely to be the etiology of the patients PVCs and do not require immediate correction.

The patient with pericarditis is reporting chest pain. After assessment, which intervention should the nurse expect to implement to provide pain relief? A. Corticosteroids B. Morphine sulfate C. Proton pump inhibitor D. Nonsteroidal antiinflammatory drugs

ANS: D Nonsteroidal antiinflammatory drugs (NSAIDs) control pain and inflammation. Corticosteroids are reserved for patients already taking corticosteroids for autoimmune conditions and those who do not respond to NSAIDs. Morphine is not necessary. Proton pump inhibitors are used to decrease stomach acid to avoid the risk of gastrointestinal bleeding from the NSAIDs.

The nurse is caring for a patient who received a mechanical aortic valve replacement 2 years ago. Current lab values include an international normalized ratio (INR) of 1.5, platelet count of 150,000/µL, and hemoglobin of 8.6g/dL. Which nursing action is most appropriate? A. Assess the vital signs. B. Start intravenous fluids. C. Monitor for signs of bleeding. D. Contact the health care provider.

ANS: D Patients with mechanical valve replacement are placed on anticoagulants and should be in a therapeutic INR range of 2.5 to 3.5. Administration of Coumadin (Warfarin) prolongs clotting time and prevents clot formation on the valve. The low INR would require a call to the health care provider for an order increase the medication dose. Vital signs would be unchanged related to the low INR. Intravenous fluids are not indicated. The patient is at risk of forming clots, not bleeding.

An 80-yr-old patient with uncontrolled type 1 diabetes is diagnosed with aortic stenosis. When conservative therapy is no longer effective, the nurse knows that the patient will need to do or have what done? A. Aortic valve replacement B. Have a pacemaker inserted C. Open commissurotomy (valvulotomy) procedure D. Percutaneous transluminal balloon valvuloplasty (PTBV) procedure

ANS: D The PTBV procedure is best for this older adult patient who is a poor surgery candidate related to the uncontrolled type 1 diabetes. Aortic valve replacement would probably not be tolerated well by this patient, although it may be done if the PTBV fails and the diabetes is controlled in the future. Open commissurotomy procedure is used for mitral stenosis. The patient is not a candidate for a pacemaker.

Which instruction by the nurse to a patient who is about to undergo Holter monitoring is accurate? a. "You may remove the monitor only to shower or bathe." b. "You should connect the monitor whenever you feel symptoms." c. "You should refrain from exercising while wearing this monitor." d. "You will need to keep a diary of your activities and symptoms."

ANS: D A Holter monitor is worn continuously for at least 24 hours while a patient continues with usual activity and keeps a diary of activities and symptoms. The patient should not take a bath or shower while wearing this monitor.

The blood pressure of an older adult patient admitted with pneumonia is 160/70 mm Hg. What is an age-related change that contributes to this finding? a. Stenosis of the heart valves b. Decreased adrenergic sensitivity c. Increased parasympathetic activity d. Loss of elasticity in arterial vessels

ANS: D An age-related change that increases the risk of systolic hypertension is a loss of elasticity in the arterial walls. Because of the increasing resistance to flow, pressure is increased within the blood vessel, and hypertension results. Valvular rigidity of aging causes murmurs, and decreased adrenergic sensitivity slows the heart rate. Blood pressure is not raised. Increased parasympathetic activity would slow the heart rate.

Which action will be included in the plan of care when the nurse is caring for a patient who is receiving nicardipine (Cardene) to treat a hypertensive emergency? a. Organize nursing activities so that the patient has undisturbed sleep for 8 hours at night. b. Keep the patient NPO to prevent aspiration caused by nausea and possible vomiting. c. Assist the patient up in the chair for meals to avoid complications associated with immobility. d. Use an automated noninvasive blood pressure machine to obtain frequent measurements.

ANS: D Frequent monitoring of BP is needed when the patient is receiving rapid-acting IV antihypertensive medications. This can be most easily accomplished with an automated BP machine or arterial line. The patient will require frequent assessments, so allowing 8 hours of undisturbed sleep is not reasonable. When patients are receiving IV vasodilators, bed rest is maintained to prevent decreased cerebral perfusion and fainting. There is no indication that this patient is nauseated or at risk for aspiration, so an NPO status is unnecessary. DIF: Cognitive Level: Apply (application)

What is an appropriate explanation for the nurse to give to a patient about the purpose of intermittent pneumatic compression devices after a surgical procedure? a. The devices keep the legs warm while the patient is not moving much. b. The devices maintain the blood flow to the legs while the patient is on bed rest. c. The devices keep the blood pressure down while the patient is stressed after surgery. d. The devices provide compression of the veins to keep the blood moving back to the heart.

ANS: D Intermittent pneumatic compression devices provide compression of the veins while the patient is not using skeletal muscles to compress the veins, which keeps the blood moving back to the heart and prevents blood pooling in the legs that could cause deep vein thrombosis. The warmth is not important. Blood flow to the legs is not maintained. Blood pressure is not decreased with the use of intermittent sequential compression stockings.

The registered nurse (RN) is caring for a patient with a hypertensive crisis who is receiving sodium nitroprusside . Which nursing action can the nurse delegate to an experienced licensed practical/vocational nurse (LPN/LVN)? a. Evaluate effectiveness of nitroprusside therapy on blood pressure (BP). b. Assess the patient's environment for adverse stimuli that might increase BP. c. Titrate nitroprusside to decrease mean arterial pressure (MAP) to 115 mm Hg. d. Set up the automatic noninvasive BP machine to take readings every 15 minutes.

ANS: D LPN/LVN education and scope of practice include the correct use of common equipment such as automatic noninvasive blood pressure machines. The other actions require advanced nursing judgment and education, and should be done by RNs. DIF: Cognitive Level: Apply (application)

Which action should the nurse take when giving the initial dose of oral labetalol to a patient with hypertension? a. Encourage the use of hard candy to prevent dry mouth. b. Teach the patient that headaches often occur with this drug. c. Instruct the patient to call for help if heart palpitations occur. d. Ask the patient to request assistance before getting out of bed.

ANS: D Labetalol decreases sympathetic nervous system activity by blocking both - and -adrenergic receptors, leading to vasodilation and a decrease in heart rate, which can cause severe orthostatic hypotension. Heart palpitations, dry mouth, dehydration, and headaches are possible side effects of other antihypertensives. DIF: Cognitive Level: Apply (application)

Which anatomic feature of the heart directly stimulates ventricular contractions? a. SA node b. AV node c. Bundle of His d. Purkinje fibers

ANS: D The Purkinje fibers move the electrical impulse or action potential through the walls of both ventricles triggering synchronized right and left ventricular contraction. The sinoatrial (SA) node initiates the electrical impulse that results in atrial contraction. The atrioventricular (AV) node receives the electrical impulse through internodal pathways. The bundle of His receives the impulse from the AV node.

For which problem is percutaneous coronary intervention (PCI) most clearly indicated? Chronic stable angina Left-sided heart failure Coronary artery disease Acute myocardial infarction

Acute myocardial infarction PCI is indicated to restore coronary perfusion in cases of myocardial infarction. Chronic stable angina and coronary artery disease are normally treated with more conservative measures initially. PCI is not relevant to the pathophysiology of heart failure.

Which statement made by a patient with hemophilia indicates a need for further teaching? A. "I will take aspirin to control pain." B. "I will avoid weight bearing until swelling has subsided." C. "I will increase wear a medic alert bracelet at all times." D. "I will stop any bleeding by applying direct pressure and ice to the area."

Answer: A Rationale: Aspirin should never be administered because it can prolong bleeding. All of the other interventions are appropriate and demonstrate understanding regarding self-care.

You are evaluating the laboratory data of the patient with suspected aplastic anemia. Which findings support this diagnosis? A. Reduced RBCs, reduced white blood cells (WBCs), and reduced platelets B. Reduced RBCs, normal WBCs, and normal platelets C. Normal RBCs, reduced WBCs, and reduced platelets D. Elevated RBCs, increased WBCs, and increased platelets

Answer: A Rationale: Because all marrow elements are affected, hemoglobin, WBC, and platelet values are decreased in aplastic anemia. Other RBC indices usually are normal.

When providing care for a patient with thrombocytopenia, you instruct the patient to A. dab his or her nose instead of blowing. B. be careful when shaving with a safety razor. C. continue with physical activities to stimulate thrombopoiesis. D. avoid aspirin because it may mask the fever that occurs with thrombocytopenia.

Answer: A Rationale: Blowing the nose forcefully should be avoided; the patient should gently pat it with a tissue if needed. Patients with thrombocytopenia should avoid aspirin; aspirin reduces platelet adhesiveness, contributing to bleeding. Patients should not perform vigorous exercise or lift weights. If the patient is weak and at risk for falling, supervise the patient when out of bed. Patients should be instructed not to shave with a blade; an electric razor should be used.

In non-Hodgkin's lymphoma, the involved cell in 90% of cases is the A. B lymphocyte. B. T lymphocyte. C. Reed-Sternberg cell. D. neutrophil.

Answer: A Rationale: In 90% of cases of non-Hodgkin's lymphoma, the B lymphocyte is the cell of origin.

Which assessment findings are consistent with thrombocytopenia? A. Petechiae, ecchymoses B. Pallor, spider angiomas C. Cyanosis, dullness D. Jaundice, purpura

Answer: A Rationale: Many patients with thrombocytopenia are asymptomatic. The most common symptom is bleeding, usually mucosal or cutaneous. Mucosal bleeding may manifest as epistaxis and gingival bleeding, and large bullous hemorrhages may appear on the buccal mucosa because of the lack of vessel protection by the submucosal tissue. Bleeding into the skin manifests as petechiae, purpura, or superficial ecchymoses

What would you expect to find in a bone marrow biopsy in the patient with acute myelogenous leukemia? A. Multiple myeloblasts B. Abundant lymphocytes C. Immature lymphoblasts D. Insufficient numbers of erythrocytes

Answer: A Rationale: Myeloblasts will be present at greater than 50%

The Schilling test for pernicious anemia involves A. administration of radioactive cobalamin and measuring its excretion in the urine. B. blood cultures for organism identification. C. the measurement of serum iron. D. the administration of iron and blood assessment of total iron binding in 24 hours.

Answer: A Rationale: Parietal cell function can be assssed with a Schilling test. After radioactive cobalamin is administered to the patient, the amount of cobalamin excreted in the urine is measured. An individual who cannot absorb cobalamin excretes only a small amount of this radioactive form.

You encourage the patient with cobalamin deficiency to seek treatment because untreated pernicious anemia may result in A. death. B. liver failure. C. heart failure. D. gastrectomy.

Answer: A Rationale: Regardless of how much cobalamin is ingested, the patient is not able to absorb it if intrinsic factor is lacking or if there is impaired absorption in the ileum. For this reason, increasing dietary cobalamin does not correct the anemia. However, the patient should be instructed about adequate dietary intake to maintain good nutrition (see Table 31-5). Parenteral (cyanocobalamin or hydroxocobalamin) or intranasal (Nascobal, CaloMist) administration of cobalamin is the treatment of choice. Without cobalamin administration, these individuals will die in 1 to 3 years.

Which patient is most likely to experience anemia caused by increased destruction of RBCs? A. An African American man who has a diagnosis of sickle cell disease B. A 59-year-old man whose alcoholism has precipitated folic acid deficiency C. A 30-year-old woman with a history of "heavy periods" accompanied by anemia D. A 3-year-old child whose impaired growth and development is attributable to thalassemia

Answer: A Rationale: The cause of sickle cell anemia involves increased hemolysis. Thalassemias and folic acid deficiencies decrease erythropoiesis, whereas the anemia related to menstruation is a direct result of blood loss.

You receive a physician's order to transfuse fresh frozen plasma to a patient suffering from an acute blood loss. Which procedure is most appropriate for infusing this blood product? A. Infuse the fresh frozen plasma as rapidly as the patient will tolerate. B. Hang the fresh frozen plasma as a piggyback to the primary IV solution. C. Infuse the fresh frozen plasma as a piggyback to a primary solution of normal saline. D. Hand the fresh frozen plasma as a piggyback to a new bag of primary IV solution without KCl.

Answer: A Rationale: The fresh frozen plasma should be administered as rapidly as possible and should be used within 2 hours of thawing. Fresh frozen plasma is infused using any straight-line infusion set. Any existing IV should be interrupted while the fresh frozen plasma is infused, unless a second IV line has been started for the transfusion.

Before beginning a transfusion of RBCs, which action would be of highest priority to you to avoid an error during this procedure? A. Check the identifying information on the unit of blood against the patient's ID bracelet. B. Select new primary IV tubing primed with lactated Ringer's solution to use for the transfusion. C. Remain with the patient for 60 minutes after beginning the transfusion to watch for signs of a transfusion reaction. D. Add the blood transfusion as a secondary line to the existing IV and use the IV controller to maintain correct flow.

Answer: A Rationale: The patient's identifying information (name, date of birth, medical record number) on the ID bracelet should exactly match the information on the blood bank tag that has been placed on the unit of blood. If any information does not match, the transfusions should not be hung because of possible error and risk to the patient.

Which individual is at high risk for a cobalamin (vitamin B12) deficiency anemia? A. A 47-year-old man who had a gastrectomy (removal of the stomach) B. A 54-year-old man with a history of irritable bowel disease and ulcerative colitis C. A 26-year-old woman who complains of heavy menstrual periods D. A 15-year-old girl who is a vegetarian

Answer: A Rationale: There are many causes of cobalamin deficiency. The most common cause is pernicious anemia, a disease in which the gastric mucosa is not secreting intrinsic factor (IF) because of antibodies being directed against the gastric parietal cells or IF itself. Other causes of cobalamin deficiency include gastrectomy, gastritis, nutritional deficiency, chronic alcoholism, and hereditary enzymatic defects of cobalamin use.

A complication of the hyperviscosity of polycythemia is A. thrombosis. B. cardiomyopathy. C. pulmonary edema. D. disseminated intravascular coagulation (DIC).

Answer: A Rationale: Thrombosis is the most likely complication. The patient with polycythemia may experience angina, heart failure, intermittent claudication, and thrombophlebitis, which may be complicated by embolization. The most common and serious acute complication is stroke due to thrombosis.

You correctly identify which descriptions as characteristic of anemia of chronic disease (select all that apply) A. normocytic. B. normochromic. C. microcytic. D. hypochromic. E. proliferative.

Answer: A & B Rationale: Anemia of chronic disease, also called anemia of inflammation, is associated with an underproduction of RBCs and mild shortening of RBC survival. The RBCs are usually normocytic, normochromic, and hypoproliferative. The anemia is usually mild, but it can be more severe.

Which points should be included in teaching the patient with sickle cell disease (select all that apply)? A. Avoid dehydration. B. Avoid high altitudes. C. Take cobalamin (vitamin B12) regularly. D. Consume dairy products frequently. E. Increase consumption of grapefruit juice.

Answer: A & B Rationale: Avoiding dehydration and high altitudes helps to prevent crises. Vitamins, dairy products, and grapefruit juice cannot help the patient to prevent attacks of sickle cell disease.

Nursing interventions for a patient with severe anemia related to peptic ulcer disease include (select all that apply) A. monitoring stools for guaiac. B. instructions about a high-iron diet. C. taking vital signs every 8 hours. D. teaching self-injection of erythropoietin.

Answer: A & B Rationale: The stool guaiac test is done to determine whetherthe cause of the iron-deficiency anemia is related to gastrointestinal bleeding. Iron should be increased in the diet. Teach the patient which foods are good sources of iron. If nutrition is already adequate, increasing iron intake by dietary means may not be practical. The patient with iron deficiency related to acute blood loss may require a transfusion of packed red blood cells (RBCs).

Which foods should you encourage patients with folic acid deficiency to include in their daily food intake (select all that apply)? A. Ready-to-eat cereal B. Wheat tortillas C. Lentils D. Strawberries E. Potatoes

Answer: A, B, & C Rationale: Whole-grain foods and beans are high in folic acid.

Priority nursing actions when caring for a hospitalized patient with new-onset temperature of 102.2° F and severe neutropenia include (select all that apply) A. administering the prescribed antibiotic STAT. B. drawing peripheral and central line blood cultures. C. ongoing monitoring of the patient's vital signs for septic shock. D. taking a full set of vital signs and notifying the physician immediately.

Answer: A, B, C, & D Rationale: Early identification of an infective organism is a priority, and samples for cultures should be obtained from various sites. Serial blood cultures (at least two) or one from a peripheral site and one from a venous access device should be done promptly. In the febrile, neutropenic patient, antibiotics should be started immediately (within 1 hour). Cultures of the nose, throat, sputum, urine, stool, obvious lesions, and the blood may be indicated. Ongoing febrile episodes or a change in the patient's assessment (or vital signs) requires a call to the physician to order additional cultures, diagnostic tests, and antimicrobial therapies.

The nursing management of a patient in sickle cell crisis includes (select all that apply) A. monitoring of the complete blood cell (CBC) count. B. blood transfusions if required and iron chelation. C. optimal pain management and oxygen therapy. D. rest as needed and deep vein thrombosis prophylaxis.

Answer: A, B, C, & D Rationale: The CBC count is monitored. Infections are common with an elevated white blood cell (WBC) count, and anemia may occur with low hemoglobin and red blood cell (RBC) levels. Oxygen may be administered to treat hypoxia and control sickling. Rest may be instituted to reduce metabolic requirements and deep vein thrombosis prophylaxis (using anticoagulants) prescribed. Transfusion therapy is indicated when an aplastic crisis occurs. Patients may require iron chelation therapy to reduce transfusion-produced iron overload. Pain occurring during an acute crisis usually is undertreated. Patients should have optimal pain control with opioid analgesics, nonsteroidal antiinflammatory agents, antineuropathic pain medications, local anesthetics, or nerve blocks.

Post-splenectomy patients have a lifelong risk of A. hemorrhage. B. infection. C. delayed healing. D. cancer.

Answer: B Rationale: After splenectomy, immunologic deficiencies may develop. IgM levels are reduced, and IgG and IgA values remain within normal limits. Postsplenectomy patients have a lifelong risk for infection. The risk for hemorrhage was removed with spleen removal. Healing should not be affected. The risk of cancer for the splenectomy patient is not greater than the regular population.

Before starting a transfusion of packed red blood cells for an anemic patient, you would arrange for a peer to monitor his or her other assigned patients for how many minutes when you begin the transfusion? A. 5 minutes B. 15 minutes C. 30 minutes D. 60 minutes

Answer: B Rationale: As part of standard procedure, you remain with the patient for the first 15 minutes after starting a blood transfusion. Patients who are likely to have a transfusion reaction will more often exhibit signs within the first 15 minutes that the blood is infusing.

The blood bank notifies you that the two units of blood ordered for an anemic patient are ready for pick up. You should take which action to prevent an adverse effect during this procedure? A. Immediately pick up both units of blood from the blood bank. B. Infuse the blood slowly for the first 15 minutes of the transfusion. C. Regulate the flow rate so that each unit takes at least 4 hours to transfuse. D. Set up the Y-tubing of the blood set with dextrose in water as the flush solution.

Answer: B Rationale: Because a transfusion reaction is more likely to occur at the beginning of a transfusion, you should initially infuse the blood at a rate no faster than 2 mL/min and remain with the patient for the first 15 minutes after hanging a unit of blood.

The patient will receive desmopressin acetate (DDAVP) as a part of the treatment plan for mild hemophilia A. The nurse knows the drug is used to stimulate blood clotting factors and expects which outcome? A. Increased red blood cell count B. Decreased bleeding time C. Increased reticulocytes D. Increased platelets

Answer: B Rationale: Beneficial effects of DDAVP include decreased bleeding time. DDAVP does not increase levels of platelets, reticulocytes, or red blood cells.

Your primary goal in the care of the patient with DIC is to A. provide emotional support. B. recognize early signs of occult or overt bleeding. C. monitor nutritional intake. D. report abnormal laboratory results.

Answer: B Rationale: Early detection of occult and overt bleeding must be your primary goal. You should assess for signs of external bleeding such as petechiae, oozing at intravenous or injection sites, and signs of internal bleeding, such as increased heart rate, changes in mental status, or pain. Emotional support is important but is a secondary goal, as is monitoring nutritional intake. It is important to report abnormal results, but this is not a priority over assessment.

A patient with a diagnosis of hemophilia fell down an escalator earlier in the day and is now experiencing bleeding in her left knee joint. Your immediate response should include A. immediate transfusion of platelets. B. resting the patient's knee to prevent hemarthroses. C. assistance with intracapsular injection of corticosteroids. D. range-of-motion exercises to prevent thrombus formation.

Answer: B Rationale: In patients with hemophilia, joint bleeding requires resting of the joint to prevent deformities from hemarthrosis. Clotting factors, not platelets or corticosteroids, are administered. Thrombus formation is not a central concern in a patient with hemophilia.

The nursing student asks the clinical instructor to explain why clinical symptoms are more important than laboratory values when the patient has experienced blood loss. The instructor correctly recognizes that A. nurses should focus on clinical symptoms because those are the parameters of nursing practice. B. blood values are often normal or even high because fluid shifts have not occurred and laboratory values are falsely high. C. laboratory values are used to supplement nursing assessments. D. laboratory findings are often falsely low in the early period of blood loss.

Answer: B Rationale: It is essential to understand that the clinical signs and symptoms the patient is experiencing are more important than the laboratory values. For example, an adult with a bleeding peptic ulcer who had a 750-mL hematemesis (15% of a normal total blood volume) within the past 30 minutes may have postural hypotension but have normal hemoglobin and hematocrit values. Over the ensuing 36 to 48 hours, most of the blood volume deficit will be replaced by the movement of fluid from the extravascular into the intravascular space. Only at these later times will the hemoglobin and hematocrit values reflect the blood loss.

When stroke volume decreases what physiological mechanism helps to maintain cardiac output? a. Heart rate of 54 b. Heart rate of 100 c. SpO2 94% d. Changing position from sitting to standing up

a. Heart rate of 54

Caring for a patient with a diagnosis of polycythemia vera will likely require you to A. encourage deep breathing and coughing. B. assist with or perform phlebotomy at the bedside. C. teach the patient how to maintain a low-activity lifestyle. D. perform thorough and regularly scheduled neurologic assessments.

Answer: B Rationale: Primary polycythemia often requires phlebotomy to reduce blood volume. The increased risk of thrombus formation that accompanies the disease requires regular exercises and ambulation. Deep-breathing and coughing exercises do not directly address the cause or common sequelae of polycythemia, and neurologic manifestations are not typical.

Which finding allows you to identify the patient's anemia as folic acid deficiency rather than cobalamin deficiency? A. Loss of appetite B. Lack of neuromuscular symptoms C. Red tongue D. Change in nail shape

Answer: B Rationale: The absence of neurologic problems is an important diagnostic finding and differentiates folic acid deficiency from cobalamin deficiency.

When caring for a patient with metastatic cancer, you note a hemoglobin level of 8.7 g/dL and hematocrit of 26%. You place highest priority on initiating interventions that can reduce A. thirst. B. fatigue. C. headache. D. abdominal pain.

Answer: B Rationale: The patient with a low hemoglobin level and hematocrit is anemic and is most likely to experience fatigue. Fatigue develops because of the lowered oxygen-carrying capacity that leads to reduced tissue oxygenation with which to carry out cellular functions.

You note a physician's order written at 10:00 AM for 2 units of packed red blood cells to be administered to a patient who is anemic secondary to chronic blood loss. If the transfusion is picked up at 11:30, you should plan to hang the unit no later than what time? A. 11:45 AM B. 12:00 noon C. 12:30 PM D. 3:30 PM

Answer: B Rationale: You must hang the unit of packed red blood cells within 30 minutes of signing them out from the blood bank.

Which nursing intervention should you prioritize in the care of a 30-year-old woman who has a diagnosis of immune thrombocytopenic purpura (ITP)? A. Administration of packed red blood cells B. Administration of clotting factors VIII and IX C. Administration of oral or intravenous corticosteroids D. Maintenance of reverse isolation and application of standard precautions

Answer: C Rationale: Common treatment modalities for ITP include corticosteroid therapy to suppress the phagocytic response of splenic macrophages. Blood transfusions, administration of clotting factors, and reverse isolation are not interventions that are indicated in the care of patients with ITP.

Disseminated intravascular coagulation (DIC) is a disorder in which A. the coagulation pathway is genetically altered, leading to thrombus formation in all major blood vessels. B. an underlying disease depletes hemolytic factors in the blood, leading to diffuse thrombotic episodes and infarcts. C. a disease process stimulates coagulation processes with resultant thrombosis and depletion of clotting factors, leading to diffuse clotting and hemorrhage. D. an inherited predisposition causes a deficiency of clotting factors that leads to overstimulation of coagulation processes in the vasculature.

Answer: C Rationale: DIC is a disease process in which there is stimulation of the coagulation process with resultant thrombosis and depletion of clotting factors, leading to diffuse clotting and hemorrhage. The paradox of this condition is characterized by the profuse bleeding that results from the depletion of platelets and clotting factors.

A 25-year-old man who was recently diagnosed with Hodgkin's disease in the pelvic area is about to begin radiation therapy and he expresses concern about becoming infertile. What should the patient be told about sexual function? A. Impotence may occur, but it will only be temporary. B. Sperm cells will mature, resulting in deformed offspring. C. Permanent sterility can occur; thus sperm banking should be considered. D. Changes in secondary sex characteristics, including breast enlargement, may occur with chemotherapy.

Answer: C Rationale: Fertility issues may be of particular concern with the disease and the related treatment.

Which organ is at greatest risk due to the effects of hemolytic anemia? A. Heart B. Spleen C. Kidney D. Liver

Answer: C Rationale: For all causes of hemolysis, a major focus of treatment is to maintain renal function. When RBCs are hemolyzed, the hemoglobin molecule is released and filtered by the kidneys. The accumulation of hemoglobin molecules can obstruct the renal tubules and lead to acute tubular necrosis

Disseminated intravascular coagulation (DIC) is initiated by intravascular release of which substance? A. Platelets B. Fibrin C. Thrombin D. Histamine

Answer: C Rationale: In DIC, abundant intravascular thrombin, the most powerful coagulant, is produced. This converts fibrinogen to fibrin and promotes platelet aggregation. Platelets do not initiate the response. Histamine does not play a role in DIC.

A factor unique to non-Hodgkin's lymphoma is that A. relapses are unlikely. B. treatment is limited to chemotherapy. C. more aggressive tumors respond effectively to treatment. D. tumors are present only in the lymph glands.

Answer: C Rationale: More aggressive non-Hodgkin lymphomas respond to treatment and are more likely to be cured. Treatment includes irradiation and chemotherapy. Relapses are common. Unlike Hodgkin's disease, non-Hodgkin's lymphomas occur in non-nodal sites.

You are aware that a major difference between Hodgkin's lymphoma and non-Hodgkin's lymphoma is that A. Hodgkin's lymphoma occurs only in young adults. B. Hodgkin's lymphoma is considered potentially curable. C. non-Hodgkin's lymphoma can present in multiple organs. D. non-Hodgkin's lymphoma is treated only with radiation therapy.

Answer: C Rationale: Non-Hodgkin's lymphoma can originate outside the lymph nodes, the method of spread can be unpredictable, and the majority of patients have widely disseminated disease.

If the patient with DIC is actively bleeding, platelets are given to correct thrombocytopenia if the count is less than A. 150,000/μL. B. 100,000/μL. C. 50,000/μL. D. 30,000/μL.

Answer: C Rationale: Platelets usually are given to correct thrombocytopenia if the platelet count is less than 20,000/μL or is less than 50,000/μL if the patient is actively bleeding.

Which findings do you expect to find for a patient with acute loss of blood? A. Weakness, lethargy, and warm, dry skin B. Restlessness, hyperthermia, and bradycardia C. Tachycardia, hypotension, and cool, clammy skin D. Widened pulse pressure, anxiety, and hypoventilation

Answer: C Rationale: Tachycardia, hypotension, and cool, clammy skin can be found in a person who has had an acute loss of blood. These are manifestations of hypovolemic shock. A person with a bleeding peptic ulcer who had a 750-mL hematemesis (15% of a normal total blood volume) within the past 30 minutes may have postural hypotension. Because blood is shunted to major organs, the skin in the periphery is cool to the touch. Tachycardia is the body's attempt to maintain adequate cardiac output.

When preparing to administer an ordered blood transfusion, you select which intravenous solution to use when priming the blood tubing? A. Lactated Ringer's B. 5% dextrose in water C. 0.9% sodium chloride D. 0.45% sodium chloride

Answer: C Rationale: The blood set should be primed before the transfusion with 0.9% sodium chloride, also known as normal saline. It is also used to flush the blood tubing after the infusion is complete to ensure the patient receives blood that is left in the tubing when the bag is empty.

A patient with acute myelogenous leukemia will soon start chemotherapy. When you are teaching the patient about the induction stage of chemotherapy, what is the best explanation? A. The drugs are started slowly to minimize side effects. B. You will develop even greater bone marrow depression with risk for bleeding and infection. C. It will be necessary to have high-dose treatment every day for several months. D. During this time you will regain energy and become more resistant to infection.

Answer: C Rationale: The chemotherapeutic treatment of acute leukemia is often divided into stages. The first stage, induction therapy, is the attempt to induce or bring about a remission. Induction is aggressive treatment that seeks to destroy leukemic cells in the tissues, peripheral blood, and bone marrow in order to eventually restore normal hematopoiesis on bone marrow recovery. During induction therapy a patient may become critically ill because the bone marrow is severely depressed by the chemotherapeutic agents.

You are caring for a patient who is to receive a transfusion of two units of packed red blood cells. After obtaining the first unit from the blood bank, you would ask which health team member in the nurses' station to assist in checking the unit before administration? A. Unit secretary B. A phlebotomist C. A physician's assistant D. Another registered nurse

Answer: D Rationale: Before hanging a transfusion, the registered nurse must check the unit with another RN or with a licensed practical (vocational) nurse, depending on agency policy.

Which finding would you recognize as an indicator of chronic myelogenous leukemia (CML)? A. Presence of an abnormal LE cell B. Numerous immature lymphoblasts C. An elevated white blood cell count D. Presence of the Philadelphia chromosome

Answer: D Rationale: CML is caused by excessive development of mature neoplastic granulocytes in the bone marrow. The excess neoplastic granulocytes move into the peripheral blood in massive numbers and ultimately infiltrate the liver and spleen. These cells contain a distinctive cytogenetic abnormality, the Philadelphia chromosome, which serves as a disease marker and results from translocation of genetic material between chromosomes 9 and 22.

The most common type of leukemia in older adults is A. acute myelocytic leukemia. B. acute lymphocytic leukemia. C. chronic myelocytic leukemia. D. chronic lymphocytic leukemia.

Answer: D Rationale: Chronic lymphocytic leukemia is a disease primarily found in older adults.

A patient with non-Hodgkin's lymphoma asks why staging is so important. The accurate response to his question is that staging is done to A. determine if you have favorable histology. B. establish your short- and long-term prognoses. C. properly code your disease for statistical purposes. D. help identify the most effective chemotherapy protocols.

Answer: D Rationale: Clinical staging is used to help guide therapy.

Multiple drugs are often used in combinations to treat leukemia and lymphoma because A. there are fewer toxic side effects. B. the chance that one drug will be effective is increased. C. the drugs are more effective without causing side effects. D. the drugs work by different mechanisms to maximize killing of malignant cells.

Answer: D Rationale: Combination therapy is the mainstay of treatment for leukemia. Multiple drugs are used to decrease drug resistance, minimize overall toxicity by using drugs with different toxic effects, and interrupt cell growth at multiple points in the cell cycle.

Complications of transfusions that can be decreased by the use of leukocyte depletion or reduction for red blood cells are A. chills and hemolysis. B. leukostasis and neutrophilia. C. fluid overload and pulmonary edema. D. transmission of cytomegalovirus and fever.

Answer: D Rationale: Infectious viruses, HIV, human herpesvirus agents transmitted by blood transfusion include hepatitis B and C type 6 (HSV-6), Epstein-Barr virus (EBV), human T-cell leukemia virus, type 1 (HTLV-1), cytomegalovirus (CMV), and malaria. Leukocyte-reduced blood products drastically reduce the risk of blood transfusion-associated viral infections, including CMV.

Which sign or symptom would you recognize as a unique characteristic specific to hemolytic anemia? A. Tachycardia B. Weakness C. Decreased RBCs D. Jaundice

Answer: D Rationale: Jaundice is likely because the increased destruction of RBCs causes an elevation in bilirubin levels. The spleen and liver may enlarge because of their hyperactivity, which is related to macrophage phagocytosis of the defective erythrocytes. The other symptoms are common to all types of anemia.

Because myelodysplastic syndrome (MDS) arises from the pluripotent hematopoietic stem cells in the bone marrow, expected laboratory results include A. an excess of T cells. B. an excess of platelets. C. an increase in lymphocytes. D. a deficiency of all cellular blood components.

Answer: D Rationale: MDS commonly manifests as infection and bleeding caused by inadequate numbers of effectively functioning circulating granulocytes or platelets.

Which instruction will you include in the discharge teaching plan of a patient with leukemia regarding health maintenance? A. "Avoid drinking large amounts of fluids." B. "Eat six small meals daily." C. "Engage in aerobic exercise three days a week." D. "Receive a yearly influenza vaccination."

Answer: D Rationale: Most patients should receive the pneumococcal vaccine (Pneumovax) at diagnosis and every 5 years, and an annual influenza vaccine.

When reviewing the patient's hematologic laboratory values after a splenectomy, you would expect to find A. leukopenia. B. RBC abnormalities. C. decreased hemoglobin. D. increased platelet count.

Answer: D Rationale: Splenectomy can have a dramatic effect in increasing peripheral RBC, WBC, and platelet counts.

You anticipate that a patient with von Willebrand's disease undergoing surgery will be treated with administration of von Willebrand factor (vWF) and A. thrombin. B. factor VI. C. factor VII. D. factor VIII.

Answer: D Rationale: Von Willebrand's disease involves a deficiency of the von Willebrand coagulation protein, variable factor VIII deficiencies, and platelet dysfunction. Treatment includes administration of vWF and factor VIII.

The care plan for a patient with aplastic anemia should include activities to minimize the risk for which complications? A. Dyspnea and pain B. Diarrhea and fatigue C. Nausea and malnutrition D. Infection and hemorrhage

Answer: D Rationale: You must assist the patient in reducing infection risk. The patient is susceptible to infection and is at risk for septic shock and death. Even a low-grade temperature (>100.4° F) should be considered a medical emergency. Thrombocytopenia manifests as a predisposition to bleeding evidenced by petechiae, ecchymosis, and epistaxis. Pain is not experienced nor is diarrhea. Nausea and malnutrition are not related to this disease except as a by-product of infection.

A male patient with COPD becomes dyspneic at rest. His baseline blood gas results are PaO2 70 mm Hg, PaCO2 52 mm Hg, and pH 7.34. What updated patient assessment requires the nurse's priority intervention? Arterial pH 7.26 PaCO2 50 mm Hg Patient in tripod position Increased sputum expectoration

Arterial pH 7.26 The patient's pH shows acidosis that supports an exacerbation of COPD along with the worsening dyspnea. The PaCO2 has improved from baseline, the tripod position helps the patient's breathing, and the increase in sputum expectoration will improve the patient's ventilation.

The nurse would assess a patient with complaints of chest pain for which clinical manifestations associated with a myocardial infarction (MI) (select all that apply.)? Flushing Ashen skin Diaphoresis Nausea and vomiting S3 or S4 heart sounds

Ashen skin -During the initial phase of an MI, catecholamines are released from the ischemic myocardial cells, causing increased sympathetic nervous system stimulation. This results in the release of glycogen, diaphoresis, and vasoconstriction of peripheral blood vessels. The patient's skin may be ashen, cool, and clammy (not flushed) as a result of this response. Nausea and vomiting may result from reflex stimulation of the vomiting center by severe pain. Ventricular dysfunction resulting from the MI may lead to the presence of the abnormal S3 and S4 heart sounds. Diaphoresis -During the initial phase of an MI, catecholamines are released from the ischemic myocardial cells, causing increased sympathetic nervous system stimulation. This results in the release of glycogen, diaphoresis, and vasoconstriction of peripheral blood vessels. The patient's skin may be ashen, cool, and clammy (not flushed) as a result of this response. Nausea and vomiting may result from reflex stimulation of the vomiting center by severe pain. Ventricular dysfunction resulting from the MI may lead to the presence of the abnormal S3 and S4 heart sounds. Nausea and vomiting -During the initial phase of an MI, catecholamines are released from the ischemic myocardial cells, causing increased sympathetic nervous system stimulation. This results in the release of glycogen, diaphoresis, and vasoconstriction of peripheral blood vessels. The patient's skin may be ashen, cool, and clammy (not flushed) as a result of this response. Nausea and vomiting may result from reflex stimulation of the vomiting center by severe pain. Ventricular dysfunction resulting from the MI may lead to the presence of the abnormal S3 and S4 heart sounds. S3 or S4 heart sounds -During the initial phase of an MI, catecholamines are released from the ischemic myocardial cells, causing increased sympathetic nervous system stimulation. This results in the release of glycogen, diaphoresis, and vasoconstriction of peripheral blood vessels. The patient's skin may be ashen, cool, and clammy (not flushed) as a result of this response. Nausea and vomiting may result from reflex stimulation of the vomiting center by severe pain. Ventricular dysfunction resulting from the MI may lead to the presence of the abnormal S3 and S4 heart sounds.

The nurse is caring for a patient who is 24 hours after pacemaker insertion. Which nursing intervention is appropriate at this time? >Reinforcing the pressure dressing as needed >Encouraging range-of-motion exercises of the involved arm >Assessing the incision for any redness, swelling, or discharge >Applying wet-to-dry dressings every 4 hours to the insertion site

Assessing the incision for any redness, swelling, or discharge Rationale: After pacemaker insertion, it is important for the nurse to observe signs of infection by assessing for any redness, swelling, or discharge from the incision site. The nonpressure dressing is kept dry until removed, usually 24 hours postoperatively. It is important for the patient to limit activity of the involved arm to minimize pacemaker lead displacement.

Postoperative care of a patient undergoing coronary artery bypass graft (CABG) surgery includes monitoring for which common complication? Dehydration Paralytic ileus Atrial dysrhythmias Acute respiratory distress syndrome

Atrial dysrhythmias Postoperative dysrhythmias, specifically atrial dysrhythmias, are common in the first 3 days after CABG surgery. Although the other complications could occur, they are not common complications.

A nurse is assessing a client and suspects the client is experiencing DIC. Which of the following physical findings should the nurse anticipate? A. Bradycardia B. Hypertension C. Epistaxis D. Xerostomia

C. Epistaxis Epistaxis is unexpected bleeding of the gums and nose and is a finding indicative of DIC

A patient has been diagnosed with acute myelogenous leukemia (AML). What should the nurse educate the patient that care will focus on? A. Leukapheresis B. Attaining remission C. One chemotherapy agent D. Waiting with active supportive care

B Attaining remission is the initial goal of care for leukemia. The methods to do this are decided based on age and cytogenetic analysis. The treatments include leukapheresis or hydroxyurea to reduce the white blood cell count and risk of leukemia-cell-induced thrombosis. A combination of chemotherapy agents will be used for aggressive treatment to destroy leukemic cells in tissues, peripheral blood, and bone marrow and minimize drug toxicity. In non-symptomatic patients with chronic lymphocytic leukemia, waiting may be done to attain remission, but not with AML.

The nurse is caring for a patient with a diagnosis of immune thrombocytopenic purpura (ITP). What is a priority nursing action in the care of this patient? A. Administration of packed red blood cells B. Administration of oral or IV corticosteroids C. Administration of clotting factors VIII and IX D. Maintenance of reverse isolation and application of standard precautions

B Common treatment modalities for ITP include corticosteroid therapy to suppress the phagocytic response of splenic macrophages. Blood transfusions, administration of clotting factors, and reverse isolation are not interventions that are indicated in the care of patients with ITP. Standard precautions are used with all patients.

A nurse is assessing a client who has pernicious anemia. Which of the following findings should the nurse expect? A. Thick, white coating on the clients tongue B. Decreased pulse rate C. Paresthesias in the hands and feet D. Joint pain in the extremities

C. Paresthesias in the hands and feet RATIONALE: Other manifestations include weight loss and fatigue -A is describing candidiasis

The nurse is providing care for older adults on a subacute, geriatric medical unit. What effect does aging have on hematologic function of older adults? A. Thrombocytosis B. Decreased hemoglobin C. Decreased WBC count D. Decreased blood volume

B Older adults frequently experience decreased hemoglobin levels as a result of changes in erythropoiesis. Decreased blood volume, decreased WBCs, and alterations in platelet number are not considered to be normal, age-related hematologic changes.

A nurse is caring for a client who has a new diagnosis of pernicious anemia. The nurse should expect the client's provider to prescribe which of the following medications for this client? A. Ferrous sulfate B. Epoetin alfa C. Vitamin B12 D. Folic acid

C. Vitamin B12

The nurse is caring for a patient with polycythemia vera. What is an important action for the nurse to initiate? A. Encourage deep breathing and coughing. B. Assist with or perform phlebotomy at the bedside. C. Teach the patient how to maintain a low-activity lifestyle. D. Perform thorough and regularly scheduled neurologic assessments.

B Primary polycythemia vera often requires phlebotomy in order to reduce blood volume. The increased risk of thrombus formation that accompanies the disease requires regular exercises and ambulation. Deep breathing and coughing exercises do not directly address the etiology or common sequelae of polycythemia, and neurologic manifestations are not typical.

A nurse is caring for a client who has scurvy. Which of the following vitamin deficiencies should the nurse identify as the cause of this disease? A. Vitamin A B. Vitamin B3 C. Vitamin C D. Vitamin D

C. Vitamin C

A nurse is developing a teaching plan for a client about preventing acute asthma attacks. Which of the following points should the nurse plan to discuss first? A. Eliminating environmental triggers that precipitate attacks B. Addressing the client's perception of the disease and what might have triggered past attacks C. Overviewing the client's medication regimen D. Explaining the manifestations of respiratory infections

B. Addressing the client's perception of the disease and what might have triggered past attacks RATIONALE: First assess the client's knowledge

The nurse is preparing to administer digoxin to a patient with heart failure. In preparation, laboratory results are reviewed with the following findings: sodium 139 mEq/L, potassium 5.6 mEq/L, chloride 103 mEq/L, and glucose 106 mg/dL. What should the nurse do next? A Withhold the daily dose until the following day. B Withhold the dose and report the potassium level. C Give the digoxin with a salty snack, such as crackers. D Give the digoxin with extra fluids to dilute the sodium level.

B Withhold the dose and report the potassium level The normal potassium level is 3.5 to 5.0 mEq/L. The patient is hyperkalemic, which makes the patient more prone to digoxin toxicity. For this reason, the nurse should withhold the dose and report the potassium level. The physician may order the digoxin to be given once the potassium level has been treated and decreases to within normal range.

A 62-yr-old male patient with disseminated intravascular coagulation (DIC) after urosepsis has a platelet count of 48,000/μL. The nurse should assess the patient for which abnormality? A. Pallor B. Purpura C. Pruritus D. Palpitation

B The normal range for a platelet count is 150,000 to 400,000/μL. Purpura is caused by decreased platelets or clotting factors, resulting in small hemorrhages into the skin or mucous membranes. Pallor is decreased or absent coloration in the conjunctiva or skin. Pruritus is an intense itching sensation. Palpitation is a sensation of feeling the heart beat, flutter, or pound in the chest.

The nurse notes a physician's order written at 10:00 AM for two units of packed red blood cells to be administered to a patient who is anemic as a result of chronic blood loss. If the transfusion is picked up at 11:30 AM, the nurse should plan to hang the unit no later than what time? A. 11:45 AM B. 12:00 noon C. 12:30 PM D. 3:30 PM

B The nurse must hang the unit of packed red blood cells within 30 minutes of signing them out from the blood bank.

When caring for a patient with metastatic cancer, the nurse notes a hemoglobin level of 8.7 g/dL and hematocrit of 26%. What associated clinical manifestations does the nurse anticipate observing? A. Thirst B. Fatigue C. Headache D. Abdominal pain

B The patient with a low hemoglobin and hematocrit is anemic and would be most likely to experience fatigue. Fatigue develops because of the lowered oxygen-carrying capacity that leads to reduced tissue oxygenation to carry out cellular functions. Thirst, headache, and abdominal pain are not related to anemia.

The thrombocytopenic patient has had a bone marrow biopsy taken from the posterior iliac crest. What nursing care is the priority for this patient after this procedure? A. Position the patient prone. B. Apply a pressure dressing. C. Administer analgesic for pain. D. Return metal objects to the patient.

B The sterile pressure dressing is applied after a bone marrow biopsy to ensure hemostasis. If bleeding is present, the patient will lie on the site and may need a rolled towel for additional pressure, thus this patient will not be in the prone position. The analgesic should have been administered pre-procedure. Metal objects would be removed for an MRI, not a bone marrow biopsy.

A nurse is providing teaching about lifestyle changes to a client who has experiences myocardial infarction and has a new prescription for a beta blocker. Which of the following client statements indicates an understanding of the teaching? A. "I should eat foods that are high in saturated fat" B. "Before taking my medication, I will count my radial pulse rate" C. "I will exercise once a week for an hour at the health club" D. "I will stop taking my medication when my blood pressure is within a normal range"

B. "Before taking my medication, I will count my radial pulse rate"

A nurse at a provider's office is reviewing information with a client scheduled for pulmonary function tests (PFTs). Which of the following information should the nurse include? A. "Do not use inhaler medications for 6 hr following the test." B. "Do not smoke tobacco for 6 to 8 hr prior to the test." C. "You will be asked to bear down and hold your breath during the test." D. "The arterial blood flow to your hand will be evaluated as part of the test."

B. "Do not smoke tobacco for 6 to 8 hr prior to the test." To ensure accurate results, the client should not smoke tobacco for 6 to 8 hr prior to the test.

A nurse is completing dietary teaching with a client who has heart failure and is prescribed a 2g sodium diet. Which of the following statements by the client indicates an understanding of the teaching? A. 'I should use salt sparingly while cooking" B. "I can have yogurt as a dessert" C. "I should use baking soda when I bake" D. "I should use canned vegetables instead of frozen"

B. "I can have yogurt as a dessert"

A nurse is caring for a client who has a tracheostomy and is receiving mechanical ventilation. When the low-pressure alarm on the ventilator sounds, it indicates which of the following to the nurse? A. Excessive airway secretions B. A leak within the ventilator's circuitry C. Decreased lung compliance D. The client coughing or attempting to talk

B. A leak within the ventilator's circuitry The low-pressure alarm means that either the ventilator tubing has come apart or the tubing detached from the client. Low-pressure alarms are often the result of a malfunction or displacement of connections somewhere between the endotracheal or tracheostomy tube and the ventilator.

The nurse supervises an unlicensed assistant personnel (UAP) who is taking the blood pressure of 58-year-old female patient admitted with heart failure. The patient is obese. The nurse should intervene if what is observed? A The UAP waits 2 minutes after position changes to take orthostatic pressures. B The UAP deflates the blood pressure cuff at a rate of 8 to 10 mm Hg per second. C The UAP takes the blood pressure with the patient's arm at the level of the heart. D The UAP takes a forearm blood pressure because the largest cuff will not fit the patient's upper arm.

B The UAP deflates the blood pressure cuff at a rate of 8 to 10 mm Hg per second. The cuff should be deflated at a rate of 2 to 3 mm Hg per second. The arm should be supported at the level of the heart for accurate blood pressure measurements. If the maximum size blood pressure cuff does not fit the upper arm, the forearm may be used. Orthostatic blood pressures should be taken within 1 to 2 minutes of repositioning the patient.

The nurse is caring for a patient admitted with a history of hypertension. The patient's medication history includes hydrochlorothiazide (Hydrodiuril) daily for the past 10 years. Which parameter would indicate the optimal intended effect of this drug therapy? A Weight loss of 2 lb B Blood pressure 128/86 C Absence of ankle edema D Output of 600 mL per 8 hours

B Blood pressure 128/86 Hydrochlorothiazide may be used alone as monotherapy to manage hypertension or in combination with other medications if not effective alone. After the first few weeks of therapy, the diuretic effect diminishes, but the antihypertensive effect remains. Since the patient has been taking this medication for 10 years, the most direct measurement of its intended effect would be the blood pressure.

The nurse admits a 73-year-old male patient with dementia for treatment of uncontrolled hypertension. The nurse will closely monitor for hypokalemia if the patient receives which medication? A Clonidine (Catapres) B Bumetanide (Bumex) C Amiloride (Midamor) D Spironolactone (Aldactone)

B Bumetanide (Bumex) Bumetanide is a loop diuretic. Hypokalemia is a common adverse effect of this medication. Amiloride is a potassium-sparing diuretic. Spironolactone is an aldosterone-receptor blocker. Hyperkalemia is an adverse effect of both amiloride and spironolactone. Clonidine is a central-acting α-adrenergic antagonist and does not cause electrolyte abnormalities.

A patient with a diagnosis of heart failure has been started on a nitroglycerin patch by his primary care provider. What should this patient be taught to avoid? A High-potassium foods B Drugs to treat erectile dysfunction C Nonsteroidal antiinflammatory drugs D Over-the-counter H2 -receptor blockers

B Drugs to treat erectile dysfunction The use of erectile drugs concurrent with nitrates creates a risk of severe hypotension and possibly death. High-potassium foods, NSAIDs, and H2-receptor blockers do not pose a risk in combination with nitrates.

What should the nurse recognize as an indication for the use of dopamine (Intropin) in the care of a patient with heart failure? A Acute anxiety B Hypotension and tachycardia C Peripheral edema and weight gain D Paroxysmal nocturnal dyspnea (PND)

B Hypotension and tachycardia Dopamine is a β-adrenergic agonist whose inotropic action is used for treatment of severe heart failure accompanied by hemodynamic instability. Such a state may be indicated by tachycardia accompanied by hypotension. PND, anxiety, edema, and weight gain are common signs and symptoms of heart failure, but these do not necessarily warrant the use of dopamine.

When teaching a patient about dietary management of stage 1 hypertension, which instruction is most appropriate? A Restrict all caffeine. B Restrict sodium intake. C Increase protein intake. D Use calcium supplements.

B Restrict sodium intake. The patient should decrease intake of sodium. This will help to control hypertension, which can be aggravated by excessive salt intake, which in turn leads to fluid retention. Caffeine and protein intake do not affect hypertension. Calcium supplements are not recommended to lower BP.

When providing dietary instruction to a patient with hypertension, the nurse would advise the patient to restrict intake of which meat? A Broiled fish B Roasted duck C Roasted turkey D Baked chicken breast

B Roasted duck Roasted duck is high in fat, which should be avoided by the patient with hypertension. Weight loss may slow the progress of atherosclerosis and overall CVD risk. The other meats are lower in fat and are therefore acceptable in the diet.

When assessing the patient for orthostatic hypotension, after taking the blood pressure (BP) and pulse (P) in the supine position, what should the nurse do next? A Repeat BP and P in this position. B Take BP and P with patient sitting. C Record the BP and P measurements. D Take BP and P with patient standing.

B Take BP and P with patient sitting. When assessing for orthostatic changes in BP after measuring BP in the supine position, the patient is placed in a sitting position and BP is measured within 1 to 2 minutes and then repositioned to the standing position with BP measured again, within 1 to 2 minutes. The results are then recorded with a decrease of 20 mm Hg or more in SBP, a decrease of 10 mm Hg or more in DBP, and/or an increase in pulse of greater than or equal to 20 beats/minute from supine to standing indicating orthostatic hypotension.

A nurse is caring for a client who has a tracheostomy and is receiving mechanical ventilation. When the low pressure alarm on the ventilator sounds, it indicates which of the following to the nurse? A. Excessive airway secretions B. A leak within the ventilators circuitry C. Decreased lung compliance D. The client coughing or attempting to talk

B. A leak within the ventilators circuitry

A nurse is caring for a client who has a history of angina and is scheduled for exercise electrocardiography at 1100. Which of the following statements by the client requires the nurse to contact the provider for possible rescheduling? A. "I smoked a cigarette this morning to calm my nerves about having this procedure" B. "I have had chest pain a couple of times since I saw my doctor in the office last week." C. "I'm still hungry after the bowl of cereal I ate at 7 a.m." D. "I didn't take my heart pills this morning because my doctor told me not to"

B. "I have had chest pain a couple of times since I saw my doctor in the office last week." Explanation: Smoking prior to this test can change the outcome and places the client at additional risk. The procedure should be rescheduled if the client has smoked before the test. Episodes of chest pain are not a contraindication to this test. It is not necessary for the client to be NPO prior to this procedure. The provider might withhold cardiovascular medications prior to this procedure to effectively monitor cardiovascular response to stress.

A nurse is reinforcing teaching with a client on the purpose of taking a bronchodilator. Which of the following client statements indicates understanding? A. "This medication can decrease my immune response." B. "I take this medication to prevent asthma attacks." C. "I need to take this medication with food." D. "This medication has a slow onset to treat my symptoms."

B. "I take this medication to prevent asthma attacks." A bronchodilator can prevent asthma attacks from occurring.

A nurse is providing teaching for a client who is scheduled for a bone marrow biopsy of the iliac crest. Which of the following statements made by the client indicates an understanding of the teaching? A. "This test will be performed while I am lying flat on my back." B. "I will need to stay in bed for about an hour after the test." C. "This test will determine which antibiotic I should take for treatment." D. "I will receive general anesthesia for the test."

B. "I will need to stay in bed for about an hour after the test." Inform the client of the need to stay on bed rest for 30 to 60 min following the test to reduce the risk for bleeding

A nurse is providing discharge teaching to an adult female client who has ineffective endocarditis about how to prevent recurrence. Which of the following statements by the client indicates an understanding of the teaching? A. "I will ask my provider to change my contraception to an intrauterine device" B. "I will notify my doctor before I have dental procedures" C. "I will avoid using antiseptic mouthwash for oral care" D. "I will wear a mask when I go out in public"

B. "I will notify my doctor before I have dental procedures" -Will need antibiotic prophylaxis

A home health nurse is teaching a client who has active tuberculosis and is following a medication regimen that includes a combination of isoniazid, rifampin, pyrazinamide, and ethambutol. Which of the following client statements indicate understanding? (Select all that apply.) A. "I can substitute one medication for another if I run out because they all fight infection." B. "I will wash my hands each time I cough." C. "I will wear a mask when I am in a public area." D. "I am glad I don't have to have any more sputum specimens." E. "I don't need to worry where I go once I start taking my medications."

B. "I will wash my hands each time I cough." C. "I will wear a mask when I am in a public area." The client should wash their hands each time they cough to prevent spreading the infection. The client should wear a mask while in public areas to prevent spreading the infection. The client has active TB, which is transmitted through the airborne route

A nurse is providing discharge teaching to a client who has emphysema. Which of the following instructions should the nurse include? A. "Be sure to take cough medicine to avoid coughing." B. "Try to drink at least 2 to 3 liters of fluid per day." C. "Try to reduce your smoking to 2 cigarettes per day." D. "Be sure to eat 3 full meals each day."

B. "Try to drink at least 2 to 3 liters of fluid per day." Although adequate hydration is essential for all clients, clients who have emphysema should drink 2 to 3 L per day to help liquefy secretions.

A nurse is teaching a client who has tuberculosis. Which of the following statements should the nurse include? A. "You will need to continue to take the multimedication regimen for 4 months." B. "You will need to provide sputum samples every 4 weeks to monitor the effectiveness of the medication." C. "You will need to remain hospitalized for treatment." D. "You will need to wear a mask at all times."

B. "You will need to provide sputum samples every 4 weeks to monitor the effectiveness of the medication." The client who has tuberculosis needs to provide sputum samples every 2 to 4 weeks to monitor the effectiveness of the medication.

A nurse at a community health center is dealing with people in reg community assessing the results of TB skin test. What would you expect to see on a positive TB skin test? A. 5mm site B. 15 mm site C. 20mm site D. 25mm site

B. 15 mm site

A nurse is providing health teaching to a group of clients. Which of the following clients is at risk for developing PAD? A. A client who has hypothyroidism B. A client who has diabetes mellitus C. A client whose daily caloric intake consists of 25% fat D. A client who consumes two 12-oz bottles of beer a day

B. A client who has diabetes mellitus

A nurse in a provider's office is reviewing the medical records of a group of clients. Which of the following clients is at risk for iron deficiency? SATA A. A client who is post menopausal B. A client who is vegetarian C. A middle adult male client D. A client who is pregnant E. A toddler who is overweight

B. A client who is vegetarian D. A client who is pregnant E. A toddler who is overweight RATIONALE: A toddler who is overweight may get most of their calories from milk and foods that are not considered healthy, putting them at risk for iron-deficiency anemia. Post-menopausal women are no longer at risk because they don't experience blood loss from menstruation

The nurse is caring for a client hospitalized with acute exacerbation of chronic obstructive pulmonary disease. Which findings would the nurse expect to note on assessment of this client? Select all that apply. A. A low arterial PCO₂ B. A hyperinflated chest noted on the chest x-ray C. Decreased oxygen saturation with mild exercise D. A widened diaphragm noted on the chest x-ray E. Pulmonary function tests that demonstrate increased vital capacity

B. A hyperinflated chest noted on the chest x-ray C. Decreased oxygen saturation with mild exercise

A nurse is developing a teaching plan for a client about preventing acute asthma attacks. Which of the following points should the nurse plan to discuss first? A. Eliminating environmental triggers that precipitate attacks B. Addressing the client's perception of the disease process and what might have triggered past attacks C. Overviewing the client's medication regimen D. Explaining manifestations of respiratory infections

B. Addressing the client's perception of the disease process and what might have triggered past attacks The nurse should apply the nursing process priority-setting framework to plan client care and prioritize nursing actions. Each step of the nursing process builds on the previous step, beginning with an assessment. Before the nurse can formulate a plan of action, implement a nursing intervention, or notify a provider of a change in the client's status, the nurse must first collect adequate data from the client. Assessing the client will provide the nurse with the knowledge to make an appropriate decision. Therefore, the nurse should first assess the client's current knowledge.

A nurse is caring for a client who has pneumonia. Assessment findings include temperature 37.8° C (100° F), respirations 30/min, blood pressure 130/76, heart rate 100/min, and SaO2 91% on room air. Prioritize the following nursing interventions. A. Administer antibiotics. B. Administer oxygen therapy. C. Perform a sputum culture. D. Instruct the client to obtain a yearly influenza vaccination.

B. Administer oxygen therapy. C. Perform a sputum culture. A. Administer antibiotics. D. Instruct the client to obtain a yearly influenza vaccination. The client's respiratory and heart rates are elevated, and her oxygen saturation is 91% on room air. Using the ABC priority framework, providing oxygen is the first intervention. Obtaining a sputum culture is the second nursing intervention. It should be done prior to administering oral medications to obtain an accurate specimen. Administration of antibiotics is the third action the nurse should take. The sputum culture should be obtained prior to antibiotic administration. The last action the nurse should take is to instruct the client to receive yearly influenza vaccinations, to reduce the risk of acquiring influenza that can lead to pneumonia.

What mask is good for patients with facial trauma or burns, and provides high humidification? A. Face tent B. Aerosol mask/face tent C. Nasal cannula D. Non-rebreather mask

B. Aerosol mask/face tent

A nurse is assessing a client who has peripheral vascular disease and a venous ulcer on the right ankle. Which of the following findings should the nurse expect in the client's affected extremity? A. Absent pedal pulses B. Ankle swelling C. Hair loss D. Skin atrophy

B. Ankle swelling RATIONALE: Other manifestations include brown pigmentation and cellulitis

A nurse is caring for a client who has a platelet count of 50,000. After discontinuing the client's peripheral IV site, which of the following actions should the nurse take? A. Apply warm compresses B. Apply pressure to the catheter removal site for 5 minutes C. Place the affected arm in a dependent position D. Clean the insertion site with alcohol

B. Apply pressure to the catheter removal site for 5 minutes

A nurse is preparing to administer packed RBCs to a client who has a Hgb of 8 g/dL. Which of the following actions should the nurse plan to take during the first 15 min of the transfusion? A. Obtain consent from the client for the transfusion. B. Assess for an acute hemolytic reaction. C. Explain the transfusion procedure to the client. D. Obtain blood culture specimens to send to the lab

B. Assess for an acute hemolytic reaction. Assess for an acute hemolytic reaction during the first 15 min of the transfusion. This form of a reaction can occur following the transfusion of as little as 10 mL of blood product.

A nurse is caring for a client who has heart failure and reports increased shortness of breath. Which of the following actions should the nurse take first? A. Obtain the client's weight. B. Assist the client into high‑Fowler's position. C. Auscultate lungs sounds. D. Check oxygen saturation with pulse oximeter

B. Assist the client into high‑Fowler's position. Using the airway, breathing, and circulation (ABC) priority approach to client care, the first action to take is to assist the client into high‑Fowler's

A nurse on a telemetry unit is caring for a client who has an irregular radial pulse. Which of the following ECG abnormalities should the nurse recognize as atrial flutter? A. P waves occurring at 0.16 seconds before each QRS complex B. Atrial rate 300/min with QRS complex of 80/min C. Ventricular rate of 82/min with an atrial rate of 80/min D. Irregular ventricular rate of 125/min with a wide QRS pattern

B. Atrial rate 300/min with QRS complex of 80/min RATIONALE: Lack of conduction between the atria and ventricles

A nurse is assessing for DIC in a client who has septic shock secondary to an untreated foot wound. Which of the following findings should the nurse expect? SATA A. Bradycardia B. Bleeding at the venipuncture site C. Petechiae on the chest and arms D. Flushed, dry skin E. Abdominal distention

B. Bleeding at the venipuncture site C. Petechiae on the chest and arms E. Abdominal distention RATIONALE: Abd distention due to internal bleeding

You are caring for a patient with a diagnosis of iron-deficiency anemia. Which clinical manifestations are you most likely to observe when assessing this patient? A. Convex nails, bright red gums, and alopecia B. Brittle nails; smooth, shiny tongue; and cheilosis C. Tenting of the skin, sunken eyes, and complaints of diarrhea D. Pale pink tongue; dull, brittle hair; and blue mucous membranes

B. Brittle nails; smooth, shiny tongue; and cheilosis Rationale: Specific clinical manifestations may be related to iron-deficiency anemia. Pallor is the most common finding, and glossitis (inflammation of the tongue) is the second most common; another finding is cheilitis (inflammation of the lips). The patient may report headache, paresthesias, and a burning sensation of the tongue, all of which are caused by lack of iron in the tissues. A sore tongue is a sign of cobalamin deficiency. Tenting skin is a sign of dehydration that often accompanies diarrhea. Blue mucous membranes are associated with cyanosis.

What does Amlodipine do? A. Beta Blocker that reduce HR & BP B. Calcium channel blocker that blocks calcium from going into the heart C. Diuretic that reduces liquids in the body D. Ace Inhibitor

B. Calcium channel blocker that blocks calcium from going into the heart

A nurse is caring for a client who has a nasogastric tube attached to low intermittent suctioning. The nurse should monitor for which of the following electrolyte imbalances? A. Hypercalcemia B. Hyponatremia C. Hyperphosphatemia D. Hyperkalemia

B. Hyponatremia Monitor the client for hyponatremia. Nasogastric losses are isotonic and contain sodium.

A nurse is caring for a client who is in the oliguric-anuric stage of acute kidney injury. The client reports diarrhea, a dull headache, palpitations, and muscle tingling and weakness. Which of the following actions should the nurse take first? A. Administer an analgesic to the client B. Check the client's electrolyte values C. Measure the client's weight D. Restrict the client's protein intake

B. Check the client's electrolyte values The nurse should apply the urgent versus nonurgent priority-setting framework when caring for the client. Using this framework, the nurse should consider urgent needs to be the priority because they pose a greater threat to the client. The nurse might also need to use Maslow's hierarchy of needs, the ABC priority-setting framework, and/or nursing knowledge to identify which finding is the most urgent. The nurse should check the client's most recent potassium value because these findings are manifestations of hyperkalemia, which can lead to cardiac dysrhythmias.

A nurse is preparing to administer packed RBCs to a client who is anemic. Which of the following actions should the nurse take? SATA A. Insert a 23-gauge angiocatheter with an IV adaptor B. Check to determine the packed RBCs are less than 1 week old C. Administer the packed RBCs over a 6 hour period D. Ask another nurse to check the packed RBCs label against the medical record E. Prime the transfusion tubing with 0.9% sodium chloride

B. Check to determine the packed RBCs are less than 1 week old D. Ask another nurse to check the packed RBCs label against the medical record E. Prime the transfusion tubing with 0.9% sodium chloride RATIONALE: NOT A- should be 18 to 20 gauge to allow the PRBCs to flow easily and prevent occlusion of the catheter; NOT C- should be over 2-4 hours to decrease risk of bacterial contamination

A older client is dx with pneumonia, what are some symptoms? A. Chronic productive cough B. Confusion and lethargy C. Digital clubbing D. Retractions

B. Confusion and lethargy

A nurse is assessing a client who has left-sided heart failure. Which of the following findings should the nurse expect? A. Pitting peripheral edema B. Crackles in the lung bases C. Jugular vein distention D. Hepatomegaly

B. Crackles in the lung bases RATIONALE: LEFT- LUNGS, Right= Rest of body

A nurse is preparing an in-service presentation about the management of MI. Death following MI is often a result of which of the following complications? A. Cardiogenic shock B. Dysrhythmias C. Heart failure D. Pulmonary edema

B. Dysrhythmias RATIONALE: Dysrhythmias are the most common cause of death following MI; nurses need to monitor ECG's carefully for dysrhythmias

A nurse is caring for a client who is experiencing acute opioid toxicity. Which of the following actions should the nurse identify as the priority? A. Insert a large-bore IV catheter B. Ensure an adequate airway C. Obtain an accurate medication history D. Prepare to administer an antagonist

B. Ensure an adequate airway

A nurse is caring for a client who is experiencing acute opioid toxicity. Which of the following actions should the nurse identify as the priority? A. Insert a large-bore IV catheter B. Ensure an adequate airway C. Obtain an accurate medication history D. Prepare to administer an antagonist

B. Ensure an adequate airway The first action the nurse should take when using the airway, breathing, and circulation (ABC) approach to client care is to ensure the client's airway is adequate, as respiratory depression is a manifestation of opioid toxicity.

A nurse is assessing a client who has a history of asthma. Which of the following factors should the nurse identify as a risk for asthma? A. Sex B. Environmental allergies C. Alcohol use D. History of diabetes

B. Environmental allergies Sex is not a risk factor associated with asthma. B. CORRECT: Environmental allergies are a risk factor associated with asthma. A client who has environmental allergies typically has other allergic problems, such as rhinitis or a skin rash.

A nurse is caring for a client who has severely elevated blood pressure. Which of the following findings should the nurse identify as a manifestation of hypertension? A. Vertigo B. Epistaxis C. Exophthalmos D. Spondylolisthesis

B. Epistaxis RATIONALE: HTN is often asymptomatic, but when it is severely elevated, it can also cause HA's, dizziness, facial flushing & fainting

A nurse is assessing a client who has a chest tube and drainage system in place. Which of the following are expected findings? (Select all that apply) A. Continuous bubbling in the water seal chamber B. Gentle constant bubbling in the suction control chamber C. Rise and fall in the level of water in the water seal chamber with inspiration D. Exposed sutures without dressing E. Drainage system upright at chest level

B. Gentle constant bubbling in the suction control chamber C. Rise and fall in the level of water in the water seal chamber with inspiration Gentle bubbling in the suction control chamber is an expected finding as air is being removed A rise and fall of the fluid level in the water seal chamber upon inspiration and expiration indicate that the drainage system is functioning properly

A nurse is caring for a client who has acute lymphocytic leukemia and reports a fever, chills, fatigue, and pallor over the past week. When checking the client's laboratory results, which of the following values should the nurse identify as contributing to the client's fatigue and pallor? A. Magnesium 2.0mEq/L B. Hgb 6.5g/dL C. WBC count 9.6/mm3 D. Creatinine 0.8mg/dL

B. Hgb 6.5g/dL RATIONALE: Normal Hgb is 12-18g/dL; a low Hgb can cause fatigue, HA, pallor, dizziness and tachycardia

A nurse is caring for a client who has chronic glomerulonephritis with oliguria. For which of the following electrolyte imbalances should the nurse monitor? A. Hypercalcemia B. Hyperkalemia C. Hypomagnesemia D. Hypophosphatemia

B. Hyperkalemia Oliguria resulting from chronic glomerulonephritis causes potassium retention, leading to levels above the expected reference range of 3.5 to 5 mEq/L. Other electrolyte imbalances common with this disorder affect sodium and phosphorus levels. Chronic glomerulonephritis eventually leads to end-stage kidney disease.

A nurse completing an assessment on a client. Which of the following findings should the nurse identify as a risk factor for coronary artery disease? SATA A. Hypothyroidism B. Hypertension C. Diabetes mellitus D. Hyperlipidemia E. Tobacco smoking

B. Hypertension C. Diabetes mellitus D. Hyperlipidemia E. Tobacco smoking

A nurse is teaching a client who has a new prescription for an ACEI to treat hypertension. The nurse should instruct the client to notify their provider if they experience which of the following adverse effects of this medication? A. Tendon pain B. Persistent cough C. Frequent urination D. Constipation

B. Persistent cough

A nurse is caring for a client who is receiving warfarin for anticoagulation therapy. Which of the following laboratory test results indicates to the nurse that the client needs an increase in the dosage? A. aPTT 38 seconds B. INR 1.1 C. PT 22 seconds D. D‑dimer negative

B. INR 1.1 INR of 1.1 is within the expected reference range for a client who is not receiving warfarin. However, this value is subtherapeutic for anticoagulation therapy. Expect the client to receive an increased dosage of warfarin until the INR is 2 to 3

A nurse is monitoring a client who has heart failure related to mitral stenosis. The client reports shortness of breath on exertion. Which of the following conditions should the nurse expect? A. Increased cardiac output B. Increased pulmonary congestion C. Decreased left atrial pressure D. Decreased pulmonary arterial pressure

B. Increased pulmonary congestion RATIONALE: Pulmonary congestion occurs due to right sided heart failure.

The blood bank notifies the nurse that the two units of blood ordered for an anemic patient are ready for pick up. Which action should the nurse take to prevent an adverse effect during this procedure? A. Immediately pick up both units of blood from the blood bank. B. Infuse the blood slowly for the first 15 minutes of the transfusion. C. Regulate the flow rate so that each unit takes at least 4 hours to transfuse. D. Set up the Y-tubing of the blood set with dextrose in water as the flush solution.

B. Infuse the blood slowly for the first 15 minutes of the transfusion. Because a transfusion reaction is more likely to occur at the beginning of a transfusion, the nurse should initially infuse the blood at a rate no faster than 2 mL/min and remain with the patient for the first 15 minutes after hanging a unit of blood. Only one unit of blood can be picked up at a time, must be infused within 4 hours, and cannot be hung with dextrose.

A patient who has sickle cell disease has developed cellulitis above the left ankle. What is the nurse's priority for this patient? A. Start IV fluids. B. Maintain oxygenation. C. Maintain distal warmth. D. Check peripheral pulses.

B. Maintain oxygenation. Maintaining oxygenation is a priority as sickling episodes are frequently triggered by low oxygen tension in the blood which is commonly caused by an infection. Antibiotics to treat cellulitis, pain control, and fluids to reduce blood viscosity will also be used, but oxygenation is the priority.

A patient whose tracheostomy was inserted 30 minutes ago is recovering in the postanesthesia recovery unit when the tracheostomy tube is expelled by coughing. What is the priority action by the nurse? A. Suction the tracheostomy opening. B. Maintain the airway with a sterile hemostat. C. Use an Ambu bag and mask to ventilate the patient. D. Insert the tracheostomy tube obturator into the stoma.

B. Maintain the airway with a sterile hemostat. As long as the patient is not in acute respiratory distress after dislodging the tracheostomy tube, the nurse should use a sterile hemostat to maintain an open airway until a sterile tracheostomy tube can be reinserted into the tracheal opening. The tracheostomy is an open surgical wound that has not had time to mature into a stoma. If the patient is in respiratory distress, the nurse will use an Ambu bag and mask to ventilate the patient temporarily.

A nurse is assisting a provider with a comprehensive physical examination of a client. When the provider uses transillumination, the nurse should explain to the client that this technique helps evaluate which of the following structures? A. Lymph nodes B. Maxillary sinuses C. Intercostal spaces D. Salivary glands

B. Maxillary sinuses RATIONALE: Clear sinus spaces allow transillumination

A nurse planning care for a client who has thrombocytopenia. Which of the following interventions should the nurse include in the plan of care? A. Restrict fluids to 1000mL per day B. Measure the client's abdominal girth daily C. Check IV sites every 4 hours for bleeding D. Administer an enema as needed for constipation

B. Measure the client's abdominal girth daily RATIONALE: This is to monitor for manifestations of internal bleeding; the nurse should check IV sites every 2 hours for bleeding (C)

The nurse is caring for a client with a nasogastric tube that is attached to low suction. The nurse monitors the client for manifestations of which disorder that the client is at risk for? A. Metabolic acidosis B. Metabolic alkalosis C. Respiratory acidosis D. Respiratory alkalosis

B. Metabolic alkalosis

A nurse is examining the ECG of a client who has frequent premature ventricular contractions (PVCs). Which of the following QRS changes should the nurse expect to see on the client's ECG? A. Narrower than usual QRS complexes B. Much greater amplitude than the usual QRS complexes C. Same polarity as the usual QRS complexes D. Immediate resumption of the usual rhythm

B. Much greater amplitude than the usual QRS complexes RATIONALE: The QRS complexes unusually have greater amplitude in height and depth in clients with PVCs

A pt has a K+ level of 3.0, what findings should you recognize as a reason for hypokalemia A. Diarrhea B. NG tube to suction C. Nausea D. Endoscopy

B. NG tube to suction

A nurse on a medical unit is caring for a client who aspirated gastric contents prior to admission. The nurse administers 100% oxygen by nonrebreather mask after the client reports severe dyspnea. Which of the following findings is a clinical manifestation of acute respiratory distress syndrome (ARDS)? A. Tympanic temperature 38°C (100.4°F) B. PaO2 50 mmHg C. Rhonchi D. Hypopnea

B. PaO2 50 mmHg This client who has manifestations of ARDS has a low PaO2 level, even after the administration of oxygen. Hypoxemia after treatment with oxygen is a manifestation of ARDS.

A nurse on a medical unit is caring for a client who aspirated gastric contents prior to admission. The nurse administers 100% oxygen by nonrebreather mask after the client reports severe dyspnea. Which of the following findings is a clinical manifestation of ARDS? A. Tympanic temperature of 100.4F B. PaO2 50mmHg C. Rhonchi D. Hypopnea

B. PaO2 50mmHg RATIONALE: ARDS= low PaO2 level, even after the administration of oxygen

A nurse on a cardiac unit is caring for a client who is on telemetry. The nurse recognizes the client's heart rate is 46/min and notifies the provider. Which of the following prescriptions might be appropriate for this client? A. Defibrillation B. Pacemaker insertion C. Synchronized cardioversion D. Administration of IV lidocaine

B. Pacemaker insertion A client who has bradycardia is a candidate for a pacemaker to increase his heart rate

A nurse is providing discharge teaching for a client who has a prescription for the transdermal nitroglycerin patch. Which of the following instructions should the nurse include in the teaching? A. Apply the new patch to the same site as the previous patch. B. Place the patch on an area of skin away from skin folds and joints. C. Keep the patch on 24 hr per day. D. Replace the patch at the onset of angina.

B. Place the patch on an area of skin away from skin folds and joints. Rationale: The nurse should instruct the client to apply the patch to an area of intact skin with enough room for the patch to fit smoothly.

A nurse is assessing a client who is receiving continuous ambulatory peritoneal dialysis. Which of the following findings should the nurse report to the provider? A. WBC 6,000/mm^3 B. Potassium 3.0 mEq/L C. Clear, pale yellow drainage D. Report of abdominal fullness

B. Potassium 3.0 mEq/L A potassium level of 3.0 mEq/L is below the expected reference range and can cause dysrhythmias. Dialysis removes fluid, waste products, and electrolytes from the blood and can cause hypokalemia.

The nurse is caring for a client who is on a mechanical ventilator. Blood gas results indicate a pH of 7.50 and a Paco2 of 30 mm Hg (30 mm Hg). The nurse has determined that the client is experiencing respiratory alkalosis. Which laboratory value would most likely be noted in this condition? A. Sodium level of 145 mEq/L (145 mmol/L) B. Potassium level of 3.0 mEq/L (3.0 mmol/L) C. Magnesium level of 1.8 (0.74 mmol/L) D. Phosphorus level of 3.0 mg/L (0.97 mmol/L)

B. Potassium level of 3.0 mEq/L (3.0 mmol/L)

A nurse is caring for a client who has hemophilia. The client reports pain and swelling in a joint following an injury. Which of the following actions should the nurse take? A. Obtain blood samples to test platelet function B. Prepare for replacement of the missing clotting factor C. Administer aspirin for the clients pain D. Place the bleeding joint in the dependent position

B. Prepare for replacement of the missing clotting factor RATIONALE: must prevent hemarthrosis (bleeding into joint space)

A nurse is monitoring the ECG of a client who has hypocalcemia. Which of the following findings should the nurse expect? A. Flattened T waves B. Prolonged QT intervals C. Shortened QT intervals D. Widened QRS complexes

B. Prolonged QT intervals RATIONALE: Other manifestations include tingling, numbness, tetany, seizures, prolonged QT intervals, and laryngospasm

A nurse is providing instructions about pursed-lip breathing for a client who has COPD. with emphysema. This breathing technique accomplishes which of the following? A. Increases oxygen intake B. Promotes Co2 elimination C. Uses the intercostal muscles D. Strengthens the diaphragm

B. Promotes Co2 elimination

A nurse is providing instructions about pursed-lip breathing for a client who has chronic obstructive pulmonary disease (COPD) with emphysema. This breathing technique accomplishes which of the following? A. Increases oxygen intake B. Promotes carbon dioxide elimination C. Uses the intercostal muscles D. Strengthens the diaphragm

B. Promotes carbon dioxide elimination A client who has COPD with emphysema should use pursed-lip breathing when experiencing dyspnea. This simple method slows the client's pace of breathing, making each breath more effective. Pursed-lip breathing releases trapped air in the lungs and prolongs exhalation in order to slow the breathing rate. This improved breathing pattern moves carbon dioxide out of the lungs more efficiently.

A patient has a tracheostomy tube after reconstructive surgery for invasive head and neck cancer. What is most important for the nurse to assess before performing tracheostomy cannula care? A. Level of consciousness B. Quality of breath sounds C. Presence of the gag reflex D. Tracheostomy cuff pressure

B. Quality of breath sounds Before performing tracheostomy care, the nurse will auscultate lung sounds to determine the presence of secretions. To prevent aspiration, secretions must be cleared either by coughing or by suctioning before performing tracheostomy cannula care.

A nurse is transfusing a unit of B-positive fresh frozen plasma whose blood type is O negative. Which of the following actions should the nurse take? A. Continue to monitor for manifestations of a transfusion reaction B. Remove the unit of plasma immediately and start an IV infusion of normal saline solution C. Continue the transfusion and repeat the type and crossmatch D. Prepare to administer a dose of diphenhydramine IV

B. Remove the unit of plasma immediately and start an IV infusion of normal saline solution RATIONALE: A client with type O can only receive type O!!!

A nurse is assessing a client who is receiving a blood transfusion. Which of the following findings is a manifestation of hemolytic transfusion reaction? A. Report of metallic taste B. Report of low back pain and apprehension C. Pallor D. Hypertension

B. Report of low back pain and apprehension Low back pain fever and chills are manifestations of a hemolytic transfusion reaction. The nurse should discontinue the transfusion and administer a 0.9% sodium chloride through a new IV tube.

A nurse is caring for an older adult client who has COPD with pneumonia. The nurse should monitor the client for which of the following acid-base imbalances? A. Respiratory alkalosis B. Respiratory acidosis C. Metabolic alkalosis D. Metabolic acidosis

B. Respiratory acidosis RATIONALE: Patient with COPD are unable to exhale CO2

A nurse is caring for an older adult client who has chronic obstructive pulmonary disease (COPD) with pneumonia. The nurse should monitor the client for which of the following acid-base imbalances? A. Respiratory alkalosis B. Respiratory acidosis C. Metabolic alkalosis D. Metabolic acidosis

B. Respiratory acidosis Respiratory acidosis is a common complication of COPD. This complication occurs because clients who have COPD are unable to exhale carbon dioxide due to a loss of elastic recoil in the lungs.

The nurse is evaluating and assessing a patient with a diagnosis of chronic emphysema. The patient is receiving oxygen at a flow rate of 5 L/min by nasal cannula. Which finding concerns the nurse immediately? A. Fine bibasilar crackles B. Respiratory rate of 8 breaths/min C. Patient sitting up and leaning over the nightstand D. A large barrel chest

B. Respiratory rate of 8 breaths/min

You anticipate the patient with hemochromatosis to be from which ethnic group? A. African American B. Hispanic American C. White European D. Chinese

C. White European Rationale: Hemochromatosis is the most common genetic disorder among whites, with an incidence of 3 to 5 cases per 1000 whites of European ancestry.

A nurse is caring for a client who was admitted for treatment of left sided HF and is receiving IV loop diuretics and digitalis therapy. The client is experiencing weakness and an irregular heart rate. Which of the following actions should the nurse take first? A. Obtain the client's current weight B. Review serum electrolyte values C. Determine the time of the last digoxin dose D. Check the clients urine output

B. Review serum electrolyte values RATIONALE: Weakness & irregular HR indicate electrolyte imbalance, an adverse effect of loop diuretics

A nurse is caring for a client who was admitted for a treatment of left-sided heart failure with intravenous loop diuretics and digitalis therapy. The client is experiencing weakness and an irregular heart rate. Which of the following actions should the nurse take first? A. Obtain clients current weight B. Review serum electrolyte values C. Determine the time of the last digoxin dose D. Check the clients urine output)

B. Review serum electrolyte values Rationale: Weakness and irregular heart rate indicate that the client is at the greatest risk for electrolyte imbalance, an adverse effect of loop diuretics. The first action the nurse should take is to review the client's electrolyte values, particularly the potassium level, because the client is at risk for dysrhythmias from hypokalemia.

A nurse is completing an integumentary assessment of a client who has anemia. Which of the following findings should the nurse expect? A. Absent turgor B. Spoon‑shaped nails C. Shiny, hairless legs D. Yellow mucous membranes

B. Spoon‑shaped nails Deformities of the nails, such as being spoon‑shaped, are findings in a client who has anemia.

What condition is a complication of uncontrolled asthma? A. Barrel chest B. Status Asthmaticus C. Pneumonia D. Thrombocytopenia

B. Status Asthmaticus

A patient is receiving 3% potassium chloride. There lab levels come back with high Sodium And high chloride. What should you do? A. Admin 0.9 NR B. Stop it C. Take vitals D. Re-take labs

B. Stop it

A nurse is administering a unit of PRBCs to a client who is postoperative. The client reports itching and hives 30 minutes after the infusion begins. Which of the following actions should the nurse take first? A. Maintain IV access with 0.9% Sodium Chloride B. Stop the infusion of blood C. Send the blood container and tubing to the blood bank D. Obtain a urine sample

B. Stop the infusion of blood

A pt has SOB, fatigue and palpitations that started 3 days ago with stable vital signs. What is the priority concern? A. Angina B. Stroke C. Tachycardia D. SOB

B. Stroke Atrial fib- Stroke, MI, clot forming situations

Which task can the registered nurse (RN) delegate to unlicensed assistive personnel (UAP) in the care of a stable patient who has a tracheostomy? A. Assessing the need for suctioning B. Suctioning the patient's oropharynx C. Assessing the patient's swallowing ability D. Maintaining appropriate cuff inflation pressure

B. Suctioning the patient's oropharynx Providing the person has been trained in correct technique, the UAP may suction the patient's oropharynx. Assessing the need for suctioning should be performed by an RN or licensed practical nurse. An RN should perform a swallowing assessment and maintain cuff inflation pressure.

A nurse in a clinic receives a phone call from a client seeking information about a new prescription for erythropoietin. Which of the following information should the nurse review with the client? A. The client needs an erythrocyte sedimentation rate (ESR) test weekly. B. The client should have their hemoglobin checked twice a week. C. Oxygen saturation levels should be monitored. D. Folic acid production will increase.

B. The client should have their hemoglobin checked twice a week. Include in the teaching that hemoglobin and hematocrit are monitored twice a week until the targeted levels are reached.

A nurse is teaching a client with cystic fibrosis about daily chest physiotherapy. Which of the following is the purpose of these treatments? A. To encourage deep breaths B. To mobilize secretions in the airways C. To dilate the bronchioles D. To stimulate the cough reflex

B. To mobilize secretions in the airways

A Pt has pneumonia with a fever and coughing and pleuritic chest pain when taking deep breaths. What should you do? A. Give pain medication B. Treat the infection C. Start an IV D. Get a x-ray

B. Treat the infection

A patient who has just arrived in the emergency department reports substernal and left arm discomfort that has been going on for about 3 hours. Which laboratory test will be most useful in determining whether the nurse should anticipate implementing the acute coronary syndrome standard protocol? A. Creatine kinase MB level B. Troponin I level C. Myoglobin level D. C-reactive protein level

B. Troponin I level

A nurse is providing discharge teaching to a client who has emphysema. Which of the following instructions should the nurse include? A. Be sure to take cough medicine to avoid coughing B. Try to drink at least 3 liters of fluid per day C. Try to reduce your smoking to 2 cigarettes per day D. Be sure to eat 3 full meals each day

B. Try to drink at least 3 liters of fluid per day

A patient has been diagnosed with stage 1A Hodgkin's lymphoma. The nurse knows that which chemotherapy regimen is most likely to be prescribed for this patient? A. Brentuximab vedotin (Adcetris) B. Two to four cycles of ABVD: doxorubicin (Adriamycin), bleomycin, vinblastine, and dacarbazine C. Four to six cycles of ABVD: doxorubicin (Adriamycin), bleomycin, vinblastine, and dacarbazine D. BEACOPP: bleomycin, etoposide, doxorubicin (Adriamycin), cyclophosphamide, vincristine (Oncovin), procarbazine, and prednisone

B. Two to four cycles of ABVD: doxorubicin (Adriamycin), bleomycin, vinblastine, and dacarbazine The patient with a favorable prognosis early-stage Hodgkin's lymphoma (stage 1A) will receive two to four cycles of ABVD. The unfavorable prognostic featured (stage 1B) Hodgkin's lymphoma would be treated with four to six cycles of chemotherapy. Advanced-stage Hodgkin's lymphoma is treated more aggressively with more cycles or with BEACOPP. Brentuximab vedotin (Adcetris) is a newer agent that will be used to treat patients who have relapsed or refractory disease.Note: Some of acronyms for drug protocols use the brand/trade name of drugs (Adriamycin, Oncovin). These brand/trade names have been discontinued but the drugs are still available as generic drugs.

A nurse is monitoring a client's ECG monitor and notes the client's rhythm has changed from normal sinus rhythm to supraventricular tachycardia. The nurse should prepare to assist with which of the following interventions? A. Initiate chest compressions B. Vagal stimulation C. Administration of atropine IV D. Defibrillation

B. Vagal stimulation RATIONALE: vagal stimulation may temporarily convert the client's HR to normal sinus rhythm, but should have the defibrillator ready at the client's bedside because vagal stimulation can cause bradydysrhythmias, ventricular dysrhythmias or asystole

What type of mask offers the MOST PRECISE amount of O2 without intubation and provides high humidification? A. Face tent B. Venturi mask C. Nasal cannula D. Non-rebreather mask

B. Venturi mask

A nurse is assessing a client who has left sided HF. Which of the following manifestations should the nurse expect to find? A. Increased abdominal girth B. Weak peripheral pulses C. JVD D. Dependent edema

B. Weak peripheral pulses RATIONALE: This represents decreased CO

A nurse in the emergency department is caring for a client who is experiencing an acute asthma attack. Which of the following assessments indicates that the respiratory status is declining? (Select all that apply.) A. SaO2 95% B. Wheezing C. Retraction of sternal muscles D. Pink mucous membranes E. Tachycardia

B. Wheezing C. Retraction of sternal muscles E. Tachycardia Wheezing indicates airway narrowing and is a manifestation indicating the client's respiratory status is declining. Retraction of sternal muscles is associated with increased work of breathing and is a manifestation that the client's respiratory status is declining. Tachycardia can be a manifestation of decreased oxygenation and an indicator that the client's respiratory status is declining.

The primary pathophysiology underlying thalassemia is A. erythropoietin deficiency. B. abnormal hemoglobin synthesis. C. autoimmunity. D. S-shaped hemoglobin.

B. abnormal hemoglobin synthesis. Rationale: Thalassemia is a group of autosomal recessive diseases that involve inadequate production of normal hemoglobin. Hemolysis also occurs in thalassemia, but insufficient production of normal hemoglobin is the predominant problem. Erythropoietin deficiency is associated with a renal disorder, and S-shaped hemoglobin is associated with sickle cell disease.

When obtaining assessment data from a patient with a microcytic, hypochromic anemia, you question the patient about A. folic acid intake. B. dietary intake of iron. C. a history of gastric surgery. D. a history of sickle cell anemia.

B. dietary intake of iron. Rationale: Iron deficiency anemia is a type of microcytic, hypochromic anemia.

The nurse caring for a client with chronic obstructive pulmonary disease (COPD) anticipates which arterial blood gas (ABG) findings? A. pH, 7.40; PaO2, 90 mm Hg; CO2, 39 mEq/L; HCO3, 23 mEq/L B. pH, 7.32; PaO2, 85 mm Hg; CO2, 57 mEq/L; HCO3, 26 mEq/L C. pH, 7.47; PaO2, 82 mm Hg; CO2, 30 mEq/L; HCO3, 31 mEq/L D. pH, 7.31; PaO2, 95 mm Hg; CO2, 22 mEq/L; HCO3, 19 mEq/L

B. pH, 7.32; PaO2, 85 mm Hg; CO2, 57 mEq/L; HCO3, 26 mEq/L

a charge nurse is reviewing the care of a client who has a chest tube connected to a water seal drainage system in a place following thoracic surgery w/ newly licensed nurse. which of the following statements by the newly licensed nurse indicates an understanding of when to notify the provider? A.) "I will notify the provider if there is a fluctuation of drainage in the tubing with inspiration." B.) "I will notify the provider if there is continuous bubbling in the water seal chamber." C.) "I will notify the provider if there is drainage of 60 milliliters in the first hour after surgery." D.) "I will notify the provider if there are several small, dark-red blood clots in the tubing."

B.) "I will notify the provider if there is continuous bubbling in the water seal chamber." rationale: Continuous bubbling in the water seal chamber suggests an air leak and requires notification of the provider. The nurse should check the system for external, correctable leaks while waiting for instructions from the provider.

a charge nurse is providing an in-service to a group of staff nurses about endotracheal suctioning. which of the following statements by a staff nurse indicates an understanding of the teaching? A.) "I will use clean technique when suctioning a client's endotracheal tube." B.) "I will use a rotating motion when removing the suction catheter." C.) "I will suction the oropharyngeal cavity prior to suctioning the endotracheal tube." D.) "I will suction a client's endotracheal tube every 2 hours."

B.) "I will use a rotating motion when removing the suction catheter." rationale: The nurse should rotate the suction catheter during withdrawal to remove secretions from the sides of the airway.

a nurse is assessing a client who's 4 hr postoperative following a total laryngectomy. which of the following findings is the priority for the nurse to report to the provider? A.) Bleeding at the surgical site B.) Decreased oxygen saturation C.) Urinary retention D.) Increased pain level

B.) Decreased oxygen saturation rationale: When using the airway, breathing, circulation approach to client care, the nurse should identify decreased oxygen saturation as the priority finding to address and report to the provider. A client who is postoperative following a total laryngectomy is at higher risk for hypoxia because of airway obstruction.

a nurse is assessing a client who has emphysema. which of the following findings should the nurse report to the provider? A.) Rhonchi on inspiration B.) Elevated temperature C.) Barrel-shaped chest D.) Diminished breath sounds

B.) Elevated temperature rationale: The nurse should report an elevated temperature to the provider because it can indicate a possible respiratory infection. Clients who have emphysema are at risk for the development of pneumonia and other respiratory infections.

a nurse is caring for a newly admitted client who has emphysema. the nurse should place the client in which of the following positions to promote effective breathing? A.) Lateral position with a pillow at the back and over the chest to support the arm B.) High-Fowler's position with the arms supported on the overbed table C.) Semi-Fowler's position with pillows supporting both arms D.) Supine position with the head of the bed elevated to 15°

B.) High-Fowler's position with the arms supported on the overbed table rationale: The nurse should place the client in a position that allows for greater expansion of the chest, such as sitting upright and leaning slightly forward while supporting both arms with pillows for comfort on the overbed table.

a nurse is caring for a client who's in respiratory distress. which of the following low-flow delivery devices should the nurse use to provide the client w/ highest level of oxygen? A.) Nasal cannula B.) Nonrebreather mask C.) Simple face mask D.) Partial rebreather mask

B.) Nonrebreather mask rationale: The nurse should use a nonrebreather mask for a client who is in respiratory distress to provide the highest oxygen level. A nonrebreather mask is made up of a reservoir bag from which the client obtains the oxygen, a one-way valve to prevent exhaled air from entering the reservoir bag, and exhalation ports with flaps that prevent room air from entering the mask. This device delivers greater than 90% FiO2.

a nurse in a provider's office is assessing a client who has COPD. which of the following findings is the priority for the nurse to report to the provider? A.) Increased anterior-posterior chest diameter B.) Productive cough with green sputum C.) Clubbing of the fingers D.) Pursed-lip breathing with exertion

B.) Productive cough with green sputum rationale: When using the urgent vs. nonurgent approach to client care, the nurse should determine that the priority finding is a productive cough with green sputum. The nurse should report this finding to the provider because it can indicate infection.

A nurse is providing discharge teaching to a client who has a new prescription for prednisone for asthma. Which of the following client statements indicates understanding? A. "I will decrease my fluid intake while taking this medication." B. "I will expect to have black, tarry stools." C. "I will take my medication with meals." D. "I will monitor for weight loss while on this medication."

C. "I will take my medication with meals." The client should take this medication with food. Taking prednisone on an empty stomach can cause gastrointestinal distress.

A nurse is caring for a client who has idiopathic thrombocytopenic purpura (ITP). The nurse should notify the provider and report possible small‑vessel clotting when which of the following is assessed? A. Petechiae on the upper chest B. Hypotension C. Cyanotic nail beds D. Severe headache

C. Cyanotic nail beds Cyanotic nail beds indicate microvascular clotting is occurring and should be immediately reported to avoid ischemic loss of the fingers or toes

A nurse in a clinic is assessing the lower extremities and ankles of a client who has a history of peripheral arterial disease. Which of the following findings should the nurse expect? A. Pitting edema B. Areas of reddish-brown pigmentation C. Dry, pale skin with minimal body hair D. Sunburned appearance with desquamation

C. Dry, pale skin with minimal body hair RATIONALE: A and B are PVD/CVI

A blood type and cross-match has been ordered for a male patient who is experiencing an upper gastrointestinal bleed. The results of the blood work indicate that the patient has type A blood. Which description explains what this means? A. The patient can be transfused with type AB blood. B. The patient may only receive a type A transfusion. C. The patient has A antigens on his red blood cells (RBCs). D. Antibodies are present on the surface of the patient's RBCs.

C An individual with type A blood has A antigens, not A antibodies, on his RBCs. An AB transfusion would result in agglutination, but he may be transfused with either type A or type O blood.

A nurse is assessing a client who has pericarditis. Which of the following manifestations should the nurse expect? A. Bradycardia with ST-segment depression B. Relief of chest pain with deep inspiration C. Dyspnea with hiccups D. Chest pain that increases when sitting upright

C. Dyspnea with hiccups RATIONALE: A client who has pericarditis will experience dyspnea, hiccups and a nonproductive cough. These manifestations can indicate HF from pericardial compression due to constrictive pericarditis or cardiac tamponade

A nurse in an ED is caring for a client who has a BP of 254/139. The nurse recognizes that the client is in a hypertensive crisis. Which of the following actions should the nurse take first? A. Initiate seizure precautions B. Tell the client to report vision changes C. Elevate the head of the client's bed D. Start a peripheral IV

C. Elevate the head of the client's bed RATIONALE: this is to reduce BP and promote oxygenation

A nurse is caring for a client for whom the respiratory therapist has just removed the endotracheal tube. Which of the following actions should the nurse take first? A. Instruct the client to cough B. Administer oxygen via facemask C. Evaluate the client for stridor D. Keep the client in a semi-to-high Fowler's position

C. Evaluate the client for stridor RATIONALE: stridor= high pitched sound that indicates laryngospasm or swelling around the glottis (the other actions are relevant, but not priority)

A client is planning care for a client who has pernicious anemia. Which of the following interventions should the nurse include int he plan? A. Administer ferrous sulfate supplementation B. Increase dietary intake of folic acid C. Initiate weekly injections of B12 D. Initiate a blood transfusion

C. Initiate weekly injections of B12

A nurse is caring for a client who presents to the ER with a BP of 254/138 mmhg. The nurse recognizes that the client is in a hypertensive crisis. Which of the following actions should the nurse take first? A. Initiate seizure precautions. B. Tell the client to report vision changes C. Elevate the head of the clients bed D. Start a peripheral IV

C. Elevate the head of the clients bed Rationale: The greatest risk to this client is organ injury due to severe hypertension. Therefore, the first action the nurse should take is to elevate the head of the client's bed to reduce blood pressure and promote oxygenation.

A 36-yr-old female patient suspected of having leukemia is scheduled for a bone marrow aspiration. What statement in the patient's health history requires immediate follow-up by the nurse? A. "I had a bad reaction to iodine once before and almost died." B. "I am taking an anti-biotic to treat a urinary tract infection." C. "I have rheumatoid arthritis and take aspirin for joint pain." D. "I have dialysis for chronic renal failure three times a week."

C Complications of bone marrow aspiration are minimal, but there is a possibility of damaging underlying structures especially if the sternum site is used. Other complications include hemorrhage particularly if the patient is thrombocytopenic and infection particularly if the patient is leukopenic. The risk of hemorrhage is increased if the patient takes aspirin because it promotes bleeding by inhibiting platelet aggregation. Contrast dye is not used during a bone marrow aspiration. A bone marrow aspiration is not contraindicated in patients who have chronic renal failure on dialysis or a urinary tract infection on an antibiotic.

A nurse is caring for a client who has COPD and is experiencing SOB. Which of the following actions should the nurse perform? A. Monitor the client's ABG results B. Instruct the client to perform controlling coughing C. Teach the client how to use pursed-lip breathing D. Place the client in an upright position

D. Place the client in an upright position

The nurse is planning health promotion teaching for a group of healthy older adults in a residential community. Which statement accurately describes expected hematologic effects of aging? A. "Platelet production increases with age and leads to easy bruising." B. "Anemia is common with aging because iron absorption is impaired." C. "Older adults with infections may have only a mild white blood cell count elevation." D. "Older adults often have poor immune function with a decreased number of lymphocytes"

C During an infection, the older adult may have only a minimal elevation in the total white blood cell count and may not have a fever. Presentation of infection can initially be nonspecific with disorientation, anorexia, and weakness. Platelets are unaffected by the aging process. However, changes in vascular integrity from aging can manifest as easy bruising. Iron absorption is not impaired in the older patient, but adequate nutritional intake of iron may be decreased. The total white blood cell count and differential are generally not affected by aging. However, a decrease in humoral antibody response and decrease in T-cell function may occur.

A nurse is caring for a client who has a major burn injury and is experiencing third spacing. Which of the following fluid or electrolyte imbalances should the nurse expect? A. Hypokalemia B. Hypernatremia C. Elevated Hct D. Decreased Hgb

C. Elevated Hct RATIONALE: elevated hematocrit as blood volume is reduced A- should be hyperkalemia B-should be hyponatremia (low Na, High K with burns)

A nurse is caring for a client who has a major burn injury and is experiencing third spacing. Which of the following fluid or electrolyte imbalances should the nurse expect? A. Hypokalemia B. Hypernatremia C. Elevated Hct D. Decreased Hgb

C. Elevated Hct The nurse should expect a client who is experiencing third spacing resulting from a major burn to have an elevated hematocrit level as blood volume is reduced by vascular dehydration.

When preparing to administer an ordered blood transfusion, which IV solution does the nurse use when priming the blood tubing? A. Lactated Ringer's B. 5% dextrose in water C. 0.9% sodium chloride D. 0.45% sodium chloride

C The blood set should be primed before the transfusion with 0.9% sodium chloride, also known as normal saline. It is also used to flush the blood tubing after the infusion is complete to ensure the patient receives blood that is left in the tubing when the bag is empty. Dextrose and lactated Ringer's solutions cannot be used with blood as they will cause RBC hemolysis.

Before beginning a transfusion of RBCs, which action by the nurse would be of highest priority to avoid an error during this procedure? A. Add the blood transfusion as a secondary line to the existing IV. B. Stay with the patient for 60 minutes after starting the transfusion. C. Check the identifying information on the unit of blood against the patient's ID bracelet. D. Prime new primary IV tubing with lactated Ringer's solution to use for the transfusion.

C The patient's identifying information (name, date of birth, medical record number) on the ID bracelet should exactly match the information on the blood bank tag that has been placed on the unit of blood. If any information does not match, the transfusions should not be hung because of possible error and risk to the patient. The transfusion is hung on blood transfusion tubing, not a secondary line, and cannot be hung with lactated Ringer's because it will cause RBC hemolysis. Usually, the patient will need continuous monitoring for 15 minutes after the transfusion is started, as this is the time most transfusion reactions occur. Then the patient should be monitored every 30 to 60 minutes during the administration.

A nurse is caring for an adult male client who is undergoing screening tests for atherosclerosis. Which of the following laboratory findings should the nurse identify as an increased risk for this disorder? A. Cholesterol level 195 B. Elevated HDL levels C. Elevated LDL levels D. Triglyceride level 135

C. Elevated LDL levels (desirable level should be <100mg/dL)

The nurse is caring for a patient with a diagnosis of disseminated intravascular coagulation (DIC). What is the first priority of care? A. Administer heparin. B. Administer whole blood. C. Treat the causative problem. D. Administer fresh frozen plasma.

C Treating the underlying cause of DIC will interrupt the abnormal response of the clotting cascade and reverse the DIC. Blood product administration occurs based on the specific component deficiencies and is reserved for patients with life-threatening hemorrhage. Heparin will be administered if the manifestations of thrombosis are present and the benefit of reducing clotting outweighs the risk of further bleeding.

A nurse is caring for a client with pneumonia who is experiencing thick oral secretions. Which of the following actions should the nurse take first? A. Provide chest physiotherapy B. Perform oropharyngeal suction C. Encourage deep breathing and coughing D. Assist the client with ambulation

C. Encourage deep breathing and coughing

The nurse teaches a 28-year-old man newly diagnosed with hypertension about lifestyle modifications to reduce his blood pressure. Which statement by the patient requires an intervention by the nurse? A "I will avoid adding salt to my food during or after cooking." B "If I lose weight, I might not need to continue taking medications." C "I can lower my blood pressure by switching to smokeless tobacco." D "Diet changes can be as effective as taking blood pressure medications."

C "I can lower my blood pressure by switching to smokeless tobacco." Nicotine contained in tobacco products (smoking and chew) cause vasoconstriction and increase blood pressure. Persons with hypertension should restrict sodium to 1500 mg/day by avoiding foods high in sodium and not adding salt in preparation of food or at meals. Weight loss can decrease blood pressure between 5 to 20 mm Hg. Following dietary recommendations (such as the DASH diet) lowers blood pressure, and these decreases compare with those achieved with blood pressure-lowering medication.

A 54-year-old male patient who had bladder surgery 2 days ago develops acute decompensated heart failure (ADHF) with severe dyspnea. Which action by the nurse would be indicated first? A Perform a bladder scan to assess for urinary retention. B Restrict the patient's oral fluid intake to 500 mL per day. C Assist the patient to a sitting position with arms on the overbed table. D Instruct the patient to use pursed-lip breathing until the dyspnea subsides.

C Assist the patient to a sitting position with arms on the overbed table. The nurse should place the patient with ADHF in a high Fowler's position with the feet horizontal in the bed or dangling at the bedside. This position helps decrease venous return because of the pooling of blood in the extremities. This position also increases the thoracic capacity, allowing for improved ventilation. Pursed-lip breathing helps with obstructive air trapping but not with acute pulmonary edema. Restricting fluids takes considerable time to have an effect.

What is the priority assessment by the nurse caring for a patient receiving IV nesiritide (Natrecor) to treat heart failure? A Urine output B Lung sounds C Blood pressure D Respiratory rate

C Blood pressure Although all identified assessments are appropriate for a patient receiving IV nesiritide, the priority assessment would be monitoring for hypotension, the main adverse effect of nesiritide.

The nurse is caring for a patient admitted with chronic obstructive pulmonary disease (COPD), angina, and hypertension. Before administering the prescribed daily dose of atenolol 100 mg PO, the nurse assesses the patient carefully. Which adverse effect is this patient at risk for, given the patient's health history? A Hypocapnia B Tachycardia C Bronchospasm D Nausea and vomiting

C Bronchospasm Atenolol is a cardioselective β1-adrenergic blocker that reduces blood pressure and could affect the β2-receptors in the lungs with larger doses or with drug accumulation. Although the risk of bronchospasm is less with cardioselective β-blockers than nonselective β-blockers, atenolol should be used cautiously in patients with COPD.

The patient has chronic hypertension. Today she has gone to the ED, and her blood pressure has risen to 200/140. What is the priority assessment for the nurse to make? A Is the patient pregnant? B Does the patient need to urinate? C Does the patient have a headache or confusion? D Is the patient taking antiseizure medications as prescribed?

C Does the patient have a headache or confusion? The nurse's priority assessments include neurologic deficits, retinal damage, heart failure, pulmonary edema, and renal failure. The headache or confusion could be seen with hypertensive encephalopathy from increased cerebral capillary permeability leading to cerebral edema. Pregnancy can lead to secondary hypertension. Needing to urinate and taking antiseizure medication do not support a hypertensive emergency.

A nurse is caring for a client with pneumonia who is experiencing thick oral secretions. Which of the following actions should the nurse take first? A. Provide chest physiotherapy B. Perform oropharyngeal suction C. Encourage deep-breathing and coughing D. Assist the client with ambulation

C. Encourage deep-breathing and coughing The first action the nurse should take when using the airway, breathing, and circulation (ABC) approach is to encourage the client to breathe deeply and cough to clear secretions from the airway.

When teaching how lisinopril (Zestril) will help lower the patient's blood pressure, which mechanism of action should the nurse use to explain it? A Blocks β-adrenergic effects. B Relaxes arterial and venous smooth muscle. C Inhibits conversion of angiotensin I to angiotensin II. D Reduces sympathetic outflow from central nervous system.

C Inhibits conversion of angiotensin I to angiotensin II. Lisinopril is an angiotensin-converting enzyme (ACE) inhibitor that inhibits the conversion of angiotensin I to angiotensin II, which reduces angiotensin II-mediated vasoconstriction and sodium and water retention. Beta blockers result in vasodilation and decreased heart rate. Direct vasodilators relax arterial and venous smooth muscle. Central acting α-adrenergic antagonists reduce sympathetic outflow from the CNS to produce vasodilation and decreased SVR and BP.

The nurse is teaching a women's group about prevention of hypertension. What information should be included in the teaching for all the women (select all that apply)? A Lose weight. B Limit nuts and seeds. C Limit sodium and fat intake. D Increase fruits and vegetables. E Exercise 30 minutes most days.

C Limit sodium and fat intake. D Increase fruits and vegetables. E Exercise 30 minutes most days. Primary prevention of hypertension is to make lifestyle modifications that prevent or delay the increase in BP. Along with exercise for 30 minutes on most days, the DASH eating plan is a healthy way to lower BP by limiting sodium and fat intake, increasing fruits and vegetables, and increasing nutrients that are associated with lowering BP. Nuts and seeds and dried beans are used for protein intake. Weight loss may or may not be necessary for the individual.

A 67-year-old woman with a history of coronary artery disease and prior myocardial infarction is admitted to the emergency department with a blood pressure of 234/148 mm Hg and started on IV nitroprusside (Nitropress). What should the nurse determine as an appropriate goal for the first hour of treatment? A Mean arterial pressure lower than 70 mm Hg B Mean arterial pressure no more than 120 mm Hg C Mean arterial pressure no lower than 133 mm Hg D Mean arterial pressure between 70 and 110 mm Hg

C Mean arterial pressure no lower than 133 mm Hg The initial treatment goal is to decrease mean arterial pressure by no more than 25% within minutes to 1 hour. If the patient is stable, the goal for BP is 160/100 to 110 mm Hg over the next 2 to 6 hours. Lowering the blood pressure too much may decrease cerebral, coronary, or renal perfusion and could precipitate a stroke, myocardial infarction, or renal failure. Additional gradual reductions toward a normal blood pressure should be implemented over the next 24 to 48 hours if the patient is clinically stable.

In caring for a patient admitted with poorly controlled hypertension, which laboratory test result should the nurse understand as indicating the presence of target organ damage? A BUN of 15 mg/dL B Serum uric acid of 3.8 mg/dL C Serum creatinine of 2.6 mg/dL D Serum potassium of 3.5 mEq/L

C Serum creatinine of 2.6 mg/dL The normal serum creatinine level is 0.6-1.3 mg/dL. This elevated level indicates target organ damage to the kidneys. The other lab results are within normal limits.

The nurse prepares to administer digoxin (Lanoxin) 0.125 mg to an 82-year-old man admitted with influenza and a history of chronic heart failure. What should the nurse assess before giving the medication? A Prothrombin time B Urine specific gravity C Serum potassium level D Hemoglobin and hematocrit

C Serum potassium level Serum potassium should be monitored because hypokalemia increases the risk for digoxin toxicity. Changes in prothrombin time, urine specific gravity, and hemoglobin or hematocrit would not require holding the digoxin dose.

A nurse is providing teaching to a client who has a chronic cough and is scheduled for a bronchoscopy. Which of the following client statements indicates an understanding of the teaching? A. "I can keep my dentures in during the procedure." B. "I am allowed only clear liquids prior to the procedure." C. "A tissue sample might be obtained during the procedure." D. "A signed consent form is not required for this procedure."

C. "A tissue sample might be obtained during the procedure." The nurse should inform the client that a tissue sample might be obtained during the procedure for biopsy testing.

A nurse is teaching a client who has polycythemia vera about self-care measures. Which of the following interventions should the nurse include? A. "Drink at least 1 liter of fluid each day" B. "Continuously wear support hose" C. "Elevate your legs when sitting" D. "Use dental floss daily"

C. "Elevate your legs when sitting" RATIONALE: To avoid pooling and clot formation

A nurse is providing discharge teaching to a client who has COPD and a new prescription for albuterol. Which of the following statements by the client indicates an understanding of the teaching? A. "This medication can increase my blood sugar levels." B. "This medication can decrease my immune response." C. "I can have an increase in my heart rate while taking this medication." D. "I can have mouth sores while taking this medication."

C. "I can have an increase in my heart rate while taking this medication." Bronchodilators, such as albuterol, can cause tachycardia

A nurse is providing discharge teaching to client who has aplastic anemia. Which of the following statements indicates that the client understands the instructions? A. "I need to stay active to prevent blood clots in my legs" B. "If I have a bad headache, I can take aspirin to get rid of it" C. "I should eliminate uncooked foods from my diet for now" D. 'I should eat more iron-fortified cereal to strengthen my blood"

C. "I should eliminate uncooked foods from my diet for now" RATIONALE: The client can help prevent infection by eating throughly cooked foods. Fresh fruit, vegetables, eggs, meat, and fish can harbor microorgansims that cooking destroys, so the client should avoid raw foods"

A nurse is caring for a client who is scheduled for a CABG in 2hr. Which of the following client statements indicates a need for further clarification by the nurse? A. "My arthritis is really bothering me because I haven't taken my aspirin in a week" B. "My BP shouldn't be high because I took my BP med this morning" C. "I took my warfarin last night according to my usual schedule" D. "I will check my blood sugar because I took a reduced dose of insulin this morning"

C. "I took my warfarin last night according to my usual schedule"

The nurse is caring for a client who is on strict bed rest and creates a plan of care with goals related to the prevention of deep vein thrombosis and pulmonary emboli. Which nursing action is most helpful in preventing these disorders from developing? A. Restricting fluids B. Placing a pillow under the knees C. Encouraging active range-of-motion exercises D. Applying a heating pad to the lower extremities

C. Encouraging active range-of-motion exercises

In caring for the patient with angina, the patient said, "While I was having a bowel movement, I started having the worst chest pain ever, like before I was admitted. I called for a nurse, then the pain went away." What further assessment data should the nurse obtain from the patient? A. "What precipitated the pain?" B. "Has the pain changed this time?" C. "In what areas did you feel this pain?" D. "What is your pain level on a 0 to 10 scale?"

C. "In what areas did you feel this pain?" Using PQRST, the assessment data not volunteered by the patient is the radiation of pain, the area the patient felt the pain, and if it radiated. The precipitating event was going to the bathroom and having a bowel movement. The quality of the pain was "like before I was admitted," although a more specific description may be helpful. Severity of the pain was the "worst chest pain ever," although an actual number may be needed. Timing is supplied by the patient describing when the pain occurred and that he had previously had this pain.

A nurse is providing discharge teaching for a client who has a newly inserted pacemaker. Which of the following instructions should the nurse include in the teaching? A. "Request a provider's prescription when traveling to alert airport security" B. "Stand at least 3 feet away while using a microwave" C. "Keep your cell phone 6 inches away from your pacemaker when making a call" D. "Avoid showering for the first 2 weeks following surgery"

C. "Keep your cell phone 6 inches away from your pacemaker when making a call" RATIONALE: This is to avoid interfering with the function of the generator inside the client's pacemaker

A nurse is providing discharge teaching to a client who had a sickle cell crisis. Which of the following statements indicates that the client understands the instructions? A. "I should try to drink at least 2liters of fluid per day" B. "I can still fly out to visit my sister in Colorado for a while" C. "Physical activity is good for me, but I need to avoid overexertion" D. "I can still go skiing during the cold winter months"

C. "Physical activity is good for me, but I need to avoid overexertion" RATIONALE: Not A- the client should drink 3-4L/day to help prevent recurrence of sickle cell crisis

A client who has thrombocytopenia asks the nurse why platelets are so important. Which of the following responses should the nurse make? A. "Platelets help the body fight infection" B. "Platelets help break down clots in the body" C. "Platelets plug breaks in blood vessels" D. "Platelets produce the molecules that carry oxygen"

C. "Platelets plug breaks in blood vessels"

A nurse is planning to instruct a client on how to perform pursed‑lip breathing. Which of the following statements should the nurse include? A. "Take quick breaths upon inhalation." B. "Place your hand over your stomach." C. "Take a deep breath in through your nose." D. "Puff your cheeks upon exhalation."

C. "Take a deep breath in through your nose." The client should take a deep breath in through the nose while performing pursed‑lip breathing. This controls the client's breathing.

A nurse is talking with a client who has class I heart failure and asks about obtaining a ventricular assist device (VAD). Which of the following statement should the nurse make? A. "VADs are only implanted during heart transplantation." B. "A VAD helps to pace the heart." C. "VADs are used when heart failure is not responsive to medications." D. "A VAD is useful for clients who also have a chronic lung issue."

C. "VADs are used when heart failure is not responsive to medications." One use for a VAD is to prolong life for clients who have become unresponsive to heart failure medications.

A client who just learned that he has varian (Prinzmetal's) angina asks the nurse how this type of angina compares with stable angina. Which of the following replies should the nurse make? A. "Exertion often brings on the pain" B. "Variant angina occurs randomly at various times" C. "Variant angina can cause changes on your electrocardiogram" D. "Reducing your cholesterol can help you experience less pain"

C. "Variant angina can cause changes on your electrocardiogram" RATIONALE: Variant angina causes ECG changes that reflect coronary artery spasms, which result in less o2 supplying the myocardium

A nurse is caring for a client who has a new diagnosis of tuberculosis and has been placed on a multimedication regimen. Which of the following instructions should the nurse give the client related to ethambutol? A. "Your urine can turn a dark orange." B. "Watch for a change in the sclera of your eyes." C. "Watch for any changes in vision." D. "Take vitamin B6 daily."

C. "Watch for any changes in vision." The client who is receiving ethambutol will need to watch for visual changes due to optic neuritis, which can result from taking this medication.

A nurse is teaching breathing techniques to a client who has emphysema. Which of the following statements indicates that the client understands the mechanics of pursed-lip breathing? A. "I'll inhale slowly through pursed lips to help me breathe better." B. "When I do my pursed-lip breathing, I'll lie down first." C. "When I breathe out through pursed lips, my airways don't collapse between breaths." D. "I'll relax my stomach muscles when I am doing my pursed-lip breathing exercises."

C. "When I breathe out through pursed lips, my airways don't collapse between breaths." Breathing through pursed lips slows exhalation and maintains inflation of the distal airways, which enhances respiration for clients who have emphysema. The client should use this technique during physical activity and episodes of dyspnea.

A nurse is providing preoperative teaching for a client who requests autologous donation in preparation for a scheduled orthopedic surgical procedure. Which of the following statements should the nurse include in the teaching? A. "You should make an appointment to donate blood 8 weeks prior to the surgery." B. "If you need an autologous transfusion, the blood your brother donates can be used." C. "You can donate blood each week if your hemoglobin is stable." D. "Any unused blood that is donated can be used for other clients."

C. "You can donate blood each week if your hemoglobin is stable." Beginning 6 weeks prior to surgery, the client can donate blood each week for autologous transfusion if their Hgb and Hct remain stable.

A nurse is preparing to administer a new prescription for isoniazid (INH) to a light-skinned client who has tuberculosis. The nurse should instruct the client to report which of the following findings as an adverse effect of the medication? A. "You might notice yellowing of your skin." B. "You might experience pain in your joints." C. "You might notice tingling of your hands." D. "You might experience a loss of appetite."

C. "You might notice tingling of your hands." Tingling of the hands can be an adverse effect of isoniazid.

A nurse in a clinic is providing teaching for a client who is scheduled to have a tuberculin skin test. Which of the following pieces of information should the nurse include? A. "If the test is positive, it means you have an active case of tuberculosis." B. "If the test is positive, you should have another tuberculin skin test in 3 weeks." C. "You must return to the clinic to have the test read in 2 or 3 days." D. "A nurse will use a small lancet to scratch the skin of your forearm before applying the tuberculin substance."

C. "You must return to the clinic to have the test read in 2 or 3 days." The client should have the skin test read in 2 to 3 days. An area of induration after 48 to 72 hours indicates exposure to the tubercle bacillus. If the client does not return to have the test read within 72 hours, another tuberculin skin test is necessary.

A nurse is preparing a client for a bone-marrow biopsy. Which of the following pieces of information should the nurse include in preoperative teaching? A. "You'll receive heavy sedation, so you might even sleep during the procedure" B. "You'll have to lie on your back throughout the procedure" C. "You'll feel a painful pulling sensation when the doctor withdraws the marrow" D. "Expect the procedure to take about an hour"

C. "You'll feel a painful pulling sensation when the doctor withdraws the marrow"

The nurse is assigned to care for several patients on a medical unit. Which patient should the nurse check on first? A. A 60-yr-old patient with a blood pressure of 92/64 mm Hg and hemoglobin of 9.8 g/dL B. A 50-yr-old patient with a respiratory rate of 26 breaths/minute and an elevated D-dimer C. A 40-yr-old patient with a temperature of 100.8F (38.2C) and a neutrophil count of 256/μL D. A 30-yr-old patient with a pulse of 112 beats/min and a white blood cell count of 14,000/μL

C. A 40-yr-old patient with a temperature of 100.8F (38.2C) and a neutrophil count of 256/μL A low-grade fever greater than 100.4°F (38°C) in a patient with a neutrophil count below 500/μL is a medical emergency and may indicate an infection. An infection in a neutropenic patient could lead to septic shock and possible death if not treated immediately.

A nurse is providing teaching to a client who has a chronic cough and is scheduled for a bronchoscopy. Which of the following statements indicates an understanding of the teaching? A. I can keep my dentures in during the procedure B. I am allowed only clear liquids prior to the procedure C. A tissue sample might be obtained during the procedure D. A signed consent form is not required for this procedure

C. A tissue sample might be obtained during the procedure

A nurse is assessing the hematologic system of an older adult client. The nurse should report which of the following findings to the provider as a possible indication of a hematologic disorder? A. Pallor B. Jaundice C. Absence of hair on the legs D. Poor nail bed capillary refill

C. Absence of hair on the legs RATIONALE: PAD (arterial insufficiency)

A nurse is reviewing prescriptions for a client who has acute dyspnea and diaphoresis. The client states that she is anxious because she feels that she cannot get enough air. Vital signs are: HR 117/min, RR 38/min, temp 38.4 (101.2), BP 100/54. Which of the following actions is the priority action at this time? A. Notify the provider B. Administer heparin via IV infusion C. Administer oxygen therapy D. Obtain a spiral CT scan

C. Administer oxygen therapy When using the airway, breathing, circulation (ABC) priority approach to care, determine that the priority finding is related to the respiratory status. Meeting oxygenation needs by administering oxygen therapy is priority action.

A 56 yo client comes to the triage area with left-sided chest pain, diaphoresis, and dizziness. What is the priority action? A. Initiate continuous ECG monitoring B. Notify the ED physician C. Administer oxygen via nasal cannula D. Establish IV access

C. Administer oxygen via nasal cannula

The nurse is caring for a patient who is to receive a transfusion of two units of packed red blood cells. After obtaining the first unit from the blood bank, the nurse would ask which health team member in the nurses' station to assist in checking the unit before administration? A. Unit secretary B. A physician's assistant C. Another registered nurse D. An unlicensed assistive personnel

C. Another registered nurse Before hanging a transfusion, the registered nurse must check the unit with another RN or with a licensed practical (vocational) nurse, depending on agency policy. The unit secretary, physician's assistant, or unlicensed assistive personnel should not be asked.

A nurse is assessing a client who had coronary artery bypass grafts for cardiac tamponade. Which of the following actions should the nurse take? A. Check for hypertension B. Auscultate for loud, bounding heart sounds C. Auscultate blood pressure for pulsus paradoxus D. Check for a pulse deficit

C. Auscultate blood pressure for pulsus paradoxus RATIONALE: A client who has cardiac tamponade will have pulsus paradoxus, when the blood pressure is at least 10mmHg higher on expiration than on inspiration; this occurs because of sudden decrease in CO from the fluid compressing the atria and ventricles

A nurse is caring for a client who reports calf pain. What is the first action the nurse should take? A. Notify the provider B. Elevate the affected extremity C. Check the affected extremity for warmth and redness D. Prepare to administer unfractionated heparin

C. Check the affected extremity for warmth and redness

A nurse is caring for a client who had an onset of chest pain 24hr ago. The nurse should identify that an increase in which of the following values is diagnostic of a MI? A. Myoglobin B. C-reactive protein C. Creatine kinase-MB D. Homocysteine

C. Creatine kinase-MB RATIONALE: This isoenzyme is specific to the myocardium and if elevated, indicates myocardial injury

A client is admitted to the ED following a MVA. The nurse notes a crackling sensation upon palpation of the right side of the client's chest. After notifying the provider, the nurse should document this finding as which of the following? A. Friction rub B. Crackles C. Crepitus D. Tactile fremitus

C. Crepitus RATIONALE: crepitus, aka subcutaneous emphysema, is a coarse crackling sensation that the nurse can feel while palpating the skin surface over the clients chest; crepitus indicates an air leak into the subcutaneous tissue, a manifestation of pneumothorax A- friction rub is scratching or squeaking B- aka rales, are wet, popping sounds heard when auscultating the client's lungs when fluid is present in the airways/alveoli D- tactile fremitus is a vibration of the chest wall that the nurse can feel when palpating the client's chest as the client repeats a syllable such as "nine, nine". Increased tactile fremitus is a clinical manifestation of pneumonia

A nurse is caring for a client for whom the respiratory therapist has just removed the endotracheal tube. Which of the following actions should the nurse take first? A. Instruct the client to cough B. Administer oxygen via face mask C. Evaluate the client for stridor D. Keep the client in a semi- to high-Fowler's position

C. Evaluate the client for stridor The first action the nurse should take using the nursing process is to assess the client. After extubation, the nurse should continuously evaluate the client's respiratory status. Stridor is a high-pitched sound during inspiration that indicates laryngospasm or swelling around the glottis. Stridor reflects a narrowed airway and might require emergency reintubation.

The nurse counsels a client diagnosed with iron deficiency anemia. The nurse determines teaching is effective if the client selects which menu? A. Broiled fish, green vegetables, milk B. Friend Chicken, yellow vegetables, fruit juice C. Flank steak, green leafy vegetables, prunes D. Grilled cheese sandwich, creamed soup, tomato salad

C. Flank steak, green leafy vegetables, prunes Also, liver & onions, spinach.

A nurse is caring for a client who has a peripherally inserted central catheter (PICC) in place. Which of the following actions should the nurse take when handling this central venous access device? SATA A. Use a 5mL syringe to flush the line B. Cleanse the insertion site with half-strength hydrogen peroxide C. Flush the line with sterile 0.9% sodium chloride before and after medication administration D. Access the PICC for blood sampling E. Perform a heparin flush of the line at least daily when not in use

C. Flush the line with sterile 0.9% sodium chloride before and after medication administration D. Access the PICC for blood sampling E. Perform a heparin flush of the line at least daily when not in use RATIONALE: NOT A because a 5mL syringe generates too much pressure and could rupture the line; the nurse should use a 10mL syringe instead; NOT B because the nurse should use chlorhexidine for cleansing the insertion site

A nurse is teaching a client who has a new prescription for ferrous sulfate. Which of the following information should the nurse include in the teaching? A. Stools will be dark red. B. Take with a glass of milk if gastrointestinal distress occurs. C. Foods high in vitamin C will promote absorption. D. Take for 14 days.

C. Foods high in vitamin C will promote absorption. Vitamin C enhances the absorption of iron by the intestinal tract.

If a patient has chest pain in the ER, which one should the nurse arrange to be done first? A. Give oxygen B. Give morphine C. Get a ECG D. Elevate the HOB

C. Get a ECG

A nurse is teaching a client about dietary modifications to control blood pressure. Which of the following food choices should the nurse identify as an indication that the client understands the instructions? A. Onion soup and salad B. Vegetarian wrap with potato chips C. Grilled chicken salad with fresh tomatoes D. Chicken bouillon and crackers

C. Grilled chicken salad with fresh tomatoes

A nurse is monitoring a client who began receiving a unit of packed RBCs 10 min ago. Which of the following findings should the nurse identify as an indication of a febrile transfusion reaction? (Select all that apply.) A. Temperature change from 37° C (98.6° F) pretransfusion to 37.2° C (99.0° F) B. Current blood pressure 178/90 mm Hg C. Heart rate change from 88/min pretransfusion to 120/min D. Client report of itching E. Client appears flushed

C. Heart rate change from 88/min pretransfusion to 120/min E. Client appears flushed Tachycardia is an indication of a febrile. A flushed appearance of the client can indicate a febrile transfusion reaction.

Which can contribute to a decrease in cardiac output? A. Heart rate of 100 B. Exercise C. Heart rate of 54 D. High cholesterol

C. Heart rate of 54

In addition to altered red blood cells (RBCs), which laboratory finding does the nurse expect for the patient with sickle cell disease? A. Leukocytosis B. Hypouricemia C. Hyperbilirubinemia D. Hypercholesteremia

C. Hyperbilirubinemia Rationale: As a result of accelerated RBC breakdown, the patient may have characteristic clinical findings of hemolysis, including jaundice and elevated serum bilirubin levels.

A nurse is providing teaching to a client who has anemia and a new prescription for epoetin alfa. Which of the following pieces of information should the nurse include in the teaching? A. Hospitalization is required when administering each treatment B. The maximum effect of the medication will occur in 6 months C. Hypertension is a common adverse effect of this medication D. Blood transfusions are needed with each treatment

C. Hypertension is a common adverse effect of this medication

A nurse is providing discharge teaching to a client who had a pulmonary embolism. Which of the following statements indicates that the client understands the information? A. I'll expect a little leg swelling since I won't be active for a while B. I'll see the doctor every week to change my vena cava filter C. I'll call the doctor if I see any blood in my urine or stool D. I'll have to take the blood thinner for a few more days

C. I'll call the doctor if I see any blood in my urine or stool RATIONALE: B- the vena cava filter remains in place either until the provider determines there is not a high risk for clot formation or permanently D- weeks to years

A nurse is caring for a client who has a 20-year history of COPD and is receiving oxygen at 2L/min via nasal cannula. The client is dyspneic and has an oxygen saturation via pulse oximetry of 85%. Which of the following actions should the nurse take? A. Place a nonrebreather mask on the client and increase the oxygen flow to 3L/min B. Prepare the client for possible endotracheal intubation and mechanical ventilation C. Increase the oxygen flow and request an ABG determination D. Position the client supine and administer an anti anxiety medication

C. Increase the oxygen flow and request an ABG determination

A nurse is caring for a client who has a 20-year history of COPD and is receiving oxygen at 2 L/min via nasal cannula. The client is dyspneic and has an oxygen saturation via pulse oximetry of 85%. Which of the following actions should the nurse take? A. Place a nonrebreather mask on the client and increase the oxygen flow to 3 L/min B. Prepare the client for possible endotracheal intubation and mechanical ventilation C. Increase the oxygen flow and request an arterial blood gas determination D. Position the client supine and administer an antianxiety medication

C. Increase the oxygen flow and request an arterial blood gas determination The client requires oxygen therapy at a rate that will keep the oxygen saturation between 88% and 92%. The nurse should increase the client's oxygen flow and evaluate its effectiveness with ABG results and oxygen saturation via pulse oximetry measurements.

A nurse is admitting a client who has a leg ulcer and a history of DM. The nurse should use which of the following focused assessments to help differentiate between an arterial ulcer and a venous stasis ulcer? A. Explore the clients family history of peripheral vascular disease B. Note the presence or absence of pain at the ulcer site C. Inquire about the presence or absence of claudication D. Ask if the client has had a recent infection

C. Inquire about the presence or absence of claudication Rationale: Knowing if the client is experiencing claudication helps differentiate venous from arterial ulcers. Clients who have arterial ulcers experience claudication, but those who have venous ulcers do not.

A nurse is assessing a client who has pharyngitis. Which of the following findings is the nurse's priority to report to the provider? A. Elevated temperature B. Swollen cervical lymph nodes C. Inspiratory stridor D. Purulent nasal discharge

C. Inspiratory stridor

A nurse is assessing a client who has pharyngitis. Which of the following findings is the nurse's priority to report to the provider? A. Elevated temperature B. Swollen cervical lymph nodes C. Inspiratory stridor D. Purulent nasal discharge

C. Inspiratory stridor When using the airway, breathing, and circulation (ABC) approach to client care, the nurse should determine that the priority finding is inspiratory stridor, which is a manifestation of airway obstruction. The nurse should notify the rapid response team and administer humidified oxygen.

A nurse is caring for a client who is scheduled to have his chest tube removed. Which of the following actions should the nurse take? A. Cover the insertion site with a hydrocolloid dressing after removal B. Provide pain medication immediately after removal C. Instruct the client to perform the Valsalva maneuver during removal D. Delegate removal of the chest tube to a licensed practical nurse (LPN)

C. Instruct the client to perform the Valsalva maneuver during removal RATIONALE: This is to maintain the appropriate amount of negative pressure in the chest in order to prevent air entry into the pleural space

You expect which laboratory finding to be abnormal for a patient with hemochromatosis? A. RBCs B. Platelets C. Iron D. Folic acid

C. Iron Rationale: The normal range for total body iron is 2 to 6 g. Individuals with hemochromatosis accumulate iron at a rate of 0.5 to 1.0 g each year and may exceed total iron concentrations of 50 g. The other values are near normal.

What is the priority to decrease sodium level? A. Balance B. Hydration C. LOC D. Elimination

C. LOC THINK Neuro

A nurse is caring for a client who had a left lower lobectomy to treat lung cancer. Which of the following factors will have a significant impact on the plan of care for this client? A. The client will need intensive smoking-cessation education B. After surgery, the prognosis for the client's with lung cancer is usually good C. Lung cancer usually has metastasized before the client presents with symptoms D. Oxygen therapy is ineffective following a lobectomy

C. Lung cancer usually has metastasized before the client presents with symptoms

A pt has a Phosphorous level of 1.8. What condition would cause this result? A. Tachycardia B. Too much milk C. Malnutrition D. Low calcium

C. Malnutrition

A nurse is reviewing the medical record of a client who is to undergo a scheduled electrical cardioversion. For which of the following findings should the nurse notify the provider? (Review the data below for additional client information.) MAR Ferrous sulfate 200 mg PO 0800 and 2000 Diazepam 2 mg PO 0800 and 2000 Isosorbide 2.5 mg PO 4 times a day AC and HS VITAL SIGNS 0800 T 99° F (37.2° C) Blood pressure 142/86 mm Hg Heart rate 88/min and irregular Respirations 20/min HISTORY AND PHYSICAL Bariatric surgery 10 years ago Dyspnea with exertion for 3 years Atrial fibrillation began 3 years ago Client reports taking the following medications for the past 6 weeks: iron supplement, multivitamin, antilipemic, and nitroglycerin A. Respiratory history B. Vital signs C. Medication history D. Medications to be administered

C. Medication history Because the client has a history of atrial fibrillation, it is recommended the client take anticoagulant therapy for 4 to 6 weeks prior to cardioversion to prevent clot dislodgement. The nurse should contact the provider regarding this safety concern.

A nurse is caring for a client who is having a possible myocardial infarction (MI). Which of the following findings should the nurse identify as an associated manifestation of an MI? A. Headache B. Hemoptysis C. Nausea D. Diarrhea

C. Nausea RATIONALE: Nausea is a manifestation of MI. Manifestations of MI include chest pain in the jaw, shoulder, or abdomen, anxiety, dizziness, dyspnea, dysrhythmias, fatigue and palpitations

A nurse in a provider's office is assessing a client who states he was recently exposed to tuberculosis. Which of the following findings is a clinical manifestation of pulmonary tuberculosis? A. Pericardial friction rub B. Weight gain C. Night sweats D. Cyanosis of the fingertips

C. Night sweats

A nurse is assessing a client who has a positive tuberculin skin test. Which of the following findings indicates that the client has active TB? A. Rhinitis B. Air hunger C. Night sweats D. Weight gain

C. Night sweats

A nurse is assessing a client who has a positive tuberculin skin test. Which of the following findings indicates that the client has active tuberculosis? A. Rhinitis B. Air hunger C. Night sweats D. Weight gain

C. Night sweats Manifestations of active tuberculosis include a fever, coughing, night sweats, anorexia, and fatigue.

A nurse in a provider's office is assessing a client who states he was recently exposed to tuberculosis. Which of the following findings is a clinical manifestation of pulmonary tuberculosis? A. Pericardial friction rub B. Weight gain C. Night sweats D. Cyanosis of the fingertips

C. Night sweats Night sweats and fevers are clinical manifestations of tuberculosis.

When providing teaching for the patient with iron-deficiency anemia who has been prescribed iron supplements, you should include taking the iron with which beverage? A. Milk B. Ginger ale C. Orange juice D. Water

C. Orange juice Rationale: Taking iron with vitamin C (ascorbic acid) or orange juice, which contains ascorbic acid, also enhances iron absorption. Milk may interfere with iron absorption. Ginger ale and water do not facilitate iron absorption.

A nurse is preparing to assist a provider with an arterial blood withdrawal from a client's radial artery for ABG measurement. Which of the following actions should the nurse plan to take? A. Hyperventilate the client with 100% oxygen prior to obtaining the specimen B. Apply ice to the site after obtaining the specimen C. Perform an Allen's test prior to obtaining the specimen D. Release the pressure applied to the puncture site 1min after the needle is withdrawn

C. Perform an Allen's test prior to obtaining the specimen RATIONALE: The Allen test is a first-line standard test used to assess the arterial blood supply of the hand; this is used to ensure the circulation to the hand is adequate from the radial artery; the radial artery is the most common site for withdrawal of ABGs

A nurse is preparing to assist a provider with an arterial blood withdrawal from a client's radial artery for ABG measurement. Which of the following actions should the nurse plan to take? A. Hyperventilate the client with 100% oxygen prior to obtaining the specimen B. Apply ice to the site after obtaining the specimen C. Perform an Allen's test prior to obtaining the specimen D. Release the pressure applied to the puncture site 1 min after the needle is withdrawn

C. Perform an Allen's test prior to obtaining the specimen The nurse should ensure that circulation to the hand is adequate from the ulnar artery in case the radial artery is injured from the blood draw. The most common site for withdrawal of arterial blood gases is the radial artery.

A pt has been ordered bacterial antibiotics for 12 weeks, which venous access device is appropriate? A. Centrally inserted cath B. Implanted infusion port C. Peripheral inserted central catheter D. Midline catheters

C. Peripheral inserted central catheter PICC line (up to 6 months)

The nurse is caring for a hospitalized patient with heart failure who is receiving captopril and spironolactone. Which laboratory value will be most important to monitor? A. Sodium level B. Blood glucose level C. Potassium level D. Alkaline phosphatase level

C. Potassium level

A nurse is assessing a client who has an abdominal aortic aneurysm (AAA). Which of the following findings indicates that the AAA is expanding? A. Increased BP and decreased pulse rate B. Jugular vein distention and peripheral edema C. Report of sudden, severe back pain D. Report of retrosternal chest pain radiating to the left arm

C. Report of sudden, severe back pain

A nurse is caring for a client who is extremely anxious and is hyperventilating. The client's ABG results are pH 7.50, PaCO2 27, and HCO3 25. The nurse should identify that the client has which of the following acid base imbalances? A. Respiratory acidosis B. Metabolic acidosis C. Respiratory alkalosis D. Metabolic alkalosis

C. Respiratory alkalosis

A nurse is caring for a client who is extremely anxious and is hyperventilating. The client's ABG results are pH 7.50, PaCO2 27 mmHg, and HCO3- 25 mEq/L. The nurse should identify that the client has which of the following acid-base imbalances? A. Respiratory acidosis B. Metabolic acidosis C. Respiratory alkalosis D. Metabolic alkalosis

C. Respiratory alkalosis Because of rapid breathing, the client is exhaling excessive amounts of carbon dioxide. This loss of carbon dioxide decreases the hydrogen ion level of the blood, which causes the pH to increase and results in respiratory alkalosis.

The nurse is caring for a patient with a tracheostomy. What is the priority nursing assessment for this patient? A. Electrolyte levels and daily weights B. Assessment of speech and swallowing C. Respiratory rate and oxygen saturation D. Pain assessment and assessment of mobility

C. Respiratory rate and oxygen saturation The priority assessment in the care of a patient with a tracheostomy focuses on airway and breathing. These assessments supersede the nurse's assessments that may also be necessary, such as nutritional status, speech, pain, and swallowing ability.

A nurse is presenting a community education program on recommended lifestyle changes to prevent angina and myocardial infarction. Which of the following changes should the nurse recommend be made first? A. Diet modification B. Relaxation exercises C. Smoking cessation D. Taking omega‑3 capsule

C. Smoking cessation According to the airway, breathing, and circulation (ABC) priority‑setting framework, the first step is to recommend the clients to stop smoking. Nicotine causes vasoconstriction, elevates blood pressure, and narrows coronary arteries.

A nurse is preparing an in-service presentation about assessing clients who are having an acute MI. What is the most common assessment finding with acute MI? A. Dyspnea B. Pain in the shoulder and left arm C. Substernal chest pain D. Palpitations

C. Substernal chest pain

A nurse is assessing a client who is receiving a transfusion of packed RBCs. Which of the following findings should the nurse identify as an acute intravascular hemolytic reaction? A. Severe hypertension B. Low body temperature C. Sudden oliguria D. Decreased respirations

C. Sudden oliguria RATIONALE: This type of transfusion reaction causes acute kidney injury resulting in sudden oliguria and hemoglobinuria. This reaction results from the client's antibodies reacting to the transfused RBCs

A nurse is assessing the respiratory status of a client who has COPD. Which of the following manifestations should the nurse identify as an indication of impending respiratory failure? A. Wheezing B. Bradypnea C. Tachycardia D. Diaphoresis

C. Tachycardia RATIONALE: tachycardia, dyspnea, restlessness, HA and increased BP are indications of impending respiratory failure

A nurse is assessing the respiratory status of a client who has COPD. Which of the following manifestations should the nurse identify as an indication of impending respiratory failure? A. Wheezing B. Bradypnea C. Tachycardia D. Diaphoresis

C. Tachycardia Tachycardia, dyspnea, restlessness, headaches, and increased blood pressure are indications of impending respiratory failure.

A nurse is teaching breathing techniques to a client who has emphysema. Which of the following statements indicates that the client understands the mechanics of pursed-lip breathing? A. I'll inhale slowly through pursed lips to help me breathe better B. When I do my pursed-lip breathing, I'll lie down first C. When I breathe out through pursed lips, my airways don't collapse between breaths D. I'll relax my stomach muscles when I am doing my pursed-lip breathing exercises

C. When I breathe out through pursed lips, my airways don't collapse between breaths

The nurse is caring for a patient with microcytic, hypochromic anemia. What teaching should the nurse provide about medication therapy? A. Take enteric-coated iron with each meal. B. Take cobalamin with green leafy vegetables. C. Take the iron with orange juice 1 hour before meals. D. Decrease the intake of the antiseizure medications to improve.

C. Take the iron with orange juice 1 hour before meals. With microcytic, hypochromic anemia may be caused by iron, vitamin B6, or copper deficiency; thalassemia; or lead poisoning. The iron prescribed should be taken with orange juice 1 hour before meals as it is best absorbed in an acid environment. Megaloblastic anemias occur with cobalamin (vitamin B12) and folic acid deficiencies. Vitamin B12 may help red blood cell (RBC) maturation if the patient has the intrinsic factor in the aplastic anemia or folic acid deficiency, but the patient should not stop taking the medications. The health care provider will prescribe changes in medications.

A nurse is assessing a client for Chvostek's sign. Which of the following techniques should the nurse use to perform this test? A. Apply a blood pressure cuff to the client's arm. B. Place the stethoscope bell over the client's carotid artery. C. Tap lightly on the client's cheek. D. Ask the client to lower their chin to their chest.

C. Tap lightly on the client's cheek. Tap the client's cheek over the facial nerve just below and anterior to the ear to elicit Chvostek's sign. A positive response is indicated when the client exhibits facial twitching on this side of the face.

A nurse is planning care for a client following placement of a chest tube 1hr ago. Which of the following actions should the nurse include in the plan of care? A. Clamp the chest tube if there is continuous bubbling in the water seal chamber B. Keep the chest tube drainage system at the level of the right atrium C. Tape all connections between the chest tube and drainage system D. Empty the collection chamber and record the amount of drainage every 8hr

C. Tape all connections between the chest tube and drainage system RATIONALE: this is to ensure airtight seal to prevent the chest tubing from accidentally disconnecting A- bubbling in the water seal chamber on forced exhalation/coughing means the system is working properly; DO NOT clamp the chest tube B- below level of chest at all times D- not until it is almost full

A nurse is planning care for a client following placement of a chest tube 1 hr ago. Which of the following actions should the nurse include in the plan of care? A. Clamp the chest tube if there is continuous bubbling in the water seal chamber B. Keep the chest tube drainage system at the level of the right atrium C. Tape all connections between the chest tube and drainage system D. Empty the collection chamber and record the amount of drainage every 8 hr

C. Tape all connections between the chest tube and drainage system The nurse should tape all connections to ensure that the system is airtight and prevent the chest tubing from accidentally disconnecting.

A newly licensed nurse is observing a cardioversion procedure and hears the team leader call out, "Stand clear." This statement indicates which of the following events is occurring? A. The cardioverter is being charged to the appropriate setting. B. The team should initiate CPR due to pulseless electrical activity. C. Team members cannot be in contact with equipment connected to the client. D. A time‑out is being called to verify correct protocols.

C. Team members cannot be in contact with equipment connected to the client. A safety concern for personnel performing cardioversion is to "stand clear" of the client and equipment connected to the client when a shock is delivered to prevent them from also receiving a shock.

A nurse is caring for a client who is postoperative following a thoracic lobectomy. The client has 2 chest tubes in place: 1 in the lower portion of the thorax and the other higher on the chest wall. When a family member asks the client why the client has 2 chest tubes, which of the following responses should the nurse make? A. Two tubes were necessary due to excessive bleeding from the area of surgery B. The tubes drain blood from 2 different lung areas C. The lower tube will drain blood, and the higher tube will remove air D. The second tube will take over if blood clots block the first tube

C. The lower tube will drain blood, and the higher tube will remove air

A patient is being treated for non-Hodgkin's lymphoma (NHL). What should the nurse first teach the patient about the treatment? A. Skin care that will be needed B. Method of obtaining the treatment C. Treatment type and expected side effects D. Gastrointestinal tract effects of treatment

C. Treatment type and expected side effects The patient should first be taught about the type of treatment and the expected and potential side effects. Nursing care is related to the area affected by the disease and treatment. Skin care will be affected if radiation is used. Not all patients will have gastrointestinal tract effects of NHL or treatment. The method of obtaining treatment will be included in the teaching about the type of treatment.

A nurse on a cardiac unit is reviewing the laboratory findings of a client who has a diagnosis of myocardial infarction (MI) and reports that his dyspnea began 2 weeks ago. Which of the following cardiac enzymes would confirm the MI occurred 14 days ago? A. CK‑MB B. Troponin I C. Troponin T D. Myoglobin

C. Troponin T The Troponin T level will still be evident 10 to 14 days following an MI.

A nurse is caring for a client who is 1hr postoperative following an aortic aneurysm repair. Which of the following findings can indicate shock and should be reported to the provider? A. Serosanguinous drainage on dressing B. Severe pain with coughing C. UO 20mL/hr D. Increase in temp from 98.2F to 99.5F

C. UO 20mL/hr RATIONALE: temp is still within the expected reference range (D); any UO <30mL/hr is a manifestation of shock

A nurse is caring for a client in the first hour following an aortic aneurysm repair. Which of the following findings can indicate shock and should be reported to the provider? A. Serosanguinous drainage on dressing B. Severe pain with coughing C. Urine output of 20 ml/hr D. Increase in temp from 36.C (98.2F)- 37.5C (99.5F)

C. Urine output of 20 ml/hr Rationale: Urine output less than 30 mL/hr is a manifestation of shock. Urine output is decreased due to a compensatory decreased blood flow to the kidneys, hypovolemia, or graft thrombosis or rupture.

A nurse is monitoring a client who had a myocardial infarction. For which of the following complications should the nurse monitor in the first 24 hr? A. Infective endocarditis B. Pericarditis C. Ventricular dysrhythmias D. Pulmonary emboli

C. Ventricular dysrhythmias

A nurse is preparing to transfuse a unit of packed red blood cells for a client who has anemia. Which of the following actions should the nurse take first? A. Hang an IV infusion of 0.9% sodium chloride with the blood B. Compare the client's identification number on the blood C. Witness the informed consent document D. Obtain pretransfusion vital signs

C. Witness the informed consent document

A nurse in a clinic is providing teaching for a client who is scheduled to have a tuberculin skin test. Which of the following pieces of information should the nurse include? A. If the test is positive, it means you have an active case of TB B. If the test is positive, you should have another TB skin test in 3 weeks C. You must return to the clinic to have the test read in 2-3 days D. A nurse will use a small lancet to scratch the skin of your forearm before applying the tuberculin substance

C. You must return to the clinic to have the test read in 2-3 days RATIONALE: A- needs chest x-ray B- needs chest x-ray next D- 0.1mL PPD intradermally to the dorsal aspect of the client's forearm

You anticipate the onset of manifestations related to thalessemia to occur by A. 6 months of age. B. age 1 year. C. age 2 year. D. adolescence.

C. age 2 year. Rationale: The manifestations, including growth and developmental deficits, develop in childhood by 2 years of age.

A nurse is caring for a client admitted with confusion and lethargy. The client was found at home unresponsive with an empty bottle of aspirin lying next to the bed. Vital signs reveal blood pressure 104/72 mm Hg, heart rate 116/min with regular rhythm, and respiratory rate 42/min and deep. Which of the following arterial blood gas findings should the nurse expect? A. pH 7.68 PaO2 96 mm Hg PaCO2 38 mm Hg HCO3 − 28 mEq/L B. pH 7.48 PaO2 100 mm Hg PaCO2 28 mm Hg HCO3 − 23 mEq/L C. pH 6.98 PaO2 100 mm Hg PaCO2 30 mm Hg HCO3 − 18 mEq/L D. pH 7.58 PaO2 96 mm Hg PaCO2 38 mm Hg HCO3 − 29 mEq/L

C. pH 6.98 PaO2 100 mm Hg PaCO2 30 mm Hg HCO3 − 18 mEq/L An aspirin toxicity would result in arterial blood gas findings of metabolic acidosis

a nurse is caring for a client who's in acute respiratory failure and is receiving mechanical ventilation. which of the following assessments is the best method for the nurse to use to determine the effectiveness of the current treatment regimen? A.) Blood pressure B.) Capillary refill C.) Arterial blood gases D.) Hear rate

C.) Arterial blood gases rationale: When using the airway, breathing, circulation approach to client care, the nurse should place priority on evaluating arterial blood gases to determine serum oxygen saturation and acid-base balance.

A nurse is caring for a client who's receiving mechanical ventilation when the low-pressure alarm sounds. which of the following situations should the nurse recognize as a possible cause of the alarm? A.) Excess secretions B.) Kinks in the tubing C.) Artificial airway cuff leak D.) Biting on the endotracheal tube

C.) Artificial airway cuff leak rationale: An artificial airway cuff leak interferes with oxygenation and causes the low-pressure alarm to sound.

a nurse is caring for a client who has a chest tube following a lobectomy. which of the following items should the nurse keep easily accessible for the client? A.) Extra drainage system B.) Suture removal kit C.) Container of sterile water D.) Non adherent pads

C.) Container of sterile water rationale: The nurse should have a container of sterile water in a location that is easily accessible for this client. The nurse should plan to place the open end of the tubing into the sterile water if the tubing becomes disconnected to prevent a pneumothorax.

a nurse is caring for a client who's 1 hr postoperative following a thoracentesis. which of the following is the priority assessment finding? A.) Pallor B.) Insertion site pain C.) Persistent cough D.) Temperature 37.3° C (99.1° F)

C.) Persistent cough rationale: When using the airway, breathing, circulation approach to client care, the nurse should determine that the priority finding is a persistent cough because this can indicate a tension pneumothorax, which is a medical emergency.

a nurse is creating a plan of care for a client who has COPD. which of the following interventions should the nurse include? A.) Schedule respiratory treatments following meals. B.) Have the client sit up in a chair for 2-hr periods three times per day. C.) Provide a diet that is high in calories and protein. D.) Combine activities to allow for longer rest periods between activities.

C.) Provide a diet that is high in calories and protein. rationale: The nurse should provide a client who has COPD with a diet that is high in calories and protein and low in carbohydrates.

a nurse is caring for a client who has asthma and is receiving albuterol. for which of the following adverse effects should the nurse monitor the client? A.) Hyperkalemia B.) Dyspnea C.) Tachycardia D.) Candidiasis

C.) Tachycardia rationale: The nurse should monitor the client for tachycardia, which is a common adverse effect of this medication, especially if the client uses albuterol on a regular basis.

a nurse is assessing a client who has bacterial pneumonia. which of the following manifestations should the nurse expect? A.) decreased fremitus B.) SaO2 95% on room air C.) temperature 38.8° C (101.8° F) D.) bradypnea

C.) temperature 38.8° C (101.8° F) rationale: An elevated temperature is an expected finding for a client who has bacterial pneumonia.

A nurse is caring for a client who is scheduled for a coronary artery bypass graft in 2 hr. Which of the following client statements indicates a need for further clarification by the nurse? A. "My arthritis is really bothering me because I haven't taken my aspiring in a week." b. "My blood pressure shouldn't be high because I took my BP medication this morning." c. "I took my warfarin last night according to my usually schedule." d. "I will check my BP because I took a reduced dose of insulin this morning."

C: "I took my warfarin last night according to my usually schedule." Rationale: Clients who are scheduled for a CABG should not take anticoagulants, such as warfarin, for several days prior to the surgery to prevent excessive bleeding.

When evaluating a patient's knowledge regarding a low-sodium, low-fat cardiac diet, the nurse recognizes additional teaching is needed when the patient selects which food? Baked flounder Angel food cake Baked potato with margarine Canned chicken noodle soup

Canned chicken noodle soup Canned soups are very high in sodium content. Patients need to be taught to read food labels for sodium and fat content.

The nurse performs discharge teaching for a patient with an implantable cardioverter-defibrillator (ICD). Which statement by the patient indicates that further teaching is needed? >"The device may set off the metal detectors in an airport." >"My family needs to keep up to date on how to perform CPR." >"I should not stand next to antitheft devices at the exit of stores." >"I can expect redness and swelling of the incision site for a few days."

Correct Answer: "I can expect redness and swelling of the incision site for a few days." Rationale: Patients should be taught to report any signs of infection at incision site (e.g., redness, swelling, drainage) or fever to their primary care providers immediately. Teach patients to inform TSA airport security of the presence of the ICD because it may set off metal detectors. If a handheld screening wand is used, it should not be placed directly over the ICD. Teach patients to avoid standing near antitheft devices in doorways of stores and public buildings and to walk through them at a normal pace. Caregivers should learn cardiopulmonary resuscitation.

The nurse is doing discharge teaching with the patient who received an implantable cardioverter-defibrillator (ICD) in the left side. Which statement by the patient indicates that further teaching is required? >"I will call the cardiologist if my ICD fires." >"I cannot fly because it will damage the ICD." >"I cannot move my left arm until it is approved." >"I cannot drive until my cardiologist says it is okay."

Correct Answer: "I cannot fly because it will damage the ICD." Rationale: The patient statement that flying will damage the ICD indicates misunderstanding about flying. The patient should be taught to inform TSA security screening agents at the airport about the ICD because it may set off the metal detector and if a hand-held screening wand is used, it should not be placed directly over the ICD. The other options indicate the patient understands the teaching.

A patient develops third-degree heart block and reports feeling chest pressure and shortness of breath. Which instructions should the nurse provide to the patient before initiating emergency transcutaneous pacing? >"The device will convert your heart rate and rhythm back to normal." >"The device uses overdrive pacing to slow the heart to a normal rate." >"The device is inserted through a large vein and threaded into your heart." >"The device delivers a current through your skin that can be uncomfortable."

Correct Answer: "The device delivers a current through your skin that can be uncomfortable." Rationale: Before initiating transcutaneous pacing therapy, it is important to tell the patient what to expect. The nurse should explain that the muscle contractions created by the pacemaker when the current passes through the chest wall are uncomfortable. Pacing for complete heart block will not convert the heart rhythm to normal. Overdrive pacing is used for very fast heart rates. Transcutaneous pacing is delivered through pacing pads adhered to the skin.

When computing a heart rate from the electrocardiography (ECG) tracing, the nurse counts 15 of the small blocks between the R waves of a patient whose rhythm is regular. What does the nurse calculate the patient's heart rate to be? 60 beats/min 75 beats/min 100 beats/min 150 beats/min

Correct Answer: 100 beats/min Rationale: Because each small block on the ECG paper represents 0.04 seconds, 1500 of these blocks represents 1 minute. By dividing the number of small blocks (15, in this case) into 1500, the nurse can calculate the heart rate in a patient whose rhythm is regular (in this case, 100).

The nurse is monitoring the electrocardiograms of several patients on a cardiac telemetry unit. The patients are directly visible to the nurse, and all the patients are observed to be sitting up and talking with visitors. Which patient's rhythm would require the nurse to take immediateaction? >A 62-yr-old man with a fever and sinus tachycardia with a rate of 110 beats/min >A 72-yr-old woman with atrial fibrillation with 60 to 80 QRS complexes per minute >A 52-yr-old man with premature ventricular contractions (PVCs) at a rate of 12 per minute >A 42-yr-old woman with first-degree AV block and sinus bradycardia at a rate of 56 beats/min

Correct Answer: A 52-yr-old man with premature ventricular contractions (PVCs) at a rate of 12 per minute Rationale: Frequent premature ventricular contractions (PVCs) (>1 every 10 beats) may reduce the cardiac output and precipitate angina and heart failure, depending on their frequency. Because PVCs in CAD or acute myocardial infarction indicate ventricular irritability, the patient's physiologic response to PVCs must be monitored. Frequent PVCs may be treated with oxygen therapy, electrolyte replacement, or antidysrhythmic agents.

A patient reporting dizziness and shortness of breath is admitted with a dysrhythmia. Which medication, if ordered, requires the nurse to carefully monitor the patient for asystole? Digoxin Adenosine Metoprolol Atropine sulfate

Correct Answer: Adenosine Rationale: IV adenosine is the first drug of choice to convert supraventricular tachycardia to a normal sinus rhythm. Adenosine is administered IV rapidly (over 1 or 2 seconds) followed by a rapid, normal saline flush. The nurse should monitor the patient's electrocardiogram continuously because a brief period of asystole after adenosine administration is common and expected. Atropine sulfate increases heart rate, while lanoxin and metoprolol slow the heart rate.

The nurse observes no P waves on the patients monitor strip. There are fine, wavy lines between the QRS complexes. The QRS complexes measure 0.08 seconds (narrow), but they occur irregularly with a rate of 120 beats/min. What does the nurse determine the rhythm to be? Sinus tachycardia Atrial fibrillation Ventricular fibrillation Ventricular tachycardia

Correct Answer: Atrial fibrillation Rationale: Atrial fibrillation is represented on the cardiac monitor by irregular R-R intervals and small fibrillatory (f) waves. There are no normal P waves because the atria are not truly contracting, just fibrillating. Sinus tachycardia is a sinus rate above 100 beats/min with normal P waves. Ventricular fibrillation is seen on the ECG without a visible P wave; an unmeasurable heart rate, PR or QRS; and the rhythm is irregular and chaotic. Ventricular tachycardia is seen as three or more premature ventricular contractions that have distorted QRS complexes with regular or irregular rhythm, and the P wave is usually buried in the QRS complex without a measurable PR interval.

The nurse observes a flat line on the patient's monitor and the patient is unresponsive without pulse. What medications does the nurse prepare to administer? >Lidocaine or amiodarone >Digoxin and procainamide >Epinephrine or vasopressin >β-Adrenergic blockers and dopamine

Correct Answer: Epinephrine or vasopressin Rationale: Normally, the patient in asystole cannot be successfully resuscitated. However, administration of epinephrine or vasopressin may prompt the return of depolarization and ventricular contraction. Lidocaine and amiodarone are used for ventricular tachycardia or ventricular fibrillation. Digoxin and procainamide are used for ventricular rate control. β-Adrenergic blockers are used to slow heart rate, and dopamine is used to increase heart rate.

The patient has a potassium level of 2.9 mEq/L, and the nurse obtains the following measurements on the rhythm strip: Heart rate of 86 with a regular rhythm, the P wave is 0.06 seconds (sec) and normal shape, the PR interval is 0.24 seconds, and the QRS is 0.09 seconds. How should the nurse document this rhythm? First-degree AV block Second-degree AV block Premature atrial contraction (PAC) Premature ventricular contraction (PVC)

Correct Answer: First-degree AV block Rationale: In first-degree atrioventricular (AV) block, there is prolonged duration of AV conduction that lengthens the PR interval above 0.20 seconds. In type I second-degree AV block, the PR interval continues to increase in duration until a QRS complex is blocked. In type II, the PR interval may be normal or prolonged, the ventricular rhythm may be irregular, and the QRS is usually greater than 0.12 seconds. PACs cause an irregular rhythm with a different-shaped P wave than the rest of the beats, and the PR interval may be shorter or longer. PVCs cause an irregular rhythm, and the QRS complex is wide and distorted in shape.

The patient is admitted with acute coronary syndrome (ACS). The ECG shows ST-segment depression and T-wave inversion. What should the nurse know that this indicates? Myocardia injury Myocardial ischemia Myocardial infarction Normal pacemaker function.

Correct Answer: Myocardial ischemia Rationale: The ST depression and T wave inversion on the ECG of a patient diagnosed with ACS indicate myocardial ischemia from inadequate supply of blood and oxygen to the heart. Myocardial injury is identified with ST-segment elevation. Myocardial infarction is identified with ST-segment elevation and a widened and deep Q wave. A pacemaker's presence is evident on the ECG by a spike leading to depolarization and contraction.

A patient informs the nurse of experiencing syncope. Which prioitiy nursing action should the nurse anticipate in the patient's subsequent diagnostic workup? >Preparing to assist with a head-up tilt-test >Assessing the patient's knowledge of pacemakers >Administering an IV dose of a β-adrenergic blocker >Teaching the patient about antiplatelet aggregators

Correct Answer: Preparing to assist with a head-up tilt-test Rationale: In patients without structural heart disease, the head-up tilt-test is a common component of the diagnostic workup after episodes of syncope. IV β-blockers are not indicated, although an IV infusion of low-dose isoproterenol may be started in an attempt to provoke a response if the head-up tilt-test did not have a response. Addressing pacemakers is premature and inappropriate at this stage of diagnosis. Patient teaching surrounding antiplatelet aggregators is not directly relevant to the patient's syncope at this time.

The nurse observes ventricular tachycardia (VT) on the patient's monitor. What evaluation made by the nurse led to this interpretation? Unmeasurable rate and rhythm Rate 150 beats/min; inverted P wave Rate 200 beats/min; P wave not visible Rate 125 beats/min; normal QRS complex

Correct Answer: Rate 200 beats/min; P wave not visible Rationale: VT is associated with a rate of 150 to 250 beats/min; the P wave is not normally visible. Rate and rhythm are not measurable in ventricular fibrillation. P wave inversion and a normal QRS complex are not associated with VT.

Which statement best describes the electrical activity of the heart represented by measuring the PR interval on the electrocardiogram (ECG)? >The length of time it takes to depolarize the atrium. >The length of time it takes for the atria to depolarize and repolarize. >The length of time for the electrical impulse to travel from the sinoatrial (SA) node to the Purkinje fibers. >The length of time it takes for the electrical impulse to travel from the sinoatrial (SA) node to the atrioventricular (AV) node.

Correct Answer: The length of time for the electrical impulse to travel from the sinoatrial (SA) node to the Purkinje fibers. Rationale: The electrical impulse in the heart must travel from the SA node through the AV node and into the Purkinje fibers in order for synchronous atrial and ventricular contraction to occur. When measuring the PR interval (the time from the beginning of the P wave to the beginning of the QRS), the nurse is identifying the length of time it takes for the electrical impulse to travel from the SA node to the Purkinje fibers. The P wave represents the length of time it takes for the impulse to travel from the SA node through the atrium, causing depolarization of the atria (atrial contraction). Atrial repolarization occurs during ventricular depolarization and is hidden by the QRS complex. The length of time it takes for the electrical impulse to travel from the SA node to the AV node is the flat line between the end of the P wave and the beginning of the Q wave on the ECG and is not usually measured.

The nurse obtains a 6-second rhythm strip and charts the following analysis: Atrial data Rate: 70, regularVariable PR interval Independent beats Ventricular data Rate: 40, regularIsolated escape beats Additional data QRS: 0.04 secP wave and QRS complexes unrelated What is the correct interpretation of this rhythm strip? >Sinus dysrhythmiaI >Third-degree heart block >Wenckebach phenomenon >Premature ventricular contractions

Correct Answer: Third-degree heart block Rationale: Third-degree heart block represents a loss of communication between the atrium and ventricles from atrioventricular node dissociation. This is depicted on the rhythm strip as no relationship between the P waves (representing atrial contraction) and QRS complexes (representing ventricular contraction). Whereas the atria are beating totally on their own at 70 beats/min, the ventricles are pacing themselves at 40 beats/min. Sinus dysrhythmia is seen with a slower heart rate with exhalation and an increased heart rate with inhalation. In Wenckebach heart block, there is a gradual lengthening of the PR interval until an atrial impulse is nonconducted and a QRS complex is blocked or missing. Premature ventricular contractions are the early occurrence of a wide, distorted QRS complex.

The nurse determines there is artifact on the patient's telemetry monitor. Which factor should the nurse assess for that could correct this issue? Disabled automaticity Electrodes in the wrong lead Too much hair under the electrodes Stimulation of the vagus nerve fibers

Correct Answer: Too much hair under the electrodes Rationale: Artifact is caused by muscle activity, electrical interference, or insecure leads and electrodes that could be caused by excessive chest wall hair. Disabled automaticity would cause an atrial dysrhythmia. Electrodes in the wrong lead will measure electricity in a different plane of the heart and may have a different wave form than expected. Stimulation of the vagus nerve fibers causes a decrease in heart rate, not artifact.

A nurse is assessing a client who is dehydrated. Which of the following findings should the nurse expect? A. Moist skin B. Distended neck veins C. Increased urinary output D. Tachycardia

D. Tachycardia Tachycardia is an attempt to maintain blood. Moist skin is a manifestation of fluid volume excess. Distended neck veins are a manifestation of fluid volume excess. Increased urinary output is a manifestation of fluid volume excess.

Cardioversion is attempted for a patient with atrial flutter and a rapid ventricular response. After delivering 50 joules by synchronized cardioversion, the patient develops ventricular fibrillation. Which action should the nurse take immediately? >Administer 250 mL of 0.9% saline solution IV by rapid bolus. >Assess the apical pulse, blood pressure, and bilateral neck vein distention. >Turn the synchronizer switch to the "off" position and recharge the device. >Ask the patient if there is any chest pain or discomfort and administer morphine sulfate.

Correct Answer: Turn the synchronizer switch to the "off" position and recharge the device. Rationale: Ventricular fibrillation produces no effective cardiac contractions or cardiac output. If during synchronized cardioversion the patient becomes pulseless or the rhythm deteriorates to ventricular fibrillation, the nurse should turn the synchronizer switch off and initiate defibrillation. Fluids, additional assessment, or treatment of pain alone will not restore an effective heart rhythm.

The nurse prepares to defibrillate a patient. Which dysrhythmia has the nurse observed in this patient? >Ventricular fibrillation >Third-degree AV block >Uncontrolled atrial fibrillation >Ventricular tachycardia with a pulse

Correct Answer: Ventricular fibrillation Defibrillation is always indicated in the treatment of ventricular fibrillation. Drug treatments are normally used in the treatment of uncontrolled atrial fibrillation and for ventricular tachycardia with a pulse (if the patient is stable). Otherwise, synchronized cardioversion is used (if the patient has a pulse). Pacemakers are the treatment of choice for third-degree heart block.

Which client is at risk for the development of a potassium level of 5.5 mEq/L (5.5 mmol/L)? A. The client with colitis B. The client with Cushing's syndrome C. The client who has been overusing laxatives D. The client who has sustained a traumatic burn

D. The client who has sustained a traumatic burn

Which of these electrolyte imbalances is the cause of "chronic alcoholism"? (select all that apply) a. hypernatremia b. hyponatremia c. hyperkalemia d. hypokalemia e. hypercalcemia f. hypocalcemia g. hyperphosphatemia h. hypophosphatemia i. hypermagnesemia j. hypomagnesemia

d. hypokalemia f. hypocalcemia i. hypermagnesemia j. hypomagnesemia

Beyond the first year after a heart transplant, the nurse knows that what is a major cause of death? A Infection B Acute rejection C Immunosuppression D Cardiac vasculopathy

D Cardiac vasculopathy Beyond the first year after a heart transplant, malignancy (especially lymphoma) and cardiac vasculopathy (accelerated CAD) are the major causes of death. During the first year after transplant, infection and acute rejection are the major causes of death. Immunosuppressive therapy will be used for posttransplant management to prevent rejection and increases the patient's risk of an infection.

Which of these electrolyte imbalances is the cause of "excess milk of magnesia"? (select all that apply) a. hypernatremia b. hyponatremia c. hyperkalemia d. hypokalemia e. hypercalcemia f. hypocalcemia g. hyperphosphatemia h. hypophosphatemia i. hypermagnesemia j. hypomagnesemia

d. hypokalemia h. hypophosphatemia i. hypermagnesemia

Direction of fluid shift "from interstitium to vessels" and mechanism of fluid movement "osmosis" matches which event factor? a. burns b. dehydration c. fluid overload d. hyponatremia

d. hyponatremia

A nurse is performing a cardiac assessment on a client. Identify the area the nurse should inspect when evaluating the point of maximal impulse. (You will find "Hot Spots" to select in the artwork below. Select only the hotspot that corresponds to your answer.)

D Rationale: Inspection of this location allows the nurse to assess for pulsations of the apex area of the heart, which is considered the apical pulse or point of maximal impulse. The point of maximal impulse is located at the left fifth intercostal space in the midclavicular line.

After having an MI, the nurse notes the patient has jugular venous distention, gained weight, developed peripheral edema, and has a heart rate of 108/minute. What should the nurse suspect is happening? A ADHF B Chronic HF C Left-sided HF D Right-sided HF

D Right-sided HF An MI is a primary cause of heart failure. The jugular venous distention, weight gain, peripheral edema, and increased heart rate are manifestations of right-sided heart failure.

A nurse is admitting a client who is in sickle cell crisis. Besides pain management, which of the following interventions should the nurse include in the client's care? A. Flexion of the extremities B. Therapeutic hypothermia C. Upright positioning D. Ample hydration

D. Ample hydration

A patient is concerned that he may have asthma. Of the symptoms that he describes to the nurse, which ones suggest asthma or risk factors for asthma? (select all that apply) a. Allergic rhinitis b. Prolonged inhalation c. Cough, especially at night d. Gastric reflux or heartburn e. History of chronic sinusitis

a. Allergic rhinitis c. Cough, especially at night d. Gastric reflux or heartburn e. History of chronic sinusitis

It is important for the nurse to assess for which manifestation(s) in a patient who has just undergone a total thyroidectomy (select all that apply)? a. Confusion b. Weight gain c. Depressed reflexes d. Circumoral numbness e. Positive Chvostek's sign

a. Confusion d. Circumoral numbness e. Positive Chvostek's sign

A nurse is caring for a client who has a chest tube. The nurse notes that the chest tube has become disconnected from the chest drainage system. Which of the following actions should the nurse take? A. Place the drainage system at the head of the client's bed B. Increase the suction to the chest drainage system C. Place the client on low-flow oxygen via nasal cannula D. Immerse the end of the chest tube in a bottle of sterile water

D. Immerse the end of the chest tube in a bottle of sterile water

When assessing laboratory values on a patient admitted with septicemia, what does the nurse expect to find? A. Increased platelets B. Decreased red blood cells C. Decreased erythrocyte sedimentation rate (ESR) D. Increased bands in the white blood cell (WBC) differential

D When infections are severe, such as in septicemia, more granulocytes are released from the bone marrow as a compensatory mechanism. To meet the increased demand, many young, immature polymorphonuclear neutrophils (bands) are released into circulation. WBCs are usually reported in order of maturity (initially with the less mature forms on the left side of a written report). Hence, the term "shift to the left" is used to denote an increase in the number of bands. Thrombocytosis occurs with inflammation and some malignant disorders. Decreased red blood cells indicate anemia. Increased red blood cells or decreased ESR is not indicative of septicemia.

A 44-year-old man is diagnosed with hypertension and receives a prescription for benazepril (Lotensin). After the nurse teaches him about the medication, which statement by the patient indicates his correct understanding? A "If I take this medication, I will not need to follow a special diet." B "It is normal to have some swelling in my face while taking this medication." C "I will need to eat foods such as bananas and potatoes that are high in potassium." D "If I develop a dry cough while taking this medication, I should notify my doctor."

D "If I develop a dry cough while taking this medication, I should notify my doctor." Benazepril is an angiotensin-converting enzyme inhibitor. The medication inhibits breakdown of bradykinin, which may cause a dry, hacking cough. Other adverse effects include hyperkalemia. Swelling in the face could indicate angioedema and should be reported immediately to the prescriber. Patients taking drug therapy for hypertension should also attempt lifestyle modifications to lower blood pressure such as a reduced-sodium diet.

In reviewing medication instructions with a patient being discharged on antihypertensive medications, which statement would be most appropriate for the nurse to make when discussing guanethidine (Ismelin)? A "A fast heart rate is a side effect to watch for while taking guanethidine." B "Stop the drug and notify your doctor if you experience any nausea or vomiting." C "Because this drug may affect the lungs in large doses, it may also help your breathing." D "Make position changes slowly, especially when rising from lying down to a standing position."

D "Make position changes slowly, especially when rising from lying down to a standing position." Guanethidine is a peripheral-acting α-adrenergic antagonist and can cause marked orthostatic hypotension. For this reason, the patient should be instructed to rise slowly, especially when moving from a recumbent to a standing position. Support stockings may also be helpful. Tachycardia or lung effects are not evident with guanethidine.

The nurse is administering a dose of digoxin (Lanoxin) to a patient with heart failure (HF). The nurse would become concerned with the possibility of digitalis toxicity if the patient reported which symptom(s)? A Muscle aches B Constipation C Pounding headache D Anorexia and nausea

D Anorexia and nausea Anorexia, nausea, vomiting, blurred or yellow vision, and cardiac dysrhythmias are all signs of digitalis toxicity. The nurse would become concerned and notify the health care provider if the patient exhibited any of these symptoms.

A stable patient with acute decompensated heart failure (ADHF) suddenly becomes dyspneic. Before positioning the patient on the bedside, what should the nurse assess first? A Urine output B Heart rhythm C Breath sounds D Blood pressure

D Blood pressure The nurse should evaluate the blood pressure before dangling the patient on the bedside because the blood pressure can decrease as blood pools in the periphery and preload decreases. If the patient's blood pressure is low or marginal, the nurse should put the patient in the semi-Fowler's position and use other measures to improve gas exchange.

A nurse is providing preoperative teaching to a client who will undergo a total laryngectomy. Which of the following statements indicates that the client understands the impact of the surgery? A. "I'm not going to be able to cough for a while after the surgery." B. "After I recover from the anesthesia, I'll be able to eat regular food again." C. "After the surgery, my voice will gradually return but might be weak." D. "I understand that I will have a permanent tracheostomy after the surgery."

D. "I understand that I will have a permanent tracheostomy after the surgery." With a partial laryngectomy, the tracheostomy is temporary. This client will have a total laryngectomy, so the tracheostomy will be permanent.

A nurse is teaching a client who has angina about a new prescription for metoprolol. Which of the following statements by the client indicates understanding of the teaching? A. "I should place the tablet under my tongue." B. "I should have my clotting time checked weekly." C. "I will report any ringing in my ears." D. "I will call my doctor if my pulse rate is less than 60."

D. "I will call my doctor if my pulse rate is less than 60." The client is advised to notify the provider if bradycardia (pulse rate less than 60) occurs.

A nurse is instructing a client on the use of an incentive spirometer. Which of the following statements by the client indicates an understanding of the teaching? A. "I will place the adapter on my finger to read my blood oxygen saturation level." B. "I will lie on my back with my knees bent." C. "I will rest my hand over my abdomen to create resistance." D. "I will take in a deep breath and hold it before exhaling."

D. "I will take in a deep breath and hold it before exhaling." The client who is using the spirometer should take in as deep a breath as possible before exhaling. As the client inhales, the needle of the spirometer rises. This promotes lung expansion.

A nurse is providing discharge teaching to a client who is postoperative following a rhinoplasty. Which of the following instructions should the nurse include? A. "Apply warm compresses to the face." B. "Take aspirin 650 mg by mouth for mild pain." C. "Close your mouth when sneezing." D. "Lie on your back with your head elevated 30° when resting."

D. "Lie on your back with your head elevated 30° when resting." The nurse should instruct the client to rest in the semi-Fowler's position to prevent aspiration of nasal secretions.

A nurse is teaching a 70-year old client about risk factors for heart failure. The client has mild asthma, dm, and coronary artery disease. Which of the following statements by the client indicates an understanding of the teaching? A. "My diabetes will not increase my risk of heart failure" B. "My asthma makes it more likely for me to have heart failure" C. "My age does not increase my risk of heart failure" D. "My coronary artery disease is a risk factor for heart failure"

D. "My coronary artery disease is a risk factor for heart failure"

The nurse instructs an African American man who has sickle cell disease about symptom management and prevention of sickle cell crisis. The nurse determines further teaching is necessary if the patient makes which statement? A. "When I take a vacation, I should not go to the mountains." B. "I should avoid contact with anyone who has a respiratory infection." C. "I may experience severe pain during a crisis and need narcotic analgesics." D. "When my vision is blurred, I will close my eyes and rest for an hour."

D. "When my vision is blurred, I will close my eyes and rest for an hour." Blurred vision should be reported immediately and may indicate a detached retina or retinopathy. Hypoxia (at high altitudes) and infection are common causes of a sickle cell crisis. Severe pain may occur during a sickle cell crisis, and narcotic analgesics are indicated for pain management.

A nurse is caring for a client who is in hypovolemic shock. While waiting for a unit of blood, the nurse should administer which of the following IV solutions? A. 0.45% sodium chloride B. Dextrose 5% in 0.9% sodium chloride C. Dextrose 10% in water D. 0.9% sodium chloride

D. 0.9% sodium chloride RATIONALE: Solutions of 0.9% sodium chloride, as well as lactated ringers solution are used for fluid volume replacement; sodium chloride (crystalloid) is a physiologically isotonic solution that replaces lost volume in the bloodstream and is the only solution to use when infusing blood products

A donor has AB- blood. Which patient or patients below can receive this type of blood safely? A. A patient with O- blood. B. A patient with A- blood. C. A patient with B- blood. D. A patient with AB- blood.

D. A patient with AB- blood. Donors with AB type blood can only donate to others who have the AB type blood, in this case AB- blood. However, they are the universal recipients in that they can receive blood for every blood type but can only donate to their same exact blood type.

A charge nurse is observing a newly licensed nurse administering an IV mediation to a client who has an implanted venous access port. Which of the following observations requires intervention by the charge nurse? A. A dressing is not applied to the port site after use B. A 22 gauge non-coring needle is used to access the port C. Blood return is noted prior to administering the medication D. A solution of 5mL heparin 1000units/mL has been prepared

D. A solution of 5mL heparin 1000units/mL has been prepared RATIONALE: Implanted ports should be flushed after each use and at least once a month when not in use. This is performed to prevent the formation of blood clots in the catheter, which would disrupt the proper functioning of the catheter. The solution of 5mL heparin should be 100 units

A nurse is assessing a client who is 85 years old. Which of the following findings should the nurse identify as a manifestation of myocardial infarction? A. Sudden hemoptysis B. Acute diarrhea C. Frontal headache D. Acute confusion

D. Acute confusion Rationale: common in ages 65 or older; other manifestations can include nausea, vomiting, dyspnea, diaphoresis, palpitations, fatigue

A pt is experiencing chest pain, diaphoresis, dyspnea and dizziness. What is the priority action? A. Get a ECG B. Give nitroglycerin C. Give morphine D. Admin O2 via nasal cannula

D. Admin O2 via nasal cannula

The health care provider has written these orders for a patient with a diagnosis of pulmonary edema. The patient's morning assessment reveals bounding peripheral pulses, a weight gain of 2 lb, pitting ankle edema, and moist crackles bilaterally. Which order takes priority at this time? A. Weigh the patient every morning. B. Maintain accurate intake and output records. C. Restrict fluids to 1500mL/day. D. Administer furosemide 40 mg IV push.

D. Administer furosemide 40 mg IV push. Bilateral moist crackles indicate fluid-filled alveoli, which interferes with gas exchange. Furosemide is a potent loop diuretic that will help mobilize the fluid in the lungs. The other orders are important but are not urgent.

A client with a 3-day history of nausea and vomiting presents to the emergency department. The client is hypoventilating and has a respiratory rate of 10 breaths per minute. The electrocardiogram (ECG) monitor displays tachycardia, with a heart rate of 120 beats per minute. Arterial blood gases are drawn and the nurse reviews the results, expecting to note which finding? A. A decreased pH and an increased Paco₂ B. An increased pH and a decreased Paco₂ C. A decreased pH and a decreased HCO₃- D. An increased pH and an increased HCO₃-

D. An increased pH and an increased HCO₃-

"The nurse is caring for a patient who is to receive a transfusion of two units of packed red blood cells. After obtaining the first unit from the blood bank, the nurse would ask which of the following health team members in the nurses' station to assist in checking the unit before administration? A. Unit Secretary B. A Phlebotomist C. A Physician's Assistant D. Another Registered Nurse

D. Another Registered Nurse Before hanging a transfusion, the registered nurse must check the unit with ANOTHER RN or with a licensed practical (vocational) nurse, depending on agency policy. Checking blood products is not in the unit secretary's or phlebotimist's scope of practice. The physician assistant is not another RN or licensed practical nurse.

A nurse is caring for a client who has thrombocytopenia and develops epistaxis. Which of the following actions should the nurse take? A. Have the client gently blow clots from the nose every 5 minutes B. Instruct the client to sit with his head hyperextended C. Apply ice compresses to the back of the clients neck D. Apply lateral pressure to the clients nose for 10 minutes

D. Apply lateral pressure to the clients nose for 10 minutes

A nurse is preparing an automated external defibrillator (AED) for a client receiving CPR after a cardiac arrest. Which of the following actions should the nurse perform first? A. Press the analyze button on the machine B. Stop CPR and move away from the client C. Push the charge button to prepare to shock D. Apply the defibrillator pads to the client's chest

D. Apply the defibrillator pads to the client's chest RATIONALE: The pads should be applied without interrupting CPR

A nurse in the emergency department is caring for a client who had an anterior MI. The client's history reveals she is 1 week post-op open cholecystectomy. The nurse should recognize that which of the following interventions is contraindicated? A. Administering IV morphine sulfate B. Administering oxygen at 2:/min via nasal cannula C. Helping the client to the bedside commode D. Assisting with thrombolytic therapy

D. Assisting with thrombolytic therapy Rationale: The nurse should recognize that major surgery within the previous 3 weeks is a contraindication for thrombolytic therapy.

A nurse in an emergency department is caring for a client who had an anterior myocardial infarction. The client's history reveals they are 1 week postoperative following an open cholecystectomy. The nurse should identify that which of the following interventions is C/I? A. Administering IV morphine sulfate B. Administering oxygen at 2L/min via nasal cannula C. Helping the client to the bedside commode D. Assisting with thrombolytic therapy

D. Assisting with thrombolytic therapy RATIONALE: major surgery within the last 3 weeks is a C/I for thrombolytic surgery

A nurse is caring for a client 2 hr after admission. The client has an SaO2 of 91%, exhibits audible wheezes, and is using accessory muscles when breathing. Which of the following classes of medications should the nurse expect to administer? A. Antibiotic B. Beta‑blocker C. Antiviral D. Beta2 agonist

D. Beta2 agonist Administer a beta2 agonist, which causes dilation of the bronchioles to relieve wheezing and open the airways.

A nurse is assessing a client following a bronchoscopy. Which of the following findings should the nurse report to the provider? A. Blood‑tinged sputum B. Dry, nonproductive cough C. Sore throat D. Bronchospasms

D. Bronchospasms Bronchospasms can indicate the client is having difficulty maintaining a patent airway. The nurse should notify the provider immediately.

A patient admitted to the emergency department 24 hours ago with complaints of chest pain was diagnosed with a ST-segment-elevation myocardial infarction (STEMI). What complication of myocardial infarction should the nurse anticipate? a. Unstable angina b. Cardiac tamponade C. Sudden cardiac death D. Cardiac dysrhythmias

D. Cardiac dysrhythmias Dysrhythmias are present in 80% to 90% of patients after myocardial infarction (MI). Unstable angina is considered a precursor to MI rather than a complication. Cardiac tamponade is a rare event, and sudden cardiac death is defined as an unexpected death from cardiac causes. Cardiac dysfunction in the period following an MI would not be characterized as sudden cardiac death.

A nurse is completing an assessment for a client who has a history of unstable angina. Which of the following findings should the nurse expect? A. Chest pain is relieved soon after resting B. Nitroglycerin relieves chest pain C. Physical exertion does not precipitate chest pain D. Chest pain lasts longer than 15 minutes

D. Chest pain lasts longer than 15 minutes

A client comes to the ED in severe respiratory distress following left-sided blunt chest trauma. The nurse notes absent breath sounds on the client's left side and a tracheal shift to the right. For which of the following procedures should the nurse prepare the client? A. Tracheostomy placement B. Thoracentesis C. CT scan of the chest D. Chest tube insertion

D. Chest tube insertion RATIONALE: insert chest tube and connect to a water seal drainage

A male patient with a long-standing history of heart failure has recently qualified for hospice care. What measure should the nurse now prioritize when providing care for this patient? A. Taper the patient off his current medications. B. Continue education for the patient and his family. C. Pursue experimental therapies or surgical options. D. Choose interventions to promote comfort and prevent suffering.

D. Choose interventions to promote comfort and prevent suffering. The central focus of hospice care is the promotion of comfort and the prevention of suffering. Patient education should continue, but providing comfort is paramount. Medications should be continued unless they are not tolerated. Experimental therapies and surgeries are not commonly used in the care of hospice patients.

A nurse in a clinic is assessing a client who has sinusitis. Which of the following techniques should the nurse use to identify manifestations of this disorder? A. Percussion of posterior lobes of lungs B. Auscultation of the trachea C. Inspection of the conjunctiva D. Palpation of the orbital areas

D. Palpation of the orbital areas A client who has sinusitis will report tenderness when the orbital, frontal, and facial areas are palpated.

Client experiencing a vaso-occlusive crisis. The nurse determines care is appropriate if which observation is made? A. Client's room is 68 degrees B. Client's intake is limited for 48 hours C. Client understands limitations of pain control D. Client receives frequent neurological assessments

D. Client receives frequent neurological assessments reasoning: the nurse will monitor for changes in the level of consciousness, reports of headache and dizziness, development of sensory and motor deficits, and seizure activity. neurological changes may reflect diminished perfusion to the central nervous system a vaso-occlusive crisis causes ineffective tissue perfusion (decrease in the oxygen resulting in the failure to nourish the tissues at the capillary level). the nurse will maintain the room temperature 72F (22C) and body warmth without overheating. this nursing action will prevent vasoconstriction and assist in circulation and perfusion the nurse must maintain adequate fluid intake and monitor the clients urine output. dehydration causes an increase in sickling and occlusion of capillaries. it is recommended that the client receive at least 200 mL per hour orally or via the parenteral route

A nurse is preparing a client for discharge following a bronchoscopy. Which of the following assessments is the nurse's monitoring priority? A. Measuring HR B. Palpating peripheral pulses C. Observing sputum for blood D. Confirming the gag reflex

D. Confirming the gag reflex

A nurse is preparing a client for discharge following a bronchoscopy. Which of the following assessments is the nurse's monitoring priority? A. Measuring heart rate B. Palpating peripheral pulses C. Observing sputum for blood D. Confirming the gag reflex

D. Confirming the gag reflex The greatest risk to the client's safety is aspiration resulting from a depressed gag reflex. The nurse's priority is to make sure the client's gag reflex has returned before discharge so that the client can maintain hydration and nutrition without risk.

A nurse is assessing a client who has right-sided heart failure. Which of the following findings should the nurse expect? A. Decreased capillary refill B. Dyspnea C. Orthopnea D. Dependent edema

D. Dependent edema RATIONALE: The subsequent systemic venous backup leads to the development of dependent edema

A nurse is providing postoperative care for a client who has 2 chest tubes in place following a lobectomy. The client asks the nurse the reasom for having 2 chest tubes. The nurse should inform the client that the lower chest tube is placed for which of the following reasons? A. Removing air from the pleural space B. Creating access for irrigating the chest cavity C. Evacuating secretions from the bronchioles and alveoli D. Draining blood and fluid from the pleural space

D. Draining blood and fluid from the pleural space

A nurse is planning care for a client during a sickle cell crisis. Which of the following interventions should the nurse include in the client's plan of care? A. Maintain the client's hips and knees in a flexed position B. Apply cold compresses to painful joints C. Withhold opioids until the crisis is resolved D. Encourage increased fluid intake

D. Encourage increased fluid intake

A nurse is caring for a client who is postoperative following a rhinoplasty. Which of the following findings should the nurse report to the surgeon? A. Nasal edema B. Mouth breathing C. Periorbital ecchymosis D. Frequent swallowing

D. Frequent swallowing

There is a hypertensive pt in ER with severe headache and nausea and a bp of 238/118. What questions should you ask first? A. Have you vomited? B. How long does your headaches last? C. Do you have heart palpitations? D. Have you consistently taken your medication?

D. Have you consistently taken your medication?

Which condition would a client benefit from a sodium restriction? A. Bradycardia B. Abdomen distinction C. Fall Risk D. Heart failure and liver cirrhosis

D. Heart failure and liver cirrhosis

A nurse is teaching a newly licensed nurse about heparin‑induced thrombocytopenia. Which of the following risk factors for this disorder should the nurse include in the teaching? A. Warfarin therapy for atrial fibrillation B. Placental abruption C. Systemic lupus erythematosus D. Heparin therapy for deep‑vein thrombosis

D. Heparin therapy for deep‑vein thrombosis The client who is receiving heparin therapy for longer than 1 week is at increased risk for the development of HIT.

A nurse in a clinic is caring for a client who has suspected anemia. Which of the following laboratory test results should the nurse expect? A. Iron 90 mcg/dL B. RBC 6.5 million/uL C. WBC 4,800 mm³ D. Hgb 10 g/dL

D. Hgb 10 g/dL Hgb of 10 g/dL is below the expected reference range and is an expected finding of anemia.

A nurse is providing preoperative teaching to a client who will undergo a total laryngectomy. Which of the following statements indicates that the client understands the impact of the surgery? A. I'm not going to be able to cough for a while after the surgery B. After I recover from the anesthesia, I'll be able to eat regular food again C. After the surgery, my voice will gradually return but might be weak D. I understand that I will have a permanent tracheostomy after the surgery

D. I understand that I will have a permanent tracheostomy after the surgery RATIONALE: With a partial laryngectomy, the tracheostomy is temporary; this client is having a total laryngectomy, so the tracheostomy will be permanent

A nurse is providing preoperative teaching to a client who is to undergo a pneumonectomy. The client states, "I am afraid coughing will hurt after surgery". Which of the following statements by the nurse is appropriate? A. After the surgeon removes the lung, you will not need to cough B. I'll make sure you get cough suppressants to keep you from straining the incision when you cough C. Don't worry. You will have a pump that delivers pain medication as needed, so you will have very little pain D. I will show you how to splint your incision while coughing

D. I will show you how to splint your incision while coughing

A nurse is caring for a client who has a history of DVT and is receiving warfarin. Which of the following client findings provides the nurse with the best evidence regarding the effectiveness of the warfarin therapy? A. Hemoglobin 14 g/dl B. Minimal bruising of extremities C. Decreased Blood pressure D. INR 2.0

D. INR 2.0 Rationale: The nurse should identify that an INR of 2.0 is within the desired reference range of 2.0 to 3.0 for a client who has a deep-vein thrombosis and is receiving warfarin to reduce the risk of new clot formation and a stroke.

A nurse is caring for a client who has a chest tube. The nurse notes that the chest tube has become disconnected from the chest drainage system. Which of the following actions should the nurse take? A. Place the drainage system at the head of the client's bed B. Increase the suction to the chest drainage system C. Place the client on low-flow oxygen via nasal cannula D. Immerse the end of the chest tube in a bottle of sterile water

D. Immerse the end of the chest tube in a bottle of sterile water If the chest tube and drainage system have become disconnected, air can enter the pleural space, producing a pneumothorax that can result in severe respiratory distress. To prevent a pneumothorax from developing, a temporary water seal can be established by immersing the end of the chest tube in an open bottle of sterile water. This allows air to escape and not enter the pleural space. A bottle of sterile water should always be readily available at the bedside for a client who has a chest tube.

A nurse is caring for a client following a stroke. Which of the following actions should the nurse take first? A. Obtain coagulation laboratory studies from the client B. Apply pneumatic compression boots to the client C. Request a referral for a speech-language pathologist D. Keep the client NPO

D. Keep the client NPO RATIONALE: keeping the client NPO due to the risk of aspiration from the stroke

A nurse is providing discharge teaching to a client who is postoperative following a rhinoplasty. Which of the following instructions should the nurse include? A. Apply warm compresses to the face B. Take aspirin 650mg by mouth for mild pain C. Close your mouth when sneezing D. Lie on your back with the HOB elevated 30° when resting

D. Lie on your back with the HOB elevated 30° when resting

A nurse is assessing a client who is receiving a unit of whole blood. Which of the following findings should the nurse identify as a manifestation of a hemolytic reaction? A. Bradycardia B. Paresthesia C. Hypertension D. Low back pain

D. Low back pain RATIONALE: Other manifestations include HA, chest pain, tachypnea, tachycardia, and dark urine

A nurse is assessing a client who has an abdominal aortic aneurysm. Which of the following manifestations should the nurse expect? A. Midsternal chest pain B. Thrill C. Pitting edema in lower extremities D. Lower back discomfort

D. Lower back discomfort

A nurse is obtaining arterial blood gases for a client who has vomited for 24 hr. The nurse should expect which of the following acid‑base imbalances to result from vomiting for 24 hr? A. Respiratory acidosis B. Respiratory alkalosis C. Metabolic acidosis D. Metabolic alkalosis

D. Metabolic alkalosis Excessive vomiting causes a loss of gastric acids and an accumulation of bicarbonate in the blood, resulting in metabolic alkalosis.

A nurse is planning care for a client who is having a percutaneous transluminal coronary angioplasty (PTCA) with stent placement. Which of the following actions should the nurse anticipate in the post-procedure plan of care? A. Instruct the client about a long-term cardiac conditioning program B. Administer scheduled doses of acetaminophen C. Check for peak laboratory markers of myocardial damage D. Monitor for bleeding

D. Monitor for bleeding RATIONALE: Bleeding is a post-procedure complication of PTCA because of the administration of heparin during the procedure and the removal of the femoral (or brachial) sheath. Manual pressure or a closure device is used to obtain hemostasis to the site. The client should remain on bed rest until hemostasis resolves

A nurse is caring for a client in a long‑term care facility who has become weak, confused, and experienced dizziness when standing. The client's temperature is 38.3° C (100.9° F), pulse 92/min, respirations 20/min, and blood pressure 108/60 mm Hg. Which of the following actions should the nurse take? A. Initiate fluid restrictions to limit intake. B. Check for peripheral edema. C. Encourage the client to ambulate to promote oxygenation. D. Monitor for orthostatic hypotension.

D. Monitor for orthostatic hypotension. Monitor for orthostatic hypotension because they have manifestations of dehydration Offer fluids when the client has manifestations of dehydration. Monitor for poor skin turgor when the client has manifestations of fluid volume deficit. Keep the client in bed and assist them to the bathroom as needed because they are at risk for falling due to manifestations of dehydration.

A nurse is planning care for a client who is postoperative following a hip arthroplasty. In the client's medical record, the nurse notes a history of chronic obstructive pulmonary disease (COPD). Which of the following oxygen-delivery methods should the nurse plan to use for this client? A. Simple face mask B. Nonrebreather mask C. Bag-valve-mask device D. Nasal cannula

D. Nasal cannula A nasal cannula delivers precise concentrations of oxygen; therefore, it is an appropriate device for a client who has COPD and requires a precise percentage of inspired oxygen.

A nurse is planning care for a client who is postoperative following a hip arthroplasty. In the client's medical record, the nurse notes a history of COPD. Which of the following oxygen-delivery methods should the nurse plan to use for this client? A. Simple face mask B. Nonrebreather C. Bag-valve mask D. Nasal cannula

D. Nasal cannula RATIONALE: provides precise concentrations of oxygen

A nurse is auscultating the lungs of a client who is having an acute asthma attack. Which of the following sounds should the nurse expect to hear? A. Soft blowing B. Loud bubbling C. Dry grating D. Noisy wheezing

D. Noisy wheezing

A nurse is auscultating the lungs of a client who is having an acute asthma attack. Which of the following sounds should the nurse expect to hear? A. Soft blowing B. Loud bubbling C. Dry grating D. Noisy wheezing

D. Noisy wheezing Asthma causes the bronchioles of the lungs to constrict, creating a wheezing sound.

A nurse preceptor is observing a newly licensed nurse on the unit who is preparing to administer a blood transfusion to an older adult client. Which of the following actions by the newly licensed nurse indicates an understanding of the procedure? A. Inserts an 18‑gauge IV catheter in the client B. Verifies blood compatibility and expiration date of the blood with an assistive personnel (AP) C. Administers dextrose 5% in 0.9% sodium chloride IV with the transfusion D. Obtains vital signs every 15 min throughout the procedure.

D. Obtains vital signs every 15 min throughout the procedure. Check the older adult client's vital signs every 15 min throughout the transfusion to allow for early detection of fluid overload or other transfusion reaction.

A nurse is caring for a client who has chronic obstructive pulmonary disease (COPD) and is experiencing shortness of breath. Which of the following actions should the nurse perform first? A. Monitor the client's arterial blood gas results B. Instruct the client to perform controlled coughing C. Teach the client how to use pursed-lip breathing D. Place the client in an upright position

D. Place the client in an upright position Using the airway, breathing, and circulation (ABC) approach to client care, the nurse should place the client in an upright position to facilitate chest expansion and proper diaphragmatic contraction. Positioning the client upright will also assist with mobilizing secretions that might be impeding airflow.

A nurse is preparing a client for coronary angiography. Which of the following findings should the nurse report to the provider prior to the procedure? A. Hemoglobin 14.4 B. History of PAD C. UO 200mL/4hr D. Previous allergic reaction to shellfish

D. Previous allergic reaction to shellfish

A nurse is preparing a client for coronary angiography. The nurse should report which of the following findings to the provider prior to the procedure? A. Hemoglobin 14.4 g/dl b. History of peripheral arterial disease. c. Urine output 200 ml/4 hr. D. Previous allergic reaction to shellfish

D. Previous allergic reaction to shellfish Rationale: The contrast medium used for coronary angiography is iodine-based. Clients who have a history of allergic reaction to shellfish often react to iodine and might need a steroid or antihistamine prior to the procedure.

The nurse instructs a client with chronic obstructive pulmonary disease (COPD) to use the pursed-lip method of breathing and evaluates the teaching by asking the client about the purpose of this type of breathing. The nurse determines that the client understands if the client states that the primary purpose of pursed-lip breathing is to promote which outcome? A. Promote oxygen intake B. Strengthen the diaphragm C. Strengthen the intercostal muscles D. Promote carbon dioxide elimination

D. Promote carbon dioxide elimination

A patient is admitted for joint replacement surgery and has a permanent tracheostomy. Which task is appropriate for the nurse to delegate to unlicensed assistive personnel (UAP)? A. Suction the tracheostomy. B. Check stoma site for skin breakdown. C. Complete tracheostomy care using sterile technique. D. Provide oral care with a toothbrush and tonsil suction tube.

D. Provide oral care with a toothbrush and tonsil suction tube. Oral care (for a stable patient with a tracheostomy) can be delegated to UAP. A registered nurse would be responsible for assessments (e.g., checking the stoma for skin breakdown) and tracheostomy suctioning and care.

A nurse in the PACU is assessing a newly admitted client and observes intercostal retractions and a high-pitched inspiratory sound. The nurse should identify these findings as manifestations of which of the following complications? A. Pulmonary edema B. Tension pneumothorax C. Flail chest D. Respiratory obstruction

D. Respiratory obstruction Intercostal retractions and a high-pitched inspiratory noise (i.e. stridor) are manifestations of an airway obstruction caused by laryngospasm and edema. The nurse should notify the rapid response team and plan to administer racemic epinephrine.

A nurse in the PACU is assessing a newly admitted client and observes intercostal retractions and a high-pitched inspiratory sound. The nurse should identify these findings as manifestations of which of the following complications? A. Pulmonary edema B. Tension pneumothorax C. Flail chest D. Respiratory obstruction

D. Respiratory obstruction RATIONALE: high pitched inspiratory noise (stridor)

A nurse is preparing a client for thoracentesis. In which of the following positions should the nurse place the client? A. Lying flat on the affected side B. Prone with the arms raised over the head C. Supine with the head of the bed D. Sitting while leaning forward over the bedside table

D. Sitting while leaning forward over the bedside table

A nurse is preparing a client for thoracentesis. In which of the following positions should the nurse place the client? A. Lying flat on the affected side B. Prone with the arms raised over the head C. Supine with the head of the bed elevated D. Sitting while leaning forward over the bedside table

D. Sitting while leaning forward over the bedside table When preparing a client for thoracentesis, the nurse should have the client sit on the edge of the bed and lean forward over the bedside table. This position maximizes the space between the client's ribs and allows aspiration of accumulated fluid and air.

A nurse is caring for a client who is receiving heparin therapy and develops hematuria. Which one of the following actions should the nurse take if the clients aPTTis 96 seconds? A. Increase the heparin infusion flow rate by 2 ml/hr B. Continue to monitor the heparin infusion as prescribed C. Request a prothrombin time D. Stop the heparin infusion

D. Stop the heparin infusion Rationale: The nurse should identify that the client's aPTT is above the critical value and the client is displaying manifestations of bleeding. Therefore, the nurse should discontinue the heparin infusion immediately and notify the provider to reduce the risk of client injury.

A nurse is caring for a client who is receiving heparin therapy and develops hematuria. Which of the following actions should the nurse take if the client's aPTT is 96 seconds? A. Increase the heparin infusion flow rate by 2mL/hr B. Continue to monitor the heparin infusion as prescribed C. Request a prothrombin time (PT) D. Stop the heparin infusion

D. Stop the heparin infusion RATIONALE: normal aPTT is 30-40 seconds. The client is above this range and is displaying manifestations of bleeding. Stop immediately and notify the provider

A nurse is caring for a client who is receiving IV ampicillin and develops urticaria and dyspnea. Which of the following actions should the nurse take first? A. Elevate the client's feet and legs B. Administer epinephrine C. Infuse 0.9% sodium chloride D. Stop the medication infusion

D. Stop the medication infusion

A client with hypertension has been told to maintain a diet low in sodium. The nurse who is teaching this client about foods that are allowed should include which food item in a list provided to the client? A. Tomato soup B. Boiled shrimp C. Instant oatmeal D. Summer squash

D. Summer squash

A nurse is showing a client who has right-sided heart failure an illustration of the heart. Which of the following blood vessels carry deoxygenated blood to the right atrium? A. Right coronary artery B. Left carotid artery C. Aorta D. Superior vena cava

D. Superior vena cava

On review of the clients' medical records, the nurse determines that which client is at risk for fluid volume excess? A. The client taking diuretics who has tenting of the skin B. The client with an ileostomy from a recent abdominal surgery C. The client who requires intermittent gastrointestinal suctioning D. The client with kidney disease and a 12-year history of diabetes mellitus

D. The client with kidney disease and a 12-year history of diabetes mellitus

A nurse is reviewing the progress notes for a client who has HF. The provider noted some improvement in the client's CO. The nurse should understand that CO reflects which of the following physiologic parameters? A. The percentage of blood the ventricles pump during each beat B. The amount of blood the LV pumps during each beat C. The amount of blood in the LV at the end of diastole D. The heart rate times the stroke volume

D. The heart rate times the stroke volume

A nurse is caring for a client who has a tracheostomy with an inflated cuff in place. Which of the following findings indicates that the nurse should suction the client's airway secretions? A. The client is unable to speak B. The client's airways secretions were last suctioned 2hr ago C. The client coughs and expectorates a large mucous plug D. The nurse auscultates coarse crackles in the lung fields

D. The nurse auscultates coarse crackles in the lung fields RATIONALE: B- the nurse should assess the need for suctioning every 2 hrs and do so as necessary

A nurse is caring for a client who has a tracheostomy with an inflated cuff in place. Which of the following findings indicates that the nurse should suction the client's airway secretions? A. The client is unable to speak. B. The client's airway secretions were last suctioned 2 hr ago. C. The client coughs and expectorates a large mucous plug. D. The nurse auscultates coarse crackles in the lung fields.

D. The nurse auscultates coarse crackles in the lung fields. The nurse should auscultate coarse crackles or rhonchi, identify a moist cough, hear or see secretions in the tracheostomy tube, and then suction the client's airway secretions.

A nurse is preparing to administer cisplatin IV to a client who has lung cancer. The nurse should identify that which of the following findings is an adverse effect of this medication? A. Hallucinations B. Pruritus C. Hand and food syndrome D. Tinnitus

D. Tinnitus RATIONALE: An adverse effect of Cisplatin is ototoxicity, manifesting as tinnitus

A nurse is preparing an in-service presentation about the basics of hematology. Which of the following provides a stimulus for the production of RBCs? A. Venous stasis B. Thrombocytopenia C. Inflammation D. Tissue hypoxia

D. Tissue hypoxia

A client who is receiving a blood transfusion pushes the call light for the nurse. When entering the room, the nurse notes that the client is flushed, dyspneic, and complaining of generalized itching. How should the nurse correctly interpret these findings? A. Bacteremia B. Fluid Overload C. Hypovolemic Shock D. Transfusion Reaction

D. Transfusion Reaction Rationale: The signs and symptoms exhibited by the client are consistent with a transfusion reaction. With bacteremia, the client would have a fever, which is not part of the clinical picture presented. With fluid (circulatory) overload, the client would have crackles in addition to dyspnea. There is no correlation between the signs mentioned in the question and hypovolemic shock. The signs identified in the question are indicative of an allergic reaction, which is one type of blood transfusion reaction.

If you are taking care of someone with COPD and they start to experience dyspnea, what should you do to promote gas exchange? A. Elevate the HOB B. Instruct the pt to elevate arms C. Turn head and cough D. Tripod position

D. Tripod position

A nurse observes tachycardia, dyspnea, a cough and distended neck veins in a client who is receiving a transfusion of packed RBCs. Which of the following interventions should the nurse use to prevent these manifestations in the client's next transfusion? A. Warm the unit of blood to room temperature before administering it B. Administer acetaminophen prior to the blood transfusion C. Give an antihistamine prior to the transfusion D. Use a transfusion pump to regulate and maintain the transfusion at a slower rate

D. Use a transfusion pump to regulate and maintain the transfusion at a slower rate

A nurse is caring for a client who has endocarditis. Which of the following findings should the nurse recognize as a potential complication? A. Ventricular depolarization B. Guillain-Barre syndrome C. Myelodysplastic syndrome D. Valvular disease

D. Valvular disease Rationale: Valvular disease or damage often occurs as a result of inflammation or infection of the endocardium.

A nurse is caring for a client who has pernicious anemia. Which of the following factors should the nurse identify with this condition? A. Iron deficiency B. Hemolytic blood loss C. Folic acid deficiency D. Vitamin B12 deficiency

D. Vitamin B12 deficiency

A nurse is reviewing the lab results of a client who has metabolic alkalosis. Which of the following laboratory values should the nurse expect? A. pH 7.31, HCO3 22, PaCO2 50 B. pH 7.48, HCO3 23, PaCO2 25 C. pH 7.32, HCO3 18, PaCO2 40 D. pH 7.49, HCO3 32, PaCO2 40

D. pH 7.49, HCO3 32, PaCO2 40

A nurse is reviewing the laboratory results of a client who has metabolic alkalosis. Which of the following laboratory values should the nurse expect? A. pH 7.31, HCO3- 22 mEq/L, PaCO2 50 mmHg B. pH 7.48, HCO3- 23 mEq/L, PaCO2 25 mmHg C. pH 7.32, HCO3- 18 mEq/L, PaCO2 40 mmHg D. pH 7.49, HCO3- 32 mEq/L, PaCO2 40 mmHg

D. pH 7.49, HCO3- 32 mEq/L, PaCO2 40 mmHg The nurse should identify that these laboratory values reflect metabolic alkalosis. The pH and bicarbonate values are greater than the expected reference range, and the PaCO2 is within the expected reference range.

a nurse is providing teaching to a client who has chronic asthma and a new prescription for montelukast. which of the following client statements indicates an understanding of the teaching? A.) "I will monitor my heart rate every day while taking this medication." B.) "I will make sure I have this medication with me at all times." C.) "I will need to carefully rinse my mouth after I take this medication." D.) "I will take this medication every night even if I don't have symptoms."

D.) "I will take this medication every night even if I don't have symptoms." rationale: Montelukast is used for the prophylactic treatment of asthma and is taken on a daily basis in the evening.

a nurse is caring for a client who has pulmonary embolism. which of the following interventions is the nurse's priority? A.) Provide a quiet environment B.) Encourage use of incentive spirometer every 1-2 hrs C.) Obtain blood sample for electrolyte study D.) Administer heparin via continuous IV infusion

D.) Administer heparin via continuous IV infusion rationale: When using the airway, breathing, circulation approach to client care, the nurse should place priority on stabilizing circulation to the lungs by administering heparin to prevent further clot formation. Therefore, this is the priority intervention.

a nurse in an ED is caring for a client who's experiencing acute respiratory failure. which of the following lab findings should the nurse expect? A.) Arterial pH 7.50 B.) PaCO2 25 mm Hg C.) SaO2 92% D.) PaO2 58 mm Hg

D.) PaO2 58 mm Hg rationale: The nurse should expect the client to have lower partial pressures of oxygen.

a nurse is caring for a client who's postoperative and has an RR of 9/min secondary to general anesthesia effects na incisional pain. which of the following ABG values indicates the client is experiencing respiratory acidosis? A.) pH 7.50, PO2 95 mm Hg, PaCO2 25 mm Hg, HCO3- 22 mEq/L B.) pH 7.50, PO2 87 mm Hg, PaCO2 35 mm Hg, HCO3- 30 mEq/L C.) pH 7.30, PO2 90 mm Hg, PaCO2 35 mm Hg, HCO3- 20 mEq/L D.) pH 7.30, PO2 80 mm Hg, PaCO2 55 mm Hg, HCO3- 22 mEq/L

D.) pH 7.30, PO2 80 mm Hg, PaCO2 55 mm Hg, HCO3- 22 mEq/L rationale: These ABG values indicate respiratory acidosis. The pH is less than 7.35 and the PaCO2 is greater than 45 mm Hg, which indicates respiratory acidosis.

When teaching the patient with chronic obstructive pulmonary disease (COPD) about smoking cessation, what information should be included related to the effects of smoking on the lungs and the increased incidence of pulmonary infections? Smoking causes a hoarse voice. Cough will become nonproductive. Decreased alveolar macrophage function Sense of smell is decreased with smoking.

Decreased alveolar macrophage function The damage to the lungs includes alveolar macrophage dysfunction that increases the incidence of infections and thus increases patient discomfort and cost to treat the infections. Other lung damage that contributes to infections includes cilia paralysis or destruction, increased mucus secretion, and bronchospasms that lead to sputum accumulation and increased cough. The patient may already be aware of respiratory mucosa damage with hoarseness and decreased sense of smell and taste, but these do not increase the incidence of pulmonary infection.

When providing nutritional counseling for patients at risk for coronary artery disease (CAD), which foods would the nurse encourage patients to include in their diet (select all that apply.)? Tofu Walnuts Tuna fish Whole milk Orange juice

Tofu -Tuna fish, tofu, and walnuts are all rich in omega-3 fatty acids, which have been shown to reduce the risks associated with CAD when consumed regularly. Walnuts -Tuna fish, tofu, and walnuts are all rich in omega-3 fatty acids, which have been shown to reduce the risks associated with CAD when consumed regularly. Tuna fish -Tuna fish, tofu, and walnuts are all rich in omega-3 fatty acids, which have been shown to reduce the risks associated with CAD when consumed regularly.

T o F. Anticholinergic drugs are more effective for COPD patients, unless having a severe asthma attack

True

T o F. For hypoxic drive, it is better to give O2 to a pt who needs it because danger of not giving it outweighs the risk

True

True or False. An intact thirst mechanism is the MAIN protection against dehydration or hyperosmolality

True

True or False. Hyperkalemia and hypokalemia can both cause dysrhythmias and respiratory arrest.

True

True or False. Reducing Na+ levels by more than 8-15 mEq/L in 8 hrs can cause rapid shifts of fluid into cells (cerebral edema)

True

True or False. The kidneys are the primary route for 90% of daily potassium intake.

True

Methylxanthines (theophylline) an alternate therapy to beta-adrenergic agonists have which side effects below? (select all that apply) a. Headache b. GI upset c. Insomnia d. Narrow angle glaucoma

a. Headache b. GI upset c. Insomnia

Identify the acid-base imbalances represented by the following laboratory values. pH: 7.26 PaCO2: 56 mm Hg HCO3-: 24 mEq/L PaO2: 68 mm Hg

Interpretation: respiratory acidosis 1. pH, PaCO2 and PaO2 are abnormal. 2. pH < 7.35 shows acidosis. 3. PaCO2 is > 45 mm Hg, matching the pH and indicating respiratory acidosis. 4. HCO3 − is normal. 5. Compensation is not evident until the kidneys have time to retain bicarbonate. 6. PaO2 is < 80 mm Hg, indicating hypoxemia, which may occur with chronic obstructive pulmonary disease (COPD).

Identify the acid-base imbalances represented by the following laboratory values. pH: 7.50 PaCO2: 30 mm Hg HCO3-: 24 mEq/L PaO2: 79 mm Hg

Interpretation: respiratory alkalosis 1. pH, PaCO2 , and PaO2 are abnormal. 2. pH > 7.45 shows alkalosis. 3. PaCO2 is < 35 mm Hg, indicating respiratory alkalosis. Respiratory alkalosis matches the pH. 4. HCO3 − is within 22 to 26 mEq/L which is normal. 5. No compensation is occurring. Although uncommon, if the HCO3 − were decreased, compensation would be present. 6. The PaO2 is < 80 mm Hg, indicating hypoxemia, which is a common cause of respiratory alkalosis.

A female patient who has type 1 diabetes mellitus has chronic stable angina that is controlled with rest. She states that over the past few months, she has required increasing amounts of insulin. What goal should the nurse use to plan care that should help prevent cardiovascular disease progression? Exercise almost every day. Avoid saturated fat intake. Limit calories to daily limit. Keep Hgb A1C (A1C) less than 7%.

Keep Hgb A1C (A1C) less than 7%. If the Hgb A1C (A1C) is kept below 7%, this means that the patient has had good control of her blood glucose over the past 3 months. The patient indicates that increasing amounts of insulin are being required to control her blood glucose. This patient may not be adhering to the dietary guidelines or therapeutic regimen, so teaching about how to maintain diet, exercise, and medications to maintain stable blood glucose levels will be needed to achieve this goal.

Which antilipemic medications should the nurse question for a patient with cirrhosis of the liver (select all that apply.)? Niacin Cholestyramine Ezetimibe (Zetia) Gemfibrozil (Lopid) Atorvastatin (Lipitor)

Ezetimibe (Zetia) -Ezetimibe (Zetia) should not be used by patients with liver impairment. Adverse effects of atorvastatin (Lipitor), a statin drug, include liver damage and myopathy. Liver enzymes must be monitored frequently and the medication stopped if these enzymes increase. Niacin's side effects subside with time, although decreased liver function may occur with high doses. Cholestyramine is safe for long-term use. Gemfibrozil (Lopid) -Ezetimibe (Zetia) should not be used by patients with liver impairment. Adverse effects of atorvastatin (Lipitor), a statin drug, include liver damage and myopathy. Liver enzymes must be monitored frequently and the medication stopped if these enzymes increase. Niacin's side effects subside with time, although decreased liver function may occur with high doses. Cholestyramine is safe for long-term use. Atorvastatin (Lipitor) -Ezetimibe (Zetia) should not be used by patients with liver impairment. Adverse effects of atorvastatin (Lipitor), a statin drug, include liver damage and myopathy. Liver enzymes must be monitored frequently and the medication stopped if these enzymes increase. Niacin's side effects subside with time, although decreased liver function may occur with high doses. Cholestyramine is safe for long-term use.

While teaching a patient with asthma about the appropriate use of a peak flow meter, what should the nurse instruct the patient to do? Keep a record of the peak flow meter numbers if symptoms of asthma are getting worse. Use the flow meter each morning after taking medications to evaluate their effectiveness. Increase the doses of the long-term control medication if the peak flow numbers decrease. Empty the lungs and then inhale quickly through the mouthpiece to measure how fast air can be inhaled.

Keep a record of the peak flow meter numbers if symptoms of asthma are getting worse. It is important to keep track of peak flow readings daily, especially when the patient's symptoms are getting worse. The patient should have specific directions as to when to call the physician based on personal peak flow numbers. Peak flow is measured by exhaling into the flow meter and should be assessed before and after medications to evaluate their effectiveness.

T o F. Over-the-counter meds that contain ephedrine &/or epinephrine are safe to use for asthma & COPD

False

True or False. Adipose tissue has a higher percentage of water in the tissue.

False

True or False. Hypocalcemia symptoms are depressed reflexes, increased BP, & psychosis.

False

Nursing assessment findings of jugular venous distention and pedal edema would be indicative of what complication of chronic obstructive pulmonary disease (COPD)? Acute respiratory failure Secondary respiratory infection Fluid volume excess resulting from cor pulmonale Pulmonary edema caused by left-sided heart failure

Fluid volume excess resulting from cor pulmonale Cor pulmonale is a right-sided heart failure caused by resistance to right ventricular outflow resulting from lung disease. With failure of the right ventricle, the blood emptying into the right atrium and ventricle would be slowed, leading to jugular venous distention and pedal edema.

When teaching the patient with bronchiectasis about manifestations to report to the health care provider, which manifestation should be included? Increasing dyspnea Temperature below 98.6° F Decreased sputum production Unable to drink 3 L low-sodium fluids

Increasing dyspnea The significant clinical manifestations to report to the health care provider include increasing dyspnea, fever, chills, increased sputum production, bloody sputum, and chest pain. Although drinking at least 3 L of low-sodium fluid will help liquefy secretions to make them easier to expectorate, the health care provider does not need to be notified if the patient cannot do this one day.

The nurse assesses the right femoral artery puncture site as soon as the patient arrives after having a stent inserted into a coronary artery. The insertion site is not bleeding or discolored. What should the nurse do next to ensure the femoral artery is intact? Palpate the insertion site for induration. Assess peripheral pulses in the right leg. Inspect the patient's right side and back. Compare the color of the left and right legs.

Inspect the patient's right side and back. The best method to determine that the right femoral artery is intact after inspection of the insertion site is to logroll the patient to inspect the right side and back for retroperitoneal bleeding. The artery can be leaking and blood is drawn into the tissues by gravity. The peripheral pulses, color, and sensation of the right leg will be assessed per agency protocol.

Identify the acid-base imbalances represented by the following laboratory values. pH: 7.44 PaCO2: 54 mm Hg HCO3-: 36 mEq/L PaO2: 90 mm Hg

Interpretation: compensated or chronic metabolic alkalosis indicated 1. PaCO2 and HCO3 − are abnormal. 2. pH is within normal range but toward alkalosis 3. PaCO2 is > 45 mm Hg, indicating respiratory acidosis. 4. HCO3 − is > 26 mEq, indicating metabolic alkalosis. Because the body will not overcompensate, the metabolic alkalosis is a closer match with the pH. 5. The high PaCO2 indicates the ability of the lungs to compensate for the metabolic alkalosis. 6. The PaO2 is within normal limits.

Identify the acid-base imbalances represented by the following laboratory values. pH: 7.35 PaCO2: 60 mm Hg HCO3-: 40 mEq/L PaO2: 84 mm Hg

Interpretation: compensated respiratory acidosis as reflected by high HCO3 − and pH in normal range 1. PaCO2 and HCO3 − are abnormal. 2. pH is within normal range but toward acidosis. 3. PaCO2 is > 45 mm Hg, indicating respiratory acidosis. 4. HCO3 − is > 26 mEq, indicating metabolic alkalosis. 5. Because the body will not overcompensate, the respiratory acidosis is a closer match with the pH. The high HCO3 − indicates the ability of the kidneys to compensate for the respiratory acidosis. 6. PaO2 is within normal limits, but on the low end, which supports the respiratory system as the causative concern.

Identify the acid-base imbalances represented by the following laboratory values. pH: 7.20 PaCO2: 25 mm Hg HCO3-: 15 mEq/L PaO2: 96 mm Hg

Interpretation: metabolic acidosis; partially compensated 1. pH, PaCO2 and HCO3 − are abnormal. 2. pH < 7.35 shows acidosis. 3. PaCO2 is < 35 mm Hg, indicating respiratory alkalosis. The PaCO2 is in the opposite direction of the pH. 4. HCO3 − is < 22 mEq, indicating metabolic acidosis, which matches the pH. 5. The low PaCO2 indicate the lungs are trying to compensate for the metabolic acidosis. 6. The PaO2 is within normal limits.

Identify the acid-base imbalances represented by the following laboratory values. pH: 7.62 PaCO2: 48 mm Hg HCO3-: 45 mEq/L PaO2: 98 mm Hg

Interpretation: metabolic alkalosis; partially compensated 1. pH, PaCO2 and HCO3 − are abnormal. 2. pH > 7.45 shows alkalosis. 3. PaCO2 is > 45 mm Hg, indicating respiratory acidosis, and is opposite of the pH. 4. HCO3 − is > 26 mEq, indicating metabolic alkalosis, which matches the pH. 5. The PaCO2 in the opposite direction of the alkalotic pH shows that the lungs are trying to compensate for the alkalosis. 6. The PaO2 is within normal limits.

The nurse is evaluating if a patient understands how to safely determine whether a metered dose inhaler (MDI) is empty. The nurse interprets that the patient understands this important information to prevent medication underdosing when the patient describes which method to check the inhaler? Place it in water to see if it floats. Keep track of the number of inhalations used. Shake the canister while holding it next to the ear Check the indicator line on the side of the canister

Keep track of the number of inhalations used. It is no longer appropriate to see if a canister floats in water or not since this is not an accurate way to determine the remaining inhaler doses. The best method to determine when to replace an inhaler is by knowing the maximum puffs available per MDI and then replacing it after the number of days when those inhalations have been used. (100 puffs/2 puffs each day = 50 days)

The nurse supervises a team including another registered nurse (RN), a licensed practical/vocational nurse (LPN/LVN), and unlicensed assistive personnel (UAP) on a medical unit. The team is caring for many patients with respiratory problems. In what situation should the nurse intervene with teaching for a team member? LPN/LVN obtained a pulse oximetry reading of 94% but did not report it. RN taught the patient about home oxygen safety in preparation for discharge. UAP report to the nurse that the patient is complaining of difficulty breathing. LPN/LVN changed the type of oxygen device based on arterial blood gas results.

LPN/LVN changed the type of oxygen device based on arterial blood gas results. It is not within the LPN scope to change oxygen devices based on analysis of lab results. It is within the scope of practice of the RN to assess, teach, and evaluate. The LPN provides care for stable patients and may adjust oxygen flow rates depending on desired oxygen saturation levels of stable patients. The UAP may obtain oxygen saturation levels, assist patients with comfort adjustment of oxygen devices, and report changes in patient's level of consciousness or difficulty breathing.

When caring for a patient with chronic obstructive pulmonary disease (COPD), the nurse identifies a nursing diagnosis of imbalanced nutrition: less than body requirements after noting a weight loss of 30 lb. Which intervention should the nurse add to the plan of care for this patient? Order fruits and fruit juices to be offered between meals. Order a high-calorie, high-protein diet with six small meals a day. Teach the patient to use frozen meals at home that can be microwaved. Provide a high-calorie, high-carbohydrate, nonirritating, frequent feeding diet.

Order a high-calorie, high-protein diet with six small meals a day. Because the patient with COPD needs to use greater energy to breathe, there is often decreased oral intake because of dyspnea. A full stomach also impairs the ability of the diaphragm to descend during inspiration, thus interfering with the work of breathing. For these reasons, the patient with COPD should eat six small meals per day taking in a high-calorie, high-protein diet, with non-protein calories divided evenly between fat and carbohydrate. The other interventions will not increase the patient's caloric intake.

When planning emergent care for a patient with a suspected myocardial infarction (MI), what should the nurse anticipate administrating? Oxygen, nitroglycerin, aspirin, and morphine Aspirin, nitroprusside, dopamine, and oxygen Oxygen, furosemide (Lasix), nitroglycerin, and meperidine Nitroglycerin, lorazepam (Ativan), oxygen, and warfarin (Coumadin)

Oxygen, nitroglycerin, aspirin, and morphine The American Heart Association's guidelines for emergency care of the patient with chest pain include the administration of oxygen, nitroglycerin, aspirin, and morphine. These interventions serve to relieve chest pain, improve oxygenation, decrease myocardial workload, and prevent further platelet aggregation. The other medications may be used later in the patient's treatment.

When teaching the patient with cystic fibrosis about the diet and medications, what is the priority information to be included in the discussion? Fat soluble vitamins and dietary salt should be avoided. Insulin may be needed with a diabetic diet if diabetes mellitus develops. Pancreatic enzymes and adequate fat, calories, protein, and vitamins are needed. Distal intestinal obstruction syndrome (DIOS) can be treated with increased water.

Pancreatic enzymes and adequate fat, calories, protein, and vitamins are needed. The patient must take pancreatic enzymes before each meal and snack and adequate fat, calories, protein, and vitamins should be eaten. Fat-soluble vitamins are needed because they are malabsorbed with the excess mucus in the gastrointestinal system. Insulin may be needed, but there is no longer a diabetic diet, and this is not priority information at this time. DIOS develops in the terminal ileum and is treated with balanced polyethylene glycol electrolyte solution (MiraLAX) to thin bowel contents.

The nurse is examining the electrocardiogram (ECG) of a patient just admitted with a suspected MI. Which ECG change is most indicative of prolonged or complete coronary occlusion? Sinus tachycardia Pathologic Q wave Fibrillatory P waves Prolonged PR interval

Pathologic Q wave The presence of a pathologic Q wave, as often accompanies STEMI, is indicative of complete coronary occlusion. Sinus tachycardia, fibrillatory P waves (e.g., atrial fibrillation), or a prolonged PR interval (first-degree heart block) are not direct indicators of extensive occlusion.

A 52-yr-old male patient has received a bolus dose and an infusion of alteplase (Activase) for an ST-segment elevation myocardial infarction (STEMI). Which patient assessment would determine the effectiveness of the medication? Presence of chest pain Blood in the urine or stool Tachycardia with hypotension Decreased level of consciousness

Presence of chest pain Alteplase is a fibrinolytic agent that is administered to patients who have had an STEMI. If the medication is effective, the patient's chest pain will resolve because the medication dissolves the thrombus in the coronary artery and results in reperfusion of the myocardium. Bleeding is a major complication of fibrinolytic therapy. Signs of major bleeding include decreased level of consciousness, blood in the urine or stool, and increased heart rate with decreased blood pressure.

The nurse teaches pursed lip breathing to a patient who is newly diagnosed with chronic obstructive pulmonary disease (COPD). The nurse reinforces that this technique will assist respiration by which mechanism? Loosening secretions so that they may be coughed up more easily Promoting maximal inhalation for better oxygenation of the lungs Preventing bronchial collapse and air trapping in the lungs during exhalation Increasing the respiratory rate and giving the patient control of respiratory patterns

Preventing bronchial collapse and air trapping in the lungs during exhalation The purpose of pursed lip breathing is to slow down the exhalation phase of respiration, which decreases bronchial collapse and subsequent air trapping in the lungs during exhalation. It does not affect secretions, inhalation, or increase the rate of breathing.

The nurse determines that the patient is not experiencing adverse effects of albuterol (Proventil) after noting which patient vital sign? Pulse rate of 72/minute Temperature of 98.4° F Oxygen saturation 96% Respiratory rate of 18/minute

Pulse rate of 72/minute Albuterol is a β2-agonist that can sometimes cause adverse cardiovascular effects. These would include tachycardia and angina. A pulse rate of 72 indicates that the patient did not experience tachycardia as an adverse effect.

The nurse is assisting a patient to learn self-administration of beclomethasone, two puffs inhaled every 6 hours. What should the nurse explain as the best way to prevent oral infection while taking this medication? Chew a hard candy before the first puff of medication. Rinse the mouth with water before each puff of medication. Ask for a breath mint following the second puff of medication. Rinse the mouth with water following the second puff of medication.

Rinse the mouth with water following the second puff of medication. Because beclamethosone is a corticosteroid, the patient should rinse the mouth with water following the second puff of medication to reduce the risk of fungal overgrowth and oral infection.

The patient has atrial fibrillation with a rapid ventricular response. What electrical treatment option does the nurse prepare the patient for? >Defibrillation >Synchronized cardioversion >Automatic external defibrillator (AED) >Implantable cardioverter-defibrillator (ICD)

Synchronized cardioversion Rationale: Synchronized cardioversion is planned for a patient with supraventricular tachydysrhythmias (atrial fibrillation with a rapid ventricular response). Defibrillation or AEDs are the treatment of choice to end ventricular fibrillation and pulseless ventricular tachycardia (VT). An ICD is used with patients who have survived sudden cardiac death, have spontaneous sustained VT, and are at high risk for future life-threatening dysrhythmias

Number the following organs in the order of the pathway of air inspired through the nose. . Number 1 is the first organ after the environment, and number 13 is the last organ before the alveoli. _______ a. Carina _______ b. Larynx _______ c. Glottis _______ d. Trachea _______ e. Epiglottis _______ f. Nasal cavity _______ g. Bronchioles _______ h. Oropharynx _______ i. Nasopharynx _______ j. Alveolar duct _______ k. Laryngopharynx _______ l. Mainstem bronchi _______ m. Segmental bronchi

___1____ f. Nasal cavity ___2____ i. Nasopharynx ___3____ h. Oropharynx ___4____ k. Laryngopharynx ___5____ e. Epiglottis ___6____ b. Larynx ___7____ c. Glottis ___8____ d. Trachea ___9____ a. Carina ___10____ l. Mainstem bronchi ___11____ m. Segmental bronchi ___12___ g. Bronchioles ___13____ j. Alveolar duct

Priority Decision: A patient just had a CVAD inserted. Number the following nursing actions related to care of the CVAD in the correct order to complete these actions. Number 1 is the first action and number 8 is the last action. _______ a. Perform hand hygiene. _______ b. Flush each line with 10 mL of normal saline. _______ c. Use strict sterile technique to change the dressing. _______ d. Clamp unused lines after flushing if not using positive pressure valve caps. _______ e. Assess the CVAD insertion site for redness, edema, warmth, drainage, and pain. _______ f. Use friction to cleanse the CVAD insertion site with chlorhexidine-based preparation. _______ g. Turn the patient's head to the side away from the CVAD insertion site when changing the caps. _______ h. Obtain chest x-ray results to verify placement of the catheter in the distal end of the superior vena cava.

___1____ h. Obtain chest x-ray results to verify placement of the catheter in the distal end of the superior vena cava. ___2____ e. Assess the CVAD insertion site for redness, edema, warmth, drainage, and pain. ___3____ a. Perform hand hygiene. ___4____ c. Use strict sterile technique to change the dressing. ___5____ f. Use friction to cleanse the CVAD insertion site with chlorhexidine-based preparation. ___6____ g. Turn the patient's head to the side away from the CVAD insertion site when changing the caps. ___7___ b. Flush each line with 10 mL of normal saline. ___8____ d. Clamp unused lines after flushing if not using positive pressure valve caps. The first nursing action after a CVAD is inserted and before it is used is to ensure proper placement with a chest x-ray. Assessments, flushing, dressing changes, and cap changes are completed according to facility policies, but hand hygiene must be completed before manipulating the CVAD to prevent infection. Strict sterile technique is used with dressing and cap changes as well as having the patient turn their face away from the insertion site to avoid contamination.

Place the most common pathophysiologic stages of pneumonia in order. Number the first stage with 1 and the last stage with 4. ________ a. Macrophages lyse the debris and normal lung tissue and function is restored. ________ b. Mucus production increases and can obstruct airflow and further decrease gas exchange. ________ c. Inflammatory response in the lungs with neutrophils is activated to engulf and kill the offending organism. ________ d. Increased capillary permeability contributes to alveolar filling with organisms and neutrophils leading to hypoxia.

___1_____ c. Inflammatory response in the lungs with neutrophils is activated to engulf and kill the offending organism. ___2____ d. Increased capillary permeability contributes to alveolar filling with organisms and neutrophils leading to hypoxia. ___3____ b. Mucus production increases and can obstruct airflow and further decrease gas exchange. ____4____ a. Macrophages lyse the debris and normal lung tissue and function is restored. With most pneumonia-causing organisms the inflammatory response results in increased blood flow and neutrophils to engulf the offending organisms. The alveoli are filled with extra fluid from increased blood flow and capillary permeability from surrounding vessels, which leads to hypoxia. Mucus production is increased and can further obstruct airflow. With bacterial pneumonia consolidation occurs when the alveoli fill with fluid and debris. Macrophages lyse and process the debris so that normal gas exchange returns.

A patient is admitted to the hospital in hypertensive emergency (BP 244/142 mmHg). Sodium nitroprusside is started to treat the elevated BP. Which management strategy(ies) would be appropriate for this patient (select all that apply)? a. Measuring hourly urine output b. Decreasing the MAP by 50% within the first hour c. Continuous BP monitoring with an intraarterial line d. Maintaining bed rest and providing tranquilizers to lower the BP e. Assessing the patient for signs and symptoms of heart failure and changes in mental status

a, c, e

Which BP-regulating mechanism(s) can result in the development of hypertension if defective (select all that apply)? a. Release of norepinephrine b. Secretion of prostaglandins c. Stimulation of the sympathetic nervous system d. Stimulation of the parasympathetic nervous system e. Activation of the renin-angiotensin-aldosterone system

a, c, e

What medications should the nurse expect to include in the teaching plan to decrease the risk of cardiovascular events and death for PAD patients (select all that apply.)? a Ramipril (Altace) b Cilostazol (Pletal) c Simvastatin (Zocor) d Clopidogrel (Plavix) e Warfarin (Coumadin) f Aspirin (acetylsalicylic acid)

a, c, f- Angiotensin-converting enzyme inhibitors (e.g., ramipril [Altace]) are used to control hypertension. Statins (e.g., simvastatin [Zocor]) are used for lipid management. Aspirin is used as an antiplatelet agent. Cilostazol (Pletal) is used for intermittent claudication, but it does not reduce CVD morbidity and mortality risks. Clopidogrel may be used if the patient cannot tolerate aspirin. Anticoagulants (e.g., warfarin [Coumadin]) are not recommended to prevent cardiovascular disease events in PAD patients.

The nurse is providing discharge teaching for a client on erythropoietin. Which statement by the client reveals to the nurse the client understands? a. "I can help control my blood pressure by limiting my salt intake and taking my blood pressure medications as directed by my health care provider (HCP)." b. "Since this medication may cause seizures, I should not drive while taking it." c. "I will have my wife inject this medicine into my arm muscle." d. "If I experience leg discomfort, I will just massage my legs until it has subsided."

a. "I can help control my blood pressure by limiting my salt intake and taking my blood pressure medications as directed by my health care provider (HCP)." This answer is correct because erythropoietin increases the blood pressure. This is due to the production of red blood cells. The client may need to increase the dose of current antihypertensives. Limiting salt intake and monitoring BP daily are also advised.

Which statement indicates the patient with asthma requires further teaching about self-care? a. "I use my corticosteroid inhaler when I feel short of breath." b. "I get a flu shot every year and see my HCP if I have an upper respiratory tract infection." c. "I use my inhaler before I visit my aunt who has a cat, but I only visit for a few minutes because of my allergies." d. "I walk 30 minutes every day but sometimes I have to use my bronchodilator inhaler before walking to prevent me from getting short of breath."

a. "I use my corticosteroid inhaler when I feel short of breath."

A patient with an MI is exhibiting anxiety while being taught about possible lifestyle changes. Which patient statement indicates to the nurse that the anxiety is relieved? a. "I'm going to take this recovery one step at a time." b. "I feel much better and am ready to get on with my life." c. "How soon do you think I will be able to go back to work?" d. "I know you are doing everything possible to save my life."

a. "I'm going to take this recovery one step at a time." This patient is indicating positive coping with a realization that recovery takes time and that lifestyle changes can be made as needed. The patient who is "just going to get on with life" is likely in denial about the seriousness of the condition and the changes that must be made. Nervous questioning about the expected duration and effect of the condition indicates the presence of anxiety, as does the statement about the HCP's role in treatment.

The nurse recognizes that additional teaching is necessary about medications when the patient with moderate asthma makes which statements (select all that apply)? a. "If I can't afford all of my medicines, I will only use the salmeterol (Serevent)." b. "I will stay inside if there is a high pollen count to prevent having an asthma attack." c. "I will rinse my mouth after using fluticasone (Flovent HFA) to prevent oral candidiasis." d. "I must have omalizumab (Xolair) injected every 2 to 4 weeks because inhalers don't help my asthma." e. "I can use my inhaler 3 times, every 20 minutes, before going to the hospital if my peak flow has not improved." f. "My gastroesophageal reflux disease (GERD) medications will help my asthma, and my asthma medications will help my GERD."

a. "If I can't afford all of my medicines, I will only use the salmeterol (Serevent)." f. "My gastroesophageal reflux disease (GERD) medications will help my asthma, and my asthma medications will help my GERD." With asthma, salmeterol (Serevent) should not be taken without inhaled corticosteroids. Gastroesophageal reflux disease (GERD) medications help asthma, but asthma medications may make GERD symptoms worse by relaxing the lower esophageal sphincter. The rest of the statements show patient understanding.

A client undergoing chemotherapy with a low hemoglobin and hematocrit has a new order for epoetin alfa. She questions the nurse as to why she is having to take this new medication. Which response by the nurse is best? a. "It is actually a hormone that will stimulate production of red blood cells within your bone marrow to help alleviate your fatigue since your red blood cells are low from receiving chemotherapy." b. "It is a medication used to stimulate production of white blood cells within the bone marrow for clients undergoing chemotherapy whose white blood cells are low." c. "It is a medication used to stimulate production of platelets within the bone marrow for clients undergoing chemotherapy whose platelets are destroyed by chemotherapy." d. "It is a medication used to stimulate production of dermis fibroblasts for clients undergoing chemotherapy since chemotherapy destroys hair follicle cells."

a. "It is actually a hormone that will stimulate production of red blood cells within your bone marrow to help alleviate your fatigue since your red blood cells are low from receiving chemotherapy." This answer is correct because chemotherapy destroys healthy blood cells. Erythropoietin is a hormone produced by the kidneys. It stimulates production of red blood cells within the bone marrow. It is usually given when the client's hemoglobin is less than 10.

Client diagnosed with pernicious anemia asks why vitamin B12 cannot be given in pill form. Which response by the nurse is best? a. "Stomach doesn't secrete the necessary substance for B12 to be absorbed orally." b. "Oral ingestion of vitamin B12 irritates the GI tract and bleeding could occur." c. "Pernicious anemia alters mucous membrane lining of the bowel and impairs absorption." d. "With severe deficiencies like yours, oral vitamin B12 does not work fast enough."

a. "Stomach doesn't secrete the necessary substance for B12 to be absorbed orally." Vitamin B12 cannot be taken orally, because one problem with pernicious anemia is an inability by the patient to absorb vitamin B12 due to low levels of intrinsic factor. Other options are incorrect.

The nurse determines that teaching about pernicious anemia has been effective when the patient says, a. "This condition can kill me unless I take injections of the vitamin for the rest of my life." b. "My symptoms can be completely reversed after I take a cobalamin (vitamin B12 ) supplement." c. "If my anemia does not respond to cobalamin therapy, my only other alternative is a bone marrow transplant." d. "The least expensive and most convenient treatment of pernicious anemia is to use a diet with foods high in cobalamin."

a. "This condition can kill me unless I take injections of the vitamin for the rest of my life." Without cobalamin replacement, individuals with pernicious anemia will die in 1 to 3 years, but the disease can be controlled with cobalamin supplements for life. Hematologic manifestations can be completely reversed with therapy, but long-standing neuromuscular complications may not be reversed. Because pernicious anemia results from an inability to absorb cobalamin, dietary intake of the vitamin is not a treatment option, nor is a bone marrow transplant.

How should hypernatremia be corrected? a. 0.45% sodium chloride (1/2 NS) b. 5% dextrose in water (DWS) c. 5% sodium chloride (5% NS) d. 3% sodium chloride (3% NS)

a. 0.45% sodium chloride (1/2 NS) SLOWLY to prevent cerebral edema & seizures

A nurse is providing dietary teaching to a client who has kidney disease. Which of the following food choices should the nurse include in the teaching as containing the lowest amount of magnesium? a. 1 large hard-boiled egg b. 1 cup bran cereal c. 1/2 cup almond d. 1 cup cooked spinach

a. 1 large hard-boiled eggs One large hard-boiled egg contains 5 mg of magnesium. Therefore, the nurse should recommend this food as containing the lowest amount of magnesium.Cereal has 112 mg. Almonds 193 mg and spinach 157 mg.

When classifying hypertension stages which of the following BPs is Stage 2? a. 151/96 b. 125/85 c. 137/79 d. 110/72

a. 151/96

What is the therapeutic INR for a patient with DVT and on Warfarin therapy? a. 2.0-3.0 b. 1.0-1.5 c. 6.0-7.0 d. 8.0-9.0

a. 2.0-3.0

After receiving change-of-shift report on a heart failure unit, which patient should the nurse assess first? a. A patient who is cool and clammy, with new-onset confusion and restlessness b. A patient who has crackles bilaterally in the lung bases and is receiving oxygen. c. A patient who had dizziness after receiving the first dose of captopril (Capoten) d. A patient who is receiving IV nesiritide (Natrecor) and has a blood pressure of 100/62

a. A patient who is cool and clammy, with new-onset confusion and restlessness The patient who has wet-cold clinical manifestations of heart failure is perfusing inadequately and needs rapid assessment and changes in management. The other patients also should be assessed as quickly as possible but do not have indications of severe decreases in tissue perfusion.

The nurse in the emergency department receives arterial blood gas results for four recently admitted patients with obstructive pulmonary disease. Which patient will require the most rapid action by the nurse? a. 22-year-old with ABG results: pH 7.28, PaCO2 60 mm Hg, and PaO2 58 mm Hg b. 34-year-old with ABG results: pH 7.48, PaCO2 30 mm Hg, and PaO2 65 mm Hg c. 45-year-old with ABG results: pH 7.34, PaCO2 33 mm Hg, and PaO2 80 mm Hg d. 65-year-old with ABG results: pH 7.31, PaCO2 58 mm Hg, and PaO2 64 mm Hg

a. 22-year-old with ABG results: pH 7.28, PaCO2 60 mm Hg, and PaO2 58 mm Hg The pH, PaCO2, and PaO2 indicate that the patient has severe uncompensated respiratory acidosis and hypoxemia. Rapid action will be required to prevent increasing hypoxemia and correct the acidosis. The other patients also should be assessed as quickly as possible but do not require interventions as quickly as the 22-year-old.

The nurse is reviewing the personal and medical history of several clients. Which finding indicates that a client is at risk for the development of a deep venous​ thrombosis? (Select all that​ apply.) a. 28 weeks' gestation b. Hypercholesterolemia c. Hormone therapy d. Diabetes mellitus e. Lung cancer

a. 28 weeks' gestation c. Hormone therapy e. Lung cancer Hormone​ therapy, lung​ cancer, and pregnancy are all risk factors for the development of DVT. Hypercholesterolemia and diabetes mellitus are risk factors for peripheral vascular​ disease, not DVT.

What is a normal phosphorus level? a. 3.0-4.5 b. 2.0-3.0 c. 1.0-2.0 d. 6.5-7.8

a. 3.0-4.5

The nurse on the intermediate care unit received change-of-shift report on four patients with hypertension. Which patient should the nurse assess first? a. 43-year-old with a (blood pressure (BP) of 160/92 who is complaining of chest pain b. 52-year-old with a BP of 212/90 who has intermittent claudication c. 50-year-old with a BP of 190/104 who has a creatinine of 1.7 mg/dL d. 48-year-old with a BP of 172/98 whose urine shows microalbuminuria

a. 43-year-old with a (blood pressure (BP) of 160/92 who is complaining of chest pain The patient with chest pain may be experiencing acute myocardial infarction, and rapid assessment and intervention are needed. The symptoms of the other patients also show target organ damage but are not indicative of acute processes.

The nurse examines the lymph nodes of a patient during a physical assessment. Which assessment finding would be of most concern to the nurse? a. A 2-cm nontender supraclavicular node b. A 1-cm mobile and nontender axillary node c. An inability to palpate any superficial lymph nodes d. Firm inguinal nodes in a patient with an infected foot

a. A 2-cm nontender supraclavicular node Enlarged and nontender nodes are suggestive of malignancies such as lymphoma. Firm nodes are an expected finding in an area of infection. The superficial lymph nodes are usually not palpable in adults, but if they are palpable, they are normally 0.5 to 1 cm and nontender.

Which rhythm pattern finding is indicative of PVCs? a. A QRS complex > 0.12 second followed by a P wave b. Continuous wide QRS complexes with a ventricular rate of 160 bpm c. P waves hidden in QRS complexes with a regular rhythm of 120 bpm d. Saw-toothed P waves with no measurable PR interval and an irregular rhythm

a. A QRS complex > 0.12 second followed by a P wave PVC is an ectopic beat that causes a wide, distorted QRS complex ≥ 0.12 second because the impulse is not conducted normally through the ventricles. Because it is premature, it precedes the P wave and the P wave may be hidden in the QRS complex, or the ventricular impulse may be conducted retrograde and the P wave may be seen following the PVC, but the rhythm is not regular. Continuous wide QRS complexes with a ventricular rate between 150 and 250 bpm are seen in ventricular tachycardia, whereas saw-toothed P waves are characteristic of atrial flutter

Which statements accurately describe heart failure (HF) (select all that apply)? a. A common cause of HF with preserved ejection fraction (HFpEF) is left ventricular dysfunction. b. A primary risk factor for HF is coronary artery disease (CAD). c. Systolic failure results in a normal left ventricular ejection fraction. d. HF with reduced ejection fraction (HFrEF) is characterized by abnormal resistance to ventricular filling. e. Hypervolemia precipitates HF by decreasing cardiac output and increasing oxygen consumption.

a. A common cause of HF with preserved ejection fraction (HFpEF) is left ventricular dysfunction. b. A primary risk factor for HF is coronary artery disease (CAD). Heart failure with preserved ejection fraction (HFpEF) (diastolic failure) is characterized by abnormal resistance to ventricular filling. Hypertension, coronary artery disease (CAD), advanced age, and diabetes are all risk factors for heart failure (HF). Ejection fraction (EF) is decreased in systolic HF. Decreased cardiac output (CO) and increased workload and oxygen requirements of the myocardium precipitate HF because of left ventricle dysfunction.

The nurse has just finished teaching a hypertensive patient about the newly prescribed ramipril (Altace). Which patient statement indicates that more teaching is needed? a. A little swelling around my lips and face is okay. b. The medication may not work as well if I take any aspirin. c. The doctor may order a blood potassium level occasionally. d. I will call the doctor if I notice that I have a frequent cough.

a. A little swelling around my lips and face is okay. Angioedema occurring with angiotensin-converting enzyme (ACE) inhibitor therapy is an indication that the ACE inhibitor should be discontinued. The patient should be taught that if any swelling of the face or oral mucosa occurs, the health care provider should be immediately notified because this could be life threatening. The other patient statements indicate that the patient has an accurate understanding of ACE inhibitor therapy.

Which statements describe anemia related to blood loss (select all that apply)? a. A major concern is prevention of shock. b. This anemia is most frequently treated with increased dietary iron intake. c. In addition to the general symptoms of anemia, this patient also manifests jaundice. d. A patient who has acute blood loss may have postural hypotension and increased heart rate. e. Initial clinical symptoms are the most reliable way to evaluate the effect and degree of blood loss.

a. A major concern is prevention of shock. d. A patient who has acute blood loss may have postural hypotension and increased heart rate. e. Initial clinical symptoms are the most reliable way to evaluate the effect and degree of blood loss. With rapid blood loss, hypovolemic shock may occur. Clinical manifestations, such as postural hypotension and increased heart rate, will be more reliable than laboratory values as they reflect the body's attempt to meet oxygen requirements. As the percentage of blood loss increases, clinical manifestations worsen.

Which patient should the nurse assign as the roommate for a patient who has aplastic anemia? a. A patient with chronic heart failure b. A patient who has viral pneumonia c. A patient who has right leg cellulitis d. A patient with multiple abdominal drains

a. A patient with chronic heart failure Patients with aplastic anemia are at risk for infection because of the low white blood cell production associated with this type of anemia, so the nurse should avoid assigning a roommate with any possible infectious process.

The nurse is caring for a patient with a fever due to pneumonia. What assessment data does the nurse obtain that correlates with the patient having a fever? (Select all that apply.) a. A temperature of 101.4° F b. Heart rate of 120 beats/min c. Respiratory rate of 20 breaths/min d. A productive cough with yellow sputum e. Reports of unable to have a bowel movement for 2 days

a. A temperature of 101.4° F b. Heart rate of 120 beats/min d. A productive cough with yellow sputum A fever is an inflammatory response related to the infectious process. A productive cough with discolored sputum (which should be clear) is an indication that the patient has pneumonia. A respiratory rate of 20 breaths/min is within normal range. Inability to have a bowel movement is not related to a diagnosis of pneumonia. A heart rate of 120 beats/min indicates that there is increased metabolism due to the fever and is related to the diagnosis of pneumonia.

Which characteristics are related to an acute hemolytic transfusion reaction (select all that apply)? a. ABO incompatibility b. Hypothermia common c. Destruction of donor RBCs d. Acute kidney injury occurs e. Hypocalcemia and hyperkalemia f. Epinephrine used for severe reaction

a. ABO incompatibility c. Destruction of donor RBCs d. Acute kidney injury occurs ABO incompatibility, destruction of donor RBCs, and acute kidney injury may occur in an acute hemolytic transfusion reaction. Hypothermia, hypocalcemia, and hyperkalemia are most likely to occur in massive blood transfusion reactions. Epinephrine may be used for severe allergic transfusion reactions, and the infusion may be restarted after treatment with antihistamines in mild cases.

A patient has a severe blockage in his right coronary artery. Which heart structures are most likely to be affected by this blockage (select all that apply)? a. AV node b. Left ventricle c. Coronary sinus d. Right ventricle e. Pulmonic valve

a. AV node b. Left ventricle d. Right ventricle

Risk Factor for or Response to Respiratory Problem: Decreased exercise or activity tolerance, dyspnea on rest or exertion, sedentary habits. Which Functional Health Pattern does it fall under? a. Activity-exercise b. Sleep-rest c. Cognitive-perceptual d. Self-perception- self-concept

a. Activity-exercise

Which of the following descriptions matches the mechanism of "active transport"? a. Adenosine triphosphate (ATP) required b. Force exerted by a fluid c. Flow of water from low-solute concentration to high-solute concentration d. Passive movement of molecules from a high concentration to lower concentration

a. Adenosine triphosphate (ATP) required

A nurse is planning care for a client who has experienced excessive fluid loss. Which of the following interventions should the nurse include in the plan of care? a. Administer IV fluids to the client evenly over 24 hr. b. Provide the client with a salt substitute. c. Assess the client for pitting edema. d. Encourage the client to rise slowly when standing up. e. Weigh the client every 8 hr.

a. Administer IV fluids to the client evenly over 24 hr d. Encourage the client to rise slowly when standing up e. Weigh the client every 8 hr Administer IV fluids to the client evenly over 24 hr is correct. A client who has excessive fluid loss is typically prescribed IV replacement fluids. Administering IV fluids rapidly over a short period of time places the client at risk for fluid volume overload. Provide the client with a salt substitute is incorrect. There is no reason to limit the client's sodium intake. The client might require electrolyte replacement, depending on the cause of fluid loss. Assess for pitting edema is incorrect. This action is appropriate for a client who has fluid overload. Encourage the client to rise slowly when standing up is correct. This action can prevent injury from falls caused by orthostatic hypotension. Weigh the client every 8 hr is correct. Weighing the client every 8 hr will provide information regarding fluid balance.

Difficulty breathing and crackles after administration of PRBCs. What is the nurse priority actions? Select all that apply. a. Administer furosemide 40 mg IVP b. Elevate HOB at least to 45 degrees c. Monitor for HTN, tachycardia, tachypnea, hypoxia d. Notify HCP

a. Administer furosemide 40 mg IVP b. Elevate HOB at least to 45 degrees c. Monitor for HTN, tachycardia, tachypnea, hypoxia d. Notify HCP All are priority actions

Which factors will the nurse consider when calculating the CURB-65 score for a patient with pneumonia (select all that apply)? a. Age b. Blood pressure c. Respiratory rate d. Oxygen saturation e. Presence of confusion f. Blood urea nitrogen (BUN) level

a. Age b. Blood pressure c. Respiratory rate e. Presence of confusion f. Blood urea nitrogen (BUN) level Data collected for the CURB-65 are mental status (confusion), BUN (elevated), blood pressure(decreased), respiratory rate (increased), and age (65 and older). The other information is also essential to assess, but are not used for CURB-65 scoring.

What are nonmodifiable risk factors for primary hypertension (select all that apply)? a. Age b. Obesity c. Gender d. Ethnicity e. Genetic link

a. Age c. Gender d. Ethnicity e. Genetic link Hypertension progresses with increasing age. It is more prevalent in men before early middle age and above the age of 64 years in women. Blacks have a higher incidence of hypertension than do whites. Children and siblings of patients with hypertension should be screened and taught about healthy lifestyles.

What is the Precaution protocol for a patient with TB? a. Airborne precautions b. Enteric precautions c. Droplet precautions d. Standard precautions

a. Airborne precautions

The nurse reviews the medication administration record (MAR) for a patient having an acute asthma attack. Which medication should the nurse administer first? a. Albuterol (Ventolin) 2.5 mg per nebulizer b. Methylprednisolone (Solu-Medrol) 60 mg IV c. Salmeterol (Serevent) 50 mcg per dry-powder inhaler (DPI) d. Triamcinolone (Azmacort) 2 puffs per metered-dose inhaler (MDI)

a. Albuterol (Ventolin) 2.5 mg per nebulizer Albuterol is a rapidly acting bronchodilator and is the first-line medication to reverse airway narrowing in acute asthma attacks. The other medications work more slowly.

A patient with acute dyspnea is scheduled for a spiral computed tomography (CT) scan. Which information obtained by the nurse is a priority to communicate to the health care provider before the CT? a. Allergy to shellfish b. Apical pulse of 104 c. Respiratory rate of 30 d. Oxygen saturation of 90%

a. Allergy to shellfish Because iodine-based contrast media is used during a spiral CT, the patient may need to have the CT scan without contrast or be premedicated before injection of the contrast media. The increased pulse, low oxygen saturation, and tachypnea all indicate a need for further assessment or intervention but do not indicate a need to modify the CT procedure.

A 20-year-old has a mandatory electrocardiogram (ECG) before participating on a college soccer team and is found to have sinus bradycardia, rate 52. Blood pressure (BP) is 114/54, and the student denies any health problems. What action by the nurse is most appropriate? a. Allow the student to participate on the soccer team. b. Refer the student to a cardiologist for further diagnostic testing. c. Tell the student to stop playing immediately if any dyspnea occurs. d. Obtain more detailed information about the students family health history.

a. Allow the student to participate on the soccer team. In an aerobically trained individual, sinus bradycardia is normal. The students normal BP and negative health history indicate that there is no need for a cardiology referral or for more detailed information about the familys health history. Dyspnea during an aerobic activity such as soccer is normal.

What are the characteristics of neutrophils (select all that apply)? a. Also known as "segs" b. Band is immature cell c. First white blood cells (WBCs) at injury site d. Arise from megakaryocyte e. Increased in individuals with allergies f. 60% to 70% of WBCs

a. Also known as "segs" b. Band is immature cell c. First white blood cells (WBCs) at injury site f. 60% to 70% of WBCs These characteristics are evident with neutrophils. Platelets arise from megakaryocytes and are stored in the spleen. Eosinophils are increased in individuals with allergies and make up 2% to 4% of white blood cells (WBCs).

Pneumonia treatment includes (select all that apply): a. Analgesics b. Steroids c. Diuretics d. Antibiotics

a. Analgesics b. Steroids d. Antibiotics

A nurse is assessing a child who has leukemia. Which of the following findings should the nurse expect? Select all that apply. a. Anorexia b. Petechiae on the extremities c. Unsteady gait d. Jaundice e. Weight gain

a. Anorexia b. Petechiae on the extremities c. Unsteady gait

Patient presents to urgent care 48 hours after a tuberculin skin test. The site looks red and raised with a 19 mm induration. The nurse knows which of the following? Select all that apply a. Anticipate orders for a chest X-ray or sputum samples b. Collect blood for the QuantiFERON-TB test c. The patient has active tuberculosis d. The patient has a tuberculosis infection e. Immediately place the patient on droplet precautions

a. Anticipate orders for a chest X-ray or sputum samples d. The patient has a tuberculosis infection

Which of the following are direct causes of chronic ischemic pain? (Select all that apply.) a. Aortic stenosis b. Acid reflux c. Pulmonary embolus d. Herpes zoster (shingles)

a. Aortic stenosis b. Acid reflux c. Pulmonary embolus

What action is included in the nurse's responsibilities in preparing to administer defibrillation? a. Applying gel pads to the patient's chest b. Setting the defibrillator to deliver 50 joules c. Setting the defibrillator to a synchronized mode d. Sedating the patient with midazolam before defibrillation

a. Applying gel pads to the patient's chest In preparation for defibrillation the nurse should apply conductive materials (e.g., saline pads, electrode gel, defibrillator gel pads) to the patient's chest to decrease electrical impedance and prevent burns. For defibrillation, the initial shock is 120 to 200 joules with biphasic defibrillators, 360 joules with monophasic, and the synchronizer switch used for cardioversion must be turned off. Be sure all staff are clear of the patient and bed before defibrillating. Sedatives may be used before cardioversion if the patient is conscious, but the patient in ventricular fibrillation is unconscious.

Patient-Centered Care: A patient with active TB continues to have positive sputum cultures after 6 months of treatment. She says she cannot remember to take the medication all the time. What is the best action for the nurse to take? a. Arrange for directly observed therapy (DOT) by a public health nurse. b. Schedule the patient to come to the clinic every day to take the medication. c. Have a patient who has recovered from TB tell the patient about his successful treatment. d. Schedule more teaching sessions so that the patient will understand the risks of noncompliance.

a. Arrange for directly observed therapy (DOT) by a public health nurse. Notification of the public health department is required. If drug compliance is questionable, follow-up of patients can be made by directly observed therapy by a public health nurse. A patient who cannot remember to take the medication usually will not remember to come to the clinic daily or will find it too inconvenient. Additional teaching or support from others is not usually effective for this type of patient.

Following assessment of a patient with pneumonia, the nurse identifies a nursing diagnosis of impaired gas exchange based on which finding? a. Arterial oxygen saturation by pulse oximetry (SpO2 ) of 86% b. Crackles in both lower lobes c. Temperature of 101.4° F (38.6° C) d. Production of greenish purulent sputum

a. Arterial oxygen saturation by pulse oximetry (SpO2 ) of 86% Oxygen saturation obtained by pulse oximetry should be >94%. An arterial oxygen saturation by pulse oximetry (SpO2 ) lower than 95% indicates hypoxemia and impaired gas exchange. Crackles, fever, and purulent sputum are all manifestations of pneumonia but do not necessarily relate to impaired gas exchange.

How do microorganisms reach the lungs and cause pneumonia (select all that apply)? a. Aspiration b. Lymphatic spread c. Inhalation of microbes in the air d. Touch contact with the infectious microbes e. Hematogenous spread from infections elsewhere in the body

a. Aspiration c. Inhalation of microbes in the air e. Hematogenous spread from infections elsewhere in the body Microorganisms that cause pneumonia reach the lungs by aspiration from the nasopharynx or oropharynx, inhalation of microbes in the air, and hematogenous spread from infections elsewhere in the body. The other causes of infection do not contribute to pneumonia.

A nurse is caring for a patient immediately following a transesophageal echocardiogram (TEE). Which assessments are appropriate for this patient? (Select all that apply.) a. Assess for return of gag reflex. b. Assess groin for hematoma or bleeding. c. Monitor vital signs and oxygen saturation. d. Position patient supine with head of bed flat. e. Assess lower extremities for circulatory compromise.

a. Assess for return of gag reflex. c. Monitor vital signs and oxygen saturation. The patient undergoing a TEE has been given conscious sedation and has had the throat numbed with a local anesthetic spray, thus eliminating the gag reflex until the effects wear off. Therefore it is imperative that the nurse assess for gag reflex return before allowing the patient to eat or drink. Vital signs and oxygen saturation are important assessment parameters resulting from the use of sedation. A TEE does not involve invasive procedures of the circulatory blood vessels. Therefore it is not necessary to monitor the patient's groin and lower extremities in relation to this procedure or to maintain a flat position.

What should the nurse do when preparing a patient for a pulmonary angiogram? a. Assess the patient for iodine allergy. b. Implement NPO orders for 6 to 12 hours before the test. c. Explain the test before the patient signs the informed consent form. d. Inform the patient that radiation isolation for 24 hours after the test is necessary

a. Assess the patient for iodine allergy. A pulmonary angiogram involves the injection of an iodine-based radiopaque dye, and iodine or shellfish allergies should be assessed before injection. A bronchoscopy requires NPO status for 6 to 12 hours before the test, and invasive tests (e.g., bronchoscopy, mediastinoscopy, biopsies) require informed consent that the HCP should obtain from the patient. Nuclear scans use radioactive materials for diagnosis, but the amounts are very small and no radiation precautions are indicated for the patient.

A newly admitted patient is diagnosed with hyponatremia. When making room assignments, the charge nurse should take which action? a. Assign the patient to a room near the nurse's station. b. Place the patient in a room nearest to the water fountain. c. Place the patient on telemetry to monitor for peaked T waves. d. Assign the patient to a semi-private room and place an order for a low-salt diet.

a. Assign the patient to a room near the nurse's station. The patient should be placed near the nurses station if confused in order for the staff to closely monitor the patient. To help improve serum sodium levels, water intake is restricted. Therefore, a confused patient should not be placed near a water fountain. Peaked T waves are a sign of hyperkalemia, not hyponatremia. A confused patient could be distracting and disruptive for another patient in a semiprivate room. This patient needs sodium replacement, not restriction.

Collaboration: The nurse is caring for a patient with COPD. Which intervention could be delegated to unlicensed assistive personnel (UAP)? a. Assist the patient to get out of bed. b. Auscultate breath sounds every 4 hours. c. Plan patient activities to minimize exertion. d. Teach the patient pursed-lip breathing technique.

a. Assist the patient to get out of bed. Assistance with positioning and activities of daily living (ADLs) is within the training of UAP. Teaching, assessing, and planning are all part of the RN's practice.

A patient with bacterial pneumonia has rhonchi and thick sputum. What is the nurses most appropriate action to promote airway clearance? a. Assist the patient to splint the chest when coughing. b. Teach the patient about the need for fluid restrictions. c. Encourage the patient to wear the nasal oxygen cannula. d. Instruct the patient on the pursed lip breathing technique.

a. Assist the patient to splint the chest when coughing. Coughing is less painful and more likely to be effective when the patient splints the chest during coughing. Fluids should be encouraged to help liquefy secretions. Nasal oxygen will improve gas exchange, but will not improve airway clearance. Pursed lip breathing is used to improve gas exchange in patients with COPD, but will not improve airway clearance.

The nurse is caring for a client diagnosed with coronary artery disease (CAD). What condition most commonly results in CAD? a. Atherosclerosis b. Diabetes mellitus c. Myocardial infarction d. Renal failure

a. Atherosclerosis Atherosclerosis (plaque formation) is the leading cause of CAD. Diabetes mellitus is a risk factor for CAD, but it isn't the most common cause. Myocardial infarction is a common result of CAD. Renal failure doesn't cause CAD, but the two conditions are related

Which one of the following inhaled medications is a short-acting bronchdilator used for emergencies in asthma? a. Atrovent b. Beclomethasone c. Fluticasone d. Indacaterol (Arcapta)

a. Atrovent

Priority Decision: A patient's tracheostomy tube becomes dislodged with vigorous coughing. What should be the nurse's first action? a. Attempt to replace the tube. b. Notify the health care provider. c. Place the patient in high Fowler's position. d. Ventilate the patient with a manual resuscitation bag until the health care provider arrives.

a. Attempt to replace the tube. If a tracheostomy tube is dislodged, the nurse should immediately attempt to replace the tube by using hemostats to spread the opening. The obturator is inserted in the replacement tube, water-soluble lubricant is applied to the tip, and the tube is inserted in the stoma at a 45-degree angle to the neck. The obturator is immediately removed to provide an airway. If the tube cannot be reinserted, the HCP should be notified and the patient should be assessed for the level of respiratory distress, positioned in semi-Fowler's position, and ventilated with a manual resuscitation bag (MRB) only if necessary, until assistance arrives. 15. b. The primary risk factors associated with head and neck cancers are heavy tobacco and alcohol use. Oral cancer may cause a change in the fit of dentures, but denture use is not a risk factor for oral cancer. Chronic infections are not known to be risk factors, although cancers in patients younger than age 50 years have been associated with human papillomavirus (HPV) infection.

A patient who was admitted the previous day with pneumonia complains of a sharp pain of 7 (based on 0 to 10 scale) whenever I take a deep breath. Which action will the nurse take next? a. Auscultate breath sounds. b. Administer the PRN morphine. c. Have the patient cough forcefully. d. Notify the patients health care provider.

a. Auscultate breath sounds. The patients statement indicates that pleurisy or a pleural effusion may have developed and the nurse will need to listen for a pleural friction rub and/or decreased breath sounds. Assessment should occur before administration of pain medications. The patient is unlikely to be able to cough forcefully until pain medication has been administered. The nurse will want to obtain more assessment data before calling the health care provider.

Two days after an acute myocardial infarction (MI), a patient complains of stabbing chest pain that increases with a deep breath. Which action will the nurse take first? a. Auscultate the heart sounds. b. Check the patients temperature. c. Notify the patients health care provider. d. Give the PRN acetaminophen (Tylenol).

a. Auscultate the heart sounds. The patients clinical manifestations and history are consistent with pericarditis, and the first action by the nurse should be to listen for a pericardial friction rub. Checking the temperature and notifying the health care provider are also appropriate actions but would not be done before listening for a rub. It is not stated for what symptom (e.g., headache) or finding (e.g., increased temperature) the PRN acetaminophen (Tylenol) is ordered.

The nurse is performing a focused respiratory assessment of a patient who is in severe respiratory distress 2 days after abdominal surgery. What is most important for the nurse to assess? a. Auscultation of bilateral breath sounds b. Percussion of anterior and posterior chest wall c. Palpation of the chest bilaterally for tactile fremitus d. Inspection for anterior and posterior chest expansion

a. Auscultation of bilateral breath sounds Important assessments obtained during a focused respiratory assessment include auscultation of lung (breath) sounds. Assessment of tactile fremitus has limited value in acute respiratory distress. It is not necessary to assess for both anterior and posterior chest expansion. Percussion of the chest wall is not essential in a focused respiratory assessment.

The complete blood count (CBC) indicates that a patient is thrombocytopenic. Which action should the nurse include in the plan of care? a. Avoid intramuscular injections. b. Encourage increased oral fluids. c. Check temperature every 4 hours. d. Increase intake of iron-rich foods.

a. Avoid intramuscular injections. Thrombocytopenia is a decreased number of platelets, which places the patient at high risk for bleeding. Neutropenic patients are at high risk for infection and sepsis and should be monitored frequently for signs of infection. Encouraging fluid intake and iron-rich food intake is not indicated in a patient with thrombocytopenia.

A pt with N-STEMI angina is discharged to return for an exercise stress test in the AM. Which should be taught: (SATA) a. Avoid smoking or strenuous exercise 3 hours before the test b. NPO after midnight c. Stop taking Metoprolol until after the test d. Wear comfortable shoes

a. Avoid smoking or strenuous exercise 3 hours before the test c. Stop taking Metoprolol until after the test d. Wear comfortable shoes

A frail older adult patient develops sudden shortness of breath while sitting in a chair. What location on the chest should the nurse begin auscultation of the lung fields? a. Bases of the posterior chest area b. Apices of the posterior lung fields c. Anterior chest area above the breasts d. Midaxillary on the left side of the chest

a. Bases of the posterior chest area Baseline data with the most information is best obtained by auscultation of the posterior chest, especially in female patients because of breast tissue interfering with the assessment or if the patient may tire easily (e.g., shortness of breath, dyspnea, weakness, fatigue). Usually auscultation proceeds from the lung apices to the bases unless it is possible the patient will tire easily. In this case, the nurse should start at the bases.

Which cells are classified as granulocytes (select all that apply)? a. Basophil b. Monocyte c. Eosinophil d. Neutrophil e. Lymphocyte f. Thrombocyte

a. Basophil c. Eosinophil d. Neutrophil Basophils, eosinophils, and neutrophils are the granulocytic leukocytes. Lymphocytes are the agranular leukocytes that form the basis of the cellular and humoral immune responses. Monocytes are agranulocytes that are potent phagocytic cells. Thrombocytes are not granulocytes or agranulocytes, and they initiate the clotting process.

A patient with a bone marrow disorder has an overproduction of myeloblasts. The nurse would expect the results of a complete blood count (CBC) to include an increase in which cell types (select all that apply)? a. Basophils b. Eosinophils c. Monocytes d. Neutrophils e. Lymphocytes

a. Basophils b. Eosinophils d. Neutrophils The myeloblast is a committed hematopoietic cell found in the bone marrow from which granulocytes develop. A disorder in which myeloblasts are overproduced would result in increased basophils, eosinophils, and neutrophils.

Priority Decision: A patient on the telemetry unit goes into VF and is unresponsive. Following initiation of the emergency call system (Code Blue), what is the next priority for the nurse in caring for this patient? a. Begin CPR. b. Get the crash cart. c. Administer amiodarone IV. d. Defibrillate with 360 joules

a. Begin CPR. Until the defibrillator is available, the patient needs CPR. Defibrillation is needed as soon as possible, so someone should bring the crash cart to the room. Amiodarone is an antidysrhythmic that is part of the advanced cardiac life support (ACLS) protocol for ventricular fibrillation. Defibrillation would be with 360 joules for monophasic defibrillators and 120 to 200 joules for biphasic defibrillators.

A patient who has bronchiectasis asks the nurse, "What conditions would warrant a call to the clinic?" a. Blood clots in the sputum b. Sticky sputum on a hot day c. Increased shortness of breath after eating a large meal d. Production of large amounts of sputum on a daily basis

a. Blood clots in the sputum

This type of anemia results in a decreased circulatory volume and can lead to hypovolemic shock. a. Blood loss b. Iron deficiency c. Aplastic d. Pernicious

a. Blood loss

The nurse is caring for a patient with a massive burn injury and possible hypovolemia. Which assessment data will be of most concern to the nurse? a. Blood pressure is 90/40 mm Hg. b. Urine output is 30 mL over the last hour. c. Oral fluid intake is 100 mL for the last 8 hours. d. There is prolonged skin tenting over the sternum.

a. Blood pressure is 90/40 mm Hg. The blood pressure indicates that the patient may be developing hypovolemic shock as a result of intravascular fluid loss due to the burn injury. This finding will require immediate intervention to prevent the complications associated with systemic hypo perfusion. The poor oral intake, decreased urine output, and skin tenting all indicate the need for increasing the patients fluid intake but not as urgently as the hypotension.

What is a possible cause for auscultation abnormal finding "wheezes"? a. Bronchoconstriction b. Partial obstruction of trachea or larynx c. Chronic hypoxemia d. Pleurisy

a. Bronchoconstriction

Which initial physical assessment finding would the nurse expect to be present in a patient with acute left-sided HF? a. Bubbling crackles and tachycardia b. Hepatosplenomegaly and tachypnea c. Peripheral edema and cool, diaphoretic skin d. Frothy, blood-tinged sputum and distended jugular veins

a. Bubbling crackles and tachycardia Early clinical manifestations of acute left-sided HF are those of interstitial edema, with bubbling crackles and tachycardia, as well as tachypnea. Later frothy, blood-tinged sputum; severe dyspnea; and orthopnea develop with alveolar edema. Severe tachycardia and cool, clammy skin are present as a result of stimulation of the SNS from hypoxemia. Systemic edema reflected by jugular vein distention, peripheral edema, and hepatosplenomegaly are characteristic of rightsided HF.

The nurse is providing care for a patient who has decreased cardiac output due to heart failure. As a basis for planning care, what should the nurse understand about cardiac output (CO)? a. CO is calculated by multiplying the patient's stroke volume by the heart rate. b. CO is the average amount of blood ejected during one complete cardiac cycle. c. CO is determined by measuring the electrical activity of the heart and the heart rate. d. CO is the patient's average resting heart rate multiplied by the mean arterial blood pressure.

a. CO is calculated by multiplying the patient's stroke volume by the heart rate. Cardiac output is determined by multiplying the patient's stroke volume by heart rate, thus identifying how much blood is pumped by the heart over a 1-minute period. Electrical activity of the heart and blood pressure are not direct components of cardiac output.

The patient has used sublingual nitroglycerin (NTG) and various long-acting nitrates but now has an ejection fraction of 38% and is considered at a high risk for a cardiac event. Which medication would first be added for vasodilation and to reduce ventricular remodeling? a. Captopril b. Clopidogrel (Plavix) c. Diltiazem (Cardizem) d. Metoprolol (Lopressor)

a. Captopril Captopril would be added. It is an angiotensin-converting enzyme (ACE) inhibitor that vasodilates and decreases endothelial dysfunction and may prevent ventricular remodeling. Clopidogrel (Plavix) is an antiplatelet agent used as an alternative for a patient unable to use aspirin. Diltiazem (Cardizem), a calcium channel blocker, may be used to decrease vasospasm but is not known to prevent ventricular remodeling. Metoprolol (Lopressor) is a β-adrenergic blocker that inhibits sympathetic nervous stimulation of the heart.

What manifestations most strongly support a diagnosis of acute rheumatic fever? a. Carditis, polyarthritis, and erythema marginatum b. Polyarthritis, chorea, and decreased antistreptolysin-O titer c. Organic heart murmurs, fever, and elevated erythrocyte sedimentation rate (ESR) d. Positive C-reactive protein, elevated white blood cells (WBCs), and subcutaneous nodules

a. Carditis, polyarthritis, and erythema marginatum Two major criteria; 1 major and 2 minor criteria plus laboratory evidence of a preceding group A streptococcal infection indicate rheumatic fever. Major criteria for the diagnosis of rheumatic fever include evidence of carditis, polyarthritis, erythema marginatum, Sydenham's chorea (often very late), and subcutaneous nodules. Minor criteria include ↑ erythrocyte sedimentation rate (ESR) and/or ↑ Creactive protein (CRP), fever, arthralgia.

46-year-old is diagnosed with thromboangiitis obliterans (Buergers disease). When the nurse is developing a discharge teaching plan for the patient, which outcome has the highest priority for this patient? a. Cessation of all tobacco use b. Control of serum lipid levels c. Maintenance of appropriate weight d. Demonstration of meticulous foot care

a. Cessation of all tobacco use Absolute cessation of nicotine use is needed to reduce the risk for amputation in patients with Buergers disease. Other therapies have limited success in treatment of this disease.

The nurse is preparing to administer a blood transfusion to a client who has anemia. Which action should the nurse take first? a. Check for the type and number of units of blood to administer b. Remain with the client for the first 15 minutes. c. Administer the PRBCs through a percutaneously inserted central catheter line with a 18-gauge needle. d. Flush PRBCs with 0.9% normal saline solution.

a. Check for the type and number of units of blood to administer

Diagnostic assessments for Tuberculosis includes (SATA): a. Chest x-ray b. PPD c. Serum electrolytes d. Sputum culture

a. Chest x-ray b. PPD d. Sputum culture

Which food item should the heart failure patient avoid? Select all that apply. a. Chips b. Fruits c. Veggies d. Grilled chicken & fries e. Canned beans f. Bread

a. Chips - NO SODIUM! d. Grilled chicken & fries - NO! e. Canned beans - NO!

Which of the following should an asthma patient avoid to prevent an attack? (select all that apply) a. Cigarette smoke b. Aspirin or NSAIDs c. Early exposure to children, fewer antibiotics, pets, etc. d. Allergens e. Air pollution f. Beta-blockers

a. Cigarette smoke b. Aspirin or NSAIDs d. Allergens e. Air pollution f. Beta-blockers

A nurse is caring for a client who had a MI 5 days ago. The client now has sudden cough, frothy pink sputum, and sob. The nurse hears bubbly, loud sounds via auscultation. Which term describes the adventitious breath sound that is going on? a. Coarse crackles b. Rhonchi c. Wheezes d. Friction rub

a. Coarse crackles

Which nursing action should the nurse take first in order to assist a patient with newly diagnosed stage 1 hypertension in making needed dietary changes? a. Collect a detailed diet history. b. Provide a list of low-sodium foods. c. Help the patient make an appointment with a dietitian. d. Teach the patient about foods that are high in potassium.

a. Collect a detailed diet history. The initial nursing action should be assessment of the patients baseline dietary intake through a thorough diet history. The other actions may be appropriate, but assessment of the patients baseline should occur first.

A nurse is assessing a client who has respiratory acidosis. Which of the following findings should the nurse except? a. Confusion b. Peripheral edema c. Facial flushing d. Hyperreflexia

a. Confusion A client who has respiratory acidosis will experience confusion from a lack of cerebral perfusion. If acidosis is not reversed, the client's level of consciousness will decrease and coma may occur. Facial flushing and warmth are manifestations of metabolic acidosis. Pale, cyanotic, dry skin is a manifestation of respiratory acidosis as ineffective breathing causes a lack of perfusion to the tissues. Hyporeflexia, not hyperreflexia, is a manifestation of respiratory acidosis. As acidosis increases, hyperkalemia can occur, causing muscle weakness, flaccid paralysis, and hyporeflexia.

6. When obtaining a health history from a 76-year-old patient with suspected CAP, what does the nurse expect the patient or caregiver to report? a. Confusion b. A recent loss of consciousness c. An abrupt onset of fever and chills d. A gradual onset of headache and sore throat

a. Confusion Confusion possibly related to hypoxia may be the only finding in older adults. Although CAP is most commonly caused by S. aureus and is associated with an acute onset with fever, chills, productive cough with purulent or bloody sputum, and pleuritic chest pain, the older patient may not have classic symptoms. A recent loss of consciousness or altered consciousness is common in those pneumonias associated with aspiration, such as anaerobic bacterial pneumonias. Other causes of pneumonia have a more gradual onset with dry, hacking cough; headache; and sore throat.

Priority Decision: A patient's rhythm strip indicates a normal HR and rhythm with normal P waves and QRS complexes, but the PR interval is 0.26 second. What is the most appropriate action by the nurse? a. Continue to assess the patient. b. Administer atropine per protocol. c. Prepare the patient for synchronized cardioversion. d. Prepare the patient for placement of a temporary pacemaker.

a. Continue to assess the patient. A rhythm pattern that is normal except for a prolonged PR interval is characteristic of a first-degree heart block. First-degree heart blocks are not treated but are observed for progression to higher degrees of heart block. Atropine is administered for bradycardia. Synchronized cardioversion is used for atrial fibrillation with a rapid ventricular response or PSVT. Pacemakers are used for higher-degree heart blocks.

The nurse recognizes that thrombolytic therapy for the treatment of an MI has not been successful when the patient displays which manifestation? a. Continues to have chest pain b. Develops gastrointestinal (GI) bleeding c. Has a marked increase in CK-MB levels within 3 hours of therapy d. Develops premature ventricular contractions and ventricular tachycardia during treatment

a. Continues to have chest pain If chest pain is unchanged, it is a sign that reperfusion was not successful. Indications that the occluded coronary artery is patent and blood flow to the myocardium is reestablished following thrombolytic therapy include return of ST-segment to baseline on the ECG; relief of chest pain; marked, rapid rise of the CK-MB within 3 hours of therapy; and the presence of reperfusion dysrhythmias. Bleeding is a complication of thrombolytic therapy but does not indicate lack of success or successful reperfusion.

Priority Decision: A patient is hospitalized after a successful resuscitation of an episode of sudden cardiac death (SCD). During the care of the patient, what nursing intervention is most important? a. Continuous ECG monitoring b. Auscultation of the carotid arteries c. Frequent assessment of heart sounds d. Monitoring airway status and respirations

a. Continuous ECG monitoring Many patients who have a sudden cardiac death (SCD) experience because of CAD do not have an acute MI but have dysrhythmias that cause death. This is probably because of electrical instability of the myocardium. To identify and treat those specific dysrhythmias, continuous monitoring is important. The other assessments can be done but are not the most important after an episode of SCD.

Increases in which blood studies are diagnostic for acute coronary syndrome (ACS) (select all that apply)? a. Copeptin b. Creatine kinase (CK-MM) c. Cardiac troponin T (cTnT) d. B-type natriuretic peptide (BNP) e. High-sensitivity C-reactive protein (hs-CRP) f. Lipoprotein-associated phospholipase A2 (Lp-PLA2 )

a. Copeptin c. Cardiac troponin T (cTnT) Copeptin is detected immediately with acute coronary syndrome (ACS) and increased levels of cardiac troponin T (cTnT) are detected within hours. Increased CK-MM is most commonly associated with skeletal muscle injury. Increased b-type natriuretic peptide (BNP) is a marker for heart failure. Increased C-reactive protein (CRP) occurs with acute inflammation as in atherosclerosis. Increased lipoprotein-associated phospholipase A2 (Lp-PLA2 ) indicates increased risk for CAD.

Which finding in a patient hospitalized with bronchiectasis is most important to report to the health care provider? a. Cough productive of bloody, purulent mucus b. Scattered rhonchi and wheezes heard bilaterally c. Respiratory rate 28 breaths/minute while ambulating in hallway d. Complaint of sharp chest pain with deep breathing

a. Cough productive of bloody, purulent mucus Hemoptysis may indicate life-threatening hemorrhage and should be reported immediately to the health care provider. The other findings are frequently noted in patients with bronchiectasis and may need further assessment but are not indicators of life-threatening complications.

The nurse is admitting a 68-yr-old preoperative patient with a suspected abdominal aortic aneurysm (AAA). The medication history reveals that the patient has been taking warfarin (Coumadin) on a daily basis. Based on this history and the patient's admission diagnosis, the nurse should prepare to administer which medication? a Vitamin K b Cobalamin c Heparin sodium d Protamine sulfate

a. Coumadin is a vitamin K antagonist anticoagulant that could cause excessive bleeding during surgery if clotting times are not corrected before surgery. For this reason, vitamin K is given as the antidote for warfarin (Coumadin).

When assessing the patient in acute respiratory distress, what should the nurse expect to observe? (Select all that apply.) a. Cyanosis b. Tripod position c. Kussmaul respirations d. Accessory muscle use e. Increased AP diameter

a. Cyanosis d. Accessory muscle use Tripod position and accessory muscle use indicate moderate to severe respiratory distress. Cyanosis may be related to anemia, decreased oxygen transfer in the lungs, or decreased cardiac output. Therefore, it is a nonspecific and unreliable indicator of only respiratory distress. Kussmaul respirations occur when the patient is in metabolic acidosis to increase CO2 excretion. Increased AP diameter occurs with lung hyperinflation from chronic obstructive pulmonary disease, cystic fibrosis, or with advanced age.

A nurse in the outpatient clinic is caring for a patient who has a magnesium level of 1.3 mg/dL. Which assessment would be most important for the nurse to make? a. Daily alcohol intake b. Intake of dietary protein c. Multivitamin/mineral use d. Use of over-the-counter (OTC) laxatives

a. Daily alcohol intake Hypomagnesemia is associated with alcoholism. Protein intake would not have a significant effect on magnesium level. OTC laxatives (such as milk of magnesia) and use of multivitamin/mineral supplements would tend to increase magnesium levels.

The health care provider prescribes spironolactone (Aldactone) for the patient with chronic HF. What diet modifications related to the use of this drug should the nurse include in the patient teaching? a. Decrease both sodium and potassium intake. b. Increase calcium intake and decrease sodium intake. c. Decrease sodium intake and increase potassium intake. d. Decrease sodium intake by using salt substitutes for seasoning.

a. Decrease both sodium and potassium intake. Spironolactone is a potassium-sparing diuretic, and when it is the only diuretic used in the treatment of HF, moderate to low levels of potassium intake should be maintained to prevent development of hyperkalemia. Sodium intake is usually reduced to at least 2400 mg/day in patients with HF, but salt substitutes cannot be freely used because many contain high concentrations of potassium. Calcium intake is not increased

Which effects contribute to making nitrates the first-line therapy for the treatment of angina (select all that apply)? a. Decrease preload b. Decrease afterload c. Dilate coronary arteries d. Decrease heart rate (HR) e. Prevent thrombosis of plaques f. Decrease myocardial contractility

a. Decrease preload b. Decrease afterload c. Dilate coronary arteries Nitrates decrease preload and afterload to decrease the coronary workload and dilate coronary arteries to increase coronary blood supply. The other options are not attributed to nitrates.

A patient has a BP of 222/148 mm Hg and confusion, nausea, and vomiting. Which goal should the nurse try to achieve by titrating medications? a. Decrease the mean arterial pressure (MAP) to 129 mm Hg b. Lower the BP to the patient's normal within the second to third hour c. Decrease the SBP to 160 mm Hg and the DBP to 100 mm Hg as quickly as possible d. Reduce the SBP to 158 mm Hg and the DBP to 90 mm Hg within the first 2 hours

a. Decrease the mean arterial pressure (MAP) to 129 mm Hg Initially, the treatment goal in hypertensive emergencies is to reduce the mean arterial pressure (MAP) by no more than 20% to 25% in the first hour, with further gradual reduction over the next 24 hours. In this case, the MAP is 222 + 2(148)/3 = 172, so decreasing it by 25% equals 129. Lowering the BP too far or too fast may cause a stroke, myocardial infarction (MI), or renal failure. Only when the patient has an aortic dissection, angina, or signs of an ischemic stroke does the SBP have to be lowered to 100 to 120 mm Hg or less as quickly as possible

Which age-related changes in the respiratory system cause decreased secretion clearance (select all that apply)? a. Decreased force of cough b. Decreased functional cilia c. Decreased chest wall compliance d. Small airway closure earlier in expiration e. Decreased functional immunoglobulin A (IgA)

a. Decreased force of cough b. Decreased functional cilia Decreased functional cilia and decreased force of cough from declining muscle strength cause decreased secretion clearance. The other options contribute to other age-related changes. Decreased compliance contributes to barrel chest appearance. Early small airway closure contributes to decreased PaO2 . Decreased immunoglobulin A (IgA) decreases the resistance to infection.

A nurse is assessing a client who has hyperkalemia. Which of the following findings should the nurse expect? a. Decreased muscle strength b. Decreased gastric motility c. Increased heart rate d. Increased blood pressure

a. Decreased muscle strength Hyperkalemia can cause muscle weakness. The nurse should monitor the client's muscle strength.

A 78-year-old patient is admitted with a BP of 180/98 mm Hg. Which age-related physical changes may contribute to this patient's hypertension (select all that apply)? a. Decreased renal function b. Increased baroreceptor reflexes c. Increased peripheral vascular resistance d. Increased adrenergic receptor sensitivity e. Increased collagen and stiffness of the myocardium f. Loss of elasticity in large arteries from arteriosclerosis

a. Decreased renal function c. Increased peripheral vascular resistance e. Increased collagen and stiffness of the myocardium f. Loss of elasticity in large arteries from arteriosclerosis The age-related changes that contribute to hypertension include decreased renal function, increased peripheral vascular resistance, increased collagen and stiffness of the myocardium, and decreased elasticity in large arteries from arteriosclerosis. The baroreceptor reflexes are blunted. The adrenergic receptor sensitivity and renin response are both decreased with aging.

A patient is scheduled to have a tunneled catheter placed for administration of chemotherapy for breast cancer. When preparing the patient for the catheter insertion, what does the nurse explain about this method of chemotherapy administration? a. Decreases the risk for extravasation at the infusion site b. Reduces the incidence of systemic side effects of the drug c. Does not become occluded as peripherally inserted catheters can d. Allows continuous infusion of the drug directly to the area of the tumor

a. Decreases the risk for extravasation at the infusion site Catheters tunneled to the distal end of the superior vena cava or the right atrium are vascular access devices inserted into central veins, which decrease the incidence of extravasation, provide for rapid dilution of chemotherapy, and reduce the need for venipunctures. Most right atrial catheters, except for a Groshong catheter, must be flushed with heparin to prevent clotting in the tubing. Regional chemotherapy administration delivers the drug directly to the tumor and is the only administration route that can decrease the systemic effects of the drugs.

To provide free water and intracellular fluid hydration for a patient with acute gastroenteritis who is NPO, the nurse would expect administration of which infusion? a. Dextrose 5% in water b. Dextrose 10% in water c. Lactated Ringer's solution d. Dextrose 5% in normal saline (0.9%)

a. Dextrose 5% in water Fluids such as 5% dextrose in water (D5W) allow water to move from the ECF to the ICF. Although D5W is physiologically isotonic, the dextrose is rapidly metabolized, leaving free water to shift into cells.

The nurse assesses a patient who has numerous petechiae on both arms. Which question should the nurse ask the patient? a. Do you take salicylates? b. Are you taking any oral contraceptives? c. Have you been prescribed antiseizure drugs? d. How long have you taken antihypertensive drugs?

a. Do you take salicylates? Salicylates interfere with platelet function and can lead to petechiae and ecchymoses. Antiseizure drugs may cause anemia, but not clotting disorders or bleeding. Oral contraceptives increase a persons clotting risk. Antihypertensives do not usually cause problems with decreased clotting.

Priority Decision: A 75-year-old patient who is breathing room air has the following arterial blood gas (ABG) results: pH 7.40, partial pressure of oxygen in arterial blood (PaO2 ) 74 mm Hg, arterial oxygen saturation (SaO2 ) 92%, partial pressure of carbon dioxide in arterial blood (PaCO2 ) 40 mm Hg. What is the most appropriate action by the nurse? a. Document the results in the patient's record. b. Repeat the ABGs within an hour to validate the findings. c. Encourage deep breathing and coughing to open the alveoli. d. Initiate pulse oximetry for continuous monitoring of the patient's oxygen

a. Document the results in the patient's record. Normal findings in arterial blood gases (ABGs) in the older adult include a small decrease in PaO2 and arterial oxygen saturation (SaO2 ) but normal pH and PaCO2 . No interventions are necessary for these findings. Usual PaO2 levels are expected in patients 60 years of age or younger.

A patient has a WBC count of 2300/µL and a neutrophil percentage of 40%. Answer the following questions. a. Does the patient have leukopenia? b. What is the patient's absolute neutrophil count? c. Does the patient have neutropenia? d. Is the patient at risk for developing a bacterial infection? If so, why?

a. Does the patient have leukopenia? Yes, as the WBC count is below 4000/μL. b. The absolute neutrophil count (ANC) is 2300 × 40% = 920/μL. c. Yes, as the ANC is <1000/μL. d. Yes, the patient is at moderate risk of infection with opportunistic pathogens and nonpathogenic organisms from normal body flora because normal phagocytic mechanisms are impaired.

The patient with chronic HF is being discharged with a diuretic, a renin-angiotensinaldosterone system (RAAS) inhibitor, and a β-adrenergic blocker. When received from the pharmacy, which medication should not be included for this patient? a. Dopamine b. Losartan (Cozaar) c. Carvedilol (Coreg) d. Hydrochlorothiazide

a. Dopamine Dopamine is a β-adrenergic agonist that is a positive inotrope given IV, not orally, and used for acute HF. Losartan (Cozaar) is an angiotensin II receptor blocker used for patients who do not tolerate ACE inhibitors. Carvedilol (Coreg) is the β-adrenergic blocker that blocks the sympathetic nervous system's negative effects on the failing heart. Hydrochlorothiazide is the diuretic.

A nurse is preparing the transfusion of a unit of blood to a patient with anemia. The nurse knows which intervention is most important to prevent a transfusion reaction? a. Double-checking the blood product's type for compatibility with the patient's blood type b. Use new tubing 0.9% sodium chloride c. Assess vital signs d. Obtain blood & urine specimen

a. Double-checking the blood product's type for compatibility with the patient's blood type

A patient with newly discovered high BP has an average reading of 158/98 mm Hg after 3 months of exercise and diet modifications. Which management strategy will be a priority for this patient? a. Drug therapy will be needed because the BP is still not at goal. b. BP monitoring should continue for 3 months to confirm a diagnosis of hypertension. c. Lifestyle changes are less important since they were not effective, and drugs will be started. d. More changes in the patient's lifestyle are needed for a longer time before starting drug therapy.

a. Drug therapy will be needed because the BP is still not at goal.

The patient comes to the HCP office with pain, edema, and warm skin on her lower left leg. What test should the nurse expect to be ordered first? a. Duplex ultrasound b. Complete blood count (CBC) c. Magnetic resonance angiography d. Computed venography (phlebogram)

a. Duplex ultrasound With manifestations of a VTE, the Duplex ultrasound is most widely used to diagnose VTE by identifying where a thrombus is found and its extent. D-dimer may also be drawn to determine if a VTE exists.

When performing discharge teaching, what should the nurse teach the patient with any type of CMP to do (select all that apply)? a. Eat a low-sodium diet. b. Go to the gym every day. c. Engage in stress reduction activities. d. Abstain from alcohol and caffeine intake. e. Avoid strenuous activity and allow for periods of rest. f. Suggest that caregivers learn cardiopulmonary resuscitation (CPR).

a. Eat a low-sodium diet. c. Engage in stress reduction activities. d. Abstain from alcohol and caffeine intake. e. Avoid strenuous activity and allow for periods of rest. f. Suggest that caregivers learn cardiopulmonary resuscitation (CPR). These topics can apply to any patient with CMP. Going to the gym could only be included within the exercise guidelines from the HCP and balanced with rest.

When providing education to a client who is prescribed anti-platelet medication, which signs should the client monitor for regularly? Select all that apply. a. Epistaxis b. Petechiae c. Hematuria d. Infection signs e. Vision changes

a. Epistaxis b. Petechiae c. Hematuria This answer is correct because epistaxis is a nosebleed. Antiplatelets can result in thrombocytopenia. This can cause bleeding from the nose. This is why it is important to monitor for epistaxis. This answer is correct because petechiae appears on the skin when platelets are low. Petechiae are tiny, purplish red spots. This is due from bleeding underneath the skin from the capillaries. This answer is correct because low platelets can cause hematuria. Hematuria is blood in the urine. It can occur due to low platelets.

Which function test fits description "Amount of air exhaled in first second of forced vital capacity"? a. FEV1 b. TLC c. Vt d. VC

a. FEV1 Forced expiratory volume in first second of expiration

Which function test fits description "Amount of air that can be quickly and forcefully exhaled after maximum inspiration"? a. FVC b. PEFR c. RV d. FRC

a. FVC Forced vital capacity

Which factor should be considered when caring for a woman with suspected coronary artery disease? a. Fatigue may be the first symptom. b. Classic signs and symptoms are expected. c. Increased risk is present before menopause. d.Women are more likely to develop collateral circulation.

a. Fatigue may be the first symptom. Fatigue, rather than pain or shortness of breath, may be the first symptom of impaired cardiac circulation. Women may not exhibit the classic signs and symptoms of ischemia such as chest pain which radiates down the left arm. Neck, throat, or back pain may be symptoms experienced by women. Risk for coronary artery disease increases four times after menopause. Men are more likely to develop collateral circulation.

Risk Factor for or Response to Respiratory Problem: Tobacco use history, gradual change in health status, family history of lung disease, sputum production, no immunizations for influenza or pneumococcal pneumonia received, travel to developing countries. Which Functional Health Pattern does it fall under? a. Health perception- health management b. Nutritional-metabolic c. Elimination d. Activity-exercise

a. Health perception- health management

A client arrives at the emergency department with a hemoglobin of 6. The health care provider (HCP) orders 2 units of packed red blood cells (PRBCs) to be infused each over 4 hours. The nurse starts the first unit of PRBCs and within the first 15 minutes, the client begins to flush, redness to the face, and begins itching. Which type of infusion reaction is the client exhibiting? a. Febrile reaction. b. Anaphylactic reaction. c. Circulatory overload. d. Hemolytic reaction.

a. Febrile reaction. This answer is correct because when a client has a febrile reaction, the client has flushing, redness, and itchy skin. Usually, the health care provider (HCP) will order tylenol and benadryl for the client and continue the transfusion.

What should the nurse inspect when assessing a patient with shortness of breath for evidence of long-standing hypoxemia? a. Fingernails b. Chest excursion c. Spinal curvatures d. Respiratory pattern

a. Fingernails Clubbing, a sign of long-standing hypoxemia, is evidenced by an increase in the angle between the base of the nail and fingernail to 180 degrees or more, usually accompanied by an increase in the depth, bulk, and sponginess of the end of the finger.

Which of the following descriptions matches the mechanism of "osmotic pressure"? a. Force determined by osmolality of a fluid b. Pressure exerted by plasma proteins c. Force exerted by a fluid d. Uses a protein carrier molecule

a. Force determined by osmolality of a fluid

What is the most important method to identify the presence of infection in a neutropenic patient? a. Frequent temperature monitoring b. Routine blood and sputum cultures c. Assessing for redness and swelling d. Monitoring WBC count

a. Frequent temperature monitoring An elevated temperature is of most significance in recognizing the presence of an infection in the neutropenic patient because there is little leukocytic response to injury with low WBC count. Minor reports of pain or other symptoms should also be reported. Monitoring WBC count will not identify infection. Cultures are indicated if the temperature is elevated but are not used to monitor for infection.

Pneumonia nursing intervention includes (select all that apply): a. Full course of antibiotics b. Drink plenty of fluids c. Maintain supine position d. Deep breathing exercises

a. Full course of antibiotics b. Drink plenty of fluids d. Deep breathing exercises

A patient who has recently started taking pravastatin (Pravachol) and niacin (Nicobid) reports the following symptoms to the nurse. Which is most important to communicate to the health care provider? a. Generalized muscle aches and pains b. Dizziness when changing positions quickly c. Nausea when taking the drugs before eating d. Flushing and pruritus after taking the medications

a. Generalized muscle aches and pains Muscle aches and pains may indicate myopathy and rhabdomyolysis, which have caused acute kidney injury and death in some patients who have taken the statin medications. These symptoms indicate that the pravastatin may need to be discontinued. The other symptoms are common side effects when taking niacin, and although the nurse should follow-up with the health care provider, they do not indicate that a change in medication is needed.

When caring for a patient who has just arrived on the medical-surgical unit after having cardiac catheterization, which nursing intervention should the nurse delegate to a licensed practical/vocational nurse (LPN/LVN)? a. Give the scheduled aspirin and lipid-lowering medication. b. Perform the initial assessment of the catheter insertion site. c. Teach the patient about the usual postprocedure plan of care. d. Titrate the heparin infusion according to the agency protocol.

a. Give the scheduled aspirin and lipid-lowering medication. Administration of oral medications is within the scope of practice for LPNs/LVNs. The initial assessment of the patient, patient teaching, and titration of IV anticoagulant medications should be done by the registered nurse (RN).

Which is a priority nursing intervention for a patient during the acute phase of rheumatic fever? a. Giving IV antibiotics as prescribed b. Managing pain with opioid analgesics c. Encouraging fluid intake for hydration d. Performing frequent active range-of-motion exercises

a. Giving IV antibiotics as prescribed

Which activity should the nurse and patient identify as a moderate-energy activity during rehabilitation after an MI? a. Golfing b. Walking at 5 mph c. Cycling at 13 mph d. Mowing the lawn by hand

a. Golfing Golfing is a moderate-energy activity that expends about 5 metabolic equivalent units (METs). It is within the 3 to 6 METs activity level desired for a patient by the time of discharge from the hospital following an MI. Walking at 5 mph and mowing the lawn by hand are high-energy activities. Cycling at 13 mph is an extremely high-energy activity.

A nurse is teaching a client about dietary modification to control blood pressure. Which of the following food choices should the nurse identify as an indication that the client understands instructions? a. Grilled chicken salad, tomatoes b. Beef vegetable soup and salad c. Chick fila grilled nuggets, fries d. Vegetarian wrap with lays potato chips

a. Grilled chicken salad, tomatoes

A nurse is educating a patient with iron deficiency on foods high in iron. Which meal, if chosen by the patient, demonstrates an understanding of iron-rich foods? a. Grilled chicken thigh, sautéed spinach, and wholegrain bread b. Friend Chicken, yellow vegetables, fruit juice c. Broiled fish, green vegetables, milk d. Grilled cheese sandwich, creamed soup, tomato salad

a. Grilled chicken thigh, sautéed spinach, and wholegrain bread

A patient in the clinic with cystic fibrosis (CF) reports increased sweating and weakness during the summer months. Which action by the nurse would be most appropriate? a. Have the patient add dietary salt to meals. b. Teach the patient about the signs of hypoglycemia. c. Suggest decreasing intake of dietary fat and calories. d. Instruct the patient about pancreatic enzyme replacements.

a. Have the patient add dietary salt to meals. Added dietary salt is indicated whenever sweating is excessive, such as during hot weather, when fever is present, or from intense physical activity. The management of pancreatic insufficiency includes pancreatic enzyme replacement of lipase, protease, and amylase (e.g., Pancreaze, Creon, Ultresa, Zenpep) administered before each meal and snack. This patient is at risk for hyponatremia based on reported symptoms. Adequate intake of fat, calories, protein, and vitamins is important. Fat-soluble vitamins (vitamins A, D, E, and K) must be supplemented because they are malabsorbed. Use of caloric supplements improves nutritional status. Hyperglycemia due to pancreatic insufficiency is more likely to occur than hypoglycemia.

The nurse reviews the blood reports of a patient who is receiving epoetin alfa (Epogen) and finds that the medication is effective. Which finding in the blood lab report enables the nurse to reach this conclusion? a. Hemoglobin levels of 15 g/dL b. Hemoglobin levels of 6 g/dL c. Hemoglobin levels of 10 g/dL d. Hemoglobin levels of 11 g/dL

a. Hemoglobin levels of 15 g/dL a normal or above hemoglobin level (anything over 12)

A nurse is assessing someone with pericarditis. What is she expected to see? a. Hiccups, dyspnea b. Chest pain when sitting upright c. Relief of chest pain with deep inhalation d. Bradycardia with st elevation

a. Hiccups, dyspnea

Based on the Joint Commission Core Measures for patients with heart failure, which topics should the nurse include in the discharge teaching plan for a patient who has been hospitalized with chronic heart failure (select all that apply)? a. How to take and record daily weight b. Importance of limiting aerobic exercise c. Date and time of follow-up appointment d. Symptoms indicating worsening heart failure e. Actions and side effects of prescribed medications

a. How to take and record daily weight c. Date and time of follow-up appointment d. Symptoms indicating worsening heart failure e. Actions and side effects of prescribed medications The Joint Commission Core Measures state that patients should be taught about prescribed medications, follow-up appointments, weight monitoring, and actions to take for worsening symptoms. Patients with heart failure are encouraged to begin or continue aerobic exercises such as walking, while self-monitoring to avoid excessive fatigue.

A nurse is assessing a client who has hypomagnesemia. Which of the following findings should the nurse expect? a. Hyperactive deep-tendon reflexes b. Increased bowel sounds c. Drowsiness d. Decreased blood pressure

a. Hyperactive deep-tendon reflexes Hyperactive deep-tendon reflexes are an expected finding for a client who has hypomagnesemia, along with muscle cramps, numbness, and tingling.

A 67-year-old patient is admitted to the hospital with a diagnosis of venous insufficiency. Which patient statement is most supportive of the diagnosis? a. I cant get my shoes on at the end of the day. b. I cant seem to ever get my feet warm enough. c. I have burning leg pains after I walk two blocks. d. I wake up during the night because my legs hurt.

a. I cant get my shoes on at the end of the day. Because the edema associated with venous insufficiency increases when the patient has been standing, shoes will feel tighter at the end of the day. The other patient statements are characteristic of peripheral artery disease (PAD).

Which patient statement to the nurse indicates a need for additional instruction about taking oral ferrous sulfate? a. I will call my health care provider if my stools turn black. b. I will take a stool softener if I feel constipated occasionally. c. I should take the iron with orange juice about an hour before eating. d. I should increase my fluid and fiber intake while I am taking iron tablets.

a. I will call my health care provider if my stools turn black. It is normal for the stools to appear black when a patient is taking iron, and the patient should not call the doctor about this. The other patient statements are correct.

A patient is scheduled for exercise nuclear imaging stress testing. The nurse explains to the patient that this test involves a. IV administration of a radioisotope at the maximum heart rate during exercise to identify the heart's response to physical stress. b. placement of electrodes inside the right-sided heart chambers through a vein to record the electrical activity of the heart directly. c. exercising on a treadmill or stationary bicycle with continuous ECG monitoring to detect ischemic changes in the heart during exercise. d. placement of a small transducer in 4 positions on the chest to record the direction and flow of blood through the heart by the reflection of sound waves.

a. IV administration of a radioisotope at the maximum heart rate during exercise to identify the heart's response to physical stress. In an exercise nuclear imaging scan, a radioisotope is injected at the maximum heart rate on a bicycle or treadmill and used to evaluate blood flow in different parts of the heart. Insertion of electrodes into the heart chambers via the venous system to record intracardiac electrical activity is an electrophysiology study. Simply monitoring electrocardiogram (ECG) activity during exercise is an exercise stress test, and an echocardiogram uses transducers to bounce sound waves off of the heart.

When the patient who is diagnosed with an MI is not relieved of chest pain with IV NTG, which medication will the nurse expect to be used? a. IV morphine sulfate b. Calcium channel blockers c. IV administration of amiodarone d. Angiotensin-converting enzyme (ACE) inhibitors

a. IV morphine sulfate Morphine sulfate decreases anxiety and cardiac workload as a vasodilator and reduces preload and myocardial O2 consumption, which relieves chest pain. Calcium channel blockers, amiodarone, and angiotensin-converting enzyme (ACE) inhibitors will not relieve chest pain related to an MI.

A patient with a type A dissection of the arch of the aorta has a decreased LOC and weak carotid pulses. What should the nurse anticipate that initial treatment of the patient will include? a. Immediate surgery to replace the torn area with a graft b. Administration of anticoagulants to prevent embolization c. Administration of packed red blood cells (RBCs) to replace blood loss d. Giving antihypertensives to maintain a mean arterial pressure of 70 to 80 mm Hg

a. Immediate surgery to replace the torn area with a graft Immediate surgery is indicated when complications (such as occlusion of the carotid arteries) occur. Otherwise, initial treatment for aortic dissection involves a period of lowering the BP and myocardial contractility to diminish the pulsatile forces in the aorta. Anticoagulants would prolong and intensify the bleeding. Blood is given only if the dissection ruptures

While reviewing a client's laboratory results, a nurse notes a serum calcium level of 0.8 mg/dL. Which of the following actions should the nurse take? a. Implement seizure precautions. b. Administer phosphate. c. Initiate diuretic therapy. d. Prepare the client for hemodialysis.

a. Implement seizure precautions The client is at risk for seizures due to low excitation threshold as a result of the client's decreased calcium level. The nurse should initiate seizure precautions to prevent injury.

Which microorganisms are associated with both CAP and HAP (select all that apply)? a. Klebsiella b. Acinetobacter c. Staphylococcus aureus d. Mycoplasma pneumoniae e. Pseudomonas aeruginosa f. Streptococcus pneumoniae

a. Klebsiella b. Acinetobacter c. Staphylococcus aureus e. Pseudomonas aeruginosa f. Streptococcus pneumoniae CAP and HAP are both associated with Klebsiella, Staphylococcus aureus, Pseudomonas aeruginosa, and Streptococcus pneumoniae. Acinetobacter is only associated with HAP. Mycoplasma pneumoniae is only associated with CAP

When teaching the patient about a new prescription for oral iron supplements, what does the nurse teach the patient to do? a. Increase fluid and dietary fiber intake. b. Take the iron preparations with meals. c. Use enteric-coated preparations taken with orange juice. d. Report the presence of black stools to the health care provider.

a. Increase fluid and dietary fiber intake. Constipation is a common side effect of oral iron supplementation, and increased fluids and fiber should be consumed to prevent this effect. Because iron is best absorbed in an acid environment and can be bound in the gastrointestinal (GI) tract by food, it should be taken before meals, unless gastric side effects of the supplements necessitate its ingestion with food. Taking iron with ascorbic acid or orange juice enhances absorption of the iron, but enteric-coated iron often is ineffective because of unpredictable release of the iron in areas of the GI tract where it can be absorbed. Black stools are an expected result of oral iron preparations.

The home health nurse cares for an alert and oriented older adult patient with a history of dehydration. Which instructions should the nurse give to this patient related to fluid intake? a. Increase fluids if your mouth feels dry. b. More fluids are needed if you feel thirsty. c. Drink more fluids in the late evening hours. d. If you feel lethargic or confused, you need more to drink.

a. Increase fluids if your mouth feels dry. An alert, older patient will be able to self-assess for signs of oral dryness such as thick oral secretions or dry-appearing mucosa. The thirst mechanism decreases with age and is not an accurate indicator of volume depletion. Many older patients prefer to restrict fluids slightly in the evening to improve sleep quality. The patient will not be likely to notice and act appropriately when changes in level of consciousness occur.

After the nurse gives IV atropine to a patient with symptomatic type 1, second-degree atrioventricular (AV) block, which finding indicates that the medication has been effective? a. Increase in the patients heart rate b. Increase in strength of peripheral pulses c. Decrease in premature atrial contractions d. Decrease in premature ventricular contractions

a. Increase in the patients heart rate Atropine will increase the heart rate and conduction through the AV node. Because the medication increases electrical conduction, not cardiac contractility, the quality of the peripheral pulses is not used to evaluate the drug effectiveness. The patient does not have premature atrial or ventricular contractions.

After the nurse gives IV atropine to a patient with symptomatic type 1, second-degree atrioventricular (AV) block, which finding indicates that the medication has been effective? a. Increase in the patients heart rate b. Increase in strength of peripheral pulses c. Decrease in premature atrial contractions d. Decrease in premature ventricular contractions

a. Increase in the patients heart rate Atropine will increase the heart rate and conduction through the AV node. Because the medication increases electrical conduction, not cardiac contractility, the quality of the peripheral pulses is not used to evaluate the drug effectiveness. The patient does not have premature atrial or ventricular contractions.

The nurse assesses the chest of a patient with pneumococcal pneumonia. Which finding would the nurse expect? a. Increased tactile fremitus b. Dry, nonproductive cough c. Hyperresonance to percussion d. A grating sound on auscultation

a. Increased tactile fremitus Increased tactile fremitus over the area of pulmonary consolidation is expected with bacterial pneumonias. Dullness to percussion would be expected. Pneumococcal pneumonia typically presents with a loose, productive cough. Adventitious breath sounds such as crackles and wheezes are typical. A grating sound is more representative of a pleural friction rub rather than pneumonia.

Three days after experiencing a myocardial infarction (MI), a patient who is scheduled for discharge asks for assistance with hygiene activities, saying, I am too nervous to take care of myself. Based on this information, which nursing diagnosis is appropriate? a. Ineffective coping related to anxiety b. Activity intolerance related to weakness c. Denial related to lack of acceptance of the MI d. Disturbed personal identity related to understanding of illness

a. Ineffective coping related to anxiety The patient data indicate that ineffective coping after the MI caused by anxiety about the impact of the MI is a concern. The other nursing diagnoses may be appropriate for some patients after an MI, but the data for this patient do not support denial, activity intolerance, or disturbed personal identity.

The nurse plans long-term goals for the patient who has had a heart transplant with the knowledge that what is the most common cause of death in heart transplant patients during the first year? a. Infection b. HF c. Embolization d. Malignant conditions

a. Infection In the first year after transplant, with the need for long-term immunosuppressant therapy to prevent rejection, the patient with a transplant is at high risk for infection, a leading cause of death in transplant patients. Acute rejection episodes may also cause death in patients with transplants, but many can be treated successfully with augmented immunosuppressive therapy. Malignancies occur in patients with organ transplants after taking immunosuppressants for a number of years.

A postoperative patient who had surgery for a perforated gastric ulcer has been receiving nasogastric suction for 3 days. The patient now has a serum sodium level of 127 mEq/L (127 mmol/L). Which prescribed therapy should the nurse question? a. Infuse 5% dextrose in water at 125 mL/hr. b. Administer IV morphine sulfate 4 mg every 2 hours PRN. c. Give IV metoclopramide (Reglan) 10 mg every 6 hours PRN for nausea. d. Administer 3% saline if serum sodium decreases to less than 128 mEq/L.

a. Infuse 5% dextrose in water at 125 mL/hr. Because the patients gastric suction has been depleting electrolytes, the IV solution should include electrolyte replacement. Solutions such as lactated Ringers solution would usually be ordered for this patient. The other orders are appropriate for a postoperative patient with gastric suction.

A nurse is caring for a patient with type AB blood who requires a blood transfusion. However, the blood bank sends up type A blood. Which action should the nurse take first? a. Infuse blood according to agency protocol b. Do not infuse blood c. Call the doctor d. Wait 4 hours to infuse blood

a. Infuse blood according to agency protocol

A patient is diagnosed with mitral stenosis and new-onset atrial fibrillation. Which interventions could the nurse delegate to unlicensed assistive personnel (UAP)? (select all that apply) a. Obtain and record daily weight. b. Determine apical-radial pulse rate. c. Observe for overt signs of bleeding. d. Teach the patient how to get a Medic Alert device. e. Obtain and record vital signs, including pulse oximetry.

a. Obtain and record daily weight. c. Observe for overt signs of bleeding. e. Obtain and record vital signs, including pulse oximetry.

Which action by a new registered nurse (RN) who is orienting to the progressive care unit indicates a good understanding of the treatment of cardiac dysrhythmias? a. Injects IV adenosine (Adenocard) over 2 seconds to a patient with supraventricular tachycardia b. Obtains the defibrillator and quickly brings it to the bedside of a patient whose monitor shows asystole c. Turns the synchronizer switch to the on position before defibrillating a patient with ventricular fibrillation d. Gives the prescribed dose of diltiazem (Cardizem) to a patient with new-onset type II second degree AV block

a. Injects IV adenosine (Adenocard) over 2 seconds to a patient with supraventricular tachycardia Adenosine must be given over 1 to 2 seconds to be effective. The other actions indicate a need for more education about treatment of cardiac dysrhythmias. The RN should hold the diltiazem until talking to the health care provider. The treatment for asystole is immediate CPR. The synchronizer switch should be off when defibrillating.

On auscultation of a patients lungs, the nurse hears low-pitched, bubbling sounds during inhalation in the lower third of both lungs. How should the nurse document this finding? a. Inspiratory crackles at the bases b. Expiratory wheezes in both lungs c. Abnormal lung sounds in the apices of both lungs d. Pleural friction rub in the right and left lower lobes

a. Inspiratory crackles at the bases Crackles are low-pitched, bubbling sounds usually heard on inspiration. Wheezes are high-pitched sounds. They can be heard during the expiratory or inspiratory phase of the respiratory cycle. The lower third of both lungs are the bases, not apices. Pleural friction rubs are grating sounds that are usually heard during both inspiration and expiration

Which action will the nurse include in the plan of care for a patient who was admitted with syncopal episodes of unknown origin? a. Instruct the patient to call for assistance before getting out of bed. b. Explain the association between various dysrhythmias and syncope. c. Educate the patient about the need to avoid caffeine and other stimulants. d. Tell the patient about the benefits of implantable cardioverter-defibrillators.

a. Instruct the patient to call for assistance before getting out of bed. A patient with fainting episodes is at risk for falls. The nurse will plan to minimize the risk by having assistance whenever the patient up. The other actions may be needed if dysrhythmias are found to be the cause of the patients syncope, but are not appropriate for syncope of unknown origin.

The nurse is admitting a patient to the clinical unit from surgery. Being alert to potential fluid volume alterations, what assessment data will be important for the nurse to monitor to identify early changes in the patient's postoperative fluid volume (select all that apply)? a. Intake and output b. Skin turgor c. Lung sounds d. Respiratory rate e. Level of consciousness

a. Intake and output b. Skin turgor c. Lung sounds d. Respiratory rate e. Level of consciousness All of these are important in assessing fluid balance in a postoperative patient. Daily weight along with these assessments will provide data about potential fluid volume abnormalities.

Priority Decision: A patient is undergoing a contrast CT scan of the spleen. What is most important for the nurse to ask the patient about before the test? a. Iodine sensitivity b. Prior blood transfusions c. Phobia of confined spaces d. Internal metal implants or appliances

a. Iodine sensitivity A contrast CT scan involves the use of an iodine-based dye that could cause a reaction if the patient is sensitive to iodine. Metal implants or internal appliances and claustrophobia should be determined before MRI. Prior blood transfusions are not a factor in this diagnostic test.

Which nutrients are essential for red blood cell (RBC) production (select all that apply)? a. Iron b. Folic acid c. Vitamin C d. Vitamin D e. Vitamin B12 f. Carbohydrates

a. Iron b. Folic acid e. Vitamin B12 Although all of the listed nutrients are helpful, iron, folic acid, and cobalamin (vitamin B12 ) are essential for erythropoiesis.

The nurse understands which is the most common type of anemia? a. Iron-deficiency anemia b. Sickle cell anemia c. Pernicious Anemia d. Pulmonary embolism

a. Iron-deficiency anemia

When assessing a patient's sleep-rest pattern related to respiratory health, what should the nurse ask the patient? (Select all that apply.) a. Is it hard for you to fall asleep? b. Do you awaken abruptly during the night? c. Do you sleep more than 8 hours per night? d. Do you need to sleep with the head elevated? e. Do you often need to urinate during the night?

a. Is it hard for you to fall asleep? b. Do you awaken abruptly during the night? d. Do you need to sleep with the head elevated? A patient with obstructive sleep apnea may have insomnia, abrupt awakenings, or both. Patients with cardiovascular disease (e.g., heart failure that may affect respiratory health) may need to sleep with the head elevated on several pillows (orthopnea). Sleeping more than 8 hours per night or needing to urinate during the night is not indicative of impaired respiratory health.

Which medications would be used in 4-drug treatment for the initial phase of TB (select all that apply)? a. Isoniazid b. Levofloxacin c. Pyrazinamide d. Rifampin (Rifadin) e. Rifabutin (Mycobutin) f. Ethambutol (Myambutol)

a. Isoniazid c. Pyrazinamide d. Rifampin (Rifadin) f. Ethambutol (Myambutol) For the first 2 months, a 4-drug regimen consists of isoniazid, pyrazinamide, rifampin (Rifadin), and ethambutol (Myambutol). Rifabutin (Mycobutin) and levofloxacin may be used if the patient develops toxicity to the primary drugs. Rifabutin may be used as first-line treatment for patients receiving medications that interact with rifampin (e.g., antiretrovirals, estradiol, warfarin).

A client who has been homeless and has spent the past 6 months living in shelters has been diagnosed with confirmed tuberculosis (TB). Which medications does the nurse expect to be ordered for the client? a .Isoniazid (INH), rifampin (Rifadin), pyrazinamide (Zinamide), ethambutol (Myambutol) b. Metronidazole (Flagyl), acyclovir (Zovirax), flunisolide (AeroBid), rifampin (Rifadin) c. Prednisone (Prednisone), guaifenesin (Organidin), ketorolac (Toradol), pyrazinamide (Zinamide) d. Salmeterol (Serevent), cromolyn sodium (Intal), dexamethasone (Decadron), isoniazid (INH)

a. Isoniazid (INH), rifampin (Rifadin), pyrazinamide (Zinamide), ethambutol (Myambutol) The combination of isoniazid, rifampin, pyrazinamide, and ethambutol is used to treat TB. Metronidazole is used to treat anaerobic bacteria and some parasites, but is not effective against TB. Acyclovir is used to treat viral infection. Flunisolide is a corticosteroid that is useful in asthma or other airway disease to prevent wheezing. Prednisone is a steroid. Guaifenesin is a mucolytic. Ketorolac is a nonsteroidal anti-inflammatory drug that is used for short-term pain relief. Salmeterol and cromolyn sodium would most likely be given to clients with respiratory difficulties such as poorly controlled asthma from allergic sources. Dexamethasone is a steroid.

When teaching the patient with angina about taking sublingual NTG tablets, what should the nurse teach the patient? a. Lie or sit and place 1 tablet under the tongue when chest pain occurs. b. Take the tablet with a large amount of water so that it will dissolve right away. c. If 1 tablet does not relieve the pain in 15 minutes, the patient should go to the hospital. d. If the tablet causes dizziness and a headache, stop the medication and call the doctor or go to the hospital.

a. Lie or sit and place 1 tablet under the tongue when chest pain occurs. A common complication of nitrates is dizziness caused by orthostatic hypotension, so the patient should sit or lie down and place the tablet under the tongue. The tablet should be allowed to dissolve under the tongue. To prevent the tablet from being swallowed, water should not be taken with it. The recommended dose for the patient for whom nitroglycerin (NTG) has been prescribed is 1 tablet taken sublingually (SL) or 1 metered spray for symptoms of angina. If symptoms are unchanged or worse after 5 minutes, the patient should contact the emergency response system (ERS) before taking more NTG. If symptoms are significantly improved by 1 dose of NTG, teach the patient or caregiver to repeat NTG every 5 minutes for a maximum of 3 doses and contact ERS if symptoms have not resolved completely. Headache is a common complication of nitrates but usually resolves with continued use of nitrates. It may be controlled with mild analgesics.

A patient who is experiencing an acute asthma attack is admitted to the emergency department. Which assessment should the nurse complete first? a. Listen to the patients breath sounds. b. Ask about inhaled corticosteroid use. c. Determine when the dyspnea started. d. Obtain the forced expiratory volume (FEV) flow rate.

a. Listen to the patients breath sounds. Assessment of the patients breath sounds will help determine how effectively the patient is ventilating and whether rapid intubation may be necessary. The length of time the attack has persisted is not as important as determining the patients status at present. Most patients having an acute attack will be unable to cooperate with an FEV measurement. It is important to know about the medications the patient is using but not as important as assessing the breath sounds.

A patient who was in a car accident had abdominal trauma. Which organs may be damaged and contribute to altered function of the hematologic system (select all that apply)? a. Liver b. Spleen c. Stomach d. Gallbladder e. Lymph nodes

a. Liver b. Spleen e. Lymph nodes The abdominal organs that are primarily involved in hematologic function are the liver, spleen, and lymph nodes. The liver filters the blood, produces procoagulants, and stores iron. The spleen's functions are hematopoietic (RBCs produced during fetal development); filtration (removes old and defective erythrocytes, iron for reuse, and bacteria); immunologic (supplies lymphocytes, monocytes, and stored immunoglobulins); and storage (RBCs and platelets). The lymph nodes filter pathogens and foreign particles from lymphatic circulation.

What is a possible cause for palpation abnormal finding "increased tactile fremitus"? a. Lung consolidation with fluid or exudate b. Air trapping c. Atelactasis d. Interstitial edema

a. Lung consolidation with fluid or exudate

A patient receives 3% NaCl solution for correction of hyponatremia. Which assessment is most important for the nurse to monitor for while the patient is receiving this infusion? a. Lung sounds b. Urinary output c. Peripheral pulses d. Peripheral edema

a. Lung sounds Hypertonic solutions cause water retention, so the patient should be monitored for symptoms of fluid excess. Crackles in the lungs may indicate the onset of pulmonary edema and are a serious manifestation of fluid excess. Bounding peripheral pulses, peripheral edema, or changes in urine output are also important to monitor when administering hypertonic solutions, but they do not indicate acute respiratory or cardiac decompensation.

After a woman had a right breast mastectomy, her right arm became severely swollen. What hematologic problem caused this? a. Lymphedema b. Right-sided heart failure c. Wound on her right hand d. Refusal to use her right arm

a. Lymphedema Lymphedema is the obstruction of lymph flow that results in accumulation of lymph fluid for the patient in the right arm following a right-sided breast mastectomy. The other options are not hematologic problems that would cause extreme swelling.

How does myelodysplastic syndrome (MDS) differ from acute leukemias? a. MDS has a slower disease progression. b. MDS does not result in bone marrow failure. c. MDS is a clonal disorder of hematopoietic cells. d. MDS affects only the production and function of platelets and WBCs.

a. MDS has a slower disease progression. Myelodysplastic syndromes, like leukemias, are a group of disorders in which hematopoietic stem cells of the bone marrow undergo clonal change and may cause eventual bone marrow failure. However, the primary difference from leukemias is that myelodysplastic cells have some degree of maturation, and the disease progression is slower than in acute leukemias.

The nurse is caring for a postoperative patient with impaired airway clearance. What nursing actions would promote airway clearance? (Select all that apply.) a. Maintain adequate fluid intake. b. Maintain a 15-degree elevation. c. Splint the chest when coughing. d. Have the patient use incentive spirometry. e. Teach the patient to cough at end of exhalation.

a. Maintain adequate fluid intake. c. Splint the chest when coughing. e. Teach the patient to cough at end of exhalation. Maintaining adequate fluid intake liquefies secretions, allowing easier expectoration. The nurse should teach the patient to splint the chest while coughing. This will reduce discomfort and allow for a more effective cough. Coughing at the end of exhalation promotes a more effective cough. Incentive spirometry promotes lung expansion. The patient should be positioned in an upright sitting position (high Fowler's) with head slightly flexed.

A patient is admitted to the hospital in a hypertensive emergency (BP 244/142 mm Hg). Sodium nitroprusside is started to treat the elevated BP. Which management strategies would be most appropriate for this patient? (select all that apply) a. Measuring hourly urine output b. Continuous BP monitoring with an arterial line c. Decreasing the MAP by 50% within the first hour d. Maintaining bed rest and giving tranquilizers to lower the BP e. Assessing the patient for signs and symptoms of heart failure and changes in mental status

a. Measuring hourly urine output b. Continuous BP monitoring with an arterial line e. Assessing the patient for signs and symptoms of heart failure and changes in mental status

Discharge teaching for a TB patient should include (select all that apply): a. Medication compliance b. Follow up visits with HCP c. Follow a low calorie, high protein diet d. Keep oral fluids to a minimum

a. Medication compliance b. Follow up visits with HCP

A patient with newly discovered high BP has an average reading of 158/98 mmHg after 3 months of exercise and diet modifications. Which management strategy will be a priority for this patient? a. Medication will be required because the BP is still not at goal b. BP monitoring should continue for another 3 months to confirm a diagnosis of hypertension c. Lifestyle changes are less important, since they were not effective, and medications will be started d. More vigorous changes in the patient's lifestyle are needed for a longer time before starting medications

a. Medication will be required because the BP is still not at goal

A patient who is lethargic and exhibits deep, rapid respirations has the following arterial blood gas (ABG) results: pH 7.32, PaO2 88 mm Hg, PaCO2 37 mm Hg, and HCO3 16 mEq/L. How should the nurse interpret these results? a. Metabolic acidosis b. Metabolic alkalosis c. Respiratory acidosis d. Respiratory alkalosis

a. Metabolic acidosis The pH and HCO3 indicate that the patient has a metabolic acidosis. The ABGs are inconsistent with the other responses.

Which of the following are the 3 classes of bronchodilators? (select all that apply) a. Methylxanthines (theophylline) b. Corticosteroids c. Anticholinergics d. Beta adrenergic agonists

a. Methylxanthines (theophylline) c. Anticholinergics d. Beta adrenergic agonists

Which action should the nurse include in the plan of care when caring for a patient admitted with acute decompensated heart failure (ADHF) who is receiving nesiritide (Natrecor)? a. Monitor blood pressure frequently. b. Encourage patient to ambulate in room. c. Titrate nesiritide slowly before stopping. d. Teach patient about home use of the drug.

a. Monitor blood pressure frequently. Nesiritide is a potent arterial and venous dilator, and the major adverse effect is hypotension. Because the patient is likely to have orthostatic hypotension, the patient should not be encouraged to ambulate. Nesiritide does not require titration and is used for ADHF but not in a home setting.

Which action will the nurse include in the plan of care for a 72-year-old woman admitted with multiple myeloma? a. Monitor fluid intake and output. b. Administer calcium supplements. c. Assess lymph nodes for enlargement. d. Limit weight bearing and ambulation.

a. Monitor fluid intake and output. A high fluid intake and urine output helps prevent the complications of kidney stones caused by hypercalcemia and renal failure caused by deposition of Bence-Jones protein in the renal tubules. Weight bearing and ambulation are encouraged to help bone retain calcium. Lymph nodes are not enlarged with multiple myeloma.

Which instruction is a key aspect of teaching for the patient on anticoagulant therapy? a. Monitor for and report any signs of bleeding. b. Do not take acetaminophen (Tylenol) for a headache. c. Decrease your dietary intake of foods containing vitamin K. d. Arrange to have blood drawn twice a week to check drug effects.

a. Monitor for and report any signs of bleeding.

The nurse notes a serum calcium level of 7.9 mg/dL for a patient who has chronic malnutrition. Which action should the nurse take next? a. Monitor ionized calcium level. b. Give oral calcium citrate tablets. c. Check parathyroid hormone level. d. Administer vitamin D supplements.

a. Monitor ionized calcium level. This patient with chronic malnutrition is likely to have a low serum albumin level, which will affect the total serum calcium. A more accurate reflection of calcium balance is the ionized calcium level. Most of the calcium in the blood is bound to protein (primarily albumin). Alterations in serum albumin levels affect the interpretation of total calcium levels. Low albumin levels result in a drop in the total calcium level, although the level of ionized calcium is not affected. The other actions may be needed if the ionized calcium is also decreased.

A patient with chronic heart failure and atrial fibrillation is treated with low-dose digitalis and a loop diuretic. What does the nurse need to do to prevent complications of this drug combination? (select all that apply) a. Monitor serum potassium levels. b. Teach the patient how to take a pulse rate. c. Withhold digitalis if pulse rhythm is irregular. d. Keep an accurate measure of intake and output. e. Teach the patient about dietary potassium restrictions.

a. Monitor serum potassium levels. b. Teach the patient how to take a pulse rate.

A patient with recurrent shortness of breath has just had a bronchoscopy. What is a priority nursing action immediately after the procedure? a. Monitor the patient for laryngeal edema. b. Assess the patient's level of consciousness. c. Monitor and manage the patient's level of pain. d. Assess the patient's heart rate and blood pressure.

a. Monitor the patient for laryngeal edema. Priorities for assessment are the patient's airway and breathing, both of which may be compromised after bronchoscopy by laryngeal edema. These assessment parameters supersede the importance of loss of consciousness (LOC), pain, heart rate, and blood pressure, although the nurse should also be assessing these.

Which nursing responsibilities are priorities when caring for a patient returning from a cardiac catheterization (select all that apply)? a. Monitoring vital signs and ECG b. Checking the catheter insertion site and distal pulses c. Helping the patient to ambulate to the bathroom to void d. Telling the patient that he will be sleepy from the general anesthesia e. Teaching the patient about the risks of the radioactive isotope injection

a. Monitoring vital signs and ECG b. Checking the catheter insertion site and distal pulses

The nurse expects which client to be in respiratory acidosis? (select all that apply) a. Morphine overdose b. Panic attack c. Sleep apnea d. COPD e. Asthma attack f. Alcohol intoxication

a. Morphine overdose c. Sleep apnea d. COPD e. Asthma attack f. Alcohol intoxication

Which statements accurately describe chronic lymphocytic leukemia (CLL) (select all that apply)? a. Most common leukemia of adults in Western countries b. Only cure is bone marrow transplant c. Neoplasm of activated B lymphocytes d. Increased incidence in survivors of atomic bombs e. Philadelphia chromosome is a diagnostic hallmark f. Mature-appearing but functionally inactive lymphocytes

a. Most common leukemia of adults in Western countries c. Neoplasm of activated B lymphocytes f. Mature-appearing but functionally inactive lymphocytes Chronic lymphocytic leukemia (CLL) is the most common leukemia in adults in Western countries. It is a neoplasm of activated B lymphocytes that are mature appearing but functionally inactive. As it progresses, pressure on nerves from enlarged lymph nodes causes pain and paralysis. Mediastinal node enlargement leads to pulmonary symptoms. The other characteristics are related to chronic myelogenous leukemia (CML).

What are characteristics of the phosphate buffer system (select all that apply)? a. Neutralizes a strong base to a weak base and water b. Resultant sodium biphosphate is eliminated by kidneys c. Free acid radicals dissociate into H+ and CO2 , buffering excess base d. Neutralizes a strong acid to yield sodium biphosphate, a weak acid, and salt e. Shifts chloride in and out of red blood cells in exchange for sodium bicarbonate, buffering both acids and bases

a. Neutralizes a strong base to a weak base and water b. Resultant sodium biphosphate is eliminated by kidneys d. Neutralizes a strong acid to yield sodium biphosphate, a weak acid, and salt Base neutralization to a weak base, water, and salt and elimination of sodium biphosphate by the kidneys are characteristic of the phosphate buffer system. Free acid radical dissociation is characteristic of the protein buffer system. Chloride shifting in and out of red blood cells is characteristic of the hemoglobin buffer system.

Which information about a patient who has been receiving thrombolytic therapy for an acute myocardial infarction (AMI) is most important for the nurse to communicate to the health care provider? a. No change in the patients chest pain b. An increase in troponin levels from baseline c. A large bruise at the patients IV insertion site d. A decrease in ST-segment elevation on the electrocardiogram

a. No change in the patients chest pain Continued chest pain suggests that the thrombolytic therapy is not effective and that other interventions such as percutaneous coronary intervention (PCI) may be needed. Bruising is a possible side effect of thrombolytic therapy, but it is not an indication that therapy should be discontinued. The decrease of the ST-segment elevation indicates that thrombolysis is occurring and perfusion is returning to the injured myocardium. An increase in troponin levels is expected with reperfusion and is related to the washout of cardiac markers into the circulation as the blocked vessel is opened.

What early manifestation(s) is the patient with primary hypertension likely to report? a. No symptoms b. Cardiac palpitations c. Dyspnea on exertion d. Dizziness and vertigo

a. No symptoms Hypertension is often asymptomatic, especially if it is mild or moderate, and has been called the silent killer. The absence of symptoms often leads to noncompliance with medical treatment and a lack of concern about the disease in patients. With severe hypertension, symptoms may include fatigue, palpitations, angina, dyspnea, and dizziness

A patient with infective endocarditis develops sudden left leg pain with pallor, paresthesia, and a loss of peripheral pulses. What should be the nurse's initial action? a. Notify the HCP of the change in perfusion. b. Start anticoagulant therapy with IV heparin. c. Elevate the leg to improve the venous return. d. Position the patient in reverse Trendelenburg.

a. Notify the HCP of the change in perfusion.

A patient who is experiencing an asthma attack develops bradycardia and a decrease in wheezing. Which action should the nurse take first? a. Notify the health care provider. b. Document changes in respiratory status. c. Encourage the patient to cough and deep breathe. d. Administer IV methylprednisolone (Solu-Medrol).

a. Notify the health care provider. The patients assessment indicates impending respiratory failure, and the nurse should prepare to assist with intubation and mechanical ventilation after notifying the health care provider. IV corticosteroids require several hours before having any effect on respiratory status. The patient will not be able to cough or deep breathe effectively. Documentation is not a priority at this time.

A patient with renal failure has been taking aluminum hydroxide/magnesium hydroxide suspension (Maalox) at home for indigestion. The patient arrives for outpatient hemodialysis and is unresponsive to questions and has decreased deep tendon reflexes. Which action should the dialysis nurse take first? a. Notify the patients' health care provider. b. Obtain an order to draw a potassium level. c. Review the magnesium level on the patient's chart. d. Teach the patient about the risk of magnesium-containing antacids

a. Notify the patients' health care provider. The health care provider should be notified immediately. The patient has a history and manifestations consistent with hypermagnesemia. The nurse should check the chart for a recent serum magnesium level and make sure that blood is sent to the laboratory for immediate electrolyte and chemistry determinations. Dialysis should correct the high magnesium levels. The patient needs teaching about the risks of taking magnesium-containing antacids. Monitoring of potassium levels also is important for patients with renal failure, but the patient's current symptoms are not consistent with hyperkalemia.

The nurse is caring for a patient immediately after repair of an abdominal aortic aneurysm. On assessment, the patient has absent popliteal, posterior tibial, and dorsalis pedis pulses. The legs are cool and mottled. Which action should the nurse take first? a. Notify the surgeon and anesthesiologist. b. Wrap both the legs in a warming blanket. c. Document the findings and recheck in 15 minutes. d. Compare findings to the preoperative assessment of the pulses.

a. Notify the surgeon and anesthesiologist. Lower extremity pulses may be absent for a short time after surgery because of vasospasm and hypothermia. Decreased or absent pulses together with a cool and mottled extremity may indicate embolization or graft occlusion. These findings should be reported to the physician immediately because this is an emergency situation. Because pulses are marked prior to surgery, the nurse would know whether pulses were present prior to surgery before notifying the health care providers about the absent pulses. Because the patients symptoms may indicate graft occlusion or multiple emboli and a possible need to return to surgery, it is not appropriate to wait 15 minutes before taking action. A warming blanket will not improve the circulation to the patients legs.

During an annual health assessment of a 66-year-old patient at the clinic, the patient tells the nurse he has not had the pneumonia vaccine. What should the nurse advise him about the best way for him to prevent pneumonia? a. Obtain a pneumococcal vaccine now and get a booster 12 months later. b. Seek medical care and antibiotic therapy for all upper respiratory infections. c. Obtain the pneumococcal vaccine if he is exposed to individuals with pneumonia. d. Obtain only the influenza vaccine every year because he should have immunity to the pneumococcus because of his age.

a. Obtain a pneumococcal vaccine now and get a booster 12 months later. He should receive his first dose of PCV13, followed at least 1 year later by a dose of PPSV23. Influenza vaccine should be taken each year. Antibiotic therapy is not appropriate for all upper respiratory infections unless secondary bacterial infections develop.

Which nursing action for a patient with chronic obstructive pulmonary disease (COPD) could the nurse delegate to experienced unlicensed assistive personnel (UAP)? a. Obtain oxygen saturation using pulse oximetry. b. Monitor for increased oxygen need with exercise. c. Teach the patient about safe use of oxygen at home. d. Adjust oxygen to keep saturation in prescribed parameters.

a. Obtain oxygen saturation using pulse oximetry. UAP can obtain oxygen saturation (after being trained and evaluated in the skill). The other actions require more education and a scope of practice that licensed practical/vocational nurses (LPN/LVNs) or registered nurses (RNs) would have.

An 80-year-old patient with a history of an abdominal aortic aneurysm arrives at the emergency department (ED) with severe back pain and absent pedal pulses. Which actions should the nurse take first? a. Obtain the blood pressure. b. Obtain blood for laboratory testing. c. Assess for the presence of an abdominal bruit. d. Determine any family history of kidney disease.

a. Obtain the blood pressure. Because the patient appears to be experiencing aortic dissection, the nurses first action should be to determine the hemodynamic status by assessing blood pressure. The other actions also may be done, but they will not provide information that will determine what interventions are needed immediately for this patient.

The nurse notes new onset confusion in an older patient who is normally alert and oriented. In which order should the nurse take the following actions? (Put a comma and a space between each answer choice [A, B, C,D].) a. Obtain the oxygen saturation. b. Check the patients pulse rate. c. Document the change in status. d. Notify the health care provider.

a. Obtain the oxygen saturation. b. Check the patients pulse rate. d. Notify the health care provider. c. Document the change in status. Assessment for physiologic causes of new onset confusion such as pneumonia, infection, or perfusion problems should be the first action by the nurse. Airway and oxygenation should be assessed first, then circulation. After assessing the patient, the nurse should notify the health care provider. Finally, documentation of the assessments and care should be done.

Which menu choice indicates that the patient understands the nurses teaching about best dietary choices for iron-deficiency anemia? a. Omelet and whole wheat toast b. Cantaloupe and cottage cheese c. Strawberry and banana fruit plate d. Cornmeal muffin and orange juice

a. Omelet and whole wheat toast Eggs and whole grain breads are high in iron. The other choices are appropriate for other nutritional deficiencies but are not the best choice for a patient with iron-deficiency anemia.

Which surgical therapy for an AAA is most likely to have the postoperative complication of renal injury? a. Open aneurysm repair (OAR) above the level of the renal arteries b. Excising only the weakened area of the artery and suturing the artery closed c. Bifurcated graft used in aneurysm repair when the AAA extends into the iliac arteries d. Endovascular graft procedure with an aortic graft inside the aneurysm via the femoral artery

a. Open aneurysm repair (OAR) above the level of the renal arteries With the aortic cross-clamping proximal and distal to the aneurysm, the open aneurysm repair (OAR) above the renal artery may cause kidney injury from lack of blood flow during the surgery. The saccular aneurysm may involve excising only the weakened area of the artery and suturing the artery closed, but this will not decrease renal blood flow. Renal blood flow will not be directly obstructed using the bifurcated graft or the minimally invasive endovascular aneurysm repair

A patient who has been receiving diuretic therapy is admitted to the emergency department with a serum potassium level of 3.0 mEq/L. The nurse should alert the health care provider immediately that the patient is on which medication? a. Oral digoxin (Lanoxin) 0.25 mg daily b. Ibuprofen (Motrin) 400 mg every 6 hours c. Metoprolol (Lopressor) 12.5 mg orally daily d. Lantus insulin 24 U subcutaneously every evening

a. Oral digoxin (Lanoxin) 0.25 mg daily Hypokalemia increases the risk for digoxin toxicity, which can cause serious dysrhythmias. The nurse will also need to do more assessment regarding the other medications, but they are not of as much concern with the potassium level.

A patient is admitted to the ICU with a diagnosis of NSTEMI. Which drugs(s) would the nurse expect the patient to receive? (select all that apply) a. Oral statin therapy b. Antiplatelet therapy c Thrombolytic therapy d. Prophylactic antibiotics e. Intravenous nitroglycerin

a. Oral statin therapy b. Antiplatelet therapy e. Intravenous nitroglycerin

Which assessment finding in a patient admitted with acute decompensated heart failure (ADHF) requires the most immediate action by the nurse? a. Oxygen saturation of 88% b. Weight gain of 1 kg (2.2 lb) c. Heart rate of 106 beats/minute d. Urine output of 50 mL over 2 hours

a. Oxygen saturation of 88% A decrease in oxygen saturation to less than 92% indicates hypoxemia. The nurse should administer supplemental oxygen immediately to the patient. An increase in apical pulse rate, 1-kg weight gain, and decreases in urine output also indicate worsening heart failure and require nursing actions, but the low oxygen saturation rate requires the most immediate nursing action.

To determine whether there is a delay in impulse conduction through the atria, the nurse will measure the duration of the patients a. P wave. b. Q wave. c. P-R interval. d. QRS complex.

a. P wave. The P wave represents the depolarization of the atria. The P-R interval represents depolarization of the atria, atrioventricular (AV) node, bundle of His, bundle branches, and the Purkinje fibers. The QRS represents ventricular depolarization. The Q wave is the first negative deflection following the P wave and should be narrow and short.

Which findings indicate that a patient is developing status asthmaticus? (select all that apply) a. PEFR <300 L/min b. Positive sputum culture c. Unable to speak in complete sentences d. Lack of response to conventional treatment e. Chest x-ray shows hyperinflated lungs and a flattened diaphragm

a. PEFR <300 L/min c. Unable to speak in complete sentences d. Lack of response to conventional treatment

The nurse evaluates that treatment for the patient with an uncomplicated aortic dissection is successful when what happens? a. Pain is relieved. b. Surgical repair is completed. c. BP is increased to normal range. d. Renal output is maintained at 30 mL/hr.

a. Pain is relieved. Relief of pain is a sign that the dissection has stabilized, and it may be treated conservatively for an extended time with drugs that lower the BP and decrease myocardial contractility. Surgery is usually indicated for Type A aortic dissection or if complications occur.

A 58-year-old patient is in a cardiac rehabilitation program. Which sign or symptom should the nurse teach the patient is a reason to stop exercising? a. Pain or dyspnea develop b. The HR exceeds 150 bpm c. The respiratory rate increases to 30 d. The HR is 30 bpm over the resting HR

a. Pain or dyspnea develop Any activity or exercise that causes dyspnea and chest pain should be stopped in the patient with CAD. The training target for a healthy 58- year-old is 80% of maximum HR, or 130 bpm. HR, rather than respiratory rate, determines the parameters for exercise.

Which patients have the greatest risk for aspiration pneumonia? (select all that apply) a. Patient with seizures b. Patient with head injury c. Patient who had thoracic surgery d. Patient who had a myocardial infarction e. Patient who is receiving nasogastric tube feeding

a. Patient with seizures b. Patient with head injury e. Patient who is receiving nasogastric tube feeding

Which statements accurately describe thrombocytopenia (select all that apply)? a. Patients with platelet deficiencies can have internal or external hemorrhage. b. The most common acquired thrombocytopenia is thrombotic thrombocytopenic purpura (TTP). c. Immune thrombocytopenic purpura (ITP) is characterized by increased platelet destruction by the spleen. d. TTP is characterized by decreased platelets, decreased RBCs, and enhanced aggregation of platelets. e. A classic clinical manifestation of thrombocytopenia that the nurse would expect to find on physical examination of the patient is ecchymosis.

a. Patients with platelet deficiencies can have internal or external hemorrhage. c. Immune thrombocytopenic purpura (ITP) is characterized by increased platelet destruction by the spleen. d. TTP is characterized by decreased platelets, decreased RBCs, and enhanced aggregation of platelets. Platelet deficiencies lead to internal and external hemorrhage. Immune thrombocytopenic purpura (ITP) is characterized by increased platelet destruction by the spleen. Thrombotic thrombocytopenic purpura (TTP) is characterized by decreased platelets and RBCs with enhanced agglutination of the platelets. ITP is the most common acquired thrombocytopenia. Petechiae, not ecchymosis, is a common manifestation of thrombocytopenia.

A patients cardiac monitor shows a pattern of undulations of varying contours and amplitude with no measurable ECG pattern. The patient is unconscious and pulseless. Which action should the nurse take first? a. Perform immediate defibrillation. b. Give epinephrine (Adrenalin) IV. c.Prepare for endotracheal intubation. d. Give ventilations with a bag-valve-mask device.

a. Perform immediate defibrillation. The patients rhythm and assessment indicate ventricular fibrillation and cardiac arrest; the initial action should be to defibrillate. If a defibrillator is not immediately available or is unsuccessful in converting the patient to a better rhythm, the other actions may be appropriate.

The nurse is caring for a patient with cor pulmonale. The nurse should monitor the patient for which expected finding? a. Peripheral edema b. Elevated temperature c. Clubbing of the fingers d. Complaints of chest pain

a. Peripheral edema Cor pulmonale causes clinical manifestations of right ventricular failure, such as peripheral edema. The other clinical manifestations may occur in the patient with other complications of chronic obstructive pulmonary disease (COPD) but are not indicators of cor pulmonale.

The nurse is caring for a patient who reports extreme weakness and glossitis, and the nurse identifies pale yellow skin. Which type of anemia does the nurse anticipate in the patient? a. Pernicious Anemia b. Sickle Cell Anemia c. Iron Deficiency Anemia d. Angina Anemia

a. Pernicious Anemia

Which manifestation of infective endocarditis is a result of fragmentation and microembolization of vegetative lesions? a. Petechiae b. Roth's spots c. Osler's nodes d. Splinter hemorrhages

a. Petechiae Petechiae from fragmentation and microembolization of vegetative lesions are seen as small hemorrhages in the conjunctiva, lips, and buccal mucosa and over the ankles, feet, and antecubital and popliteal areas. Roth's spots are hemorrhagic retinal lesions seen with funduscopic examination. Osler's nodes are lesions on the fingertips or toes. The cause of Roth's spots and Osler's nodes is not clear. Splinter hemorrhages are black longitudinal streaks that occur on nail beds. They may be caused by vessel damage from vasculitis or microemboli.

During discharge teaching for an older adult patient with chronic obstructive pulmonary disease (COPD) and pneumonia, which vaccine should the nurse recommend that this patient receive? a. Pneumococcal b. Staphylococcus aureus c. Haemophilus influenzae d. Bacille-Calmette-Guérin (BCG)

a. Pneumococcal The pneumococcal vaccine is important for patients with a history of heart or lung disease, recovering from a severe illness, age 65 years or older, or living in a long-term care facility. A S. aureus vaccine has been researched but not yet been effective. The H. influenzae vaccine would not be recommended as adults do not need it unless they are immunocompromised. The BCG vaccine is for infants in parts of the world where tuberculosis is prevalent.

Patient with altered consciousness is hospitalized and at risk for aspiration pneumonia. What nursing intervention is indicated to prevent pneumonia? a. Position to side, protect airway b. Check placement of the tube before feeding and residual feeding; keep head of bed up after feedings or continuously with continuous feedings c. Check gag reflex before feeding or offering fluids d. Cut food in small bites, encourage thorough chewing, and provide soft foods that are easier to swallow than liquids

a. Position to side, protect airway

A nurse preparing a patient for a bone marrow biopsy knows which site will most likely be used? a. Posterior iliac crest b. Anterior iliac crest c. Anterior thigh d. Deltoid

a. Posterior iliac crest

What are characteristics of vasospastic disease (Raynaud's phenomenon) (select all that apply)? a. Predominant in young females b. May be associated with autoimmune disorders c. Precipitated by exposure to cold, caffeine, and tobacco d. Involves small cutaneous arteries of the fingers and toes e. Inflammation of small and medium-sized arteries and veins f. Episodes involve white, blue, and red color changes of fingertips

a. Predominant in young females b. May be associated with autoimmune disorders c. Precipitated by exposure to cold, caffeine, and tobacco d. Involves small cutaneous arteries of the fingers and toes f. Episodes involve white, blue, and red color changes of fingertips Raynaud's phenomenon is predominant in young females and may be associated with autoimmune disorders (e.g., rheumatoid arthritis, scleroderma, systemic lupus erythematosus). Incidents occur with cold, emotional upsets, and caffeine or tobacco use because of vasoconstrictive effects. Small cutaneous arteries are involved and cause color changes of the fingertips or toes. When conservative management is ineffective, it may be treated with nifedipine (Procardia).

The client diagnosed with pericarditis is experiencing cardiac tamponade. Which collaborative intervention should the nurse anticipate for this client? a. Prepare for a pericardiocentesis b .Request STAT cardiac enzymes. c .Perform a 12-lead electrocardiogram. d. Assess the client's heart and lung sounds.

a. Prepare for a pericardiocentesis The pericardiocentesis removes fluid from the pericardial sac and is the emergency tx for cardiac tamponade.

In preparation for an AAA repair surgery, what should the nurse include in patient teaching? a. Prepare the bowel on the night before surgery with laxatives or an enema. b. Use moisturizing soap to clean the skin three times the day before surgery. c. Eat a high-protein and high-carbohydrate breakfast to help with healing postoperatively. d. Take the prescribed oral antibiotic the morning of surgery before going to the operating room.

a. Prepare the bowel on the night before surgery with laxatives or an enema. Usually aortic surgery patients will have a bowel preparation, skin cleansing with an antimicrobial agent on the day before surgery, nothing by mouth after midnight on the day of the surgery, and IV antibiotics immediately before the incision is made. Patients with a history of CVD will receive a β-adrenergic blocker preoperatively to reduce morbidity and mortality. Each surgeon's protocol may be different.

Following a thyroidectomy, a patient complains of a tingling feeling around my mouth. Which assessment should the nurse complete immediately? a. Presence of the Chvostek's sign b. Abnormal serum potassium level c. Decreased thyroid hormone level d. Bleeding on the patients dressing

a. Presence of the Chvostek's sign The patients' symptoms indicate possible hypocalcemia, which can occur secondary to parathyroid injury/removal during thyroidectomy. There is no indication of a need to check the potassium level, the thyroid hormone level, or for bleeding.

Safe administration of PRBCs? Select all that apply. a. Prime tubing with 0.9% sodium chloride (normal saline) b. Obtain and document a full set of Vital Signs c. Check name and blood product with second nurse d. Administer 0.9% sodium chloride after infusion e. Monitor for s/s of transfusion reaction like volume overload

a. Prime tubing with 0.9% sodium chloride (normal saline) b. Obtain and document a full set of Vital Signs c. Check name and blood product with second nurse d. Administer 0.9% sodium chloride after infusion e. Monitor for s/s of transfusion reaction like volume overload All are considered safe administration

Risk factors for acquiring pneumonia include (select all that apply): a. Prolonged immobility b. Aspiration c. Mechanical ventilation d. Young age

a. Prolonged immobility b. Aspiration c. Mechanical ventilation

A patient with aplastic anemia has impaired oral mucous membranes. This problem can be related to the effects of what deficiencies (select all that apply)? a. RBCs b. Ferritin c. Platelets d. Coagulation factor VIII e. White blood cells (WBCs)

a. RBCs c. Platelets e. White blood cells (WBCs) Aplastic anemia may cause an inflamed, painful tongue. Oxygen is not delivered without RBCs. The thrombocytopenia may contribute to blood-filled bullae in the mouth and gingival bleeding. The leukopenia may lead to stomatitis and oral ulcers and infections. MCV will be normal or slightly increased. Ferritin and coagulation factors are not affected in aplastic anemia.

Using light pressure with the index and middle fingers, the nurse cannot palpate any of the patient's superficial lymph nodes. How should the nurse respond to this assessment? a. Record this finding as normal. b. Reassess the lymph nodes using deeper pressure. c. Ask the patient about any history of radiation therapy. d. Notify the health care provider (HCP) that x-rays of the nodes will be necessary.

a. Record this finding as normal. Superficial lymph nodes are evaluated by light palpation, but they are not normally palpable. It may be normal to find small (< 1.0 cm), mobile, firm, nontender nodes. Deep lymph nodes are detected radiographically

Which BP-regulating mechanism(s) can result in the development of hypertension if defective? (select all that apply) a. Release of norepinephrine b. Secretion of prostaglandins c. Stimulation of the sympathetic nervous system d. Stimulation of the parasympathetic nervous system e. Activation of the renin-angiotensin-aldosterone system

a. Release of norepinephrine c. Stimulation of the sympathetic nervous system d. Stimulation of the parasympathetic nervous system

A nurse is preparing to administer a unit of PRBCs to a client. Which action should the nurse take? a. Remain with the client for the first 15 minutes. b. Discontinue the IV catheter if a blood transfusion reaction occurs. c. Administer the PRBCs through a percutaneously inserted central catheter line with a 20-gauge needle. d. Flush PRBCs with 5% dextrose and 0.45% normal saline solution.

a. Remain with the client for the first 15 minutes.

A 20-year-old patient has acute infective endocarditis. What topics should the nurse ask the patient about during the health history (select all that apply)? a. Renal dialysis b. IV drug abuse c. Recent dental work d. Cardiac catheterization e. Recent urinary tract infection

a. Renal dialysis b. IV drug abuse c. Recent dental work d. Cardiac catheterization e. Recent urinary tract infection Recent dental, urologic, surgical, or gynecologic procedures and history of IV drug abuse, heart disease, cardiac catheterization or surgery, renal dialysis, and infections all increase the risk of infective endocarditis.

The nurse assesses a patient with chronic obstructive pulmonary disease (COPD) who has been admitted with increasing dyspnea over the last 3 days. Which finding is most important for the nurse to report to the health care provider? a. Respirations are 36 breaths/minute. b. Anterior-posterior chest ratio is 1:1. c. Lung expansion is decreased bilaterally. d. Hyperresonance to percussion is present.

a. Respirations are 36 breaths/minute. The increase in respiratory rate indicates respiratory distress and a need for rapid interventions such as administration of oxygen or medications. The other findings are common chronic changes occurring in patients with COPD.

A nurse is admitting a client who has status asthmaticus. The client's ABG results are pH 7.32, PaO2 74 mmhg, PaC02 56 mm hg, and HCO3- 26 mEq/L. The nurse should interpret these laboratory values as which of the following imbalances. a. Respiratory acidosis b. Respiratory alkalosis c. Metabolic acidosis d. Metabolic alkalosis

a. Respiratory acidosis Status asthmaticus causes inadequate gas exchange, resulting in a low pH and PaO2, an elevated PaCO2, and an HCO3- within the expected reference range. These laboratory values indicate respiratory acidosis.

The patient has chronic venous insufficiency and a venous ulcer. The unlicensed assistive personnel (UAP) decides to apply compression stockings because that is what patients 'always' have ordered. Which assessment finding would indicate the application of compression stockings could harm the patient? a. Rest pain b. High blood pressure c. Elevated blood sugar d. Dry, itchy, flaky skin

a. Rest pain Rest pain occurs as peripheral artery disease (PAD) progresses and involves multiple arterial segments. Compression stockings should not be used on patients with PAD. Elevated blood glucose, possibly indicating uncontrolled diabetes mellitus, and hypertension may or may not indicate arterial problems. Dry, itchy, flaky skin indicates venous insufficiency. The RN should be the one to obtain the order and instruct the UAP to apply compression stockings if they are ordered.

The patient's arterial blood gas results show the PaO2 at 65 mmHg and SaO2 at 80%. What other manifestations should the nurse expect to observe in this patient? a. Restlessness, tachypnea, tachycardia, and diaphoresis b. Unexplained confusion, dyspnea at rest, hypotension, and diaphoresis c. Combativeness, retractions with breathing, cyanosis, and decreased output d. Coma, accessory muscle use, cool and clammy skin, and unexplained fatigue

a. Restlessness, tachypnea, tachycardia, and diaphoresis With inadequate oxygenation, early manifestations include restlessness, tachypnea, tachycardia, and diaphoresis, decreased urinary output, and unexplained fatigue. Unexplained confusion, dyspnea at rest, hypotension, and diaphoresis; combativeness, retractions with breathing, cyanosis, and decreased urinary output; coma, accessory muscle use, cool and clammy skin, and unexplained fatigue are later manifestations of inadequate oxygenation.

Which signs and symptoms should the nurse expect to find when assessing a patient with infective endocarditis who uses IV cocaine? (select all that apply) a. Retinal hemorrhages b. Splinter hemorrhages c. Presence of Osler's nodes d. Painless nodules over bony prominences e. Erythematous macules on the palms and soles

a. Retinal hemorrhages b. Splinter hemorrhages c. Presence of Osler's nodes e. Erythematous macules on the palms and soles

The nurse cares for a client diagnosed with immune thrombocytopenia purpura. Which nursing diagnosis is a priority when caring for this client? a. Risk for injury b. Weight gain​ c. Poor skin turgor​ d. Bradypnea​

a. Risk for injury

Risk Factor for or Response to Respiratory Problem: Loss of roles at work or home, exposure to respiratory toxins at work. Which Functional Health Pattern does it fall under? a. Role-relationship b. Sexuality-reproductive c. Coping- stress tolerance d. Value-belief

a. Role-relationship

Which information obtained by the nurse assessing a patient admitted with multiple myeloma is most important to report to the health care provider? a. Serum calcium level is 15 mg/dL. b. Patient reports no stool for 5 days. c. Urine sample has Bence-Jones protein. d. Patient is complaining of severe back pain.

a. Serum calcium level is 15 mg/dL. Hypercalcemia may lead to complications such as dysrhythmias or seizures, and should be addressed quickly. The other patient findings will also be discussed with the health care provider, but are not life threatening.

The nurse is reviewing the laboratory test results for a patient who has recently been diagnosed with hypertension. Which result is most important to communicate to the health care provider? a. Serum creatinine of 2.8 mg/dL b. Serum potassium of 4.5 mEq/L c. Serum hemoglobin of 14.7 g/dL d. Blood glucose level of 96 mg/dL

a. Serum creatinine of 2.8 mg/dL The elevated creatinine indicates renal damage caused by the hypertension. The other laboratory results are normal.

A patient who is being admitted to the emergency department with intermittent chest pain gives the following list of medications to the nurse. Which medication has the most immediate implications for the patients care? a. Sildenafil (Viagra) b. Furosemide (Lasix) c. Captopril (Capoten) d. Warfarin (Coumadin)

a. Sildenafil (Viagra) The nurse will need to avoid giving nitrates to the patient because nitrate administration is contraindicated in patients who are using sildenafil because of the risk of severe hypotension caused by vasodilation. The other home medications also should be documented and reported to the health care provider but do not have as immediate an impact on decisions about the patients treatment.

The nurse is caring for a patient who is receiving a blood transfusion. The client begins to exhibit dyspnea, crackles in lung bases, and becomes restless. Which action should the nurse anticipate? a. Slow the infusion rate b. Check vital signs c. Stop transfusion d. Give oxygen

a. Slow the infusion rate

An outpatient comes to the clinic for 2 units of blood. In the first 10 minutes of the 1st unit, the client's lung sounds have audible crackles, VS: HR 128, RR 48 dyspnea, O2 sat 85%, and jugular vein distention is assessed. Which of these interventions should be implemented as priority? Select all that apply. a. Slow the infusion to run over 4 hours maximum. b. Raise the HOB up 45 degrees and begin oxygen BNC at 2-4 L/M. c. Discontinue all IV fluids infusing with the blood. d. Notify the health care provider (HCP) immediately. e. Stop the infusion immediately.

a. Slow the infusion to run over 4 hours maximum. b. Raise the HOB up 45 degrees and begin oxygen BNC at 2-4 L/M. c. Discontinue all IV fluids infusing with the blood. d. Notify the health care provider (HCP) immediately. This answer is correct because the blood is being transfused too fast, so the nurse must slow the infusion to run over 4 hours maximum. This answer is correct because the client's respiratory rate is 48 with dyspnea, and an O2 sat of 85%, so the nurse must raise the HOB up at least to 45 degrees and administer oxygen BNC at 2-4 L/M to ensure that the client's O2 sat is at least 92%. This answer is correct because discontinuing any other IV fluids infusing with the blood such as 0.9% normal saline is pertinent to reduce the fluid overload. This answer is correct because notifying the health care provider (HCP) is priority to ensure the client receives other orders/interventions such as diuretics to reduce the fluid overload.

Which person should the nurse identify as having the highest risk for abdominal aortic aneurysm? a A 70-yr-old man with high cholesterol and hypertension b A 40-yr-old woman with obesity and metabolic syndrome c A 60-yr-old man with renal insufficiency who is physically inactive d A 65-yr-old woman with hyperhomocysteinemia and substance abuse

a. The most common etiology of descending abdominal aortic aneurysm (AAA) is atherosclerosis. Male gender, age 65 years or older, and tobacco use are the major risk factors for AAAs of atherosclerotic origin. Other risk factors include the presence of coronary or peripheral artery disease, high blood pressure, and high cholesterol.

Patient-Centered Care: The surgery area calls the transfer report for a 68-year-old, postmenopausal, female patient who smokes and takes hormone therapy. She is returning to the clinical unit after a lengthy hip replacement surgery. Which factors present in this patient increase her risk for developing venous thromboembolism (VTE) related to Virchow's triad (select all that apply)? a. Smoking b. IV therapy c. Dehydration d. Estrogen therapy e. Orthopedic surgery f. Prolonged immobilization

a. Smoking b. IV therapy d. Estrogen therapy e. Orthopedic surgery f. Prolonged immobilization This patient is a smoker and on hormone therapy, both of which increase blood hypercoagulability. She will have an IV, and her fractured hip can cause VTE by damaging the venous endothelium. She is an older patient who has had an orthopedic surgery and may have experienced prolonged immobility postinjury and through her "lengthy hip replacement surgery," which contributes to venous stasis. These are representative of Virchow's triad in this patient. The other options are also related to Virchow's triad but not present in this patient via the transfer report.

A nurse is assessing a client who is receiving hydrochlorothiazide and notes that the client is confused and lethargic. Which of the following laboratory values should the nurse report to the provider? a. Sodium 128 mEq/L b. Potassium 4.8 mEq/L c. Calcium 9.1 mg/dL d. Magnesium 2.0 mEq/L

a. Sodium 128 mEq/L This level is below the expected reference range and is the likely cause of the client's altered mental status. The nurse should report this finding to the provider and monitor the client for weakened respiratory effort.

Which treatments would the nurse expect to implement in the management plan of a patient with cystic fibrosis? (select all that apply) a. Sperm banking b. IV corticosteroids on a chronic basis c. Airway clearance techniques (e.g., Acapella) d. GoLYTELY given as needed for severe constipation e. Inhaled tobramycin to combat Pseudomonas infection

a. Sperm banking c. Airway clearance techniques (e.g., Acapella) d. GoLYTELY given as needed for severe constipation e. Inhaled tobramycin to combat Pseudomonas infection

A patient is admitted to the emergency department complaining of sudden onset shortness of breath and is diagnosed with a possible pulmonary embolus. How should the nurse prepare the patient for diagnostic testing to confirm the diagnosis? a. Start an IV so contrast media may be given. b. Ensure that the patient has been NPO for at least 6 hours. c. Inform radiology that radioactive glucose preparation is needed. d. Instruct the patient to undress to the waist and remove any metal objects.

a. Start an IV so contrast media may be given. Spiral computed tomography (CT) scans are the most commonly used test to diagnose pulmonary emboli, and contrast media may be given IV. A chest x-ray may be ordered but will not be diagnostic for a pulmonary embolus. Preparation for a chest x-ray includes undressing and removing any metal. Bronchoscopy is used to detect changes in the bronchial tree, not to assess for vascular changes, and the patient should be NPO 6 to 12 hours before the procedure. Positron emission tomography (PET) scans are most useful in determining the presence of malignancy, and a radioactive glucose preparation is used

A patient in the outpatient clinic has a new diagnosis of peripheral artery disease (PAD). Which group of medications will the nurse plan to include when providing patient teaching about PAD management? a. Statins b. Antibiotics c. Thrombolytics d. Anticoagulants

a. Statins Current research indicates that statin use by patients with PAD improves multiple outcomes. There is no research that supports the use of the other medication categories in PAD.

During the nursing assessment of a patient with anemia, what specific information should the nurse ask the patient about? a. Stomach surgery b. Recurring infections c. Corticosteroid therapy d. Oral contraceptive use

a. Stomach surgery The parietal cells of the stomach secrete intrinsic factor, a substance necessary for the absorption of cobalamin (vitamin B12 ), and if all or part of the stomach is removed, the lack of intrinsic factor can lead to impaired RBC production and pernicious anemia. Recurring infections indicate decreased WBCs and immune response, and corticosteroid therapy may cause a neutrophilia and lymphopenia. Oral contraceptive use is strongly associated with changes in blood coagulation

Priority Decision: While receiving a unit of packed RBCs, the patient develops chills and a temperature of 102.2° F (39° C). What is the priority action for the nurse to take? a. Stop the transfusion and instill normal saline. b. Notify the health care provider and the blood bank. c. Add a leukocyte reduction filter to the blood administration set. d. Recognize this as a mild allergic transfusion reaction and slow the transfusion.

a. Stop the transfusion and instill normal saline. Chills and fever are symptoms of an acute hemolytic or febrile transfusion reaction, and if these develop, the nurse should stop the transfusion, infuse saline through the IV line, notify the HCP and blood bank immediately, recheck the ID tags, and monitor vital signs and urine output. The addition of a leukocyte reduction filter may prevent a febrile reaction but is not helpful once the reaction has occurred. Mild and transient allergic reactions indicated by itching and hives may permit restarting the transfusion after treatment with antihistamines.

While the nurse is feeding a patient, the patient appears to choke on the food. Which symptoms indicate to the nurse that the patient has a partial airway obstruction (select all that apply)? a. Stridor b. Cyanosis c. Wheezing d. Bradycardia e. Rapid respiratory rate

a. Stridor b. Cyanosis c. Wheezing With partial airway obstruction, choking, stridor, use of accessory muscles, suprasternal and intercostals retraction, flaring nostrils, wheezing, restlessness, tachycardia, cyanosis, and change in level of consciousness may occur. Partial airway obstruction may progress to complete obstruction without prompt assessment and treatment.

Older clients experiencing anginal pain with complaints of fatigue or weakness usually are medicated with which of the following types of medication? a. Sublingual nitroglycerin b. Cardiac glycosides c. HMG-CoA reductase inhibitors d. Morphine sulfate

a. Sublingual nitroglycerin

After which diagnostic study should the nurse observe the patient for symptoms of a pneumothorax? a. Thoracentesis b. Pulmonary function test c. Ventilation-perfusion scan d. Positron emission tomography (PET) scan

a. Thoracentesis The greatest chance for a pneumothorax occurs with a thoracentesis because of the possibility of lung tissue injury during this procedure. Ventilation-perfusion scans and positron emission tomography (PET) scans involve injections, but no manipulation of the respiratory tract is involved. Pulmonary function tests are noninvasive.

Collaboration: In planning the care for a patient with a tracheostomy who has been stable and is to be discharged later in the day, the registered nurse (RN) may delegate which interventions to the licensed practical/vocational nurse (LPN/VN) (select all that apply)? a. Suction the tracheostomy. b. Provide tracheostomy care. c. Determine the need for suctioning. d. Assess the patient's swallowing ability. e. Teach the patient about home tracheostomy care.

a. Suction the tracheostomy. b. Provide tracheostomy care. c. Determine the need for suctioning. LPNs may determine the need for suctioning, suction the tracheostomy, and determine whether the patient has improved after the suctioning when caring for stable patients. They also may perform tracheostomy care using sterile technique. The patient's swallowing ability is assessed by a speech therapist, videofluoroscopy, or fiberoptic endoscopic evaluations. The RN will teach the patient about home tracheostomy care.

An older adult patient living alone is admitted to the hospital with pneumococcal pneumonia. Which clinical manifestation is consistent with the patient being hypoxic? a. Sudden onset of confusion b. Oral temperature of 102.3° F c. Coarse crackles in lung bases d. Clutching chest on inspiration

a. Sudden onset of confusion Confusion or stupor (related to hypoxia) may be the only clinical manifestation of pneumonia in an older adult patient. An elevated temperature, coarse crackles, and pleuritic chest pain with guarding may occur with pneumonia, but these symptoms do not indicate hypoxia.

The nurse hears a murmur between the S1 and S2 heart sounds at the patients left fifth intercostal space and midclavicular line. How will the nurse record this information? a. Systolic murmur heard at mitral area b. Systolic murmur heard at Erbs point c. Diastolic murmur heard at aortic area d. Diastolic murmur heard at the point of maximal impulse

a. Systolic murmur heard at mitral area The S1 signifies the onset of ventricular systole. S2 signifies the onset of diastole. A murmur occurring between these two sounds is a systolic murmur. The mitral area is the intersection of the left fifth intercostal space and the midclavicular line. The other responses describe murmurs heard at different landmarks on the chest and/or during the diastolic phase of the cardiac cycle

A nurse is assessing a client that is dehydrated. Which finding should the nurse expect? (Select All That Apply)​ a. Tachycardia​ b. Tachypnea​ c. Hypertension​ d. Jugular Vein Distension​ e. Poor skin turgor​ f. Pulmonary edema

a. Tachycardia​ b. Tachypnea e. Poor skin turgor​

A nurse is assessing a client with fluid volume overload. Which finding should the nurse expect? (Select All That Apply) ​ a. Tachycardia​ b. Bradypnea​ c. Hypertension​ d. Jugular Vein Distension​ e. Weight gain​ f. Edema

a. Tachycardia​ c. Hypertension​ d. Jugular Vein Distension​ e. Weight gain​ f. Edema

Nursing Interventions for pneumonia (select all that apply): a. Teach coughing and deep breathing to patient b. Monitor vital signs c. Encourage & assist with ambulation d. Maintain in supine position as often as possible.

a. Teach coughing and deep breathing to patient b. Monitor vital signs c. Encourage & assist with ambulation

A patient who is 2 days post-femoral-popliteal bypass graft to the right leg is being cared for on the vascular unit. Which action by a licensed practical/vocational nurse (LPN/LVN) caring for the patient requires the registered nurse (RN) to intervene? a. The LPN/LVN has the patient sit in a chair for 90 minutes. b. The LPN/LVN assists the patient to walk 40 feet in the hallway. c. The LPN/LVN gives the ordered aspirin 160 mg after breakfast. d. The LPN/LVN places the patient in a Fowlers position for meals.

a. The LPN/LVN has the patient sit in a chair for 90 minutes. The patient should avoid sitting for long periods because of the increased stress on the suture line caused by leg edema and because of the risk for venous thromboembolism (VTE). The other actions by the LPN/LVN are appropriate.

What should the nurse emphasize when teaching a patient who is newly prescribed clonidine (Catapres)? a. The drug should never be stopped abruptly. b. The drug should be taken early in the day to prevent nocturia. c. The first dose should be taken when the patient is in bed for the night. d. Because aspirin will decrease the drug's effectiveness, Tylenol should be used instead

a. The drug should never be stopped abruptly. Centrally acting α-adrenergic blockers may cause severe rebound hypertension if the drugs are abruptly discontinued, and patients should be taught about this effect because many are not consistently compliant with drug therapy. Diuretics should be taken early in the day to prevent nocturia. The profound orthostatic hypotension that occurs with first-dose peripheral acting α-adrenergic blockers can be prevented by taking the initial dose at bedtime. Aspirin use may decrease the effectiveness of ACE inhibitors.

When the patient is being examined for venous thromboembolism (VTE) in the calf, what diagnostic test should the nurse expect to teach the patient about first? a Duplex ultrasound b Contrast venography c Magnetic resonance venography d Computed tomography venography

a. The duplex ultrasound is the most widely used test to diagnose VTE. Contrast venography is rarely used now. Magnetic resonance venography is less accurate for calf veins than pelvic and proximal veins. Computed tomography venography may be used but is invasive and much more expensive than the duplex ultrasound.

What are the characteristics of the carbonic acid-bicarbonate buffer system (select all that apply)? a. The lungs eliminate CO2 b. Neutralizes HCl acid to yield carbonic acid and salt c. H2CO3 formed by neutralization dissociates into H2O and CO2 d. Shifts H+ in and out of cell in exchange for other cations, such as potassium and sodium e. Free basic radicals dissociate into ammonia and OH− , which combines with H+ to form water

a. The lungs eliminate CO2 b. Neutralizes HCl acid to yield carbonic acid and salt c. H2CO3 formed by neutralization dissociates into H2O and CO2 CO2 elimination by the lungs, neutralized HCl, and H2CO3 formation are all part of the carbonic acid-bicarbonate buffer system. Shifts of H+ in and out of the cell are characteristics of the cellular buffer system. Free basic radical dissociation is characteristic of the protein buffer system.

When assessing a patient, you note a pulse deficit of 23 beats. This finding may be caused by a. dysrhythmias. b. heart murmurs. c. gallop rhythms. d. pericardial friction rubs.

a. dysrhythmias.

Which intervention by a new nurse who is caring for a patient who has just had an implantable cardioverter-defibrillator (ICD) inserted indicates a need for more education about care of patients with ICDs? a. The nurse assists the patient to do active range of motion exercises for all extremities. b. The nurse assists the patient to fill out the application for obtaining a Medic Alert ID. c. The nurse gives amiodarone (Cordarone) to the patient without first consulting with the health care provider. d. The nurse teaches the patient that sexual activity usually can be resumed once the surgical incision is healed.

a. The nurse assists the patient to do active range of motion exercises for all extremities. The patient should avoid moving the arm on the ICD insertion site until healing has occurred in order to prevent displacement of the ICD leads. The other actions by the new nurse are appropriate for this patient.

The nurse is caring for a 78-year-old patient with aortic stenosis. Which assessment data obtained by the nurse would be most important to report to the health care provider? a. The patient complains of chest pressure when ambulating. b. A loud systolic murmur is heard along the right sternal border. c. A thrill is palpated at the second intercostal space, right sternal border. d. The point of maximum impulse (PMI) is at the left midclavicular line.

a. The patient complains of chest pressure when ambulating. Chest pressure (or pain) occurring with aortic stenosis is caused by cardiac ischemia, and reporting this information would be a priority. A systolic murmur and thrill are expected in a patient with aortic stenosis. A PMI at the left midclavicular line is normal.

After teaching a patient with newly diagnosed Raynauds phenomenon about how to manage the condition, which action by the patient demonstrates that the teaching has been effective? a. The patient exercises indoors during the winter months. b. The patient places the hands in hot water when they turn pale. c. The patient takes pseudoephedrine (Sudafed) for cold symptoms. d. The patient avoids taking nonsteroidal antiinflammatory drugs (NSAIDs).

a. The patient exercises indoors during the winter months. Patients should avoid temperature extremes by exercising indoors when it is cold. To avoid burn injuries, the patient should use warm, rather than hot, water to warm the hands. Pseudoephedrine is a vasoconstrictor, and should be avoided. There is no reason to avoid taking NSAIDs with Raynauds phenomenon.

The nurse is caring for a 64-year-old patient admitted with mitral valve regurgitation. Which information obtained by the nurse when assessing the patient should be communicated to the health care provider immediately? a. The patient has bilateral crackles. b. The patient has bilateral, 4+ peripheral edema. c. The patient has a loud systolic murmur across the precordium. d. The patient has a palpable thrill felt over the left anterior chest.

a. The patient has bilateral crackles. Crackles that are audible throughout the lungs indicate that the patient is experiencing severe left ventricular failure with pulmonary congestion and needs immediate interventions such as diuretics. A systolic murmur and palpable thrill would be expected in a patient with mitral regurgitation. Although 4+ peripheral edema indicates a need for a change in therapy, it does not need to be addressed urgently.

A patient has a serum calcium level of 7.0 mEq/L. Which assessment finding is most important for the nurse to report to the health care provider? a. The patient is experiencing laryngeal stridor. b. The patient complains of generalized fatigue. c. The patient's bowels have not moved for 4 days. d. The patient has numbness and tingling of the lips.

a. The patient is experiencing laryngeal stridor. Hypocalcemia can cause laryngeal stridor, which may lead to respiratory arrest. Rapid action is required to correct the patients calcium level. The other data are also consistent with hypocalcemia, but do not indicate a need for as immediate action as laryngospasm

A 19-year-old woman with immune thrombocytopenic purpura (ITP) has an order for a platelet transfusion. Which information indicates that the nurse should consult with the health care provider before obtaining and administering platelets? a. The platelet count is 42,000/mL. b. Petechiae are present on the chest. c. Blood pressure (BP) is 94/56 mm Hg. d. Blood is oozing from the venipuncture site.

a. The platelet count is 42,000/mL. Platelet transfusions are not usually indicated until the platelet count is below 10,000 to 20,000/mL unless the patient is actively bleeding. Therefore the nurse should clarify the order with the health care provider before giving the transfusion. The other data all indicate that bleeding caused by ITP may be occurring and that the platelet transfusion is appropriate.

Which statement(s) about fluid in the human body is (are) true (select all that apply)? a. The primary hypothalamic mechanism of water intake is thirst. b. Third spacing refers to the abnormal movement of fluid into interstitial spaces. c. A cell surrounded by hypoosmolar fluid will shrink and die as water moves out of the cell. d. A cell surrounded by hyperosmolar fluid will shrink and die as water moves out of the cell. e. Concentrations of Na + and K+ in interstitial and intracellular fluids

a. The primary hypothalamic mechanism of water intake is thirst. d. A cell surrounded by hyperosmolar fluid will shrink and die as water moves out of the cell. e. Concentrations of Na + and K+ in interstitial and intracellular fluids With fluid volume deficit, the osmoreceptors stimulate thirst. Hyperosmolar extracellular fluid (ECF) draws fluid out of the cells. The sodium-potassium pump maintains the fluid balance between the intracellular fluid (ICF) and ECF. Third spacing is when fluid moves into spaces that normally have little or no fluid. A cell surrounded by hypoosmolar fluid will swell and burst as water moves into the cell.

Assessment of a patient's peripheral IV site reveals that phlebitis has developed over the past several hours. Which intervention should the nurse implement first? a Remove the patient's IV catheter. b Apply an ice pack to the affected area. c Decrease the IV rate to 20 to 30 mL/hr. d Administer prophylactic anticoagulants

a. The priority intervention for superficial phlebitis is removal of the offending IV catheter. Decreasing the IV rate is insufficient. Anticoagulants are not normally required, and warm, moist heat is often therapeutic.

A patient with no history of heart disease has a rhythm strip that shows an occasional distorted P wave followed by normal AV and ventricular conduction. About what should the nurse question the patient? a. The use of caffeine b. The use of sedatives c. Any aerobic training d. Holding of breath during exertion

a. The use of caffeine A distorted P wave with normal conduction of the impulse through the ventricles is characteristic of a PAC. In a normal heart, this dysrhythmia is frequently associated with emotional stress or the use of caffeine, tobacco, or alcohol. Sedatives rarely slow the heart rate (HR). Aerobic conditioning and holding of breath during exertion (Valsalva maneuver) often cause bradycardia, but the P wave is not distorted.

A patient has an admitting diagnosis of acute left-sided infective endocarditis. What is the best test to confirm this diagnosis? a. Three blood cultures b. Complete blood count c. Cardiac catheterization d. Transesophageal echocardiogram

a. Three blood cultures Three positive blood cultures drawn over a 1-hour period from 3 different sites are the primary diagnostic tool for infective endocarditis. Although a complete blood count (CBC) will reveal a mild leukocytosis, this is a nonspecific finding. Cardiac catheterization is used when surgical intervention is being considered. Transesophageal echocardiograms can identify vegetations on valves but are used when blood cultures are negative

Which guidelines should the nurse use to teach an older adult with CAD how to manage the treatment program for angina?? a. To sit for 2 to 5 minutes before standing when getting out of bed b. To exercise only twice a week to avoid unnecessary strain on the heart c. That lifestyle changes are not as necessary as they would be in a younger person d. That aspirin therapy is contraindicated in older adults because of the risk for bleeding

a. To sit for 2 to 5 minutes before standing when getting out of bed Orthostatic hypotension may cause dizziness and falls in older adults taking antianginal agents that decrease preload. Patients should be cautioned to change positions slowly. Daily exercise programs are indicated for older adults and may increase performance, endurance, and ability to tolerate stress. A change in lifestyle behaviors may increase the quality of life and reduce the risks of CAD, even in the older adult. Aspirin is often used in these patients and is not contraindicated.

T o F. Ninety percent of pulmonary embolisms develop from venous thromboembolism a. True b. False

a. True

The nurse is assessing a client with a suspected diagnosis of hypocalcemia. Which clinical manifestation would the nurse expect to note in the client? a. Twitching b. Hypoactive bowel sounds c. Negative Trousseau's sign d. Hypoactive deep tendon reflexes

a. Twitching A serum calcium level lower than 8.6 mg/dL indicates hypocalcemia. Signs of hypocalcemia include paresthesias followed by numbness, hyperactive deep tendon reflexes, and a positive Trousseau's or Chvostek's sign. Additional signs of hypocalcemia include increased neuromuscular excitability, muscle cramps, twitching, tetany, seizures, irritability, and anxiety. Gastrointestinal symptoms include increased gastric motility, hyperactive bowel sounds, abdominal cramping, and diarrhea.

Which statements accurately describe heart failure with preserved ejection fraction (HFpEF)? (select all that apply) a. Uncontrolled hypertension is the primary cause. b. Left ventricular ejection fraction may be within normal limits. c. The pathophysiology involves ventricular relaxation and filling. d. Multiple evidence-based therapies have been shown to decrease mortality. e. Therapies focus on symptom control and treatment of underlying conditions.

a. Uncontrolled hypertension is the primary cause. b. Left ventricular ejection fraction may be within normal limits. c. The pathophysiology involves ventricular relaxation and filling. e. Therapies focus on symptom control and treatment of underlying conditions.

The nurse is caring for a mechanically ventilated patient with a cuffed tracheostomy tube. Which action by the nurse would best determine if the cuff has been properly inflated? a. Use a manometer to ensure cuff pressure is at an appropriate level. b. Check the amount of cuff pressure ordered by the health care provider. c. Suction the patient first with a fenestrated inner cannula to clear secretions. d. Insert the decannulation plug before the nonfenestrated inner cannula is removed.

a. Use a manometer to ensure cuff pressure is at an appropriate level. Measurement of cuff pressure using a manometer to ensure that cuff pressure is 20 mm Hg or lower will avoid compression of the tracheal wall and capillaries. Never insert the decannulation plug in a tracheostomy tube until the cuff is deflated and the nonfenestrated inner cannula is removed. Otherwise, the patients airway is occluded. A health care providers order is not required to determine safe cuff pressure. A nonfenestrated inner cannula must be used to suction a patient to prevent tracheal damage occurring from the suction catheter passing through the fenestrated openings.

During a routine health examination, a 48-year-old patient is found to have a total cholesterol level of 224 mg/dL (5.8 mmol/L) and an LDL level of 140 mg/dL (3.6 mmol/L). What does the nurse teach the patient based on the Therapeutic Lifestyle Changes diet (select all that apply)? a. Use fat-free milk. b. Abstain from alcohol use. c. Reduce red meat in the diet. d. Eliminate intake of simple sugars. e. Avoid foods prepared with egg yolks.

a. Use fat-free milk. c. Reduce red meat in the diet. e. Avoid foods prepared with egg yolks. Therapeutic Lifestyle Changes diet recommendations emphasize reduction in saturated fat and cholesterol intake. Whole milk products, red meats, and eggs as well as butter, stick margarine, lard, and solid shortening should be reduced or eliminated from diets. If triglyceride levels are high, alcohol and simple sugars should be reduced.

Which method of low, constant oxygen administration is the safest system to use for a patient with COPD exacerbation? a. Venturi mask b. Nasal cannula c. Simple face mask d. Nonrebreather mask

a. Venturi mask A Venturi mask is helpful to administer low, constant O2 concentrations to patients with COPD and can be set to administer a varied percentage of O2 . The amount of O2 inhaled via the nasal cannula depends on room air and the patient's breathing pattern. The simple face mask must have a tight seal and may generate heat under the mask. The non-rebreather mask is more useful for short-term therapy with patients needing high O2 concentrations.

The nurse is preparing the patient for and will assist the health care provider with a thoracentesis in the patient's room. Which of the following is the sixth action? a. Verify breath sounds in all fields. b. Obtain the supplies that will be used. c. Send labeled specimen containers to the laboratory. d. Direct the family members to the waiting room. e. Observe for signs of hypoxia during the procedure. f. Instruct the patient not to talk during the procedure. g. Position the patient sitting upright with the elbows on an over-the bed table.

a. Verify breath sounds in all fields. Breath sounds in all lobes are verified to be sure that there was no damage to the lung.

A nurse is caring for a patient with pernicious anemia secondary to total gastrectomy. The nurse would question which provider's order? a. Vitamin B12 supplement, PO b. Vitamin B12 supplement, IV c. Vitamin B12 supplement, IM

a. Vitamin B12 supplement, PO B12 is NOT taken orally for patient with pernicious anemia

Which characteristics describe the anticoagulant warfarin (Coumadin) (select all that apply)? a. Vitamin K is the antidote b. Protamine sulfate is the antidote c. May be given orally or subcutaneously d. May be given intravenously or subcutaneously e. Monitor dosage using international normalized ratio (INR) f. Monitor dosage using activated partial thromboplastin time (aPTT)

a. Vitamin K is the antidote e. Monitor dosage using international normalized ratio (INR) Warfarin (Coumadin) is a vitamin K antagonist, so vitamin K is the antidote. It is monitored with the INR. It is only given orally. Protamine sulfate is the antidote for unfractionated heparin (UH) and LMWH. UH can be given subcutaneously or IV. It is monitored with activated partial thromboplastin time (aPTT). Hirudin derivatives are given IV or subcutaneously, do not have an antidote, and are monitored with aPTT. Argatroban, a synthetic thrombin inhibitor, is given only IV and is monitored with aPTT. Factor Xa inhibitor, fondaparinux (Arixtra), is given subcutaneously and does not need routine coagulation testing. Rivaroxaban (Xarelto), another factor Xa inhibitor, is given orally

Patient with heart failure who is constipated. What would the nurse recommend? (select all that apply) a. Walking b. Increase fiber c. Stool Softeners d. Drink extra water

a. Walking b. Increase fiber c. Stool Softeners

Following assessment of a patient with pneumonia, the nurse identifies a nursing diagnosis of ineffective airway clearance. Which assessment data best supports this diagnosis? a. Weak, nonproductive cough effort b. Large amounts of greenish sputum c. Respiratory rate of 28 breaths/minute d. Resting pulse oximetry (SpO2) of 85%

a. Weak, nonproductive cough effort The weak, nonproductive cough indicates that the patient is unable to clear the airway effectively. The other data would be used to support diagnoses such as impaired gas exchange and ineffective breathing pattern.

Symptoms of pneumonia include? (select all that apply) a. Weakness b. Non-productive cough c. Rust colored sputum d. Fever

a. Weakness c. Rust colored sputum d. Fever

A few days after experiencing a myocardial infarction (MI) and successful percutaneous coronary intervention, the patient states, I just had a little chest pain. As soon as I get out of here, I'm going for my vacation as planned. Which reply would be most appropriate for the nurse to make? a. What do you think caused your chest pain? b. Where are you planning to go for your vacation? c. Sometimes plans need to change after a heart attack. d. Recovery from a heart attack takes at least a few weeks.

a. What do you think caused your chest pain? When the patient is experiencing denial, the nurse should assist the patient in testing reality until the patient has progressed beyond this step of the emotional adjustment to MI. Asking the patient about vacation plans reinforces the patients plan, which is not appropriate in the immediate post-MI period. Reminding the patient in denial about the MI is likely to make the patient angry and lead to distrust of the nursing staff.

The nurse identifies the nursing diagnosis of activity intolerance for a patient with asthma. In patients with asthma, the nurse assesses for which etiologic factor for this nursing diagnosis? a. Work of breathing b. Fear of suffocation c. Effects of medications d. Anxiety and restlessness

a. Work of breathing When the patient does not have sufficient gas exchange to engage in activity, the etiologic factor is often the work of breathing. When patients with asthma do not have effective respirations, they use all available energy to breathe and have little left over for purposeful activity. Fear of suffocation, effects of medications or anxiety, and restlessness are not etiologies for activity intolerance for a patient with asthma.

An older patient is receiving standard multidrug therapy for tuberculosis (TB). The nurse should notify the health care provider if the patient exhibits which finding? a. Yellow-tinged skin b. Orange-colored sputum c. Thickening of the fingernails d. Difficulty hearing high-pitched voices

a. Yellow-tinged skin Noninfectious hepatitis is a toxic effect of isoniazid (INH), rifampin, and pyrazinamide, and patients who develop hepatotoxicity will need to use other medications. Changes in hearing and nail thickening are not expected with the four medications used for initial TB drug therapy. Presbycusis is an expected finding in the older adult patient. Orange discoloration of body fluids is an expected side effect of rifampin and not an indication to call the health care provider.

A client asks the nurse how long injections of vitamin B12 will be needed. Which response by the nurse is best? a. You may need lifelong injections b. You may need 6 months injections c. You may need 24 hours injections d. You may need 3 days injections

a. You may need lifelong injections

Priority Decision: The nurse is preparing to administer a blood transfusion. Number the actions in order of priority (1 is first priority action; 10 is last priority action). a. ________ Verify the order for the transfusion. b. ________ Ensure that the patient has a patent 16- to 22-gauge IV. c. ________ Prime the transfusion tubing and filter with normal saline. d. ________ Verify that the physician has discussed risks, benefits, and alternatives with the patient. e. ________ Obtain the blood product from the blood bank. f. ________ Ask another licensed person (nurse or MD) to assist in verifying the product identification and the patient identification. g. ________ Document outcomes in the patient record. Document vital signs, names of personnel, and starting and ending times. h. ________ Adjust the infusion rate and continue to monitor the patient every 30 minutes for up to an hour after the product is infused. i. ________ Infuse the first 50 mL over 15 minutes, staying with the patient. j. ________ Obtain the patient's vital signs before starting the transfusion.

a. ____1____ Verify the order for the transfusion. d. ___2_____ Verify that the physician has discussed risks, benefits, and alternatives with the patient. b. ___3____ Ensure that the patient has a patent 16- to 22-gauge IV. c. ___4_____ Prime the transfusion tubing and filter with normal saline. j. ____5____ Obtain the patient's vital signs before starting the transfusion. e. __6______ Obtain the blood product from the blood bank. f. ___7_____ Ask another licensed person (nurse or MD) to assist in verifying the product i. ____8____ Infuse the first 50 mL over 15 minutes, staying with the patient. identification and the patient identification. h. __9______ Adjust the infusion rate and continue to monitor the patient every 30 minutes for up to an hour after the product is infused. g. __10______ Document outcomes in the patient record. Document vital signs, names of personnel, and starting and ending times.

A patient is admitted to the hospital with possible acute pericarditis. The nurse should plan to teach the patient about the purpose of a. echocardiography. b. daily blood cultures. c. cardiac catheterization. d. 24-hour Holter monitor.

a. echocardiography. Echocardiograms are useful in detecting the presence of the pericardial effusions associated with pericarditis. Blood cultures are not indicated unless the patient has evidence of sepsis. Cardiac catheterization and 24-hour Holter monitor is not a diagnostic procedure for pericarditis.

A young adult patient who denies any history of smoking is seen in the clinic with a new diagnosis of chronic obstructive pulmonary disease (COPD). It is most appropriate for the nurse to teach the patient about a. a 1-antitrypsin testing. b. use of the nicotine patch. c. continuous pulse oximetry. d. effects of leukotriene modifiers.

a. a 1-antitrypsin testing. When COPD occurs in young patients, especially without a smoking history, a genetic deficiency in a1-antitrypsin should be suspected. Because the patient does not smoke, a nicotine patch would not be ordered. There is no indication that the patient requires continuous pulse oximetry. Leukotriene modifiers would be used in patients with asthma, not with COPD.

A student nurse asks the RN what can be measured by arterial blood gas (ABG). The RN tells the student that the ABG can measure (select all that apply) a. acid-base balance. b. oxygenation status. c. acidity of the blood. d. bicarbonate (HCO3-). e. compliance and resistance.

a. acid-base balance. b. oxygenation status. c. acidity of the blood. d. bicarbonate (HCO3-).

During a visit to a 78-year-old with chronic heart failure, the home care nurse finds that the patient has ankle edema, a 2-kg weight gain over the past 2 days, and complains of feeling too tired to get out of bed. Based on these data, the best nursing diagnosis for the patient is a. activity intolerance related to fatigue. b. disturbed body image related to weight gain. c. impaired skin integrity related to ankle edema. d. impaired gas exchange related to dyspnea on exertion.

a. activity intolerance related to fatigue. The patients statement supports the diagnosis of activity intolerance. There are no data to support the other diagnoses, although the nurse will need to assess for other patient problems.

A patient with dilated cardiomyopathy has new onset atrial fibrillation that has been unresponsive to drug therapy for several days. The priority teaching needed for this patient would include information about a. anticoagulant therapy. b. permanent pacemakers. c. electrical cardioversion. d. IV adenosine (Adenocard).

a. anticoagulant therapy. Atrial fibrillation therapy that has persisted for more than 48 hours requires anticoagulant treatment for 3 weeks before attempting cardioversion. This is done to prevent embolization of clots from the atria. Cardioversion may be done after several weeks of anticoagulation therapy. Adenosine is not used to treat atrial fibrillation. Pacemakers are routinely used for patients with brady dysrhythmias. Information does not indicate that the patient has a slow heart rate.

Propranolol (Inderal) is prescribed for a patient diagnosed with hypertension. The nurse should consult with the health care provider before giving this medication when the patient reveals a history of a. asthma. b. daily alcohol use. c. peptic ulcer disease. d. myocardial infarction (MI).

a. asthma. Nonselective b-blockers block b1 and b2-adrenergic receptors and can cause bronchospasm, especially in patients with a history of asthma. b-Blockers will have no effect on the patients peptic ulcer disease or alcoholuse. b-Blocker therapy is recommended after MI.

When a person's blood pressure rises, the homeostatic mechanism to compensate for an elevation involves stimulation of a. baroreceptors that inhibit the sympathetic nervous system, causing vasodilation. b. chemoreceptors that inhibit the sympathetic nervous system, causing vasodilation. c. baroreceptors that inhibit the parasympathetic nervous system, causing vasodilation. d. chemoreceptors that stimulate the sympathetic nervous system, causing an increased heart rate.

a. baroreceptors that inhibit the sympathetic nervous system, causing vasodilation.

To auscultate for S3 or S4 gallops in the mitral area, the nurse listens with the a. bell of the stethoscope with the patient in the left lateral position. b. diaphragm of the stethoscope with the patient in a supine position. c. bell of the stethoscope with the patient sitting and leaning forward. d. diaphragm of the stethoscope with the patient lying flat on the left side.

a. bell of the stethoscope with the patient in the left lateral position. Gallop rhythms generate low-pitched sounds and are most easily heard with the bell of the stethoscope. Sounds associated with the mitral valve are accentuated by turning the patient to the left side, which brings the heart closer to the chest wall. The diaphragm of the stethoscope is best to use for the higher-pitched sounds such as S1 and S2

Post bronchoscopy priority findings to report to the provider immediately?(Select all that apply) a. bright red blood tinged sputum b. dysphagia c. sore throat and hoarseness d. no gag reflex after thirty minutes e. stridor and increased dyspnea

a. bright red blood tinged sputum b. dysphagia e. stridor and increased dyspnea

Direction of fluid shift "from blood vessels to interstitium" and mechanism of fluid movement "plasma hydrostatic pressure" matches which event factor? a. burns b. dehydration c. fluid overload d. hyponatremia

a. burns

The key anatomic landmark that separates the upper respiratory tract from the lower respiratory tract is the a. carina. b. larynx. c. trachea. d. epiglottis.

a. carina.

An appropriate nursing intervention for a patient with non-Hodgkins lymphoma whose platelet count drops to 18,000/L during chemotherapy is to a. check all stools for occult blood. b. encourage fluids to 3000 mL/day. c. provide oral hygiene every 2 hours. d. check the temperature every 4 hours.

a. check all stools for occult blood. Because the patient is at risk for spontaneous bleeding, the nurse should check stools for occult blood. A low platelet count does not require an increased fluid intake. Oral hygiene is important, but it is not necessary to provide oral care every 2 hours. The low platelet count does not increase risk for infection, so frequent temperature monitoring is not indicated.

The best method for determining the risk for aspiration in a patient with a tracheostomy is to a. consult a speech therapist for swallowing assessment. b. have the patient drink plain water and assess for coughing. c. ask the patient to rate the perceived degree of swallowing difficulty. d. assess for sputum changes 48 hours after the patient drinks small amount of blue dye.

a. consult a speech therapist for swallowing assessment.

When caring for a patient with thrombocytopenia, the nurse instructs the patient to a. dab his or her nose instead of blowing. b. be careful when shaving with a safety razor. c. continue with physical activities to stimulate thrombopoiesis. d. avoid aspirin because it may mask the fever that occurs with thrombocytopenia.

a. dab his or her nose instead of blowing.

The plan of care for the patient with chronic obstructive pulmonary disease (COPD) should include (select all that apply) a. exercise such as walking. b. high flow rate of O2 administration. c. low-dose chronic oral corticosteroid therapy. d. use of peak flow meter to monitor the progression of COPD. e. breathing exercises, such as pursed-lip breathing that focus on exhalation.

a. exercise such as walking. e. breathing exercises, such as pursed-lip breathing that focus on exhalation.

The nursing care for a patient with hyponatremia and fluid volume excess includes a. fluid restriction. b. administration of hypotonic IV fluids. c. administration of a cation-exchange resin. d. placement of an indwelling urinary catheter.

a. fluid restriction.

A patient in the intensive care unit with acute decompensated heart failure (ADHF) complains of severe dyspnea and is anxious, tachypneic, and tachycardic. All of the following medications have been ordered for the patient. The nurses priority action will be to a. give IV morphine sulfate 4 mg. b. give IV diazepam (Valium) 2.5 mg. c. increase nitroglycerin (Tridil) infusion by 5 mcg/min. d. increase dopamine (Intropin) infusion by 2 mcg/kg/min.

a. give IV morphine sulfate 4 mg. Morphine improves alveolar gas exchange, improves cardiac output by reducing ventricular preload and afterload, decreases anxiety, and assists in reducing the subjective feeling of dyspnea. Diazepam may decrease patient anxiety, but it will not improve the cardiac output or gas exchange. Increasing the dopamine may improve cardiac output, but it will also increase the heart rate and myocardial oxygen consumption. Nitroglycerin will improve cardiac output and may be appropriate for this patient, but it will not directly reduce anxiety and will not act as quickly as morphine to decrease dyspnea.

Nursing care for a patient immediately after a bone marrow biopsy and aspiration includes (select all that apply) a. giving analgesics as needed. b. preparing to start a blood transfusion. c. giving pre-procedure and post-procedure antibiotic medications. d. having the patient lie still to keep the sterile pressure dressing intact. e. monitoring vital signs and assessing the site for excess drainage or bleeding.

a. giving analgesics as needed. d. having the patient lie still to keep the sterile pressure dressing intact. e. monitoring vital signs and assessing the site for excess drainage or bleeding.

Which of these electrolyte imbalances is the cause of "diabetes insipidus"? (select all that apply) a. hypernatremia b. hyponatremia c. hyperkalemia d. hypokalemia e. hypercalcemia f. hypocalcemia g. hyperphosphatemia h. hypophosphatemia i. hypermagnesemia j. hypomagnesemia

a. hypernatremia

Which of these electrolyte imbalances is the cause of "loop and thiazide diuretics"? (select all that apply) a. hypernatremia b. hyponatremia c. hyperkalemia d. hypokalemia e. hypercalcemia f. hypocalcemia g. hyperphosphatemia h. hypophosphatemia i. hypermagnesemia j. hypomagnesemia

a. hypernatremia b. hyponatremia d. hypokalemia e. hypercalcemia f. hypocalcemia

Which of these electrolyte imbalances is the cause of "hyperaldosteronism"? (select all that apply) a. hypernatremia b. hyponatremia c. hyperkalemia d. hypokalemia e. hypercalcemia f. hypocalcemia g. hyperphosphatemia h. hypophosphatemia i. hypermagnesemia j. hypomagnesemia

a. hypernatremia d. hypokalemia

Which of these electrolyte imbalances is the cause of "osmotic diuresis"? (select all that apply) a. hypernatremia b. hyponatremia c. hyperkalemia d. hypokalemia e. hypercalcemia f. hypocalcemia g. hyperphosphatemia h. hypophosphatemia i. hypermagnesemia j. hypomagnesemia

a. hypernatremia d. hypokalemia

Which of these electrolyte imbalances is the cause of "fleet enemas"? (select all that apply) a. hypernatremia b. hyponatremia c. hyperkalemia d. hypokalemia e. hypercalcemia f. hypocalcemia g. hyperphosphatemia h. hypophosphatemia i. hypermagnesemia j. hypomagnesemia

a. hypernatremia d. hypokalemia f. hypocalcemia g. hyperphosphatemia

Factors that cause sodium retention_____,_______,_______& cause potassium excretion. Select all that apply. a. hyponatremia b. low blood volume c. aldosterone excretion d. hypotonic IV fluids

a. hyponatremia b. low blood volume c. aldosterone excretion

The nurse caring for a patient with type A hemophilia being admitted to the hospital with severe pain and swelling in the right knee will a. immobilize the joint. b. apply heat to the knee. c. assist the patient with light weight bearing. d. perform passive range of motion to the knee.

a. immobilize the joint. The initial action should be total rest of the knee to minimize bleeding. Ice packs are used to decrease bleeding. Range of motion (ROM) and weight-bearing exercise are contraindicated initially, but after the bleeding stops, ROM and physical therapy are started.

The charge nurse observes a new registered nurse (RN) doing discharge teaching for a patient with hypertension who has a new prescription for enalapril (Vasotec). The charge nurse will need to intervene if the new RN tells the patient to a. increase the dietary intake of high-potassium foods. b. make an appointment with the dietitian for teaching. c. check the blood pressure (BP) with a home BP monitor at least once a day. d. move slowly when moving from lying to sitting to standing.

a. increase the dietary intake of high-potassium foods. The ACE inhibitors cause retention of potassium by the kidney, so hyperkalemia is a possible adverse effect. The other teaching by the new RN is appropriate for a patient with newly diagnosed hypertension who has just started therapy with enalapril.

The lungs act as an acid-base buffer by a. increasing respiratory rate and depth when CO2 levels in the blood are high, reducing acid load. b. increasing respiratory rate and depth when CO2 levels in the blood are low, reducing base load. c. decreasing respiratory rate and depth when CO2 levels in the blood are high, reducing acid load. d. decreasing respiratory rate and depth when CO2 levels in the blood are low, increasing acid load.

a. increasing respiratory rate and depth when CO2 levels in the blood are high, reducing acid load.

Nursing interventions for a patient with severe anemia related to peptic ulcer disease include (select all that apply) a. instructions for high-iron diet. b. taking vital signs every 8 hours. c. monitoring stools for occult blood. d. teaching self-injection of erythropoietin. e. administration of cobalamin (vitamin B12) injections.

a. instructions for high-iron diet. c. monitoring stools for occult blood.

The nurse suspects stable angina rather than MI pain in the patient who reports that his chest pain a. is relieved by NTG. b. is a sensation of tightness or squeezing. c. does not radiate to the neck, back, or arms. d. is precipitated by physical or emotional exertion.

a. is relieved by NTG. One of the primary differences between the pain of angina and the pain of an MI is that angina pain is usually relieved by rest or NTG, which reduces the oxygen demand of the heart, while MI pain is not. Both angina and MI pain can cause a pressure or squeezing sensation; may or may not radiate to the neck, back, arms, fingers, and jaw; and may be precipitated by exertion.

Signs of fluid volume excess: Select all that apply. a. jugular venous distention b. bounding pulses c. pulmonary edema d. orthostatic hypotension

a. jugular venous distention b. bounding pulses c. pulmonary edema

Direction of fluid shift "from blood vessels to interstitium" and mechanism of fluid movement "oncotic pressure" matches which event factor? a. low serum albumin b. administration of 10% glucose c. application of elastic bandages d. dehydration

a. low serum albumin

When reviewing laboratory results of an older patient with an infection, the nurse would expect to find a. mild leukocytosis. b. decreased platelet count. c. increased hemoglobin and hematocrit levels. d. decreased erythrocyte sedimentation rate (ESR).

a. mild leukocytosis.

The nursing management of a patient in sickle cell crisis includes (select all that apply) a. monitoring CBC. b. optimal pain management and O2 therapy. c. blood transfusions if needed and iron chelation. d. rest as needed and deep vein thrombosis prophylaxis. e. administration of IV iron and diet high in iron content.

a. monitoring CBC. b. optimal pain management and O2 therapy. c. blood transfusions if needed and iron chelation. d. rest as needed and deep vein thrombosis prophylaxis.

The nurse suspects cardiac tamponade in a patient who has acute pericarditis. To assess for the presence of pulsus paradoxus, the nurse should a. note when Korotkoff sounds are auscultated during both inspiration and expiration. b. subtract the diastolic blood pressure (DBP) from the systolic blood pressure (SBP). c. check the electrocardiogram (ECG) for variations in rate during the respiratory cycle. d. listen for a pericardial friction rub that persists when the patient is instructed to stop breathing.

a. note when Korotkoff sounds are auscultated during both inspiration and expiration. Pulsus paradoxus exists when there is a gap of greater than 10 mm Hg between when Korotkoff sounds can be heard during only expiration and when they can be heard throughout the respiratory cycle. The other methods described would not be useful in determining the presence of pulsus paradoxus.

During administration of a hypertonic IV solution, the mechanism involved in equalizing the fluid concentration between ECF and the cells is a. osmosis. b. diffusion. c. active transport. d. facilitated diffusion.

a. osmosis.

A nurse is caring for a client who requires nasogastric suctioning. Which of the following set of laboratory results indicates that the client has metabolic alkalosis? a. pH 7.51, PaO2 94 mm Hg, PaCO2 36 mm Hg, HCO3- 31 mEq/L b. pH 7.48, PaO2 89 mm Hg, PaCO2 30 mm Hg, HCO3- 26 mEq/L c. pH 7.31, PaO2 77 mm Hg, PaCO2 52 mm Hg, HCO3- 23 mEq/L d. pH 7.26, PaO2 84 mm Hg, PaCO2 38 mm Hg, HCO3- 20 mEq/L

a. pH 7.51, Pa02 94 mm Hg, PaC02 36 mm Hg, HCO3- 31 mEq/L An elevated pH and HCO3- with a PaCO2 within the expected reference range indicates metabolic alkalosis.

An elevated level of this electrolyte is common in acute kidney injury & chronic kidney disease. a. phosphate b. calcium c. sodium d. magnesium

a. phosphate

The standard policy on the cardiac unit states, Notify the health care provider for mean arterial pressure (MAP) less than 70 mm Hg. The nurse will need to call the health care provider about the a. postoperative patient with a BP of 116/42. b. newly admitted patient with a BP of 150/87. c.patient with left ventricular failure who has a BP of 110/70. d. patient with a myocardial infarction who has a BP of 140/86.

a. postoperative patient with a BP of 116/42. The mean arterial pressure (MAP) is calculated using the formula MAP = (systolic BP + 2 diastolic BP)/3. The MAP for the postoperative patient in answer 3 is 67. The MAP in the other three patients is higher than 70 mm Hg

While caring for a patient with aortic stenosis, the nurse identifies a nursing diagnosis of acute pain related to decreased coronary blood flow. A priority nursing intervention for this patient would be to a. promote rest to decrease myocardial oxygen demand. b. teach the patient about the need for anticoagulant therapy. c. teach the patient to use sublingual nitroglycerin for chest pain. d. raise the head of the bed 60 degrees to decrease venous return.

a. promote rest to decrease myocardial oxygen demand. Rest is recommended to balance myocardial oxygen supply and demand and to decrease chest pain. The patient with aortic stenosis requires higher preload to maintain cardiac output, so nitroglycerin and measures to decrease venous return are contraindicated. Anticoagulation is not recommended unless the patient has atrial fibrillation.

Which of the following acid-base imbalances is the common cause for respiratory failure? a. respiratory acidosis b. respiratory alkalosis c. metabolic alkalosis d. metabolic acidosis

a. respiratory acidosis

Which of the following acid-base imbalances is the common cause for sedative or opioid overdose? a. respiratory acidosis b. respiratory alkalosis c. metabolic alkalosis d. metabolic acidosis

a. respiratory acidosis

Which of the following acid-base imbalances matches the mechanism "increased carbonic acid (H2CO3)"? a. respiratory acidosis b. respiratory alkalosis c. metabolic alkalosis d. metabolic acidosis

a. respiratory acidosis

Which of the following are cations? Select all that apply. a. sodium b. phosphate c. calcium d. potassium e. bicarbonate f. chloride

a. sodium c. calcium d. potassium

Which patient is at highest risk for venous thromboembolism (VTE)? a. A 62-yr-old man with spider veins who is having arthroscopic knee surgery b. A 32-yr-old woman who smokes, takes oral contraceptives, and is planning a trip to Europe c. A 26-yr-old woman who is 3 days postpartum and received maintenance IV fluids for 12 hours during her labor d. An active 72-yr-old man at home recovering from transurethral resection of the prostate for benign prostatic hyperplasia

b. A 32-yr-old woman who smokes, takes oral contraceptives, and is planning a trip to Europe

Priority nursing actions when caring for a hospitalized patient with a new-onset temperature of 102.2°F (39°C) and severe neutropenia include (select all that apply) a. starting the prescribed antibiotic STAT. b. drawing peripheral and central line blood cultures. c. ongoing monitoring of the patient's vital signs for septic shock. d. taking a full set of vital signs and notifying the physician immediately. e. administering transfusions of WBCs treated to decrease immunogenicity.

a. starting the prescribed antibiotic STAT. b. drawing peripheral and central line blood cultures. c. ongoing monitoring of the patient's vital signs for septic shock. d. taking a full set of vital signs and notifying the physician immediately.

A nurse is assessing a client following a gunshot wound to the chest. For which of the following finders should the nurse monitor to detect a pneumothorax? (Select all that apply) a. tachypnea b. deviation of the trachea c. bradycardia d. decreased use of accessory muscles e. pleuritic pain

a. tachypnea b. deviation of the trachea e. pleuritic pain

The nurse teaches the patient with any venous disorder that the best way to prevent venous stasis and increase venous return is to a. take short walks. b. sit with the legs elevated. c. frequently rotate the ankles. d. always wear elastic compression stockings.

a. take short walks. During walking, the muscles of the legs continuously knead the veins, promoting movement of venous blood toward the heart. Walking is the best measure to prevent venous stasis and will be increased gradually. Elevating the legs will decrease edema. The other methods will help venous return, but they do not provide the benefit that ambulation does.

These are clinical findings of fluid volume deficit: Select all that apply. a. tenting of the skin b. increased RR and temperature c. muscle weakness d. elevated BP

a. tenting of the skin b. increased RR and temperature c. muscle weakness

A complication of the hyperviscosity of polycythemia is a. thrombosis. b. cardiomyopathy. c. pulmonary edema. d. disseminated intravascular coagulation (DIC).

a. thrombosis.

When teaching a patient about the long-term consequences of rheumatic fever, the nurse should discuss the possibility of a. valvular heart disease. b. pulmonary hypertension. c. superior vena cava syndrome. d. hypertrophy of the right ventricle.

a. valvular heart disease.

Symptoms of hypernatremia with decreased ECF volume: Select all that apply. a. weight loss b. elevated BP c. agitation, seizures, coma d. hypotension e. pulmonary edema f. flushed skin

a. weight loss c. agitation, seizures, coma d. hypotension f. flushed skin

An older woman is admitted to the medical unit with GI bleeding. Assessment findings that indicate fluid volume deficit include (select all that apply) a. weight loss. b. dry oral mucosa. c. full bounding pulse. d. engorged neck veins. e. decreased central venous pressure.

a. weight loss. b. dry oral mucosa. e. decreased central venous pressure.

The nurse is caring for a patient who was admitted to the coronary care unit following an acute myocardial infarction (AMI) and percutaneous coronary intervention the previous day. Teaching for this patient would include a. when cardiac rehabilitation will begin. b. the typical emotional responses to AMI. c. information regarding discharge medications. d. the pathophysiology of coronary artery disease.

a. when cardiac rehabilitation will begin. Early after an AMI, the patient will want to know when resumption of usual activities can be expected. At this time, the patients anxiety level or denial will interfere with good understanding of complex information such as the pathophysiology of coronary artery disease (CAD). Teaching about discharge medications should be done closer to discharge. The nurse should support the patient by decreasing anxiety rather than discussing the typical emotional responses to myocardial infarction (MI).

a nurse is assisting a provider who's performing a thoracentesis at the beside of a client. which of the following actions should the nurse take? a.) Wear goggles and a mask during the procedure. b.) Cleanse the procedure area with an antiseptic solution. c.) Instruct the client to take deep breaths during the procedure. d.) Position the client laterally on the affected side before the procedure. e.) Apply pressure to the site after the procedure.

a.) Wear goggles and a mask during the procedure. b.) Cleanse the procedure area with an antiseptic solution. e.) Apply pressure to the site after the procedure. rationale: a.) Wear goggles and a mask during the procedure is correct. The nurse and provider should both wear goggles and a mask to reduce the risk for exposure to pleural fluid. b.) Cleanse the procedure area with an antiseptic solution is correct. The use of an antiseptic solution decreases the risk for infection, which is increased due to the invasive nature of the procedure. c.) Instruct the client to take deep breaths during the procedure is incorrect. The nurse should instruct the client to remain as still as possible during the procedure to reduce the risk for puncturing the pleura or lung. d.) Position the client laterally on the affected side before the procedure is incorrect. The nurse should position the client in a sitting position leaning over the bedside table or laterally on the unaffected side to promote access to the site and encourage drainage of pleural fluid. e.) Apply pressure to the site after the procedure is correct. The application of pressure decreases the risk for bleeding at the procedure site.

The nurse is reviewing the laboratory test results for a 68-yr-old patient whose warfarin (Coumadin) therapy was terminated during the preoperative period. On postoperative day 2, the international normalized ratio (INR) result is 2.7. Which action by the nurse is most appropriate? a Hold the daily dose of warfarin. b Administer the daily dose of warfarin. c Teach the patient signs and symptoms of bleeding. d Call the physician to request an increased dose of warfarin.

b Administer the daily dose of warfarin. The therapeutic range for INR is 2.0 to 3.0 for many clinical diagnoses. To maintain therapeutic values, the nurse will administer the medication as ordered. Holding the medication would lower the INR, which would increase the risk of clot formation. Conversely, the higher the INR is, the more prolonged the clotting time. Calling the health care provider is not indicated. Although teaching is important, administering the medication is a higher priority at this time.

A patient has this rhythm, which interventions would the nurse need to perform? (select all that apply) a. Give diuretics for fluid overload. b. Check for dehydration. c. Give oxygen for hypoxia. d. Treat the pt for an elevated temperature.

b. Check for dehydration. c. Give oxygen for hypoxia. d. Treat the pt for an elevated temperature.

Which assessment findings of the left lower extremity will the nurse identify as consistent with arterial occlusion (select all that apply.)? a Edematous b Cold and mottled c Complaints of paresthesia d Pulse not palpable with Doppler e Capillary refill less than three seconds f Erythema and warmer than right lower extremity

b, c, d- Arterial occlusion may result in loss of limb if not timely revascularized. When an artery is occluded, perfusion to the extremity is impaired or absent. On assessment, the nurse would note a cold, mottled extremity with impaired sensation or numbness. The pulse would not be identified, even with a Doppler. In contrast, the nurse would find edema, erythema, and increased warmth in the presence of a venous occlusion (deep vein thrombosis). Capillary refill would be greater than 3 seconds in an arterial occlusion and less than 3 seconds with a venous occlusion.

A patient with rheumatic heart disease with carditis asks the nurse how long his activity will be restricted. What is the best answer by the nurse? a. "Full activity will be allowed as soon as acute symptoms have subsided." b. "Bed rest will be continued until symptoms of heart failure are controlled." c. "Nonstrenuous activities can be performed as soon as antibiotics are started." d. "Bed rest must be maintained until antiinflammatory therapy has been discontinued."

b. "Bed rest will be continued until symptoms of heart failure are controlled." When carditis is present in the patient with rheumatic fever, ambulation is postponed until any symptoms of heart failure are controlled with treatment, and activity cannot be resumed until acute inflammation has subsided. In the patient without cardiac involvement, ambulation may be permitted as soon as acute symptoms have subsided, and normal activity can be resumed when antiinflammatory therapy is discontinued.

A nurse is providing teaching about lifestyle changes to a client who had a myocardial infarction and has a new prescription for a beta blocker. Which of the following client statements indicates an understanding of the teaching? a. "I should eat food that are high in saturated in fat." b. "Before taking my medication, I will count my radial pulse rate." c. "I will exercise once a week for an hour at the health club." d. "I will stop taking my medication when my blood pressure is within normal range"

b. "Before taking my medication, I will count my radial pulse rate." A beta blocker will induce bradycardia. The client should take her pulse rate for 1 min before self-administration.

A patient with a history of hypertension treated with a diuretic and an angiotensin-converting enzyme (ACE) inhibitor arrives in the emergency department complaining of a severe headache and nausea and has a blood pressure (BP) of 238/118 mm Hg. Which question should the nurse ask to follow up on these findings? a. "Have you recently taken any antihistamines?" b. "Have you consistently taken your medications?" c. "Did you take any acetaminophen (Tylenol) today?" d. "Have there been recent stressful events in your life?"

b. "Have you consistently taken your medications?" Sudden withdrawal of antihypertensive medications can cause rebound hypertension and hypertensive crisis. Although many over-the-counter medications can cause hypertension, antihistamines and acetaminophen do not increase BP. Stressful events will increase BP but not usually to the level seen in this patient.

After teaching a patient with chronic stable angina about nitroglycerin, the nurse recognizes the need for further teaching when the patient makes which statement? a. "I will replace my nitroglycerin supply every 6 months." b. "I can take up to five tablets every 3 minutes for relief of my chest pain." c. "I will take acetaminophen (Tylenol) to treat the headache caused by nitroglycerin." d. "I will take the nitroglycerin 10 minutes before planned activity that usually causes chest pain."

b. "I can take up to five tablets every 3 minutes for relief of my chest pain." The recommended dose of nitroglycerin is one tablet taken sublingually (SL) or one metered spray for symptoms of angina. If symptoms are unchanged or worse after 5 minutes, the patient should be instructed to activate the emergency medical services (EMS) system. If symptoms are improved, repeat the nitroglycerin every 5 minutes for a maximum of three doses and contact EMS if symptoms have not resolved completely.

The nurse recognizes that additional teaching is needed when the patient with asthma says a. "I should exercise every day if my symptoms are controlled." b. "I may use over-the-counter bronchodilator drugs occasionally if I develop chest tightness." c. "I should inform my spouse about my medications and how to get help if I have a severe asthma attack." d. "A diary to record my medication use, symptoms, PEF rates, and activity levels will help in adjusting my therapy."

b. "I may use over-the-counter bronchodilator drugs occasionally if I develop chest tightness." Nonprescription drugs should not be used by patients with asthma because of dangers associated with rebound bronchospasm, interactions with prescribed drugs, and undesirable side effects. All the other responses are appropriate for the patient with asthma.

After teaching about ways to decrease risk factors for CAD, the nurse recognizes that further instruction is needed when the patient says a. "I can keep my blood pressure normal with medication." b. "I would like to add weight lifting to my exercise program." c. "I can change my diet to decrease my intake of saturated fats." d. "I will change my lifestyle to reduce activities that increase my stress."

b. "I would like to add weight lifting to my exercise program."

The patient with VTE is receiving therapy with heparin and asks the nurse whether the drug will dissolve the clot in her leg. What is the best response by the nurse? a. "This drug will break up and dissolve the clot so that circulation in the vein can be restored." b. "The purpose of the heparin is to prevent growth of the clot or formation of new clots where the circulation is slowed." c. "Heparin won't dissolve the clot, but it will inhibit the inflammation around the clot and delay the development of new clots." d. "The heparin will dilate the vein, preventing turbulence of blood flow around the clot that may cause it to break off and travel to the lungs."

b. "The purpose of the heparin is to prevent growth of the clot or formation of new clots where the circulation is slowed." Anticoagulant therapy with heparin or warfarin (Coumadin) does not dissolve clots but prevents propagation of the clot, development of new thrombi, and embolization. Clot lysis occurs naturally through the body's intrinsic fibrinolytic system or by the administration of thrombolytic agents.

A hospitalized patient with a history of chronic stable angina tells the nurse that she is having chest pain. The nurse bases his actions on the knowledge that ischemia a. will always progress to myocardial infarction. b. can be relieved by rest, nitroglycerin, or both. c. is often associated with vomiting and extreme fatigue. d. indicates that irreversible myocardial damage is occurring.

b. can be relieved by rest, nitroglycerin, or both.

A client with pernicious anemia asks why she must take vitamin B12 injections for the rest of her life. What is the nurse's best response? a."The reason for your vitamin deficiency is an inability to absorb the vitamin because the stomach is not producing sufficient acid." b."The reason for your vitamin deficiency is an inability to absorb the vitamin because the stomach is not producing sufficient intrinsic factor." c."The reason for your vitamin deficiency is an excessive excretion of the vitamin because of kidney dysfunction." d."The reason for your vitamin deficiency is an increased requirement for the vitamin because of rapid red blood cell production."

b. "The reason for your vitamin deficiency is an inability to absorb the vitamin because the stomach is not producing sufficient intrinsic factor." Most clients with pernicious anemia have deficient production of intrinsic factor in the stomach. Intrinsic factor attaches to the vitamin in the stomach and forms a complex that allows the vitamin to be absorbed in the small intestine. The stomach is producing enough acid, there is not an excessive excretion of the vitamin, and there is not a rapid production of RBCs in this condition.

A nurse is admitting a client who takes 40 mg furosemide daily for heart failure and has experienced 3 days of vomiting. The nurse suspects hypokalemia. Which of the following medications should the nurse prepare to administer? a. Sodium polystyrene sulfonate 30 g/day b. 0.9% sodium chloride with 10 mEq/L of potassium chloride at 100 mL/hr c. Bumetanide 8 mg/day d. 100 mL of dextrose 10% in water with 10 units of insulin

b. 0.9% sodium chloride with 10 mEq/L of potassium chloride at 100 mL/hr This IV solution will provide adequate fluid and potassium replacement to offset the losses from vomiting. The typical amount of potassium chloride to administer IV is 5 to 10 mEq/hr and not to exceed 20 mEq/hr. The dilution should be 1 mEq to 10 mL of 0.9% sodium chloride.

Ordered Response: Reaction to blood transfusion. List from 1-6 the order of how a blood transfusion is performed if a reaction occurs. a. Hang new IV infusion set b. STOP transfusion c. Call HCP d. Start 0.9% sodium chloride infusion e. Assess BP, HR, RR f. Obtain blood and urine specimens

b. 1 - STOP transfusion a. 2 - Hang new IV infusion set d. 3 - Start 0.9% sodium chloride infusion c. 4 - Call HCP e. 5 - Assess BP, HR, RR f. 6 - Obtain blood and urine specimens

A nurse is providing dietary teaching to a client who has heart failure and is receiving furosemide. Which of the following foods should the nurse recommend as containing the greatest amount of potassium? a. 1/2 cup chopped celery b. 1 cup plain yogurt c. 1 slice whole grain bread d. 1/2 cup cooked tofu

b. 1 cup plain yogurt One cup of plain yogurt contains 380 g of potassium. Therefore, the nurse should recommend this food as containing the greatest amount of potassium.

What is a normal magnesium level? a. 3.0-4.5 b. 1.3-2.1 c. 1.0-2.0 d. 2.0-3.0

b. 1.3-2.1

What is the normal range for Hgb? a. 8-9 b. 12-18 c. 1-7 d. 3-10

b. 12-18

Which blood pressure (BP) finding by the nurse indicates that no changes in therapy are needed for a patient with stage 1 hypertension who has a history of diabetes mellitus? a. 102/60 mm Hg b. 128/76 mm Hg c. 139/90 mm Hg d. 136/82 mm Hg

b. 128/76 mm Hg The goal for antihypertensive therapy for a patient with hypertension and diabetes mellitus is a BP <130/80 mm Hg. The BP of 102/60 may indicate overtreatment of the hypertension and an increased risk for adverse drug effects. The other two blood pressures indicate a need for modifications in the patients treatment.

One liter of fluid is equal to: a. 1.5 lbs. b. 2.2 lbs. c. 3.7 lbs. d. 4.4 lbs.

b. 2.2 lbs.

Several patients call the outpatient clinic and ask to make an appointment as soon as possible. Which patient should the nurse schedule to be seen first? a. 44-year-old with sickle cell anemia who says my eyes always look sort of yellow b. 23-year-old with no previous health problems who has a nontender lump in the axilla c. 50-year-old with early-stage chronic lymphocytic leukemia who reports chronic fatigue d. 19-year-old with hemophilia who wants to learn to self-administer factor VII replacement

b. 23-year-old with no previous health problems who has a nontender lump in the axilla The patients age and presence of a nontender axillary lump suggest possible lymphoma, which needs rapid diagnosis and treatment. The other patients have questions about treatment or symptoms that are consistent with their diagnosis but do not need to be seen urgently.

Which of the following tests is the gold standard for diagnosing TB? a. Interferon Release assays b. 3 consecutive sputum specimens c. Purified protein derivative (PPD) d. Chest x-ray

b. 3 consecutive sputum specimens

When using CURB-65, how many points would a 70 yr old with RR-32 & confusion score? a. 2 points b. 3 points c. 4 points d. 1 point

b. 3 points

After receiving change-of-shift report for several patients with neutropenia, which patient should the nurse assess first? a. 56-year-old with frequent explosive diarrhea b. 33-year-old with a fever of 100.8 F (38.2 C) c. 66-year-old who has white pharyngeal lesions d. 23-year old who is complaining of severe fatigue

b. 33-year-old with a fever of 100.8 F (38.2 C) Any fever in a neutropenic patient indicates infection and can quickly lead to sepsis and septic shock. Rapid assessment and (if prescribed) initiation of antibiotic therapy within 1 hour are needed. The other patients also need to be assessed but do not exhibit symptoms of potentially life-threatening problems.

What is the normal hematocrit level? a. 25-52% b. 37-52% c. 60-82% d. 46-72%

b. 37-52%

The nurse is caring for a patient with chronic obstructive pulmonary disorder (COPD) and pneumonia who has an order for arterial blood gases to be drawn. What is the minimum length of time the nurse should plan to hold pressure on the puncture site? a. 2 minutes b. 5 minutes c. 10 minutes d. 15 minutes

b. 5 minutes After obtaining blood for an arterial blood gas measurement, the nurse should hold pressure on the puncture site for 5 minutes by the clock to be sure that bleeding has stopped. An artery is an elastic vessel under much higher pressure than veins, and significant blood loss or hematoma formation could occur if the time is insufficient.

Which of the following is the most accurate measure of volume status? a. intake & output b. daily weight c. BUN, sodium & hematocrit levels d. Specific gravity of the urine

b. daily weight

Which patient has early clinical manifestations of hypoxemia? a. A 67-yr-old patient who has dyspnea while resting in the bed or in a reclining chair. b. A 72-yr-old patient who has four new premature ventricular contractions per minute. c. A 94-yr-old patient who has renal insufficiency, anemia, and decreased urine output. d. A 48-yr-old patient who is intoxicated and acutely disoriented to time and place.

b. A 72-yr-old patient who has four new premature ventricular contractions per minute. Early clinical manifestations of hypoxemia include dysrhythmias (e.g., premature ventricular contractions), unexplained decreased level of consciousness (e.g., disorientation), dyspnea on exertion, and unexplained decreased urine output.

A nurse is providing health teaching for a group of clients. Which of the following clients is at risk for developing peripheral arterial disease? A. A client who has hypothyroidism b. A client who has diabetes mellitus c. A client whose daily caloric intake consists of 25% fat d. A client who consumes two bottles of beer a day

b. A client who has diabetes mellitus Rationale: Diabetes mellitus places the client at risk for microvascular damage and progressive peripheral arterial disease.

Which statements explain the measurement of pulsus paradoxus with cardiac tamponade (select all that apply)? a. A difference of < 10 mm Hg occurs. b. A difference of > 10 mm Hg occurs. c. It is measured with an automatic sphygmomanometer. d. Rapidly inflate the cuff until you hear sounds throughout the respiratory cycle. e. Subtract the number when sounds are heard in the respiratory cycle from the number when the first Korotkoff sound during expiration is heard.

b. A difference of > 10 mm Hg occurs. e. Subtract the number when sounds are heard in the respiratory cycle from the number when the first Korotkoff sound during expiration is heard. Pulsus paradoxus is measured with a manually operated sphygmomanometer. The cuff is deflated slowly until the first Korotkoff sound during expiration is heard and the number is noted. The slow deflation of the cuff is continued until sounds are heard throughout the respiratory cycle and that number is subtracted from the first number. When the difference is > 10 mm Hg, cardiac tamponade may be present. The difference is normally < 10 mm Hg.

The nurse receives a change-of-shift report on the following patients with chronic obstructive pulmonary disease (COPD). Which patient should the nurse assess first? a. A patient with loud expiratory wheezes b. A patient with a respiratory rate of 38/minute c. A patient who has a cough productive of thick, green mucus d. A patient with jugular venous distention and peripheral edema

b. A patient with a respiratory rate of 38/minute A respiratory rate of 38/minute indicates severe respiratory distress, and the patient needs immediate assessment and intervention to prevent possible respiratory arrest. The other patients also need assessment as soon as possible, but they do not need to be assessed as urgently as the tachypneic patient.

A 73-yr-old man with dementia has a venous ulcer related to chronic venous insufficiency. The nurse should provide teaching on which type of diet for this patient and his caregiver? a Low-fat diet b High-protein diet c Calorie-restricted diet d High-carbohydrate diet

b. A patient with a venous ulcer should have a balanced diet with adequate protein, calories, and micronutrients; this type of diet is essential for healing. Nutrients most important for healing include protein, vitamins A and C, and zinc. Foods high in protein (e.g., meat, beans, cheese, tofu), vitamin A (green leafy vegetables), vitamin C (citrus fruits, tomatoes, cantaloupe), and zinc (meat, seafood) must be provided. Restricting fat or calories is not helpful for wound healing or in patients of normal weight. For overweight individuals with no active venous ulcer, a weight-loss diet should be considered.

After the nurse has received change-of-shift report, which patient should the nurse assess first? a. A patient with pneumonia who has crackles in the right lung base b. A patient with possible lung cancer who has just returned after bronchoscopy c. A patient with hemoptysis and a 16-mm induration with tuberculin skin testing d. A patient with chronic obstructive pulmonary disease (COPD) and pulmonary function testing(PFT) that indicates low forced vital capacity

b. A patient with possible lung cancer who has just returned after bronchoscopy Because the cough and gag are decreased after bronchoscopy, this patient should be assessed for airway patency. The other patients do not have clinical manifestations or procedures that require immediate assessment by the nurse.

When caring for a patient who is hospitalized with active tuberculosis (TB), the nurse observes a student nurse who is assigned to take care of a patient. Which action, if performed by the student nurse, would require an intervention by the nurse? a. The patient is offered a tissue from the box at the bedside. b. A surgical face mask is applied before visiting the patient. c. A snack is brought to the patient from the unit refrigerator. d. Hand washing is performed before entering the patients room.

b. A surgical face mask is applied before visiting the patient. A high-efficiency particulate-absorbing (HEPA) mask, rather than a standard surgical mask, should be used when entering the patients room because the HEPA mask can filter out 100% of small airborne particles. Hand washing before entering the patients room is appropriate. Because anorexia and weight loss are frequent problems in patients with TB, bringing food to the patient is appropriate. The student nurse should perform hand washing after handling a tissue that the patient has used, but no precautions are necessary when giving the patient an unused tissue.

After obtaining patient histories, which patient does the nurse identify as having the highest risk for CAD? a. A white man, age 54 years, who is a smoker and has a stressful lifestyle b. A white woman, age 75 years, with a BP of 172/100 mm Hg and who is physically inactive c. An Asian woman, age 45 years, with a cholesterol level of 240 mg/dL and a BP of 130/74 mm Hg d. An obese Hispanic man, age 65 years, with a cholesterol level of 195 mg/dL and a BP of 128/76 mm Hg

b. A white woman, age 75 years, with a BP of 172/100 mm Hg and who is physically inactive This white woman has 1 unmodifiable risk factor (age) and 2 major modifiable risk factors (hypertension and physical inactivity). Her gender risk is as high as a man's because she is 75 years of age. The white man has 1 unmodifiable risk factor (gender), 1 major modifiable risk factor (smoking), and 1 contributing modifiable risk factor (stressful lifestyle). The Asian woman has only 1 major modifiable risk factor (hyperlipidemia), and Asians in the United States have fewer myocardial infarctions (MIs) than do whites. The Hispanic man has an unmodifiable risk factor related to age and 1 major modifiable risk factor (obesity). Hispanics have slightly lower rates of CAD than non-Hispanic whites or blacks.

What causes the anemia of sickle cell disease? a. Intracellular hemolysis of sickled RBCs b. Accelerated breakdown of abnormal RBCs c. Autoimmune antibody destruction of RBCs d. Isoimmune antibody-antigen reactions with RBCs

b. Accelerated breakdown of abnormal RBCs Because RBCs become abnormal related to hypoxia or infection in sickle cell anemia, the spleen accelerates RBC breakdown as the sickling increases. Antibody reactions with RBCs may be seen in other types of hemolytic anemias but are not present in sickle cell anemia.

A patient with rheumatic fever has subcutaneous nodules, erythema marginatum, and polyarthritis. Based on these findings, which nursing diagnosis would be most appropriate? a. Pain related to permanent joint fixation b. Activity intolerance related to arthralgia c. Risk for infection related to open skin lesions d. Risk for impaired skin integrity related to pruritus

b. Activity intolerance related to arthralgia The patients joint pain will lead to difficulty with activity. The skin lesions seen in rheumatic fever are not open or pruritic. Although acute joint pain will be a problem for this patient, joint inflammation is a temporary clinical manifestation of rheumatic fever and is not associated with permanent joint changes.

Priority Decision: A patient diagnosed with class 3 TB 1 week ago is admitted to the hospital with symptoms of chest pain and coughing. What nursing action has the highest priority? a. Administering the patient's antitubercular drugs b. Admitting the patient to an airborne infection isolation room c. Preparing the patient's room with suction equipment and extra linens d. Placing the patient in an intensive care unit, where he can be closely monitored

b. Admitting the patient to an airborne infection isolation room A patient with class 3 TB has clinically active disease, and airborne infection isolation is required for active disease until the patient is noninfectious, indicated by negative sputum smears. Cardiac monitoring and observation will be done with the patient in isolation. The nurse will administer the antitubercular drugs after the patient is in isolation. There should be no need for suction or extra linens after the TB patient is receiving drug therapy.

A patient with thrombocytopenia with active bleeding is to receive 2 units of platelets. To administer the platelets, what should the nurse do? a. Check for ABO compatibility. b. Agitate the bag periodically during the transfusion. c. Take vital signs every 15 minutes during the procedure. d. Refrigerate the second unit until the first unit has transfused.

b. Agitate the bag periodically during the transfusion. Because platelets adhere to the plastic bags, the bag should be gently agitated throughout the transfusion. Platelets do not have A, B, or Rh antibodies, and ABO compatibility is not a consideration. Baseline vital signs should be taken before the transfusion is started, and the nurse should stay with the patient during the first 15 minutes. Platelets are stored at room temperature for 1 to 5 days.

What is a possible cause for percussion abnormal finding "hyperresonance"? a. Lung consolidation with fluid or exudate b. Air trapping c. Atelactasis d. Interstitial edema

b. Air trapping

Which medication would you give to a patient with asthma? Select all that apply a. Inhaled salmeterol b. Albuterol inhaler c. Nebulizer Ipratropium d. IV methamphetamines e. IV Methylprednisolone

b. Albuterol inhaler c. Nebulizer Ipratropium e. IV Methylprednisolone

To which patients should the nurse teach the Therapeutic Lifestyle Changes diet to reduce the risk of CAD? a. Patients who have had an MI b. All patients to reduce CAD risk c. Those with 2 or more risk factors for CAD d. Those with a cholesterol level > 200 mg/dL (5.2 mmol/L)

b. All patients to reduce CAD risk The Therapeutic Lifestyle Changes diet includes recommendations for all people, not just those with risk factors, to decrease the risk for CAD.

What causes the pulmonary vasoconstriction leading to the development of cor pulmonale in the patient with COPD? a. Increased viscosity of the blood b. Alveolar hypoxia and hypercapnia c. Long-term low-flow oxygen therapy d. Administration of high concentrations of oxygen

b. Alveolar hypoxia and hypercapnia Constriction of the pulmonary vessels, leading to pulmonary hypertension, is caused by alveolar hypoxia and the acidosis that results from hypercapnia. Polycythemia is a contributing factor in cor pulmonale because it increases the viscosity of blood and the pressure needed to circulate the blood but does not cause vasoconstriction. Long-term lowflow oxygen therapy dilates pulmonary vessels and is used to treat cor pulmonale. High oxygen administration is not related to cor pulmonale.

A client in the intensive care unit is having active bleeding and the health care provider (HCP) orders 4 units of PRBC's to be infused ASAP. The client's hemoglobin has dropped to 5 gm/dL. As the nurse begins the first infusion, the client begins audibly wheezing, breaks out in a rash/hives over the body, and has a blood pressure of 72/38. Which type of infusion reaction is the client experiencing? a. Febrile reaction. b. Anaphylactic reaction. c. Circulatory overload. d. Hemolytic reaction.

b. Anaphylactic reaction. This answer is correct because when a client has an anaphylactic reaction, the client will experience wheezing, hives, and hypotension. If the client is not treated immediately, the result could be shock and could be fatal.

A patient with a pH of 7.29 has metabolic acidosis. Which value is useful in determining whether the cause of the acidosis is an acid gain or a bicarbonate loss? a. PaCO2 b. Anion gap c. Serum Na + level d. Bicarbonate level

b. Anion gap Anion Gap = Na + − (HCO3 − + Cl − ). Calculate the anion gap by subtracting the serum bicarbonate and chloride levels from the serum sodium level. It should normally be 8 to 12 mmol/L. The anion gap is increased in metabolic acidosis associated with acid gain (e.g., diabetic ketoacidosis) but is normal in metabolic acidosis caused by bicarbonate loss (e.g., diarrhea).

A patient with chronic HF has atrial fibrillation and a left ventricular ejection fraction (LVEF) of 18%. To decrease the risk of complications from these conditions, what drug does the nurse anticipate giving? a. Diuretic b. Anticoagulant c. β-Adrenergic blocker d. Potassium supplement

b. Anticoagulant Thrombus formation occurs in the heart when the chambers do not contract normally and empty completely. Both atrial fibrillation and very low left ventricular output (LVEF < 20%) lead to thrombus formation, which is treated with anticoagulants to prevent the release of emboli into the circulation as well as antidysrhythmics or cardioversion to control atrial fibrillation.

Which drugs would the nurse expect to be prescribed for patients with a mechanical valve replacement? a. Oral nitrates b. Anticoagulants c. Atrial antidysrhythmics d. β-adrenergic blocking agents

b. Anticoagulants Patients with mechanical valves have an increased risk for thromboembolism and require long-term anticoagulation to prevent systemic or pulmonary embolization. Nitrates are contraindicated for the patient with aortic stenosis because an adequate preload is necessary to open the stiffened aortic valve. Antidysrhythmics are used only if dysrhythmias occur and β-adrenergic blocking drugs may be used to control the heart rate if needed.

The patient is admitted with angina, syncope, and dyspnea on exertion. In the assessment, the nurse notes a systolic murmur with a prominent S4 . What will the nurse suspect is occurring with this patient? a. Mitral valve stenosis b. Aortic valve stenosis c. Acute mitral valve regurgitation d. Chronic mitral valve regurgitation

b. Aortic valve stenosis Aortic valve stenosis is identified with the triad of angina, syncope, and dyspnea on exertion, as well as the systolic murmur and prominent S4 heart sound. Mitral valve stenosis manifests as exertional dyspnea, hemoptysis, fatigue, atrial fibrillation, and a diastolic murmur. Acute mitral valve regurgitation has a new systolic murmur with pulmonary edema and cardiogenic shock rapidly developing. Chronic mitral valve regurgitation is identified with weakness, fatigue, exertional dyspnea, palpitations, an S3 gallop, and holosystolic murmur

Which anemia is manifested with pancytopenia? a. Thalassemia b. Aplastic anemia c. Megaloblastic anemia d. Anemia of chronic disease

b. Aplastic anemia Aplastic anemia has a decrease of all blood cell types and hypocellular bone marrow. Thalassemia is characterized by inadequate production of normal Hgb and decreased erythrocyte production. Megaloblastic anemias (cobalamin deficiency and folic acid deficiency anemias) are caused by impaired DNA synthesis, which results in the presence of large red blood cells (RBCs). Anemia of chronic disease occurs with chronic inflammation, autoimmune and infectious disorders, heart failure, malignancies, or bleeding episodes. It manifests with underproduction of RBCs and shortened RBC survival.

Which actions could the nurse delegate to unlicensed assistive personnel (UAP) who are providing care for a patient who is at risk for venous thromboembolism? a. Monitor for any bleeding after anticoagulation therapy is started. b. Apply sequential compression device whenever the patient is in bed. c. Ask the patient about use of herbal medicines or dietary supplements. d. Instruct the patient to call immediately if any shortness of breath occurs.

b. Apply sequential compression device whenever the patient is in bed. UAP training includes the use of equipment that requires minimal nursing judgment, such as sequential compression devices. Patient assessment and teaching require more education and critical thinking and should be done by the registered nurse (RN).

When planning care for a patient hospitalized with a streptococcal infective endocarditis (IE), which intervention is a priority for the nurse to include? a. Monitor labs for streptococcal antibodies. b. Arrange for placement of a long-term IV catheter. c. Teach the importance of completing all oral antibiotics. d. Encourage the patient to begin regular aerobic exercise.

b. Arrange for placement of a long-term IV catheter. Treatment for IE involves 4 to 6 weeks of IV antibiotic therapy in order to eradicate the bacteria, which will require a long-term IV catheter such as a peripherally inserted central catheter (PICC) line. Rest periods and limiting physical activity to a moderate level are recommended during the treatment for IE. Oral antibiotics are not effective in eradicating the infective bacteria that cause IE. Blood cultures, rather than antibody levels, are used to monitor the effectiveness of antibiotic therapy.

Which conditions characterize critical limb ischemia (select all that apply)? a. Cold feet b. Arterial leg ulcers c. Venous leg ulcers d. Gangrene of the leg e. No palpable peripheral pulses f. Rest pain lasting more than 2 weeks

b. Arterial leg ulcers d. Gangrene of the leg f. Rest pain lasting more than 2 weeks Arterial leg ulcers and/or gangrene of the leg caused by PAD and chronic ischemic rest pain lasting more than 2 weeks characterize critical limb ischemia. Optimal therapy is revascularization via bypass surgery

When titrating IV nitroglycerin (Tridil) for a patient with a myocardial infarction (MI), which action will the nurse take to evaluate the effectiveness of the medication? a. Monitor heart rate. b. Ask about chest pain. c. Check blood pressure. d. Observe for dysrhythmias.

b. Ask about chest pain. The goal of IV nitroglycerin administration in MI is relief of chest pain by improving the balance between myocardial oxygen supply and demand. The nurse also will monitor heart rate and blood pressure (BP) and observe for dysrhythmias, but these parameters will not indicate whether the medication is effective.

A 19-year-old student comes to the student health center at the end of the semester complaining that, My heart is skipping beats. An electrocardiogram (ECG) shows occasional premature ventricular contractions (PVCs). What action should the nurse take next? a. Start supplemental O2 at 2 to 3 L/min via nasal cannula. b. Ask the patient about current stress level and caffeine use. c. Ask the patient about any history of coronary artery disease. d. Have the patient taken to the hospital emergency department (ED).

b. Ask the patient about current stress level and caffeine use. In a patient with a normal heart, occasional PVCs are a benign finding. The timing of the PVCs suggests stress or caffeine as possible etiologic factors. It is unlikely that the patient has coronary artery disease, and this should not be the first question the nurse asks. The patient is hemodynamically stable, so there is no indication that the patient needs to be seen in the ED or that oxygen needs to be administered.

A patient with hypertension who has just started taking atenolol (Tenormin) returns to the health clinic after 2 weeks for a follow-up visit. The blood pressure (BP) is unchanged from the previous visit. Which action should the nurse take first? a. Tell the patient why a change in drug dosage is needed. b. Ask the patient if the medication is being taken as prescribed. c. Inform the patient that multiple drugs are often needed to treat hypertension. d. Question the patient regarding any lifestyle changes made to help control BP.

b. Ask the patient if the medication is being taken as prescribed. Because nonadherence with antihypertensive therapy is common, the nurse's initial action should be to determine whether the patient is taking the atenolol as prescribed. The other actions also may be implemented, but these would be done after assessing patient adherence with the prescribed therapy.

After 2 months of tuberculosis (TB) treatment with isoniazid (INH), rifampin (Rifadin), pyrazinamide (PZA), and ethambutol, a patient continues to have positive sputum smears for acid-fast bacilli (AFB). Which action should the nurse take next? a. Teach about treatment for drug-resistant TB treatment. b. Ask the patient whether medications have been taken as directed. c. Schedule the patient for directly observed therapy three times weekly. d. Discuss with the health care provider the need for the patient to use an injectable antibiotic.

b. Ask the patient whether medications have been taken as directed. The first action should be to determine whether the patient has been compliant with drug therapy because negative sputum smears would be expected if the TB bacillus is susceptible to the medications and if the medications have been taken correctly. Assessment is the first step in the nursing process. Depending on whether the patient has been compliant or not, different medications or directly observed therapy may be indicated. The other options are interventions based on assumptions until an assessment has been completed.

A pt admitted with an MI 3 days ago is reporting chest pain with deep breaths. Which intervention should be done FIRST? a. Obtain a 12-lead ECG and call the HCP. b. Give the patient something for pain. c. Place a nonrebreather mask on the patient. b. Assess vital signs and heart sounds for a friction rub.

b. Assess vital signs and heart sounds for a friction rub.

What causes most organ damage in hypertension? a. Increased fluid pressure exerted against organ tissue b. Atherosclerotic changes in vessels that supply the organs c. Erosion and thinning of blood vessels in organs from constant pressure d. Increased hydrostatic pressure causing leakage of plasma into organ interstitial spaces

b. Atherosclerotic changes in vessels that supply the organs Elevated BP causes endothelial damage, which causes the inner lining of arterioles to become thickened and stiffened and affects coronary circulation, cerebral circulation, peripheral vessels, and renal and retinal blood vessels. The narrowed vessels lead to ischemia and ultimately to damage of these organs.

A nurse in an urgent care center is assessing a client who reports a sudden onset of irregular palpitations, fatigue, and dizziness. The nurse finds a rapid and irregular heart rate with significant pulse deficit. Which of the following dysrhythmias should the nurse expect to find on the ECG? a. Asystole b. Atrial fibrillation c. Ventricular fibrillation d. Ventricular tachycardia

b. Atrial fibrillation

When admitting a patient with a nonST-segment-elevation myocardial infarction (NSTEMI) to the intensive care unit, which action should the nurse perform first? a. Obtain the blood pressure. b. Attach the cardiac monitor. c. Assess the peripheral pulses. d. Auscultate the breath sounds.

b. Attach the cardiac monitor. Because dysrhythmias are the most common complication of myocardial infarction (MI), the first action should be to place the patient on a cardiac monitor. The other actions also are important and should be accomplished as quickly as possible.

Following a laryngectomy a patient coughs violently during suctioning and dislodges the tracheostomy tube. Which action should the nurse take first? a. Cover stoma with sterile gauze and ventilate through stoma. b. Attempt to reinsert the tracheostomy tube with the obturator in place. c. Assess the patients oxygen saturation and notify the health care provider. d. Ventilate the patient with a manual bag and face mask until the health care provider arrives.

b. Attempt to reinsert the tracheostomy tube with the obturator in place. The first action should be to attempt to reinsert the tracheostomy tube to maintain the patients airway. Assessing the patients oxygenation is an important action, but it is not the most appropriate first action in this situation. Covering the stoma with a dressing and manually ventilating the patient may be an appropriate action if the nurse is unable to reinsert the tracheostomy tube. Ventilating with a facemask is not appropriate for a patient with a total laryngectomy because there is a complete separation between the upper airway and the trachea.

Which intervention will be included in the nursing care plan for a patient with immune thrombocytopenic purpura (ITP)? a. Assign the patient to a private room. b. Avoid intramuscular (IM) injections. c. Use rinses rather than a soft toothbrush for oral care. d. Restrict activity to passive and active range of motion.

b. Avoid intramuscular (IM) injections. IM or subcutaneous injections should be avoided because of the risk for bleeding. A soft toothbrush can be used for oral care. There is no need to restrict activity or place the patient in a private room.

A patient with a sinus node dysfunction has a permanent pacemaker inserted. Before discharge, what should the nurse include when teaching the patient? a. Avoid cooking with microwave ovens. b. Avoid standing near antitheft devices in doorways. c. Use mild analgesics to control the chest spasms caused by the pacing current. d. Start lifting the arm above the shoulder right away to prevent a "frozen shoulder."

b. Avoid standing near antitheft devices in doorways. The patient should avoid standing near antitheft devices in doorways of department stores and libraries but walking through them at normal pace is fine. High-output electrical generators or large magnets, such as those used in MRI, can reprogram pacemakers and should be avoided unless the pacemaker is MRI safe. Microwave ovens pose no problems to pacemaker function, but the affected arm should not be raised above the shoulder until approved by cardiologist. The pacing current of an implanted pacemaker is not felt by the patient, but an external transcutaneous pacemaker may cause uncomfortable chest muscle contractions.

Which dietary modification helps meet the nutritional needs of patients with COPD? a. Eating a high-carbohydrate, low-fat diet b. Avoiding foods that require a lot of chewing c. Preparing most foods of the diet to be eaten hot d. Drinking fluids with meals to promote digestion

b. Avoiding foods that require a lot of chewing Eating is an effort for patients with COPD, and often these patients do not eat because of fatigue, dyspnea, altered taste, and decreased appetite. Foods that require much chewing cause more exhaustion and should be avoided. A low-carbohydrate diet is indicated if the patient has hypercapnia because carbohydrates are metabolized into carbon dioxide. Cold foods seem to give less of a sense of fullness than hot foods, and fluids should be avoided at meals to prevent a full stomach.

A patient has ST segment changes that support an acute inferior wall myocardial infarction. Which lead would be best for monitoring the patient? a. I b. II c. V2 d. V6

b. II Leads II, III, and AVF reflect the inferior area of the heart and the ST segment changes. Lead II will best capture any electrocardiographic (ECG) changes that indicate further damage to the myocardium. The other leads do not reflect the inferior part of the myocardial wall and will not provide data about further ischemic changes in that area.

A 92-year-old female patient is being admitted to the emergency department with severe shortness of breath. Being aware of the patient's condition, what approach should the nurse use to assess the patient's lungs (select all that apply)? a. Apex to base b. Base to apex c. Lateral sequence d. Anterior then posterior e. Posterior then anterior

b. Base to apex e. Posterior then anterior This patient is older and short of breath. To obtain the most information, auscultate the posterior to avoid breast tissue and start at the base because of her respiratory difficulty and the chance that she will tire easily. Important sounds may be missed if the other strategies are used first

When auscultating the chest of an older patient in mild respiratory distress, it is best to a. Begin listening at the apices. b. Begin listening at the lung bases. c. Begin listening on the anterior chest. d. Ask the patient to breathe through the nose with the mouth closed.

b. Begin listening at the lung bases.

Which assessment finding for a patient who is receiving IV furosemide (Lasix) to treat stage 2 hypertension is most important to report to the health care provider? a. Blood glucose level of 175 mg/dL b. Blood potassium level of 3.0 mEq/L c. Most recent blood pressure (BP) reading of 168/94 mm Hg d. Orthostatic systolic BP decrease of 12 mm Hg

b. Blood potassium level of 3.0 mEq/L Hypokalemia is a frequent adverse effect of the loop diuretics and can cause life-threatening dysrhythmias. The health care provider should be notified of the potassium level immediately and administration of potassium supplements initiated. The elevated blood glucose and BP also indicate a need for collaborative interventions but will not require action as urgently as the hypokalemia. An orthostatic drop of 12 mm Hg is common and will require intervention only if the patient is symptomatic.

A nurse is caring for a client in the first 8 hr following coronary artery bypass graft (CABG) surgery. Which of the following client findings should the nurse report to the provider? A. Mediastinal drainage 100 ml/hr b. Blood pressure 160/80 mm Hg C. Temperature 37.1° C (98.8° F) D. Potassium 4.0 meq/L

b. Blood pressure 160/80 mm Hg Rationale: The nurse should report an elevated blood pressure following a CABG because increased vascular pressure can cause bleeding at the incision sites.

A patient with pancytopenia of unknown origin is scheduled for the following diagnostic tests. The nurse will provide a consent form to sign for which test? a. ABO blood typing b. Bone marrow biopsy c. Abdominal ultrasound d. Complete blood count (CBC)

b. Bone marrow biopsy A bone marrow biopsy is a minor surgical procedure that requires the patient or guardian to sign a surgical consent form. The other procedures do not require a signed consent by the patient or guardian.

A patient with acute shortness of breath is admitted to the hospital. Which action should the nurse take during the initial assessment of the patient? a. Ask the patient to lie down to complete a full physical assessment. b. Briefly ask specific questions about this episode of respiratory distress. c. Complete the admission database to check for allergies before treatment. d. Delay the physical assessment to first complete pulmonary function tests.

b. Briefly ask specific questions about this episode of respiratory distress. When a patient has severe respiratory distress, only information pertinent to the current episode is obtained, and a more thorough assessment is deferred until later. Obtaining a comprehensive health history or full physical examination is unnecessary until the acute distress has resolved. Brief questioning and a focused physical assessment should be done rapidly to help determine the cause of the distress and suggest treatment. Checking for allergies is important, but it is not appropriate to complete the entire admission database at this time. The initial respiratory assessment must be completed before any diagnostic tests or interventions can be ordered.

While providing discharge instructions to the patient who has had an implantable cardioverter-defibrillator (ICD) inserted, the nurse teaches the patient that if the ICD fires, it is important that he or she should do what? a. Lie down. b. Call the cardiologist. c. Push the reset button on the pulse generator. d. Immediately take his or her antidysrhythmic medication.

b. Call the cardiologist. If the implanted cardioverter-defibrillator delivers a shock, the patient has experienced a lethal dysrhythmia and needs to notify the cardiologist. The patient will want to lie down to allow recovery from the dysrhythmia. In the event that the patient loses consciousness or there is repetitive firing, a call should be placed to the emergency response system (ERS) by anyone who finds the patient.

A nurse is assessing a client who has a serum calcium level of 8.1 mg/dL. Which of the following findings is the priority for the nurse to assess? a. Deep-tendon reflexes b. Cardiac rhythm c. Peripheral sensation d. Bowel sounds

b. Cardiac rhythm When using the airway, breathing, circulation approach to client care, the nurse should determine that assessing the cardiac rhythm is the priority. Calcium levels below the expected reference range can cause ECG changes, bradycardia, or tachycardia.

Which finding is associated with a blue color around the lips and conjunctiva? a. Finger clubbing b. Central cyanosis c. Peripheral cyanosis d. Delayed capillary filling time

b. Central cyanosis Central cyanosis is evident with a blue tinge in the lips, conjunctiva, or tongue. Finger clubbing results from endocarditis, congenital defects, or prolonged O2 deficiency. Peripheral cyanosis is evident with blue-tinged extremities or in the nose and ears. Decreased capillary refill may be seen in reduced capillary perfusion or anemia

Priority Decision: A patient with stage 2 hypertension who is taking chlorothiazide (Diuril) and lisinopril (Zestril) has prazosin (Minipress) added to the medication regimen. What is most important for the nurse to teach the patient to do? a. Weigh every morning to monitor for fluid retention. b. Change position slowly and avoid prolonged standing. c. Use sugarless gum or candy to help relieve dry mouth. d. Take the pulse daily to note any slowing of the heart rate.

b. Change position slowly and avoid prolonged standing. Chlorothiazide is a thiazide diuretic that causes orthostatic hypotension. Prazosin is an α-adrenergic blocker that causes dilation of arterioles and veins and causes orthostatic hypotension. The patient may feel dizzy, weak, and faint when assuming an upright position after sitting or lying down and should be taught to change positions slowly, avoid standing for long periods, do leg exercises to increase venous return, and lie or sit down when dizziness occurs. Direct-acting vasodilators often cause fluid retention; dry mouth may occur with diuretic use, and centrally acting α- and β-blockers may cause bradycardia.

Patient with a feeding tube is hospitalized and at risk for aspiration pneumonia. What nursing intervention is indicated to prevent pneumonia? a. Position to side, protect airway b. Check placement of the tube before feeding and residual feeding; keep head of bed up after feedings or continuously with continuous feedings c. Check gag reflex before feeding or offering fluids d. Cut food in small bites, encourage thorough chewing, and provide soft foods that are easier to swallow than liquids

b. Check placement of the tube before feeding and residual feeding; keep head of bed up after feedings or continuously with continuous feedings

Which action will the admitting nurse include in the care plan for a 30-year old woman who is neutropenic? a. Avoid any injections. b. Check temperature every 4 hours. c. Omit fruits or vegetables from the diet. d. Place a No Visitors sign on the door.

b. Check temperature every 4 hours. The earliest sign of infection in a neutropenic patient is an elevation in temperature. Although unpeeled fresh fruits and vegetables should be avoided, fruits and vegetables that are peeled or cooked are acceptable. Injections may be required for administration of medications such as filgrastim (Neupogen). The number of visitors may be limited and visitors with communicable diseases should be avoided, but a no visitors policy is not needed.

A patient comes to the clinic complaining of frequent, watery stools for the last 2 days. Which action should the nurse take first? a. Obtain the baseline weight. b. Check the patients' blood pressure. c. Draw blood for serum electrolyte levels. d. Ask about any extremity numbness or tingling.

b. Check the patients' blood pressure. Because the patient's history suggests that fluid volume deficit may be a problem, assessment for adequate circulation is the highest priority. The other actions are also appropriate, but are not as essential as determining the patient's perfusion status.

Collaboration: In planning care for a patient who has just returned to the unit following a PCI, the nurse may delegate which activity to unlicensed assistive personnel (UAP)? a. Monitor the IV fluids and measure urine output. b. Check vital signs and report changes in HR, BP, or pulse oximetry. c. Explain to the patient the need for frequent vital signs and pulse checks. d. Assess circulation to the extremity used by checking pulses, skin temperature, and color.

b. Check vital signs and report changes in HR, BP, or pulse oximetry. The UAP can check vital signs and report results to the RN. The other actions include assessment, teaching, and monitoring of IV fluids, which are all responsibilities of the RN.

In the patient with a dysrhythmia, which assessment indicates decreased cardiac output (CO)? a. Hypertension and bradycardia b. Chest pain and decreased mentation c. Abdominal distention and hepatomegaly d. Bounding pulses and a ventricular heave

b. Chest pain and decreased mentation Symptoms of decreased cardiac output (CO) related to dysrhythmias include a sudden drop in BP and symptoms of hypoxemia, such as decreased mentation, chest pain, and dyspnea. Peripheral pulses are weak and the HR may be increased or decreased, depending on the type of dysrhythmia present.

In the patient with supraventricular tachycardia, which assessment indicates decreased cardiac output? a. Hypertension and dyspnea b. Chest pain and palpitations c. Abdominal distention and tachypnea d. Bounding pulses and a systolic murmur

b. Chest pain and palpitations Correct answer: b Rationale: Manifestations of decreased cardiac output in the patient with supraventricular tachycardia include hypotension, angina, palpitations, and dyspnea.

The patient is diagnosed with a superficial vein thrombosis (SVT). Which characteristic should the nurse know about SVT? a. Embolization to lungs may result in death. b. Clot may extend to deeper veins if untreated. c. Vein is tender to pressure and there is edema. d. Typically found in the iliac, inferior, or superior vena cava

b. Clot may extend to deeper veins if untreated. If left untreated, a superficial vein thrombosis (SVT) may extend to deeper veins and VTE may occur. VTE may embolize to the lungs and have tenderness to pressure and edema. SVTs usually occur in superficial leg veins and have tenderness, itchiness, redness, warmth, pain, inflammation, and induration along the course of the superficial vein.

Priority Decision: A patient who is postoperative following repair of an AAA has been receiving IV fluids at 125 mL/hr continuously for the last 12 hours. Urine output for the last 4 hours has been 60 mL, 42 mL, 28 mL, and 20 mL, respectively. What is the priority action that the nurse should take? a. Monitor for a couple more hours. b. Contact the HCP and report the decrease in urine output. c. Send blood for electrolytes, blood urea nitrogen (BUN), and creatinine. d. Decrease the rate of infusion to prevent blood leakage at the suture line.

b. Contact the HCP and report the decrease in urine output. The decreasing urine output is evidence that either the patient needs volume replacement or there is reduced renal blood flow. The HCP will want to be notified as soon as possible of this change in condition and will request results of daily blood urea nitrogen (BUN) and serum creatinine levels. The other options are incorrect.

Which characteristics are associated with LDLs (select all that apply)? a. Increases with exercise b. Contains the most cholesterol c. Has an affinity for arterial walls d. Carries lipids away from arteries to liver e. High levels correlate most closely with CAD f. The higher the level, the lower the risk for CAD

b. Contains the most cholesterol c. Has an affinity for arterial walls e. High levels correlate most closely with CAD LDLs contain more cholesterol than the other lipoproteins, have an attraction for arterial walls, and correlate most closely with increased incidence of atherosclerosis and CAD. HDLs increase with exercise and carry lipids away from arteries to the liver for metabolism. A high HDL level is associated with a lower risk of CAD.

A patient with a small AAA is not a good surgical candidate. What should the nurse teach the patient is the best way to prevent expansion of the lesion? a. Avoid strenuous physical exertion. b. Control hypertension with prescribed therapy. c. Comply with prescribed anticoagulant therapy. d. Maintain a low-calcium diet to prevent calcification of the vessel.

b. Control hypertension with prescribed therapy. Increased systolic BP (SBP) continually puts pressure on the diseased area of the artery, promoting its expansion. Small aneurysms can be treated by decreasing BP, modifying atherosclerosis risk factors, and monitoring the size of the aneurysm. Anticoagulants are used during surgical treatment of aneurysms, but physical activity is not known to increase their size. Calcium intake is not related to calcification in arteries.

A patient has a platelet count of 50,000/µL and is diagnosed with ITP. What does the nurse anticipate that initial treatment will include? a. Splenectomy b. Corticosteroids c. Administration of platelets d. Immunosuppressive therapy

b. Corticosteroids Corticosteroids are used in initial treatment of ITP because they suppress the phagocytic response of splenic macrophages, decreasing platelet destruction. They also depress autoimmune antibody formation and reduce capillary leakage. All of the other therapies may be used but only in patients who are unresponsive to corticosteroid therapy and severely reduced platelet counts.

The patient is calling the clinic with a cough. What assessment should be made first before the nurse advises the patient? a. Frequency, family history, hematemesis b. Cough sound, sputum production, pattern c. Weight loss, activity tolerance, orthopnea d. Smoking status, medications, residence location

b. Cough sound, sputum production, pattern The sound of the cough, sputum production and description, and the pattern of the cough's occurrence (including acute or chronic) and what its occurrence is related to are the first assessments to be made to determine the severity. Frequency of the cough will not provide a lot of information. Family history can help to determine a genetic cause of the cough. Hematemesis is vomiting blood and not as important as hemoptysis. Smoking is an important risk factor for chronic obstructive pulmonary disease, and lung cancer and may cause a cough. Medications may or may not contribute to a cough as does residence location. Weight loss, activity intolerance, and orthopnea may be related to respiratory or cardiac problems but are not as important when dealing with a cough.

A patient is admitted for hypovolemia associated with multiple draining wounds. Which assessment would be the most accurate way for the nurse to evaluate fluid balance? a. Skin turgor b. Daily weight c. Presence of edema d. Hourly urine output

b. Daily weight Daily weight is the most easily obtained and accurate means of assessing volume status. Skin turgor varies considerably with age. Considerable excess fluid volume may be present before fluid moves into the interstitial space and causes edema. Although very important, hourly urine outputs do not take account of fluid intake or of fluid loss through insensible loss, sweating, or loss from the gastrointestinal tract or wounds.

CPR is in progress when the pt arrived in the ER. Three doses of Epi were given, which intervention should be FIRST? a. Give another dose of Epi since it has been 5 minutes since the last dose. b. b. Debrillate with 200 joules. c. Start high-quality chest compressions. d. Perform a sternal thump.

b. Debrillate with 200 joules.

Pulmonary rehabilitation (PR) is designed to reduce symptoms and improve the patient's quality of life. Along with improving exercise capacity, what are anticipated results of PR (select all that apply)? a. Decreased FEV1 b. Decreased depression c. Increased oxygen need d. Decreased fear of exercise e. Decreased hospitalizations

b. Decreased depression d. Decreased fear of exercise e. Decreased hospitalizations Decreasing depression, fear of exercise, and hospitalizations along with improving exercise capacity are benefits of pulmonary rehabilitation (PR). Decreased FEV1 and increased oxygen need are not beneficial.

When assessing a newly admitted patient, the nurse notes a murmur along the left sternal border. To document more information about the murmur, which action will the nurse take next? a. Find the point of maximal impulse. b. Determine the timing of the murmur. c. Compare the apical and radial pulse rates. d. Palpate the quality of the peripheral pulses.

b. Determine the timing of the murmur. Murmurs are caused by turbulent blood flow, such as occurs when blood flows through a damaged valve. Relevant information includes the position in which the murmur is heard best (e.g., sitting and leaning forward), the timing of the murmur in relation to the cardiac cycle (e.g., systole, diastole), and where on the thorax the murmur is heard best. The other information is also important in the cardiac assessment but will not provide information that is relevant to the murmur

To improve the physical activity level for a mildly obese 71-year-old patient, which action should the nurse plan to take? a. Stress that weight loss is a major benefit of increased exercise. b. Determine what kind of physical activities the patient usually enjoys. c. Tell the patient that older adults should exercise for no more than 20 minutes at a time. d. Teach the patient to include a short warm-up period at the beginning of physical activity.

b. Determine what kind of physical activities the patient usually enjoys. Because patients are more likely to continue physical activities that they already enjoy, the nurse will plan to ask the patient about preferred activities. The goal for older adults is 30 minutes of moderate activity on most days. Older adults should plan for a longer warm-up period. Benefits of exercises, such as improved activity tolerance, should be emphasized rather than aiming for significant weight loss in older mildly obese adults.

When counseling the patient about sexual activity following an MI, what should the nurse do? a. Wait for the patient to ask about resuming sexual activity. b. Discuss sexual activity while teaching about other physical activity. c. Have the patient ask the health care provider when sexual activity can be resumed. d. Inform the patient that impotence is a common long-term complication following MI.

b. Discuss sexual activity while teaching about other physical activity. Resumption of sexual activity is often difficult for patients to approach. It is reported that most cardiac patients do not resume sexual activity after MI. The nurse can give the patient permission to discuss concerns about sexual activity by introducing it as a physical activity when other physical activities are discussed. HCPs may have preferences about the timing of resumption of sexual activity. The nurse should discuss this with the HCP and the patient but addressing the patient's concerns is a nursing responsibility. Patients should be taught that impotence after MI is common but that it usually disappears after several attempts

A patients complete blood count (CBC) shows a hemoglobin of 19 g/dL and a hematocrit of 54%. Which question should the nurse ask to determine possible causes of this finding? a. Have you had a recent weight loss? b. Do you have any history of lung disease? c. Have you noticed any dark or bloody stools? d. What is your dietary intake of meats and protein?

b. Do you have any history of lung disease? The hemoglobin and hematocrit results indicate polycythemia, which can be associated with chronic obstructive pulmonary disease (COPD). The other questions would be appropriate for patients who are anemic.

Risk Factor for or Response to Respiratory Problem: Decreased fluid intake, anorexia and rapid weight loss, obesity. Which Functional Health Pattern does it fall under? a. Health perception- health management b. Nutritional-metabolic c. Elimination d. Activity-exercise

b. Nutritional-metabolic

A young adult female patient with cystic fibrosis (CF) tells the nurse that she is not sure about getting married and having children some day. Which initial response by the nurse is best? a. Are you aware of the normal lifespan for patients with CF? b. Do you need any information to help you with that decision? c. Many women with CF do not have difficulty conceiving children. d. You will need to have genetic counseling before making a decision.

b. Do you need any information to help you with that decision? The nurses initial response should be to assess the patients knowledge level and need for information. Although the lifespan for patients with CF is likely to be shorter than normal, it would not be appropriate for the nurse to address this as the initial response to the patients comments. The other responses have accurate information, but the nurse should first assess the patients understanding about the issues surrounding pregnancy.

An older adult patient who is malnourished presents to the emergency department with a serum protein level of 5.2 g/dL. The nurse would expect which clinical manifestation? a. Pallor b. Edema c. Confusion d. Restlessness

b. Edema The normal range for total protein is 6.4 to 8.3 g/dL. Low serum protein levels cause a decrease in plasma oncotic pressure and allow fluid to remain in interstitial tissues, causing edema. Confusion, restlessness, and pallor are not associated with low serum protein levels.

Priority Decision: A patient with symptomatic mitral valve prolapse has atrial and ventricular dysrhythmias. In addition to monitoring for decreased cardiac output related to the dysrhythmias, what is an important nursing intervention related to the dysrhythmias? a. Monitor breathing pattern related to hypervolemia. b. Encourage calling for assistance when getting out of bed. c. Give sleeping pills to decrease paroxysmal nocturnal dyspnea. d. Teach the patient exercises to prevent recurrence of dysrhythmias.

b. Encourage calling for assistance when getting out of bed. Dysrhythmias often cause palpitations, lightheadedness, and dizziness and the patient should be carefully attended to prevent falls. Hypervolemia and paroxysmal nocturnal dyspnea (PND) would be apparent in the patient with heart failure. Exercises will not prevent dysrhythmias.

What accurately describes the pathophysiology of CAD? a. Partial or total occlusion of the coronary artery occurs during the stage of raised fibrous plaque. b. Endothelial changes may be caused by chemical irritants, such as hyperlipidemia or by tobacco use. c. Collateral circulation in the coronary circulation is more likely to be present in the young patient with CAD. d. The leading theory of atherogenesis proposes that infection and fatty dietary intake are the basic underlying causes of atherosclerosis.

b. Endothelial changes may be caused by chemical irritants, such as hyperlipidemia or by tobacco use. The etiology of CAD includes atherosclerosis as the major cause. The pathophysiology of atherosclerosis development and resulting atheromas is related to endothelial injury and inflammation, which can be the result of tobacco use, hyperlipidemia, hypertension, toxins, diabetes, high homocysteine levels, and infection causing a local inflammatory response in the inner lining of the vessel walls. Partial or total occlusion occurs in the complicated lesion stage. Extra collateral circulation occurs in the presence of chronic ischemia. Therefore it is more likely to occur in an older patient.

A nurse is caring for a patient with a diagnosis of deep venous thrombosis (DVT). The patient has an order to receive 30 mg enoxaparin (Lovenox). Which injection site should the nurse use to administer this medication safely? a Buttock, upper outer quadrant b Abdomen, anterior-lateral aspect c Back of the arm, 2 inches away from a mole d Anterolateral thigh, with no scar tissue nearby

b. Enoxaparin (Lovenox) is a low-molecular-weight (LMW) heparin that is given as a deep subcutaneous injection in the right and left anterolateral abdomen. All subcutaneous injections should be given away from scars, lesions, or moles

Which assessment finding would alert the nurse that a postoperative patient is not receiving the beneficial effects of enoxaparin (Lovenox)? a Crackles bilaterally in the lung bases b Pain and swelling in a lower extremity c Absence of arterial pulse in a lower extremity d Abdominal pain with decreased bowel sounds

b. Enoxaparin is a low-molecular-weight heparin used to prevent the development of deep vein thromboses (DVTs) in the postoperative period. Pain and swelling in a lower extremity can indicate development of DVT and therefore may signal ineffective medication therapy.

What covers the larynx during swallowing? a. Trachea b. Epiglottis c. Turbinates d. Parietal pleura

b. Epiglottis The epiglottis is a small flap closing over the larynx during swallowing. The trachea connects the larynx and the bronchi. The turbinates in the nose warm and moisturize inhaled air. The parietal pleura is a membrane that lines the chest cavity.

While obtaining subjective assessment data from a patient with hypertension, the nurse recognizes that a modifiable risk factor for the development of hypertension is a. A low-calcium diet. b. Excess alcohol intake. c. A family history of hypertension. d. Consumption of a high-protein diet.

b. Excess alcohol intake.

A patient with acute pericarditis has markedly distended jugular veins, decreased BP, tachycardia, tachypnea, and muffled heart sounds. What does the nurse recognize as the cause of these findings? a. The pericardial space is obliterated with scar tissue and thickened pericardium. b. Excess pericardial fluid compresses the heart and prevents adequate diastolic filling. c. The parietal and visceral pericardial membranes adhere to each other, preventing normal myocardial contraction d. Fibrin accumulation on the visceral pericardium infiltrates the myocardium, creating myocardial dysfunction

b. Excess pericardial fluid compresses the heart and prevents adequate diastolic filling. The patient is experiencing cardiac tamponade that consists of excess fluid in the pericardial sac, which compresses the heart and the adjoining structures, preventing normal filling and cardiac output. A scarred and thickened pericardium, adherent pericardial membranes, and fibrin accumulation occur in chronic constrictive pericarditis

A patient admitted to the hospital for evaluation of chest pain has no abnormal serum cardiac biomarkers 4 hours after the onset of pain. What noninvasive diagnostic test can be used to differentiate angina from other types of chest pain? a. 12-lead ECG b. Exercise stress test c. Coronary angiogram d. Transesophageal echocardiogram

b. Exercise stress test An exercise stress test will reveal ECG changes that indicate impaired coronary circulation when the oxygen demand of the heart is increased. A single ECG is not conclusive for CAD, and negative findings do not rule out CAD. Coronary angiography can detect narrowing of coronary arteries but is an invasive procedure. Echocardiograms of various types may identify abnormalities of myocardial wall motion under stress but are indirect measures of CAD.

Which serum lipid elevation, along with high LDL, is strongly associated with CAD? a. Apolipoproteins b. Fasting triglycerides c. Total serum cholesterol d. High-density lipoprotein (HDL)

b. Fasting triglycerides High fasting triglyceride levels are associated with cardiovascular disease and diabetes. Apolipoproteins are found in varying amounts on the HDLs and activate enzyme or receptor sites that promote removal of fat from plasma, which is protective. The apolipoprotein A and apolipoprotein B ratio must be done to predict CAD. High total serum cholesterol must be calculated with HDL for a ratio over time to determine an increased risk of CAD. High HDLs are associated with a lower risk of CAD.

When obtaining a health history from a patient suspected of having early TB, the nurse should ask the patient about what manifestations? a. Chest pain, hemoptysis, and weight loss b. Fatigue, low-grade fever, and night sweats c. Cough with purulent mucus and fever with chills d. Pleuritic pain, nonproductive cough, and temperature elevation at night

b. Fatigue, low-grade fever, and night sweats TB usually develops insidiously with fatigue, malaise, anorexia, low-grade fevers, and night sweats, a dry cough, and unexplained weight loss. Pleuritic pain, flu-like symptoms, and a productive cough may occur with an acute sudden presentation; but dyspnea and hemoptysis are late symptoms.

Which assessment finding in a patient who has received omalizumab (Xolair) is most important to report immediately to the health care provider? a. Pain at injection site b. Flushing and dizziness c. Peak flow reading 75% of normal d. Respiratory rate 22 breaths/minute

b. Flushing and dizziness Flushing and dizziness may indicate that the patient is experiencing an anaphylactic reaction, and immediate intervention is needed. The other information should also be reported, but do not indicate possibly life- threatening complications of omalizumab therapy.

In the patient with HF, which medications or treatments require careful monitoring of the patient's serum potassium level to prevent further cardiac dysfunction (select all that apply)? a. Enalapril (Vasotec) b. Furosemide (Lasix) c. Nesiritide (Natrecor) d. Spironolactone (Aldactone) e. Metoprolol CR/XL (Toprol XL)

b. Furosemide (Lasix) d. Spironolactone (Aldactone) Furosemide is a diuretic that eliminates potassium and spironolactone is a potassium-sparing diuretic that retains potassium. The other treatments and medications are used for patients with HF, but they do not directly affect serum potassium levels.

Which action should the nurse perform when preparing a patient with supraventricular tachycardia for cardioversion who is alert and has a blood pressure of 110/66 mm Hg? a. Turn the synchronizer switch to the off position. b. Give a sedative before cardioversion is implemented. c. Set the defibrillator/cardioverter energy to 360 joules. d. Provide assisted ventilations with a bag-valve-mask device.

b. Give a sedative before cardioversion is implemented. When a patient has a nonemergency cardioversion, sedation is used just before the procedure. The synchronizer switch is turned on for cardioversion. The initial level of joules for cardioversion is low (e.g., 50). Assisted ventilations are not indicated for this patient.

A client with chronic heart failure took cold medicine for her flu. She presents with new productive cough with pink frothy sputum and worsening crackles. What action should the nurse take first? a. Assess lung sounds b. Give bumetanide IV Push c. Notify the HCP d. Clock out for lunch

b. Give bumetanide IV Push - YES "New, sudden, worsening, rapid symptoms" # 1 Action is Furosemide = "Body Dried"

A patient who had a transverse colectomy for diverticulosis 18 hours ago has nasogastric suction and is complaining of anxiety and incisional pain. The patient's respiratory rate is 32 breaths/minute and the arterial blood gases (ABGs) indicate respiratory alkalosis. Which action should the nurse take first? a. Discontinue the nasogastric suction. b. Give the patient the PRN IV morphine sulfate 4 mg. c. Notify the health care provider about the ABG results. d. Teach the patient how to take slow, deep breaths when anxious.

b. Give the patient the PRN IV morphine sulfate 4 mg. The patient's respiratory alkalosis is caused by the increased respiratory rate associated with pain and anxiety. The nurses first action should be to medicate the patient for pain. Although the nasogastric suction may contribute to the alkalosis, it is not appropriate to discontinue the tube when the patient needs gastric suction. The health care provider may be notified about the ABGs but is likely to instruct the nurse to medicate for pain. The patient will not be able to take slow, deep breaths when experiencing pain.

Which action will the nurse in the hypertension clinic take in order to obtain an accurate baseline blood pressure (BP) for a new patient? a. Deflate the BP cuff at a rate of 5 to 10 mm Hg per second. b. Have the patient sit in a chair with the feet flat on the floor. c. Assist the patient to the supine position for BP measurements. d. Obtain two BP readings in the dominant arm and average the results.

b. Have the patient sit in a chair with the feet flat on the floor. The patient should be seated with the feet flat on the floor. The BP is obtained in both arms, and the results of the two arms are not averaged. The patient does not need to be in the supine position. The cuff should be deflated at 2 to 3 mm Hg per second.

When preparing to administer a unit of packed red blood cells to a client with anemia, which nursing actions are appropriate? Select all that apply. a. Ensure that the patient has a functioning central venous access catheter. b. Have two licensed nurses verify the prescription. c. Assess the patient's HgB and Hct. d. Auscultate the patient's heart and lung sounds. e. Prepare a bag of lactated Ringer's to administer with the blood.

b. Have two licensed nurses verify the prescription. c. Assess the patient's HgB and Hct. d. Auscultate the patient's heart and lung sounds.

A patient with a history of hypertension treated with a diuretic and an angiotensin-converting enzyme (ACE) inhibitor arrives in the emergency department complaining of a severe headache and nausea and has a blood pressure (BP) of 238/118 mm Hg. Which question should the nurse ask first? a. Did you take any acetaminophen (Tylenol) today? b. Have you been consistently taking your medications? c. Have there been any recent stressful events in your life? d. Have you recently taken any antihistamine medications?

b. Have you been consistently taking your medications? Sudden withdrawal of antihypertensive medications can cause rebound hypertension and hypertensive crisis. Although many over-the-counter medications can cause hypertension, antihistamines and acetaminophen do not increase BP. Stressful events will increase BP but not usually to the level seen in this patient.

A patient with anemia states... "I'm a universal donor" ... patient has which blood type? a. A type b. O type c. AB type d. B Type

b. O type

The nurse is admitting a patient with possible rheumatic fever. Which question on the admission health history will be most pertinent to ask? a. Do you use any illegal IV drugs? b. Have you had a recent sore throat? c. Have you injured your chest in the last few weeks? d. Do you have a family history of congenital heart disease

b. Have you had a recent sore throat? Rheumatic fever occurs as a result of an abnormal immune response to a streptococcal infection. Although illicit IV drug use should be discussed with the patient before discharge, it is not a risk factor for rheumatic fever, and would not be as pertinent when admitting the patient. Family history is not a risk factor for rheumatic fever. Chest injury would cause musculoskeletal chest pain rather than rheumatic fever.

During the assessment, the nurse identifies crackles in the lungs and an S3 heart sound. Which complication of MI should the nurse suspect and further investigate? a. Pericarditis b. Heart failure c. Ventricular aneurysm d. Papillary muscle dysfunction

b. Heart failure Left-sided HF, which can escalate to cardiogenic shock, initially occurs and manifests as mild dyspnea, restlessness, agitation, pulmonary congestion with crackles, and/or S3 or S4 heart sounds. Right-sided HF includes jugular vein distention, hepatic congestion, or lower extremity edema. Pericarditis is a common complication identified with chest pain that is aggravated by inspiration, coughing, and moving the upper body. Ventricular aneurysm is manifested with HF, dysrhythmias, and angina. Papillary muscle dysfunction is suspected with a new systolic apical murmur.

What sounds can be auscultated in a patient with cardiac valve problems (select all that apply)? a. Arterial bruit b. Heart murmurs c. Pulsus alternans d. Third heart sound (S3 ) e. Pericardial friction rub f. Fourth heart sound (S4 )

b. Heart murmurs d. Third heart sound (S3 ) f. Fourth heart sound (S4 ) The heart murmurs are produced by turbulent blood flow across diseased heart valves, S3 is heard with mitral valve regurgitation, and S4 is heard with aortic stenosis. Arterial bruits are from turbulent peripheral blood flow. Pulsus alternans, seen in heart failure, is a variation in the strength of each pulse when palpated. Pericardial friction rub is the sound heard with pericarditis.

A nurse is reviewing the laboratory report of a client who has fluid volume excess. Which of the following laboratory values should the nurse expect? a. Hemoglobin 20 g/dL b. Hematocrit 34% c. BUN 25 mg/dL d. Urine specific gravity 1.050

b. Hematocrit 34% This hematocrit level is below the expected reference range. A 2+ pitting edema indicates fluid overload, which can cause hemodilution and a decreased hematocrit.

A 45-year-old patient has symptoms including arthralgia, impotence, weight loss, and liver enlargement. His laboratory results include an elevated serum iron, total iron binding capacity (TIBC), and serum ferritin levels. Which disorder does this describe and which treatment will be used? a. Thalassemia; combination chemotherapy b. Hemochromatosis; deferoxamine (Desferal) c. Myelodysplastic syndrome; blood transfusions d. Delayed transfusion reaction; deferasirox (Exjade)

b. Hemochromatosis; deferoxamine (Desferal) The symptoms describe hemochromatosis, which is treated with iron chelating agents to remove accumulated iron via the kidneys. Thalassemia involves inadequate production of normal hemoglobin, is asymptomatic or has growth and development deficits, and is treated with blood transfusion and chelating agents (e.g., deferasirox). Myelodysplastic syndrome has disordered and ineffective hematopoiesis and is treated supportively with hematologic monitoring, antibiotic therapy, transfusions, iron chelators, or intensive chemotherapy and/or hematopoietic stem cell transplantation (HSCT). Delayed transfusion reactions exhibit delayed hemolytic reactions, infections, and iron overload and have no acute treatment unless severe enough to warrant further transfusions.

A patient with a hematologic disorder has a smooth, shiny, red tongue. Which laboratory result would the nurse expect to see? a. Neutrophils 45% b. Hemoglobin (Hgb) 9.6 g/dL (96 g/L) c. WBC count 13,500/µL d. RBC count 6.4 × 10 6 /µL

b. Hemoglobin (Hgb) 9.6 g/dL (96 g/L) A smooth, shiny, reddened tongue is an indication of irondeficiency anemia or pernicious anemia that would be reflected by a decreased hemoglobin level. The decreased neutrophils would be indicative of neutropenia. The increased WBC count could be indicative of an infection and the increased RBC count of polycythemia.

Nursing interventions for the patient with aplastic anemia are directed toward the prevention of which complications? a. Fatigue and dyspnea b. Hemorrhage and infection c. Thromboemboli and gangrene d. Cardiac dysrhythmias and heart failure

b. Hemorrhage and infection Hemorrhage from thrombocytopenia and infection from neutropenia are the greatest risks for the patient with aplastic anemia. The patient will experience fatigue from anemia, but bleeding and infection are the major causes of death in aplastic anemia.

The nurse is reviewing laboratory results and notes an aPTT level of 28 seconds. The nurse should notify the health care provider in anticipation of adjusting which medication? a. Aspirin b. Heparin c. Warfarin d. Erythropoietin

b. Heparin Activated partial thromboplastin time (aPTT) assesses intrinsic coagulation by measuring factors I, II, V, VIII, IX, X, XI, XII. aPTT is increased (prolonged) in heparin administration. aPTT is used to monitor whether heparin is at a therapeutic level (needs to be greater than the normal range of 25 to 35 sec). Prothrombin time (PT) and international normalized ratio (INR) are most commonly used to test for therapeutic levels of warfarin (Coumadin). Aspirin affects platelet function. Erythropoietin is used to stimulate red blood cell production.

The nurse is caring for a patient with a recent history of deep vein thrombosis (DVT) who is scheduled for an emergency appendectomy. Vitamin K is ordered for immediate administration. The international normalized ratio (INR) value is 1.0. Which nursing action is most appropriate? a. Administer the medication as ordered. b. Hold the medication and record in the electronic medical record. c. Hold the medication until the lab result is repeated to verify results. d. Administer the medication and seek an increased dose from the health care provider.

b. Hold the medication and record in the electronic medical record. Vitamin K is the antidote to warfarin (Coumadin), which the patient has most likely been taking before admission for treatment of DVT. Warfarin is an anticoagulant that impairs the ability of the blood to clot. Therefore, it may be necessary to give vitamin K before surgery to reduce the risk of hemorrhage. However, the INR value is normal, and vitamin K is not required, so the medication would be held and recorded in the electronic medical record.

Which method is used to evaluate the ECG responses to normal activity over a period of 1 or 2 days? a. Serial ECGs b. Holter monitoring c. 6-minute walk test d. Event monitor or loop recorder

b. Holter monitoring Holter monitoring involves placing electrodes on the chest attached to a recorder that will record ECG rhythm for 24 to 48 hours while the patient engages in normal activities of daily living (ADLs). The recording is later analyzed for dysrhythmias. Serial ECGs are frequent but not continuous ECGs. The 6-minute walk test measures the distance walked in 6 minutes to determine response to treatments and functional capacity for ADLs. An event monitor or loop recorder is used to record infrequent rhythm disturbances when the patient activates the recording with symptom occurrence.

An 82 year old client with COPD presents with dyspnea, restlessness, pursed lips & in tripod position complaining of anxiety, pain, and "not being able to breathe." The nurse should question which order? Select all that apply. a. Ipratropium b. Hydromorphone c. Rescue inhaler d. Oxygen via nasal cannula 3 L/min e. Diazepam

b. Hydromorphone e. Diazepam

The nurse is encouraging a sedentary patient with major risks for CAD to perform physical exercise on a regular basis. In addition to decreasing the risk factor of physical inactivity, the nurse tells the patient that exercise will directly contribute to reducing which risk factors? a. Diabetes and hypertension b. Hyperlipidemia and obesity c. Increased serum lipids and stressful lifestyle d. Hypertension and increased serum homocysteine

b. Hyperlipidemia and obesity Increased exercise without an increase in caloric intake will result in weight loss, reducing the risk associated with obesity. Exercise increases lipid metabolism and increases HDL, thus reducing CAD risk. Exercise may indirectly reduce the risk of CAD by controlling hypertension, promoting glucose metabolism in diabetes, and reducing stress. While high blood levels of homocysteine are linked to an increased risk for CAD, reducing homocysteine levels has not been shown to reduce the risk of heart disease.

While caring for an 84-year-old patient, the nurse monitors the patient's fluid and electrolyte balance, recognizing what as a normal change of aging? a. Hyperkalemia b. Hyponatremia c. Decreased insensible fluid loss d. Increased plasma oncotic pressures

b. Hyponatremia A decrease in renin and aldosterone and an increase in ADH and atrial natriuretic peptide (ANP) lead to decreased sodium reabsorption and increased water retention by the kidney, both of which lead to hyponatremia. Loss of subcutaneous tissue and thinning dermis of aging lead to increased moisture lost through the skin. Plasma oncotic pressure is often decreased because of lack of protein intake.

A patient is taking diuretic drugs. Which fluid or electrolyte imbalance can occur in this patient (select all that apply)? a. Hyperkalemia b. Hyponatremia c. Hypocalcemia d. Hypotonic fluid loss e. Hypertonic fluid loss

b. Hyponatremia c. Hypocalcemia Because of the osmotic pressure of sodium, water will be excreted with the sodium lost with the diuretic. A change in the relative concentration of sodium will not be seen, but an isotonic fluid loss will occur. Diuretics can also cause a loss of calcium in the urine.

A patient's arterial blood gas (ABG) results include the following: pH 7.32, PaO2 84 mm Hg, PaCO2 49 mm Hg, and SaO2 84%. For what should the nurse assess the patient? a. Tetany b. Hypoventilation c. Pleural friction rub d. Kussmaul respirations

b. HypoventilationThe arterial blood gas analysis indicates respiratory acidosis. Hypoventilation with tachypnea from respiratory distress can cause respiratory acidosis.

The nurse provides dietary teaching for a patient with chronic obstructive pulmonary disease (COPD) who has a low body mass index (BMI). Which patient statement indicates that the teaching has been effective? a. I will drink lots of fluids with my meals. b. I can have ice cream as a snack every day. c. I will exercise for 15 minutes before meals. d. I will decrease my intake of meat and poultry.

b. I can have ice cream as a snack every day. High-calorie foods like ice cream are an appropriate snack for patients with COPD. Fluid intake of 3 L/day is recommended, but fluids should be taken between meals rather than with meals to improve oral intake of solid foods. The patient should avoid exercise for an hour before meals to prevent fatigue while eating. Meat and dairy products are high in protein and are good choices for the patient with COPD.

The nurse has started discharge teaching for a patient who is to continue warfarin (Coumadin) following hospitalization for venous thromboembolism (VTE). The nurse determines that additional teaching is needed when the patient says which of the following? a. I should get a Medic Alert device stating that I take Coumadin. b. I should reduce the amount of green, leafy vegetables that I eat. c. I will need routine blood tests to monitor the effects of the Coumadin. d. I will check with my health care provider before I begin any new medications.

b. I should reduce the amount of green, leafy vegetables that I eat. Patients taking warfarin are taught to follow a consistent diet with regard to foods that are high in vitamin K, such as green, leafy vegetables. The other patient statements are accurate.

The nurse provides discharge instructions to a patient who was hospitalized for pneumonia. Which statement, if made by the patient, indicates a good understanding of the instructions? a. I will call the doctor if I still feel tired after a week. b. I will continue to do the deep breathing and coughing exercises at home. c. I will schedule two appointments for the pneumonia and influenza vaccines. d. Ill cancel my chest x-ray appointment if Im feeling better in a couple weeks.

b. I will continue to do the deep breathing and coughing exercises at home. Patients should continue to cough and deep breathe after discharge. Fatigue is expected for several weeks. The Pneumovax and influenza vaccines can be given at the same time in different arms. Explain that a follow-up chest x-ray needs to be done in 6 to 8 weeks to evaluate resolution of pneumonia.

When evaluating the effectiveness of preoperative teaching with a patient scheduled for coronary artery bypass graft (CABG) surgery using the internal mammary artery, the nurse determines that additional teaching is needed when the patient says which of the following? a. They will circulate my blood with a machine during the surgery. b. I will have small incisions in my leg where they will remove the vein. c. They will use an artery near my heart to go around the area that is blocked. d. I will need to take an aspirin every day after the surgery to keep the graft open.

b. I will have small incisions in my leg where they will remove the vein. When the internal mammary artery is used there is no need to have a saphenous vein removed from the leg. The other statements by the patient are accurate and indicate that the teaching has been effective.

Myocardial ischemia occurs because of increased oxygen demand and decreased oxygen supply. What factors and disorders result in increased oxygen demand (select all that apply)? a. Hypovolemia or anemia b. Increased cardiac workload with aortic stenosis c. Narrowed coronary arteries from atherosclerosis d. Angina in the patient with atherosclerotic coronary arteries e. Left ventricular hypertrophy caused by chronic hypertension f. Sympathetic nervous system stimulation by drugs, emotions, or exertion

b. Increased cardiac workload with aortic stenosis d. Angina in the patient with atherosclerotic coronary arteries e. Left ventricular hypertrophy caused by chronic hypertension f. Sympathetic nervous system stimulation by drugs, emotions, or exertion Increased oxygen demand is caused by increasing the workload of the heart, including left ventricular hypertrophy with hypertension, sympathetic nervous stimulation, and anything precipitating angina. Hypovolemia, anemia, and narrowed coronary arteries contribute to decreased oxygen supply

Which effects result from sympathetic nervous system stimulation of βadrenergic receptors (select all that apply)? a. Vasoconstriction b. Increased heart rate c. Decreased heart rate d. Increased rate of impulse conduction e. Decreased rate of impulse conduction f. Increased force of cardiac contraction

b. Increased heart rate d. Increased rate of impulse conduction f. Increased force of cardiac contraction The sympathetic nervous system increases the heart rate, the speed of impulse conduction through the atrioventricular (AV) node, and the force of atrial and ventricular contractions via the β-adrenergic receptors.

What is the pathophysiologic mechanism that results in the pulmonary edema of leftsided HF? a. Increased right ventricular preload b. Increased pulmonary hydrostatic pressure c. Impaired alveolar oxygen and carbon dioxide exchange d. Increased lymphatic flow of pulmonary extravascular fluid

b. Increased pulmonary hydrostatic pressure In left-sided HF, blood backs up into the pulmonary veins and capillaries. This increased hydrostatic pressure in the vessels causes fluid to move out of the vessels and into the pulmonary interstitial space. When increased lymphatic flow cannot remove enough fluid from the interstitial space, fluid moves into the alveoli, resulting in pulmonary edema and impaired alveolar oxygen and carbon dioxide exchange. Initially the right side of the heart is not involved.

How does the nurse expect the client to show compensation for the following ABG values? Ph 7.20, PaO2 82 mm Hg, PaCO2 37 mm Hg, HCO3 15 mEq/L (metabolic acidosis) a. Decreased respiratory rate b. Increased respiratory rate c. Increased renal retention of H+ d. Decreased renal excretion of HCO3

b. Increased respiratory rate

Patients are at risk for which complications in the first year after heart transplantation? (select all that apply) a. Cancer b. Infection c. Rejection d. Vasculopathy e. Sudden cardiac death

b. Infection c. Rejection e. Sudden cardiac death

IV potassium chloride (KCl) 60 mEq is prescribed for treatment of a patient with severe hypokalemia. Which action should the nurse take? a. Administer the KCl as a rapid IV bolus. b. Infuse the KCl at a rate of 10 mEq/hour. c. Only give the KCl through a central venous line. d. Discontinue cardiac monitoring during the infusion.

b. Infuse the KCl at a rate of 10 mEq/hour. IV KCl is administered at a maximal rate of 10 mEq/hr. Rapid IV infusion of KCl can cause cardiac arrest. Although the preferred concentration for KCl is no more than 40 mEq/L, concentrations up to 80 mEq/L may be used for some patients. KCl can cause inflammation of peripheral veins, but it can be administered by this route. Cardiac monitoring should be continued while patient is receiving potassium because of the risk for dysrhythmias.

A nurse is caring for a client who is experiencing respiratory distress as a result of pulmonary edema. Which of the following actions should the nurse take first? a. Assist with intubation. b. Initiate high-flow oxygen therapy. c. Administer a rapid-acting diuretic. d. Provide cardiac monitoring.

b. Initiate high-flow oxygen therapy The priority action the nurse should take when using the airway, breathing, circulation approach to client care is to administer high-flow oxygen therapy by face mask at 5 to 6 L/min to keep the client's oxygen saturation above 90%.

The clinic nurse makes a follow-up telephone call to a patient with asthma. The patient reports having a baseline peak flow reading of 600 L/minute and the current peak flow is 420 L/minute. Which action should the nurse take first? a. Tell the patient to go to the hospital emergency department. b. Instruct the patient to use the prescribed albuterol (Proventil). c. Ask about recent exposure to any new allergens or asthma triggers. d. Question the patient about use of the prescribed inhaled corticosteroids.

b. Instruct the patient to use the prescribed albuterol (Proventil). The patients peak flow is 70% of normal, indicating a need for immediate use of short-acting b2-adrenergic SABA medications. Assessing for correct use of medications or exposure to allergens also is appropriate, but would not address the current decrease in peak flow. Because the patient is currently in the yellow zone, hospitalization is not needed.

A 67-yr-old man with peripheral artery disease is seen in the primary care clinic. Which symptom reported by the patient would indicate to the nurse that the patient is experiencing intermittent claudication? a Patient complains of chest pain with strenuous activity. b Patient says muscle leg pain occurs with continued exercise. c Patient has numbness and tingling of all his toes and both feet. d Patient states the feet become red if he puts them in a dependent position.

b. Intermittent claudication is an ischemic muscle ache or pain that is precipitated by a consistent level of exercise, resolves within 10 minutes or less with rest, and is reproducible. Angina is the term used to describe chest pain with exertion. Paresthesia is the term used to describe numbness or tingling in the toes or feet. Reactive hyperemia is the term used to describe redness of the foot; if the limb is in a dependent position the term is dependent rubor.

The nurse is assessing a patient with myocarditis before administering the scheduled dose of digoxin (Lanoxin). Which finding is most important for the nurse to communicate to the health care provider? a. Leukocytosis b. Irregular pulse c. Generalized myalgia d. Complaint of fatigue

b. Irregular pulse Myocarditis predisposes the heart to digoxin-associated dysrhythmias and toxicity. The other findings are common symptoms of myocarditis and there is no urgent need to report these.

After the nurse teaches the patient about the use of carvedilol (Coreg) in preventing anginal episodes, which statement by a patient indicates that the teaching has been effective? a. Carvedilol will help my heart muscle work harder. b. It is important not to suddenly stop taking the carvedilol. c. I can expect to feel short of breath when taking carvedilol. d. Carvedilol will increase the blood flow to my heart muscle.

b. It is important not to suddenly stop taking the carvedilol. Patients who have been taking b-adrenergic blockers can develop intense and frequent angina if the medication is suddenly discontinued. Carvedilol (Coreg) decreases myocardial contractility. Shortness of breath that occurs when taking b-adrenergic blockers for angina may be due to bronchospasm and should be reported to the health care provider. Carvedilol works by decreasing myocardial oxygen demand, not by increasing blood flow to the coronary arteries.

When developing a teaching plan for a 76-year-old patient newly diagnosed with peripheral artery disease (PAD), which instructions should the nurse include? a. Exercise only if you do not experience any pain. b. It is very important that you stop smoking cigarettes. c. Try to keep your legs elevated whenever you are sitting. d. Put elastic compression stockings on early in the morning.

b. It is very important that you stop smoking cigarettes. Smoking cessation is essential for slowing the progression of PAD to critical limb ischemia and reducing the risk of myocardial infarction and death. Circulation to the legs will decrease if the legs are elevated. Patients with PAD are taught to exercise to the point of feeling pain, rest, and then resume walking. Support hose are not used for patients with PAD.

A patient is receiving 35% oxygen via a Venturi mask. To ensure the correct amount of oxygen delivery, which action by the nurse is most important? a. Teach the patient to keep mask on at all times. b. Keep the air entrainment ports clean and unobstructed. c. Give a high enough flow rate to keep the bag from collapsing. d. Drain moisture condensation from the oxygen tubing every hour.

b. Keep the air entrainment ports clean and unobstructed. The air entrainment ports regulate the oxygen percentage delivered to the patient, so they must be unobstructed. A high oxygen flow rate is needed when giving oxygen by partial rebreather or non-rebreather masks. Draining oxygen tubing is necessary when caring for a patient receiving mechanical ventilation. The mask is uncomfortable and can be removed when the patient eats.

A patient with a chronic cough has a bronchoscopy. After the procedure, which intervention by the nurse is most appropriate? a. Elevate the head of the bed to 80 to 90 degrees. b. Keep the patient NPO until the gag reflex returns. c. Place on bed rest for at least 4 hours after bronchoscopy. d. Notify the health care provider about blood-tinged mucus.

b. Keep the patient NPO until the gag reflex returns. Risk for aspiration and maintaining an open airway is the priority. Because a local anesthetic is used to suppress the gag/cough reflexes during bronchoscopy, the nurse should monitor for the return of these reflexes before allowing the patient to take oral fluids or food. Blood-tinged mucus is not uncommon after bronchoscopy. The patient does not need to be on bed rest, and the head of the bed does not need to be in the high-Fowlers position

A diabetic patients arterial blood gas (ABG) results are pH 7.28; PaCO2 34 mm Hg; PaO2 85 mm Hg; HCO3 18 mEq/L. The nurse would expect which finding? a. Intercostal retractions b. Kussmaul respirations c. Low oxygen saturation (SpO2) d. Decreased venous O2 pressure

b. Kussmaul respirations Kussmaul (deep and rapid) respirations are a compensatory mechanism for metabolic acidosis. The low pH and low bicarbonate result indicate metabolic acidosis. Intercostal retractions, a low oxygen saturation rate, and a decrease in venous O2 pressure would not be caused by acidosis

Which action is appropriate for the nurse to delegate to unlicensed assistive personnel (UAP)? a. Listen to a patients lung sounds for wheezes or rhonchi. b. Label specimens obtained during percutaneous lung biopsy. c. Instruct a patient about how to use home spirometry testing. d. Measure induration at the site of a patients intradermal skin test.

b. Label specimens obtained during percutaneous lung biopsy. Labeling of specimens is within the scope of practice of UAP. The other actions require nursing judgment and should be done by licensed nursing personnel.

A nurse is caring for a client who is being treated for HF and has prescriptions for furosemide. The nurse should plan to monitor for which of the following as an adverse effect of this medication? A. SOB b. Lightheadedness c. Dry cough d. Metallic taste

b. Lightheadedness Rationale: Furosemide can cause a substantial drop in blood pressure, resulting in lightheadedness or dizziness.

When caring for a patient on the first postoperative day after an abdominal aortic aneurysm repair, which assessment finding is most important for the nurse to communicate to the health care provider? a. Presence of flatus b. Loose, bloody stools c. Hypoactive bowel sounds d. Abdominal pain with palpation

b. Loose, bloody stools Loose, bloody stools at this time may indicate intestinal ischemia or infarction, and should be reported immediately because the patient may need an emergency bowel resection. The other findings are normal on the first postoperative day after abdominal surgery.

A patient with pneumonia is having difficulty clearing the airway because of pain, fatigue, and thick secretions. What is an expected outcome for this patient? a. SpO2 is 90% b. Lungs clear to auscultation c. Patient tolerates walking in hallway d. Patient takes 3 to 4 shallow breaths before coughing to minimize pain

b. Lungs clear to auscultation Clear lung sounds indicate that the airways are clear. SpO2 of 95% to 100% indicates appropriate gas exchange. Tolerating walking in the hallway indicates appropriate gas exchange and activity tolerance, not improved airway clearance. Deep breaths are necessary to move mucus from distal airways, but this is not an outcome for this nursing diagnosis.

Molecular cytogenetics and gene analysis may be done to diagnose, stage, and help determine treatment options for various hematologic disorders. Which sites are preferred to obtain the sample for this testing (select all that apply)? a. Skin sample b. Lymph node c. Bone marrow d. Arterial blood e. Inner cheek mucosa

b. Lymph node c. Bone marrow Bone marrow and lymph node biopsies are preferred methods to obtain the sample for gene analysis. If a large number of abnormal cells are circulating in the blood, peripheral blood may be used. The other options will not provide the desired information.

The nurse is admitting a patient diagnosed with an acute exacerbation of chronic obstructive pulmonary disease (COPD).What is the best way for the nurse to determine the appropriate oxygen flow rate? a. Minimize oxygen use to avoid oxygen dependency. b. Maintain the pulse oximetry level at 90% or greater. c. Administer oxygen according to the patients level of dyspnea. d. Avoid administration of oxygen at a rate of more than 2 L/minute.

b. Maintain the pulse oximetry level at 90% or greater. The best way to determine the appropriate oxygen flow rate is by monitoring the patients oxygenation either by arterial blood gases (ABGs) or pulse oximetry. An oxygen saturation of 90% indicates adequate blood oxygen level without the danger of suppressing the respiratory drive. For patients with an exacerbation of COPD, an oxygen flow rate of 2 L/min may not be adequate. Because oxygen use improves survival rate in patients with COPD, there is no concern about oxygen dependency. The patients perceived dyspnea level may be affected by other factors (such as anxiety) besides blood oxygen level.

While participating in a community health fair, a nurse is providing information to a client who has a blood pressure of 149/95 during screening. Which of the following action should the nurse take? a. Encourage client to go urgent care b. Make client keep a record of bp to take to his provider c. Instruct client to do a 4 month follow up d. He is not at risk for hypertension

b. Make client keep a record of bp to take to his provider

The nurse is caring for an 86-year-old visually impaired client with thrombocytopenia. Which intervention is most important to implement? a. Making sure oral care is provided with a soft toothbrush. b. Making sure the client's call light is within reach. c. Requesting an order for a urinary catheter. d. Orient the client to the room.

b. Making sure the client's call light is within reach. This answer is correct because the client is a fall risk because of age and visual impairment. Client safety is always the number 1 priority. If the client who is thrombocytopenic falls, the client is at risk for a major bleed. The call light being within reach will alert the nurse when the client requires assistance.

What characteristics should the nurse be aware of in planning care for the patient with Hodgkin's lymphoma? a. Staging of Hodgkin's lymphoma is not important to predict prognosis. b. Management of the patient being treated for Hodgkin's lymphoma includes measures to prevent infection. c. Hodgkin's lymphoma is characterized by proliferation of malignant activated B cells that destroy the kidneys. d. An important nursing intervention in the care of patients with Hodgkin's lymphoma is increasing fluids to manage hypercalcemia.

b. Management of the patient being treated for Hodgkin's lymphoma includes measures to prevent infection. The patient is monitored for infection because leukopenia and thrombocytopenia may develop from the disease or usually as a consequence of treatment. Staging of Hodgkin's disease is important to determine treatment. Multiple myeloma is characterized by proliferation of malignant activated B cells that destroy the bones. The intervention of increasing fluid to manage hypercalcemia is used with multiple myeloma.

Priority Decision: A patient is admitted to the emergency department with an acute asthma attack. Which patient assessment is of greatest concern to the nurse? a. The presence of a pulsus paradoxus b. Markedly decreased breath sounds with no wheezing c. A respiratory rate of 34 breaths/min and increased pulse and BP d. Use of accessory muscles of respiration and a feeling of suffocation

b. Markedly decreased breath sounds with no wheezing Decreased or absent breath sounds may indicate a significant decrease in air movement resulting from exhaustion and an inability to generate enough muscle force to ventilate and is an ominous sign. The other symptoms are expected in an asthma attack but are not life threatening.

With which disorder is hyperkalemia often associated? a. Hypoglycemia b. Metabolic acidosis c. Respiratory alkalosis d. Decreased urine potassium levels

b. Metabolic acidosis In metabolic acidosis, hydrogen ions in the blood are taken into the cell in exchange for potassium ions as a means of buffering excess acids. This results in an increase in serum potassium until the kidneys have time to excrete the excess potassium

Which clinical findings should the nurse expect in a person with an acute lower extremity VTE? (select all that apply) a. Pallor and coolness of foot and calf b. Mild to moderate calf pain and tenderness c. Grossly decreased or absent pedal pulses d. Unilateral edema and induration of the thigh e. Palpable cord along a superficial varicose vein

b. Mild to moderate calf pain and tenderness d. Unilateral edema and induration of the thigh

When caring for a patient with renal failure on a low phosphate diet, the nurse will inform unlicensed assistive personnel (UAP) to remove which food from the patient's food tray? a. Grape juice b. Milk carton c. Mixed green salad d. Fried chicken breast

b. Milk carton Foods high in phosphate include milk and other dairy products, so these are restricted on low-phosphate diets. Green, leafy vegetables; high-fat foods; and fruits/juices are not high in phosphate and are not restricted.

What is the rationale for using docusate sodium (Colace) for a patient after an MI? a. Relieves cardiac workload b. Minimizes vagal stimulation c. Controls ventricular dysrhythmias d. Prevents the binding of fibrinogen to platelets

b. Minimizes vagal stimulation Docusate sodium (Colace) is a stool softener, which prevents straining and provoking dysrhythmias. It does not do any of the other options. Antidysrhythmics are used to control ventricular dysrhythmias; morphine sulfate is used to decrease anxiety and cardiac workload; and glycoprotein IIb/IIIa inhibitors and antiplatelets prevent the binding of fibrinogen to platelets

Which heart valve sound is heard best at the left midclavicular line at the level of the fifth ICS? a. Aortic b. Mitral c. Tricuspid d. Pulmonic

b. Mitral

Collaboration: The unit is very busy and short staffed. What could the RN delegate to the unlicensed assistive personnel (UAP)? a. Administer antihypertensive medications to stable patients. b. Obtain orthostatic BP readings for older patients. c. Check BP readings for the patient receiving IV sodium nitroprusside. d. Teach about home BP monitoring and use of automatic BP monitoring equipment.

b. Obtain orthostatic BP readings for older patients. The UAP may check postural changes in BP as directed. The LPN may administer antihypertensive medications to stable patients. The RN must monitor the patient receiving IV sodium nitroprusside, as the patient is in a hypertensive crisis. The RN must also do the teaching related to home BP monitoring.

When teaching the patient with PAD about modifying risk factors associated with the condition, what should the nurse emphasize? a. Amputation is the ultimate outcome if the patient does not alter lifestyle behaviors. b. Modifications will reduce the risk of other atherosclerotic conditions, such as stroke. c. Risk-reducing behaviors started after angioplasty can stop the progression of the disease. d. Maintenance of normal body weight is the most important factor in controlling arterial disease.

b. Modifications will reduce the risk of other atherosclerotic conditions, such as stroke. PAD occurs as a result of atherosclerosis and the risk factors are the same as for other diseases associated with atherosclerosis, such as CAD, cerebrovascular disease, and aneurysms. Major risk factors are tobacco use, hyperlipidemia, elevated C-reactive protein, diabetes, obesity, and uncontrolled hypertension. The risk for amputation is high in patients with severe occlusive disease, but this is not the best approach to encourage patients to make lifestyle modifications

What is included in the nursing care of the patient with a cuffed tracheostomy tube? a. Change the tube every 3 days. b. Monitor cuff pressure every 8 hours. c. Perform mouth care every 12 hours. d. Assess arterial blood gases every 8 hours.

b. Monitor cuff pressure every 8 hours. Cuff pressure should be monitored at least every 8 hours to ensure that an air leak around the cuff does not occur and that the pressure is not too high to allow adequate tracheal capillary perfusion. Respiratory therapists in some institutions will record the cuff pressure, but the nurse must be able to assess cuff pressure and identify if there is a problem maintaining cuff pressure. Tracheostomy tubes are changed monthly when needed for long-term use. Mouth care should be performed a minimum of every 8 hours and more often as needed to remove dried secretions. Arterial blood gases (ABGs) are not routinely assessed with tracheostomy tube placement unless symptoms of respiratory distress continue.

Which nursing action should be included in the plan of care after endovascular repair of an abdominal aortic aneurysm? a. Record hourly chest tube drainage. b. Monitor fluid intake and urine output. c. Check the abdominal incision for any redness. d. Teach the reason for a prolonged recovery period.

b. Monitor fluid intake and urine output. Because renal artery occlusion can occur after endovascular repair, the nurse should monitor parameters of renal function such as intake and output. Chest tubes will not be needed for endovascular surgery, the recovery period will be short, and there will not be an abdominal wound.

Which action can the registered nurse (RN) who is caring for a critically ill patient with multiple IV lines delegate to an experienced licensed practical/vocational nurse (LPN/LVN)? a. Administer IV antibiotics through the implantable port. b. Monitor the IV sites for redness, swelling, or tenderness. c. Remove the patients no tunneled subclavian central venous catheter. d. Adjust the flow rate of the 0.9% normal saline in the peripheral IV line.

b. Monitor the IV sites for redness, swelling, or tenderness. An experienced LPN/LVN has the education, experience, and scope of practice to monitor IV sites for signs of infection. Administration of medications, adjustment of infusion rates, and removal of central catheters in critically ill patients require RN level education and scope of practice.

The client diagnosed with congestive heart failure is complaining of leg cramps at night. Which nursing interventions should be implemented? a .Check the client for peripheral edema and make sure the client takes a diuretic early in the day. b. Monitor the client's potassium level and assess the client's intake of bananas and orange juice. c. Monitor ECG dysrhythmias specifically looking for tall, peaked T waves. d. Instruct the client to ambulate frequently and perform calf-muscle stretching exercises daily.

b. Monitor the client's potassium level and assess the client's intake of bananas and orange juice. The most probable cause of the leg cramping is hypokalemia as a result of diuretic medication. Bananas and orange juice are foods that are high in potassium.

The nurse teaches a patient about the transmission of pulmonary tuberculosis (TB). Which statement, if made by the patient, indicates that teaching was effective? a. I will avoid being outdoors whenever possible. b. My husband will be sleeping in the guest bedroom. c. I will take the bus instead of driving to visit my friends. d. I will keep the windows closed at home to contain the germs.

b. My husband will be sleeping in the guest bedroom. Teach the patient how to minimize exposure to close contacts and household members. Homes should be well ventilated, especially the areas where the infected person spends a lot of time. While still infectious, the patient should sleep alone, spend as much time as possible outdoors, and minimize time in congregate settings or on public transportation.

Which characteristics describe unstable angina (select all that apply)? a. Usually precipitated by exertion b. New-onset angina with minimal exertion c. Occurs only when the person is recumbent d. Characterized by increased duration or severity e. Usually occurs in response to coronary artery spasm

b. New-onset angina with minimal exertion d. Characterized by increased duration or severity Unstable angina is new-onset angina occurring at rest or with minimal exertion and increases in frequency, duration, or severity. Chronic stable angina is usually precipitated by exertion. Angina decubitus occurs when the person is recumbent. Prinzmetal's angina is often caused by a coronary artery spasm.

The nurse obtains the following information from a patient newly diagnosed with prehypertension. Which finding is most important to address with the patient? a. Low dietary fiber intake b. No regular aerobic exercise c. Weight 5 pounds above ideal weight d. Drinks a beer with dinner on most nights

b. No regular aerobic exercise The recommendations for preventing hypertension include exercising aerobically for 30 minutes most days of the week. A weight that is 5 pounds over the ideal body weight is not a risk factor for hypertension. The Dietary Approaches to Stop Hypertension (DASH) diet is high in fiber, but increasing fiber alone will not prevent hypertension from developing. The patients alcohol intake is within guidelines and will not increase the hypertension risk.

A patient with septicemia develops prolonged bleeding from venipuncture sites and blood in the stools. Which action is most important for the nurse to take? a. Avoid venipunctures. b. Notify the patients physician. c. Apply sterile dressings to the sites. d. Give prescribed proton-pump inhibitors.

b. Notify the patients physician. The patients new onset of bleeding and diagnosis of sepsis suggest that disseminated intravascular coagulation (DIC) may have developed, which will require collaborative actions such as diagnostic testing, blood product administration, and heparin administration. The other actions also are appropriate, but the most important action should be to notify the physician so that DIC treatment can be initiated rapid.

The nurse is preparing the patient for and will assist the health care provider with a thoracentesis in the patient's room. Which of the following is the first action? a. Verify breath sounds in all fields. b. Obtain the supplies that will be used. c. Send labeled specimen containers to the laboratory. d. Direct the family members to the waiting room. e. Observe for signs of hypoxia during the procedure. f. Instruct the patient not to talk during the procedure. g. Position the patient sitting upright with the elbows on an over-the bed table.

b. Obtain the supplies that will be used. The nurse will gather the supplies as soon as the order to do a thoracentesis is given.

The nurse is planning to administer a transfusion of packed red blood cells (PRBCs) to a patient with blood loss from gastrointestinal hemorrhage. Which action can the nurse delegate to unlicensed assistive personnel (UAP)? a. Verify the patient identification (ID) according to hospital policy. b. Obtain the temperature, blood pressure, and pulse before the transfusion. c. Double-check the product numbers on the PRBCs with the patient ID band. d. Monitor the patient for shortness of breath or chest pain during the transfusion.

b. Obtain the temperature, blood pressure, and pulse before the transfusion. UAP education includes measurement of vital signs. UAP would report the vital signs to the registered nurse (RN). The other actions require more education and a larger scope of practice and should be done by licensed nursing staff members.

The nurse is caring for a patient who is 2 days post MI. The patient reports that she is experiencing chest pain when she takes a deep breath. Which action would be a priority? a. Notify the provider STAT and obtain a 12-lead ECG. b. Obtain vital signs and auscultate for a pericardial friction rub. c. Apply high-flow O2 by face mask and auscultate breath sounds. d. Medicate the patient with as-needed analgesic and reevaluate in 30 minutes.

b. Obtain vital signs and auscultate for a pericardial friction rub.

The nurse is caring for a patient with critical limb ischemia who has just arrived on the nursing unit after having percutaneous transluminal balloon angioplasty. Which action should the nurse perform first? a. Begin oral intake. b. Obtain vital signs. c. Assess pedal pulses. d. Start discharge teaching.

b. Obtain vital signs. Bleeding is a possible complication after catheterization of the femoral artery, so the nurses first action should be to assess for changes in vital signs that might indicate hemorrhage. The other actions are also appropriate but can be done after determining that bleeding is not occurring.

The microorganisms Pneumocystis jiroveci (PJP) and cytomegalovirus (CMV) are associated with which type of pneumonia? a. Necrotizing pneumonia b. Opportunistic pneumonia c. HAP d. CAP

b. Opportunistic pneumonia People at risk for opportunistic pneumonia include those with altered immune responses. Pneumocystis jiroveci rarely causes pneumonia in healthy individuals but is the most common cause of pneumonia in persons with human immunodeficiency (HIV) disease. Cytomegalovirus (CMV) occurs in people with an impaired immune response. Necrotizing pneumonia is caused by Staphylococcus, Klebsiella, and Streptococcus. Hospital-acquired pneumonia (HAP) is frequently caused by Pseudomonas aeruginosa, Escherichia coli, Klebsiella, and Acinetobacter. Community-acquired pneumonia (CAP) is most commonly caused by Streptococcus pneumonia.

Which admission order written by the health care provider for a patient admitted with infective endocarditis (IE) and a fever would be a priority for the nurse to implement? a. Administer ceftriaxone (Rocephin) 1 g IV. b. Order blood cultures drawn from two sites. c. Give acetaminophen (Tylenol) PRN for fever. d. Arrange for a transesophageal echocardiogram.

b. Order blood cultures drawn from two sites. Treatment of the IE with antibiotics should be started as quickly as possible, but it is essential to obtain blood cultures before initiating antibiotic therapy to obtain accurate sensitivity results. The echocardiogram and acetaminophen administration also should be implemented rapidly, but the blood cultures (and then administration of the antibiotic) have the highest priority.

The nurse determines that a patient is experiencing common adverse effects from the inhaled corticosteroid beclomethasone (Beclovent) after what occurs? a. Hypertension and pulmonary edema b. Oropharyngeal candidiasis and hoarseness c. Elevation of blood glucose and calcium levels d. Adrenocortical dysfunction and hyperglycemia

b. Oropharyngeal candidiasis and hoarseness Oropharyngeal candidiasis and hoarseness are common adverse effects from the use of inhaled corticosteroids because the medication can lead to overgrowth of organisms and local irritation if the patient does not rinse the mouth following each dose.

A nurse is planning care for a client who has a serum potassium level of 3.0 mEq/L. The nurse should plan to monitor the client for which of the following findings? a. Hyperactive deep-tendon reflexes b. Orthostatic hypotension c. Rapid, deep respirations d. Strong, bounding pulse

b. Orthostatic hypotension Hypokalemia can lead to hypotension. The nurse should monitor the client for orthostatic hypotension.

The emergency department nurse is evaluating the effectiveness of therapy for a patient who has received treatment during an asthma attack. Which assessment finding is the best indicator that the therapy has been effective? a. No wheezes are audible. b. Oxygen saturation is >90%. c. Accessory muscle use has decreased. d. Respiratory rate is 16 breaths/minute.

b. Oxygen saturation is >90%. The goal for treatment of an asthma attack is to keep the oxygen saturation >90%. The other patient data may occur when the patient is too fatigued to continue with the increased work of breathing required in an asthma attack.

Which function test fits description "Maximum rate of airflow during forced expiration"? a. FVC b. PEFR c. RV d. FRC

b. PEFR Peak expiratory flow rate

A nurse is caring for a client who reports difficulty breathing and tingling in both hands. His respiratory rate is 36/min and he appears very restless. Which of the following values should the nurse anticipate to be outside the expected reference range if the client is experiencing respiratory alkalosis? a. PaO2 b. PaCO2 c. Sodium d. Bicarbonate

b. PaCO2 With respiratory alkalosis, the PaCO2 level is decreased.

Risk Factor for or Response to Respiratory Problem: Sexual activity altered by respiratory symptoms. Which Functional Health Pattern does it fall under? a. Role-relationship b. Sexuality-reproductive c. Coping- stress tolerance d. Value-belief

b. Sexuality-reproductive

Triggered by stress, hypoxia or trauma, this anemia results in RBCs forming a crescent shape. a. Thalassemia b. Sickle Cell c. Hemolytic d. Pernicious

b. Sickle Cell

Which assessment finding by the nurse caring for a patient who has had coronary artery bypass grafting using a right radial artery graft is most important to communicate to the health care provider? a. Complaints of incisional chest pain b. Pallor and weakness of the right hand c. Fine crackles heard at both lung bases d. Redness on both sides of the sternal incision

b. Pallor and weakness of the right hand The changes in the right hand indicate compromised blood flow, which requires immediate evaluation and actions such as prescribed calcium channel blockers or surgery. The other changes are expected and/or require nursing interventions.

What is a possible cause for auscultation abnormal finding "stridor"? a. Bronchoconstriction b. Partial obstruction of trachea or larynx c. Chronic hypoxemia d. Pleurisy

b. Partial obstruction of trachea or larynx

A patient is scheduled for a computed tomography (CT) of the chest with contrast media. Which assessment findings should the nurse immediately report to the health care provider (select all that apply)? a. Patient is claustrophobic. b. Patient is allergic to shellfish. c. Patient recently used a bronchodilator inhaler. d. Patient is not able to remove a wedding band. e. Blood urea nitrogen (BUN) and serum creatinine levels are elevated.

b. Patient is allergic to shellfish. e. Blood urea nitrogen (BUN) and serum creatinine levels are elevated. Because the contrast media is iodine-based and may cause dehydration and decreased renal blood flow, asking about iodine allergies (such as allergy to shellfish) and monitoring renal function before the CT scan are necessary. The other actions are not contraindications for CT of the chest, although they may be for other diagnostic tests, such as magnetic resonance imaging (MRI) or pulmonary function testing (PFT).

After receiving the following information about four patients during change-of-shift report, which patient should the nurse assess first? a. Patient with acute pericarditis who has a pericardial friction rub b. Patient who has just returned to the unit after balloon valvuloplasty c. Patient who has hypertrophic cardiomyopathy and a heart rate of 116 d. Patient with a mitral valve replacement who has an anticoagulant scheduled

b. Patient who has just returned to the unit after balloon valvuloplasty The patient who has just arrived after balloon valvuloplasty will need assessment for complications such as bleeding and hypotension. The information about the other patients is consistent with their diagnoses and does not indicate any complications or need for urgent assessment or intervention.

After receiving change-of-shift report on a heart failure unit, which patient should the nurse assess first? a. Patient who is taking carvedilol (Coreg) and has a heart rate of 58 b. Patient who is taking digoxin and has a potassium level of 3.1 mEq/L c. Patient who is taking isosorbide dinitrate/hydralazine (BiDil) and has a headache d. Patient who is taking captopril (Capoten) and has a frequent nonproductive cough

b. Patient who is taking digoxin and has a potassium level of 3.1 mEq/L The patients low potassium level increases the risk for digoxin toxicity and potentially fatal dysrhythmias. The nurse should assess the patient for other signs of digoxin toxicity and then notify the health care provider about the potassium level. The other patients also have side effects of their medications, but their symptoms do not indicate potentially life-threatening complications.

The nurse who works in the vascular clinic has several patients with venous insufficiency scheduled today. Which patient should the nurse assign to an experienced licensed practical/vocational nurse (LPN/LVN)? a. Patient who has been complaining of increased edema and skin changes in the legs b. Patient who needs wound care for a chronic venous stasis ulcer on the right lower leg c. Patient who has a history of venous thromboembolism and is complaining of some dyspnea d. Patient who needs teaching about the use of elastic compression stockings for venous insufficiency

b. Patient who needs wound care for a chronic venous stasis ulcer on the right lower leg LPN education and scope of practice includes wound care. The other patients, which require more complex assessments or education, should be managed by the RN.

After receiving report on the following patients, which patient should the nurse assess first? a. Patient with rheumatic fever who has sharp chest pain with a deep breath b. Patient with acute aortic regurgitation whose blood pressure is 86/54 mm Hg c. Patient with infective endocarditis who has a murmur and splinter hemorrhages d. Patient with dilated cardiomyopathy who has bilateral crackles at the lung bases

b. Patient with acute aortic regurgitation whose blood pressure is 86/54 mm Hg Hypotension in patients with acute aortic regurgitation may indicate cardiogenic shock. The nurse should immediately assess this patient for other findings such as dyspnea or chest pain. The findings in the other patients are typical of their diagnoses and do not indicate a need for urgent assessment and intervention.

Which patient at the cardiovascular clinic requires the most immediate action by the nurse? a. Patient with type 2 diabetes whose current blood glucose level is 145 mg/dL b. Patient with stable angina whose chest pain has recently increased in frequency c. Patient with familial hypercholesterolemia and a total cholesterol of 465 mg/dL d. Patient with chronic hypertension whose blood pressure today is 172/98 mm Hg

b. Patient with stable angina whose chest pain has recently increased in frequency The history of more frequent chest pain suggests that the patient may have unstable angina, which is part of the acute coronary syndrome spectrum. This will require rapid implementation of actions such as cardiac catheterization and possible percutaneous coronary intervention. The data about the other patients suggest that their conditions are stable.

What finding is an indication of marked bronchoconstriction with air trapping and hyperinflation of the lungs in a patient with asthma? a. Arterial oxygen saturation (SaO2 ) of 85% b. Peak (expiratory) flow meter (PEF) rate of < 200 L/min c. Forced expiratory volume in 1 second (FEV1 ) of 85% of predicted d. Chest x-ray showing a flattened diaphragm

b. Peak (expiratory) flow meter (PEF) rate of < 200 L/min Peak expiratory flow rates (PEFRs) are normally up to 600 L/min and in a severe asthma attack may be as low as 100 to 150 L/min. An arterial oxygen saturation (SaO2 ) of 85% and a forced expiratory volume in 1 second (FEV1 ) of 85% of predicted are typical of mild to well controlled asthma. A flattened diaphragm may be present in the patient with long-standing asthma but does not reflect current bronchoconstriction.

Risk Factor for or Response to Respiratory Problem: Sleep apnea. Awakening with dyspnea, wheezing, or cough. Night sweats. Which Functional Health Pattern does it fall under? a. Activity-exercise b. Sleep-rest c. Cognitive-perceptual d. Self-perception- self-concept

b. Sleep-rest

What describes the refractory phase? a. Abnormal electrical impulses b. Period in which heart tissue cannot be stimulated c. Areas of the heart do not repolarize at the same rate because of depressed conduction d. Sodium migrates rapidly into the cell, so it is positive compared to the outside of the cell

b. Period in which heart tissue cannot be stimulated Refractory phase is the period in which heart tissue cannot be stimulated. Ectopic foci produce abnormal electrical impulses. Reentrant excitation causing premature beats may occur when areas of the heart do not repolarize simultaneously with depressed conduction. Depolarization of cardiac cells occurs when sodium migrates rapidly into the cell.

A client who has a new diagnosis of hypertension has a prescription for an ACE inhibitor. The nurse instructs the client about adverse effects of the medication. The client demonstrates an understanding of the teaching by stating that he will notify his provider if he experiences which of the following? A. Tendon pain b. Persistent cough c. Frequent urination d. Constipation

b. Persistent cough Rationale: A persistent cough is an adverse effect of ACE inhibitors. The client should report this finding to the provider and discontinue the medication.

During physical assessment of a patient with thrombocytopenia, what would the nurse expect to find? a. Sternal tenderness b. Petechiae and purpura c. Jaundiced sclera and skin d. Tender enlarged lymph nodes

b. Petechiae and purpura Petechiae are small, flat, red, or reddish-brown pinpoint microhemorrhages that occur on the skin when platelet levels are low. When petechiae are numerous, they group, causing reddish bruises known as purpura. Sternal tenderness is associated with leukemias. Jaundice occurs when anemias are of a hemolytic origin, resulting in accumulation of bile pigments from RBCs. Enlarged, tender lymph nodes are associated with infection or cancer

The nurse develops a plan of care to prevent aspiration in a high-risk patient. Which nursing action will be most effective? a. Turn and reposition immobile patients at least every 2 hours. b. Place patients with altered consciousness in side-lying positions. c. Monitor for respiratory symptoms in patients who are immunosuppressed. d. Insert nasogastric tube for feedings for patients with swallowing problems.

b. Place patients with altered consciousness in side-lying positions. The risk for aspiration is decreased when patients with a decreased level of consciousness are placed in a side- lying or upright position. Frequent turning prevents pooling of secretions in immobilized patients but will not decrease the risk for aspiration in patients at risk. Monitoring of parameters such as breath sounds and oxygen saturation will help detect pneumonia in immunocompromised patients, but it will not decrease the risk for aspiration. Conditions that increase the risk of aspiration include decreased level of consciousness (e.g., seizure, anesthesia, head injury, stroke, alcohol intake), difficulty swallowing, and nasogastric intubation with or without tube feeding. With loss of consciousness, the gag and cough reflexes are depressed, and aspiration is more likely to occur. Other high-risk groups are those who are seriously ill, have poor dentition, or are receiving acid-reducing medications.

While in the recovery room, a patient with a total laryngectomy is suctioned and has bloody mucus with some clots. Which nursing interventions would apply? (select all that apply) a. Notify the health care provider at once. b. Place the patient in semi-Fowler's position. c. Use a bag-valve-mask (BVM) and begin rescue breathing for the patient d. Instill 10 mL of normal saline into the tracheostomy tube to loosen secretions. e. Continue patient assessment, including O2 saturation, respiratory rate, and breath sounds.

b. Place the patient in semi-Fowler's position. e. Continue patient assessment, including O2 saturation, respiratory rate, and breath sounds.

A transesophageal echocardiogram (TEE) is ordered for a patient with possible endocarditis. Which action included in the standard TEE orders will the nurse need to accomplish first? a. Start an IV line. b. Place the patient on NPO status. c. Administer O2 per nasal cannula. d. Give lorazepam (Ativan) 1 mg IV.

b. Place the patient on NPO status. The patient will need to be NPO for 6 hours preceding the TEE, so the nurse should place the patient on NPO status as soon as the order is received. The other actions also will need to be accomplished but not until just before or during the procedure.

The nurse and unlicensed assistive personnel (UAP) on the telemetry unit are caring for four patients. Which nursing action can be delegated to the UAP? a. Teaching a patient scheduled for exercise electrocardiography about the procedure b. Placing electrodes in the correct position for a patient who is to receive ECG monitoring c. Checking the catheter insertion site for a patient who is recovering from a coronary angiogram d. Monitoring a patient who has just returned to the unit after a transesophageal echocardiogram

b. Placing electrodes in the correct position for a patient who is to receive ECG monitoring UAP can be educated in standardized lead placement for ECG monitoring. Assessment of patients who have had procedures where airway maintenance (transesophageal echocardiography) or bleeding (coronary angiogram) is a concern must be done by the registered nurse (RN). Patient teaching requires RN level education and scope of practice.

A patient with hemophilia is hospitalized with acute knee pain and swelling. What is an appropriate nursing intervention for the patient? a. Wrapping the knee with an elastic bandage b. Placing the patient on bed rest and applying ice to the joint c. Administering nonsteroidal antiinflammatory drugs (NSAIDs) as needed for pain d. Gently performing range-of-motion (ROM) exercises to the knee to prevent adhesions

b. Placing the patient on bed rest and applying ice to the joint During an acute bleeding episode in a joint, it is important to rest the involved joint totally and slow bleeding with application of ice. Drugs that decrease platelet aggregation, such as aspirin or nonsteroidal antiinflammatory drugs (NSAIDs), should not be used for pain. As soon as bleeding stops, mobilization of the affected area is encouraged with range-of-motion (ROM) exercises and physical therapy.

After assisting at the bedside with a thoracentesis, the nurse should continue to assess the patient for signs and symptoms of what? a. Bronchospasm b. Pneumothorax c. Pulmonary edema d. Respiratory acidosis

b. Pneumothorax Because thoracentesis involves the introduction of a catheter into the pleural space, there is a risk of pneumothorax. Thoracentesis does not carry a significant potential for causing bronchospasm, pulmonary edema, or respiratory acidosis.

Which subjective data related to the cardiovascular system should be obtained from the patient (select all that apply)? a. Annual income b. Smoking history c. Religious preference d. Number of pillows used to sleep e. Blood for basic laboratory studies

b. Smoking history c. Religious preference d. Number of pillows used to sleep

The patient with acute pericarditis is having a pericardiocentesis. What complication should the nurse monitor the patient for after surgery? a. Pneumonia b. Pneumothorax c. Myocardial infarction (MI) d. Cerebrovascular accident (CVA)

b. Pneumothorax Pneumothorax may occur as a needle is inserted into the pericardial space to remove fluid for analysis and relieve cardiac pressure with pericardiocentesis. Other complications could include dysrhythmias, further cardiac tamponade, pneumomediastinum myocardial laceration, and coronary artery laceration. Pneumonia, myocardial infarction (MI), and cerebrovascular accident (CVA) would not be expected

To what was the resurgence in tuberculosis (TB) resulting from the emergence of multidrug-resistant (MDR) strains of Mycobacterium tuberculosis related? a. A lack of effective means to diagnose TB b. Poor compliance with drug therapy in patients with TB c. Indiscriminate use of antitubercular drugs in treatment of other infections d. Increased population of immunosuppressed persons with acquired immunodeficiency syndrome (AIDS)

b. Poor compliance with drug therapy in patients with TB Drug-resistant strains of tuberculosis (TB) have developed because TB patients' compliance with drug therapy has been poor, and there has been general decreased vigilance in monitoring and follow-up of TB treatment. TB can be diagnosed effectively with sputum cultures. Antitubercular drugs are almost exclusively used for TB infections. The incidence of TB is at epidemic proportions in patients with HIV, but this does not account for multidrug-resistant strains of TB.

Priority Decision: A patient with chronic HF is treated with hydrochlorothiazide, digoxin, and lisinopril. To prevent the risk of digitalis toxicity with these drugs, what is most important that the nurse monitor for this patient? a. HR b. Potassium levels c. BP d. Gastrointestinal function

b. Potassium levels Hypokalemia is one of the most common causes of digitalis toxicity because low serum potassium levels enhance ectopic pacemaker activity. When a patient is receiving potassium-excreting diuretics, such as hydrochlorothiazide or furosemide, it is essential to monitor the patient's serum potassium levels to prevent digitalis toxicity. Monitoring the HR assesses for complications related to digoxin but does not prevent toxicity.

Which collaborative problem will the nurse include in a care plan for a patient admitted to the hospital with idiopathic aplastic anemia? a. Potential complication: seizures b. Potential complication: infection c. Potential complication: neurogenic shock d. Potential complication: pulmonary edema

b. Potential complication: infection Because the patient with aplastic anemia has pancytopenia, the patient is at risk for infection and bleeding. There is no increased risk for seizures, neurogenic shock, or pulmonary edema.

Which rhythm abnormality has an increased risk of ventricular tachycardia and ventricular fibrillation? a. PAC b. Premature ventricular contraction (PVC) on the T wave c. Accelerated idioventricular rhythm d. PVC couplet

b. Premature ventricular contraction (PVC) on the T wave When premature ventricular contraction (PVC) falls on the T wave of the preceding beat, R-on-T phenomenon occurs. Because the ventricle is repolarizing and there is increased excitability of cardiac cells, there is an increased risk of ventricular tachycardia or ventricular fibrillation. The other options do not increase this risk.

Which of the following descriptions matches the mechanism of "oncotic pressure"? a. Force determined by osmolality of a fluid b. Pressure exerted by plasma proteins c. Force exerted by a fluid d. Uses a protein carrier molecule

b. Pressure exerted by plasma proteins

In providing care for a patient hospitalized with an acute exacerbation of polycythemia vera, the nurse gives priority to which activity? a. Maintaining protective isolation b. Promoting leg exercises and ambulation c. Protecting the patient from injury or falls d. Promoting hydration with a large oral fluid intake

b. Promoting leg exercises and ambulation Active or passive leg exercises and ambulation should be implemented to prevent thrombus formation. Thrombus and embolization are the major complications of polycythemia vera because of hypervolemia and hyperviscosity. Isolation is not needed and falls are not expected. Hydration therapy is important to decrease blood viscosity. However, because the patient already has hypervolemia, a careful balance of intake and output must be maintained and fluids are not increased injudiciously.

What advice about sexual activity should the nurse give to a male patient who has had an MI? a. The patient should use the superior position. b. Prophylactic NTG may be used if angina occurs. c. Foreplay may cause too great an increase in HR. d. Performance can be enhanced with the use of sildenafil (Viagra).

b. Prophylactic NTG may be used if angina occurs. It is common for a patient who has chest pain on exertion to have some angina during sexual stimulation or intercourse. Teach the patient to use NTG prophylactically. Positions during intercourse are a matter of individual choice, and foreplay is desirable because it allows a gradual increase in HR. Sildenafil (Viagra) should be used cautiously in men with CAD and should not be used with nitrates.

Which finding by the nurse for a patient with a nursing diagnosis of impaired gas exchange will be most useful in evaluating the effectiveness of treatment? a. Even, unlabored respirations b. Pulse oximetry reading of 92% c. Respiratory rate of 18 breaths/minute d. Absence of wheezes, rhonchi, or crackles

b. Pulse oximetry reading of 92% For the nursing diagnosis of impaired gas exchange, the best data for evaluation are arterial blood gases (ABGs) or pulse oximetry. The other data may indicate either improvement or impending respiratory failure caused by fatigue.

An asymptomatic patient admitted to a cardiac floor has this rhythm. Which should the nurse do FIRST? a. Defibrillate at 200 joules. b. Put on transcutaneous pacer. c. Give Epinephrine. d. Give Atropine.

b. Put on transcutaneous pacer.

A patient with hypertension who has just started taking atenolol (Tenormin) returns to the health clinic after 2 weeks for a follow-up visit. The blood pressure (BP) is unchanged from the previous visit. Which action should the nurse take first? a. Inform the patient about the reasons for a possible change in drug dosage. b. Question the patient about whether the medication is actually being taken. c. Inform the patient that multiple drugs are often needed to treat hypertension. d. Question the patient regarding any lifestyle changes made to help control BP.

b. Question the patient about whether the medication is actually being taken. Because noncompliance with antihypertensive therapy is common, the nurses initial action should be to determine whether the patient is taking the atenolol as prescribed. The other actions also may be implemented, but these would be done after assessing patient compliance with the prescribed therapy.

When obtaining a nursing history for a patient with myocarditis, what should the nurse specifically question the patient about? a. Prior use of digoxin for treatment of cardiac problems b. Recent symptoms of a viral illness, such as fever and malaise c. A history of coronary artery disease (CAD) with or without an MI d. A recent streptococcal infection requiring treatment with penicillin

b. Recent symptoms of a viral illness, such as fever and malaise Viruses are the most common cause of myocarditis in the United States, and early manifestations of myocarditis are often those of systemic viral infections. Myocarditis may also be associated with autoimmune disorders as well as with other microorganisms, drugs, or toxins. The patient with myocarditis is predisposed to drug-related dysrhythmias and toxicity with digoxin, so it is used very cautiously, if at all, in treatment of the condition, but digoxin does not lead to myocarditis. A streptococcal infection is more likely to lead to rheumatic fever.

During care of a patient with a cuffed tracheostomy, the nurse notes that the tracheostomy tube has an inner cannula. To care for the tracheostomy appropriately, what should the nurse do? a. Deflate the cuff, then remove and suction the inner cannula. b. Remove the inner cannula and replace it per institutional guidelines. c. Remove the inner cannula if the patient shows signs of airway obstruction. d. Keep the inner cannula in place at all times to prevent dislodging the tracheostomy tube.

b. Remove the inner cannula and replace it per institutional guidelines. An inner cannula is a second tubing that fits inside the outer tracheostomy tube. Disposable inner cannulas are frequently used, but nondisposable ones can be removed and cleaned of mucus that has accumulated on the inside of the tube. Many tracheostomy tubes do not have inner cannulas because when humidification is adequate, accumulation of mucus should not occur. Cuff deflation is no longer recommended. When signs of airway obstruction occur, suction is needed

What is characteristic of a partial rebreather mask? a. Used for long-term O2 therapy b. Reservoir bag conserves oxygen c. Provides highest oxygen concentrations d. Most comfortable and causes the least restriction on activities

b. Reservoir bag conserves oxygen The partial rebreather mask has O2 flow into the reservoir bag and mask during inhalation. The O2 -conserving cannula is used for long-term therapy at home versus during hospitalization. The Venturi mask can deliver the highest concentrations of O2 . The nasal cannula is the most comfortable and mobile delivery device.

The patient is admitted postradiation therapy with symptoms of cardiomyopathy (CMP). Which type of CMP should the nurse suspect that the patient is experiencing? a. Dilated b. Restrictive c. Takotsubo d. Hypertrophic

b. Restrictive A secondary cause of restrictive cardiomyopathy (CMP) is radiation treatment to the thorax with stiffness of the ventricular wall occurring. Dilated CMP may have a genetic link, follow infectious myocarditis, or be related to an autoimmune process or excess alcohol ingestion. Takotsubo CMP is an acute stress-related syndrome that mimics acute coronary syndrome. It is most common in postmenopausal women. Hypertrophic CMP has a genetic link in about one-half of all cases and is often seen in young athletic individuals.

A P wave on an ECG represents an impulse arising at the a. SA node and repolarizing the atria. b. SA node and depolarizing the atria. c. AV node and depolarizing the atria. d. AV node and spreading to the bundle of His.

b. SA node and depolarizing the atria.

A patient suddenly develops severe hypertension from cirrhosis, what type of hypertension do they have? a. Primary hypertension b. Secondary hypertension c. Idiopathic hypertension d. Essential hypertension

b. Secondary hypertension

Which clinical manifestations can the nurse expect to see in both patients with Buerger's disease and patients with Raynaud's phenomenon? (select all that apply) a. Intermittent low-grade fevers b. Sensitivity to cold temperatures c. Gangrenous ulcers on fingertips d. Color changes of fingers and toes e. Episodes of superficial vein thrombosis

b. Sensitivity to cold temperatures c. Gangrenous ulcers on fingertips d. Color changes of fingers and toes

The nurse is reviewing a patient's morning laboratory results. Which result is of greatest concern? a. Serum Na + of 150 mEq/L b. Serum Mg 2 + of 1.1 mEq/L c. Serum PO4 3 − of 4.5 mg/dL d. Serum Ca 2 + (total) of 8.6 mg/dL

b. Serum Mg 2 + of 1.1 mEq/L With a low magnesium level there is an increased risk for hypokalemia and hypocalcemia as well as altered sodium-potassium pump and altered carbohydrate and protein metabolism. Hypokalemia could lead to dysrhythmias and severe muscle weakness. The sodium and phosphate levels are also not within normal limits. However, the implications are not as life-threatening. The calcium level is normal.

A patient is admitted to the emergency department with severe fatigue and confusion. Laboratory studies are done. Which laboratory value will require the most immediate action by the nurse? a. Arterial blood pH is 7.32. b. Serum calcium is 18 mg/dL. c. Serum potassium is 5.1 mEq/L. d. Arterial oxygen saturation is 91%.

b. Serum calcium is 18 mg/dL. The serum calcium is well above the normal level and puts the patient at risk for cardiac dysrhythmias. The nurse should initiate cardiac monitoring and notify the health care provider. The potassium, oxygen saturation, and pH are also abnormal, and the nurse should notify the health care provider about these values as well, but they are not immediately life threatening.

A nurse is evaluating a client who is receiving IV fluids to treat isotonic dehydration. Which of the following laboratory findings indicates that the fluid therapy has been effective? a. BUN 26 mg/dL b. Serum sodium 142 mEq/L c. Hct 56% d. Urine specific gravity 1.035

b. Serum sodium 142 mEq/L Isotonic dehydration includes loss of water and electrolytes due to a decrease in oral intake of water and salt. A serum sodium level of 142 mEq/L is within the expected reference range and indicates that the fluid therapy has been effective.BUN is elevated. HCT is elevated and USG is elevated.

A patient who is recovering from an acute myocardial infarction (AMI) asks the nurse about when sexual intercourse can be resumed. Which response by the nurse is best? a. Most patients are able to enjoy intercourse without any complications. b. Sexual activity uses about as much energy as climbing two flights of stairs. c. The doctor will provide sexual guidelines when your heart is strong enough. d. Holding and cuddling are good ways to maintain intimacy after a heart attack.

b. Sexual activity uses about as much energy as climbing two flights of stairs. Sexual activity places about as much physical stress on the cardiovascular system as most moderate-energy activities such as climbing two flights of stairs. The other responses do not directly address the patients question or may not be accurate for this patient.

A 62-yr-old Hispanic male patient with diabetes mellitus has been diagnosed with peripheral artery disease (PAD). The patient is a smoker with a history of gout. To prevent complications, which factor is priority in patient teaching? a Gender b Smoking c Ethnicity d Comorbidities

b. Smoking is the most significant factor for this patient. PAD is a marker of advanced systemic atherosclerosis. Therefore, tobacco cessation is essential to reduce PAD progression, CVD events, and mortality. Diabetes mellitus and hyperuricemia are also risk factors. Being male or Hispanic are not risk factors for PAD.

After admitting a patient from home to the medical unit with a diagnosis of pneumonia, which provider orders must the nurse verify have been completed before administering a dose of cefuroxime? a. Orthostatic blood pressures b. Sputum culture and sensitivity c. Pulmonary function evaluation d. Serum laboratory studies ordered for AM

b. Sputum culture and sensitivity The nurse should ensure that the sputum for culture and sensitivity was sent to the laboratory before administering the cefuroxime because this is community-acquired pneumonia. It is important that the organisms are correctly identified (by the culture) before the antibiotic takes effect. The test will also determine whether the proper antibiotic has been ordered (sensitivity testing). Although antibiotic administration should not be unduly delayed while waiting for the patient to expectorate sputum, orthostatic blood pressures, pulmonary function evaluation, and serum laboratory tests will not be affected by the administration of antibiotics.

A client has driven himself to the ER. He is 51 yo has a history of hypertension and informs the nurse that his father died of a heart attack at 60. The client is presently complaining of indigestion. The nurse connects him to an ecg monitor and begins administering oxygen at 2 l/minute. The nurse next action would be to: a. Call the doctor b. Start an IV line c. Provide chest xray d. Draw blood for lab studies.

b. Start an IV line Advanced cardiac life support recommends that at least one or two intravenous lines be inserted in one or both of the antecubital spaces.

A patient whose heart monitor shows sinus tachycardia, rate 132, is apneic and has no palpable pulses. What is the first action that the nurse should take? a. Perform synchronized cardioversion. b. Start cardiopulmonary resuscitation (CPR). c. Administer atropine per agency dysrhythmia protocol. d. Provide supplemental oxygen via non-rebreather mask

b. Start cardiopulmonary resuscitation (CPR). The patients clinical manifestations indicate pulseless electrical activity and the nurse should immediately start CPR. The other actions would not be of benefit to this patient.

Priority Decision: To decrease the patient's sense of panic during an acute asthma attack, what is the best action for the nurse to do? a. Leave the patient alone to rest in a quiet, calm environment. b. Stay with the patient and encourage slow, pursed-lip breathing. c. Reassure the patient that the attack can be controlled with treatment. d. Let the patient know that frequent monitoring is being done by measuring vital signs and arterial oxygen saturation by pulse oximetry (SpO2 ).

b. Stay with the patient and encourage slow, pursed-lip breathing. The patient in an acute asthma attack is very anxious and fearful. It is best to stay with the patient and interact in a calm, unhurried manner. Helping the patient breathe with pursed lips will facilitate expiration of trapped air and help the patient gain control of breathing. Pursed-lip breathing is also used with COPD for this same reason. The other options will not decrease the panic of an acute asthma attack.

Which nursing action could the registered nurse (RN) working in a skilled care hospital unit delegate to an experienced licensed practical/vocational nurse (LPN/LVN) caring for a patient with a permanent tracheostomy? a. Assess the patients risk for aspiration. b. Suction the tracheostomy when needed. c. Teach the patient about self-care of the tracheostomy. d. Determine the need for replacement of the tracheostomy tube.

b. Suction the tracheostomy when needed. Suctioning of a stable patient can be delegated to LPNs/LVNs. Patient assessment and patient teaching should be done by the RN.

Which assessment finding in a patient who is admitted with infective endocarditis (IE) is most important to communicate to the health care provider? a. Generalized muscle aching b. Sudden onset right flank pain c. Janeways lesions on the palms d. Temperature 100.7 F (38.1 C)s.

b. Sudden onset right flank pain Sudden onset of flank pain indicates possible embolization to the kidney and may require diagnostic testing such as a renal arteriogram and interventions to improve renal perfusion. The other findings are typically found in IE, but do not require any new intervention

A patient is admitted to the hospital with a diagnosis of abdominal aortic aneurysm. Which signs and symptoms would suggest that the aneurysm has ruptured? a. Rapid onset of shortness of breath and hemoptysis b. Sudden, severe low back pain and bruising along his flank c. Gradually increasing substernal chest pain and diaphoresis d. Sudden, patchy blue mottling on feet and toes and rest pain

b. Sudden, severe low back pain and bruising along his flank

What keeps alveoli from collapsing? a. Carina b. Surfactant c. Empyema d. Thoracic cag

b. Surfactant Surfactant is a lipoprotein that lowers the surface tension in the alveoli. It reduces the pressure needed to inflate the alveoli and decreases the tendency of the alveoli to collapse. The other options do not maintain inflation of the alveoli. The carina is the point of bifurcation of the trachea into the right and left bronchi. Empyema is a collection of pus in the thoracic cavity. The thoracic cage is formed by the ribs and protects the thoracic organs.

A patient with infective endocarditis of a prosthetic mitral valve develops a left hemiparesis and visual changes. What should the nurse expect to be included in interprofessional care of the patient? a. Embolectomy b. Surgical valve replacement c. Administration of anticoagulants d. Higher-than-usual antibiotic dosages

b. Surgical valve replacement Early valve replacement followed by prolonged antibiotic and anticoagulant therapy is recommended for these patients. Drug therapy for patients who develop endocarditis of prosthetic valves is often unsuccessful in eliminating the infection and preventing embolization.

The health care providers progress note for a patient states that the complete blood count (CBC) shows a shift to the left. Which assessment finding will the nurse expect? a. Cool extremities b. Pallor and weakness c. Elevated temperature d. Low oxygen saturation

c. Elevated temperature The term shift to the left indicates that the number of immature polymorphonuclear neutrophils (bands) is elevated and that finding is a sign of infection. There is no indication that the patient is at risk for hypoxemia, pallor/weakness, or cool extremities.

Which patient teaching points should the nurse include when providing discharge instructions to a patient with a new permanent pacemaker and the caregiver? (select all that apply) a. Avoid or limit air travel. b. Take and record a daily pulse rate. c. Obtain and wear a Medic Alert ID device at all times. d. Avoid lifting arm on the side of the pacemaker above shoulder e. Do not use a microwave oven because it interferes with pacemaker function.

b. Take and record a daily pulse rate. c. Obtain and wear a Medic Alert ID device at all times. d. Avoid lifting arm on the side of the pacemaker above shoulder Correct answers: b, c, d Rationale: Pacemaker discharge teaching should include: Air travel is not restricted. The patient should tell airport security of the presence of a pacemaker because it may set off the metal detector. A hand-held screening wand should not pass directly over the pacemaker. Manufacturer information varies about the effect of metal detectors on pacemaker function. The patient should monitor the pulse and tell the HCP if it drops below a predetermined rate. The patient should have and wear a Medic Alert ID device at all times. The patient must avoid lifting the arm on the pacemaker side above the shoulder until approved by the HCP. Microwave ovens are safe to use. They do not interfere with pacemaker function.

Which patient teaching points should the nurse include when providing discharge instructions to a patient with a new permanent pacemaker and the caregiver? (select all that apply) a. Avoid or limit air travel. b. Take and record a daily pulse rate. c. Obtain and wear a Medic Alert ID device at all times. d. Avoid lifting arm on the side of the pacemaker above shoulder. e. Do not use a microwave oven because it interferes with pacemaker function.

b. Take and record a daily pulse rate. c. Obtain and wear a Medic Alert ID device at all times. d. Avoid lifting arm on the side of the pacemaker above shoulder.

A patient with iron deficiency anemia is to be discharged home with iron replacement therapy. Which instruction would be most important to maximize iron absorption? a. Take iron an hour after meals. b. Take iron with a glass of orange juice. c. Report black stools to the provider immediately. d. Enteric-coated iron tablets can be crushed and mixed with orange juice

b. Take iron with a glass of orange juice. Iron is best absorbed in an acidic environment. The patient should be instructed to take iron an hour before meals with a glass of orange juice. Black, tarry stools are to be expected with iron supplements. Enteric-coated iron tablets should not be crushed or split.

Which problem reported by a patient with hemophilia is most important for the nurse to communicate to the physician? a. Leg bruises b. Tarry stools c. Skin abrasions d. Bleeding gums

b. Tarry stools Melena is a sign of gastrointestinal bleeding and requires collaborative actions such as checking hemoglobin and hematocrit and administration of coagulation factors. The other problems indicate a need for patient teaching about how to avoid injury, but are not indicators of possible serious blood loss.

When developing a community health program to decrease the incidence of rheumatic fever, which action would be most important for the community health nurse to include? a. Vaccinate high-risk groups in the community with streptococcal vaccine. b. Teach community members to seek treatment for streptococcal pharyngitis. c. Teach about the importance of monitoring temperature when sore throats occur. d. Teach about prophylactic antibiotics to those with a family history of rheumatic fever.

b. Teach community members to seek treatment for streptococcal pharyngitis. The incidence of rheumatic fever is decreased by treatment of streptococcal infections with antibiotics. Family history is not a risk factor for rheumatic fever. There is no immunization that is effective in decreasing the incidence of rheumatic fever. Teaching about monitoring temperature will not decrease the incidence of rheumatic fever.

A patient with cystic fibrosis (CF) has blood glucose levels that are consistently between 180 to 250mg/dL. Which nursing action will the nurse plan to implement? a. Discuss the role of diet in blood glucose control. b. Teach the patient about administration of insulin. c. Give oral hypoglycemic medications before meals. d. Evaluate the patients home use of pancreatic enzymes.

b. Teach the patient about administration of insulin. The glucose levels indicate that the patient has developed CF-related diabetes, and insulin therapy is required. Because the etiology of diabetes in CF is inadequate insulin production, oral hypoglycemic agents are not effective. Patients with CF need a high-calorie diet. Inappropriate use of pancreatic enzymes would not be a cause of hyperglycemia in a patient with CF.

A 30-year-old man with acute myelogenous leukemia develops an absolute neutrophil count of 850/L while receiving outpatient chemotherapy. Which action by the outpatient clinic nurse is most appropriate? a. Discuss the need for hospital admission to treat the neutropenia. b. Teach the patient to administer filgrastim (Neupogen) injections. c. Plan to discontinue the chemotherapy until the neutropenia resolves. d. Order a high-efficiency particulate air (HEPA) filter for the patients home.

b. Teach the patient to administer filgrastim (Neupogen) injections. The patient may be taught to self-administer filgrastim injections. Although chemotherapy may be stopped with severe neutropenia (neutrophil count less than 500/L), administration of filgrastim usually allows the chemotherapy to continue. Patients with neutropenia are at higher risk for infection when exposed to other patients in the hospital. HEPA filters are expensive and are used in the hospital, where the number of pathogens is much higher than in the patients home environment.

A patient with chronic obstructive pulmonary disease (COPD) has rhonchi throughout the lung fields and a chronic, nonproductive cough. Which nursing intervention will be most effective? a. Change the oxygen flow rate to the highest prescribed rate. b. Teach the patient to use the Flutter airway clearance device. c. Reinforce the ongoing use of pursed lip breathing techniques. d. Teach the patient about consistent use of inhaled corticosteroids.

b. Teach the patient to use the Flutter airway clearance device. Airway clearance devices assist with moving mucus into larger airways where it can more easily be expectorated. The other actions may be appropriate for some patients with COPD, but they are not indicated for this patients problem of thick mucus secretions.

The nurse notes that a patients cardiac monitor shows that every other beat is earlier than expected, has no visible P wave, and has a QRS complex that is wide and bizarre in shape. How will the nurse document the rhythm? a. Ventricular couplets b. Ventricular bigeminy c. Ventricular R-on-T phenomenon d. Multifocal premature ventricular contractions

b. Ventricular bigeminy Ventricular bigeminy describes a rhythm in which every other QRS complex is wide and bizarre looking. Pairs of wide QRS complexes are described as ventricular couplets. There is no indication that the premature ventricular contractions (PVCs) are multifocal or that the R-on-T phenomenon is occurring.

If a patient with blood type O Rh + is given AB Rh − blood, what would the nurse expect to happen? a. The patient's Rh factor will react with the RBCs of the donor blood. b. The anti-A and anti-B antibodies in the patient's blood will hemolyze the donor blood. c. The anti-A and anti-B antibodies in the donor blood will hemolyze the patient's blood. d. No adverse reaction is expected because the patient has no antibodies against the donor blood.

b. The anti-A and anti-B antibodies in the patient's blood will hemolyze the donor blood. A patient with type O Rh + blood has no A or B antigens on the RBC but does have anti-A and anti-B antibodies in the blood and has an Rh antigen. Type AB Rh − blood has both A and B antigens on the RBC but no Rh antigen and no anti-A or anti-B antibodies. If the type AB Rh − blood is given to the patient with type O Rh + blood, the antibodies in the patient's blood will react with the antigens in the donor blood, causing hemolysis of the donor cells. There will be no Rh reaction because the donor blood has no Rh antigen.

During preoperative preparation of the patient scheduled for an AAA, why should the nurse establish baseline data for the patient? a. All physiologic processes will be changed postoperatively. b. The cause of the aneurysm is a systemic vascular disease. c. Surgery will be canceled if any physiologic function is not normal. d. BP and heart rate (HR) will be maintained well below baseline levels during the postoperative period.

b. The cause of the aneurysm is a systemic vascular disease. Because atherosclerosis is a systemic disease, the patient with an AAA is likely to have cardiac, pulmonary, cerebral, or lower extremity vascular problems that should be noted and monitored throughout the perioperative period. Postoperatively, the BP is balanced: high enough to keep adequate flow through the artery to prevent thrombosis but low enough to prevent bleeding at the surgical site.

A 53-year-old patient with Stage D heart failure and type 2 diabetes asks the nurse whether heart transplant is a possible therapy. Which response by the nurse is most appropriate? a. Because you have diabetes, you would not be a candidate for a heart transplant. b. The choice of a patient for a heart transplant depends on many different factors. c. Your heart failure has not reached the stage in which heart transplants are needed. d. People who have heart transplants are at risk for multiple complications after surgery.

b. The choice of a patient for a heart transplant depends on many different factors. Indications for a heart transplant include end-stage heart failure (Stage D), but other factors such as coping skills, family support, and patient motivation to follow the rigorous posttransplant regimen are also considered. Diabetic patients who have well-controlled blood glucose levels may be candidates for heart transplant. Although heart transplants can be associated with many complications, this response does not address the patients question.

Which client is most likely to receive erythropoietin? a. The client with congestive heart failure. b. The client with chronic kidney disease. c. The client with liver failure. d. The asplenic client.

b. The client with chronic kidney disease. This answer is correct since erythropoietin is a hormone produced by the kidneys. If the kidney is injured, red blood cell production will be low. This will result in low hemoglobin and hematocrit.

8. The patient who is being admitted has had a history of uncontrolled hypertension. High SVR is most likely to cause damage to which organ? a. Brain b. Heart c. Retina d. Kidney

b. The increased systemic vascular resistance (SVR) of hypertension directly increases the workload of the heart, and heart failure occurs when the heart can no longer pump effectively against the increased resistance. The heart may be indirectly damaged by atherosclerotic changes in the blood vessels, as are the brain, retina, and kidney.

The nurse is caring for a newly admitted patient with vascular insufficiency. The patient has a new order for enoxaparin (Lovenox) 30 mg subcutaneously. What should the nurse do to correctly administer this medication? a Spread the skin before inserting the needle. b Leave the air bubble in the prefilled syringe. c Use the back of the arm as the preferred site. d Sit the patient at a 30-degree angle before administration.

b. The nurse should not expel the air bubble from the prefilled syringe because it should be injected to clear the needle of medication and avoid leaving medication in the needle track in the tissue.

Which assessment data collected by the nurse who is admitting a patient with chest pain suggest that the pain is caused by an acute myocardial infarction (AMI)? a. The pain increases with deep breathing. b. The pain has lasted longer than 30 minutes. c. The pain is relieved after the patient takes nitroglycerin. d. The pain is reproducible when the patient raises the arms.

b. The pain has lasted longer than 30 minutes. Chest pain that lasts for 20 minutes or more is characteristic of AMI. Changes in pain that occur with raising the arms or with deep breathing are more typical of musculoskeletal pain or pericarditis. Stable angina is usually relieved when the patient takes nitroglycerin.

When assessing a pregnant patient with eclampsia who is receiving IV magnesium sulfate, which finding should the nurse report to the health care provider immediately? a. The bibasilar breath sounds are decreased. b. The patellar and triceps reflexes are absent. c. The patient has been sleeping most of the day. d. The patient reports feeling sick to my stomach.

b. The patellar and triceps reflexes are absent. The loss of the deep tendon reflexes indicates that the patients magnesium level may be reaching toxic levels. Nausea and lethargy also are side effects associated with magnesium elevation and should be reported, but they are not as significant as the loss of deep tendon reflexes. The decreased breath sounds suggest that the patient needs to cough and deep breathe to prevent atelectasis.

The nurse is assessing a patient who has been admitted to the intensive care unit (ICU) with a hypertensive emergency. Which finding is most important to report to the health care provider? a. Urine output over 8 hours is 250 mL less than the fluid intake. b. The patient cannot move the left arm and leg when asked to do so. c. Tremors are noted in the fingers when the patient extends the arms. d. The patient complains of a headache with pain at level 7 of 10 (0 to 10 scale).

b. The patient cannot move the left arm and leg when asked to do so. The patient's inability to move the left arm and leg indicates that a stroke may be occurring and will require immediate action to prevent further neurologic damage. The other clinical manifestations are also likely caused by the hypertension and will require rapid nursing actions, but they do not require action as urgently as the neurologic changes.

The nurse is assessing a patient who has been admitted to the intensive care unit (ICU) with a hypertensive emergency. Which finding is most important to report to the health care provider? a. Urine output over 8 hours is 250 mL less than the fluid intake. b. The patient cannot move the left arm and leg when asked to do so. c. Tremors are noted in the fingers when the patient extends the arms. d. The patient complains of a headache with pain at level 8/10 (0 to 10 scale).

b. The patient cannot move the left arm and leg when asked to do so. The patients inability to move the left arm and leg indicates that a hemorrhagic stroke may be occurring and will require immediate action to prevent further neurologic damage. The other clinical manifestations are also likely caused by the hypertension and will require rapid nursing actions, but they do not require action as urgently as the neurologic changes.

When admitting a patient for a cardiac catheterization and coronary angiogram, which information about the patient is most important for the nurse to communicate to the health care provider? a. The patients pedal pulses are +1. b. The patient is allergic to shellfish. c. The patient had a heart attack a year ago. d. The patient has not eaten anything today.

b. The patient is allergic to shellfish. The contrast dye used for the procedure is iodine based, so patients who have shellfish allergies will require treatment with medications such as corticosteroids and antihistamines before the angiogram. The other information is also communicated to the health care provider but will not require a change in the usual precardiac catheterization orders or medications.

Which information obtained by the nurse caring for a patient with thrombocytopenia should be immediately communicated to the health care provider? a. The platelet count is 52,000/L. b. The patient is difficult to arouse. c. There are purpura on the oral mucosa. d. There are large bruises on the patients back.

b. The patient is difficult to arouse. Difficulty in arousing the patient may indicate a cerebral hemorrhage, which is life threatening and requires immediate action. The other information should be documented and reported but would not be unusual in a patient with thrombocytopenia.

The nurse teaches a patient about pursed lip breathing. Which action by the patient would indicate to the nurse that further teaching is needed? a. The patient inhales slowly through the nose. b. The patient puffs up the cheeks while exhaling. c. The patient practices by blowing through a straw. d. The patients ratio of inhalation to exhalation is 1:3.

b. The patient puffs up the cheeks while exhaling. The patient should relax the facial muscles without puffing the cheeks while doing pursed lip breathing. The other actions by the patient indicate a good understanding of pursed lip breathing.

Which patient requires the most rapid assessment and care by the emergency department nurse? a. The patient with hemochromatosis who reports abdominal pain b. The patient with neutropenia who has a temperature of 101.8 F c. The patient with sickle cell anemia who has had nausea and diarrhea for 24 hours d. The patient with thrombocytopenia who has oozing after having a tooth extracted

b. The patient with neutropenia who has a temperature of 101.8 F A neutropenic patient with a fever is assumed to have an infection and is at risk for rapidly developing sepsis. Rapid assessment, cultures, and initiation of antibiotic therapy are needed. The other patients also require rapid assessment and care but not as urgently as the neutropenic patient.

A patient with varicose veins has been prescribed compression stockings. Which nursing instruction would be most appropriate? a "Try to keep your stockings on 24 hours a day, as much as possible." b "While you're still lying in bed in the morning, put on your stockings." c "Dangle your feet at your bedside for 5 minutes before putting on your stockings." d "Your stockings will be most effective if you can remove them for a few minutes several times a day."

b. The patient with varicose veins should apply stockings in bed before rising in the morning. Stockings should not be worn continuously and should not be removed several times daily. Dangling at the bedside before application is likely to decrease their effectiveness.

A patient was just diagnosed with acute arterial ischemia in the left leg secondary to atrial fibrillation. Which early clinical manifestation must be reported to the physician immediately to save the patient's limb? a Paralysis b Paresthesia c Cramping d Referred pain

b. The physician must be notified immediately if any of the six Ps of acute arterial ischemia occur to prevent ischemia from quickly progressing to tissue necrosis and gangrene. The six Ps are paresthesia, pain, pallor, pulselessness, and poikilothermia, with paralysis being a very late sign indicating the death of nerves to the extremity. Crampy leg sensation is more common with varicose veins. The pain is not referred.

The patient had aortic aneurysm repair 6 hours ago. What priority nursing action will maintain graft patency? a Assess output for renal dysfunction. b Use IV fluids to maintain adequate BP. c Use oral antihypertensives to maintain cardiac output. d Maintain a low BP to prevent pressure on surgical site.

b. The priority is to maintain an adequate blood pressure (BP) (determined by the surgeon) to maintain graft patency. A prolonged low BP may result in graft thrombosis, and hypertension may cause undue stress on arterial anastomoses resulting in leakage of blood or rupture at the suture lines, which is when IV antihypertensives may be used. Renal output will be assessed when the aneurysm repair is above the renal arteries to assess graft patency, not maintain it.

A 68-year-old woman with acute myelogenous leukemia (AML) asks the nurse whether the planned chemotherapy will be worth undergoing. Which response by the nurse is appropriate? a. If you do not want to have chemotherapy, other treatment options include stem cell transplantation. b. The side effects of chemotherapy are difficult, but AML frequently goes into remission with chemotherapy. c. The decision about treatment is one that you and the doctor need to make rather than asking what I would do. d. You dont need to make a decision about treatment right now because leukemias in adults tend to progress quite slowly.

b. The side effects of chemotherapy are difficult, but AML frequently goes into remission with This response uses therapeutic communication by addressing the patients question and giving accurate information. The other responses either give inaccurate information or fail to address the patients question, which will discourage the patient from asking the nurse for information.

What is the antidote to warfarin? a. Protamine sulfate b. Vitamin K c. Acetylcysteine d. Digoxin immune fab

b. Vitamin K This answer is correct because warfarin is a vitamin K antagonist. This makes vitamin K the antidote to warfarin. Vitamin K produces prothrombin, which is a protein that clots blood.

When does the nurse record the presence of an increased anteroposterior (AP) diameter of the chest? a. There is a prominent protrusion of the sternum. b. The width of the chest is equal to the depth of the chest. c. There is equal but diminished movement of the 2 sides of the chest. d. The patient cannot fully expand the lungs because of kyphosis of the spine.

b. The width of the chest is equal to the depth of the chest. An increased anterior-posterior (AP) diameter is characteristic of a barrel chest, in which the AP diameter is about equal to the side-to-side diameter. Normally the AP diameter should be ⅓ to ½ the side-to-side diameter. A prominent protrusion of the sternum is the pectus carinatum and diminished movement of both sides of the chest indicates decreased chest excursion. Lack of lung expansion caused by kyphosis of the spine results in shallow breathing with decreased chest expansion.

A patient with an acute myocardial infarction (MI) develops the following ECG pattern: atrial rate of 82 bpm and regular; ventricular rate of 46 bpm and regular; P wave and QRS complex are normal but there is no relationship between the P wave and the QRS complex. What dysrhythmia does the nurse identify this as and what treatment is expected? a. Sinus bradycardia treated with atropine b. Third-degree heart block treated with a pacemaker c. Atrial fibrillation treated with electrical cardioversion d. Type I second-degree AV block treated with observation

b. Third-degree heart block treated with a pacemaker Third-degree or complete heart block is recognized with the atrial and ventricular dissociation and treated with a pacemaker. Sinus bradycardia does not have atrial and ventricular dissociation. Atrial fibrillation does not have normal P waves, as they are stimulated by ectopic foci. In type I second-degree AV heart block, the PR interval gradually lengthens and a QRS complex is dropped. Then the cycle begins again

A patient with critical limb ischemia had peripheral artery bypass surgery to improve circulation. What nursing care should be provided on postoperative day 1? a Keep patient on bed rest. b Assist patient to walk several times. c Have patient sit in the chair several times. d Place patient on their side with knees flexed.

b. To avoid blockage of the graft or stent, the patient should walk several times on postoperative day 1 and subsequent days. Having the patient's knees flexed for sitting in a chair or in bed increase the risk of venous thrombosis and may place stress on the suture lines.

A 50-year-old woman who weighs 95 kg has a history of tobacco use, high blood pressure, high sodium intake, and sedentary lifestyle. Which is the most important risk factor for peripheral artery disease (PAD) to address in the nursing plan of care? a. Salt intake b. Tobacco use c. Excess weight d. Sedentary lifestyle

b. Tobacco use

A patient with a history of a transfusion-related acute lung injury (TRALI) is to receive a transfusion of packed red blood cells (PRBCs). Which action by the nurse will decrease the risk for TRALI for this patient? a. Infuse the PRBCs slowly over 4 hours. b. Transfuse only leukocyte-reduced PRBCs. c. Administer the scheduled diuretic before the transfusion. d. Give the PRN dose of antihistamine before the transfusion.

b. Transfuse only leukocyte-reduced PRBCs. TRALI is caused by a reaction between the donor and the patient leukocytes that causes pulmonary inflammation and capillary leaking. The other actions may help prevent respiratory problems caused by circulatory overload or by allergic reactions, but they will not prevent TRAL.

The nurse identifies the nursing diagnosis of decreased cardiac output related to valvular insufficiency for the patient with infective endocarditis (IE) based on which assessment finding(s)? a. Fever, chills, and diaphoresis b. Urine output less than 30 mL/hr c. Petechiae on the inside of the mouth and conjunctiva d. Increase in heart rate of 15 beats/minute with walking

b. Urine output less than 30 mL/hr Decreased renal perfusion caused by inadequate cardiac output will lead to decreased urine output. Petechiae, fever, chills, and diaphoresis are symptoms of IE, but are not caused by decreased cardiac output. An increase in pulse rate of 15 beats/minute is normal with exercise.

Which instruction should the nurse include in an exercise teaching plan for a patient with chronic obstructive pulmonary disease (COPD)? a. Stop exercising if you start to feel short of breath. b. Use the bronchodilator before you start to exercise. c. Breathe in and out through the mouth while you exercise. d. Upper body exercise should be avoided to prevent dyspnea.

b. Use the bronchodilator before you start to exercise. Use of a bronchodilator before exercise improves airflow for some patients and is recommended. Shortness of breath is normal with exercise and not a reason to stop. Patients should be taught to breathe in through the nose and out through the mouth (using a pursed lip technique). Upper-body exercise can improve the mechanics of breathing in patients with COPD.

The nurse is caring for a patient who has a central venous access device (CVAD). Which action by the nurse is appropriate? a. Avoid using friction when cleaning around the CVAD insertion site. b. Use the push-pause method to flush the CVAD after giving medications. c. Obtain an order from the health care provider to change CVAD dressing. d. Position the patients face toward the CVAD during injection cap changes.

b. Use the push-pause method to flush the CVAD after giving medications. The push-pause enhances the removal of debris from the CVAD lumen and decreases the risk for clotting. To decrease infection risk, friction should be used when cleaning the CVAD insertion site. The dressing should be changed whenever it becomes damp, loose, or visibly soiled. The patient should turn away from the CVAD during cap changes.

A nurse is watching a client's ECG monitor and notes that the client's rhythm has changed from a normal sinus rhythm to supraventricular tachycardia. The client is conscious with a HR of 200-210 bpm and has a faint radial pulse. The nurse should anticipate assisting with which of the following interventions? A. Initiate chest compressions b. Vagal stimulation c. Administration of atropine IV d. Defibrillation

b. Vagal stimulation Rationale: The nurse should identify that vagal stimulation might temporarily convert the client's heart rate to normal sinus rhythm. The nurse should have a defibrillator and resuscitation equipment at the client's bedside because vagal stimulation can cause bradydysrhythmias, ventricular dysrhythmias, or asystole.

A nurse is assessing a client who has left-sided HF. Which of the following manifestations should the nurse expect to find? A. Inc abdominal girth b. Weak peripheral pulses c. Jugular vein distention d. Dependent edema

b. Weak peripheral pulses Rationale: Weak peripheral pulses are related to decreased cardiac output resulting from left-sided heart failure.

A patient with PAD has a nursing diagnosis of ineffective tissue perfusion. What should be included in the teaching plan for this patient (select all that apply)? a. Apply cold compresses when the legs become swollen. b. Wear protective footwear and avoid hot or cold extremes. c. Walk at least 30 minutes per day, at least 3 times per week. d. Use nicotine replacement therapy as a substitute for smoking. e. Inspect lower extremities for pulses, temperature, and any injury.

b. Wear protective footwear and avoid hot or cold extremes. c. Walk at least 30 minutes per day, at least 3 times per week. e. Inspect lower extremities for pulses, temperature, and any injury. Protecting feet and legs from injury is important. Walking exercise increases oxygen extraction in the legs and improves skeletal muscle metabolism. The patient with PAD should walk at least 30 minutes a day, at least 3 times per week. Exercise should be stopped when pain occurs and resumed when the pain subsides. The lower extremities should be assessed at regular intervals for changes. Cold compresses and nicotine in all forms causes vasoconstriction and should be avoided.

A nurse in an emergency department is caring for a child who is experiencing an acute asthma attack. Which of the following assessments indicates that the respiratory status is declining? (Select all that apply). a. SaO2 95% b. Wheezing c. Retraction of sternal muscles d. Pink mucous membranes c. Tachycardia e. Premature Ventricular Complexes (PVCs)

b. Wheezing c. Retraction of sternal muscles c. Tachycardia e. Premature Ventricular Complexes (PVCs)

A patient with sickle cell anemia asks the nurse why the sickling crisis does not stop when oxygen therapy is started. Which explanation should the nurse give to the patient? a. Sickling occurs in response to decreased blood viscosity, which is not affected by oxygen therapy. b. When RBCs sickle, they occlude small vessels, which causes more local hypoxia and more sickling. c. The primary problem during a sickle cell crisis is destruction of the abnormal cells, resulting in fewer RBCs to carry oxygen. d. Oxygen therapy does not alter the shape of the abnormal erythrocytes but only allows for increased oxygen concentration in hemoglobin.

b. When RBCs sickle, they occlude small vessels, which causes more local hypoxia and more sickling. During a sickle cell crisis, the sickling cells clog small capillaries, and the resulting hemostasis promotes a self-perpetuating cycle of local hypoxia, deoxygenation of more erythrocytes, and more sickling. Administration of oxygen may reverse sickling at first, but eventually the sickling becomes irreversible because of cell membrane damage from recurrent sickling.

A nurse is preparing to teach a group of women in a community volunteer group about heart disease. What should the nurse include in the teaching plan? a. Women are less likely to delay seeking treatment than men. b. Women are more likely to have noncardiac symptoms of heart disease. c. Women are often less ill when presenting for treatment of heart disease. d. Women have more symptoms of heart disease at a younger age than men.

b. Women are more likely to have noncardiac symptoms of heart disease. Women often have atypical angina symptoms and nonpain symptoms. Women experience the onset of heart disease about 10 years later than men. Women are often more ill on presentation and delay longer in seeking care than men.

A patient is scheduled for a cardiac catheterization with coronary angiography. Before the test, the nurse informs the patient that a. it will be important to lie completely still during the procedure. b. a flushed feeling may be noted when the contrast dye is injected. c. monitored anesthesia care will be provided during the procedure. d. arterial pressure monitoring will be required for 24 hours after the test.

b. a flushed feeling may be noted when the contrast dye is injected. A sensation of warmth or flushing is common when the contrast material is injected, which can be anxiety- producing unless it has been discussed with the patient. The patient may receive a sedative drug before the procedure, but monitored anesthesia care is not used. Arterial pressure monitoring is not routinely used after the procedure to monitor blood pressure. The patient is not immobile during cardiac catheterization and may be asked to cough or take deep breaths.

During the assessment of a 25-year-old patient with infective endocarditis (IE), the nurse would expect to find a. substernal chest pressure. b. a new regurgitant murmur. c. a pruritic rash on the chest. d. involuntary muscle movement.

b. a new regurgitant murmur. New regurgitant murmurs occur in IE because vegetations on the valves prevent valve closure. Substernal chest discomfort, rashes, and involuntary muscle movement are clinical manifestations of other cardiac disorders such as angina and rheumatic fever.

In teaching a patient about coronary artery disease, the nurse explains that the changes that occur in this disorder include (select all that apply) a. diffuse involvement of plaque formation in coronary veins. b. abnormal levels of cholesterol, especially low-density lipoproteins. c. accumulation of lipid and fibrous tissue within the coronary arteries. d. development of angina due to a decreased blood supply to the heart muscle. e. chronic vasoconstriction of coronary arteries leading to permanent vasospasm.

b. abnormal levels of cholesterol, especially low-density lipoproteins. c. accumulation of lipid and fibrous tissue within the coronary arteries. d. development of angina due to a decreased blood supply to the heart muscle.

An anticoagulant such as warfarin that interferes with prothrombin production will alter the clotting mechanism during a. platelet aggregation. b. activation of thrombin. c. the release of tissue thromboplastin. d. stimulation of factor activation complex.

b. activation of thrombin.

When caring for a patient who is recovering from a sudden cardiac death (SCD) event and has no evidence of an acute myocardial infarction (AMI), the nurse will anticipate teaching the patient that a. sudden cardiac death events rarely reoccur. b. additional diagnostic testing will be required. c. long-term anticoagulation therapy will be needed. d. limited physical activity after discharge will be needed to prevent future events.

b. additional diagnostic testing will be required. Diagnostic testing (e.g., stress test, Holter monitor, electrophysiologic studies, cardiac catheterization) is used to determine the possible cause of the SCD and treatment options. SCD is likely to recur. Anticoagulation therapy will not have any effect on the incidence of SCD, and SCD can occur even when the patient is resting.

Direction of fluid shift "from blood vessels to interstitium" and mechanism of fluid movement "osmosis" matches which event factor? a. low serum albumin b. administration of 10% glucose c. application of elastic bandages d. dehydration

b. administration of 10% glucose

An appropriate nursing intervention for a hospitalized patient with severe hemolytic anemia is to a. provide a diet high in vitamin K. b. alternate periods of rest and activity. c. teach the patient how to avoid injury. d. place the patient on protective isolation.

b. alternate periods of rest and activity. Nursing care for patients with anemia should alternate periods of rest and activity to encourage activity without causing undue fatigue. There is no indication that the patient has a bleeding disorder, so a diet high in vitamin K or teaching about how to avoid injury is not needed. Protective isolation might be used for a patient with aplastic anemia, but it is not indicated for hemolytic anemia.

To detect early signs or symptoms of inadequate oxygenation, the nurse would examine the patient for a. dyspnea and hypotension. b. apprehension and restlessness. c. cyanosis and cool, clammy skin. d. increased urine output and diaphoresis.

b. apprehension and restlessness.

Examples of third spacing are: Select all that apply. a. peripheral edema b. ascites c. burn blister d. lymphedema

b. ascites c. burn blister

The nurse is caring for a patient who had abdominal surgery yesterday. Today the patient's lung sounds in the lower lobes are diminished. The nurse knows this could be related to the occurrence of: a. pain. b. atelectasis. c. pneumonia. d. pleural effusion.

b. atelectasis. After surgery, there is an increased risk for atelectasis from anesthesia as well as restricted breathing from pain. Without deep breathing to stretch the alveoli, surfactant secretion to hold the alveoli open is not promoted. Pneumonia will occur later after surgery. Pleural effusion occurs because of blockage of lymphatic drainage or an imbalance between intravascular and oncotic fluid pressures, which is not expected in this case.

To assess the patient with pericarditis for evidence of a pericardial friction rub, the nurse should a. listen for a rumbling, low-pitched, systolic murmur over the left anterior chest. b. auscultate by placing the diaphragm of the stethoscope on the lower left sternal border. c. ask the patient to cough during auscultation to distinguish the sound from a pleural friction rub. d. feel the precordial area with the palm of the hand to detect vibrations with cardiac contraction.

b. auscultate by placing the diaphragm of the stethoscope on the lower left sternal border. Pericardial friction rubs are heard best with the diaphragm at the lower left sternal border. The nurse should ask the patient to hold his or her breath during auscultation to distinguish the sounds from a pleural friction rub. Friction rubs are not typically low pitched or rumbling and are not confined to systole. Rubs are not assessed by palpation.

The health care provider prescribes an infusion of heparin (Hep-Lock) and daily partial thromboplastin time (PTT) testing for a patient with venous thromboembolism (VTE). The nurse will plan to a. decrease the infusion when the PTT value is 65 seconds. b. avoid giving any IM medications to prevent localized bleeding. c. monitor posterior tibial and dorsalis pedis pulses with the Doppler. d. have vitamin K available in case reversal of the heparin is needed.

b. avoid giving any IM medications to prevent localized bleeding. IM injections are avoided in patients receiving anticoagulation. A PTT of 65 seconds is within the therapeutic range. Vitamin K is used to reverse warfarin. Pulse quality is not affected by VTE.

The nurse notes scleral jaundice in a patient being admitted with hemolytic anemia. The nurse will plan to check the laboratory results for the a. Schilling test. b. bilirubin level. c. stool occult blood test. d. gastric analysis testing.

b. bilirubin level. Jaundice is caused by the elevation of bilirubin level associated with red blood cell (RBC) hemolysis. The other tests would not be helpful in monitoring or treating a hemolytic anemia.

A 28-year-old man with von Willebrand disease is admitted to the hospital for minor knee surgery. The nurse will review the coagulation survey to check the a. platelet count. b. bleeding time. c. thrombin time. d. prothrombin time.

b. bleeding time. The bleeding time is affected by von Willebrand disease. Platelet count, prothrombin time, and thrombin time are normal in von Willebrand disease.

Laboratory test results indicate increased fibrin split products (FSPs). An appropriate nursing action is to monitor the patient for a. fever. b. bleeding. c. faintness. d. thrombotic episodes.

b. bleeding. During fibrinolysis by plasmin, the fibrin clot is split into smaller molecules known as FSPs or FDPs. Increased FSPs impair platelet aggregation, reduce prothrombin, prevent fibrin stabilization, and lead to bleeding.

The nurse will suspect that the patient with stable angina is experiencing a side effect of the prescribed metoprolol (Lopressor) if the a. patient is restless and agitated. b. blood pressure is 90/54 mm Hg. c. patient complains about feeling anxious. d. cardiac monitor shows a heart rate of 61 beats/minute.

b. blood pressure is 90/54 mm Hg. Patients taking b-adrenergic blockers should be monitored for hypotension and bradycardia. Because this class of medication inhibits the sympathetic nervous system, restlessness, agitation, hypertension, and anxiety will not be side effects.

A routine complete blood count indicates that an active 80-year-old man may have myelodysplastic syndrome. The nurse will plan to teach the patient about a. blood transfusion b. bone marrow biopsy. c. filgrastim (Neupogen) administration. d. erythropoietin (Epogen) administration.

b. bone marrow biopsy. Bone marrow biopsy is needed to make the diagnosis and determine the specific type of myelodysplastic syndrome. The other treatments may be necessary if there is progression of the myelodysplastic syndrome, but the initial action for this asymptomatic patient will be a bone marrow biopsy.

When auscultating over the patients abdominal aorta, the nurse hears a humming sound. The nurse documents this finding as a a. thrill. b. bruit. c. murmur. d. normal finding.

b. bruit. A bruit is the sound created by turbulent blood flow in an artery. Thrills are palpable vibrations felt when there is turbulent blood flow through the heart or in a blood vessel. A murmur is the sound caused by turbulent blood flow through the heart. Auscultating a bruit in an artery is not normal and indicates pathology

The nurse expects the long-term treatment of a patient with hyperphosphatemia from renal failure will include a. fluid restriction. b. calcium supplements. c. magnesium supplements. d. increased intake of dairy products.

b. calcium supplements.

Diltiazem (Cardizem) is ordered for a patient with newly diagnosed Prinzmetals (variant) angina. When teaching the patient, the nurse will include the information that diltiazem will a. reduce heart palpitations. b. decrease spasm of the coronary arteries. c. increase the force of the heart contractions. d. help prevent plaque from forming in the coronary arteries.

b. decrease spasm of the coronary arteries. Prinzmetals angina is caused by coronary artery spasm. Calcium channel blockers (e.g., diltiazem, amlodipine [Norvasc]) are a first-line therapy for this type of angina. Lipid-lowering drugs help reduce atherosclerosis (i.e., plaque formation), and b -adrenergic blockers decrease sympathetic stimulation of the heart (i.e., palpitations). Medications or activities that increase myocardial contractility will increase the incidence of angina by increasing oxygen demand.

Direction of fluid shift "from extracellular compartment to the cell" and mechanism of fluid movement "interstitial hydrostatic pressure" matches which event factor? a. burns b. dehydration c. fluid overload d. hyponatremia

b. dehydration

When obtaining assessment data from a patient with a microcytic, hypochromic anemia, the nurse would ask the patient about a. folic acid intake. b. dietary intake of iron. c. a history of gastric surgery. d. a history of sickle cell anemia.

b. dietary intake of iron.

When assessing subjective data related to the respiratory health of a patient with emphysema, the nurse asks about (select all that apply) a. date of last chest x-ray. b. dyspnea during rest or exercise. c. pulmonary function test results. d. ability to sleep through the entire night. e. prescription and over-the-counter medication.

b. dyspnea during rest or exercise. d. ability to sleep through the entire night. e. prescription and over-the-counter medication.

The part of the vascular system responsible for hemostasis is the a. thin capillary vessels. b. endothelial layer of the arteries. c. elastic middle layer of the veins. d. smooth muscle of the arterial wall.

b. endothelial layer of the arteries.

It is important for the nurse providing care for a patient with sickle cell crisis to a. limit the patients intake of oral and IV fluids. b. evaluate the effectiveness of opioid analgesics. c. encourage the patient to ambulate as much as tolerated. d. teach the patient about high-protein, high-calorie foods.

b. evaluate the effectiveness of opioid analgesics. Pain is the most common clinical manifestation of a crisis and usually requires large doses of continuous opioids for control. Fluid intake should be increased to reduce blood viscosity and improve perfusion. Rest is usually ordered to decrease metabolic requirements. Patients are instructed about the need for dietary folic acid, but high-protein, high-calorie diets are not emphasized.

While obtaining subjective assessment date from a patient with hypertension, the nurse recognizes that a modifiable risk factor for the development of hypertension is: a. a low-calcium diet b. excessive alcohol consumption c. a family history of hypertension d. consumption of a high-protein diet

b. excessive alcohol consumption

A patient who is receiving methotrexate for severe rheumatoid arthritis develops a megaloblastic anemia. The nurse will anticipate teaching the patient about increasing oral intake of a. iron. b. folic acid. c. cobalamin (vitamin B12). d. ascorbic acid (vitamin C).

b. folic acid. Methotrexate use can lead to folic acid deficiency. Supplementation with oral folic acid supplements is the usual treatment. The other nutrients would not correct folic acid deficiency, although they would be used to treat other types of anemia.

What sound will you hear with COPD? a. wheeze b. hyperresonance c. tubular or hollow d. no sound

b. hyperresonance due to trapped air

Which of these electrolyte imbalances is the cause of "primary polydipsia"? (select all that apply) a. hypernatremia b. hyponatremia c. hyperkalemia d. hypokalemia e. hypercalcemia f. hypocalcemia g. hyperphosphatemia h. hypophosphatemia i. hypermagnesemia j. hypomagnesemia

b. hyponatremia

Which of these electrolyte imbalances is the cause of "early burn stage"? (select all that apply) a. hypernatremia b. hyponatremia c. hyperkalemia d. hypokalemia e. hypercalcemia f. hypocalcemia g. hyperphosphatemia h. hypophosphatemia i. hypermagnesemia j. hypomagnesemia

b. hyponatremia c. hyperkalemia

A person who lives at a high altitude may normally have an increased Hgb and RBC count because a. high altitudes cause vascular fluid loss, leading to hemoconcentration. b. hypoxia caused by decreased atmospheric O2 stimulates erythropoiesis. c. the function of the spleen in removing old RBCs is impaired at high altitudes. d. impaired production of platelets leads to proportionally higher red cell counts.

b. hypoxia caused by decreased atmospheric O2 stimulates erythropoiesis.

When reviewing a patient's hematologic laboratory values after a splenectomy, the nurse would expect to find a. RBC abnormalities. b. increased WBC count. c. decreased hemoglobin. d. decreased platelet count.

b. increased WBC count.

In teaching the patient with pernicious anemia about the disease, the nurse explains that it results from a lack of a. folic acid. b. intrinsic factor. c. extrinsic factor. d. cobalamin intake

b. intrinsic factor. Pernicious anemia is a type of cobalamin (vitamin B12 ) deficiency that results when parietal cells in the stomach fail to secrete enough intrinsic factor to absorb ingested cobalamin. Folic acid deficiency may contribute to folic acid deficiency anemia, not pernicious anemia. Extrinsic factor may be a factor in some cobalamin deficiencies but not in pernicious anemia. Lack of cobalamin intake can cause cobalamin deficiency but not pernicious anemia. Increasing cobalamin intake cannot improve pernicious anemia without intrinsic factor to aid its absorption.

Which of the following ABG's would the nurse expect to see when a client has shallow breathing with periods of apnea and develops acidosis? a. pH 7.30, PaCO2 44 mmhg, HCO3- 18 mEq/L b. pH 7.29, PaCO2 62 mmhg, HCO3- 23 mEq/L c. pH 7.35, PaCO2 33 mmhg, HCO3- 18 mEq/L d. pH 7.48, PaCO2 36 mmhg, HCO3- 30 mEq/L

b. pH 7.29, PaCO2 62 mmhg, HCO3- 23 mEq/L Respiratory acidosis. If you are apneic with shallow breathing, then you are retaining CO2 which is acidic. Your kidneys work slower to compensate and it will be normal or slightly lower. This is uncompensated example.

The patient has had COPD for years, and his ABGs usually show hypoxia (PaO2 < 60 mm Hg or SaO2 < 88%) and hypercapnia (PaCO2 > 45 mm Hg). Which ABG results show movement toward respiratory acidosis and further hypoxia indicating respiratory failure? a. pH 7.35, PaO2 62 mm Hg, PaCO2 45 mm Hg b. pH 7.34, PaO2 45 mm Hg, PaCO2 65 mm Hg c. pH 7.42, PaO2 90 mm Hg, PaCO2 43 mm Hg d. pH 7.46, PaO2 92 mm Hg, PaCO2 32 mm Hg

b. pH 7.34, PaO2 45 mm Hg, PaCO2 65 mm Hg These results show worsening respiratory function and failure with the pH at 7.34, the lower PaO2 , and the higher PaCO2 . The pH results of 7.35 and 7.42 show potential normal results for the patient described. The pH of 7.46 shows alkalosis, respiratory with the low PaCO2 , but the HCO3 − results are needed to be sure.

A registered nurse (RN) is observing a student nurse who is doing a physical assessment on a patient. The RN will need to intervene immediately if the student nurse a. presses on the skin over the tibia for 10 seconds to check for edema. b. palpates both carotid arteries simultaneously to compare pulse quality. c. documents a murmur heard along the right sternal border as a pulmonic murmur. d. places the patient in the left lateral position to check for the point of maximal impulse.

b. palpates both carotid arteries simultaneously to compare pulse quality. The carotid pulses should never be palpated at the same time to avoid vagal stimulation, dysrhythmias, and decreased cerebral blood flow. The other assessment techniques also need to be corrected. However, they are not dangerous to the patient.

Which of the following acid-base imbalances is the common cause for mechanical over ventilation? a. respiratory acidosis b. respiratory alkalosis c. metabolic alkalosis d. metabolic acidosis

b. respiratory alkalosis

Which of the following acid-base imbalances is the common cause for response to anxiety, fear, and pain? a. respiratory acidosis b. respiratory alkalosis c. metabolic alkalosis d. metabolic acidosis

b. respiratory alkalosis

Which of the following acid-base imbalances matches the mechanism "decreased carbonic acid (H2CO3)"? a. respiratory acidosis b. respiratory alkalosis c. metabolic alkalosis d. metabolic acidosis

b. respiratory alkalosis

Meeting the developmental tasks of young adults with cystic fibrosis becomes a major problem primarily because a. they eventually need a lung transplant. b. they must also adapt to a chronic disease. c. any children they have will develop cystic fibrosis. d. their illness keeps them from becoming financially independent.

b. they must also adapt to a chronic disease. The presence of a chronic disease that is present at birth, delayed sexual development, difficulty in marrying and having children, and the many treatments needed by those with CF affects all relationships and development of these patients. Although a lung transplant may be needed, not all CF patients need one. Not all children will inherit CF (e.g., 25% chance for offspring with both parents having the defective gene). Many men with CF are sterile. Women may have difficulty becoming pregnant. Educational and vocational goals may be met in those who maintain treatment programs and health.

Reasons why a patient might have hyperkalemia: Select all that apply. a. insulin therapy b. tissue breakdown c. diarrhea & vomiting d. tumor lysis syndrome e. renal failure f. salt substitutes

b. tissue breakdown d. tumor lysis syndrome e. renal failure f. salt substitutes

A patient has a 6-cm thoracic aortic aneurysm that was discovered during a routine chest x-ray. When obtaining an admission history from the patient, it will be most important for the nurse to ask about a. low back pain. b. trouble swallowing. c. abdominal tenderness. d. changes in bowel habits.

b. trouble swallowing. Difficulty swallowing may occur with a thoracic aneurysm because of pressure on the esophagus. The other symptoms will be important to assess for in patients with abdominal aortic aneurysms.

When evaluating the discharge teaching for a patient with chronic peripheral artery disease (PAD), the nurse determines a need for further instruction when the patient says, I will a. have to buy some loose clothes that do not bind across my legs or waist. b. use a heating pad on my feet at night to increase the circulation and warmth in my feet. c. change my position every hour and avoid long periods of sitting with my legs crossed. d. walk to the point of pain, rest, and walk again until the pain returns for at least 30 minutes 3 times a week.

b. use a heating pad on my feet at night to increase the circulation and warmth in my feet. Because the patient has impaired circulation and sensation to the feet, the use of a heating pad could lead to burns. The other patient statements are correct and indicate that teaching has been successful.

Appropriate discharge teaching for the patient with a permanent tracheostomy after a total laryngectomy for cancer would include (select all that apply) a. encouraging regular exercise such as swimming. b. washing around the stoma daily with a moist washcloth. c. encouraging participation in postlaryngectomy support group. d. providing pictures and "hands-on" instruction for tracheostomy care. e. teaching how to hold breath and trying to gag to promote swallowing reflex.

b. washing around the stoma daily with a moist washcloth. c. encouraging participation in postlaryngectomy support group. d. providing pictures and "hands-on" instruction for tracheostomy care.

A patient who has hypertension just had an MI. Which type of medication should the nurse expect to be added to decrease the cardiac workload? a. ACE inhibitor b. β-adrenergic blocker c. Calcium channel blocker d. Angiotensin II receptor blocker (ARB)

b. β-adrenergic blocker It is recommended that patients with hypertension and after an MI be on β-adrenergic blockers indefinitely to decrease oxygen demand. They inhibit sympathetic nervous stimulation of the heart; reduce heart rate, contractility, and BP; and decrease afterload. Although calcium channel blockers decrease heart rate, contractility, and BP, they are not used unless the patient cannot tolerate β-adrenergic blockers. ACE inhibitors and angiotensin II receptor blockers (ARBs) are used for vasodilation.

A nurse is caring for a client who is receiving furosemide daily. During the morning assessment, the client tells the nurse that he is "feeling weak in the legs." Which of the following actions should the nurse take first? a. Monitor the client's bowel sounds. b. Review the client's daily laboratory results. c. Auscultate the client's lungs. d. Palpate the client's peripheral pulses.

c. Auscultate the client's lungs Using the airway, breathing, circulation approach to client care, the first action the nurse should take is to auscultate the client's lungs to assess for respiratory changes due to weakness of the respiratory muscles.

A nurse is providing discharge teaching to a client who has pulmonary TB and a new prescription for rifampin. Which of the following instructions should the nurse include? a. "Ringing in the ears is an adverse effect of this medication." b. "Have your skin test repeated in 4 months to show a positive result." c. "Expect your urine and other secretions to be orange while taking this medication." d. "Remember to take this medication with a sip of water just before your first bite of each meal."

c. "Expect your urine and other secretions to be orange while taking this medication." rationale: The nurse should inform the client that rifampin will turn urine and other secretions orange. Rifampin is hepatotoxic, so the nurse should also instruct the client to notify the provider if manifestations of hepatitis occur, including jaundice, fatigue, or malaise.

A patient had a right total knee replacement 2 days ago. Upon auscultation of the patient's posterior chest, the nurse detects discontinuous, high-pitched breath sounds just before the end of inspiration in the lower portion of both lungs. Which statement most appropriately reflects how the nurse should document the breath sounds? a. "Bibasilar wheezes present on inspiration." b. "Diminished breath sounds in the bases of both lungs." c. "Fine crackles posterior right and left lower lung fields." d. "Expiratory wheezing scattered throughout the lung fields."

c. "Fine crackles posterior right and left lower lung fields." Fine crackles are described as a series of short-duration, discontinuous, high-pitched sounds heard just before the end of inspiration.

A 2400-mg sodium diet is prescribed for a patient with chronic HF. The nurse recognizes that additional teaching is necessary when the patient makes which statement? a. "I should limit my milk intake to 2 cups a day." b. "I can eat fresh fruits and vegetables without worrying about sodium content." c. "I can eat most foods as long as I do not add salt when cooking or at the table." d. "I need to read the labels on prepared foods and medicines for their sodium content."

c. "I can eat most foods as long as I do not add salt when cooking or at the table." Not adding salt to foods will not eliminate enough sodium for the 2400-mg sodium diet. All foods that are high in sodium should be eliminated in a 2400-mg sodium diet, in addition to the elimination of salt during cooking. Examples include obviously salted foods as well as unexpected sodium sources that are identified by reading the label of prepared foods and medicines.

The nurse is teaching the patient with human immunodeficiency virus (HIV) about the diagnosis of a fungal lung infection with Candida albicans. What patient statement indicates to the nurse that further teaching is required? a. "I will be given amphotericin B to treat the fungus." b. "I got this fungus because I am immunocompromised." c. "I need to be isolated from my family and friends so they won't get it." d. "The effectiveness of my therapy can be monitored with fungal serology titers."

c. "I need to be isolated from my family and friends so they won't get it." The patient with an opportunistic fungal infection does not need to be isolated because it is not transmitted from person to person. This immunocompromised patient will be likely to have a serious infection so it will be treated with IV amphotericin B. The effectiveness of the therapy can be monitored with fungal serology titers.

A nurse is providing discharge teaching to a client who has a temporary tracheostomy. which of the following statements by the client indicates an understanding of the teaching? a. "I should dip a cotton-tipped applicator into full-strength hydrogen peroxide to cleanse around my stoma" b. "I should cut a 4" gauze dressing and place it around my trach tube to absorb drainage" c. "I should remove the old twill ties after the new ties are in place" d. "I should apply suction while inserting the catheter into my trach tube"

c. "I should remove the old twill ties after the new ties are in place" rationale: As a safety measure, the nurse should teach the client to wait until the new ties are in place to remove the old ties. This practice can prevent accidental decannulation.

Priority Decision: Following teaching about medications for PAD, the nurse determines that more instruction is needed when the patient makes which statement? a. "I should take 1 aspirin a day to prevent clotting in my legs." b. "The lisinopril I use for my blood pressure may help me walk further without pain." c. "I will need to have frequent blood tests to evaluate the effect of the pentoxifylline I will be taking." d. "Cilostazol should help me increase my walking distance and speed and help prevent pain in my legs."

c. "I will need to have frequent blood tests to evaluate the effect of the pentoxifylline I will be taking." Pentoxifylline allows blood cells to pass through small vessels, but there are no blood tests related to it. Warfarin (Coumadin), which needs international normalized ration (INR) blood tests, is not recommended for prevention of cardiovascular disease (CVD) events in patients with PAD. All the other statements are correct in relation to treatment of PAD.

A patient with AML is considering a hematopoietic stem cell transplant and asks the nurse what is involved. What is the best response the nurse can give the patient? a. "Your bone marrow is destroyed by radiation, and new bone marrow cells from a matched donor are injected into your bones." b. "A specimen of your bone marrow may be aspirated and treated to destroy any leukemic cells and then reinfused when your disease becomes worse." c. "Leukemic cells and bone marrow stem cells are eliminated with chemotherapy and/or total-body radiation, and new bone marrow cells from a donor are infused." d. "During chemotherapy and/or total-body irradiation to destroy all of your blood cells, you may be given transfusions of RBCs and platelets to prevent complications."

c. "Leukemic cells and bone marrow stem cells are eliminated with chemotherapy and/or total-body radiation, and new bone marrow cells from a donor are infused." Whether the donor bone marrow is from a human leukocyte antigen (HLA)-matched donor or taken from the patient during a remission for later use, HSCT always involves the use of chemotherapy and/or totalbody radiation to eliminate leukemic cells and the patient's bone marrow stem cells totally before IV infusion of the donor cells. A severe pancytopenic period follows the transplant, during which the patient must be in protective isolation and during which RBC and platelet transfusions may be given.

Which action should the nurse take first when a patient complains of acute chest pain and dyspnea soon after insertion of a centrally inserted IV catheter? a. Notify the health care provider. b. Offer reassurance to the patient. c. Auscultate the patient's breath sounds. d. Give the prescribed PRN morphine sulfate IV.

c. Auscultate the patient's breath sounds. The initial action should be to assess the patient further because the history and symptoms are consistent with several possible complications of central line insertion, including embolism and pneumothorax. The other actions may be appropriate, but further assessment of the patient is needed before notifying the health care provider, offering reassurance, or administration of morphine.

A 38-year-old man is treated for hypertension with triamterene and hydrochlorothiazide and metoprolol (Lopressor). Four months after his last clinic visit, his BP returns to pretreatment levels, and he admits he has not been taking his medication regularly. What is the nurse's best response to this patient? a. "Try always to take your medication when you carry out another daily routine so that you do not forget to take it." b. "You probably would not need to take medications for hypertension if you would exercise more and stop smoking." c. "The drugs you are taking cause sexual dysfunction in many patients. Are you experiencing any problems in this area?" d. "You need to remember that hypertension can be only controlled with medication, not cured, and you must always take your medication."

c. "The drugs you are taking cause sexual dysfunction in many patients. Are you experiencing any problems in this area?" Sexual dysfunction, which can occur with many of the antihypertensive drugs, including thiazide and potassium-sparing diuretics and β-adrenergic blockers, can be a major reason that a male patient does not adhere to his treatment regimen. It is helpful for the nurse to raise the subject because sexual problems may be easier for the patient to discuss and handle once it has been explained that the drug may be the source of the problem.

A client is donating blood. The nurse tells the client that an indirect Coombs' test will be performed on the blood. The client asks the nurse about the purpose of the test. Which response should the nurse provide to the client? a. "The test detects the presence of hepatitis B virus." b. "The test detects the amount of hemoglobin in the blood." c. "The test detects circulating antibodies against red blood cells (RBCs)." d. "The test detects the presence of human immunodeficiency virus (HIV)."

c. "The test detects circulating antibodies against red blood cells (RBCs)."

A nurse is providing teaching for a client who has venous insufficiency of the lower extremities. Which of the following statements by the client indicates an understanding of the teaching? a. "If my stockings feel tight, I'll just roll them down for a while." b. "I'll put on my elastic stockings at the first sign of swelling." c. "When I sit down to watch television, I'll be sure to put my feet up." d. "It's okay to cross my legs as long as it's for less than an hour."

c. "When I sit down to watch television, I'll be sure to put my feet up." Venous insufficiency makes it difficult for blood flow to return to the heart. Elevating the feet will increase the return. The client should elevate them for at least 20 min several times per day.

At what point in the healing process of the myocardium following an infarct does early scar tissue result in an unstable heart wall? a. 2 to 4 days after MI b. 4 to 10 days after MI c. 10 to 14 days after MI d. 6 weeks after MI

c. 10 to 14 days after MI At 10 to 14 days after MI, the new scar tissue is weak and is vulnerable to increased stress because of the unstable state of healing at this point, as well as the increasing physical activity of the patient. At 2 to 4 days, removal of necrotic tissue is taking place by phagocytic cells. By 4 to 10 days, the necrotic tissue has been cleared and a collagen matrix for scar tissue has been deposited. Healing with scar-tissue replacement of the necrotic area is usually complete in 6 weeks.

The nurse is interpreting a tuberculin skin test (TST) for a patient with end-stage renal disease due to diabetes. Which finding would indicate a positive reaction? a. Acid-fast bacilli cultured at the injection site b. 15-mm area of redness at the TST injection site c. 11-mm area of induration at the TST injection site d. Wheal formed immediately after intradermal injection

c. 11-mm area of induration at the TST injection site An area of induration 10 mm or larger would be a positive reaction in a person with end-stage renal disease. Reddened, flat areas do not indicate a positive reaction. A wheal appears when the TST is administered that indicates correct administration of the intradermal antigen. Presence of acid-fast bacilli in the sputum indicates active tuberculosis.

When the nurse is screening patients for possible peripheral arterial disease, indicate where the posterior tibial artery will be palpated. a. 1 b. 2 c. 3 d. 4

c. 3 The posterior tibial site is located behind the medial malleolus of the tibia.

In which patient would a mechanical prosthetic valve be preferred over a biologic valve for valve replacement? a. 41-year-old man with peptic ulcer disease b. 22-year-old woman who wants to have children c. 35-year-old man with a history of seasonal asthma d. 62-year-old woman with early Alzheimer's disease

c. 35-year-old man with a history of seasonal asthma Mechanical prosthetic valves require long-term anticoagulation, and this is a factor in deciding about the type of valve to use for replacement. Patients who cannot take anticoagulant therapy, such as patients at risk for hemorrhage, women of childbearing age, patients who may not be compliant with anticoagulation therapy, and patients over age 65 years may be candidates for the less durable biologic valves

A patient has a junctional escape rhythm on the monitor. The nurse will expect the patient to have a heart rate of _____ beats/minute. a. 15 to 20 b. 20 to 40 c. 40 to 60 d. 60 to 100

c. 40 to 60 If the sinoatrial (SA) node fails to discharge, the atrioventricular (AV) node will automatically discharge at the normal rate of 40 to 60 beats/minute. The slower rates are typical of the bundle of His and the Purkinje system and may be seen with failure of both the SA and AV node to discharge. The normal SA node rate is 60 to 100 beats/minute.

After receiving report, which patient admitted to the emergency department should the nurse assess first? a. 67-year-old who has a gangrenous left foot ulcer with a weak pedal pulse b. 58-year-old who is taking anticoagulants for atrial fibrillation and has black stools c. 50-year-old who is complaining of sudden sharp and worst ever upper back pain d. 39-year-old who has right calf tenderness, redness, and swelling after a long plane ride

c. 50-year-old who is complaining of sudden sharp and worst ever upper back pain The patients presentation is consistent with dissecting thoracic aneurysm, which will require rapid intervention. The other patients do not need urgent interventions.

Which instruction will the nurse plan to include in discharge teaching for the patient admitted with a sickle cell crisis? a. Take a daily multivitamin with iron. b. Limit fluids to 2 to 3 quarts per day. c. Avoid exposure to crowds when possible. d. Drink only two caffeinated beverages daily.

c. Avoid exposure to crowds when possible. Exposure to crowds increases the patients risk for infection, the most common cause of sickle cell crisis. There is no restriction on caffeine use. Iron supplementation is generally not recommended. A high-fluid intake is recommended.

The nurse is caring for a group of patients. Which patient is at risk of aspiration? a. A 58-yr-old patient with absent bowel sounds 12 hours after abdominal surgery b. A 67-yr-old patient who had a cerebrovascular accident with expressive dysphasia c. A 26-yr-old patient with continuous enteral tube feedings through a nasogastric tube d. A 92-yr-old patient with viral pneumonia and coarse crackles throughout the lung fields

c. A 26-yr-old patient with continuous enteral tube feedings through a nasogastric tube Conditions that increase the risk of aspiration include decreased level of consciousness, difficulty swallowing (dysphagia), and nasogastric intubation with or without enteral nutrition. With loss of consciousness, the gag and cough reflexes are depressed, and aspiration is more likely to occur. Dysphasia is difficulty with speech. Absent bowel sounds and coarse crackles do not increase the risk for aspiration.

Priority Decision: Following an ascending aortic aneurysm repair, what is an important finding that the nurse should report immediately to the HCP? a. Shallow respirations and poor coughing b. Decreased drainage from the chest tubes c. A change in level of consciousness and inability to speak d. Lower extremity pulses that are decreased from the preoperative baseline

c. A change in level of consciousness and inability to speak During repair of an ascending aortic aneurysm, the blood supply to the carotid arteries may be interrupted, leading to neurologic complications manifested by a decreased level of consciousness (LOC) and altered pupil responses to light as well as changes in facial symmetry, speech, upper extremity movement, and hand grasp quality. The thorax is opened for ascending aortic surgery, and shallow breathing, poor cough, and decreasing chest drainage are expected. Lower limb pulses may normally be decreased or absent for a short time following surgery.

Which manifestation is an indication that a patient is having hypertensive emergency? a. Symptoms of a stroke with an elevated BP b. A systolic BP (SBP) > 180 mm Hg and a diastolic BP (DBP) > 110 mm Hg c. A sudden rise in BP accompanied by neurologic impairment d. A severe elevation of BP that occurs over several days or weeks

c. A sudden rise in BP accompanied by neurologic impairment Hypertensive emergency, a type of hypertensive crisis, is a situation in which a patient's BP is severely elevated with evidence of acute target organ disease (e.g., cerebrovascular, cardiovascular, renal, or retinal). The neurologic manifestations are often similar to the presentation of a stroke but do not show the focal or lateralizing symptoms of stroke. Hypertensive crises are defined by the degree of organ damage and how rapidly the BP rises, not by specific BP measurements. A hypertensive urgency is a less severe crisis, in which a patient's BP becomes severely elevated over hours or days but there is no evidence of target organ damage.

What is an advantage of a tracheostomy over an endotracheal (ET) tube for long-term management of an upper airway obstruction? a. A tracheostomy is safer to perform in an emergency. b. An ET tube has a higher risk of tracheal pressure necrosis. c. A tracheostomy tube allows for more comfort and mobility. d. An ET tube is more likely to lead to lower respiratory tract infection.

c. A tracheostomy tube allows for more comfort and mobility. With a tracheostomy (versus an endotracheal [ET] tube), patient comfort is increased because there is no tube in the mouth. Because the tube is more secure, mobility is improved. The ET tube is more easily inserted in an emergency situation. It is preferable to perform a tracheostomy in the operating room because it requires careful dissection, but it can be performed with local anesthetic in the intensive care unit (ICU) or in an emergency. With a cuff, tracheal pressure necrosis is as much a risk with a tracheostomy tube as with an ET tube, and infection is also as likely to occur because the defenses of the upper airway are bypassed.

When caring for a patient after a cardiac catheterization with coronary angiography, which finding should be of most concern to the nurse? a. Swelling at the catheter insertion site b. Development of raised wheals on the patient's trunk c. Absence of pulses distal to the catheter insertion site d. Patient pain at the insertion site at 4 on a scale of 0 to 10

c. Absence of pulses distal to the catheter insertion site An absence of pulses distal to the catheter insertion site indicates that clotting is occluding blood flow to the extremity and is an emergency that requires immediate medical attention. Some swelling and pain at the site are expected, but the site is also monitored for bleeding, and a pressure dressing or compression device may be applied. Hives may occur as a result of iodine sensitivity and will require treatment but the priority is the lack of pulses.

Interprofessional care of the patient with NSTEMI differs from that of a patient with STEMI in that NSTEMI is more often initially treated with what? a. PCI b. CABG c. Acute intensive drug therapy d. Reperfusion therapy with thrombolytics

c. Acute intensive drug therapy An NSTEMI is an ACS that indicates a transient thrombosis or incomplete coronary artery occlusion. Treatment involves intensive drug therapy with antiplatelets, glycoprotein IIb/IIIa inhibitors, antithrombotics, and heparin to prevent clot extension. In addition, IV NTG is used. Reperfusion therapy using thrombolytics, CABG, or PCI is used for treatment of STEMI

Patient-Centered Care: A 60-year-old male farmer is diagnosed with multiple myeloma. He has pain in his ribs with movement and his diagnostic studies show hypercalcemia. What nursing interventions should be implemented for this patient as the interprofessional care is being initiated (select all that apply)? a. Provide privacy b. Complete bed rest c. Adequate hydration d. Prepare for dialysis e. Assess for infection f. Encourage ambulation

c. Adequate hydration e. Assess for infection f. Encourage ambulation Because he has hypercalcemia, adequate hydration and ambulation, not bed rest, are implemented to dilute calcium and prevent protein precipitates from causing renal tubular obstruction and help bones resorb some calcium with weight bearing. Assessing for infection must be done because the excess plasma cells are monoclonal and ineffective against infection. Privacy would be provided for all patients. Although this patient may need dialysis in the late states of multiple myeloma, it is not needed at this time. Analgesia for rib pain will be provided as prescribed by the HCP.

A patient with TB has been admitted to the hospital and is placed on airborne precautions and in an isolation room. What should the nurse teach the patient? (select all that apply) a. Expect routine TB testing to evaluate the infection. b. No visitors will be allowed while in airborne isolation. c. Adherence to precautions includes coughing into a paper tissue. d. Take all medications for full length of time to prevent multidrug-resistant TB. e. Wear a standard isolation mask if leaving the airborne infection isolation room.

c. Adherence to precautions includes coughing into a paper tissue. d. Take all medications for full length of time to prevent multidrug-resistant TB. e. Wear a standard isolation mask if leaving the airborne infection isolation room.

postoperative patient receiving a transfusion of packed red blood cells develops chills, fever, headache, and anxiety 35 minutes after the transfusion is started. After stopping the transfusion, what action should the nurse take? a. Draw blood for a new crossmatch. b. Send a urine specimen to the laboratory. c. Administer PRN acetaminophen (Tylenol). d. Give the PRN diphenhydramine (Benadryl).

c. Administer PRN acetaminophen (Tylenol). The patients clinical manifestations are consistent with a febrile, nonhemolytic transfusion reaction. The transfusion should be stopped and antipyretics administered for the fever as ordered. A urine specimen is needed if an acute hemolytic reaction is suspected. Diphenhydramine (Benadryl) is used for allergic reactions. This type of reaction does not indicate incorrect crossmatching.

A patient seen in the asthma clinic has recorded daily peak flows that are 75% of the baseline. Which action will the nurse plan to take next? a. Increase the dose of the leukotriene inhibitor. b. Teach the patient about the use of oral corticosteroids. c. Administer a bronchodilator and recheck the peak flow. d. Instruct the patient to keep the next scheduled follow-up appointment.

c. Administer a bronchodilator and recheck the peak flow. The patients peak flow reading indicates that the condition is worsening (yellow zone). The patient should take the bronchodilator and recheck the peak flow. Depending on whether the patient returns to the green zone, indicating well-controlled symptoms, the patient may be prescribed oral corticosteroids or a change in dosing of other medications. Keeping the next appointment is appropriate, but the patient also needs to be taught how to control symptoms now and use the bronchodilator.

Which action will the nurse include in the plan of care for a patient who has thalassemia major? a. Teach the patient to use iron supplements. b. Avoid the use of intramuscular injections. c. Administer iron chelation therapy as needed. d. Notify health care provider of hemoglobin 11g/dL.

c. Administer iron chelation therapy as needed. The frequent transfusions used to treat thalassemia major lead to iron toxicity in patients unless iron chelation therapy is consistently used. Iron supplementation is avoided in patients with thalassemia. There is no need to avoid intramuscular injections. The goal for patients with thalassemia major is to maintain a hemoglobin of 10g/dL or greater.

While caring for a patient with respiratory disease, the nurse observes that the patients SpO2 drops from 93% to 88% while the patient is ambulating in the hallway. What is the priority action of the nurse? a. Notify the health care provider. b. Document the response to exercise. c. Administer the PRN supplemental O2. d. Encourage the patient to pace activity.

c. Administer the PRN supplemental O2. The drop in SpO2 to 85% indicates that the patient is hypoxemic and needs supplemental oxygen when exercising. The other actions are also important, but the first action should be to correct the hypoxemia.

The nurse notes that a patient who was admitted with diabetic ketoacidosis has rapid, deep respirations. Which action should the nurse take? a. Give the prescribed PRN lorazepam (Ativan). b. Start the prescribed PRN oxygen at 2 to 4 L/min. c. Administer the prescribed normal saline bolus and insulin. d. Encourage the patient to take deep, slow breaths with guided imagery.

c. Administer the prescribed normal saline bolus and insulin. The rapid, deep (Kussmaul) respirations indicate a metabolic acidosis and the need for correction of the acidosis with a saline bolus to prevent hypovolemia followed by insulin administration to allow glucose to reenter the cells. Oxygen therapy is not indicated because there is no indication that the increased respiratory rate is related to hypoxemia. The respiratory pattern is compensatory, and the patient will not be able to slow the respiratory rate. Lorazepam administration will slow the respiratory rate and increase the level of acidosis.

What are the priority nursing interventions 8 hours after an abdominal aortic aneurysm repair? a. Assessing nutritional status and dietary preferences b. Initiating IV heparin and monitoring anticoagulation c. Administering IV fluids and watching kidney function d. Elevating the legs and applying compression stockings

c. Administering IV fluids and watching kidney function

Which action for a patient with neutropenia is appropriate for the registered nurse (RN) to delegate to a licensed practical/vocational nurse (LPN/LVN)? a. Assessing the patient for signs and symptoms of infection b. Teaching the patient the purpose of neutropenic precautions c. Administering subcutaneous filgrastim (Neupogen) injection d. Developing a discharge teaching plan for the patient and family

c. Administering subcutaneous filgrastim (Neupogen) injection Administration of subcutaneous medications is included in LPN/LVN education and scope of practice. Patient education, assessment, and developing the plan of care require RN level education and scope of practice.

A child involved in a motor vehicle accident has a thready pulse, decreased blood pressure, rapid respiration, and clammy skin with pallor. The child is not responding to secondary fluid bolus. Which treatment would the nurse anticipate administering to ensure the safety of the child? a. Administration of platelets b. Administration of fresh frozen plasma c. Administration of red blood cells d. Administration of insulin

c. Administration of red blood cells

Which patient is most likely to be in the fibrous stage of development of coronary artery disease (CAD)? a. Age 40 years, thrombus adhered to the coronary artery wall b. Age 50 years, rapid onset of disease with hypercholesterolemia c. Age 32 years, thickened coronary arterial walls with narrowed vessel lumen d. Age 19 years, high low-density lipoprotein (LDL) cholesterol, lipid-filled smooth muscle cells

c. Age 32 years, thickened coronary arterial walls with narrowed vessel lumen The fibrous plaque stage has progressive changes that can be seen by age 30 years. Collagen covers the fatty streak and forms a fibrous plaque in the artery. The thrombus adheres to the arterial wall in the complicated lesion stage. Rapid onset of coronary artery disease (CAD) with hypercholesterolemia may be related to familial hypercholesterolemia, not a stage of CAD development. The fatty streak stage is the earliest stage of atherosclerosis and can be seen by age 20 years.

The nurse suspects the client is having an MI. They report epigastric tightness, nausea, SOB & pallor. Put in order a. Give Nitroglycerin b. Obtain cardiac biomarkers c. Apply nasal cannula @ 2L d. Obtain a 12-lead ECG

c. Apply nasal cannula @ 2L d. Obtain a 12-lead ECG a. Give Nitroglycerin b. Obtain cardiac biomarkers

Risk Factor for or Response to Respiratory Problem: Constipation, incontinence. Which Functional Health Pattern does it fall under? a. Health perception- health management b. Nutritional-metabolic c. Elimination d. Activity-exercise

c. Elimination

A patient is hospitalized with pneumonia. Which diagnostic test should be used to measure the efficiency of gas exchange in the lung and tissue oxygenation? a. Thoracentesis b. Bronchoscopy c. Arterial blood gases d. Pulmonary function tests

c. Arterial blood gases Arterial blood gases are used to assess the efficiency of gas exchange in the lung and tissue oxygenation as is pulse oximetry. Thoracentesis is used to obtain specimens for diagnostic evaluation, remove pleural fluid, or instill medication into the pleural space. Bronchoscopy is used for diagnostic purposes, to obtain biopsy specimens, and to assess changes resulting from treatment. Pulmonary function tests measure lung volumes and airflow to diagnose pulmonary disease, monitor disease progression, evaluate disability, and evaluate response to bronchodilators.

A 60-year-old male enters the emergency room complaining of burning pain under his sternum that radiates to his jaw. He has been experiencing this pain for three weeks, periodically, with episodes that occur as often as three times a week. The nurse proceeds in which of the following ways? a. Administer a dose of ibuprofen b. Prepares the patient for immediate transfer to the cardiac lab c. Ask the patient if he has antacids d. Immediately calls the provider

c. Ask the patient if he has antacids The patient is describing symptoms of heartburn. While it is important to rule out a cardiac-related cause, the nurse must do more assessment before reaching a conclusion.

Which action should the nurse take when administering the initial dose of oral labetalol (Normodyne) to a patient with hypertension? a. Encourage the use of hard candy to prevent dry mouth. b. Instruct the patient to ask for help if heart palpitations occur. c. Ask the patient to request assistance when getting out of bed. d. Teach the patient that headaches may occur with this medication.

c. Ask the patient to request assistance when getting out of bed. Labetalol decreases sympathetic nervous system activity by blocking both - and b-adrenergic receptors, leading to vasodilation and a decrease in heart rate, which can cause severe orthostatic hypotension. Heart palpitations, dry mouth, dehydration, and headaches are possible side effects of other antihypertensives.

Which action by the nurse will determine if the therapies ordered for a patient with chronic constrictive pericarditis are effective? a. Assess for the presence of a paradoxical pulse. b. Monitor for changes in the patients sedimentation rate. c. Assess for the presence of jugular venous distention (JVD). d. Check the electrocardiogram (ECG) for ST segment changes.

c. Assess for the presence of jugular venous distention (JVD). Because the most common finding on physical examination for a patient with chronic constrictive pericarditis is jugular venous distention, a decrease in JVD indicates improvement. Paradoxical pulse, ST-segment ECG changes, and changes in sedimentation rates occur with acute pericarditis but are not expected in chronic constrictive pericarditis.

A patient with a tracheostomy has a new order for a fenestrated tracheostomy tube. Which action should the nurse include in the plan of care in collaboration with the speech therapist? a. Leave the tracheostomy inner cannula inserted at all times. b. Place the decannulation cap in the tube before cuff deflation. c. Assess the ability to swallow before using the fenestrated tube. d. Inflate the tracheostomy cuff during use of the fenestrated tube.

c. Assess the ability to swallow before using the fenestrated tube. Because the cuff is deflated when using a fenestrated tube, the patients risk for aspiration should be assessed before changing to a fenestrated tracheostomy tube. The decannulation cap is never inserted before cuff deflation because to do so would obstruct the patients airway. The cuff is deflated and the inner cannula removed to allow air to flow across the patients vocal cords when using a fenestrated tub.

Which is the most appropriate nursing intervention when preparing to administer synthetic erythropoietin to a patient with anemia from chronic renal disease? a. Give the drug in the patient's right deltoid muscle. b. Hold the dose for a HgB less than 10 g/dL. c. Assess the patient's blood pressure prior to administration. d. Check the patient's urine output to ensure an output of at least 30 mL/hr

c. Assess the patient's blood pressure prior to administration.

What does the nurse include when planning for postural drainage for the patient with COPD? a. Schedules the procedure 1 hour before and after meals b. Has the patient cough before positioning to clear the lungs c. Assesses the patient's tolerance for dependent (head-down) positions d. Ensures that percussion and vibration are done before positioning the patient

c. Assesses the patient's tolerance for dependent (head-down) positions Many postural drainage positions require placement in Trendelenburg position, but patients with head injury, heart disease, hemoptysis, chest trauma, and others should not be placed in these positions. Postural drainage should be done 1 hour before and 3 hours after meals if possible. Coughing, percussion, and vibration are all done after the patient has been positioned.

A patient is recovering from a myocardial infarction (MI) and develops chest pain on day 3 that increases when taking a deep breath and is relieved by leaning forward. Which action should the nurse take next? a. Assess the feet for pedal edema. b. Palpate the radial pulses bilaterally. c. Auscultate for a pericardial friction rub. d. Check the heart monitor for dysrhythmias.

c. Auscultate for a pericardial friction rub. The patients symptoms are consistent with the development of pericarditis, a possible complication of MI. The other assessments listed are not consistent with the description of the patients symptoms.

A patient with a history of chronic heart failure is admitted to the emergency department (ED) with severe dyspnea and a dry, hacking cough. Which action should the nurse do first? a. Auscultate the abdomen. b. Check the capillary refill. c. Auscultate the breath sounds. d. Assess the level of orientation.

c. Auscultate the breath sounds. This patients severe dyspnea and cough indicate that acute decompensated heart failure (ADHF) is occurring. ADHF usually manifests as pulmonary edema, which should be detected and treated immediately to prevent ongoing hypoxemia and cardiac/respiratory arrest. The other assessments will provide useful data about the patients volume status and also should be accomplished rapidly, but detection (and treatment) of pulmonary complications is the priority.

A patient with hemophilia comes to the clinic for treatment. What should the nurse anticipate that he or she will need to administer? a. Whole blood b. Thromboplastin c. Coagulation factor d. Fresh frozen plasma

c. Coagulation factor Although whole blood and fresh frozen plasma contain the clotting factors that are deficient in hemophilia, specific coagulation factors have been developed that are purer and safer in preventing infection transmission. Thromboplastin is factor III and is not deficient in patients with hemophilia.

Which diagnostic test will be most useful to the nurse in determining whether a patient admitted with acute shortness of breath has heart failure? a. Serum troponin b. Arterial blood gases c. B-type natriuretic peptide d. 12-lead electrocardiogram

c. B-type natriuretic peptide B-type natriuretic peptide (BNP) is secreted when ventricular pressures increase, as they do with heart failure. Elevated BNP indicates a probable or very probable diagnosis of heart failure. A twelve-lead electrocardiogram, arterial blood gases, and troponin may also be used in determining the causes or effects of heart failure but are not as clearly diagnostic of heart failure as BNP.

Which diagnostic test is most useful in differentiating dyspnea related to pulmonary effects of HF from dyspnea related to pulmonary disease? a. Exercise stress testing b. Cardiac catheterization c. B-type natriuretic peptide (BNP) levels d. Determination of blood urea nitrogen (BUN)

c. B-type natriuretic peptide (BNP) levels b-type natriuretic peptide (BNP) is released from the ventricles in response to increased blood volume in the heart and is a good marker for HF. If BNP is elevated, shortness of breath is caused by HF; if BNP is normal, dyspnea is caused by pulmonary disease. BNP opposes the actions of the RAAS, resulting in vasodilation and reduction in blood volume. Exercise stress testing and cardiac catheterization are more important tests to diagnose CAD, and although the blood urea nitrogen (BUN) may be elevated in HF, it is a reflection of decreased renal perfusion.

During the patient's acute postoperative period following repair of an AAA, the nurse should ensure that which goal is achieved? a. Hypothermia is maintained to decrease oxygen need. b. IV fluids are given to maintain urine output of 100 mL/hr. c. BP and all peripheral pulses are assessed at least every hour. d. The patient's BP is kept lower than baseline to prevent leaking at the incision line.

c. BP and all peripheral pulses are assessed at least every hour. The BP and peripheral pulses are evaluated every hour in the acute postoperative period to ensure that BP is adequate to maintain graft patency and that extremities are being perfused. BP is kept within normal range. If BP is too low, thrombosis of the graft may occur; if it is too high, it may cause leaking or rupture at the suture line. Hypothermia is induced during surgery, but the patient is rewarmed as soon as surgery is over. Fluid replacement to maintain urine output at 100 mL/hr would increase the BP too much and only 30 mL/hr of urine is needed to show adequate renal perfusion

A patient is recovering from an uncomplicated MI. Which rehabilitation guideline is a priority to include in the teaching plan? a. Refrain from sexual activity for a minimum of 3 weeks. b. Plan a diet program that aims for a 1- to 2-lb weight loss per week. c. Begin an exercise program that aims for at least 5 30-minute sessions per week. d. Consider the use of erectile agents and prophylactic NTG before engaging in sexual activity.

c. Begin an exercise program that aims for at least 5 30-minute sessions per week.

Which topic will the nurse plan to include in discharge teaching for a patient with systolic heart failure and an ejection fraction of 33%? a. Need to begin an aerobic exercise program several times weekly b. Use of salt substitutes to replace table salt when cooking and at the table c. Benefits and side effects of angiotensin-converting enzyme (ACE) inhibitors d. Importance of making an annual appointment with the primary care provider

c. Benefits and side effects of angiotensin-converting enzyme (ACE) inhibitors The core measures for the treatment of heart failure established by The Joint Commission indicate that patients with an ejection fraction (EF) <40% receive an ACE inhibitor to decrease the progression of heart failure. Aerobic exercise may not be appropriate for a patient with this level of heart failure, salt substitutes are not usually recommended because of the risk of hyperkalemia, and the patient will need to see the primary care provider more frequently than annually.

The nurse is caring for a client diagnosed with a deep venous thrombosis. Which nursing assessment is a priority​? a. Level of consciousness b. Stroke scale assessment c. Bilateral breath sounds d. EKG rhythm & rate

c. Bilateral breath sounds Pulmonary embolism is a complication of DVT. Assessing the​ client's respiratory​ status, including auscultating bilateral breath​ sounds, is appropriate. The​ client's airway and breathing take priority. Atrial fibrillation and ischemic stroke are risk factors for development of DVT. Assessing level of consciousness and performing a stroke scale assessment are appropriate to assess for ischemic stroke. Obtaining EKG rhythm and rate is appropriate to assess for atrial fibrillation.

The nurse obtains the following data when assessing a patient who experienced an ST-segment-elevation myocardial infarction (STEMI) 2 days previously. Which information is most important to report to the health care provider? a. The troponin level is elevated. b. The patient denies ever having a heart attack. c. Bilateral crackles are auscultated in the mid-lower lobes. d. The patient has occasional premature atrial contractions (PACs).

c. Bilateral crackles are auscultated in the mid-lower lobes. The crackles indicate that the patient may be developing heart failure, a possible complication of myocardial infarction (MI). The health care provider may need to order medications such as diuretics or angiotensin- converting enzyme (ACE) inhibitors for the patient. Elevation in troponin level at this time is expected. PACs are not life-threatening dysrhythmias. Denial is a common response in the immediate period after the MI.

A patient complains of shortness of breath and having to sleep on three pillows to breathe comfortably at night. During the nurse's examination, what findings will suggest that the cause of this patient's dyspnea is due to heart disease rather than respiratory disease? a. Increased anteroposterior diameter b. Clubbing of the fingers c. Bilateral peripheral edema d. Increased tactile fremitus

c. Bilateral peripheral edema

An outpatient who has chronic heart failure returns to the clinic after 2 weeks of therapy with metoprolol (Toprol XL). Which assessment finding is most important for the nurse to report to the health care provider? a. 2+ pedal edema b. Heart rate of 56 beats/minute c. Blood pressure (BP) of 88/42 mm Hg d. Complaints of fatigue

c. Blood pressure (BP) of 88/42 mm Hg The patients BP indicates that the dose of metoprolol may need to be decreased because of hypotension. Bradycardia is a frequent adverse effect of b-adrenergic blockade, but the rate of 56 is not unusual with -adrenergic blocker therapy. b-Adrenergic blockade initially will worsen symptoms of heart failure in many patients, and patients should be taught that some increase in symptoms, such as fatigue and edema, is expected during the initiation of therapy with this class of drugs.

The nurse is caring for a patient with a descending aortic dissection. Which assessment finding is most important to report to the health care provider? a. Weak pedal pulses b. Absent bowel sounds c. Blood pressure 137/88 mm Hg d. 25 mL urine output over last hour

c. Blood pressure 137/88 mm Hg The blood pressure is typically kept at less than 120 mm Hg systolic to minimize extension of the dissection. The nurse will need to notify the health care provider so that b-blockers or other antihypertensive medications can be prescribed. The other findings are typical with aortic dissection and should also be reported but do not require immediate action.

A nurse is caring for a client who had dehydration and is receiving IV fluids. Which assessing for complications, the nurse should recognize which of the following manifestations as a sign of fluid overload? a. Increased urine specific gravity b. Hypoactive bowel sounds c. Bounding peripheral pulses d. Decreased respiratory rate

c. Bounding peripheral pulses Fluid overload results in increased vascular volume and places a greater workload on the heart. Thus, an expected finding is bounding peripheral pulses.

Which statement accurately describes mitral valve prolapse? a. Rapid onset prevents left chamber dilation b. May be caused by pulmonary hypertension c. Buckling of valve into left atrium during ventricular systole d. Rapid development of pulmonary edema and cardiogenic shock

c. Buckling of valve into left atrium during ventricular systole Mitral valve prolapse is the buckling of the valve leaflets into the left atrium during ventricular systole. The rapid onset that prevents left chamber dilation and the rapid development of pulmonary edema and cardiogenic shock occur with acute mitral regurgitation. Pulmonary hypertension may contribute to tricuspid valve disease.

Why does a patient's respiratory rate increase when there is an excess of carbon dioxide in the blood? a. CO2 displaces oxygen on hemoglobin, leading to a decreased PaO2 . b. CO2 causes an increase in the amount of hydrogen ions available in the body. c. CO2 combines with water to form carbonic acid, which lowers the pH of cerebrospinal fluid. d. CO2 directly stimulates chemoreceptors in the medulla to increase

c. CO2 combines with water to form carbonic acid, which lowers the pH of cerebrospinal fluid. A combination of excess CO2 and H2O results in carbonic acid, which lowers the pH of cerebrospinal fluid and stimulates an increase in the respiratory rate. Peripheral chemoreceptors in the carotid and aortic bodies also respond to increases in PaCO2 to stimulate the respiratory center. Excess CO2 does not increase the amount of hydrogen ions available in the body but does combine with the hydrogen of water to form an acid.

Which finding about a patient with polycythemia vera is most important for the nurse to report to the health care provider? a. Hematocrit 55% b. Presence of plethora c. Calf swelling and pain d. Platelet count 450,000/mL

c. Calf swelling and pain The calf swelling and pain suggest that the patient may have developed a deep vein thrombosis, which will require diagnosis and treatment to avoid complications such as pulmonary embolus. The other findings will also be reported to the health care provider but are expected in a patient with this diagnosis.

An ER pt. started having SOB & palpitations yesterday, HR-123 to 150, BP-99/71 what is the primary intervention? a. Start anticoagulants to prevent clots b. Synchronized cardioversion c. Cardizem to control rate d. Amiodarone to convert the rhythm

c. Cardizem to control rate

Why is the classification of pneumonia as community-acquired pneumonia (CAP) or hospital-acquired pneumonia (HAP) clinically useful? a. Atypical pneumonia syndrome is more likely to occur in HAP. b. Diagnostic testing does not have to be used to identify causative agents. c. Causative agents can be predicted, and empiric treatment is often effective. d. IV antibiotic therapy is necessary for HAP, but oral therapy is adequate for CAP.

c. Causative agents can be predicted, and empiric treatment is often effective. Pneumonia that has its onset in the community is usually caused by different microorganisms than pneumonia that develops related to hospitalization and treatment can be empiric—based on observations and experience without knowing the exact causative organism. Frequently, a causative organism cannot be identified from cultures, and treatment is based on experience.

Patient with a local anesthetic to throat is hospitalized and at risk for aspiration pneumonia. What nursing intervention is indicated to prevent pneumonia? a. Position to side, protect airway b. Check placement of the tube before feeding and residual feeding; keep head of bed up after feedings or continuously with continuous feedings c. Check gag reflex before feeding or offering fluids d. Cut food in small bites, encourage thorough chewing, and provide soft foods that are easier to swallow than liquids

c. Check gag reflex before feeding or offering fluids

Priority Decision: A pulse oximetry monitor indicates that the patient has a drop in arterial oxygen saturation by pulse oximetry (SpO2 ) from 95% to 85% over several hours. What is the first action the nurse should take? a. Order stat ABGs to confirm the SpO2 with a SaO2. b. Start oxygen administration by nasal cannula at 2 L/min. c. Check the position of the probe on the finger or earlobe. d. Notify the health care provider of the change in baseline PaO2 .

c. Check the position of the probe on the finger or earlobe. Pulse oximetry is inaccurate if the probe is loose, if there is low perfusion, or when skin color is dark. Before other measures are taken, the nurse should check the probe site. If the probe is intact at the site and perfusion is adequate, an ABG analysis will be ordered by the HCP to verify accuracy, and oxygen may be administered, depending on the patient's condition and the assessment of respiratory and cardiac status.

A nurse is reviewing the laboratory results of several clients who have peripheral arterial disease. The nurse should plan to provide dietary teaching for the client who has which laboratory values? A. Cholesterol 180 mg/dl, HDL 70 mg/dl, LDL 90 mg/dl b. Cholesterol 185 mg/dl, HDL 50 mg/dl, LDL 120 mg/dl c. Cholesterol 190 mg/dl, HDL 25 mg/dl, LDL 160 mg/dl d. Cholesterol 195 mg/dl, HDL 55 mg/dl, LDL 125 mg/dl

c. Cholesterol 190 mg/dl, HDL 25 mg/dl, LDL 160 mg/dl Rationale: These laboratory values for HDL and LDL are outside of the expected reference range and indicate that the nurse should provide dietary teaching to the client. The expected reference range for cholesterol is less than 200 mg/dL; for HDL is above 45 mg/dL for males and above 55 mg/dL for females; and for LDL is less than 130 mg/dL.

What is a possible cause for inspection abnormal finding "finger clubbing"? a. Bronchoconstriction b. Partial obstruction of trachea or larynx c. Chronic hypoxemia d. Pleurisy

c. Chronic hypoxemia

When auscultating the patient's lower lungs, the nurse hears low-pitched sounds similar to blowing through a straw under water on inspiration. How should the nurse document these sounds? a. Stridor b. Vesicular c. Coarse crackles d. Bronchovesicular

c. Coarse crackles Coarse crackles are a series of long-duration, discontinuous, low-pitched sounds caused by air passing through an airway intermittently occluded by mucus, an unstable bronchial wall, or a fold of mucosa. Coarse crackles are evident on inspiration and at times expiration. Stridor is a continuous crowing sound of constant pitch from partial obstruction of larynx or trachea. Vesicular sounds are relatively soft, low-pitched, gentle, rustling sounds. They are heard over all lung areas except the major bronchi. Bronchovesicular sounds are normal sounds heard anteriorly over the mainstem bronchi on either side of the sternum and posteriorly between the scapulae with a medium pitch and intensity.

Risk Factor for or Response to Respiratory Problem: Decreased cognitive function with restlessness, irritability. Chest pain or pain with breathing. Which Functional Health Pattern does it fall under? a. Activity-exercise b. Sleep-rest c. Cognitive-perceptual d. Self-perception- self-concept

c. Cognitive-perceptual

What is the first priority of interprofessional care for a patient with a suspected acute aortic dissection? a. Reduce anxiety. b. Monitor chest pain. c. Control blood pressure. d. Increase myocardial contractility.

c. Control blood pressure.

Risk Factor for or Response to Respiratory Problem: Dyspnea-anxiety-dyspnea cycle, poor coping with stress of chronic respiratory problems. Which Functional Health Pattern does it fall under? a. Role-relationship b. Sexuality-reproductive c. Coping- stress tolerance d. Value-belief

c. Coping- stress tolerance

A nurse is caring for a client who had an onset of chest pain 24 hr ago. The nurse should recognize that an increase in which of the following is diagnostic of a MI? A. Myoglobin b. C-reactive protein c. Creatine kinase- MB d. Homocysteine

c. Creatine kinase- MB Rationale: Creatine kinase-MB is the isoenzyme specific to the myocardium. Elevated creatine kinase-MB indicates myocardial muscle injury.

What is the primary BP effect of β-adrenergic blockers, such as atenolol (Tenormin)? a. Vasodilation of arterioles by blocking movement of calcium into cells b. Decrease Na + and water reabsorption by blocking the effect of aldosterone c. Decrease CO by decreasing rate and strength of the heart and renin secretion by the kidneys d. Vasodilation caused by inhibiting sympathetic outflow from the central nervous system (CNS)

c. Decrease CO by decreasing rate and strength of the heart and renin secretion by the kidneys Cardioselective β-adrenergic blockers decrease CO, reduce sympathetic vasoconstrictor tone, and decrease renin secretion by kidneys. Calcium channel blockers reduce BP by causing blocked movement of calcium into cells, which causes vasodilation of arterioles. Spironolactone blocks the effect of aldosterone. Central adrenergic antagonists, such as clonidine (Catapres) inhibit sympathetic outflow from the central nervous system (CNS).

During treatment with reteplase (Retavase) for a patient with a STEMI, which finding should most concern the nurse? a. Oozing of blood from the IV site b. BP of 102/60 mm Hg with an HR of 78 bpm c. Decrease in the responsiveness of the patient d. Intermittent accelerated idioventricular rhythms

c. Decrease in the responsiveness of the patient Decreasing level of consciousness (LOC) may reflect hypoxemia resulting from internal bleeding, which is always a risk with thrombolytic therapy. Oozing of blood is expected, as are reperfusion dysrhythmias. BP is low but not considered abnormal because the pulse is within normal range. Idioventricular dysrhythmias are common with reperfusion.

The long-term care nurse is evaluating the effectiveness of protein supplements for an older resident who has a low serum total protein level. Which assessment finding indicates that the patient's condition has improved? a. Hematocrit 28% b. Absence of skin tenting c. Decreased peripheral edema d. Blood pressure 110/72 mm Hg

c. Decreased peripheral edema Edema is caused by low oncotic pressure in individuals with low serum protein levels. The decrease in edema indicates an improvement in the patient's protein status. Good skin turgor is an indicator of fluid balance, not protein status. A low hematocrit could be caused by poor protein intake. Blood pressure does not provide a useful clinical tool for monitoring protein status.

In the patient experiencing ventricular fibrillation (VF), what is the rationale for using defibrillation? a. Enhance repolarization and relaxation of ventricular myocardial cells b. Provide an electrical impulse that stimulates normal myocardial contractions c. Depolarize the cells of the myocardium to allow the SA node to resume pacemaker function d. Deliver an electrical impulse to the heart at the time of ventricular contraction to convert the heart to a sinus rhythm

c. Depolarize the cells of the myocardium to allow the SA node to resume pacemaker function The intent of defibrillation is to apply an electrical current to the heart that will depolarize the cells of the myocardium so that subsequent repolarization of the cells will allow the SA node to resume the role of pacemaker. An artificial pacemaker provides an electrical impulse that stimulates normal myocardial contractions. Synchronized cardioversion involves delivery of a shock that is programmed to occur during the QRS complex of the ECG, but this cannot be done during ventricular fibrillation because there is no normal ventricular contraction or QRS complex.

Patient-Centered Care: A 62-year-old woman smokes a pack of cigarettes per day and has a BP 138/88 mm Hg. She has no symptoms of CAD, but a recent LDL level was 154 mg/dL (3.98 mmol/L). Based on these findings, the nurse would expect that which treatment plan would be used first for this patient? a. Diet and drug therapy b. Exercise instruction only c. Diet therapy and smoking cessation d. Drug therapy and smoking cessation

c. Diet therapy and smoking cessation Without the total serum cholesterol and HDL results, diet therapy and smoking cessation are indicated for this patient without CAD who has prehypertension and an LDL level ≥ 130 mg/dL. When the patient's LDL level is 75 to 189 mg/dL with a 10-year risk for cardiovascular disease (CVD) of 7.5% or above, drug therapy would be added to diet therapy. Because tobacco use is related to increased BP and LDL level, the benefit of smoking cessation is almost immediate. Exercise is indicated to reduce risk factors throughout treatment.

The nurse completes a shift assessment on a patient admitted in the early phase of heart failure. When auscultating the patients lungs, which finding would the nurse most likely hear? a. Continuous rumbling, snoring, or rattling sounds mainly on expiration b. Continuous high-pitched musical sounds on inspiration and expiration c. Discontinuous, high-pitched sounds of short duration heard on inspiration d. A series of long-duration, discontinuous, low-pitched sounds during inspiration

c. Discontinuous, high-pitched sounds of short duration heard on inspiration Fine crackles are likely to be heard in the early phase of heart failure. Fine crackles are discontinuous, high- pitched sounds of short duration heard on inspiration. Rhonchi are continuous rumbling, snoring, or rattling sounds mainly on expiration. Course crackles are a series of long-duration, discontinuous, low-pitched sounds during inspiration. Wheezes are continuous high-pitched musical sounds on inspiration and expiration.

The nurse should instruct a pt who is prescribed digoxin (Lanoxin) on which of the following info? a. How to manage nausea that can be associated w/ taking digoxin. b. Foods that should be eaten while taking this drug. c. Do not take the meds & to not take it if the pulse is under 60 beats per minute. d. Checking the pulse for one minute each day & recording the result on a notepad

c. Do not take the meds & to not take it if the pulse is under 60 beats per minute.

A patient with asthma has the following arterial blood gas (ABG) results early in an acute asthma attack: pH 7.48, partial pressure of carbon dioxide in arterial blood (PaCO2 ) 30 mm Hg, partial pressure of oxygen in arterial blood (PaO2 ) 78 mm Hg. What is the most appropriate action by the nurse? a. Prepare the patient for mechanical ventilation. b. Have the patient breathe in a paper bag to raise the PaCO2 . c. Document the findings and monitor the ABGs for a trend toward acidosis. d. Reduce the patient's oxygen flow rate to keep the PaO2 at the current level.

c. Document the findings and monitor the ABGs for a trend toward acidosis. Early in an asthma attack, an increased respiratory rate and hyperventilation create a respiratory alkalosis with increased pH and decreased PaCO2 , accompanied by hypoxemia. As the attack progresses, pH shifts to normal, then decreases, with arterial blood gases (ABGs) that reflect respiratory acidosis with hypoxemia. During the attack, high-flow oxygen should be provided. Breathing in a paper bag, although used to treat some types of hyperventilation, would increase the hypoxemia.

A common collaborative problem related to both hyperkalemia and hypokalemia is which potential complication? a. Seizures b. Paralysis c. Dysrhythmias d. Acute kidney injury

c. Dysrhythmias Potassium maintains normal cardiac rhythm, transmission and conduction of nerve impulses, and contraction of muscles. Cardiac cells have the most clinically significant changes with potassium imbalances because of changes in cardiac conduction. Although paralysis may occur with severe potassium imbalances, cardiac changes are seen earlier and much more commonly.

The nurse evaluates that nursing interventions to promote airway clearance in a patient admitted with COPD are successful based on which finding? a. Absence of dyspnea b. Improved mental status c. Effective and productive coughing d. PaO2 within normal range for the patient

c. Effective and productive coughing Airway clearance is most directly evaluated as successful if the patient can engage in effective and productive coughing. Absence of dyspnea, improved mental status, and PaO2 within normal range for the patient show improved respiratory status but do not evaluate airway clearance.

A 46-year-old service-counter worker undergoes sclerotherapy for treatment of superficial varicose veins at an outpatient center. Which instructions should the nurse provide to the patient before discharge? a. Sitting at the work counter, rather than standing, is recommended. b. Exercise, such as walking or jogging, can cause recurrence of varicosities. c. Elastic compression stockings should be applied before getting out of bed. d. Taking an aspirin daily will help prevent clots from forming around venous valves.

c. Elastic compression stockings should be applied before getting out of bed. Elastic compression stockings are applied with the legs elevated to reduce pressure in the lower legs. Walking is recommended to prevent recurrent varicosities. Sitting and standing are both risk factors for varicose veins and venous insufficiency. An aspirin a day is not adequate to prevent venous thrombosis and would not be recommended for the patient who had just had sclerotherapy.

A patient who has chest pain is admitted to the emergency department (ED) and all of the following are ordered. Which one should the nurse arrange to be completed first? a. Chest x-ray b. Troponin level c. Electrocardiogram (ECG) d. Insertion of a peripheral IV.

c. Electrocardiogram (ECG) The priority for the patient is to determine whether an acute myocardial infarction (AMI) is occurring so that reperfusion therapy can begin as quickly as possible. ECG changes occur very rapidly after coronary artery occlusion, and an ECG should be obtained as soon as possible. Troponin levels will increase after about 3 hours. Data from the chest x-ray may impact the patients care but are not helpful in determining whether the patient is experiencing a myocardial infarction (MI). Peripheral access will be needed but not before the ECG

A patient who has chest pain is admitted to the emergency department (ED), and all of the following items are prescribed. Which one should the nurse arrange to be completed first? a. Chest x-ray. b. Troponin level. c. Electrocardiogram (ECG). d. Insertion of a peripheral IV.

c. Electrocardiogram (ECG). Electrocardiogram (ECG)Rationale: The priority for the patient is to determine whether an acute myocardial infarction (AMI) is occurring so that the appropriate therapy can begin as quickly as possible. ECG changes occur very rapidly after coronary artery occlusion, and an ECG should be obtained as soon as possible. Troponin levels will increase after about 3 hours. Data from the chest x-ray may impact the patient's care but are not helpful in determining whether the patient is experiencing a myocardial infarction. Peripheral access will be needed but not before the ECG.

A patient who had a total hip replacement had an intraoperative hemorrhage 14 hours ago. Which laboratory result would the nurse expect to find? a. Hematocrit of 46% b. Hemoglobin of 13.8 g/dL c. Elevated reticulocyte count d. Decreased white blood cell (WBC) count

c. Elevated reticulocyte count Hemorrhage causes the release of reticulocytes (immature red blood cells) from the bone marrow into circulation. The hematocrit and hemoglobin levels are normal. The WBC count is not affected by bleeding.

A patient with chronic obstructive pulmonary disease (COPD) has poor gas exchange. Which action by the nurse would be most appropriate? a. Have the patient rest in bed with the head elevated to 15 to 20 degrees. b. Ask the patient to rest in bed in a high-Fowlers position with the knees flexed. c. Encourage the patient to sit up at the bedside in a chair and lean slightly forward. d. Place the patient in the Trendelenburg position with several pillows behind the head.

c. Encourage the patient to sit up at the bedside in a chair and lean slightly forward. Patients with COPD improve the mechanics of breathing by sitting up in the tripod position. Resting in bed with the head elevated in a semi-Fowlers position would be an alternative position if the patient was confined to bed, but sitting in a chair allows better ventilation. The Trendelenburg position or sitting upright in bed with the knees flexed would decrease the patients ability to ventilate well.

A postoperative patient asks the nurse why the physician ordered daily administration of enoxaparin (Lovenox). Which reply by the nurse is most appropriate? a "This medication will help prevent breathing problems after surgery, such as pneumonia." b "This medication will help lower your blood pressure to a safer level, which is very important after surgery." c "This medication will help prevent blood clots from forming in your legs until your level of activity, such as walking, returns to normal." d "This medication is a narcotic pain medication that will help take away any muscle aches caused by positioning on the operating room table."

c. Enoxaparin is an anticoagulant that is used to prevent DVTs postoperatively. All other options do not describe the action or purpose of enoxaparin.

A patient who is taking rifampin (Rifadin) for tuberculosis calls the clinic and reports having orange discolored urine and tears. Which is the best response by the nurse? a. Ask if the patient is experiencing shortness of breath, hives, or itching. b. Ask the patient about any visual abnormalities such as red-green color discrimination. c. Explain that orange discolored urine and tears are normal while taking this medication. d. Advise the patient to stop the drug and report the symptoms to the health care provider.

c. Explain that orange discolored urine and tears are normal while taking this medication. Orange-colored body secretions are a side effect of rifampin. The patient does not have to stop taking the medication. The findings are not indicative of an allergic reaction. Alterations in red-green color discrimination commonly occurs when taking ethambutol (Myambutol), which is a different TB medication.

Which type of transfusion reaction occurs with leukocyte or plasma protein incompatibility and may be avoided with leukocyte reduction filters? a. Allergic reaction b. Acute hemolytic reaction c. Febrile, nonhemolytic reaction d. Massive blood transfusion reaction

c. Febrile, nonhemolytic reaction Febrile nonhemolytic reaction is the most common transfusion reaction. Allergic reactions occur with sensitivity to foreign plasma proteins and can be treated prophylactically with antihistamines. Acute hemolytic reactions are related to the infusion of ABO-incompatible blood or components with 10 mL or more of RBCs. Massive blood transfusion reactions occur when patients receive more RBCs or blood than the total blood volume.

Which of the following descriptions matches the mechanism of "osmosis"? a. Uses a protein carrier molecule b. Force exerted by a fluid c. Flow of water from low-solute concentration to high-solute concentration d. Passive movement of molecules from a high concentration to lower concentration

c. Flow of water from low-solute concentration to high-solute concentration

Which medications are the most effective in improving asthma control by reducing bronchial hyperresponsiveness, blocking the late-phase reaction, and inhibiting migration of inflammatory cells (select all that apply)? a. Zileuton (Zyflo CR) b. Omalizumab (Xolair) c. Fluticasone (Flovent HFA) d. Salmeterol (Serevent) e. Montelukast (Singulair) f. Budesonide g. Beclomethasone (Qvar) h. Theophylline i. Mometasone (Asmanex Twisthaler)

c. Fluticasone (Flovent HFA) f. Budesonide g. Beclomethasone (Qvar) i. Mometasone (Asmanex Twisthaler) These are the corticosteroids described. Zileuton (Zyflo CR) and montelukast (Singulair) are leukotriene modifiers that interfere with the synthesis or block the action of the leukotriene inflammatory mediators that cause bronchoconstriction. Omalizumab (Xolair) is a monoclonal antibody to immunoglobulin (Ig)E, which prevents IgE from attaching to mast cells and prevents the release of chemical mediators. Salmeterol (Serevent) is a long-acting β2 -adrenergic agonist bronchodilator. Theophylline is a methylxanthine used when other longterm bronchodilators are not available or affordable.

When teaching the patient with mild asthma about the use of the peak flow meter, what should the nurse teach the patient to do? a. Always carry the flowmeter in case an asthma attack occurs. b. Use the flowmeter to check the status of the patient's asthma every time the patient takes quick-relief medication. c. Follow the written asthma action plan (e.g., take quick-relief medication) if the expiratory flow rate is in the yellow zone. d. Use the flowmeter by emptying the lungs, closing the mouth around the mouthpiece, and inhaling through the meter as quickly as possible.

c. Follow the written asthma action plan (e.g., take quick-relief medication) if the expiratory flow rate is in the yellow zone. A yellow zone reading with the peak flow meter indicates that the patient's asthma is getting worse and quick-relief medications should be used. The meter is routinely used each morning before taking medications after the personal best peak flow number has been determined. It does not always have to be on hand. The meter measures the ability to empty the lungs and involves blowing through the meter.

Which of the following descriptions matches the mechanism of "hydrostatic pressure"? a. Force determined by osmolality of a fluid b. Pressure exerted by plasma proteins c. Force exerted by a fluid d. Uses a protein carrier molecule

c. Force exerted by a fluid

A 65 yo male comes into the ED with complaints of crushing chest pain that radiates into his shoulder and left arm. The admitting diagnosis is acute MI. Immediate admission orders include Oxygen at NC 4L/min, blood work, chest xray, an ECG, and 2 mg of IV morphine. What should the nurse do first? a. Call the doctor b. Obtain a 12 lead ecg c. Give Morphine d. Provide blood work

c. Give Morphine Although obtaining the ECG, chest x-ray, and blood work are all important, the nurse's priority action would be to relieve the crushing chest pain. Opioids may be used for pain control in addition to sublingual nitroglycerin if the blood pressure is adequate. All patients with STEMI and NSTEMI require immediately chewed aspirin 160 mg to 325 mg. Furthermore, the patient should have intravenous access and oxygen supplementation if oxygen saturation is less than 91%.

A patient who is on the progressive care unit develops atrial flutter, rate 150, with associated dyspnea and chest pain. Which action that is included in the hospital dysrhythmia protocol should the nurse do first? a. Obtain a 12-lead electrocardiogram (ECG). b. Notify the health care provider of the change in rhythm. c. Give supplemental O2 at 2 to 3 L/min via nasal cannula. d. Assess the patients vital signs including oxygen saturation.

c. Give supplemental O2 at 2 to 3 L/min via nasal cannula. Because this patient has dyspnea and chest pain in association with the new rhythm, the nurses initial actions should be to address the patients airway, breathing, and circulation (ABC) by starting with oxygen administration. The other actions also are important and should be implemented rapidly.

The nurse assesses a patient who has been hospitalized for 2 days. The patient has been receiving normal saline IV at 100 mL/hr, has a nasogastric tube to low suction, and is NPO. Which assessment finding would be a priority for the nurse to report to the health care provider? a. Oral temperature of 100.1 F b. Serum sodium level of 138 mEq/L (138 mmol/L) c. Gradually decreasing level of consciousness (LOC) d. Weight gain of 2 pounds (1 kg) above the admission weight

c. Gradually decreasing level of consciousness (LOC) The patient's history and change in LOC could be indicative of fluid and electrolyte disturbances: extracellular fluid (ECF) excess, ECF deficit, hyponatremia, hypernatremia, hypokalemia, or metabolic alkalosis. Further diagnostic information is needed to determine the cause of the change in LOC and the appropriate interventions. The weight gain, elevated temperature, crackles, and serum sodium level also will be reported, but do not indicate a need for rapid action to avoid complications.

What is the most important measure in the treatment of venous leg ulcers? a. Elevation of the affected leg b. Application of topical antibiotics c. Graduated compression stockings d. Application of moist to dry dressing

c. Graduated compression stockings Although leg elevation, moist dressings, and systemic antibiotics are useful in treatment of venous stasis ulcers, the most important factor is compression, which minimizes venous stasis, venous hypertension, and edema and prevents recurrence. Compression may be applied with various methods including stockings, elastic bandages or wraps, or a Velcro wrap, among others.

A patient diagnosed with secondary hypertension asks why it is called secondary and not primary. What is the best explanation for the nurse to provide? a. Has a more gradual onset than primary hypertension b. Does not cause the target organ damage that occurs with primary hypertension c. Has a specific cause, such as renal disease, that often can be treated by medicine or surgery d. Is caused by age-related changes in BP regulatory mechanisms in people over 65 years of age

c. Has a specific cause, such as renal disease, that often can be treated by medicine or surgery Secondary hypertension has an underlying cause that will be treated, in contrast to primary or essential hypertension, which has no single known cause. Isolated systolic hypertension occurs when the systolic BP (SBP) is consistently equal to or over 130 mm Hg and the diastolic BP (DBP) is consistently equal to or over at 80 mm Hg.

Following an acute myocardial infarction (AMI), a patient ambulates in the hospital hallway. When the nurse is evaluating the patients response to the activity, which assessment data would indicate that the exercise level should be decreased? a. Blood pressure (BP) changes from 118/60 to 126/68 mm Hg. b. Oxygen saturation drops from 99% to 95%. c. Heart rate increases from 66 to 92 beats/minute. d. Respiratory rate goes from 14 to 20 breaths/minute.

c. Heart rate increases from 66 to 92 beats/minute. A change in heart rate of more than 20 beats over the resting heart rate indicates that the patient should stop and rest. The increases in BP and respiratory rate, and the slight decrease in oxygen saturation, are normal responses to exercise.

Which nursing intervention will be most effective when assisting the patient with coronary artery disease (CAD) to make appropriate dietary changes? a. Give the patient a list of low-sodium, low-cholesterol foods that should be included in the diet. b. Emphasize the increased risk for heart problems unless the patient makes the dietary changes. c. Help the patient modify favorite high-fat recipes by using monosaturated oils when possible. d. Inform the patient that a diet containing no saturated fat and minimal salt will be necessary.

c. Help the patient modify favorite high-fat recipes by using monosaturated oils when possible. Lifestyle changes are more likely to be successful when consideration is given to the patients values and preferences. The highest percentage of calories from fat should come from monosaturated fats. Although low- sodium and low-cholesterol foods are appropriate, providing the patient with a list alone is not likely to be successful in making dietary changes. Completely removing saturated fat from the diet is not a realistic expectation. Up to 7% of calories in the therapeutic lifestyle changes (TLC) diet can come from saturated fat. Telling the patient about the increased risk without assisting further with strategies for dietary change is unlikely to be successful.

Heparin is ordered for a patient with a nonST-segment elevation myocardial infarction (NSTEMI). What is the purpose of the heparin? a. Heparin enhances platelet aggregation. b. Heparin decreases coronary artery plaque size. c. Heparin prevents the development of new clots in the coronary arteries. d. Heparin dissolves clots that are blocking blood flow in the coronary arteries.

c. Heparin prevents the development of new clots in the coronary arteries. Heparin helps prevent the conversion of fibrinogen to fibrin and decreases coronary artery thrombosis. It does not change coronary artery plaque, dissolve already formed clots, or enhance platelet aggregation.

A patient with chronic kidney disease has hyperphosphatemia. What is a commonly associated electrolyte imbalance? a. Hypokalemia b. Hyponatremia c. Hypocalcemia d. Hypomagnesemia

c. Hypocalcemia Kidneys are the major route of phosphate excretion, a function that is impaired in renal failure. A reciprocal relationship exists between phosphorus and calcium, and high serum phosphate levels of kidney failure cause low calcium concentration in the serum.

A 65-year-old male client has been admitted for complications due to chronic heart failure, and is receiving several medications. Medications include digoxin, Lasix, and metoprolol. The nurse enters the client room and he informs her that he is feeling nauseous and has vomited. She suspects the client is experiencing digoxin toxicity. What has the most likely cause the client to experience digoxin toxicity? a. Digoxin level of 1.5 b. Change in heart rate due to metoprolol c. Hypokalemia due to Lasix d. Left ventricular hypertrophy

c. Hypokalemia due to Lasix Digoxin toxicity is often precipitated by low potassium levels (hypokalemia). Hypokalemia is a risk with the taking of loop diuretics such as Lasix, so monitoring of potassium levels should be performed to prevent digoxin toxicity.

The nurse teaches a patient about pulmonary function testing (PFT). Which statement, if made by the patient, indicates teaching was effective? a. I will use my inhaler right before the test. b. I wont eat or drink anything 8 hours before the test. c. I should inhale deeply and blow out as hard as I can during the test. d. My blood pressure and pulse will be checked every 15 minutes after the test.

c. I should inhale deeply and blow out as hard as I can during the test. For PFT, the patient should inhale deeply and exhale as long, hard, and fast as possible. The other actions are not needed with PFT. The administration of inhaled bronchodilators should be avoided 6 hours before the procedure.

After the nurse has finished teaching a patient about the use of sublingual nitroglycerin (Nitrostat), which patient statement indicates that the teaching has been effective? a. I can expect some nausea as a side effect of nitroglycerin. b. I should only take the nitroglycerin if I start to have chest pain. c. I will call an ambulance if I still have pain after taking 3 nitroglycerin 5 minutes apart. d. Nitroglycerin helps prevent a clot from forming and blocking blood flow to my heart.

c. I will call an ambulance if I still have pain after taking 3 nitroglycerin 5 minutes apart. The emergency medical services (EMS) system should be activated when chest pain or other symptoms are not completely relieved after 3 sublingual nitroglycerin tablets taken 5 minutes apart. Nitroglycerin can be taken to prevent chest pain or other symptoms from developing (e.g., before intercourse). Gastric upset (e.g., nausea) is not an expected side effect of nitroglycerin. Nitroglycerin does not impact the underlying pathophysiology of coronary artery atherosclerosis.

A patient with heart failure has a new order for captopril (Capoten) 12.5 mg PO. After administering the first dose and teaching the patient about the drug, which statement by the patient indicates that teaching has been effective? a. I will be sure to take the medication with food. b. I will need to eat more potassium-rich foods in my diet. c. I will call for help when I need to get up to use the bathroom. d. I will expect to feel more short of breath for the next few days.

c. I will call for help when I need to get up to use the bathroom. Captopril can cause hypotension, especially after the initial dose, so it is important that the patient not get up out of bed without assistance until the nurse has had a chance to evaluate the effect of the first dose. The angiotensin-converting enzyme (ACE) inhibitors are potassium sparing, and the nurse should not teach the patient to purposely increase sources of dietary potassium. Increased shortness of breath is expected with the initiation of b-adrenergic blocker therapy for heart failure, not for ACE inhibitor therapy. ACE inhibitors are best absorbed when taken an hour before eating.

In preparation for discharge, the nurse teaches a patient with chronic stable angina how to use the prescribed short-acting and long-acting nitrates. Which patient statement indicates that the teaching has been effective? a. I will check my pulse rate before I take any nitroglycerin tablets. b. I will put the nitroglycerin patch on as soon as I get any chest pain. c. I will stop what I am doing and sit down before I put the nitroglycerin under my tongue. d. I will be sure to remove the nitroglycerin patch before taking any sublingual nitroglycerin.

c. I will stop what I am doing and sit down before I put the nitroglycerin under my tongue. The patient should sit down before taking the nitroglycerin to decrease cardiac workload and prevent orthostatic hypotension. Transdermal nitrates are used prophylactically rather than to treat acute pain and can be used concurrently with sublingual nitroglycerin. Although the nurse should check blood pressure before giving nitroglycerin, patients do not need to check the pulse rate before taking nitrates.

Which statement by a patient with restrictive cardiomyopathy indicates that the nurses discharge teaching about self-management has been most effective? a. I will avoid taking aspirin or other anti-inflammatory drugs. b. I will need to limit my intake of salt and fluids even in hot weather. c. I will take antibiotics when my teeth are cleaned at the dental office. d. I should begin an exercise program that includes things like biking or swimming.

c. I will take antibiotics when my teeth are cleaned at the dental office. Patients with restrictive cardiomyopathy are at risk for infective endocarditis and should use prophylactic antibiotics for any procedure that may cause bacteremia. The other statements indicate a need for more teaching by the nurse. Dehydration and vigorous exercise impair ventricular filling in patients with restrictive cardiomyopathy. There is no need to avoid salt (unless ordered), aspirin, or NSAIDs.

Following a splenectomy for the treatment of ITP, what laboratory test result would the nurse expect to find? a. Decreased RBCs b. Decreased WBCs c. Increased platelets d. Increased immunoglobulins

c. Increased platelets Splenectomy may be indicated for treatment for ITP, and when the spleen is removed, platelet counts increase significantly in most patients. In any of the disorders in which the spleen removes excessive blood cells, splenectomy will most often increase peripheral RBC, WBC, and platelet counts.

The nurse is performing a respiratory assessment for a patient admitted with pneumonia. Which clinical manifestation would the nurse expect to find? a. Hyperresonance on percussion b. Vesicular breath sounds in all lobes c. Increased vocal fremitus on palpation d. Fine crackles in all lobes on auscultation

c. Increased vocal fremitus on palpation A typical physical examination finding for a patient with pneumonia is increased vocal fremitus on palpation. Other signs of pulmonary consolidation include bronchial breath sounds, egophony, and crackles in the affected area. With pleural effusion, there may be dullness to percussion over the affected area.

The body compensates for anemia by: a. Sending more blood to the GI tract & skin b. Increasing the heart rate and decreasing respirations c. Increasing the production of erythropoietin d. Decreasing blood pressure

c. Increasing the production of erythropoietin

What is the underlying cause of lymphadenopathy, splenomegaly, and hepatomegaly in leukemia? a. The development of infection at these sites b. Increased compensatory production of blood cells by these organs c. Infiltration of the organs by increased numbers of WBCs in the blood d. Normal hypertrophy of the organs in an attempt to destroy abnormal cells

c. Infiltration of the organs by increased numbers of WBCs in the blood d. Normal hypertrophy of the organs in an attempt to destroy abnormal cells Almost all leukemias cause some degree of hepatosplenomegaly because of infiltration of these organs as well as the bone marrow, lymph nodes, bones, and CNS by excessive WBCs in the blood.

A patient with possible disseminated intravascular coagulation arrives in the emergency department with a blood pressure of 82/40, temperature 102 F (38.9 C), and severe back pain. Which physician order will the nurse implement first? a. Administer morphine sulfate 4 mg IV. b. Give acetaminophen (Tylenol) 650 mg. c. Infuse normal saline 500 mL over 30 minutes. d. Schedule complete blood count and coagulation studies.

c. Infuse normal saline 500 mL over 30 minutes. The patients blood pressure indicates hypovolemia caused by blood loss and should be addressed immediately to improve perfusion to vital organs. The other actions also are appropriate and should be rapidly implemented, but improving perfusion is the priority for this patient.

The nurse and an unlicensed assistive personnel (UAP) are caring for a group of clients. Which nursing intervention should the nurse perform? a. Measure the client's output from the indwelling catheter. b. Record the client's intake and out on the I&O sheet. c. Instruct the client on appropriate fluid restrictions. d. Provide water for a client diagnosed with diabetes insipidus.

c. Instruct the client on appropriate fluid restrictions. The nurse cannot delegate teaching.

Which assessment finding obtained by the nurse when assessing a patient with acute pericarditis should be reported immediately to the health care provider? a. Pulsus paradoxus 8 mm Hg b. Blood pressure (BP) of 168/94 c. Jugular venous distention (JVD) to jaw level d. Level 6 (0 to 10 scale) chest pain with a deep breath

c. Jugular venous distention (JVD) to jaw level The JVD indicates that the patient may have developed cardiac tamponade and may need rapid intervention to maintain adequate cardiac output. Hypertension would not be associated with complications of pericarditis, and the BP is not high enough to indicate that there is any immediate need to call the health care provider. A pulsus paradoxus of 8 mm Hg is normal. Level 6/10 chest pain should be treated but is not unusual with pericarditis.

Priority Decision: A 76-year-old woman has an Hgb of 7.3 g/dL (73 g/L) and is experiencing ataxia, confusion, weakness, and fatigue on admission to the hospital. What is the priority nursing intervention for this patient? a. Provide a darkened, quiet room. b. Have the family stay with the patient. c. Keep top bedside rails up and call bell in close reach. d. Question the patient about possible causes of anemia

c. Keep top bedside rails up and call bell in close reach. Keeping the top bedside rails up and the call bell within reach will facilitate safety and safe mobility for this patient. In the older adult, confusion, ataxia, fatigue, and weakness are common manifestations of anemia and place the patient at risk for injury. Nursing interventions should include safety precautions to prevent falls and injury when these symptoms are present. The nurse, not the patient's family, is responsible for the patient, and although a quiet room may promote rest, it is not as important as protection of the patient.

What is an example of an appropriate IV solution to treat an extracellular fluid volume deficit? a. D5W b. 3% saline c. Lactated Ringer's solution d. D5W in ½ normal saline (0.45%)

c. Lactated Ringer's solution An isotonic solution does not change the osmolality of the blood and does not cause fluid shifts between the ECF and ICF. In the case of ECF loss, an isotonic solution, such as lactated Ringer's solution, is ideal because it stays in the extracellular compartment. A hypertonic solution would pull fluid from the cells into the ECF, resulting in cellular fluid loss and possible vascular overload.

What causes the pain that occurs with myocardial ischemia? a. Death of myocardial tissue b. Dysrhythmias caused by cellular irritability c. Lactic acid accumulation during anaerobic metabolism d. Increased pressure in the ventricles and pulmonary vessels

c. Lactic acid accumulation during anaerobic metabolism When the coronary arteries are occluded, contractility ceases after several minutes, depriving the myocardial cells of glucose and oxygen for aerobic metabolism. Anaerobic metabolism begins, and lactic acid accumulates, irritating myocardial nerve fibers that then transmit a pain message to the cardiac nerves and upper thoracic posterior roots. The other factors may occur during vessel occlusion but are not the source of pain.

A patient who has a large amount of carbon dioxide in the blood also has what in the blood? a. Large amount of carbonic acid and low hydrogen ion concentration b. Small amount of carbonic acid and low hydrogen ion concentration c. Large amount of carbonic acid and high hydrogen ion concentration d. Small amount of carbonic acid and high hydrogen ion concentration

c. Large amount of carbonic acid and high hydrogen ion concentration The amount of CO2 in the blood directly relates to carbonic acid concentration and subsequently hydrogen ion concentration. The CO2 combines with water in the blood to form carbonic acid and in cases in which CO2 is retained in the blood, acidosis occurs.

What phrase describes transmyocardial laser revascularization (TMR)? a. Structure applied to hold vessels open b. Requires anticoagulation following the procedure c. Laser-created channels in the heart muscle to allow blood flow to ischemic areas d. Surgical construction of new vessels to carry blood beyond obstructed coronary artery

c. Laser-created channels in the heart muscle to allow blood flow to ischemic areas Transmyocardial laser revascularization (TMR) is a treatment used for patients with inoperable CAD. It uses a high-energy laser to create channels in the heart to allow blood to flow to the ischemic area and can be done using a left anterior thoracotomy incision or with coronary artery bypass graft (CABG) surgery. A stent is the structure used to hold vessels open and requires anticoagulation following the procedure. Surgical construction of new vessels is done with CABG surgery

Collaboration: A registered nurse (RN) is working with a licensed practical nurse/licensed vocational nurse (LPN/VN) in caring for a group of patients on a cardiac telemetry unit. Which nursing activity could be delegated to the LPN? a. Explain the reason for planning frequent periods of rest. b. Evaluate the patient's understanding of his disease process. c. Monitor BP, heart rate (HR), respiratory rate (RR), and arterial oxygen saturation by pulse oximetry (SpO2 ) before, during, and after ambulation. d. Teach the patient which activities to choose that will gradually increase endurance.

c. Monitor BP, heart rate (HR), respiratory rate (RR), and arterial oxygen saturation by pulse oximetry (SpO2 ) before, during, and after ambulation. Monitoring vital signs before and after ambulation is the collection of data. Instructions should be provided to the LPN about what changes in these vital signs should be reported to the RN. The other actions listed are RN responsibilities

What accurately describes electrocardiographic (ECG) monitoring? a. Depolarization of the cells in the ventricles produces the T wave on the ECG. b. An abnormal cardiac impulse that arises in the atria, ventricles, or atrioventricular (AV) junction can create a premature beat that is known as "an artifact." c. Lead placement for V1 includes 1 lead each for right arm, right leg, left arm, and left leg with the fifth lead on the fourth intercostal space to the right of the sternal border. d. If the sinoatrial (SA) node fails to discharge an impulse or discharges very slowly, a secondary pacemaker in the AV node is able to discharge at a rate of 30 to 40 times per minute.

c. Lead placement for V1 includes 1 lead each for right arm, right leg, left arm, and left leg with the fifth lead on the fourth intercostal space to the right of the sternal border. The V1 leads are placed toward each limb and centrally at the fourth intercostal space to the right of the sternal border. Depolarization of the ventricular cells produces the QRS interval on the electrocardiogram (ECG). The T wave is produced by repolarization of the ventricular cells. Abnormal cardiac impulses from the atria, ventricles, or atrioventricular (AV) junction create ectopic beats. Artifacts are seen with leads or electrodes that are not secure, with muscle activity or electrical interference. The rate produced by the AV node pacing in a junctional escape rhythm is 40 to 60 bpm. If the His-Purkinje system is blocked, the heart rate is 20 to 40 bpm

Which arteries are the major providers of coronary circulation (select all that apply)? a. Left marginal artery b. Right marginal artery c. Left circumflex artery d. Right coronary artery e. Posterior descending artery f. Left anterior descending artery

c. Left circumflex artery d. Right coronary artery f. Left anterior descending artery The left circumflex and left anterior descending arteries branch from the left coronary artery. The left coronary artery and right coronary artery arise from the aorta to supply the atria, ventricles, and interventricular septum.

A 21-year-old woman is scheduled for percutaneous transluminal balloon valvuloplasty to treat mitral stenosis. Which information should the nurse include when explaining the advantages of valvuloplasty over valve replacement to the patient? a. Biologic valves will require immunosuppressive drugs after surgery. b. Mechanical mitral valves need to be replaced sooner than biologic valves. c. Lifelong anticoagulant therapy will be needed after mechanical valve replacement. d. Ongoing cardiac care by a health care provider is not necessary after valvuloplasty.

c. Lifelong anticoagulant therapy will be needed after mechanical valve replacement. Long-term anticoagulation therapy is needed after mechanical valve replacement, and this would restrict decisions about career and childbearing in this patient. Mechanical valves are durable and last longer than biologic valves. All valve repair procedures are palliative, not curative, and require lifelong health care. Biologic valves do not activate the immune system, and immunosuppressive therapy is not needed.

A pediatric clinic nurse is providing teaching to a parent whose 11-month-old has iron deficiency anemia. What might the nurse suggest to increase the client's iron levels? a. Consideration of a blood transfusion. b. Encourage intake of red meat. c. Limit excessive milk intake. d. Start the child on an iron supplement.

c. Limit excessive milk intake. This answer is correct because by limiting excessive milk intake, the child will be hungry for food. This is when the parent can encourage oral intake of iron-rich foods. Also encourage foods high in vitamin C since it enhances iron absorption within the body.

A patient with a venous thromboembolism (VTE) is started on enoxaparin (Lovenox) and warfarin (Coumadin). The patient asks the nurse why two medications are necessary. Which response by the nurse is most appropriate? a. Taking two blood thinners reduces the risk for another clot to form. b. Lovenox will start to dissolve the clot, and Coumadin will prevent any more clots from forming. c. Lovenox will work right away, but Coumadin takes several days to have an effect on preventing clots. d. Because of the risk for a blood clot in the lungs, it is important for you to take more than one blood thinner.

c. Lovenox will work right away, but Coumadin takes several days to have an effect on preventing clots. Low molecular weight heparin (LMWH) is used because of the immediate effect on coagulation and discontinued once the international normalized ratio (INR) value indicates that the warfarin has reached a therapeutic level. LMWH has no thrombolytic properties. The use of two anticoagulants is not related to the risk for pulmonary embolism, and two are not necessary to reduce the risk for another VTE. Furthermore, anticoagulants should not be described as blood thinners.

A second 12-lead ECG performed on a patient 4 hours after the onset of chest pain reveals ST segment elevation. What does the nurse recognize that this finding indicates? a. Transient ischemia typical of unstable angina b. Lack of permanent damage to myocardial cells c. MI associated with prolonged and complete coronary thrombosis d. MI associated with transient or incomplete coronary artery occlusion

c. MI associated with prolonged and complete coronary thrombosis A differentiation is made between MIs that have ST-segment elevations on ECG and those that do not because chest pain accompanied by ST-segment elevations is associated with prolonged and complete coronary thrombosis and is treated with reperfusion therapy. The other options are incorrect

A nurse is assessing a newly admitted patient with chronic heart failure who forgot to take prescribed medications and seems confused. The patient complains of just blowing up and has peripheral edema and shortness of breath. Which assessment should the nurse complete first? a. Skin turgor b. Heart sounds c. Mental status d. Capillary refill

c. Mental status Increases in extracellular fluid (ECF) can lead to swelling of cells in the central nervous system, initially causing confusion, which may progress to coma or seizures. Although skin turgor, capillary refill, and heart sounds also may be affected by increases in ECF, these are signs that do not have as immediate impact on patient outcomes as cerebral edema.

The nurse is caring for a patient newly admitted with heart failure secondary to dilated cardiomyopathy. Which intervention would be a priority? a. Encourage caregivers to learn CPR. b. Consider a consultation with hospice for palliative care. c. Monitor the patient's response to prescribed medications. d. Arrange for the patient to enter a cardiac rehabilitation program.

c. Monitor the patient's response to prescribed medications.

A patient who is receiving dobutamine (Dobutrex) for the treatment of acute decompensated heart failure (ADHF) has the following nursing interventions included in the plan of care. Which action will be most appropriate for the registered nurse (RN) to delegate to an experienced licensed practical/vocational nurse (LPN/LVN)? a. Assess the IV insertion site for signs of extravasation. b. Teach the patient the reasons for remaining on bed rest. c. Monitor the patients blood pressure and heart rate every hour. d. Titrate the rate to keep the systolic blood pressure >90 mm Hg.

c. Monitor the patients blood pressure and heart rate every hour. An experienced LPN/LVN would be able to monitor BP and heart rate and would know to report significant changes to the RN. Teaching patients, making adjustments to the drip rate for vasoactive medications, and monitoring for serious complications such as extravasation require RN level education and scope of practice.

Priority Decision: A patient is admitted to the hospital for evaluation and treatment of thrombocytopenia. Which action is most important for the nurse to implement? a. Taking the temperature every 4 hours to assess for fever b. Maintaining the patient on strict bed rest to prevent injury c. Monitoring the patient for headaches, vertigo, or confusion d. Removing the oral crusting and scabs with a soft brush 4 times a day

c. Monitoring the patient for headaches, vertigo, or confusion The major complication of thrombocytopenia is hemorrhage, and it may occur in any area of the body. Cerebral hemorrhage may be fatal, and evaluation of mental status for central nervous system (CNS) alterations to identify CNS bleeding is very important. Fever is not a common finding in thrombocytopenia. Protection from injury to prevent bleeding is an important nursing intervention, but strict bed rest is not indicated. Oral care is performed very gently with minimum friction and soft swabs.

Which descriptions are characteristic of iron-deficiency anemia (select all that apply)? a. Lack of intrinsic factor b. Autoimmune-related disease c. Most common type of anemia d. Associated with chronic blood loss e. May occur with removal of the stomach f. May occur with removal of the duodenum

c. Most common type of anemia d. Associated with chronic blood loss f. May occur with removal of the duodenum Iron-deficiency anemia is the most common type of anemia and occurs with chronic blood loss or malabsorption in the duodenum so it may occur with duodenal removal. The other options are associated with cobalamin deficiency.

Which respiratory defense mechanism is most impaired by smoking? a. Cough reflex b. Filtration of air c. Mucociliary clearance d. Reflex bronchoconstriction

c. Mucociliary clearance Ciliary action impaired by smoking and increased mucus production may be caused by the irritants in tobacco smoke, leading to impairment of the mucociliary clearance system. Smoking does not directly affect filtration of air, the cough reflex, or reflex bronchoconstriction, but it does impair the respiratory defense mechanism provided by alveolar macrophages.

An older patient receiving iso-osmolar continuous tube feedings develops restlessness, agitation, and weakness. Which laboratory result should the nurse report to the health care provider immediately? a. K+ 3.4 mEq/L (3.4 mmol/L) b. Ca+2 7.8 mg/dL (1.95 mmol/L) c. Na+ 154 mEq/L (154 mmol/L) d. PO4-3 4.8 mg/dL (1.55 mmol/L)

c. Na+ 154 mEq/L (154 mmol/L) The elevated serum sodium level is consistent with the patients' neurologic symptoms and indicates a need for immediate action to prevent further serious complications such as seizures. The potassium and calcium levels vary slightly from normal but do not require immediate action by the nurse. The phosphate level is normal.

Which assessment finding for a patient who has been admitted with a right calf venous thromboembolism (VTE) requires immediate action by the nurse? a. Erythema of right lower leg b. Complaint of right calf pain c. New onset shortness of breath d. Temperature of 100.4 F (38 C)

c. New onset shortness of breath New onset dyspnea suggests a pulmonary embolus, which will require rapid actions such as oxygen administration and notification of the health care provider. The other findings are typical of VTE.

The nurse assesses a patient with pernicious anemia. Which assessment finding would the nurse expect? a. Yellow-tinged sclerae b. Shiny, smooth tongue c. Numbness of the extremities d. Gum bleeding and tenderness

c. Numbness of the extremities Extremity numbness is associated with cobalamin (vitamin B12) deficiency or pernicious anemia. Loss of the papillae of the tongue occurs with chronic iron deficiency. Yellow-tinged sclera is associated with hemolytic anemia and the resulting jaundice. Gum bleeding and tenderness occur with thrombocytopenia or neutropenia.

Which nursing action can the registered nurse (RN) delegate to experienced unlicensed assistive personnel (UAP) working as a telemetry technician on the cardiac care unit? a. Decide whether a patients heart rate of 116 requires urgent treatment. b. Monitor a patients level of consciousness during synchronized cardioversion. c. Observe cardiac rhythms for multiple patients who have telemetry monitoring. d. Select the best lead for monitoring a patient admitted with acute coronary syndrome.

c. Observe cardiac rhythms for multiple patients who have telemetry monitoring. UAP serving as telemetry technicians can monitor cardiac rhythms for individuals or groups of patients. Nursing actions such as assessment and choice of the most appropriate lead based on ST segment elevation location require RN-level education and scope of practice.

The nurse is caring for a patient with pneumonia unresponsive to two different antibiotics. Which action is most important for the nurse to complete before administering a newly prescribed antibiotic? a. Teach the patient to cough and deep breathe. b. Take the temperature, pulse, and respiratory rate. c. Obtain a sputum specimen for culture and Gram stain. d. Check the patient's oxygen saturation by pulse oximetry.

c. Obtain a sputum specimen for culture and Gram stain. A sputum specimen for culture and Gram stain to identify the organism should be obtained before beginning antibiotic therapy. However, antibiotic administration should not be delayed if a specimen cannot be readily obtained because delays in antibiotic therapy can increase morbidity and mortality risks.

The nurse will plan discharge teaching about the need for prophylactic antibiotics when having dental procedures for which patient? a. Patient admitted with a large acute myocardial infarction. b. Patient being discharged after an exacerbation of heart failure. c. Patient who had a mitral valve replacement with a mechanical valve. d. Patient being treated for rheumatic fever after a streptococcal infection.

c. Patient who had a mitral valve replacement with a mechanical valve. Current American Heart Association guidelines recommend the use of prophylactic antibiotics before dental procedures for patients with prosthetic valves to prevent infective endocarditis (IE). The other patients are not at risk for IE.

A patient who is complaining of a racing heart and feeling anxious comes to the emergency department. The nurse places the patient on a heart monitor and obtains the following electrocardiographic (ECG) tracing. Which action should the nurse take next? a. Prepare to perform electrical cardioversion. b. Have the patient perform the Valsalva maneuver. c. Obtain the patients vital signs including oxygen saturation. d. Prepare to give a b-blocker medication to slow the heart rate.

c. Obtain the patients vital signs including oxygen saturation. The patient has sinus tachycardia, which may have multiple etiologies such as pain, dehydration, anxiety, and myocardial ischemia. Further assessment is needed before determining the treatment. Vagal stimulation or b-blockade may be used after further assessment of the patient. Electrical cardioversion is used for some tachydysrhythmias, but would not be used for sinus tachycardia.

A patient with chronic obstructive pulmonary disease (COPD) has a nursing diagnosis of imbalanced nutrition: less than body requirements. Which intervention would be most appropriate for the nurse to include in the plan of care? a. Encourage increased intake of whole grains. b. Increase the patients intake of fruits and fruit juices. c. Offer high-calorie snacks between meals and at bedtime. d. Assist the patient in choosing foods with high vegetable and mineral content.

c. Offer high-calorie snacks between meals and at bedtime. Eating small amounts more frequently (as occurs with snacking) will increase caloric intake by decreasing the fatigue and feelings of fullness associated with large meals. Patients with COPD should rest before meals. Foods that have a lot of texture like whole grains may take more energy to eat and get absorbed and lead to decreased intake. Although fruits, juices, and vegetables are not contraindicated, foods high in protein are a better choice.

Which patients should the nurse expect to be scheduled for percutaneous transluminal balloon valvuloplasty? a. Any patient with aortic regurgitation b. Older patients with aortic regurgitation c. Older patients with stenosis of any valve d. Young adult patients with mild mitral valve stenosis

c. Older patients with stenosis of any valve This procedure has been used for repair of mitral, tricuspid, and pulmonic stenosis and less often for aortic stenosis. It is usually used for older patients and for those patients who are poor surgical risks because it is relatively easy and has good results and few complications.

The client who has undergone an exploratory laparotomy and subsequent removal of a large intestinal tumor has a nasogastric tube in place and an IV running at 150mL/hr via an IV pump. Which data should be reported to the HCP? a.The pump keeps sounding an alarm indicating the high pressure has been reached. b.Intake is 1800 mL, NG output is 550mL, and foley output is 950mL. c.On auscultation, crackles and rhonchi in all lung fields are noted. d.Client has negative pedal edema and an increasing level of consciousness.

c. On auscultation, crackles and rhonchi in all lung fields are noted. Crackles & rhonchi in all lung fields indicate the body is not able to process the amount of fluid being infused. This should be brought to the HCP's attention.

A patient recovering from heart surgery develops pericarditis and complains of level 6 (0 to 10 scale) chest pain with deep breathing. Which ordered PRN medication will be the most appropriate for the nurse to give? a. Fentanyl 1 mg IV b. IV morphine sulfate 4 mg c. Oral ibuprofen (Motrin) 600 mg d. Oral acetaminophen (Tylenol) 650 mg

c. Oral ibuprofen (Motrin) 600 mg The pain associated with pericarditis is caused by inflammation, so nonsteroidal antiinflammatory drugs (NSAIDs) (e.g., ibuprofen) are most effective. Opioid analgesics are usually not used for the pain associated with pericarditis.

A nurse is assessing a client who has right side heart failure. Which of the following findings should the nurse expect? a. Decreased cap refill b. Dyspnea c. Orthopnea, dependent edema

c. Orthopnea, dependent edema

The patient's laboratory results show a marked decrease in RBCs, WBCs, and platelets. What term should the nurse use when reporting the results to the HCP? a. Hemolysis b. Leukopenia c. Pancytopenia d. Thrombocytosis

c. Pancytopenia Pancytopenia is decreased RBCs, WBCs, and platelets. Hemolysis is RBC destruction. Leukopenia is WBC < 5000/μL. Thrombocytosis is increased platelets, and thrombocytopenia is decreased platelets.

During the assessment of a patient with cobalamin deficiency, what manifestation would the nurse expect to find in the patient? a. Icteric sclera b. Hepatomegaly c. Paresthesia of the hands and feet d. Intermittent heartburn with acid reflux

c. Paresthesia of the hands and feet Neurologic manifestations of weakness, paresthesia of the feet and hands, and impaired thought processes are characteristic of cobalamin deficiency and pernicious anemia. Hepatomegaly and jaundice often occur with hemolytic anemia. The patient with cobalamin deficiency often has achlorhydria or decreased stomach acidity and would not experience effects of gastric hyperacidity.

When reviewing the results of an 83-year-old patient's diagnostic studies, which finding would be of most concern to the nurse? a. Platelets 150,000/µL b. Serum iron 50 mcg/dL . c. Partial thromboplastin time (PTT) 60 seconds d. Erythrocyte sedimentation rate (ESR) 35 mm in 1 hour

c. Partial thromboplastin time (PTT) 60 seconds As a person ages the partial thromboplastin time (PTT) is normally decreased, so an abnormally high PTT of 60 seconds is an indication that bleeding could readily occur. Platelets are unaffected by aging and 150,000/μL is a normal count. Serum iron levels are decreased and the erythrocyte sedimentation rate (ESR) is significantly increased with aging, as are reflected in these values.

Pulse oximetry may not be a reliable indicator of oxygen saturation in which patient? a. Patient with a fever b. Patient who is anesthetized c. Patient in hypovolemic shock d. Patient receiving oxygen therapy

c. Patient in hypovolemic shock Poor peripheral perfusion that occurs with hypovolemia or other conditions that cause peripheral vasoconstriction will cause inaccurate pulse oximetry, and ABGs may have to be used to monitor oxygenation status and ventilation status in these patients. Pulse oximetry would not be affected by fever or anesthesia and is a method of monitoring arterial oxygen saturation in patients who are receiving oxygen therapy.

A nurse is reviewing the medical record of a client who had diabetes mellitus and is recieving regular insulin by continous IV infustion to treat diabetic ketoacidosis. Which of the following findings should the nurse report to the provider? a. Urine output of 30 mL/hr b. Blood glucose of 180 mg/dL c. Serum potassium 3.0 mEq/L d. BUN 18 mg/dL

c. Serum potassium 3.0 mEq/L This serum potassium level is outside the expected reference range. The nurse should report this finding to the provider.

The nurse is reviewing the laboratory results for newly admitted patients on the cardiovascular unit. Which patient laboratory result is most important to communicate as soon as possible to the health care provider? a. Patient whose triglyceride level is high b. Patient who has very low homocysteine level c. Patient with increase in troponin T and troponin I level d. Patient with elevated high-sensitivity C-reactive protein level

c. Patient with increase in troponin T and troponin I level The elevation in troponin T and I indicates that the patient has had an acute myocardial infarction. Further assessment and interventions are indicated. The other laboratory results are indicative of increased risk for coronary artery disease but are not associated with acute cardiac problems that need immediate intervention

After receiving change-of-shift report, which patient should the nurse assess first? a. Patient with serum potassium level of 5.0 mEq/L who is complaining of abdominal cramping b. Patient with serum sodium level of 145 mEq/L who has a dry mouth and is asking for a glass of water c. Patient with serum magnesium level of 1.1 mEq/L who has tremors and hyperactive deep tendon reflexes d. Patient with serum phosphorus level of 4.5 mg/dL who has multiple soft tissue calcium-phosphate precipitates

c. Patient with serum magnesium level of 1.1 mEq/L who has tremors and hyperactive deep tendon reflexes The low magnesium level and neuromuscular irritability suggest that the patient may be at risk for seizures. The other patients have mild electrolyte disturbances and/or symptoms that require action, but they are not at risk for life-threatening complications.

The abnormal assessment findings of dullness and hyperresonance are found with which assessment technique? a. Inspection b. Palpation c. Percussion d. Auscultation

c. Percussion Dullness and hyperresonance are found in the lungs using percussion, not the other assessment techniques.

Behavior changes, paresthesia, ataxia and poor memory are symptoms of: a. Hemolytic anemia b. Aplastic anemia c. Pernicious anemia d. Thalassemia

c. Pernicious anemia

A pt. came in when she started to feel dizzy 1 hr ago. Vitals:BP-130/77, HR-155, RR-22, Sats-95%. Which should be FIRST? a. Give Adenosine over 2secs with 20ml saline flush. b. Give sedation and pain meds in anticipation of synchronized cardioversion. c. Place 2 L NC on the pt. d. Have pt. perform the Valsalva maneuver.

c. Place 2 L NC on the pt.

A patient in metabolic alkalosis is admitted to the emergency department, and pulse oximetry (SpO2) indicates that the O2 saturation is 94%. Which action should the nurse take next? a. Administer bicarbonate. b. Complete a head-to-toe assessment. c. Place the patient on high-flow oxygen. d. Obtain repeat arterial blood gases (ABGs).

c. Place the patient on high-flow oxygen. Although the O2 saturation is adequate, the left shift in the oxyhemoglobin dissociation curve will decrease the amount of oxygen delivered to tissues, so high oxygen concentrations should be given. Bicarbonate would worsen the patients condition. A head-to-toe assessment and repeat ABGs may be implemented. However, the priority intervention is to give high-flow oxygen.

How does the nurse assess the patient's chest expansion? a. Put the palms of the hands against the chest wall. b. Put the index fingers on either side of the trachea. c. Place the thumbs at the midline of the lower chest. d. Place 1 hand on the lower anterior chest and 1 hand on the upper abdomen

c. Place the thumbs at the midline of the lower chest. To assess the extent and symmetry of chest movement, the nurse places the hands over the lower anterior chest wall along the costal margin and moves them inward until the thumbs meet at the midline and then asks the patient to breathe deeply and observes the movement of the thumbs away from each other. The palms are placed against the chest wall to assess tactile fremitus. To determine the tracheal position, the nurse places the index fingers on either side of the trachea just above the suprasternal notch and gently presses backward.

As fluid circulates through the capillaries, there is movement of fluid between the capillaries and interstitium. What describes the fluid movement that would cause edema (select all that apply)? a. Plasma hydrostatic pressure is less than plasma oncotic pressure. b. Plasma oncotic pressure is higher than interstitial oncotic pressure. c. Plasma hydrostatic pressure is higher than plasma oncotic pressure. d. Plasma hydrostatic pressure is less than interstitial hydrostatic pressure. e. Interstitial hydrostatic pressure is lower than plasma hydrostatic pressure.

c. Plasma hydrostatic pressure is higher than plasma oncotic pressure. e. Interstitial hydrostatic pressure is lower than plasma hydrostatic pressure. At the arterial end of the capillary, capillary hydrostatic pressure exceeds plasma oncotic pressure and fluid moves into the interstitial space. At the capillary level, hydrostatic pressure is the major force causing fluid to shift from vascular to the interstitial space. The other options would not cause edema.

A patient is being treated with chemotherapy. The nurse revises the patient's care plan based on which result? a. WBC count 4000/µL b. RBC count 4.3 × 10 6 /µL c. Platelets 50,000/µL d. Hematocrit (Hct) 39%

c. Platelets 50,000/µL Any platelet count < 150,000/μL is considered thrombocytopenia and could place the patient at risk for bleeding, necessitating special consideration in nursing care. Chemotherapy may cause bone marrow suppression and a depletion of all blood cells. The other factors are all within normal range.

The nurse is evaluating the telemetry ECG rhythm strip. How should the nurse document the distorted P wave causing an irregular rhythm? a. Atrial flutter b. Sinus bradycardia c. Premature atrial contraction (PAC) d. Paroxysmal supraventricular tachycardia (PSVT)

c. Premature atrial contraction (PAC) The premature atrial contraction (PAC) has a distorted P wave that may feel like a skipped beat to the patient. Atrial flutter is an atrial tachydysrhythmia with recurring, regular, saw-toothed flutter waves from the same focus in the right or possibly left atrium. Sinus bradycardia has a regular heart rate <100 bpm. Paroxysmal supraventricular tachycardia (PSVT) starts in an ectopic focus above the bundle of His and may be triggered by PAC. If seen, the P wave may have an abnormal shape and has a spontaneous start and termination with a rate of 150 to 220 bpm

A patient newly diagnosed with asthma is being discharged. The nurse anticipates including which topic in the discharge teaching? a. Use of long-acting b-adrenergic medications b. Side effects of sustained-release theophylline c. Self-administration of inhaled corticosteroids d. Complications associated with oxygen therapy

c. Self-administration of inhaled corticosteroids Inhaled corticosteroids are more effective in improving asthma than any other drug and are indicated for all patients with persistent asthma. The other therapies would not typically be first-line treatments for newly diagnosed asthma.

Priority Decision: A patient with an acute MI is having multifocal PVCs and couplets. He is alert and has a BP reading of 118/78 mm Hg with an irregular pulse of 86 bpm. What is the priority nursing action at this time? a. Continue to assess the patient. b. Ask the patient to perform Valsalva maneuver. c. Prepare to administer antidysrhythmic drugs per protocol. d. Be prepared to administer cardiopulmonary resuscitation (CPR).

c. Prepare to administer antidysrhythmic drugs per protocol. Multifocal PVCs in a patient with an MI indicate significant ventricular irritability that may lead to ventricular tachycardia or ventricular fibrillation. Preparing to administer antidysrhythmics, such as β-adrenergic blockers, procainamide, amiodarone, or lidocaine, is the priority to control the dysrhythmias. Valsalva maneuver may be used to treat PSVT. The nurse must always be ready to perform cardiopulmonary resuscitation (CPR), but drugs may prevent this need.

A nurse cares for a client who has a serum potassium of 7.5 mEq/L and is exhibiting cardiovascular changes. Which prescription should the nurse implement first? a. Prepare to administer sodium polystyrene sulfate (Kayexalate) 15 g by mouth. b. Provide a heart healthy, low-potassium diet. c. Prepare to administer dextrose 20% and 10 units of regular insulin IV push. d. Prepare the client for hemodialysis treatment.

c. Prepare to administer dextrose 20% and 10 units of regular insulin IV push. A client with a high serum potassium level and cardiac changes should be treated immediately to reduce the extracellular potassium level. Potassium movement into the cells is enhanced by insulin by increasing the activity of sodium-potassium pumps. Insulin will decrease both serum potassium and glucose levels and therefore should be administered with dextrose to prevent hypoglycemia. Kayexalate may be ordered, but this therapy may take hours to reduce potassium levels. Dialysis may also be needed, but this treatment will take much longer to implement and is not the first prescription the nurse should implement. Decreasing potassium intake may help prevent hyperkalemia in the future but will not decrease the client's current potassium level.

A surgical repair is planned for a patient who has a 5.5-cm abdominal aortic aneurysm (AAA). On physical assessment of the patient, what should the nurse expect to find? a. Hoarseness and dysphagia b. Severe back pain with flank ecchymosis c. Presence of a bruit in the periumbilical area d. Weakness in the lower extremities progressing to paraplegia

c. Presence of a bruit in the periumbilical area Although most abdominal aortic aneurysms (AAAs) are asymptomatic, on physical examination a pulsatile mass in the periumbilical area slightly to the left of the midline may be detected and bruits may be audible with a stethoscope placed over the aneurysm. Hoarseness and dysphagia may occur with aneurysms of the ascending aorta and the aortic arch. Severe back pain with flank ecchymosis is usually present on rupture of an AAA and neurovascular loss in the lower extremities may occur from pressure of a thoracic aneurysm.

What types of angina can occur in the absence of CAD (select all that apply)? a. Silent ischemia b. Nocturnal angina c. Prinzmetal's angina d. Microvascular angina e. Chronic stable angina

c. Prinzmetal's angina d. Microvascular angina Prinzmetal's angina and microvascular angina may occur in the absence of CAD but with arterial spasm in Prinzmetal's angina or abnormalities of the coronary microcirculation. Silent ischemia is prevalent in persons with diabetes and contributes to asymptomatic myocardial ischemia. Nocturnal angina occurs only at night. Chronic stable angina refers to chest pain that occurs with the same pattern of onset, duration, and intensity intermittently over a long period of time.

What is the pathophysiologic mechanism of cystic fibrosis leading to obstructive lung disease? a. Fibrosis of mucous glands and destruction of bronchial walls b. Destruction of lung parenchyma from inflammation and scarring c. Production of secretions low in sodium chloride and resulting thickened mucus d. Increased serum levels of pancreatic enzymes that are deposited in the bronchial mucosa

c. Production of secretions low in sodium chloride and resulting thickened mucus Cystic fibrosis (CF) is an autosomal recessive, multisystem disease involving gene mutations that make secretions of the lungs, pancreas, intestines low in sodium chloride and thus water, so they are abnormally thick and sticky. This leads to a chronic, diffuse, obstructive pulmonary disorder in almost all patients. Exocrine pancreatic insufficiency occurs in about 85% to 90% of patients with CF. Fibrosis occurs in the subepithelium of the lungs and pancreas, which plugs the exocrine ducts. Bronchiectasis occurs when bronchial walls are changed.

A nurse is assessing a client who is using PCA following a thoracotomy. The client is short of breath, appears restless, and has a respiratory rate of 28/min. The client's ABG results are pH 7.52, PoO2 89 mm hg, and HCO3- 24 mEq/L. Which of the following actions should the nurse take? a. Instruct the client to cough forcefully. b. Assist the client with ambulation. c. Provide calming interventions. d. Discontinue the PCA.

c. Provide calming interventions The client's respiratory rate is above the expected range. Calming the client should decrease the respiratory rate, which will cause the client's carbon dioxide levels to increase. This will help correct the pH imbalance.

A murmur is heard at the second left intercostal space along the left sternal border. Which valve is this? a. Aortic b. Mitral c. Pulmonic d. Tricuspid

c. Pulmonic

Which breathing technique should the nurse teach the patient with moderate COPD to promote exhalation? a. Huff coughing b. Thoracic breathing c. Pursed lip breathing d. Diaphragmatic breathing

c. Pursed lip breathing Pursed lip breathing prolongs exhalation and prevents bronchiolar collapse and air trapping. Huff coughing is a technique used to increase coughing patterns to remove secretions. Thoracic breathing is not as effective as diaphragmatic breathing and is the method most naturally used by patients with COPD. Diaphragmatic breathing emphasizes the use of the diaphragm to increase maximum inhalation, but it may increase the work of breathing and dyspnea.

The nurse is preparing the patient for and will assist the health care provider with a thoracentesis in the patient's room. Which of the following is the last action? a. Verify breath sounds in all fields. b. Obtain the supplies that will be used. c. Send labeled specimen containers to the laboratory. d. Direct the family members to the waiting room. e. Observe for signs of hypoxia during the procedure. f. Instruct the patient not to talk during the procedure. g. Position the patient sitting upright with the elbows on an over-the bed table.

c. Send labeled specimen containers to the laboratory. This is done before sending the sample to the laboratory if there is no one else who can send the sample to the laboratory.

A nurse who is caring for patient with a tracheostomy tube in place has just auscultated rhonchi bilaterally. If the patient is unsuccessful in coughing up secretions, what action should the nurse take? a. Encourage increased incentive spirometer use. b. Encourage the patient to increase oral fluid intake. c. Put on sterile gloves and use a sterile catheter to suction. d. Preoxygenate the patient for 3 minutes before suctioning.

c. Put on sterile gloves and use a sterile catheter to suction. This patient needs suctioning now to secure a patent airway. Sterile gloves and a sterile catheter are used when suctioning a tracheostomy. Preoxygenation for 3 minutes is not necessary. Incentive spirometer (IS) use opens alveoli and can induce coughing, which can mobilize secretions. However, the patient with a tracheostomy may not be able to use an incentive spirometer. Increasing oral fluid intake would not moisten and help mobilize secretions in a timely manner.

A 63 year old client with history of asthma is prescribed Inderal (propanol) to control hypertension. Before administered it, which of the following actions should the nurse take first? a. Monitor the apical pulse rate b. Instruct patient to take it food c. Question the provider about the order d. Caution the patient to rise slowly when standing.

c. Question the provider about the order

Which function test fits description "Amount of air remaining in lungs after forced expiration"? a. FVC b. PEFR c. RV d. FRC

c. RV Residual volume

A 32-yr-old woman is prescribed diltiazem (Cardizem) for Raynaud's phenomenon. To evaluate the effectiveness of the medication, which assessment will the nurse perform? a Improved skin turgor b Decreased cardiac rate c Improved finger perfusion d Decreased mean arterial pressure

c. Raynaud's phenomenon is an episodic vasospastic disorder of small cutaneous arteries, most frequently involving the fingers and toes. Diltiazem (Cardizem) is a calcium channel blocker that relaxes smooth muscles of the arterioles by blocking the influx of calcium into the cells, thus reducing the frequency and severity of vasospastic attacks. Perfusion to the fingertips is improved, and vasospastic attacks are reduced. Diltiazem may decrease heart rate and blood pressure, but that is not the purpose in Raynaud's phenomenon. Skin turgor is most often a reflection of hydration status.

Collaboration: The nursing care area is very busy with new surgical patients. Which care could the registered nurse (RN) delegate to the unlicensed assistive personnel (UAP) for a patient with VTE? a. Assess the patient's use of herbs. b. Measure the patient for elastic compression stockings. c. Remind the patient to flex and extend the legs and feet every 2 hours. d. Teach the patient to call emergency response system with signs of pulmonary embolus

c. Remind the patient to flex and extend the legs and feet every 2 hours. The RN could delegate to the UAP the task to remind the patient to flex and extend the legs and feet every 2 hours while in bed. Measuring for elastic compression stockings may be delegated to the LPN. The RN must assess and teach the patient.

An increase in which blood cell indicates an increased rate of erythropoiesis? a. Basophil b. Monocyte c. Reticulocyte d. Lymphocyte

c. Reticulocyte Increased reticulocytes, or immature red blood cells (RBCs), indicate an increased rate of erythropoiesis or stimulation of erythrocyte (RBC) production by the bone marrow. Basophils are stimulated by granulocyte colony-stimulating factor in response to an antigen or by tissue injury. Monocytes and lymphocytes respond to tissue injury, including infection

A patient with a tricuspid valve disorder will have impaired blood flow between the a. Vena cava and right atrium b. Left atrium and left ventricle c. Right atrium and right ventricle

c. Right atrium and right ventricle

When a patient reports chest pain, why must unstable angina be identified and rapidly treated? a. The pain may be severe and disabling. b. Electrocardiogram (ECG) changes and dysrhythmias may occur during an attack. c. Rupture of unstable plaque may cause complete thrombosis of the vessel lumen. d. Spasm of a major coronary artery may cause total occlusion of the vessel with progression to MI.

c. Rupture of unstable plaque may cause complete thrombosis of the vessel lumen. Unstable angina is associated with the rupture of a once-stable atherosclerotic plaque, exposing the intima to blood and stimulating platelet aggregation and local vasoconstriction with thrombus formation. Patients with unstable angina need immediate hospitalization and monitoring because the lesion is at increased risk of complete thrombosis of the lumen with progression to MI. Any type of angina may be associated with severe pain, electrocardiogram (ECG) changes, and dysrhythmias. Prinzmetal's angina is characterized by coronary artery spasm.

A patient with chest pain that is unrelieved by nitroglycerin is admitted to the coronary care unit for observation and diagnosis. While the patient has continuous ECG monitoring, what finding would most concern the nurse? a. Occasional PVCs b. QRS complex change c. ST segment elevation d. A PR interval of 0.18 second

c. ST segment elevation ST segment elevation indicates injury or infarction of an area of the heart. A widened QRS occurs with second-degree AV block, type II and antidysrhythmic drugs. Occasional PVCs may be normal or may be the result of electrolyte imbalance or hypoxia. They require continued observation. A PR interval of 0.18 second is within normal range.

Which electrocardiographic (ECG) change is most important for the nurse to report to the health care provider when caring for a patient with chest pain? a. Inverted P wave b. Sinus tachycardia c. ST-segment elevation d. First-degree atrioventricular block

c. ST-segment elevation The patient is likely to be experiencing an ST-segment-elevation myocardial infarction (STEMI). Immediate therapy with percutaneous coronary intervention (PCI) or thrombolytic medication is indicated to minimize myocardial damage. The other ECG changes may also suggest a need for therapy, but not as rapidly.

When a patient with splenomegaly is scheduled for splenectomy, which action will the nurse include in the preoperative plan of care? a. Discourage deep breathing to reduce risk for splenic rupture. b. Teach the patient to use ibuprofen (Advil) for left upper quadrant pain. c. Schedule immunization with the pneumococcal vaccine (Pneumovax). d. Avoid the use of acetaminophen (Tylenol) for 2 weeks prior to surgery.

c. Schedule immunization with the pneumococcal vaccine (Pneumovax). Asplenic patients are at high risk for infection with Pneumococcus and immunization reduces this risk. There is no need to avoid acetaminophen use before surgery, but nonsteroidal anti-inflammatory drugs (NSAIDs) may increase bleeding risk and should be avoided. The enlarged spleen may decrease respiratory depth and the patient should be encouraged to take deep breaths.

A patient has recently started on digoxin (Lanoxin) in addition to furosemide (Lasix) and captopril (Capoten) for the management of heart failure. Which assessment finding by the home health nurse is a priority to communicate to the health care provider? a. Presence of 1 to 2+ edema in the feet and ankles b. Palpable liver edge 2 cm below the ribs on the right side c. Serum potassium level 3.0 mEq/L after 1 week of therapy d. Weight increase from 120 pounds to 122 pounds over 3 days

c. Serum potassium level 3.0 mEq/L after 1 week of therapy Hypokalemia can predispose the patient to life-threatening dysrhythmias (e.g., premature ventricular contractions), and potentiate the actions of digoxin and increase the risk for digoxin toxicity, which can also cause life-threatening dysrhythmias. The other data indicate that the patients heart failure requires more effective therapies, but they do not require nursing action as rapidly as the low serum potassium level.

A patient who has a small cell carcinoma of the lung develops syndrome of inappropriate antidiuretic hormone (SIADH). The nurse should notify the health care provider about which assessment finding? a. Reported weight gain b. Serum hematocrit of 42% c. Serum sodium level of 120 mg/dL d. Total urinary output of 280 mL during past 8 hours

c. Serum sodium level of 120 mg/dL Hyponatremia is the most important finding to report. SIADH causes water retention and a decrease in serum sodium level. Hyponatremia can cause confusion and other central nervous system effects. A critically low value likely needs to be treated. At least 30 mL/hr of urine output indicates adequate kidney function. The hematocrit level is normal. Weight gain is expected with SIADH because of water retention.

In the patient with chest pain, which results can distinguish unstable angina from an MI? a. ECG changes present at the onset of the pain b. A chest x-ray showing left ventricular hypertrophy c. Serum troponin levels increased 4 to 6 hours after the onset d. Creatine kinase MB (CK-MB) elevations that peak 6 hours after the infarct

c. Serum troponin levels increased 4 to 6 hours after the onset Cardiac-specific troponin T and troponin I have a greater sensitivity and specificity for myocardial injury than creatine kinase MB (CK-MB), are released 4 to 6 hours after the onset of MI, peak in 10 to 24 hours, and return to baseline over 10 to 14 days. CK-MB levels begin to rise 6 hours after an acute MI, peak in about 18 hours, and return to normal within 24 to 36 hours. ECG changes are often not apparent immediately after infarct and may be normal when the patient seeks medical attention. An enlarged heart, determined by x-ray, indicates cardiac stress but is not diagnostic of acute MI.

The registered nurse (RN) is caring for a patient with a hypertensive crisis who is receiving sodium nitroprusside (Nipride). Which nursing action can the nurse delegate to an experienced licensed practical/vocational nurse(LPN/LVN)? a. Titrate nitroprusside to decrease mean arterial pressure (MAP) to 115 mm Hg. b. Evaluate effectiveness of nitroprusside therapy on blood pressure (BP). c. Set up the automatic blood pressure machine to take BP every 15 minutes. d. Assess the patients environment for adverse stimuli that might increase BP.

c. Set up the automatic blood pressure machine to take BP every 15 minutes. LPN/LVN education and scope of practice include the correct use of common equipment such as automatic blood pressure machines. The other actions require advanced nursing judgment and education, and should be done by RNs.

During the nursing assessment of the patient with a type B aortic dissection, what should the nurse expect the patient to manifest? a. Altered level of consciousness (LOC) with dizziness and weak carotid pulses b. A cardiac murmur characteristic of aortic valve insufficiency c. Severe "ripping" back or abdominal pain with decreased urine output d. Severe hypertension and orthopnea and dyspnea of pulmonary edema

c. Severe "ripping" back or abdominal pain with decreased urine output A Type B aortic dissection involves the distal descending aorta and is usually characterized by a sudden, severe, tearing pain in the back. As it progresses down the aorta, the kidneys, abdominal organs, and lower extremities may begin to show evidence of ischemia. Type A aortic dissections of the ascending aorta and aortic arch may affect the heart and circulation to the head, with the development of cerebral ischemia, murmurs, ventricular failure, and pulmonary edema.

What should the nurse include when teaching the patient with COPD about the need for physical exercise? a. All patients with COPD should be able to increase walking gradually up to 20 minutes per day. b. A bronchodilator inhaler should be used to relieve exercise-induced dyspnea immediately after exercise. c. Shortness of breath is expected during exercise but should return to baseline within 5 minutes after the exercise. d. Monitoring the heart rate before and after exercise is the best way to determine how much exercise can be tolerated.

c. Shortness of breath is expected during exercise but should return to baseline within 5 minutes after the exercise. Shortness of breath usually increases during exercise, but the activity is not being overdone if breathing returns to baseline within 5 minutes after stopping. Bronchodilators can be administered 10 minutes before exercise but should not be administered for at least 5 minutes after activity to allow recovery. Patients are encouraged to walk 15 to 20 minutes per day with gradual increases, but actual patterns will depend on patient tolerance. Dyspnea most often limits exercise and is a better sign of exercise tolerance than is heart rate in the patient with COPD.

An elderly client with worsening COPD presents to the emergency department with fatigue and altered level of consciousness. Upon assessment the nurse finds O2 saturation of 87%, and ABG: pH 7.21, PaCO2 75, and PaO2 55 mm Hg. Which immediate intervention is best? a. Apply oxygen 4 LPM via nasal cannula. b. Call respiratory for STAT albuterol treatment. c. Sit the patient upright and apply Bilevel Positive Airway Pressure BiPAP d. Start looking for other jobs in cosmetic surgery

c. Sit the patient upright and apply Bilevel Positive Airway Pressure BiPAP HyperCAPnic (High CO2) = BiPAP

A nurse is assessing a client who has a phosphorus level of 2.4 mg/dL. Which of the following findings should the nurse expect? a. Hepatic failure b. Abdominal pain c. Slow peripheral pulses d. Increase in cardiac output

c. Slow peripheral pulses Hypophosphatemia causes slow peripheral pulses that are difficult to detect and can eventually result in cardiac muscle damage.

What accurately describes the alveolar sacs? a. Line the lung pleura b. Warm and moisturize inhaled air c. Terminal structures of the respiratory tract d. Contain dead air that is not available for gas exchange

c. Terminal structures of the respiratory tract Alveolar sacs are terminal structures of the respiratory tract, where gas exchange takes place. The visceral pleura lines the lungs and forms a closed, double-walled sac with the parietal pleura. Turbinates warm and moisturize inhaled air. The 150 mL of air is dead space in the trachea and bronchi.

The nurse is starting a client's 3rd unit of PRBCs. The client begins complaining of severe back pain, becomes apprehensive, and VS: T 100.9F, P 126, RR 28, BP 80/54. Which intervention should the nurse perform as priority? a. Administer tylenol and benadryl and continue the infusion. b. Slow the infusion because the client is in circulatory overload. c. Stop the infusion because the client is having a hemolytic reaction. d. Stop the infusion because the client is having an anaphylactic reaction.

c. Stop the infusion because the client is having a hemolytic reaction. This answer is correct because the client is having a hemolytic blood transfusion reaction and the nurse must stop the infusion immediately.

A patient admitted to the coronary care unit (CCU) with an ST-segment-elevation myocardial infarction (STEMI) is restless and anxious. The blood pressure is 86/40 and heart rate is 123. Based on this information, which nursing diagnosis is a priority for the patient? a. Acute pain related to myocardial infarction b. Anxiety related to perceived threat of death c. Stress overload related to acute change in health d. Decreased cardiac output related to cardiogenic shock

c. Stress overload related to acute change in health All the nursing diagnoses may be appropriate for this patient, but the hypotension and tachycardia indicate decreased cardiac output and shock from the damaged myocardium. This will result in decreased perfusion to all vital organs (e.g., brain, kidney, heart) and is a priority.

The nurse concludes that a client does not have an increased magnesium level based on which finding? a. Hypotension b. Bradycardia c. Supraventricular tachycardia (SVT) d. Flushing and sweating

c. Supraventricular tachycardia (SVT) SVT is seen in decreased mag levels, as are premature ventricular contractions and ventricular fibrillation.

The nurse is assigned to care for a patient in the emergency department admitted with an exacerbation of asthma. The patient has received a β-adrenergic bronchodilator and supplemental oxygen. If the patient's condition does not improve, the nurse should anticipate what as the most likely next step in treatment? a. IV fluids b. Biofeedback therapy c. Systemic corticosteroids d. Pulmonary function testing

c. Systemic corticosteroids Systemic corticosteroids speed the resolution of asthma exacerbations and are indicated if the initial response to the β-adrenergic bronchodilator is insufficient. IV fluids may be used, but not to improve ventilation. Biofeedback therapy and pulmonary function testing may be used after recovery to assist the patient and monitor the asthma.

Which is a symptom of iron deficiency anemia? a. Tachypnea b. Hypertension c. Tachycardia d. Dusky skin color

c. Tachycardia This answer is correct because one of the symptoms of iron deficiency anemia is tachycardia. Low iron is a result of inadequate red blood cells. The heart beats faster to compensate for the lack of red blood cells.

What is a primary nursing responsibility after obtaining a blood specimen for ABGs? a. Add heparin to the blood specimen. b. Apply pressure to the puncture site for 2 full minutes. c. Take the specimen immediately to the laboratory in an iced container. d. Avoid any changes in oxygen intervention for 15 minutes following the procedure.

c. Take the specimen immediately to the laboratory in an iced container. Samples for ABGs must be iced to keep the gases dissolved in the blood (unless the specimen is to be analyzed in < 1 minute) and taken directly to the laboratory. The syringe used to obtain the specimen is rinsed with heparin before the specimen is taken and pressure is applied to the arterial puncture site for 5 minutes after obtaining the specimen. Changes in oxygen therapy or interventions should be avoided for 15 minutes before the specimen is drawn because these changes might alter blood gas values.

The health care provider writes an order for bacteriologic testing for a patient who has a positive tuberculosis skin test. Which action should the nurse take? a. Teach about the reason for the blood tests. b. Schedule an appointment for a chest x-ray. c. Teach about the need to get sputum specimens for 2 to 3 consecutive days. d. Instruct the patient to expectorate three specimens as soon as possible.

c. Teach about the need to get sputum specimens for 2 to 3 consecutive days. Sputum specimens are obtained on 2 to 3 consecutive days for bacteriologic testing for M. tuberculosis The patient should not provide all the specimens at once. Blood cultures are not used for tuberculosis testing. A chest x-ray is not bacteriologic testing. Although the findings on chest x-ray examination are important, it is not possible to make a diagnosis of TB solely based on chest x-ray findings because other diseases can mimic the appearance of TB.

Which action will the nurse implement for a patient who arrives for a calcium-scoring CT scan? a. Insert an IV catheter. b. Administer oral sedative medications. c. Teach the patient about the procedure. d. Confirm that the patient has been fasting.

c. Teach the patient about the procedure. The nurse will need to teach the patient that the procedure is rapid and involves little risk. None of the other actions are necessary.

When planning care for the patient with hypertrophic CMP, what should the nurse include? a. Ventricular pacing b. Administration of vasodilators c. Teach the patient to avoid strenuous activity and dehydration d. Surgery for cardiac transplantation will have to be done soon

c. Teach the patient to avoid strenuous activity and dehydration Nursing interventions for the patient with hypertrophic CMP are to improve ventricular filling by reducing ventricular contractility and relieving left ventricular outflow obstruction to relieve symptoms and prevent complications. Strenuous activity and dehydration will increase systemic vascular resistance and should be avoided. Atrioventricular pacing will allow the septum to move away from the left ventricular wall and reduce the degree of outflow obstruction. Vasodilators may decrease venous return and further increase obstruction of blood flow from the heart. The surgery that could be done involves cutting into the thickened septal wall and removing some of the ventricular muscle.

An older patient has been diagnosed with possible white coat hypertension. Which action will the nurse plan to take next? a. Schedule the patient for regular blood pressure (BP) checks in the clinic. b. Instruct the patient about the need to decrease stress levels. c. Tell the patient how to self-monitor and record BPs at home. d. Inform the patient that ambulatory blood pressure monitoring will be needed.

c. Tell the patient how to self-monitor and record BPs at home. Having the patient self-monitor BPs at home will provide a reliable indication about whether the patient has hypertension. Regular BP checks in the clinic are likely to be high in a patient with white coat hypertension. Ambulatory blood pressure monitoring may be used if the data from self-monitoring are unclear. Although elevated stress levels may contribute to hypertension, instructing the patient about this is unlikely to reduce BP.

A patient has a parenteral nutrition infusion of 25% dextrose. A student nurse asks the nurse why a peripherally inserted central catheter was inserted. Which response by the nurse is most appropriate? a. There is a decreased risk for infection when 25% dextrose is infused through a central line. b. The prescribed infusion can be given much more rapidly when the patient has a central line. c. The 25% dextrose is hypertonic and will be more rapidly diluted when given through a central line. d. The required blood glucose monitoring is more accurate when samples are obtained from a central line.

c. The 25% dextrose is hypertonic and will be more rapidly diluted when given through a central line. The 25% dextrose solution is hypertonic. Shrinkage of red blood cells can occur when solutions with dextrose concentrations greater than 10% are administered IV. Blood glucose testing is not more accurate when samples are obtained from a central line. The infection risk is higher with a central catheter than with peripheral IV lines. Hypertonic or concentrated IV solutions are not given rapidly.

A patient is scheduled to have CABG surgery. What should the nurse explain is involved with the procedure? a. A synthetic graft will be used as a tube for blood flow from the aorta to a coronary artery distal to an obstruction. b. A stenosed coronary artery will be resected, and a synthetic arterial tube graft will be inserted to replace the diseased artery. c. The internal mammary artery will be detached from the chest wall and attached to a coronary artery distal to the stenosis. d. Reversed segments of a saphenous artery from the aorta will be anastomosed to the coronary artery distal to an obstruction.

c. The internal mammary artery will be detached from the chest wall and attached to a coronary artery distal to the stenosis. The most common method of coronary artery bypass involves leaving the internal mammary artery attached to its origin from the subclavian artery but dissecting it from the chest wall and anastomosing it distal to an obstruction in a coronary artery. Other grafts options include using the saphenous vein, and/or radial artery.

A new RN is preparing to administer packed red blood cells (PRBCs) to a client whose anemia was caused by blood loss after surgery. Which action by the new RN requires that you, as charge nurse, intervene immediately?" a. The new RN waits 20 minutes after obtaining the PRBCs before starting the infusion. b. The new RN starts an intravenous line for the transfusion using a 22-gauge catheter. c. The new RN primes the transfusion set using 5% dextrose in lactated Ringer's solution. d. The new RN tells the client that the PRBCs may cause a serious transfusion reaction."

c. The new RN primes the transfusion set using 5% dextrose in lactated Ringer's solution. Normal saline, an isotonic solution, should be used when priming the IV line to avoid causing hemolysis of RBCs. Ideally, blood products should be infused as soon as possible after they are obtained; however, a 20-minute delay would not be unsafe. Large-gauge IV catheters are preferable for blood administration; if a smaller catheter must be used, normal saline may be used to dilute the RBCs. Although it is appropriate to instruct clients to notify the nurse if symptoms of a transfusion reaction such as shortness of breath or chest pain occur, it will cause unnecessary anxiety to indicate that a serious reaction is likely to occur. Focus: Prioritization

Which action by a nurse who is giving fondaparinux (Arixtra) to a patient with a lower leg venous thromboembolism (VTE) indicates that more education about the drug is needed? a. The nurse avoids rubbing the injection site after giving the drug. b. The nurse injects the drug into the abdominal subcutaneous tissue. c. The nurse ejects the air bubble in the syringe before giving the drug. d. The nurse fails to assess the partial thromboplastin time (PTT) before giving the drug.

c. The nurse ejects the air bubble in the syringe before giving the drug. The air bubble is not ejected before giving fondaparinux to avoid loss of medication. The other actions by the nurse are appropriate.

A patient is scheduled for an open surgical valvuloplasty of the mitral valve. What information about this surgery should the nurse use to plan care? a. Cardiopulmonary bypass is not required with this procedure. b. Valve repair is a palliative measure, while valve replacement is curative. c. The operative mortality rate is lower in valve repair than in valve replacement. d. Patients with valve repair do not need postoperative anticoagulation as do those who have valve replacement.

c. The operative mortality rate is lower in valve repair than in valve replacement. Repair of mitral or tricuspid valves has a lower operative mortality rate than does replacement and is usually the surgical procedure of choice for these valvular diseases. Open repair is more precise than closed repair and requires cardiopulmonary bypass during surgery. All types of valve surgery are palliative, not curative, and patients require lifelong health care. Anticoagulation therapy is used for all valve surgery for at least some time postoperatively.

The nurse interviews a patient with a new diagnosis of chronic obstructive pulmonary disease (COPD). Which information is most helpful in confirming a diagnosis of chronic bronchitis? a. The patient tells the nurse about a family history of bronchitis. b. The patients history indicates a 30 pack-year cigarette history. c. The patient complains about a productive cough every winter for 3 months. d. The patient denies having any respiratory problems until the last 12 months.

c. The patient complains about a productive cough every winter for 3 months. A diagnosis of chronic bronchitis is based on a history of having a productive cough for 3 months for at least 2 consecutive years. There is no family tendency for chronic bronchitis. Although smoking is the major risk factor for chronic bronchitis, a smoking history does not confirm the diagnosis.

Which information obtained by the nurse who is admitting the patient for magnetic resonance imaging (MRI) will be most important to report to the health care provider before the MRI? a. The patient has an allergy to shellfish. b. The patient has a history of atherosclerosis. c. The patient has a permanent ventricular pacemaker. d. The patient took all the prescribed cardiac medications today.

c. The patient has a permanent ventricular pacemaker. MRI is contraindicated for patients with implanted metallic devices such as pacemakers. The other information also will be reported to the health care provider but does not impact on whether or not the patient can have an MRI.

The nurse assesses a patient with a history of asthma. Which assessment finding indicates that the nurse should take immediate action? a. Pulse oximetry reading of 91% b. Respiratory rate of 26 breaths/minute c. Use of accessory muscles in breathing d. Peak expiratory flow rate of 240 L/minute

c. Use of accessory muscles in breathing Use of accessory muscle indicates that the patient is experiencing respiratory distress and rapid intervention is needed. The other data indicate the need for ongoing monitoring and assessment but do not suggest that immediate treatment is required.

During change-of-shift report, the nurse obtains the following information about a hypertensive patient who received the first dose of nadolol (Corgard) during the previous shift. Which information indicates that the patient needs immediate intervention? a. The patient's pulse has dropped from 68 to 57 beats/min. b. The patient complains that the fingers and toes feel quite cold. c. The patient has developed wheezes throughout the lung fields. d. The patient's blood pressure (BP) reading is now 158/91 mm Hg.

c. The patient has developed wheezes throughout the lung fields. The most urgent concern for this patient is the wheezes, which indicate that bronchospasm (a common adverse effect of the noncardioselective -blockers) is occurring. The nurse should immediately obtain an O2 saturation measurement, apply supplemental O2, and notify the health care provider. The mild decrease in heart rate and complaint of cold fingers and toes are associated with -receptor blockade but do not require any change in therapy. The BP reading may indicate that a change in medication type or dose may be indicated. However, this is not as urgently needed as addressing the bronchospasm.

During change-of-shift report, the nurse obtains the following information about a hypertensive patient who received the first dose of nadolol (Corgard) during the previous shift. Which information indicates that the patient needs immediate intervention? a. The patients most recent blood pressure (BP) reading is 158/91 mm Hg. b. The patients pulse has dropped from 68 to 57 beats/minute. c. The patient has developed wheezes throughout the lung fields. d. The patient complains that the fingers and toes feel quite cold.

c. The patient has developed wheezes throughout the lung fields. The most urgent concern for this patient is the wheezes, which indicate that bronchospasm (a common adverse effect of the noncardioselective b -blockers) is occurring. The nurse should immediately obtain an oxygen saturation measurement, apply supplemental oxygen, and notify the health care provider. The mild decrease in heart rate and complaint of cold fingers and toes are associated with b -receptor blockade but do not require any change in therapy. The BP reading may indicate that a change in medication type or dose may be indicated. However, this is not as urgently needed as addressing the bronchospasm.

A patient is diagnosed with both human immunodeficiency virus (HIV) and active tuberculosis (TB) disease. Which information obtained by the nurse is most important to communicate to the health care provider? a. The Mantoux test had an induration of 7 mm. b. The chest-x-ray showed infiltrates in the lower lobes. c. The patient is being treated with antiretrovirals for HIV infection. d. The patient has a cough that is productive of blood-tinged mucus.

c. The patient is being treated with antiretrovirals for HIV infection. Drug interactions can occur between the antiretrovirals used to treat HIV infection and the medications used to treat TB. The other data are expected in a patient with HIV and TB.

The nurse teaches a patient with chronic bronchitis about a new prescription for Advair Diskus (combined fluticasone and salmeterol). Which action by the patient would indicate to the nurse that teaching about medication administration has been successful? a. The patient shakes the device before use. b. The patient attaches a spacer to the Diskus. c. The patient rapidly inhales the medication. d. The patient performs huff coughing after inhalation.

c. The patient rapidly inhales the medication. The patient should inhale the medication rapidly. Otherwise the dry particles will stick to the tongue and oral mucosa and not get inhaled into the lungs. Advair Diskus is a dry powder inhaler; shaking is not recommended. Spacers are not used with dry powder inhalers. Huff coughing is a technique to move mucus into larger airways to expectorate. The patient should not huff cough or exhale forcefully after taking Advair in order to keep the medication in the lungs.

The nurse is caring for a patient with chronic obstructive pulmonary disease (COPD). Which information obtained from the patient would prompt the nurse to consult with the health care provider before administering the prescribed theophylline? a. The patient reports a recent 15-pound weight gain. b. The patient denies any shortness of breath at present. c. The patient takes cimetidine (Tagamet) 150 mg daily. d. The patient complains about coughing up green mucus.

c. The patient takes cimetidine (Tagamet) 150 mg daily. Cimetidine interferes with the metabolism of theophylline, and concomitant administration may lead rapidly to theophylline toxicity. The other patient information would not affect whether the theophylline should be administered or not.

The nurse takes an admission history on a patient with possible asthma who has new-onset wheezing and shortness of breath. Which information may indicate a need for a change in therapy? a. The patient has chronic inflammatory bowel disease. b. The patient has a history of pneumonia 6 months ago. c. The patient takes propranolol (Inderal) for hypertension. d. The patient uses acetaminophen (Tylenol) for headaches.

c. The patient takes propranolol (Inderal) for hypertension. b-Blockers such as propranolol can cause bronchospasm in some patients with asthma. The other information will be documented in the health history but does not indicate a need for a change in therapy.

When teaching a patient about a bone marrow examination, what should the nurse explain? a. The procedure will be done under general anesthesia because it is so painful. b. The patient will not have any pain after the area at the puncture site is anesthetized. c. The patient will experience a brief, very sharp pain during aspiration of the bone marrow. d. There will be no pain during the procedure, but an ache will be present several days afterward.

c. The patient will experience a brief, very sharp pain during aspiration of the bone marrow. d. There will be no pain The aspiration of bone marrow content is done with local anesthesia at the site of the puncture, but the aspiration causes a suction pain that is quite painful but very brief. There is generally only residual soreness following the test.

A patient with right lower-lobe pneumonia has been treated with IV antibiotics for 3 days. Which assessment data obtained by the nurse indicates that the treatment has been effective? a. Bronchial breath sounds are heard at the right base. b. The patient coughs up small amounts of green mucus. c. The patients white blood cell (WBC) count is 9000/L. d. Increased tactile fremitus is palpable over the right chest.

c. The patients white blood cell (WBC) count is 9000/L. The normal WBC count indicates that the antibiotics have been effective. All the other data suggest that a change in treatment is needed.

Which information will the nurse include when teaching a patient who is scheduled for a radiofrequency catheter ablation for treatment of atrial flutter? a. The procedure will prevent or minimize the risk for sudden cardiac death. b. The procedure will use cold therapy to stop the formation of the flutter waves. c. The procedure will use electrical energy to destroy areas of the conduction system. d. The procedure will stimulate the growth of new conduction pathways between the atria.

c. The procedure will use electrical energy to destroy areas of the conduction system. Radiofrequency catheter ablation therapy uses electrical energy to burn or ablate areas of the conduction system as definitive treatment of atrial flutter (i.e., restore normal sinus rhythm) and tachydysrhythmias. All other statements regarding the procedure are incorrect.

The nurse observes a student who is listening to a patients lungs who is having no problems with breathing. Which action by the student indicates a need to review respiratory assessment skills? a. The student starts at the apices of the lungs and moves to the bases. b. The student compares breath sounds from side to side avoiding bony areas. c. The student places the stethoscope over the posterior chest and listens during inspiration. d. The student instructs the patient to breathe slowly and a little more deeply than normal through the mouth.

c. The student places the stethoscope over the posterior chest and listens during inspiration. Listening only during inspiration indicates the student needs a review of respiratory assessment skills. At each placement of the stethoscope, listen to at least one cycle of inspiration and expiration. During chest auscultation, instruct the patient to breathe slowly and a little deeper than normal through the mouth. Auscultation should proceed from the lung apices to the bases, comparing opposite areas of the chest, unless the patient is in respiratory distress or will tire easily. If so, start at the bases (see Fig. 26-7). Place the stethoscope over lung tissue, not over bony prominences.

A patient is being discharged with plans for home O2 therapy provided by an O2 concentrator with a portable O2 -concentrator unit. In preparing the patient to use the equipment, what should the nurse teach the patient? a. The portable unit will last about 6 to 8 hours. b. The unit is strictly for portable and emergency use. c. The unit concentrates O2 from the air, providing a continuous O2 supply. d. Weekly delivery of 1 large cylinder of O2 will be needed for a 7- to 10-day supply of O2 .

c. The unit concentrates O2 from the air, providing a continuous O2 supply. Oxygen concentrators or extractors continuously supply O2 concentrated from the air. Portable liquid O2 units will hold about 6 to 8 hours of O2 , but because of the expense, they are only used for portable and emergency use. Portable O2 -conserving units slow the use of oxygen. Compressed O2 comes in various tank sizes. It requires weekly deliveries of 4 to 5 large tanks to meet a 7- to 10-day supply.

You are taking care of a male patient who has the following laboratory values from his CBC: WBC 6.5 × 103/μL, Hgb 13.4 g/dL, Hct 40%, platelets 50 × 103/μL. What are you most concerned about? a. The patient is neutropenic. b. The patient has an infection. c. There is an increased risk for bleeding. d. Fall risk precautions are needed due to anemia

c. There is an increased risk for bleeding.

Palpation is the assessment technique used to find which abnormal assessment findings (select all that apply)? a. Stridor b. Finger clubbing c. Tracheal deviation d. Limited chest expansion e. Increased tactile fremitus f. Use of accessory muscles

c. Tracheal deviation d. Limited chest expansion e. Increased tactile fremitus Palpation identifies tracheal deviation, limited chest expansion, and increased tactile fremitus. Stridor is identified with auscultation. Finger clubbing and accessory muscle use are identified with inspection.

The nurse is palpating the patient's chest during a focused respiratory assessment in the emergency department. Which finding is a medical emergency? a. Increased tactile fremitus b. Diminished chest movement c. Tracheal deviation to the left d. Decreased anteroposterior (AP) diameter

c. Tracheal deviation to the left Tracheal deviation is a medical emergency when it is caused by a tension pneumothorax. Tactile fremitus increases with pneumonia or pulmonary edema and decreases in pleural effusion or lung hyperinflation. Diminished chest movement occurs with barrel chest, restrictive disease, and neuromuscular disease.

What are manifestations of acute coronary syndrome (ACS) (select all that apply)? a. Dysrhythmia b. Stable angina c. Unstable angina d. ST-segment-elevation myocardial infarction (STEMI) e. Non-ST-segment-elevation myocardial infarction (NSTEMI)

c. Unstable angina d. ST-segment-elevation myocardial infarction (STEMI) e. Non-ST-segment-elevation myocardial infarction (NSTEMI) Unstable angina, ST-segment-elevation myocardial infarction (STEMI), and non-ST-segment-elevation myocardial infarction (NSTEMI) are conditions that are manifestations of acute coronary syndrome (ACS). The other options are not manifestations of ACS.

Employee health test results reveal a tuberculosis (TB) skin test of 16-mm induration and a negative chest x-ray for a staff nurse working on the pulmonary unit. The nurse has no symptoms of TB. Which information should the occupational health nurse plan to teach the staff nurse? a. Standard four-drug therapy for TB b. Need for annual repeat TB skin testing c. Use and side effects of isoniazid (INH) d. Bacille Calmette-Gurin (BCG) vaccine

c. Use and side effects of isoniazid (INH) The nurse is considered to have a latent TB infection and should be treated with INH daily for 6 to 9 months. The four-drug therapy would be appropriate if the nurse had active TB. TB skin testing is not done for individuals who have already had a positive skin test. BCG vaccine is not used in the United States for TB and would not be helpful for this individual, who already has a TB infection.

To detect and treat the most common complication of MI, what should the nurse do? a. Measure hourly urine output. b. Auscultate the chest for crackles. c. Use continuous cardiac monitoring. d. Take vital signs every 2 hours for the first 8 hours.

c. Use continuous cardiac monitoring. The most common complication of MI is dysrhythmias. Continuous cardiac monitoring allows identification and treatment of dysrhythmias that may cause further deterioration of the cardiovascular status or death. Measurement of hourly urine output and vital signs is indicated to detect symptoms of the complication of cardiogenic shock. Crackles, dyspnea, and tachycardia may indicate the onset of heart failure (HF).

The nursing student is seeking assistance in hearing the patient's abnormal heart sounds. What should the nurse tell the student to do for a more effective assessment? a. Use the diaphragm of the stethoscope with the patient prone. b. Use the diaphragm of the stethoscope with the patient supine. c. Use the bell of the stethoscope with the patient leaning forward. d. Use the bell of the stethoscope with the patient on the right side.

c. Use the bell of the stethoscope with the patient leaning forward. The bell of the stethoscope will enable better hearing of the lowpitched extra heart sounds. Having the patient lean forward best enables hearing the aortic and pulmonic areas; having the patient on the left side will enhance the mitral area sounds; both of these positions bring the heart closer to the chest wall. Having the patient supine or prone will not improve the auscultation.

A patient with aortic valve endocarditis develops dyspnea, crackles in the lungs, and restlessness. What should the nurse suspect that the patient is experiencing? a. Pulmonary embolization from valve vegetations b. Vegetative embolization to the coronary arteries c. Valvular incompetence with resulting heart failure d. Nonspecific manifestations that accompany infectious diseases

c. Valvular incompetence with resulting heart failure The dyspnea, crackles, and restlessness that the patient is manifesting are manifestations of heart failure and decreased cardiac output (CO) that occurs in up to 80% of patients with aortic valve endocarditis because of aortic valve incompetence. Pulmonary emboli occur in right-sided endocarditis. Vegetative embolization from the aortic valve occurs throughout the arterial system and may affect any body organ.

What describes the action of the natriuretic peptides and nitric oxide in their counterregulatory processes in response to HF? a. Excretion of potassium b. Increased release of ADH c. Vasodilation and decreased BP d. Decreased glomerular filtration rate and edema

c. Vasodilation and decreased BP Both the natriuretic peptides and nitric oxide contribute to vasodilation, decreased BP, and decreased afterload. The natriuretic peptides also increase excretion of sodium by increasing glomerular filtration rate and diuresis (renal effects) as well as interfere with ADH release and inhibit aldosterone and renin secretion (hormonal effects).

The nurse is caring for a preoperative patient who has an order for vitamin K by subcutaneous injection. The nurse should verify that which laboratory study is abnormal before administering the dose? a Hematocrit (Hct) b Hemoglobin (Hgb) c Prothrombin time (PT) d Partial thromboplastin time (PTT)

c. Vitamin K counteracts hypoprothrombinemia and/or reverses the effects of warfarin (Coumadin) and thus decreases the risk of bleeding. High values for either the PT or the international normalized ratio demonstrate the need for this medication.

Which function test fits description "Volume of air inhaled and exhaled with each breath"? a. FEV1 b. TLC c. Vt d. VC

c. Vt Tidal volume

The nurse is caring for a patient who has been receiving warfarin (Coumadin) and digoxin (Lanoxin) as treatment for atrial fibrillation. Because the warfarin has been discontinued before surgery, the nurse should diligently assess the patient for which complication early in the postoperative period until the medication is resumed? a Decreased cardiac output b Increased blood pressure c Cerebral or pulmonary emboli d Excessive bleeding from incision or IV sites

c. Warfarin is an anticoagulant that is used to prevent thrombi from forming on the walls of the atria during atrial fibrillation. When the medication is terminated, thrombi could again form. If one or more thrombi detach from the atrial wall, they could travel as cerebral emboli from the left atrium or pulmonary emboli from the right atrium.

The nurse should be alert for which manifestations in a patient receiving a loop diuretic? a. Restlessness and agitation b. Paresthesias and irritability c. Weak, irregular pulse and poor muscle tone d. Increased blood pressure and muscle spasms

c. Weak, irregular pulse and poor muscle tone

A patient with ST-segment elevation in three contiguous electrocardiographic (ECG) leads is admitted to the emergency department (ED) and diagnosed as having an ST-segment-elevation myocardial infarction (STEMI). Which question should the nurse ask to determine whether the patient is a candidate for thrombolytic therapy? a. Do you have any allergies? b. Do you take aspirin on a daily basis? c. What time did your chest pain begin? d. Can you rate your chest pain using a 0 to 10 scale?

c. What time did your chest pain begin? Thrombolytic therapy should be started within 6 hours of the onset of the myocardial infarction (MI), so the time at which the chest pain started is a major determinant of the appropriateness of this treatment. The other information will also be needed, but it will not be a factor in the decision about thrombolytic therapy.

A patient is scheduled for pulmonary function testing. Which action should the nurse take to prepare the patient for this procedure? a. Give the rescue medication immediately before testing. b. Administer oral corticosteroids 2 hours before the procedure. c. Withhold bronchodilators for 6 to 12 hours before the examination. d. Ensure that the patient has been NPO for several hours before the test.

c. Withhold bronchodilators for 6 to 12 hours before the examination. Bronchodilators are held before pulmonary function testing (PFT) so that a baseline assessment of airway function can be determined. Testing is repeated after bronchodilator use to determine whether the decrease in lung function is reversible. There is no need for the patient to be NPO. Oral corticosteroids should be held before PFTs. Rescue medications (which are bronchodilators) would not be given until after the baseline pulmonary function was assessed.

A nurse is preparing to administer oral potassium for a client who has a potassium level of 5.5 mEq/L. Which of the following actions should the nurse take first? a. Administer a hypertonic solution. b. Repeat the potassium level. c. Withhold the medication. d. Monitor for paresthesia.

c. Withhold the medication The greatest risk to this client is injury from hyperkalemia. Therefore, the priority action is to withhold the oral potassium and notify the provider.

The nurse determines that treatment of HF has been successful when the patient experiences a. weight loss and diuresis. b. warm skin and less fatigue. c. clear lung sounds and decreased HR. d. absence of chest pain and improved level of consciousness (LOC).

c. clear lung sounds and decreased HR. Successful treatment of HF is indicated by an absence of symptoms of pulmonary edema and hypoxemia, such as clear lung sounds and a normal HR. Weight loss and diuresis, warm skin, less fatigue, and improved level of consciousness (LOC) may occur without resolution of pulmonary symptoms. Chest pain is not a common finding in HF unless coronary artery perfusion is impaired

During the administration of the thrombolytic agent to a patient with an acute myocardial infarction (AMI), the nurse should stop the drug infusion if the patient experiences a. bleeding from the gums. b. increase in blood pressure. c. a decrease in level of consciousness. d. a nonsustained episode of ventricular tachycardia.

c. a decrease in level of consciousness. The change in level of consciousness indicates that the patient may be experiencing intracranial bleeding, a possible complication of thrombolytic therapy. Some bleeding of the gums is an expected side effect of the therapy but not an indication to stop infusion of the thrombolytic medication. A decrease in blood pressure could indicate internal bleeding. A nonsustained episode of ventricular tachycardia is a common reperfusion dysrhythmia and may indicate that the therapy is effective.

DIC is a disorder in which a. the coagulation pathway is genetically altered, leading to thrombus formation in all major blood vessels. b. an underlying disease depletes hemolytic factors in the blood, leading to diffuse thrombotic episodes and infarcts. c. a disease process stimulates coagulation processes with resultant thrombosis, as well as depletion of clotting factors, leading to diffuse clotting and hemorrhage. d. an inherited predisposition causes a deficiency of clotting factors that leads to overstimulation of coagulation processes in the vasculature.

c. a disease process stimulates coagulation processes with resultant thrombosis, as well as depletion of clotting factors, leading to diffuse clotting and hemorrhage.

Priority Decision: During care of the patient following femoral bypass graft surgery, the nurse immediately notifies the health care provider (HCP) if the patient has a. fever and redness at the incision site. b. 2 + edema of the extremity and pain at the incision site. c. a loss of palpable pulses and numbness and tingling of the feet. d. increasing ankle-brachial indices and serous drainage from the incision

c. a loss of palpable pulses and numbness and tingling of the feet. Loss of palpable pulses, numbness and tingling of the extremity, extremity pallor, cyanosis, or cold are indications of occlusion of the bypass graft and need immediate medical attention. Pain, redness, and serous drainage at the incision site are expected postoperatively. Ankle brachial index measurements are not recommended because of increased risk for graft thrombosis, but this would decrease with occlusion.

IV sodium nitroprusside (Nipride) is ordered for a patient with acute pulmonary edema. During the first hours of administration, the nurse will need to titrate the nitroprusside rate if the patient develops a. ventricular ectopy. b. a dry, hacking cough. c. a systolic BP <90 mm Hg. d. a heart rate <50 beats/minute.

c. a systolic BP <90 mm Hg. Sodium nitroprusside is a potent vasodilator, and the major adverse effect is severe hypotension. Coughing and bradycardia are not adverse effects of this medication. Nitroprusside does not cause increased ventricular ectopy.

Which medications would be most appropriate to administer to a patient experiencing an acute asthma attack? (select all that apply) a. montelukast (Singulair) b. inhaled hypertonic saline c. albuterol (Proventil HFA) d. ipratropium (Atrovent HFA) e. salmeterol (Serevent Diskus)

c. albuterol (Proventil HFA) d. ipratropium (Atrovent HFA)

Direction of fluid shift "from interstitium to vessels" and mechanism of fluid movement "interstitial hydrostatic pressure" matches which event factor? a. low serum albumin b. administration of 10% glucose c. application of elastic bandages d. dehydration

c. application of elastic bandages

The nurse can best determine adequate arterial oxygenation of the blood by assessing a. heart rate. b. hemoglobin level. c. arterial oxygen partial pressure. d. arterial carbon dioxide partial pressure.

c. arterial oxygen partial pressure.

A 54-year-old woman with acute myelogenous leukemia (AML) is considering treatment with a hematopoietic stem cell transplant (HSCT). The best approach for the nurse to assist the patient with a treatment decision is to a. emphasize the positive outcomes of a bone marrow transplant. b. discuss the need for adequate insurance to cover post-HSCT care. c. ask the patient whether there are any questions or concerns about HSCT. d. explain that a cure is not possible with any other treatment except HSCT.

c. ask the patient whether there are any questions or concerns about HSCT. Offering the patient an opportunity to ask questions or discuss concerns about HSCT will encourage the patient to voice concerns about this treatment and also will allow the nurse to assess whether the patient needs more information about the procedure. Treatment of AML using chemotherapy is another option for the patient. It is not appropriate for the nurse to ask the patient to consider insurance needs in making this decision.

A patient with chronic heart failure who is taking a diuretic and an angiotensin-converting enzyme (ACE) inhibitor and who is on a low-sodium diet tells the home health nurse about a 5-pound weight gain in the last 3 days. The nurses priority action will be to a. have the patient recall the dietary intake for the last 3 days. b. ask the patient about the use of the prescribed medications. c. assess the patient for clinical manifestations of acute heart failure. d. teach the patient about the importance of restricting dietary sodium.

c. assess the patient for clinical manifestations of acute heart failure. The 5-pound weight gain over 3 days indicates that the patients chronic heart failure may be worsening. It is important that the patient be assessed immediately for other clinical manifestations of decompensation, such as lung crackles. A dietary recall to detect hidden sodium in the diet, reinforcement of sodium restrictions, and assessment of medication compliance may be appropriate interventions but are not the first nursing actions indicated.

A 23-year-old patient tells the health care provider about experiencing cold, numb fingers when running during the winter and Raynauds phenomenon is suspected. The nurse will anticipate teaching the patient about tests for a. hyperglycemia. b. hyperlipidemia. c. autoimmune disorders. d. coronary artery disease.

c. autoimmune disorders. Secondary Raynauds phenomenon may occur in conjunction with autoimmune diseases such as rheumatoid arthritis. Patients should be screened for autoimmune disorders. Raynauds phenomenon is not associated with hyperlipidemia, hyperglycemia, or coronary artery disease.

A patient with a tricuspid valve disorder has impaired blood flow between the a. vena cava and right atrium. b. left atrium and left ventricle. c. right atrium and right ventricle. d. right ventricle and pulmonary artery.

c. right atrium and right ventricle.

A 52-year-old patient has a new diagnosis of pernicious anemia. The nurse determines that the patient understands the teaching about the disorder when the patient states, I a. need to start eating more red meat and liver. b. will stop having a glass of wine with dinner. c. could choose nasal spray rather than injections of vitamin B12. d. will need to take a proton pump inhibitor like omeprazole (Prilosec).

c. could choose nasal spray rather than injections of vitamin B12. Because pernicious anemia prevents the absorption of vitamin B12, this patient requires injections or intranasal administration of cobalamin. Alcohol use does not cause cobalamin deficiency. Proton pump inhibitors decrease the absorption of vitamin B12. Eating more foods rich in vitamin B12 is not helpful because the lack of intrinsic factor prevents absorption of the vitamin.

The major advantage of a Venturi mask is that it can a. deliver up to 80% O2. b. provide continuous 100% humidity. c. deliver a precise concentration of O2. d. be used while a patient eats and sleeps.

c. deliver a precise concentration of O2.

An expected finding in the assessment of an 81-year-old patient is a. a narrowed pulse pressure. b. diminished carotid artery pulses. c. difficulty isolating the apical pulse. d. an increased heart rate in response to stress.

c. difficulty isolating the apical pulse.

In a severely anemic patient, the nurse would expect to find a. cyanosis and cardiomegaly. b. pulmonary edema and fibrosis. c. dyspnea at rest and tachycardia. d. ventricular dysrhythmias and wheezing.

c. dyspnea at rest and tachycardia.

Fluid and electrolytes are transported by this method which occurs with the use of proteins a. diffusion b. active transport c. facilitated diffusion d. osmosis e. hydrostatic pressure f. oncotic pressure

c. facilitated diffusion

Direction of fluid shift "from cell to extracellular compartment and from interstitium to vessels" and mechanism of fluid movement "tissue oncotic pressure" matches which event factor? a. burns b. dehydration c. fluid overload d. hyponatremia

c. fluid overload

A patient with multiple myeloma becomes confused and lethargic. The nurse would expect that these clinical manifestations may be explained by diagnostic results that indicate a. hyperkalemia. b. hyperuricemia. c. hypercalcemia. d. CNS myeloma.

c. hypercalcemia.

Which of these electrolyte imbalances is the cause of "chronic kidney disease"? (select all that apply) a. hypernatremia b. hyponatremia c. hyperkalemia d. hypokalemia e. hypercalcemia f. hypocalcemia g. hyperphosphatemia h. hypophosphatemia i. hypermagnesemia j. hypomagnesemia

c. hyperkalemia f. hypocalcemia g. hyperphosphatemia i. hypermagnesemia

During the respiratory assessment of an older adult, the nurse would expect to find (select all that apply) a. a vigorous reflex cough. b. increased chest expansion. c. increased residual volume. d. decreased lung sounds at base of lungs. e. increased anteroposterior (AP) chest diameter.

c. increased residual volume. d. decreased lung sounds at base of lungs. e. increased anteroposterior (AP) chest diameter.

While assessing a 68-year-old with ascites, the nurse also notes jugular venous distention (JVD) with the head of the patients bed elevated 45 degrees. The nurse knows this finding indicates a. decreased fluid volume. b. jugular vein atherosclerosis. c. increased right atrial pressure. d. incompetent jugular vein valves.

c. increased right atrial pressure. The jugular veins empty into the superior vena cava and then into the right atrium, so JVD with the patient sitting at a 45-degree angle reflects increased right atrial pressure. JVD is an indicator of excessive fluid volume (increased preload), not decreased fluid volume. JVD is not caused by incompetent jugular vein valves or atherosclerosis.

The most common finding in people at risk for sudden cardiac death is a. aortic valve disease. b. mitral valve disease. c. left ventricular dysfunction. d. atherosclerotic heart disease.

c. left ventricular dysfunction.

While assessing the lymph nodes, the nurse should a. apply gentle, firm pressure to deep lymph nodes. b. palpate the deep cervical and supraclavicular nodes last. c. lightly palpate superficial lymph nodes with the pads of the fingers. d. use the tips of the second, third, and fourth fingers to apply deep palpation.

c. lightly palpate superficial lymph nodes with the pads of the fingers.

Which of the following acid-base imbalances is the common cause for baking soda used as antacid? a. respiratory acidosis b. respiratory alkalosis c. metabolic alkalosis d. metabolic acidosis

c. metabolic alkalosis

Which of the following acid-base imbalances is the common cause for prolonged vomiting? a. respiratory acidosis b. respiratory alkalosis c. metabolic alkalosis d. metabolic acidosis

c. metabolic alkalosis

Which of the following acid-base imbalances matches the mechanism "increased base bicarbonate (HCO3-)"? a. respiratory acidosis b. respiratory alkalosis c. metabolic alkalosis d. metabolic acidosis

c. metabolic alkalosis

During discharge teaching with a 68-year-old patient who had a mitral valve replacement with a mechanical valve, the nurse instructs the patient on the a. use of daily aspirin for anticoagulation. b. correct method for taking the radial pulse. c. need for frequent laboratory blood testing. d. need to avoid any physical activity for 1 month.

c. need for frequent laboratory blood testing. Anticoagulation with warfarin (Coumadin) is needed for a patient with mechanical valves to prevent clotting on the valve. This will require frequent international normalized ratio (INR) testing. Daily aspirin use will not be effective in reducing the risk for clots on the valve. Monitoring of the radial pulse is not necessary after valve replacement. Patients should resume activities of daily living as tolerated.

The nurse is aware that a major difference between Hodgkin's lymphoma and non-Hodgkin's lymphoma is that a. Hodgkin's lymphoma occurs only in young adults. b. Hodgkin's lymphoma is considered potentially curable. c. non-Hodgkin's lymphoma can manifest in multiple areas. d. non-Hodgkin's lymphoma is treated only with radiation therapy.

c. non-Hodgkin's lymphoma can manifest in multiple areas.

The nurse plans discharge teaching for a patient with chronic heart failure who has prescriptions for digoxin (Lanoxin) and hydrochlorothiazide (HydroDIURIL). Appropriate instructions for the patient include a. limit dietary sources of potassium. b. take the hydrochlorothiazide before bedtime. c. notify the health care provider if nausea develops. d. skip the digoxin if the pulse is below 60 beats/minute.

c. notify the health care provider if nausea develops. Nausea is an indication of digoxin toxicity and should be reported so that the provider can assess the patient for toxicity and adjust the digoxin dose, if necessary. The patient will need to include potassium-containing foods in the diet to avoid hypokalemia. Patients should be taught to check their pulse daily before taking the digoxin and if the pulse is less than 60, to call their provider before taking the digoxin. Diuretics should be taken early in the day to avoid sleep disruption.

Which is a modifiable risk factor for a client with hypertension? a. Age b. Race c. Obesity d. Family history of hypertension

c. obesity

A patient admitted with syncope has continuous ECG monitoring. An examination of the rhythm strip reveals the following: atrial rate 74 beats/min and regular; ventricular rate 62 beats/min and irregular; P wave normal shape; PR interval lengthens progressively until a P wave is not conducted; QRS normal shape. The priority nursing intervention would be to a. give epinephrine 1 mg IV push. b. prepare for synchronized cardioversion. c. observe for symptoms of hypotension or angina. d. apply transcutaneous pacemaker pads on the patient.

c. observe for symptoms of hypotension or angina.

A patient admitted with syncope has continuous ECG monitoring. An examination of the rhythm strip reveals the following: atrial rate 74 beats/min and regular; ventricular rate 62 beats/min and irregular; P wave normal shape; PR interval lengthens progressively until a P wave is not conducted; QRS normal shape. The priority nursing intervention would be to a. give epinephrine 1 mg IV push. b. prepare for synchronized cardioversion. c. observe for symptoms of hypotension or angina. d. apply transcutaneous pacemaker pads on the patient.

c. observe for symptoms of hypotension or angina. Correct answer: c Rationale: The rhythm is a second-degree atrioventricular (AV) block, type I (i.e., Mobitz I or Wenckebach heart block). It is characterized by a gradual lengthening of the PR interval. Type I AV block is usually a result of myocardial ischemia or infarction. It is typically transient and well tolerated. The nurse should assess for bradycardia, hypotension, and angina. The symptomatic patient may need atropine or a temporary pacemaker.

The nurse interprets the following ECG at the nurse's station. Which action should the nurse implement first? a. administer epinephrine IVP b. prepare to defibrillate the client c. palpate carotid pulse d. start cardiopulmonary resuscitation (CPR)

c. palpate carotid pulse The nurse must first assess before they act to confirm client status. We always assess clients over monitors. (rhythm = Vfib).

A patient who has chronic heart failure tells the nurse, I was fine when I went to bed, but I woke up in the middle of the night feeling like I was suffocating! The nurse will document this assessment finding as a. orthopnea. b. pulsus alternans. c. paroxysmal nocturnal dyspnea. d. acute bilateral pleural effusion.

c. paroxysmal nocturnal dyspnea. Paroxysmal nocturnal dyspnea is caused by the reabsorption of fluid from dependent body areas when the patient is sleeping and is characterized by waking up suddenly with the feeling of suffocation. Pulsus alternans is the alternation of strong and weak peripheral pulses during palpation. Orthopnea indicates that the patient is unable to lie flat because of dyspnea. Pleural effusions develop over a longer time period.

A client is admitted to the hospital with headache, weakness, and lethargy. The arterial blood gas results are pH 7.33, PaCO2 49, HCO3 29. How should the nurse interpret findings? a. fully compensated respiratory acidosis b. uncompensated metabolic alkalosis c. partially compensated respiratory acidosis d. partially compensated metabolic acidosis

c. partially compensated respiratory acidosis The pH is low, indicating acidosis; the PaCO2 is elevated indicating a respiratory basis; and the HCO3 is elevated, indicating compensatory mechanisms are partially working. Note: Fully compensated we have normal pH & abnormal CO2 and HCO3. Partially compensated we have abnormal CO2 and HCO3, but our pH is partial to one over the other. Uncompensated is when we have a normal CO2 and abnormal pH & HCO3.

Best indicator of ventilator associated pneumonia (VAP)? a. negative chest xray b. 94% pulse oximetry c. positive sputum culture d. decreased white blood cells (WBC)

c. positive sputum culture

A patient with diabetes mellitus and chronic stable angina has a new order for captopril (Capoten). The nurse should teach the patient that the primary purpose of captopril is to a. lower heart rate. b. control blood glucose levels. c. prevent changes in heart muscle. d. reduce the frequency of chest pain.

c. prevent changes in heart muscle. The purpose for angiotensin-converting enzyme (ACE) inhibitors in patients with chronic stable angina who are at high risk for a cardiac event is to decrease ventricular remodeling. ACE inhibitors do not directly impact angina frequency, blood glucose, or heart rate.

The nurse performing an assessment with a patient who has chronic peripheral artery disease (PAD) of the legs and an ulcer on the right second toe would expect to find a. dilated superficial veins. b. swollen, dry, scaly ankles. c. prolonged capillary refill in all the toes. d. a serosanguineous drainage from the ulcer.

c. prolonged capillary refill in all the toes. Capillary refill is prolonged in PAD because of the slower and decreased blood flow to the periphery. The other listed clinical manifestations are consistent with chronic venous disease.

While admitting an 82-year-old with acute decompensated heart failure to the hospital, the nurse learns that the patient lives alone and sometimes confuses the water pill with the heart pill. When planning for the patients discharge the nurse will facilitate a a. consult with a psychologist. b. transfer to a long-term care facility. c. referral to a home health care agency. d. arrangements for around-the-clock care.

c. referral to a home health care agency. The data about the patient suggest that assistance in developing a system for taking medications correctly at home is needed. A home health nurse will assess the patients home situation and help the patient develop a method for taking the two medications as directed. There is no evidence that the patient requires services such as a psychologist consult, long-term care, or around-the-clock home care.

The nurse working on the heart failure unit knows that teaching an older female patient with newly diagnosed heart failure is effective when the patient states that a. she will take furosemide (Lasix) every day at bedtime. b. the nitroglycerin patch is applied when any chest pain develops. c. she will call the clinic if her weight goes from 124 to 128 pounds in a week. d. an additional pillow can help her sleep if she is feeling short of breath at night.

c. she will call the clinic if her weight goes from 124 to 128 pounds in a week. Teaching for a patient with heart failure includes information about the need to weigh daily and notify the health care provider about an increase of 3 pounds in 2 days or 3 to 5 pounds in a week. Nitroglycerin patches are used primarily to reduce preload (not to prevent chest pain) in patients with heart failure and should be used daily, not on an as needed basis. Diuretics should be taken earlier in the day to avoid nocturia and sleep disturbance. The patient should call the clinic if increased orthopnea develops, rather than just compensating by further elevating the head of the bed.

When caring for a patient with mitral valve stenosis, it is most important that the nurse assess for a. diastolic murmur. b. peripheral edema. c. shortness of breath on exertion. d. right upper quadrant tenderness.

c. shortness of breath on exertion. The pressure gradient changes in mitral stenosis lead to fluid backup into the lungs, resulting in hypoxemia and dyspnea. The other findings also may be associated with mitral valve disease but are not indicators of possible hypoxemia.

When caring for a patient with infective endocarditis of the tricuspid valve, the nurse should monitor the patient for the development of a. flank pain. b. splenomegaly. c. shortness of breath. d. mental status changes.

c. shortness of breath. Embolization from the tricuspid valve would cause symptoms of pulmonary embolus. Flank pain, changes in mental status, and splenomegaly would be associated with embolization from the left-sided valves.

The use of radiofrequency catheter ablation therapy to "burn" areas of the cardiac conduction system is indicated for the treatment of a. sinus arrest. b. heart blocks. c. tachydysrhythmias. d. premature ventricular beats.

c. tachydysrhythmias. Catheter ablation therapy uses radiofrequency energy to ablate or "burn" accessory pathways or ectopic sites in the atria, AV node, or ventricles that cause tachydysrhythmias. It is not used for sinus arrest, heart blocks, or premature ventricular beats.

Important teaching for the patient scheduled for a radiofrequency catheter ablation procedure includes explaining that a. ventricular bradycardia may be induced and treated during the procedure. b. catheter will be placed in both femoral arteries to allow double-catheter use. c. the procedure will destroy areas of the conduction system that are causing rapid heart rhythms. d. general anesthetic will be given to prevent the awareness of any "sudden cardiac death" experiences.

c. the procedure will destroy areas of the conduction system that are causing rapid heart rhythms.

Important teaching for the patient scheduled for a radiofrequency catheter ablation procedure includes explaining that a. ventricular bradycardia may be induced and treated during the procedure. b. catheter will be placed in both femoral arteries to allow double-catheter use. c. the procedure will destroy areas of the conduction system that are causing rapid heart rhythms. d. general anesthetic will be given to prevent the awareness of any "sudden cardiac death" experiences.

c. the procedure will destroy areas of the conduction system that are causing rapid heart rhythms. Correct answer: c Rationale: Radiofrequency catheter ablation therapy involves the use of electrical energy to "burn" or ablate areas of the conduction system as definitive treatment of tachydysrhythmias.

A patient reports dizziness and shortness of breath for several days. During cardiac monitoring in the emergency department (ED), the nurse obtains the following electrocardiographic (ECG) tracing. The nurse interprets this heart rhythm as a. junctional escape rhythm. b. accelerated idioventricular rhythm. c. third-degree atrioventricular (AV) block. d. sinus rhythm with premature atrial contractions (PACs).

c. third-degree atrioventricular (AV) block. The inconsistency between the atrial and ventricular rates and the variable P-R interval indicate that the rhythm is third-degree AV block. Sinus rhythm with PACs will have a normal rate and consistent P-R intervals with occasional PACs. An accelerated idioventricular rhythm will not have visible P waves.

The nurse has received the laboratory results for a patient who developed chest pain 4 hours ago and may be having a myocardial infarction. The most important laboratory result to review will be a. myoglobin. b. low-density lipoprotein (LDL) cholesterol. c. troponins T and I. d. creatine kinase-MB (CK-MB).

c. troponins T and I. Cardiac troponins start to elevate 4 to 6 hours after myocardial injury and are highly specific to myocardium. They are the preferred diagnostic marker for myocardial infarction. Myoglobin rises in response to myocardial injury within 30 to 60 minutes. It is rapidly cleared from the body, thus limiting its use in the diagnosis of myocardial infarction. LDL cholesterol is useful in assessing cardiovascular risk but is not helpful in determining whether a patient is having an acute myocardial infarction. Creatine kinase (CK-MB) is specific to myocardial injury and infarction and increases 4 to 6 hours after the infarction occurs. It is often trended with troponin levels.

Which statement indicates the patient is experiencing anger as the psychologic response to his acute MI? a. "Yes, I'm having a little chest pain. It's no big deal." b. "I don't think I can take care of myself at home yet." c. "What's going to happen if I have another heart attack?" d. "I hope my wife is happy after harping about my eating habits."

d. "I hope my wife is happy after harping about my eating habits." Anger about the MI may be directed at family, staff, or the medical regimen. Stating that the chest pain is no big deal is denial. Relaying an inability to care for self relates to dependency. Questioning what will happen if there is another attack is expressing anxiety and fear. Depression may be expressed related to changes in lifestyle. Realistic acceptance is seen with actively engaging in changing modifiable risk factors

Which cardiac valve disorder causes a sudden onset of cardiovascular collapse? a. Mitral valve stenosis b. Tricuspid valve disease c. Pulmonic valve stenosis d. Acute aortic regurgitation

d. Acute aortic regurgitation Acute aortic regurgitation causes a sudden cardiovascular collapse. With mitral valve stenosis, dyspnea is a prominent symptom, and embolization may result from chronic atrial fibrillation. With tricuspid and pulmonic valve diseases, stenosis occurs more often than regurgitation. Tricuspid valve stenosis results in right atrial enlargement and elevated systemic venous pressures. Pulmonic valve stenosis results in right ventricular hypertension and hypertrophy.

The nurse evaluates that discharge teaching for a patient hospitalized with pneumonia has been effective when the patient makes which statement about measures to prevent a relapse? a. "I will seek immediate medical treatment for any upper respiratory infections." b. "I should continue to do deep breathing and coughing exercises for at least 12 weeks." c. "I will increase my food intake to 2400 calories a day to keep my immune system well." d. "I must have a follow-up chest x-ray in 6 to 8 weeks to evaluate the pneumonia's resolution."

d. "I must have a follow-up chest x-ray in 6 to 8 weeks to evaluate the pneumonia's resolution." The follow-up chest x-ray examination will be done in 6 to 8 weeks to evaluate pneumonia resolution. A patient should seek medical treatment for upper respiratory infections that persist for more than 7 days. It may be important for the patient to continue with coughing and deep breathing exercises for 6 to 8 weeks, not 12 weeks, until all the infection has cleared from the lungs. Increased fluid intake, not caloric intake, is required to liquefy secretions.

Which statement by the patient with chronic HF should cause the nurse to determine that additional discharge teaching is needed? a. "I will call my health clinic if I wake up breathless at night." b. "I will look for sodium content on labels of foods and over-the-counter medicines." c. "I plan to organize my household tasks so I don't have to constantly go up and down the stairs." d. "I should weigh myself every morning and go on a diet if I gain more than 2 or 3 pounds in 2 days."

d. "I should weigh myself every morning and go on a diet if I gain more than 2 or 3 pounds in 2 days." Further teaching is needed if the patient believes a weight gain of 2 to 3 pounds in 2 days is an indication for dieting. In a patient with HF, this type of weight gain reflects fluid retention and is a sign of HF that should be reported to the HCP. The other options show patient understanding of the HF management teaching

A nurse is caring for a client who has a history of angina and is schedules for a stress test at 1100. Which of the following statements by the client requires the nurse to contact the provider for possible rescheduling? A. "I'm still hungry after the bowl of cereal I ate at 7am." b. "I didn't take my heart pills this morning because the doctor told me not to." c. "I have had chest pain a couple of times since I saw my doctor in the office last week." d. "I smoked a cigarette this morning to calm my nerves about having this procedure."

d. "I smoked a cigarette this morning to calm my nerves about having this procedure." Rationale: Smoking prior to this test can change the outcome and places the client at additional risk. The procedure should be rescheduled if the client has smoked before the test.

A nurse is teaching nutritional strategies to a client who has a low serum calcium level and an allergy to milk. Which of the following statements by the client indicates an understanding of the teaching? a. "I will eat more cheese because I can't drink milk." b. "I need to avoid foods with vitamin D because I am allergic to milk." c. "I will stop taking my calcium supplements if they irritate my stomach."" d. I will add broccoli and kale to my diet."

d. "I will add broccoli and kale to my diet." The nurse should recommend broccoli and kale, which are good sources of calcium as alternatives to milk products.

When teaching the patient about going from a metered-dose inhaler (MDI) to a dry powder inhaler (DPI), which patient statement indicates to the nurse that the patient needs more teaching? a. "I do not need to use the spacer like I used to." b. "I will hold my breath for 10 seconds or longer if I can." c. "I will not shake this inhaler like I did with my old inhaler." d. "I will store it in the bathroom, so I will be able to clean it when I need to."

d. "I will store it in the bathroom, so I will be able to clean it when I need to." Storing the dry powder inhaler (DPI) in the bathroom will expose it to moisture, which could cause clumping of the medication and an altered dose. The other statements show patient understanding.

Following discharge teaching with a male patient with an AAA repair, the nurse determines that further instruction is needed when the patient makes which statement? a. "I should avoid heavy lifting for 6 weeks." b. "I may have some sexual dysfunction because of the surgery." c. "I should maintain a low-fat and low-cholesterol diet to help keep the new graft open." d. "I will take the pulses in my legs and let the doctor know if they get too fast or too slow."

d. "I will take the pulses in my legs and let the doctor know if they get too fast or too slow." Patients are taught to palpate peripheral pulses to identify changes in their quality or strength, but the rate is not a significant factor in peripheral perfusion. The color and temperature of the extremities are important for patients to observe. The remaining statements are true.

A nurse is providing teaching for a client who is at risk for developing respiratory acidosis following surgery. Which of the following statements by the client indicates an understanding of the teaching? a. "I should conserve energy by limiting my physical activity." b. "I will wait until my pain is at least six out of ten before I use the PCA." c. "I will limit my daily fluid intake to two to three glasses." d. "I will use the incentive spirometer every hour."

d. "I will use the incentive spirometer every hour." Respiratory depression and limited chest expansion are both causes of respiratory acidosis. Using an incentive spirometer will promote adequate chest expansion. The nurse should identify this statement as indicating an understanding of the teaching.

A patient with VTE is to be discharged on long-term warfarin (Coumadin) therapy and is taught about prevention and continuing treatment of VTE. The nurse determines that discharge teaching for the patient has been effective when the patient makes which statement? a. "I should expect that Coumadin will cause my stools to be somewhat black." b. "I should avoid all dark green and leafy vegetables while I am taking Coumadin." c. "Massaging my legs several times a day will help increase my venous circulation." d. "Swimming is a good activity to include in my exercise program to increase my circulation."

d. "Swimming is a good activity to include in my exercise program to increase my circulation." Exercise programs for patients recovering from VTE should emphasize swimming, which is particularly beneficial because of the gentle, even pressure of the water. Coumadin will not blacken stools. If this occurs, it could be a sign of gastrointestinal bleeding. Dark green and leafy vegetables have high amounts of vitamin K and should not be increased during Coumadin therapy, but they do not need to be restricted. The legs must not be massaged because of the risk for dislodging any clots that may be present.

Which statement by a patient after mechanical valve replacement indicates to the nurse that further instruction is needed? a. "I may begin an exercise program to gradually increase my cardiac tolerance." b. "I will always need to have my blood checked once a month for its clotting function." c. "I should take prophylactic antibiotics before I have dental or invasive medical procedures." d. "The biggest risk I have during invasive health procedures is bleeding because of my anticoagulants."

d. "The biggest risk I have during invasive health procedures is bleeding because of my anticoagulants." The greatest risk to a patient who has an artificial valve is the development of endocarditis with invasive medical or dental procedures, not bleeding. Before any of these procedures, antibiotic prophylaxis is necessary to prevent infection. Planning of an exercise program and monitoring anticoagulant therapy will be done.

The husband of a patient with severe COPD tells the nurse that he and his wife have not had any sexual activity since she was diagnosed with COPD because she becomes too short of breath. What is the nurse's best response? a. "You need to discuss your feelings and needs with your wife so that she knows what you expect of her." b. "There are other ways to maintain intimacy besides sexual intercourse that will not make her short of breath." c. "You should explore other ways to meet your sexual needs since your wife is no longer capable of sexual activity." d. "Would you like me to talk with you and your wife about some modifications that can be made to maintain sexual activity?"

d. "Would you like me to talk with you and your wife about some modifications that can be made to maintain sexual activity?" Specific guidelines for sexual activity help preserve energy and prevent dyspnea, and maintenance of sexual activity is important to the healthy psychologic well-being of the patient. Open communication between partners is needed so that the modifications can be made with consideration of both partners.

A patient has been receiving oxygen per nasal cannula while hospitalized for COPD. The patient asks the nurse whether oxygen use will be needed at home. What is the most appropriate response by the nurse? a. "Long-term home oxygen therapy should be used to prevent respiratory failure." b. "Oxygen will not be needed until or unless you are in the terminal stages of this disease." c. "Long-term home oxygen therapy should be used to prevent heart problems related to COPD." d. "You will not need oxygen until your oxygen saturation drops to 88% and you have symptoms of hypoxia."

d. "You will not need oxygen until your oxygen saturation drops to 88% and you have symptoms of hypoxia." Long-term oxygen therapy in the home will not be considered until the oxygen saturation is less than or equal to 88% and the patient has signs of tissue hypoxia, such as cor pulmonale, erythrocytosis, or impaired mental status. PaO2 less than 55 mm Hg will also allow home oxygen therapy to be considered.

What accurately describes the PR interval (select all that apply)? a. 0.16 seconds b. < 0.12 seconds c. 0.06 to 0.12 seconds d. 0.12 to 0.20 seconds e. Time of depolarization and repolarization of ventricles f. Measured from beginning of P wave to beginning of QRS complex

d. 0.12 to 0.20 seconds f. Measured from beginning of P wave to beginning of QRS complex The expected PR interval is 0.12 to 0.20 seconds and is measured from the beginning of the P wave to the beginning of the QRS complex. The T wave is 0.16 seconds, the QRS interval is < 0.12 seconds, the P wave is 0.06 to 0.12 seconds, and the QT interval is the time of depolarization and repolarization of the ventricles.

A nurse is caring for a client who has a sodium level of 155 mEq/L. Which of the following IV fluids should the nurse anticipate the provider to prescribe? a. Dextrose 5% in 0.9% sodium chloride b. Dextrose 5% in lactated Ringer's c. 3% sodium chloride d. 0.45% sodium chloride

d. 0.45% sodium chloride A sodium level of 155 mEq/L is an indication of hypernatremia. The nurse should anticipate a prescription for a hypotonic solution. The 0.45% sodium chloride is a hypotonic solution used to provide free water and treat cellular dehydration, which promotes waste elimination by the kidneys.Dextrose 5% in 0.9% sodium chloride is a hypertonic solution. The 3% sodium chloride is a hypertonic solution. Lactated Ringer's solution contains sodium and other electrolytes and is not indicated for hypernatremia.

Priority Decision: A patient hospitalized for evaluation of unstable angina has severe chest pain and calls the nurse. Prioritize the subsequent interventions from 1 (highest priority) to 6 (lowest priority). The appropriate medical prescriptions and protocols are available to the nurse. a. Notify the provider. b. Obtain a 12-lead ECG. c. Check the patient's vital signs. d. Apply oxygen per nasal cannula. e. Perform a focused assessment of the chest. f. Assess pain (PQRST) and medicate as ordered.

d. 1 c. 2 b. 3 f. 4 e. 5 a. 6 A patient having chest pain needs to have the pain assessed and relieved as quickly as possible. Applying oxygen may help relieve the pain. Following an assessment of the vital signs, it is important to know if the pain is accompanied by ECG changes. Then perform a detailed assessment of the pain using PQRST and medicate as ordered. Perform a focused assessment of the heart and lungs before reporting the findings to the provider.

What is the goal of total cholesterol? a. 100 or less b. 40 or more c. 150 or less d. 200 or less

d. 200 or less

A client is admitted to the intensive care unit with thrombocytopenia. The client's platelet count is 120,000 μl. The health care provider (HCP) ordered 5 bags of platelets to be infused on this client. How long should the nurse infuse each bag of platelets? a. 5 to 10 minutes. b. 10 to 20 minutes. c. 20 to 30 minutes. d. 30 to 60 minutes.

d. 30 to 60 minutes. This answer is correct because infusing platelets over 30 to 60 minutes is the recommended time to keep clients from having an infusion reaction.

A CHF pt admitted 3 days ago begins to complain about SOB with this rhythm began-HR 95, BP 89/59, RR 32. Which FIRST? a. Start an Amiodarone IV. b. Start Heparin IV. c. Perform synchronized cardioversion. d. 3L nasal cannula.

d. 3L nasal cannula.

Which leukemia is seen in 80% of adults with acute leukemia and exhibits proliferation of precursors of granulocytes? a. Hairy cell leukemia b. Biphenotypic leukemia c. Acute lymphocytic leukemia (ALL) d. Acute myelogenous leukemia (AML)

d. Acute myelogenous leukemia (AML) Acute myelogenous leukemia (AML) is seen in 80% of adults with acute leukemia and is characterized by hyperplasia of the bone marrow with uncontrolled proliferation of myeloblasts, the precursors of granulocytes. Hairy cell leukemia is a rare cancer with hairy-looking abnormal lymphocytes. Biphenotypic leukemia is a rare form of both types of acute leukemia. Acute lymphocytic leukemia (ALL), the other acute leukemia, is most common in children and is characterized by small, immature lymphocytes, primarily of B-cell origin, proliferated in the bone marrow.

After receiving change-of-shift report about the following four patients, which patient should the nurse assess first? a. 39-year-old with pericarditis who is complaining of sharp, stabbing chest pain b. 56-year-old with variant angina who is to receive a dose of nifedipine (Procardia) c. 65-year-old who had a myocardial infarction (MI) 4 days ago and is anxious about the planned discharge d. 59-year-old with unstable angina who has just returned to the unit after having a percutaneous coronary intervention (PCI)

d. 59-year-old with unstable angina who has just returned to the unit after having a percutaneous coronary intervention (PCI) This patient is at risk for bleeding from the arterial access site for the PCI, so the nurse should assess the patients blood pressure, pulse, and the access site immediately. The other patients should also be assessed as quickly as possible, but assessment of this patient has the highest priority.

The GI tract normally secretes _____ mls daily, most of this is reabsorbed a. 2,000 b. 3,500 c. 6,000 d. 8,000

d. 8,000

The evaluation team for cardiac transplantation is evaluating patients. Which patient is most likely to receive the most benefit from a new heart? a. A 24-year-old man with Down syndrome who has received excellent care from parents in their 60s b. A 46-year-old single woman with a limited support system who has alcohol-induced cardiomyopathy c. A 60-year-old man with inoperable CAD who has not been compliant with lifestyle changes and rehabilitation programs d. A 52-year-old woman with end-stage CAD who has limited financial resources but is emotionally stable and has strong social support

d. A 52-year-old woman with end-stage CAD who has limited financial resources but is emotionally stable and has strong social support The 52-year-old woman does not have any contraindications for cardiac transplantation, even though she lacks the indication of adequate financial resources. The 24-year-old man does not have a current cardiac diagnosis. The postoperative transplant regimen is complex and rigorous, and patients who have not been compliant with other treatments or who may not have the means to understand the care would not be good candidates. A history of drug or alcohol abuse is usually a contraindication to heart transplant.

The nurse has received change-of-shift report about the following patients on the progressive care unit. Which patient should the nurse see first? a. A patient who is in a sinus rhythm, rate 98, after having electrical cardioversion 2 hours ago b. A patient with new onset atrial fibrillation, rate 88, who has a first dose of warfarin (Coumadin) due c. A patient with second-degree atrioventricular (AV) block, type 1, rate 60, who is dizzy when ambulating d. A patient whose implantable cardioverter-defibrillator (ICD) fired two times today who has a dose of amiodarone (Cordarone) due

d. A patient whose implantable cardioverter-defibrillator (ICD) fired two times today who has a dose of amiodarone (Cordarone) due The frequent firing of the ICD indicates that the patients ventricles are very irritable, and the priority is to assess the patient and administer the amiodarone. The other patients may be seen after the amiodarone is administered.

The patient's PR interval comprises 6 small boxes on the ECG graph. What does the nurse determine that this indicates? a. A normal finding b. A problem with ventricular depolarization c. A disturbance in the repolarization of the atria d. A problem with conduction from the SA node to the ventricular cells

d. A problem with conduction from the SA node to the ventricular cells The normal PR interval is 0.12 to 0.20 seconds and reflects the time taken for the impulse to spread through the atria, AV node and bundle of His, the bundle branches, and Purkinje fibers. A PR interval of 6 small boxes is 0.24 second and indicates that the conduction of the impulse from the atria to the Purkinje fibers is delayed.

In reviewing the laboratory results of a patient with hemophilia A, what would the nurse expect to find? a. An absence of factor IX b. A decreased platelet count c. A prolonged bleeding time d. A prolonged partial thromboplastin time (PTT)

d. A prolonged partial thromboplastin time (PTT) A prolonged PTT occurs when there is a deficiency of clotting factors, such as factor VIII associated with hemophilia A. Factor IX is deficient in hemophilia B and prolonged bleeding time, and decreased platelet counts are associated with platelet deficiencies.

Which laboratory test will the nurse use to determine whether filgrastim (Neupogen) is effective for a patient with acute lymphocytic leukemia who is receiving chemotherapy? a. Platelet count b. Reticulocyte count c. Total lymphocyte count d. Absolute neutrophil count

d. Absolute neutrophil count Filgrastim increases the neutrophil count and function in neutropenic patients. Although total lymphocyte, platelet, and reticulocyte counts also are important to monitor in this patient, the absolute neutrophil count is used to evaluate the effects of filgrastim.

Which laboratory result will the nurse expect to show a decreased value if a patient develops heparin- induced thrombocytopenia (HIT)? a. Prothrombin time b. Erythrocyte count c. Fibrinogen degradation products d. Activated partial thromboplastin time

d. Activated partial thromboplastin time Platelet aggregation in HIT causes neutralization of heparin, so that the activated partial thromboplastin time will be shorter and more heparin will be needed to maintain therapeutic levels. The other data will not be affected by HIT.

The patient is being dismissed from the hospital after acute coronary syndrome and will be attending rehabilitation. What information would be taught in the early recovery phase of rehabilitation? a. Therapeutic lifestyle changes should become lifelong habits. b. Physical activity is always started in the hospital and continued at home. c. Attention will focus on management of chest pain, anxiety, dysrhythmias, and other complications. d. Activity level is gradually increased under cardiac rehabilitation team supervision and with electrocardiographic (ECG) monitoring.

d. Activity level is gradually increased under cardiac rehabilitation team supervision and with electrocardiographic (ECG) monitoring. In the early recovery phase after the patient is dismissed from the hospital, the activity level is gradually increased under supervision and with ECG monitoring. The late recovery phase includes therapeutic lifestyle changes that become lifelong habits. In the first phase of recovery, activity is dependent on the severity of the angina or myocardial infarction, and attention is focused on the management of chest pain, anxiety, dysrhythmias, and other complications. With early recovery phase, the cardiac rehabilitation team may suggest that physical activity be initiated at home, but this is not always done.

What is a nursing intervention that is indicated for the patient during a sickle cell crisis? a. Frequent ambulation b. Application of antiembolism hose c. Restriction of sodium and oral fluids d. Administration of large doses of continuous opioid analgesics

d. Administration of large doses of continuous opioid analgesics Because pain is the most common symptom with a sickle cell crisis and may last for 4 to 6 days, pain control is an essential part of treatment. Rest is indicated to reduce metabolic needs. Although thrombosis does occur in capillaries, antiembolism stockings that primarily affect venous circulation are not indicated; anticoagulants are more effective. Fluids and electrolytes are administered to reduce blood viscosity and maintain renal function.

A nurse is caring for a client who was admitted with major burns to the head, neck, and chest. Which of the following complications should the nurse identify as the greatest risk to the client? a. Hypothermia b. Hyponatremia c. Fluid imbalance d. Airway obstruction

d. Airway obstruction

Priority Decision: Which medication should the nurse anticipate being used first in the emergency department for relief of severe respiratory distress related to asthma? a. Prednisone orally b. Tiotropium inhaler c. Fluticasone inhaler d. Albuterol nebulizer

d. Albuterol nebulizer The albuterol nebulizer will rapidly cause bronchodilation and be easier to use in an emergency than an inhaler. It will be used every 20 minutes to 4 hours as needed. The tiotropium inhaler is only approved for chronic obstructive pulmonary disease (COPD). Oral or inhaled corticosteroids will be used to decrease the inflammation and provide better symptom control after the emergency is over.

A patient with an acute MI has sinus tachycardia of 126 bpm. The nurse recognizes that if this dysrhythmia is not treated, what is the worst thing the patient is likely to experience? a. Hypertension b. Escape rhythms c. Ventricular tachycardia d. An increase in infarct size

d. An increase in infarct size Although many factors can cause a sinus tachycardia, in the patient who has had an acute myocardial infarction (MI), tachycardia increases myocardial oxygen need in a heart that already has impaired circulation and may lead to increasing angina and further ischemia and necrosis.

The nurse is preparing an injection of erythropoietin for an 11-month-old client diagnosed with Ewing's sarcoma. Which part of the body is the preferred method for administration? a. Deltoid b. Vastus lateralis c. Abdomen d. Anterior thigh

d. Anterior thigh This answer is correct because the thigh is the most preferred site for injections for children under 12 months. Erythropoietin is given subcutaneously. The drug would be injected into the fatty tissue of the anterior thigh.

Which topic should the nurse include in patient teaching for a patient with a venous stasis ulcer on the left lower leg? a. Need to increase carbohydrate intake b. Methods of keeping the wound area dry c. Purpose of prophylactic antibiotic therapy d. Application of elastic compression stockings

d. Application of elastic compression stockings Compression of the leg is essential to healing of venous stasis ulcers. High dietary intake of protein, rather than carbohydrates, is needed. Prophylactic antibiotics are not routinely used for venous ulcers. Moist dressings are used to hasten wound healing.

A patient develops sinus bradycardia at a rate of 32 beats/minute, has a blood pressure (BP) of 80/42 mm Hg, and is complaining of feeling faint. Which actions should the nurse take next? a. Recheck the heart rhythm and BP in 5 minutes. b. Have the patient perform the Valsalva maneuver. c. Give the scheduled dose of diltiazem (Cardizem). d. Apply the transcutaneous pacemaker (TCP) pads.

d. Apply the transcutaneous pacemaker (TCP) pads. The patient is experiencing symptomatic bradycardia, and treatment with TCP is appropriate. Continued monitoring of the rhythm and BP is an inadequate response. Calcium channel blockers will further decrease the heart rate, and the diltiazem should be held. The Valsalva maneuver will further decrease the rate.

The nurse is planning care and teaching for a patient with venous leg ulcers. What is the most important patient action in healing and control of this condition? a. Following activity guidelines. b. Using moist environment dressings. c. Taking horse chestnut seed extract daily. d. Applying graduated compression stockings.

d. Applying graduated compression stockings.

An alcoholic and homeless patient is diagnosed with active tuberculosis (TB). Which intervention by the nurse will be most effective in ensuring adherence with the treatment regimen? a. Arrange for a friend to administer the medication on schedule. b. Give the patient written instructions about how to take the medications. c. Teach the patient about the high risk for infecting others unless treatment is followed. d. Arrange for a daily noon meal at a community center where the drug will be administered.

d. Arrange for a daily noon meal at a community center where the drug will be administered. Directly observed therapy is the most effective means for ensuring compliance with the treatment regimen, and arranging a daily meal will help ensure that the patient is available to receive the medication. The other nursing interventions may be appropriate for some patients but are not likely to be as helpful for this patient.

A patient who has a history of chronic obstructive pulmonary disease (COPD) was hospitalized for increasing shortness of breath and chronic hypoxemia (SaO2 levels of 89% to 90%). In planning for discharge, which action by the nurse will be most effective in improving compliance with discharge teaching? a. Start giving the patient discharge teaching on the day of admission. b. Have the patient repeat the instructions immediately after teaching. c. Accomplish the patient teaching just before the scheduled discharge. d. Arrange for the patients caregiver to be present during the teaching.

d. Arrange for the patients caregiver to be present during the teaching. Hypoxemia interferes with the patients ability to learn and retain information, so having the patients caregiver present will increase the likelihood that discharge instructions will be followed. Having the patient repeat the instructions will indicate that the information is understood at the time, but it does not guarantee retention of the information. Because the patient is likely to be distracted just before discharge, giving discharge instructions just before discharge is not ideal. The patient is likely to be anxious and even more hypoxemic than usual on the day of admission, so teaching about discharge should be postponed.

A nurse is caring for a client who is pending surgery for his AAA. Vital signs are: 95% O2, HR 102, rr 22, bp 165/99. Which of the following actions should a nurse take? a. Monitor patient urinary output b. Withhold patient's pain meds preop c. Take vitals q1 hour d. Give blood pressure meds

d. Give blood pressure meds

The patient with Parkinson's disease has a pulse oximetry reading of 72% but has no other signs of decreased oxygenation. What is the most likely explanation for the low SpO2 level? a. Anemia b. Dark skin color c. Thick acrylic nails d. Artifact

d. Artifact Motion is the most likely cause of the low SpO2 for this patient with Parkinson's disease. Anemia, dark skin color, and thick acrylic nails as well as low perfusion, bright fluorescent lights, and intravascular dyes may also cause an inaccurate pulse oximetry result. There is no mention of these or reason to suspect these in this question.

A patient who is taking a potassium-wasting diuretic for treatment of hypertension complains of generalized weakness. It is most appropriate for the nurse to take which action? a. Assess for facial muscle spasms. b. Ask the patient about loose stools. c. Suggest that the patient avoid orange juice with meals. d. Ask the health care provider to order a basic metabolic panel.

d. Ask the health care provider to order a basic metabolic panel. Generalized weakness is a manifestation of hypokalemia. After the health care provider orders the metabolic panel, the nurse should check the potassium level. Facial muscle spasms might occur with hypocalcemia. Orange juice is high in potassium and would be advisable to drink if the patient was hypokalemic. Loose stools are associated with hyperkalemia.

Priority Decision: A patient hospitalized for 1 week with subacute infective endocarditis is afebrile and has no signs of heart damage. Discharge with outpatient antibiotic therapy is planned. During discharge planning with the patient, what is it most important for the nurse to do? a. Plan how his needs will be met while he continues on bed rest. b. Encourage the use of diversional activities to relieve boredom and restlessness. c. Teach the patient to avoid crowds and exposure to upper respiratory infections. d. Assess the patient's home environment in terms of family assistance and hospital access.

d. Assess the patient's home environment in terms of family assistance and hospital access. The patient with outpatient antibiotic therapy needs vigilant home nursing care. It is most important to determine the adequacy of the home environment for successful management of the patient. The patient is at risk for life-threatening complications, such as embolization and pulmonary edema, and must be able to access a hospital if needed. Bed rest will not be necessary for the patient without heart damage. Avoiding infections and planning diversional activities are indicated for the patient but are not the most important factors while he is on outpatient antibiotic therapy.

Which nursing action would be of highest priority when suctioning a patient with a tracheostomy? a. Auscultating lung sounds after suctioning is complete b. Giving antianxiety medications 30 minutes before suctioning c. Instilling 5 mL of normal saline into the tracheostomy tube before suctioning d. Assessing the patient's oxygen saturation before, during, and after suctioning

d. Assessing the patient's oxygen saturation before, during, and after suctioning

The health care provider orders a liver/spleen scan for a patient who has been in a motor vehicle accident. Which action should the nurse take before this procedure? a. Check for any iodine allergy. b. Insert a large-bore IV catheter. c. Place the patient on NPO status. d. Assist the patient to a flat position.

d. Assist the patient to a flat position. During a liver/spleen scan, a radioactive isotope is injected IV and images from the radioactive emission are used to evaluate the structure of the spleen and liver. An indwelling IV catheter is not needed. The patient is placed in a flat position before the scan.

The nurse palpates the posterior chest while the patient says 99 and notes absent fremitus. Which action should the nurse take next? a. Palpate the anterior chest and observe for barrel chest. b. Encourage the patient to turn, cough, and deep breathe. c. Review the chest x-ray report for evidence of pneumonia. d. Auscultate anterior and posterior breath sounds bilaterally

d. Auscultate anterior and posterior breath sounds bilaterally To assess for tactile fremitus, the nurse should use the palms of the hands to assess for vibration when the patient repeats a word or phrase such as 99. After noting absent fremitus, the nurse should then auscultate the lungs to assess for the presence or absence of breath sounds. Absent fremitus may be noted with pneumothorax or atelectasis. The vibration is increased in conditions such as pneumonia, lung tumors, thick bronchial secretions, and pleural effusion. Turning, coughing, and deep breathing is an appropriate intervention for atelectasis, but the nurse needs to first assess breath sounds. Fremitus is decreased if the hand is farther from the lung or the lung is hyperinflated (barrel chest).The anterior of the chest is more difficult to palpate for fremitus because of the presence of large muscles and breast tissue.

What describes the SA node's ability to discharge an electrical impulse spontaneously? a. Excitability b. Contractility c. Conductivity d. Automaticity

d. Automaticity Automaticity describes the ability to discharge an electrical impulse spontaneously. Excitability is a property of myocardial tissue that enables it to be depolarized by an impulse. Contractility is the ability of the chambers to respond mechanically to an impulse. Conductivity is the ability to transmit an impulse along a membrane.

To determine the effects of therapy for a patient who is being treated for heart failure, which laboratory result will the nurse plan to review? a. Troponin b. Homocysteine (Hcy) c. Low-density lipoprotein (LDL) d. B-type natriuretic peptide (BNP)

d. B-type natriuretic peptide (BNP) Increased levels of BNP are a marker for heart failure. The other laboratory results would be used to assess for myocardial infarction (troponin) or risk for coronary artery disease (Hcy and LDL)

When assessing the respiratory system of an older patient, which finding indicates that the nurse should take immediate action? a. Weak cough effort b. Barrel-shaped chest c. Dry mucous membranes d. Bilateral crackles at lung bases

d. Bilateral crackles at lung bases Crackles in the lower half of the lungs indicate that the patient may have an acute problem such as heart failure. The nurse should immediately accomplish further assessments, such as oxygen saturation, and notify the health care provider. A barrel-shaped chest, hyperresonance to percussion, and a weak cough effort are associated with aging. Further evaluation may be needed, but immediate action is not indicated. An older patient has a less forceful cough and fewer and less functional cilia. Mucous membranes tend to be drier.

Which respiratory assessment finding does the nurse interpret as abnormal? a. Inspiratory chest expansion of 1 inch b. Symmetric chest expansion and contraction c. Resonance (to percussion) over the lung bases d. Bronchial breath sounds in the lower lung fields

d. Bronchial breath sounds in the lower lung fields

Priority Decision: What should the nurse emphasize when teaching women about the risks and incidence of CAD? a. Smoking is not as significant a risk factor for CAD in women as it is in men. b. Women seek treatment sooner than do men when they have symptoms of CAD. c. Estrogen replacement therapy in postmenopausal women decreases the risk for CAD. d. CAD is the leading cause of death in women, with a higher mortality rate after myocardial infarction (MI) than in men

d. CAD is the leading cause of death in women, with a higher mortality rate after myocardial infarction (MI) than in men CAD is the number-one killer of American women, and women have a much higher mortality rate within 1 year following MI than do men. Smoking carries specific problems for women because smoking has been linked to a decrease in natural estrogen levels and to early menopause, and it has been identified as the most powerful contributor to CAD in women under the age of 50 years. Fewer women than men present with classic manifestations, and women delay seeking care longer than men. Estrogen replacement does not always reduce the risk for CAD, even though natural estrogen lowers low-density lipoprotein (LDL) and raises high-density lipoprotein (HDL) cholesterol.

A thoracic aortic aneurysm is found when a patient has a routine chest x-ray. The nurse anticipates that additional diagnostic testing to determine the size and structure of the aneurysm will include which test? a. Angiography b. Ultrasonography c. Echocardiography d. CT scan

d. CT scan A CT scan is the most accurate test to determine the length and diameter of the aneurysm and whether a thrombus is present. The other tests may also be used, but the CT scan yields the most descriptive results.

What is a priority nursing intervention in the care of a patient with a diagnosis of chronic venous insufficiency (CVI)? a Application of topical antibiotics to venous ulcers b Maintaining the patient's legs in a dependent position c Administration of oral and/or subcutaneous anticoagulants d Teaching the patient the correct use of compression stockings

d. CVI requires conscientious and consistent application of compression stockings. Anticoagulants are not necessarily indicated and antibiotics, if required, are typically oral or IV, not topical. The patient should avoid prolonged positioning with the limb in a dependent position.

In a patient with a positive Chvostek's sign, the nurse would expect the IV administration of which medication? a. Calcitonin b. Vitamin D c. Loop diuretics d. Calcium gluconate

d. Calcium gluconate Chvostek's sign is a contraction of facial muscles in response to a tap over the facial nerve. This indicates the neuromuscular irritability of low calcium levels. IV calcium is the treatment used to prevent laryngeal spasms and respiratory arrest. Calcitonin and loop diuretics are treatments for hypercalcemia. Oral vitamin D supplements are part of the treatment for hypocalcemia but not for impending tetany.

A patients cardiac monitor shows sinus rhythm, rate 64. The P-R interval is 0.18 seconds at 1:00 AM, 0.22 seconds at 2:30 PM, and 0.28 seconds at 4:00 PM. Which action should the nurse take next? a. Place the transcutaneous pacemaker pads on the patient. b. Administer atropine sulfate 1 mg IV per agency dysrhythmia protocol. c. Document the patients rhythm and assess the patients response to the rhythm. d. Call the health care provider before giving the next dose of metoprolol (Lopressor).

d. Call the health care provider before giving the next dose of metoprolol (Lopressor). The patient has progressive first-degree atrioventricular (AV) block, and the b-blocker should be held until discussing the medication with the health care provider. Documentation and assessment are appropriate but not fully adequate responses. The patient with first-degree AV block usually is asymptomatic, and a pacemaker is not indicated. Atropine is sometimes used for symptomatic bradycardia, but there is no indication that this patient is symptomatic.

A patient who has had chest pain for several hours is admitted with a diagnosis of rule out acute myocardial infarction (AMI). Which laboratory test should the nurse monitor to help determine whether the patient has had an AMI? a. Myoglobin b. Homocysteine c. C-reactive protein d. Cardiac-specific troponin

d. Cardiac-specific troponin Troponin levels increase about 4 to 6 hours after the onset of myocardial infarction (MI) and are highly specific indicators for MI. Myoglobin is released within 2 hours of MI, but it lacks specificity and its use is limited. The other laboratory data are useful in determining the patients risk for developing coronary artery disease (CAD) but are not helpful in determining whether an acute MI is in progress.

A patient who has just been admitted with pulmonary edema is scheduled to receive the following medications. Which medication should the nurse question before giving? a. Furosemide (Lasix) 60 mg b. Captopril (Capoten) 25 mg c. Digoxin (Lanoxin) 0.125 mg d. Carvedilol (Coreg) 3.125 mg

d. Carvedilol (Coreg) 3.125 mg Although carvedilol is appropriate for the treatment of chronic heart failure, it is not used for patients with acute decompensated heart failure (ADHF) because of the risk of worsening the heart failure. The other medications are appropriate for the patient with ADHF.

In a patient with sodium imbalances, the primary clinical manifestations are related to alterations in what body system? a. Kidneys b. Cardiovascular system c. Musculoskeletal system d. Central nervous system

d. Central nervous system As water shifts into and out of cells in response to the osmolality of the blood, the cells that are most sensitive to shrinking or swelling are those of the brain, resulting in neurologic symptoms.

A patient has just been diagnosed with hypertension and has been started on captopril (Capoten). Which information is important to include when teaching the patient about this medication? a. Check blood pressure (BP) in both arms before taking the medication. b. Increase fluid intake if dryness of the mouth is a problem. c. Include high-potassium foods such as bananas in the diet. d. Change position slowly to help prevent dizziness and falls.

d. Change position slowly to help prevent dizziness and falls. The angiotensin-converting enzyme (ACE) inhibitors frequently cause orthostatic hypotension, and patients should be taught to change position slowly to allow the vascular system time to compensate for the position change. Increasing fluid intake may counteract the effect of the medication, and the patient is taught to use gum or hard candy to relieve dry mouth. The BP should be taken in the nondominant arm by newly diagnosed patients in the morning, before taking the medication, and in the evening. Because ACE inhibitors cause potassium retention, increased intake of high-potassium foods is inappropriate.

After 4 days with palpitations & dyspnea, the pt presents to the ER with this rhythm. What should the nurse do FIRST? a. Perform synchronized cardioversion. b. Start Nitroglycerin IV. c. Perform radiofrequency catheter ablation immediately. d. Check PTT and start Heparin IV

d. Check PTT and start Heparin IV

A patient is being evaluated for post-thrombotic syndrome. Which assessment will the nurse perform? a. Ask about leg pain with exercise. b. Determine the ankle-brachial index. c. Assess capillary refill in the patients toes. d. Check for presence of lipodermatosclerosis.

d. Check for presence of lipodermatosclerosis. Clinical signs of post-thrombotic syndrome include lipodermatosclerosis. In this situation, the skin on the lower leg becomes scarred, and the leg becomes tapered like an inverted bottle. The other assessments would be done for patients with peripheral arterial disease.

Which action could the nurse delegate to unlicensed assistive personnel (UAP) trained as electrocardiogram (ECG) technicians working on the cardiac unit? a. Select the best lead for monitoring a patient with an admission diagnosis of Dressler syndrome. b. Obtain a list of herbal medications used at home while admitting a new patient with pericarditis. c. Teach about the need to monitor the weight daily for a patient who has hypertrophic cardiomyopathy. d. Check the heart monitor for changes in rhythm while a patient who had a valve replacement ambulates.

d. Check the heart monitor for changes in rhythm while a patient who had a valve replacement ambulates. Under the supervision of registered nurses (RNs), UAP check the patients cardiac monitor and obtain information about changes in heart rate and rhythm with exercise. Teaching and obtaining information about home medications (prescribed or complementary) and selecting the best leads for monitoring patients require more critical thinking and should be done by the RN.

A nurse is completing an assessment for unstable angina. What should the nurse expect? a. Chest pain that is relieved with rest b. Nitroglycerin relieves it c. Physical exertion does not cause chest pain d. Chest pain lasting longer than 15 minute

d. Chest pain lasting longer than 15 minute

When caring for a patient with acute coronary syndrome who has returned to the coronary care unit after having angioplasty with stent placement, the nurse obtains the following assessment data. Which data indicate the need for immediate action by the nurse? a. Heart rate 102 beats/min b. Pedal pulses 1+ bilaterally c. Blood pressure 103/54 mm Hg d. Chest pain level 7 on a 0 to 10 point scale

d. Chest pain level 7 on a 0 to 10 point scale The patients chest pain indicates that restenosis of the coronary artery may be occurring and requires immediate actions, such as administration of oxygen and nitroglycerin, by the nurse. The other information indicates a need for ongoing assessments by the nurse.

A patient with severe chronic obstructive pulmonary disease (COPD) tells the nurse, I wish I were dead! I'm just a burden on everybody. Based on this information, which nursing diagnosis is most appropriate? a. Complicated grieving related to expectation of death b. Ineffective coping related to unknown outcome of illness c. Deficient knowledge related to lack of education about COPD d. Chronic low self-esteem related to increased physical dependence

d. Chronic low self-esteem related to increased physical dependence The patients statement about not being able to do anything for himself or herself supports this diagnosis. Although deficient knowledge, complicated grieving, and ineffective coping may also be appropriate diagnoses for patients with COPD, the data for this patient do not support these diagnoses.

Which component of normal hemostasis involves the processes of protein C and protein S and plasminogen? a. Activation b. Aggregation c. Platelet plug formation d. Clot retraction and dissolution

d. Clot retraction and dissolution Protein C and protein S are examples of anticoagulants that are involved in clot retraction and dissolution. Fibrinolysis also keeps blood in its fluid form by thrombin-activating conversion of plasminogen to plasmin. Plasmin attacks fibrin or fibrinogen and splits it into smaller elements known as fibrin split products (FSPs) or fibrin degradation products (FDPs).

The strict vegetarian is at highest risk for the development of which anemia? a. Thalassemia b. Iron-deficiency anemia c. Folic acid deficiency anemia d. Cobalamin deficiency anemia

d. Cobalamin deficiency anemia Because red meats are the primary dietary sources of cobalamin, a strict vegetarian is most at risk for cobalamin deficiency anemia. Meats are also an important source of iron and folic acid, but whole grains, legumes, and green leafy vegetables also supply these nutrients. Thalassemia is not related to dietary deficiencies.

A 73-year-old patient has an SpO2 of 70%. What other assessment should the nurse consider before making a judgment about the adequacy of the patient's oxygenation? a. What the oxygenation status is with a stress test b. Trend and rate of development of the hyperkalemia c. Comparison of patient's SpO2 values with the normal values d. Comparison of patient's current vital signs with normal vital signs

d. Comparison of patient's current vital signs with normal vital signs The respiratory rate, pulse rate, and BP will all increase with decreased oxygenation when compared to the patient's own normal results. The position of the oximeter should also be assessed. The oxygenation status with a stress test would not assist the nurse in caring for the patient now. Hyperkalemia is not occurring and will not directly affect oxygenation initially. The arterial oxygen saturation by pulse oximetry (SpO2 ) compared with normal values will not be helpful in this older patient or in a patient with respiratory disease as the patient's expected normal will not be the same as standard normal values.

Patient with difficulty swallowing is hospitalized and at risk for aspiration pneumonia. What nursing intervention is indicated to prevent pneumonia? a. Position to side, protect airway b. Check placement of the tube before feeding and residual feeding; keep head of bed up after feedings or continuously with continuous feedings c. Check gag reflex before feeding or offering fluids d. Cut food in small bites, encourage thorough chewing, and provide soft foods that are easier to swallow than liquids

d. Cut food in small bites, encourage thorough chewing, and provide soft foods that are easier to swallow than liquids

Which observation should indicate to the nurse the presence of the complication of graft thrombosis after an AAA repair? a. Cardiac dysrhythmias or chest pain b. Absent bowel sounds, abdominal distention, or diarrhea c. Increased temperature and increased white blood cell count d. Decreased pulses and cool, painful extremities below the level of repair

d. Decreased pulses and cool, painful extremities below the level of repair Decreased or absent pulses in conjunction with cool, painful extremities below the level of repair indicate graft thrombosis. Dysrhythmias or chest pain indicates myocardial ischemia. Absent bowel sounds, abdominal distention, diarrhea, or bloody stools indicate bowel infarction. Increased temperature and white blood cells (WBCs), surgical site inflammation, or drainage indicates graft infection.

After swallowing, a 73-yr-old patient is coughing and has a wet voice. What changes of aging could be contributing to this abnormal finding? a. Decreased response to hypercapnia b. Decreased number of functional alveoli c. Increased calcification of costal cartilage d. Decreased respiratory defense mechanisms

d. Decreased respiratory defense mechanisms Aspiration occurs more easily in the older patient related to decreased respiratory defense mechanisms (e.g., decreases in immunity, ciliary function, cough force, sensation in pharynx). Changes of aging include a decreased response to hypercapnia, decreased number of functional alveoli, and increased calcification of costal cartilage, but these do not increase the risk of aspiration.

What is a compensatory mechanism for metabolic alkalosis? a. Shifting of bicarbonate into cells in exchange for chloride b. Kidney conservation of bicarbonate and excretion of hydrogen ions c. Deep, rapid respirations (Kussmaul respirations) to increase CO2 excretion d. Decreased respiratory rate and depth to retain CO2 and kidney excretion of bicarbonate

d. Decreased respiratory rate and depth to retain CO2 and kidney excretion of bicarbonate Decreased respiratory rate and kidney excretion of HCO3 − compensates for metabolic alkalosis. Shifting of bicarbonate for Cl − may buffer acute respiratory alkalosis. The kidney conserves bicarbonate and excretes hydrogen to compensate for respiratory acidosis. Kussmaul respirations occur with metabolic acidosis to compensate.

What stimulates aldosterone secretion from the adrenal cortex? a. Excessive water intake b. Increased serum osmolality c. Decreased serum potassium d. Decreased sodium and water

d. Decreased sodium and water Aldosterone is secreted by the adrenal cortex in response to a decrease in plasma volume (loss of water) and resulting decreased renal perfusion; decreased serum sodium, increased serum potassium, or adrenocorticotropic hormone (ACTH).

The nurse is preparing the patient for and will assist the health care provider with a thoracentesis in the patient's room. Which of the following is the second action? a. Verify breath sounds in all fields. b. Obtain the supplies that will be used. c. Send labeled specimen containers to the laboratory. d. Direct the family members to the waiting room. e. Observe for signs of hypoxia during the procedure. f. Instruct the patient not to talk during the procedure. g. Position the patient sitting upright with the elbows on an over-the bed table.

d. Direct the family members to the waiting room. Hopefully the family will have some time to discuss this before they are instructed to leave the room, unless it is an emergency.

A patient who has been receiving a heparin infusion and warfarin (Coumadin) for a deep vein thrombosis (DVT) is diagnosed with heparin-induced thrombocytopenia (HIT) when her platelet level drops to 110,000/L. Which action will the nurse include in the plan of care? a. Use low-molecular-weight heparin (LMWH) only. b. Administer the warfarin (Coumadin) at the scheduled time. c. Teach the patient about the purpose of platelet transfusions. d. Discontinue heparin and flush intermittent IV lines using normal saline.

d. Discontinue heparin and flush intermittent IV lines using normal saline. All heparin is discontinued when the HIT is diagnosed. The patient should be instructed to never receive heparin or LMWH. Warfarin is usually not given until the platelet count has returned to 150,000/L. The platelet count does not drop low enough in HIT for a platelet transfusion, and platelet transfusions increase the risk for thrombosis.

While doing the admission assessment for a thin 76-year-old patient, the nurse observes pulsation of the abdominal aorta in the epigastric area. Which action should the nurse take? a. Teach the patient about aneurysms. b. Notify the hospital rapid response team. c. Instruct the patient to remain on bed rest. d. Document the finding in the patient chart.

d. Document the finding in the patient chart. Visible pulsation of the abdominal aorta is commonly observed in the epigastric area for thin individuals. The nurse should simply document the finding in the admission assessment. Unless there are other abnormal findings (such as a bruit, pain, or hyper/hypotension) associated with the pulsation, the other actions are not necessary.

A client with no history of CV disease comes into the ambulatory clinic with flulike symptoms. The client suddenly complains of chest pain. Which of the following questions would best help a nurse to discriminate pain cause by a non-cardiac problem? a. Have you ever had this pain before? b. Can you describe the pain to me c. Can you rate this pain on a pain scale d. Does the pain get worse when you breathe in?

d. Does the pain get worse when you breathe in?

During the physical assessment of the patient with severe anemia, which finding is of the most concern to the nurse? a. Anorexia b. Bone pain c. Hepatomegaly d. Dyspnea at rest

d. Dyspnea at rest Dyspnea at rest indicates that the patient is making an effort to provide adequate amounts of oxygen to the tissues. If oxygen needs are not met, angina, myocardial infarction, heart failure, and pulmonary and systemic congestion can occur. The other manifestations are present in severe anemia, but they do not reflect hypoxemia, a priority problem.

Which statements accurately describe dilated CMP (select all that apply)? a. Characterized by ventricular stiffness b. The least common type of CMP c. The hyperdynamic systolic function creates a diastolic failure d. Echocardiogram reveals cardiomegaly with thin ventricular walls e. Often follows an infective myocarditis or exposure to toxins or drugs f. Differs from chronic heart failure in that there is no ventricular hypertrophy

d. Echocardiogram reveals cardiomegaly with thin ventricular walls e. Often follows an infective myocarditis or exposure to toxins or drugs f. Differs from chronic heart failure in that there is no ventricular hypertrophy Dilated CMP, the most common type of CMP, reveals cardiomegaly with thin ventricular walls on echocardiogram, as there is no ventricular hypertrophy, and may follow an infective myocarditis. In addition, stasis of blood in the ventricles may contribute to systemic embolization. Restrictive CMP is the least common type and is characterized by ventricular stiffness. Hypertrophic CMP is characterized by massive ventricular hypertrophy and rapid, forceful contraction of the left ventricle, impaired relaxation (diastole), and obstructed aortic valve outflow.

Your patient is in bed & suddenly becomes SOB. What do you do first? a. Call the HCP b. Apply supplemental oxygen c. Turn the patient to the side d. Elevate the head of the bed(HOB)

d. Elevate the head of the bed(HOB)

The nurse is caring for a patient who has a calcium level of 12.1 mg/dL. Which nursing action should the nurse include on the care plan? a. Maintain the patient on bed rest. b. Auscultate lung sounds every 4 hours. c. Monitor for Trousseaus and Chvostek's signs. d. Encourage fluid intake up to 4000 mL every day.

d. Encourage fluid intake up to 4000 mL every day. To decrease the risk for renal calculi, the patient should have a fluid intake of 3000 to 4000 mL daily. Ambulation helps decrease the loss of calcium from bone and is encouraged in patients with hypercalcemia. Trousseaus and Chvostek's signs are monitored when there is a possibility of hypocalcemia. There is no indication that the patient needs frequent assessment of lung sounds, although these would be assessed every shift.

Which function test fits description "The volume of air in the lungs after normal exhalation"? a. FVC b. PEFR c. RV d. FRC

d. FRC Functional residual capacity

The acronym FACES is used to help teach patients to identify early symptoms of HF. What does this acronym mean? a. Frequent activity leads to cough in the elderly and swelling b. Factors of risk: activity, cough, emotional upsets, salt intake c. Follow activity plan, continue exercise, and know signs of problems d. Fatigue, limitation of activities, chest congestion/cough, edema, shortness of breath

d. Fatigue, limitation of activities, chest congestion/cough, edema, shortness of breath FACES is used to teach patients to identify early HF symptoms. F = Fatigue; A = Activity limitations; C = Chest congestion/cough; E = Edema; S = Shortness of breath. The other options are not correct.

Which drugs are most commonly used to treat hypertensive crises? a. Labetalol and bumetanide (Bumex) b. Esmolol (Brevibloc) and captopril (Captopril) c. Enalaprilat (Vasotec) and minoxidil (Minoxidil) d. Fenoldopam (Corlopam) and sodium nitroprusside (Nitropress)

d. Fenoldopam (Corlopam) and sodium nitroprusside (Nitropress) Hypertensive crises are treated with IV administration of antihypertensive drugs, including the vasodilators sodium nitroprusside, fenoldopam, and nicardipine; adrenergic blockers, such as labetalol, esmolol, and phentolamine; the ACE inhibitor enalaprilat; the calcium channel blocker clevidipine; nitroglycerin for myocardial ischemia; hydralazine with other medications; and oral captopril. Sodium nitroprusside is the most effective parenteral drug for hypertensive emergencies

Which statement describes Janewayʼs lesions as manifestations of infective endocarditis? a. Hemorrhagic retinal lesions b. Black longitudinal streaks in nail beds c. Painful red or purple lesions on fingers or toes d. Flat, red, painless spots on the palm of hands and soles of feet

d. Flat, red, painless spots on the palm of hands and soles of feet Janeway's lesions are flat, painless, small red spots found on the fingertips, palms of hands, the soles of feet, and toes. Hemorrhagic retinal lesions are Roth's spots. Black streaks on the nails are splinter hemorrhages. Painful lesions on the fingertips and toes are Osler's nodes.

A 20-year-old female patient is in the emergency department for anorexia and fatigue. She takes phenytoin (Dilantin) for a seizure disorder and oral contraceptives. For which type of anemia is this patient most at risk? a. Aplastic anemia b. Hemolytic anemia c. Iron-deficiency anemia d. Folic acid deficiency anemia

d. Folic acid deficiency anemia Folic acid deficiency megaloblastic anemia is related to dietary deficiency as seen in anorexia and with the use of oral contraceptives and antiseizure medications. The other anemias are unrelated to this patient's history.

What is a significant finding in the health history of a patient during an assessment of the cardiovascular system? a. Metastatic cancer b. Calcium supplementation c. Frequent viral pharyngitis d. Frequent use of recreational drugs

d. Frequent use of recreational drugs Recreational or abused drugs, especially stimulants, such as cocaine and methamphetamine, are a growing cause of dysrhythmias and problems associated with tachycardia. IV injection of abused drugs is a risk factor for inflammatory and infectious conditions of the heart. Although calcium is involved in the contraction of muscles, calcium supplementation is not a significant factor in heart disease, nor is metastatic cancer. Streptococcal, but not viral, pharyngitis is a risk factor for rheumatic heart disease.

A client is admitted to the hospital with acid-base imbalances. ABG results are pH: 7.35, PaCO2: 52, HCO3: 30. How should the nurse interpret these findings? a. Uncompensated metabolic acidosis b. Partially compensated respiratory acidosis c. Uncompensated respiratory alkalosis d. Fully compensated respiratory acidosis

d. Fully compensated respiratory acidosis The pH is "normal" but trending towards acid side, and the PaCO2 is elevated (acid), so it is fully compensated respiratory acidosis. Note: Fully compensated we have normal pH & abnormal CO2 and HCO3. Partially compensated we have abnormal CO2 and HCO3, but our pH is partial to one over the other. Uncompensated is when we have a normal CO2 and abnormal pH & HCO3.

Priority Decision: A patient is admitted to the emergency department with ADHF. Which IV medication would the nurse expect to administer first? a. Digoxin (Lanoxin) b. Morphine sulfate c. Nesiritide (Natrecor) d. Furosemide (Lasix)

d. Furosemide (Lasix) Diuretics are the first line for treating patients with volume overload. They decrease sodium reabsorption at various sites within the kidneys, enhancing sodium and water loss. Decreasing intravascular volume with diuretics reduces volume returning to the LV (preload). This allows for more efficient LV pumping, decreased pulmonary vascular pressures, and improved alveolar gas exchange. IV Nesiritide is a recombinant form of BNP used for short-term treatment of acute decompensated heart failure (ADHF) after a failed response to IV diuretics. Digoxin requires a loading dose and time to work, so it is not recommended for emergency treatment of ADHF. Morphine sulfate relieves dyspnea but has more adverse effects.

Which aneurysm is uniform in shape and a circumferential dilation of the artery? a. False aneurysm b. Pseudoaneurysm c. Saccular aneurysm d. Fusiform aneurysm

d. Fusiform aneurysm The fusiform aneurysm is circumferential and relatively uniform in shape. The false aneurysm or pseudoaneurysm is not an aneurysm but a disruption of all the arterial wall layers with bleeding that is contained by surrounding anatomic structures. Saccular aneurysms are the pouchlike bulge of an artery.

Which of the following factors can worsen asthma symptoms? a. Peripheral vascular disease b. Hygiene hypothesis c. During exercise d. Gastroesophageal reflux disease

d. Gastroesophageal reflux disease

Postural drainage with percussion and vibration is ordered twice daily for a patient with chronic bronchitis. Which intervention should the nurse include in the plan of care? a. Schedule the procedure 1 hour after the patient eats. b. Maintain the patient in the lateral position for 20 minutes. c. Perform percussion before assisting the patient to the drainage position. d. Give the ordered albuterol (Proventil) before the patient receives the therapy.

d. Give the ordered albuterol (Proventil) before the patient receives the therapy. Bronchodilators are administered before chest physiotherapy. Postural drainage, percussion, and vibration should be done 1 hour before or 3 hours after meals. Patients remain in each postural drainage position for 5 minutes. Percussion is done while the patient is in the postural drainage position.

A patient with hyperlipidemia has a new order for colesevelam (Welchol). Which nursing action is most appropriate when giving the medication? a. Have the patient take this medication with an aspirin. b. Administer the medication at the patients usual bedtime. c. Have the patient take the colesevelam with a sip of water. d. Give the patients other medications 2 hours after the colesevelam.

d. Give the patients other medications 2 hours after the colesevelam. The bile acid sequestrants interfere with the absorption of many other drugs, and giving other medications at the same time should be avoided. Taking an aspirin concurrently with the colesevelam may increase the incidence of gastrointestinal side effects such as heartburn. An increased fluid intake is encouraged for patients taking the bile acid sequestrants to reduce the risk for constipation. For maximum effect, colesevelam should be administered with meals.

Twenty-four hours after a patient had a tracheostomy, the tube is accidentally dislodged after a coughing episode. Which action should the nurse take first? a. Call the health care provider. b. Place obturator in the tracheostomy tube. c. Position patient in a semi-Fowler's position. d. Grasp the retention sutures to spread the tracheostomy opening.

d. Grasp the retention sutures to spread the tracheostomy opening. However, the nurse should first grasp the retention sutures to spread the tracheostomy opening.

Which patient is at risk for hypernatremia? a. Has an aldosterone deficiency b. Has prolonged vomiting and diarrhea c. Receives excessive IV 5% dextrose solution d. Has impaired consciousness and decreased thirst sensitivity

d. Has impaired consciousness and decreased thirst sensitivity A major cause of hypernatremia is a water deficit, which can occur in those with a decreased sensitivity to thirst, the major protection against hyperosmolality. All other conditions lead to hyponatremia

A patient with pancytopenia has a bone marrow aspiration from the left posterior iliac crest. Which action would be important for the nurse to take after the procedure? a. Elevate the head of the bed to 45 degrees. b. Apply a sterile 2-inch gauze dressing to the site. c. Use a half-inch sterile gauze to pack the wound. d. Have the patient lie on the left side for 1 hour.

d. Have the patient lie on the left side for 1 hour. To decrease the risk for bleeding, the patient should lie on the left side for 30 to 60 minutes. After a bone marrow biopsy, the wound is small and will not be packed with gauze. A pressure dressing is used to cover the aspiration site. There is no indication to elevate the patients head.

The nurse obtains a health history from a 65-year-old patient with a prosthetic mitral valve who has symptoms of infective endocarditis (IE). Which question by the nurse is most appropriate? a. Do you have a history of a heart attack? b. Is there a family history of endocarditis? c. Have you had any recent immunizations? d. Have you had dental work done recently?

d. Have you had dental work done recently? Dental procedures place the patient with a prosthetic mitral valve at risk for infective endocarditis (IE). Myocardial infarction (MI), immunizations, and a family history of endocarditis are not risk factors for IE.

The nurse is performing tuberculosis (TB) skin tests in a clinic that has many patients who have immigrated to the United States. Which question is most important for the nurse to ask before the skin test? a. Is there any family history of TB? b. How long have you lived in the United States? c. Do you take any over-the-counter (OTC) medications? d. Have you received the bacille Calmette-Gurin (BCG) vaccine for TB?

d. Have you received the bacille Calmette-Gurin (BCG) vaccine for TB? Patients who have received the BCG vaccine will have a positive Mantoux test. Another method for screening (such as a chest x-ray) will need to be used in determining whether the patient has a TB infection. The other information also may be valuable but is not as pertinent to the decision about doing TB skin testing.

A 55-year-old patient with increasing dyspnea is being evaluated for a possible diagnosis of chronic obstructive pulmonary disease (COPD). When teaching a patient about pulmonary function testing (PFT) for this condition, what is the most important question the nurse should ask? a. Are you claustrophobic? b. Are you allergic to shellfish? c. Do you have any metal implants or prostheses? d. Have you taken any bronchodilators in the past 6 hours?

d. Have you taken any bronchodilators in the past 6 hours? Pulmonary function testing will help establish the COPD diagnosis. Bronchodilators should be avoided at least 6 hours before the test. PFTs do not involve being placed in an enclosed area such as for magnetic resonance imaging (MRI). Contrast dye is not used for PFTs. The patient may still have PFTs done if metal implants or prostheses are present, as these are contraindications for an MRI.

A 52-year-old man is admitted to the emergency department with severe chest pain. On what basis would the nurse suspect an MI? a. He has pale, cool, clammy skin. b. He reports nausea and vomited once at home. c. He says he is anxious and has a feeling of impending doom. d. He reports he has had no relief of the pain with rest or position change.

d. He reports he has had no relief of the pain with rest or position change. The subjective report of the pain from an MI is usually severe. It usually is unrelieved by NTG, rest, or position change and usually lasts more than the 15 or 20 minutes typical of angina pain. All the other symptoms may occur with angina as well as with an MI.

A nurse is assessing a client who has Acute Respiratory Distress Syndrome (ARDS). Which of the following findings should the nurse report to the provider? a. Decreased bowel sounds b. Oxygen saturation 92% c. CO2 24 mEq/L d. Intercostal retractions

d. Intercostal retractions rationale: The nurse should report intercostal retractions to the provider because this finding indicates increasing respiratory compromise in a client who has ARDS.

What is a possible cause for auscultation abnormal finding "fine crackles"? a. Lung consolidation with fluid or exudate b. Air trapping c. Atelactasis d. Interstitial edema

d. Interstitial edema

Which nursing intervention for a patient who had an open repair of an abdominal aortic aneurysm 2 days previously is appropriate for the nurse to delegate to unlicensed assistive personnel (UAP)? a. Monitor the quality and presence of the pedal pulses. b. Teach the patient the signs of possible wound infection. c. Check the lower extremities for strength and movement. d. Help the patient to use a pillow to splint while coughing.

d. Help the patient to use a pillow to splint while coughing. Assisting a patient who has already been taught how to cough is part of routine postoperative care and within the education and scope of practice for UAP. Patient teaching and assessment of essential postoperative functions such as circulation and movement should be done by RNs.

A mental health nurse notices her client is always eating crushed ice from a cup. Which diagnostic study would be most appropriate for this client? a. CT scan of the brain b. Electroencephalogram c. Vitamin B12 lab study. d. Hemoglobin & hematocrit.

d. Hemoglobin & hematocrit. This answer is correct because pica causes iron deficiency anemia and vice versa. When a client continuously eats items with no nutritional value, it is because they are deficient in nutrients and/or they become deficient in nutrients. Nutrients such as iron. By testing hemoglobin & hematocrit, anemia can be diagnosed and a treatment plan can begin.

When assessing a newly admitted patient, the nurse notes pallor of the skin and nail beds. The nurse should ensure that which laboratory test has been ordered? a. Platelet count b. Neutrophil count c. White blood cell count d. Hemoglobin (Hgb) level

d. Hemoglobin (Hgb) level Pallor of the skin or nail beds is indicative of anemia, which would be indicated by a low Hgb level. Platelet counts indicate a persons clotting ability. A neutrophil is a type of white blood cell that helps to fight infection.

Which lab study will the nurse monitor for her client who is receiving IV erythropoietin? a. Red blood cell count b. White blood cell count c. Platelet count d. Hemoglobin and hematocrit

d. Hemoglobin and hematocrit This answer is correct because erythropoietin stimulates red blood cell production. Hemoglobin is a protein in red blood cells that carries oxygen. Thus, hemoglobin and hematocrit should be monitored to watch for anemia.

What is the initial antibiotic treatment for pneumonia based on? a. The severity of symptoms b. The presence of characteristic leukocytes c. Gram stains and cultures of sputum specimens d. History and physical examination and characteristic chest x-ray findings

d. History and physical examination and characteristic chest x-ray findings Prompt treatment of pneumonia with appropriate antibiotics is important in treating bacterial and mycoplasma pneumonia, and antibiotics are often administered on the basis of the history, physical examination, and a chest x-ray indicating a typical pattern characteristic of a particular organism without further testing. It is more significant if it is CAP or HAP than the severity of pneumonia symptoms. Blood and sputum cultures take 24 to 72 hours for results, and microorganisms often cannot be identified with either Gram stain or cultures.

It is time for a client's enoxaparin injection. The nurse reviews the client's CBC and notes the following values: WBC 5.9 mm, RBC 4.9 mm, hemoglobin 14.6 g/dL, hematocrit 43.5 g/dL and platelets 99,000. Which action by the nurse is most appropriate? a. Notify the health care provider (HCP). b. Assess for signs of bleeding. c. Obtain vital signs. d. Hold the medication.

d. Hold the medication. This answer is correct because the platelet count is low. Normal platelets are between 150,000 and 400,000. When platelets fall below 100,000, the medications that affect platelets should be held. These include heparin, aspirin, clopidogrel, and enoxaparin. The health care provider (HCP) should be notified and the nurse should obtain vital signs and assess for signs of bleeding.

Dietary teaching that includes eating dietary sources of potassium is indicated for the hypertensive patient taking which drug? a. Enalapril b. Labetalol c. Spironolactone d. Hydrochlorothiazide

d. Hydrochlorothiazide Hydrochlorothiazide is a thiazide diuretic that causes sodium and potassium loss through the kidneys. High-potassium foods should be included in the diet, or potassium supplements may be used to prevent hypokalemia. Enalapril and spironolactone may cause hyperkalemia by inhibiting the action of aldosterone, and potassium supplements should not be used by patients taking these drugs. As a combined α/β-blocker, labetalol does not affect potassium levels

Which information should the nurse include when teaching a patient with newly diagnosed hypertension? a. Increasing physical activity will control blood pressure (BP) for most patients. b. Most patients are able to control BP through dietary changes. c. Annual BP checks are needed to monitor treatment effectiveness. d. Hypertension is usually asymptomatic until target organ damage occurs.

d. Hypertension is usually asymptomatic until target organ damage occurs. Hypertension is usually asymptomatic until target organ damage has occurred. Lifestyle changes (e.g., physical activity, dietary changes) are used to help manage blood pressure, but drugs are needed for most patients. Home BP monitoring should be taught to the patient and findings checked by the health care provider frequently when starting treatment for hypertension and then every 3 months once stable.

The nurse establishes the nursing diagnosis of ineffective health maintenance related to lack of knowledge regarding long-term management of rheumatic fever when a 30-year-old recovering from rheumatic fever without carditis says which of the following? a. I will need prophylactic antibiotic therapy for 5 years. b. I will need to take aspirin or ibuprofen (Motrin) to relieve my joint pain. c. I will call the doctor if I develop excessive fatigue or difficulty breathing. d. I will be immune to further episodes of rheumatic fever after this infection.

d. I will be immune to further episodes of rheumatic fever after this infection. Patients with a history of rheumatic fever are more susceptible to a second episode. Patients with rheumatic fever without carditis require prophylaxis until age 20 and for a minimum of 5 years. The other patient statements are correct and would not support the nursing diagnosis of ineffective health maintenance.

The most common type of anemia, it is the result of nutritional deficits and will require supplements to treat. a. Folic Acid Deficiency b. Aplastic c. Sickle Cell d. Iron Deficiency

d. Iron Deficiency

Spironolactone (Aldactone), an aldosterone antagonist, is prescribed for a patient. Which statement by the patient indicates that the teaching about this medication has been effective? a. I will try to drink at least 8 glasses of water every day. b. I will use a salt substitute to decrease my sodium intake. c. I will increase my intake of potassium-containing foods. d. I will drink apple juice instead of orange juice for breakfast.

d. I will drink apple juice instead of orange juice for breakfast. Because spironolactone is a potassium-sparing diuretic, patients should be taught to choose low-potassium foods (e.g., apple juice) rather than foods that have higher levels of potassium (e.g., citrus fruits). Because the patient is using spironolactone as a diuretic, the nurse would not encourage the patient to increase fluid intake. Teach patients to avoid salt substitutes, which are high in potassium.

Which statement made by a patient with coronary artery disease after the nurse has completed teaching about therapeutic lifestyle changes (TLC) diet indicates that further teaching is needed? a. I will switch from whole milk to 1% milk. b. I like salmon and I will plan to eat it more often. c. I can have a glass of wine with dinner if I want one. d. I will miss being able to eat peanut butter sandwiches.

d. I will miss being able to eat peanut butter sandwiches. Although only 30% of the daily calories should come from fats, most of the fat in the TLC diet should come from monosaturated fats such as are found in nuts, olive oil, and canola oil. The patient can include peanut butter sandwiches as part of the TLC diet. The other patient comments indicate a good understanding of the TLC diet.

After providing a patient with discharge instructions on the management of a new permanent pacemaker, the nurse knows that teaching has been effective when the patient states a. I will avoid cooking with a microwave oven or being near one in use. b. It will be 1 month before I can take a bath or return to my usual activities. c. I will notify the airlines when I make a reservation that I have a pacemaker. d. I wont lift the arm on the pacemaker side up very high until I see the doctor.

d. I wont lift the arm on the pacemaker side up very high until I see the doctor. The patient is instructed to avoid lifting the arm on the pacemaker side above the shoulder to avoid displacing the pacemaker leads. The patient should notify airport security about the presence of a pacemaker before going through the metal detector, but there is no need to notify the airlines when making a reservation. Microwave oven use does not affect the pacemaker. The insertion procedure involves minor surgery that will have a short recovery period.

The nurse admits a patient who has a diagnosis of an acute asthma attack. Which statement indicates that the patient may need teaching regarding medication use? a. I have not had any acute asthma attacks during the last year. b. I became short of breath an hour before coming to the hospital. c. I've been taking Tylenol 650 mg every 6 hours for chest-wall pain. d. I've been using my albuterol inhaler more frequently over the last 4 days.

d. I've been using my albuterol inhaler more frequently over the last 4 days. The increased need for a rapid-acting bronchodilator should alert the patient that an acute attack may be imminent and that a change in therapy may be needed. The patient should be taught to contact a health care provider if this occurs. The other data do not indicate any need for additional teaching.

A patient with pneumonia has a fever of 101.4 F (38.6 C), a nonproductive cough, and an oxygen saturation of 88%. The patient complains of weakness, fatigue, and needs assistance to get out of bed. Which nursing diagnosis should the nurse assign as the highest priority ? a. Hyperthermia related to infectious illness b. Impaired transfer ability related to weakness c. Ineffective airway clearance related to thick secretions d. Impaired gas exchange related to respiratory congestion

d. Impaired gas exchange related to respiratory congestion All these nursing diagnoses are appropriate for the patient, but the patients oxygen saturation indicates that all body tissues are at risk for hypoxia unless the gas exchange is improved.

A patient experienced sudden cardiac death (SCD) and survived. Which preventive treatment should the nurse expect to be implemented? a. External pacemaker b. An electrophysiologic study (EPS) c. Medications to prevent dysrhythmias d. Implantable cardioverter-defibrillator (ICD)

d. Implantable cardioverter-defibrillator (ICD) An ICD is the most common approach to preventing recurrence of SCD. An external pacemaker may be used in the hospital but will not be used for the patient living daily life at home. An EPS may be done to determine if a recurrence is likely and determine the most effective medication treatment. Medications to prevent dysrhythmias are used but are not the best prevention of SCD.

Tobacco smoke causes defects in multiple areas of the respiratory system. What is a long-term effect of smoking? a. Bronchospasm and hoarseness b. Decreased mucus secretions and cough c. Increased function of alveolar macrophages d. Increased risk of infection and hyperplasia of mucous glands

d. Increased risk of infection and hyperplasia of mucous glands Increased risk of infection, hyperplasia of mucous glands, cancer, chronic cough, chronic bronchitis, and COPD are the long-term effects of smoking. Bronchospasm and hoarseness are acute effects of smoking.

Which medication is a long-acting β2 -adrenergic agonist and DPI that is used only for COPD? a. Roflumilast (Daliresp) b. Salmeterol (Serevent) c. Ipratropium (Atrovent HFA) d. Indacaterol (Arcapta Neohaler)

d. Indacaterol (Arcapta Neohaler) Indacaterol (Arcapta Neohaler) is a β2 -adrenergic agonist administered via DPI that is used only for COPD. Roflumilast (Daliresp) is an oral medication used for COPD. Salmeterol (Serevent) is a DPI, but it is also used in asthma with inhaled corticosteroids. Ipratropium (Atrovent HFA) is used for COPD, but it is delivered via metered-dose inhaler or nebulizer.

What does the nursing responsibility in the management of the patient with hypertensive urgency include? a. Monitoring hourly urine output for drug effectiveness b. Titrating IV drug dosages based on BP and HR measurements every 2 to 3 minutes c. Providing continuous electrocardiographic (ECG) monitoring to detect side effects of the drugs d. Instructing the patient to follow up with a health care provider within 24 hours after outpatient treatment

d. Instructing the patient to follow up with a health care provider within 24 hours after outpatient treatment Hypertensive urgencies are often treated with oral drugs on an outpatient basis, but it is important for the patient to be seen by a HCP within 24 hours to evaluate the effectiveness of the treatment. Hourly urine measurements, titration of IV drugs, and Electrocardiogram (ECG) monitoring are indicated for hypertensive emergencies.

Which medication shows improvement for hypertension and angina in black patients with HFrEF? a. Captopril b. Nitroglycerin c. Spironolactone (Aldactone) d. Isosorbide dinitrate and hydralazine (Bidil)

d. Isosorbide dinitrate and hydralazine (Bidil) Isosorbide dinitrate and hydralazine (Bidil) is recommended for use in black patients with HFrEF to treat hypertension and angina. Captopril is used for hypertension by all patients. Nitroglycerin is used with hydralazine for patients who cannot tolerate RAAS inhibitors (angiotensin-converting enzyme [ACE] inhibitors or angiotensin II receptor blocker [ARBs]) for HF management. Spironolactone (Aldactone) is used for hypertension.

In teaching a patient with hypertension about controlling the illness, the nurse recognizes that a. All patients with elevated BP need drug therapy. b. Obese persons must achieve a normal weight to lower BP. c. It is not necessary to limit salt in the diet if taking a diuretic. d. Lifestyle modifications are needed for all persons with elevated BP.

d. Lifestyle modifications are needed for all persons with elevated BP.

A patient who has non-Hodgkins lymphoma is receiving combination treatment with rituximab (Rituxan) and chemotherapy. Which patient assessment finding requires the most rapid action by the nurse? a. Anorexia b. Vomiting c. Oral ulcers d. Lip swelling

d. Lip swelling Lip swelling in angioedema may indicate a hypersensitivity reaction to the rituximab. The nurse should stop the infusion and further assess for anaphylaxis. The other findings may occur with chemotherapy, but are not immediately life threatening.

Which indirect thrombin inhibitor is only given subcutaneously and does not need routine coagulation tests? a. Warfarin (Coumadin) b. Unfractionated heparin c. Hirudin derivatives (bivalirudin [Angiomax]) d. Low-molecular-weight heparin (enoxaparin [Lovenox])

d. Low-molecular-weight heparin (enoxaparin [Lovenox]) Low-molecular-weight heparin (LMWH) (enoxaparin [Lovenox]) is only given subcutaneously and does not need routine coagulation testing. Unfractionated heparin is the only other indirect thrombin inhibitor option. It can be given subcutaneously or IV and therapeutic effects must be monitored with coagulation testing

Priority Decision: The nurse caring for a patient immediately following a transesophageal echocardiogram (TEE) should consider which action to be the highest priority? a. Monitor the ECG. b. Monitor pulse oximetry. c. Assess vital signs (BP, HR, RR, temperature). d. Maintain NPO status until gag reflex has returned

d. Maintain NPO status until gag reflex has returned All actions will be done but to perform a transesophageal echocardiogram (TEE). The throat must be numbed. Until sensation returns, as evidenced by the gag reflex, the patient is at risk of aspiration, so this action has the highest priority (priority related to airway—airway, breathing and circulation [ABCs]).

To help prevent embolization of a thrombus in a patient with acute VTE and severe edema and limb pain, what should the nurse teach the patient to do first? a. Dangle on the edge of the bed q2-3hr. b. Ambulate around the bed 3 to 4 times a day. c. Keep the affected leg elevated above the level of the heart. d. Maintain bed rest until edema is relieved and anticoagulation is established.

d. Maintain bed rest until edema is relieved and anticoagulation is established. With acute VTE, prevention of emboli formation, decreased edema and pain can be achieved initially by bed rest and limiting movement of the involved extremity. Ambulation will be the next priority. Dangling the legs promotes venous stasis and further clot formation. Elevating the affected limb will promote venous return, but it does not prevent embolization.

A 54-year-old patient who has no structural heart disease has an episode of syncope. A head-up tilt-test is performed to rule out cardioneurogenic syncope. The nurse explains to the patient that if cardioneurogenic syncope is the problem, the patient will experience what? a. No change in HR or BP b. Palpitations and dizziness c. Tachydysrhythmias and chest pain d. Marked bradycardia and hypotension

d. Marked bradycardia and hypotension One of the most common causes of syncope is cardioneurogenic syncope, or "vasovagal" syncope. In this type of syncope, there is accentuated adrenergic activity in the upright position, with intense activation of cardiopulmonary receptors resulting in marked bradycardia and hypotension, cerebral hypoperfusion, and syncope. Normally testing with the head-up tilt-test causes activation of the renin-angiotensinaldosterone system and compensation to increase CO and maintain BP when blood pools in the extremities.

If a patient is in status asthmaticus, which of the following interventions would the nurse expect? a. Bronchodilators b. Corticosteroids c. O2 via non-rebreather d. Mechanical ventilation

d. Mechanical ventilation

A nurse is assessing a client who has dehydration. Which of the following assessments is the priority? a. Skin turgor b. Urine output c. Weight d. Mental status

d. Mental status The greatest risk to this client is injury from declining mental status or a fall from worsened dehydration. Therefore, assessing the client's mental status is the priority.

While working in the outpatient clinic, the nurse notes that a patient has a history of intermittent claudication. Which statement by the patient would support this information? a. When I stand too long, my feet start to swell. b. I get short of breath when I climb a lot of stairs. c. My fingers hurt when I go outside in cold weather. d. My legs cramp whenever I walk more than a block.

d. My legs cramp whenever I walk more than a block. Cramping that is precipitated by a consistent level of exercise is descriptive of intermittent claudication. Finger pain associated with cold weather is typical of Raynauds phenomenon. Shortness of breath that occurs with exercise is not typical of intermittent claudication, which is reproducible. Swelling associated with prolonged standing is typical of venous disease.

Following successful treatment of Hodgkins lymphoma for a 55-year-old woman, which topic will the nurse include in patient teaching? a. Potential impact of chemotherapy treatment on fertility b. Application of soothing lotions to treat residual pruritus c. Use of maintenance chemotherapy to maintain remission d. Need for follow-up appointments to screen for malignancy

d. Need for follow-up appointments to screen for malignancy The chemotherapy used in treating Hodgkins lymphoma results in a high incidence of secondary malignancies; follow-up screening is needed. The fertility of a 55-year-old woman will not be impacted by chemotherapy. Maintenance chemotherapy is not used for Hodgkins lymphoma. Pruritus is a clinical manifestation of lymphoma, but should not be a concern after treatment.

When the nurse is monitoring a patient who is undergoing exercise (stress) testing on a treadmill, which assessment finding requires the most rapid action by the nurse? a. Patient complaint of feeling tired b. Pulse change from 87 to 101 beats/minute c. Blood pressure (BP) increase from 134/68 to 150/80 mm Hg d. Newly inverted T waves on the electrocardiogram

d. Newly inverted T waves on the electrocardiogram ECG changes associated with coronary ischemia (such as T-wave inversions and ST segment depression) indicate that the myocardium is not getting adequate oxygen delivery and that the exercise test should be terminated immediately. Increases in BP and heart rate (HR) are normal responses to aerobic exercise. Feeling tired is also normal as the intensity of exercise increases during the stress testing.

The nurse monitors the patient receiving treatment for ADHF with the knowledge that marked hypotension is most likely to occur with the IV administration of which medication? a. Milrinone b. Furosemide c. Nitroglycerin d. Nitroprusside

d. Nitroprusside Although all of these drugs may cause hypotension, nitroprusside is a potent dilator of both arteries and veins and may cause such marked hypotension that an inotropic agent (e.g., dobutamine) administration may be necessary to maintain the BP during its administration. Furosemide may cause hypotension because of diuretic-induced depletion of intravascular fluid volume. Milrinone has a positive inotropic effect in addition to peripheral vasodilation. Nitroglycerin is a vasodilator and can decrease BP but not as severely as nitroprusside. It primarily dilates veins and increases myocardial oxygen supply.

A patient's ABGs include a PaO2 of 88 mm Hg and a PaCO2 of 38 mm Hg, and mixed venous blood gases include a partial pressure of oxygen in venous blood (PvO2 ) of 40 mm Hg and partial pressure of carbon dioxide in venous blood (PvCO2 ) of 46 mm Hg. What do these findings indicate? a. Impaired cardiac output b. Unstable hemodynamics c. Inadequate delivery of oxygen to the tissues d. Normal capillary oxygen-carbon dioxide exchange

d. Normal capillary oxygen-carbon dioxide exchange Normal venous blood gas values reflect the normal uptake of oxygen from arterial blood and the release of carbon dioxide from cells into the blood, resulting in a much lower PaO2 and an increased PaCO2 . The pH is also decreased in mixed venous blood gases because of the higher partial pressure of carbon dioxide in venous blood (PvCO2 ). Normal mixed venous blood gases also have much lower partial pressure of oxygen in venous blood (PvO2 ) and venous oxygen saturation (SvO2 ) than ABGs. Mixed venous blood gases are used when patients are hemodynamically unstable to evaluate the amount of oxygen delivered to the tissue and the amount of oxygen consumed by the tissues.

A patient admitted with acute dyspnea is newly diagnosed with dilated cardiomyopathy. Which information will the nurse plan to teach the patient about managing this disorder? a. A heart transplant should be scheduled as soon as possible. b. Elevating the legs above the heart will help relieve dyspnea. c. Careful compliance with diet and medications will prevent heart failure. d. Notify the doctor about any symptoms of heart failure such as shortness of breath.

d. Notify the doctor about any symptoms of heart failure such as shortness of breath. The patient should be instructed to notify the health care provider about any worsening of heart failure symptoms. Because dilated cardiomyopathy does not respond well to therapy, even patients with good compliance with therapy may have recurrent episodes of heart failure. Elevation of the legs above the heart will worsen symptoms (although this approach is appropriate for a patient with hypertrophic cardiomyopathy). The patient with terminal or end-stage cardiomyopathy may consider heart transplantation.

While assessing a patient who was admitted with heart failure, the nurse notes that the patient has jugular venous distention (JVD) when lying flat in bed. Which action should the nurse take next? a. Document this finding in the patients record. b. Obtain vital signs, including oxygen saturation. c. Have the patient perform the Valsalva maneuver. d. Observe for JVD with the patient upright at 45 degrees.

d. Observe for JVD with the patient upright at 45 degrees. When the patient is lying flat, the jugular veins are at the level of the right atrium, so JVD is a common (but not a clinically significant) finding. Obtaining vital signs and oxygen saturation is not warranted at this point. JVD is an expected finding when a patient performs the Valsalva maneuver because right atrial pressure increases. JVD that persists when the patient is sitting at a 30- to 45-degree angle or greater is significant. The nurse will document the JVD in the medical record if it persists when the head is elevated.

What should the nurse teach the patient who has endocarditis about preventing recurrence of the infection? a. Start on antibiotic therapy when exposed to persons with infections. b. Take one aspirin a day to prevent vegetative lesions from forming around the valves. c. Always maintain continuous antibiotic therapy to prevent the development of any systemic infection. d. Obtain prophylactic antibiotic therapy before certain invasive medical or dental procedures (e.g., dental cleaning).

d. Obtain prophylactic antibiotic therapy before certain invasive medical or dental procedures (e.g., dental cleaning). Prophylactic antibiotic therapy should be started before invasive dental, medical, or surgical procedures to prevent recurrence of endocarditis. Symptoms of infection should be treated promptly, but antibiotics are not used for exposure to infection. Continuous antibiotic therapy is indicated only in patients with implanted devices or ongoing invasive procedure

The health care provider has prescribed bed rest with the feet elevated for a patient admitted to the hospital with venous thromboembolism. Which action by the nurse to elevate the patients feet is best? a. The patient is placed in the Trendelenburg position. b. Two pillows are positioned under the affected leg. c. The bed is elevated at the knee and pillows are placed under the feet. d. One pillow is placed under the thighs and two pillows are placed under the lower legs.

d. One pillow is placed under the thighs and two pillows are placed under the lower legs. The purpose of elevating the feet is to enhance venous flow from the feet to the right atrium, which is best accomplished by placing two pillows under the feet and one under the thighs. Placing the patient in the Trendelenburg position will lower the head below heart level, which is not indicated for this patient. Placing pillows under the calf or elevating the bed at the knee may cause blood stasis at the calf level.

A lymph node biopsy is most often performed to diagnose a. leukemia. b. cause of lymphedema. c. hemorrhagic tendencies. d. neoplastic cells in lymph nodes.

d. neoplastic cells in lymph nodes. Lymph node biopsy is usually done to determine whether malignant cells are present in lymph nodes and can be used to diagnose lymphomas as well as metastatic spread from any malignant tumor in the body. Leukemias may infiltrate lymph nodes, but biopsy of the nodes is more commonly used to detect any type of neoplastic cells.

A patient hospitalized with chronic obstructive pulmonary disease (COPD) is being discharged home on oxygen therapy. Which instruction should the nurse include in the discharge teaching? a. Storage of oxygen tanks will require adequate space in the home. b. Travel opportunities will be limited because of the use of oxygen. c. Oxygen flow should be increased if the patient has more dyspnea. d. Oxygen use can improve the patients prognosis and quality of life.

d. Oxygen use can improve the patients prognosis and quality of life. The use of home oxygen improves quality of life and prognosis. Because increased dyspnea may be a symptom of an acute process such as pneumonia, the patient should notify the physician rather than increasing the oxygen flow rate if dyspnea becomes worse. Oxygen can be supplied using liquid, storage tanks, or concentrators, depending on individual patient circumstances. Travel is possible using portable oxygen concentrators.

Which of the following descriptions matches the mechanism of "diffusion"? a. Uses a protein carrier molecule b. Force exerted by a fluid c. Flow of water from low-solute concentration to high-solute concentration d. Passive movement of molecules from a high concentration to lower concentration

d. Passive movement of molecules from a high concentration to lower concentration

The nurse is caring for a 70-year-old who uses hydrochlorothiazide (HydroDIURIL) and enalapril (Norvasc), but whose self-monitored blood pressure (BP) continues to be elevated. Which patient information may indicate a need for a change? a. Patient takes a daily multivitamin tablet. b. Patient checks BP daily just after getting up. c. Patient drinks wine three to four times a week. d. Patient uses ibuprofen (Motrin) daily to treat osteoarthritis.

d. Patient uses ibuprofen (Motrin) daily to treat osteoarthritis. Because use of nonsteroidal anti-inflammatory drugs (NSAIDs) can prevent adequate BP control, the patient may need to avoid the use of ibuprofen. A multivitamin tablet will help supply vitamin D, which may help lower BP. BP decreases while sleeping, so self-monitoring early in the morning will result in obtaining pressures that are at their lowest. The patients alcohol intake is not excessive.

The nurse is caring for a 70-yr-old patient who uses hydrochlorothiazide and enalapril (Norvasc) but whose self-monitored blood pressure (BP) continues to be elevated. Which patient information may indicate a need for a change? a. Patient takes a daily multivitamin tablet. b. Patient checks BP daily just after getting up. c. Patient drinks wine three to four times a week. d. Patient uses ibuprofen (Motrin) treat osteoarthritis.

d. Patient uses ibuprofen (Motrin) treat osteoarthritis. Because use of nonsteroidal antiinflammatory drugs (NSAIDs) can prevent adequate BP control, the patient may need to avoid the use of ibuprofen. A multivitamin tablet will help supply vitamin D, which may help lower BP. BP decreases while sleeping, so self-monitoring early in the morning will result in obtaining pressures that are at their lowest. The patient's alcohol intake is not excessive.

Which treatment is used first for the patient with a confirmed MI to open the blocked artery within 90 minutes of arrival to a health care facility? a. TMR b. Stent placement c. Coronary artery bypass graft (CABG) d. Percutaneous coronary intervention (PCI)

d. Percutaneous coronary intervention (PCI) Emergent percutaneous coronary intervention (PCI) is the first treatment for patients with a confirmed MI within 90 minutes of arriving at the facility with an interventional cardiac catheterization laboratory. TMR, stent placement, and CABG are usually done to facilitate circulation in nonemergency situations.

A young adult patient with cystic fibrosis (CF) is admitted to the hospital with increased dyspnea. Which intervention should the nurse include in the plan of care? a. Schedule a sweat chloride test. b. Arrange for a hospice nurse visit. c. Place the patient on a low-sodium diet. d. Perform chest physiotherapy every 4 hours.

d. Perform chest physiotherapy every 4 hours. Routine scheduling of airway clearance techniques is an essential intervention for patients with CF. A sweat chloride test is used to diagnose CF, but it does not provide any information about the effectiveness of therapy. There is no indication that the patient is terminally ill. Patients with CF lose excessive sodium in their sweat and require high amounts of dietary sodium.

A patient is admitted to the hospital with a suspected acute pericarditis. What is the best method for the nurse to use in assessing for the presence of a pericardial friction rub? a. Timing the sounds with the respiratory pattern b. Place the bell of the stethoscope at the apical area of the heart c. Use the diaphragm of the stethoscope to auscultate for a high-pitched continuous rumbling sound d. Place the stethoscope at the lower left sternal border, patient leaning forward and holding breath

d. Place the stethoscope at the lower left sternal border, patient leaning forward and holding breath The stethoscope diaphragm at the lower left sternal border with the patient leaning forward is the best method to use to hear the high-pitched, grating sound of a pericardial friction rub. The sound does not radiate widely and occurs with the heartbeat. To differentiate a pericardial friction rub from a pleural friction rub, have the patient hold their breath. The rub will still be heard if it is cardiac in nature

For the client who is taking enoxaparin, which lab values are most important to monitor? a. PTT b. PT c. INR d. Platelets

d. Platelets This answer is correct because enoxaparin is a low molecular weight heparin that inactivates clotting. It is used post procedures or when clients are on bedrest to help prevent thrombosis or emboli. Monitoring the platelets while the client is taking enoxaparin will ensure the platelets do not become too low and put the client at risk for hemorrhage.

The patient is admitted with pneumonia, and the nurse hears a grating sound when she assesses the patient. How should the nurse document this sound? a. Stridor b. Bronchophony c. Course crackles d. Pleural friction rub

d. Pleural friction rub Pleural friction rub occurs with pneumonia and is a grating or creaking sound. Stridor is a continuous musical or crowing sound and unrelated to pneumonia. Bronchophony occurs with pneumonia but is a spoken or whispered word that is more distinct than normal on auscultation. Course crackles sound like blowing through a straw under water and occur in pneumonia when there is severe congestion. See Table 25.8 for more thorough descriptions of these sounds and their possible etiologies and significance.

What is a possible cause for auscultation abnormal finding "pleural friction rub"? a. Bronchoconstriction b. Partial obstruction of trachea or larynx c. Chronic hypoxemia d. Pleurisy

d. Pleurisy

Priority Decision: A patient with acute pericarditis has pain from pericardial inflammation. What is the best nursing intervention for the patient? a. Administer opioids as prescribed on a round-the-clock schedule. b. Promote progressive relaxation exercises with the use of deep, slow breathing. c. Position the patient on the right side with the head of the bed elevated 15 degrees. d. Position the patient in Fowler's position with a padded table for the patient to lean on.

d. Position the patient in Fowler's position with a padded table for the patient to lean on. Relief from pericardial pain is often obtained by sitting up and leaning forward. Pain is increased by lying flat. Antiinflammatory medications may also be used to help control pain, but opioids are not usually indicated. The pain has a sharp, pleuritic quality that changes with respiration, and patients take shallow breaths.

Priority Decision: During an acute exacerbation of mild COPD, the patient is severely short of breath. The nurse identifies a nursing diagnosis of impaired breathing, etiology: alveolar hypoventilation and anxiety. What is the best nursing action? a. Prepare and administer routine bronchodilator medications. b. Perform chest physiotherapy to promote removal of secretions. c. Administer oxygen at 5 L/min until the shortness of breath is relieved. d. Position the patient upright with the elbows resting on the over-the-bed table.

d. Position the patient upright with the elbows resting on the over-the-bed table. The tripod position with an elevated backrest and supported upper extremities to fix the shoulder girdle maximizes respiratory excursion and an effective breathing pattern. Staying with the patient and encouraging pursed lip breathing also helps. Rescue short-acting, not routine bronchodilators, will be ordered but can also increase nervousness and anxiety. Postural drainage is not tolerated by a patient in acute respiratory distress, and oxygen is titrated to an effective rate based on ABGs because of the possibility of carbon dioxide narcosis.

What is the primary principle involved in the various airway clearance devices used for mobilizing secretions? a. Vibration b. Inhalation therapy c. Chest physiotherapy d. Positive expiratory pressure

d. Positive expiratory pressure Positive expiratory pressure (PEP) is the principle behind the airway clearance devices that mobilize secretions and benefit patients. Vibration, a form of chest physiotherapy, and inhalation therapy are therapies to assist patients with excessive secretions or to increase bronchodilation, but they are not principles of airway clearance device function

A patient with a recent history of a dry cough has had a chest x-ray that revealed the presence of nodules. In an effort to determine whether the nodules are malignant or benign, what is the primary care provider likely to order? a. Thoracentesis b. Pulmonary angiogram c. CT scan of the patient's chest d. Positron emission tomography (PET)

d. Positron emission tomography (PET) PET is used to distinguish benign and malignant pulmonary nodules. Because malignant lung cells have an increased uptake of glucose, the PET scan (which uses an IV radioactive glucose preparation) can demonstrate increased uptake of glucose in malignant lung cells. This differentiation cannot be made using CT, a pulmonary angiogram, or thoracentesis.

A nurse is caring for a client who requires continuous cardiac monitoring. The nurse identifies a prolonged PR interval and a widened QRS complex. Which of the following laboratory values supports this finding? a. Sodium 152 mEq/L b. Chloride 102 mEq/L c. Magnesium 1.8 mEq/L d. Potassium 6.1 mEq/L

d. Potassium 6.1 mEq/L Hyperkalemia can cause a prolonged PR interval; a wide QRS complex; flat or absent P waves; and tall, peaked T waves.

A patient who was admitted with a myocardial infarction experiences a 45-second episode of ventricular tachycardia, then converts to sinus rhythm with a heart rate of 98 beats/minute. Which of the following actions should the nurse take next? a. Immediately notify the health care provider. b. Document the rhythm and continue to monitor the patient. c. Perform synchronized cardioversion per agency dysrhythmia protocol. d. Prepare to give IV amiodarone (Cordarone) per agency dysrhythmia protocol.

d. Prepare to give IV amiodarone (Cordarone) per agency dysrhythmia protocol. The burst of sustained ventricular tachycardia indicates that the patient has significant ventricular irritability, and antidysrhythmic medication administration is needed to prevent further episodes. The nurse should notify the health care provider after the medication is started. Defibrillation is not indicated given that the patient is currently in a sinus rhythm. Documentation and continued monitoring are not adequate responses to this situation.

In addition to smoking cessation, what treatment is included for COPD to slow the progression of the disease? a. Use of bronchodilator drugs b. Use of inhaled corticosteroids c. Lung volume reduction surgery d. Prevention of respiratory tract infections

d. Prevention of respiratory tract infections Smoking cessation is one of the most important factors in preventing further damage to the lungs in COPD, but prevention of infections that further increase lung damage is also important. The patient is very susceptible to infections, and these infections make the disease worse, creating a vicious cycle. Bronchodilators, inhaled corticosteroids, and lung volume-reduction surgery help control symptoms, but these are symptomatic measures.

The nurse is educating a client with COPD about pursed-lip breathing and the client asks the nurse about the purpose of this type of breathing. The nurse responds, knowing that the primary purpose of pursed-lip breathing is to? a. Promote oxygen intake b. Strengthen the diaphragm c. Strengthen the intercostal muscles d. Promote carbon dioxide elimination

d. Promote carbon dioxide elimination This will be asked a lot in the hospital setting. If they don't ask, it is important to tell them why it works. Pursed lip breathing facilitates maximal expiration for clients with obstructive lung disease. This type of breathing allows better expiration by increasing airway pressure that keeps air passage open during exhalation. IF the patient, puffs his cheeks out, why would this not work? Demonstrate: that the change is pressure is happening in the cheeks or mouth rather than inside the lungs. The cheeks are representing what we want to happen in the lungs.

Priority Decision: In planning care for the patient with bronchiectasis, which nursing intervention is the priority? a. Relieve or reduce pain b. Prevent paroxysmal coughing c. Prevent spread of the disease to others d. Promote drainage and removal of mucus

d. Promote drainage and removal of mucus Mucus production is increased in bronchiectasis and collects in the dilated, pouched bronchi. A major goal of treatment is to promote drainage and removal of the mucus, primarily through ACT, including deep breathing, coughing, and especially postural drainage. Pleuritic chest pain and prevention of coughing will occur with the removal of mucus. The disease is not contagious.

Priority Decision: What is the most important role of the nurse in preventing rheumatic fever? a. Teach patients with infective endocarditis to adhere to antibiotic prophylaxis. b. Identify patients with valvular heart disease who are at risk for rheumatic fever. c. Encourage the use of antibiotics for treatment of all infections involving a sore throat. d. Promote the early diagnosis and immediate treatment of group A streptococcal pharyngitis

d. Promote the early diagnosis and immediate treatment of group A streptococcal pharyngitis Adequate treatment of group A streptococcal pharyngitis can prevent initial attacks of rheumatic fever and the development of rheumatic heart disease. Because streptococcal infection accounts for only about 20% of acute pharyngitis, cultures should be done to identify the organism and direct antibiotic therapy. Viral infections should not be treated with antibiotics. Prophylactic therapy is indicated in those who have valvular heart disease or have had rheumatic heart disease

The health care provider orders a pulmonary angiogram for a patient admitted with dyspnea and hemoptysis. For which problem is this test most commonly used as a diagnostic measure? a. TB b. Cancer of the lung c. Airway obstruction d. Pulmonary embolism

d. Pulmonary embolism A pulmonary angiogram outlines the pulmonary vasculature and is useful to diagnose obstructions or pathologic conditions of the pulmonary vessels, such as a pulmonary embolus. The tissue changes of TB and cancer of the lung may be diagnosed by chest x-ray or CT scan, MRI, or positron emission tomography (PET) scans. Airway obstruction is most often diagnosed with pulmonary function testing.

When analyzing the rhythm of a patients electrocardiogram (ECG), the nurse will need to investigate further upon finding a(n) a. isoelectric ST segment. b. P-R interval of 0.18 second. c. Q-T interval of 0.38 second. d. QRS interval of 0.14 second.

d. QRS interval of 0.14 second. Because the normal QRS interval is 0.04 to 0.10 seconds, the patients QRS interval of 0.14 seconds indicates that the conduction through the ventricular conduction system is prolonged. The P-R interval and Q-T interval are within normal range, and ST segment should be isoelectric (flat).

In an adult patient with bronchiectasis, what is a health history likely to reveal? a. Chest trauma b. Childhood asthma c. Smoking or oral tobacco use d. Recurrent lower respiratory tract infections

d. Recurrent lower respiratory tract infections In adults, most forms of bronchiectasis are associated with bacterial infections that damage the bronchial walls. In children, CF is the prominent cause of bronchiectasis. The incidence of bronchiectasis has decreased with the use of measles and pertussis vaccines and better treatment of lower respiratory tract infections.

Using the illustrated technique, the nurse is assessing for which finding in a patient with chronic obstructive pulmonary disease (COPD)? a. Hyperresonance b. Tripod positioning c. Accessory muscle use d. Reduced chest expansion

d. Reduced chest expansion The technique for palpation for chest expansion is shown in the illustrated technique. Reduced chest movement would be noted on palpation of a patients chest with COPD. Hyperresonance would be assessed through percussion. Accessory muscle use and tripod positioning would be assessed by inspection.

The nurse is caring for a patient who is receiving IV furosemide (Lasix) and morphine for the treatment of acute decompensated heart failure (ADHF) with severe orthopnea. Which clinical finding is the best indicator that the treatment has been effective? a. Weight loss of 2 pounds in 24 hours b. Hourly urine output greater than 60 mL c. Reduction in patient complaints of chest pain d. Reduced dyspnea with the head of bed at 30 degrees

d. Reduced dyspnea with the head of bed at 30 degrees Because the patients major clinical manifestation of ADHF is orthopnea (caused by the presence of fluid in the alveoli), the best indicator that the medications are effective is a decrease in dyspnea with the head of the bed at 30 degrees. The other assessment data also may indicate that diuresis or improvement in cardiac output has occurred, but are not as specific to evaluating this patients response.

A patient had a nonST-segment-elevation myocardial infarction (NSTEMI) 3 days ago. Which nursing intervention included in the plan of care is most appropriate for the registered nurse (RN) to delegate to an experienced licensed practical/vocational nurse (LPN/LVN)? a. Evaluation of the patients response to walking in the hallway b. Completion of the referral form for a home health nurse follow-up c. Education of the patient about the pathophysiology of heart disease d. Reinforcement of teaching about the purpose of prescribed medications

d. Reinforcement of teaching about the purpose of prescribed medications LPN/LVN education and scope of practice include reinforcing education that has previously been done by the RN. Evaluating the patient response to exercise after a NSTEMI requires more education and should be done by the RN. Teaching and discharge planning/ documentation are higher level skills that require RN education and scope of practice.

Priority Decision: On assessment of a central venous access device (CVAD) site, the nurse notes that the transparent dressing is loose along 2 sides. What should the nurse do immediately? a. Wait and change the dressing when it is due. b. Tape the 2 loose sides down and document. c. Apply a gauze dressing over the transparent dressing and tape securely. d. Remove the dressing and apply a new transparent dressing using sterile technique.

d. Remove the dressing and apply a new transparent dressing using sterile technique. The greatest risk with central venous access device (CVAD) is systemic infection. Dressings that are loose should be changed at once to reduce this risk.

What compensatory mechanism involved in both chronic heart failure and acute decompensated heart failure leads to fluid retention and edema? a. Ventricular dilation b. Ventricular hypertrophy c. Increased systemic blood pressure d. Renin-angiotensin-aldosterone activation

d. Renin-angiotensin-aldosterone activation

A patient who was involved in a motor vehicle crash has had a tracheostomy placed to allow for continued mechanical ventilation. How should the nurse interpret the following arterial blood gas results: pH 7.48, PaO2 85 mm Hg, PaCO2 32 mm Hg, and HCO3 25 mEq/L? a. Metabolic acidosis b. Metabolic alkalosis c. Respiratory acidosis d. Respiratory alkalosis

d. Respiratory alkalosis The pH indicates that the patient has alkalosis and the low PaCO2indicates a respiratory cause. The other responses are incorrect based on the pH and the normal HCO3. This is uncompensated.

Priority nursing management for a patient with myocarditis includes interventions related to a. meticulous skin care. b. antibiotic prophylaxis. c. tight glycemic control. d. oxygenation and ventilation.

d. oxygenation and ventilation.

Which statement by a patient indicates good understanding of the nurses teaching about prevention of sickle cell crisis? a. Home oxygen therapy is frequently used to decrease sickling. b. There are no effective medications that can help prevent sickling. c. Routine continuous dosage narcotics are prescribed to prevent a crisis. d. Risk for a crisis is decreased by having an annual influenza vaccination.

d. Risk for a crisis is decreased by having an annual influenza vaccination. Because infection is the most common cause of a sickle cell crisis, influenza, Haemophilus influenzae, pneumococcal pneumonia, and hepatitis immunizations should be administered. Although continuous dose opioids and oxygen may be administered during a crisis, patients do not receive these therapies to prevent crisis. Hydroxyurea (Hydrea) is a medication used to decrease the number of sickle cell crises.

The patient is brought to the emergency department with acute coronary syndrome (ACS). What changes should the nurse expect to see on the ECG if only myocardial injury has occurred? a. Absent P wave b. A wide QT interval c. Tall, peaked T wave d. ST-segment elevation

d. ST-segment elevation ST segment elevation is seen in myocardial injury. An absent or buried P wave can occur with PVCs, ventricular tachycardia, or ventricular fibrillation. A wide pathologic QT interval affects repolarization and is caused by drugs and electrolyte imbalances. Tall, peaked T waves may be seen with electrolyte imbalance.

During discharge teaching of a patient with newly diagnosed sickle cell disease, what should the nurse teach the patient to do? a. Limit fluid intake. b. Avoid humid weather. c. Eliminate exercise from the lifestyle. d. Seek early medical intervention for upper respiratory infections.

d. Seek early medical intervention for upper respiratory infections. The patient with sickle cell disease is particularly prone to upper respiratory infection, and infection can precipitate a sickle cell crisis. Patients should seek medical attention quickly to counteract upper respiratory infections because pneumonia is the most common infection in patients with sickle cell disease. Fluids should be increased to decrease blood viscosity, which may precipitate a crisis. Moderate activity is permitted. Dehydration in hot weather may precipitate a sickling episode, but humid weather alone will not do so.

Risk Factor for or Response to Respiratory Problem: Inability to maintain lifestyle, altered self-esteem. Which Functional Health Pattern does it fall under? a. Activity-exercise b. Sleep-rest c. Cognitive-perceptual d. Self-perception- self-concept

d. Self-perception- self-concept

Which patient information is most important for the nurse to monitor when evaluating the effectiveness of deferoxamine (Desferal) for a patient with hemochromatosis? a. Skin color b. Hematocrit c. Liver function d. Serum iron level

d. Serum iron level Because iron chelating agents are used to lower serum iron levels, the most useful information will be the patients iron level. The other parameters will also be monitored, but are not the most important to monitor when determining the effectiveness of deferoxamine.

The nurse prepares a patient with a left-sided pleural effusion for a thoracentesis. How should the nurse position the patient? a. Supine with the head of the bed elevated 30 degrees b. In a high-Fowlers position with the left arm extended c. On the right side with the left arm extended above the head d. Sitting upright with the arms supported on an over bed table

d. Sitting upright with the arms supported on an over bed table The upright position with the arms supported increases lung expansion, allows fluid to collect at the lung bases, and expands the intercostal space so that access to the pleural space is easier. The other positions would increase the work of breathing for the patient and make it more difficult for the health care provider performing the thoracentesis.

Which values indicate a need for the use of continuous oxygen therapy? a. SpO2 of 92%; PaO2 of 65 mm Hg b. SpO2 of 95%; PaO2 of 70 mm Hg c. SpO2 of 90%; PaO2 of 60 mm Hg d. SpO2 of 88%; PaO2 of 55 mm Hg

d. SpO2 of 88%; PaO2 of 55 mm Hg An SpO2 of 88% and a PaO2 of 55 mm Hg indicate inadequate oxygenation and are the criteria for continuous oxygen therapy (see Table 25.10). These values may be adequate for patients with chronic hypoxemia if no cardiac problems occur but will affect the patients' activity tolerance.

Priority Decision: After the health care provider sees a patient hospitalized with a stroke who developed a fever and adventitious lung sounds, the following orders are written. Which order should the nurse implement first? a. Anterior/posterior and lateral chest x-rays b. Start IV levofloxacin 500 mg every 24 hr now c. Complete blood count (CBC) with differential d. Sputum specimen for Gram stain and culture and sensitivity

d. Sputum specimen for Gram stain and culture and sensitivity A sputum specimen for Gram stain and culture should be obtained before starting antibiotic therapy and while waiting for the antibiotic to be delivered from the pharmacy in a hospitalized patient with suspected pneumonia. Then antibiotics should be started without delay. If the sputum specimen cannot be obtained rapidly, the chest x-ray will be done to assess the typical pattern characteristic of the infecting organism. Blood cell tests will not be altered significantly by delaying the tests until after the first dose of antibiotics.

Which is the priority nursing action for a patient who develops a high fever and tachycardia during administration of a unit of packed red blood cells? a. Administer 650 mg acetaminophen PO. b. Notify the provider. c. Assess the remainder of the patient's vital signs. d. Stop the infusion

d. Stop the infusion

About ten minutes after the nurse begins an infusion of packed RBCs, the patient complains of chills, chest and back pain, and nausea. His face is flushed, and he's anxious. Which is the priority nursing action? a. Administering antihistamines STAT for an allergic reaction. b. Notifying the physician of a possible transfusion reaction. c. Obtaining a urine and serum specimen to send to the lab immediately. d. Stopping the transfusion and maintaining a patent IV catheter.

d. Stopping the transfusion and maintaining a patent IV catheter. The patient is experiencing a transfusion reaction. The immediate nursing action is to stop the transfusion and maintain a patent IV line. The other options may be indicated but aren't the priority in this case.

Which of these electrolyte imbalances is the cause of "prolonged immobilization"? (select all that apply) a. hypernatremia b. hyponatremia c. hyperkalemia d. hypokalemia e. hypercalcemia f. hypocalcemia g. hyperphosphatemia h. hypophosphatemia i. hypermagnesemia j. hypomagnesemia

e. hypercalcemia

What is a major method of preventing infection in the patient with neutropenia? a. Prophylactic antibiotics b. A diet that eliminates fresh fruits and vegetables c. High-efficiency particulate air (HEPA) filtration d. Strict hand washing by all persons in contact with the patient

d. Strict hand washing by all persons in contact with the patient Despite its seeming simplicity, hand washing before, during, and after care of the patient with neutropenia is the major method to prevent transmission of harmful pathogens to the patient. IV antibiotics are administered when febrile episodes occur. Some oral antibiotics may be used prophylactically in some neutropenic patients. High-efficiency particulate air (HEPA) filtration will be used for the neutropenic patient at home to reduce the number of aerosolized pathogens.

Which treatment should the nurse anticipate for an otherwise healthy person with an initial VTE? a. IV argatroban as an inpatient b. IV unfractionated heparin as an inpatient c. Subcutaneous unfractionated heparin as an outpatient d. Subcutaneous low-molecular-weight heparin as an outpatient

d. Subcutaneous low-molecular-weight heparin as an outpatient

What is included in the correct technique for BP measurements? a. Always take the BP in both arms. b. Position the patient supine for all readings. c. Place the cuff loosely around the upper arm. d. Take readings at least 2 times at least 1 minute apart.

d. Take readings at least 2 times at least 1 minute apart. Correct technique in measuring BP includes taking and averaging 2 or more readings at least 1 minute apart. Initially BP measurements should be taken in both arms to detect any differences. If there is a difference, the arm with the higher reading should be used for all subsequent BP readings. The patient may be supine or sitting. The important points are that the arm being used is at the heart level and the cuff needs to fit snugly

A patient with chronic obstructive pulmonary disease (COPD) has a nursing diagnosis of impaired breathing pattern related to anxiety. Which nursing action is most appropriate to include in the plan of care? a. Titrate oxygen to keep saturation at least 90%. b. Discuss a high-protein, high-calorie diet with the patient. c. Suggest the use of over-the-counter sedative medications. d. Teach the patient how to effectively use pursed lip breathing.

d. Teach the patient how to effectively use pursed lip breathing. Pursed lip breathing techniques assist in prolonging the expiratory phase of respiration and decrease air trapping. There is no indication that the patient requires oxygen therapy or an improved diet. Sedative medications should be avoided because they decrease respiratory drive.

A nurse has been caring for a patient with tuberculosis (TB) and has a TB skin test performed. When is the nurse considered infected? a. There is no redness or induration at the injection site. b. There is an induration of only 5 mm at the injection site. c. A negative skin test is followed by a negative chest x-ray. d. Testing causes a 10-mm red, indurated area at the injection site.

d. Testing causes a 10-mm red, indurated area at the injection site. A 10-mm red indurated injection site could be a positive result for a nurse as an employee in a high-risk setting. Because antibody production in response to infection with the tuberculosis (TB) bacillus may not be sufficient to produce a reaction to TB skin testing immediately after infection, 2-step testing is recommended for individuals likely to be tested often, such as health care professionals. An initial negative skin test should be repeated in 1 to 3 weeks and if the second test is negative, the individual can be considered uninfected. All other answers indicate a negative response to skin testing.

The patient reports tenderness when she touches her leg over a vein. The nurse assesses warmth and a palpable cord in the area. The nurse knows the patient needs treatment to prevent which sequela? a Pulmonary embolism b Pulmonary hypertension c Post-thrombotic syndrome d Venous thromboembolism

d. The clinical manifestations are characteristic of a superficial vein thrombosis. If untreated, the clot may extend to deeper veins, and venous thromboembolism may occur. Pulmonary embolism, pulmonary hypertension, and postthrombotic syndrome are the sequelae of venous thromboembolism.

When reviewing the 12-lead electrocardiograph (ECG) for a healthy 79-year-old patient who is having an annual physical examination, what will be of most concern to the nurse? a. The PR interval is 0.21 seconds. b. The QRS duration is 0.13 seconds. c. There is a right bundle-branch block. d. The heart rate (HR) is 42 beats/minute.

d. The heart rate (HR) is 42 beats/minute. The resting HR does not change with aging, so the decrease in HR requires further investigation. Bundle-branch block and slight increases in PR interval or QRS duration are common in older individuals because of increases in conduction time through the AV node, bundle of His, and bundle branches

A PPD test is positive if an HIV patient has an induration of: a. The redness at the site is 15 mm or more. b. The induration is 1 mm c. There is no induration d. The induration is 5 mm or >

d. The induration is 5 mm or >

A patient was admitted for possible ruptured aortic aneurysm. No back pain was reported. Ten minutes later, the nurse notes sinus tachycardia 138 beats/min, blood pressure is palpable at 65 mm Hg, increasing waist circumference, and no urine output. How should the nurse interpret the findings? a Tamponade will soon occur. b The renal arteries are involved. c Perfusion to the legs is impaired. d Bleeding into the abdomen is likely.

d. The lack of back pain indicates the patient is most likely exsanguinating into the abdominal space, and the bleeding is likely to continue without surgical repair. A blockade of the blood flow will not occur in the abdominal space as it would in the retroperitoneal space, where surrounding anatomic structures may control the bleeding. The lack of urine output does not indicate renal artery involvement but that the bleeding is occurring above the renal arteries, which decreases the blood flow to the kidneys. There are no assessment data indicating decreased perfusion to the legs.

The nurse completes an admission assessment on a patient with asthma. Which information given by patient is most indicative of a need for a change in therapy? a. The patient uses albuterol (Proventil) before any aerobic exercise. b. The patient says that the asthma symptoms are worse every spring. c. The patients heart rate increases after using the albuterol (Proventil) inhaler. d. The patients only medications are albuterol (Proventil) and salmeterol (Serevent).

d. The patients only medications are albuterol (Proventil) and salmeterol (Serevent). Long-acting b2-agonists should be used only in patients who also are using an inhaled corticosteroid for long-term control. Salmeterol should not be used as the first-line therapy for long-term control. Using a bronchodilator before exercise is appropriate. The other information given by the patient requires further assessment by the nurse, but is not unusual for a patient with asthma.

A female patient has a total cholesterol level of 232 mg/dL (6.0 mmol/L) and a high-density lipoprotein (HDL) of 65 mg/dL (1.68 mmol/L). A male patient has a total cholesterol level of 200 mg/dL (5.172 mmol/L) and an HDL of 32 mg/dL (0.83 mmol/L). Based on these findings, which patient has the highest cardiac risk? a. The man, because his HDL is lower b. The woman, because her HDL is higher c. The woman, because her cholesterol is higher d. The man, because his cholesterol-to-HDL ratio is higher

d. The man, because his cholesterol-to-HDL ratio is higher A risk assessment for CAD is determined by comparing the total cholesterol to high-density lipoprotein (HDL), and a ratio can be calculated by dividing the total cholesterol level by the HDL level. The ratio provides more information than either value alone, and an increased ratio indicates an increased risk. The female patient has a ratio of 3.57, which is average risk, compared with the male patient's ratio of 6.25, which is increased risk.

The nurse is preparing to administer a scheduled dose of enoxaparin (Lovenox) 30 mg subcutaneously. What should the nurse do to administer this medication correctly? a Remove the air bubble in the prefilled syringe. b Aspirate before injection to prevent IV administration. c Rub the injection site after administration to enhance absorption. d Pinch the skin between the thumb and forefinger before inserting the needle.

d. The nurse should gather together or "bunch up" the skin between the thumb and the forefinger before inserting the needle into the subcutaneous tissue. The nurse should not remove the air bubble in the prefilled syringe, aspirate, nor rub the site after injection.

The nurse analyzes the results of a patients arterial blood gases (ABGs). Which finding would require immediate action? a. The bicarbonate level (HCO3) is 31 mEq/L. b. The arterial oxygen saturation (SaO2) is 92%. c. The partial pressure of CO2 in arterial blood (PaCO2) is 31 mm Hg. d. The partial pressure of oxygen in arterial blood (PaO2) is 59 mm Hg.

d. The partial pressure of oxygen in arterial blood (PaO2) is 59 mm Hg. All the values are abnormal, but the low PaO2 indicates that the patient is at the point on the oxyhemoglobin dissociation curve where a small change in the PaO2 will cause a large drop in the O2 saturation and a decrease in tissue oxygenation. The nurse should intervene immediately to improve the patients oxygenation.

After the nurse teaches the patient with stage 1 hypertension about diet modifications that should be implemented, which diet choice indicates that the teaching has been effective? a. The patient avoids eating nuts or nut butters. b. The patient restricts intake of chicken and fish. c. The patient has two cups of coffee in the morning. d. The patient has a glass of low-fat milk with each meal.

d. The patient has a glass of low-fat milk with each meal. For the prevention of hypertension, the Dietary Approaches to Stop Hypertension (DASH) recommendations include increasing the intake of calcium-rich foods. Caffeine restriction and decreased protein intake are not included in the recommendations. Nuts are high in beneficial nutrients and 4 to 5 servings weekly are recommended in the DASH diet.

The nurse teaches a patient how to administer formoterol (Perforomist) through a nebulizer. Which action by the patient indicates good understanding of the teaching? a. The patient attaches a spacer before using the inhaler. b. The patient coughs vigorously after using the inhaler. c. The patient activates the inhaler at the onset of expiration. d. The patient removes the facial mask when misting has ceased.

d. The patient removes the facial mask when misting has ceased. A nebulizer is used to administer aerosolized medication. A mist is seen when the medication is aerosolized, and when all of the medication has been used, the misting stops. The other options refer to inhaler use. Coughing vigorously after inhaling and activating the inhaler at the onset of expiration are both incorrect techniques when using an inhaler.

Which information given by a patient admitted with chronic stable angina will help the nurse confirm this diagnosis? a. The patient states that the pain wakes me up at night. b. The patient rates the pain at a level 3 to 5 (0 to 10 scale). c. The patient states that the pain has increased in frequency over the last week. d. The patient states that the pain goes away with one sublingual nitroglycerin tablet.

d. The patient states that the pain goes away with one sublingual nitroglycerin tablet. Chronic stable angina is typically relieved by rest or nitroglycerin administration. The level of pain is not a consistent indicator of the type of angina. Pain occurring at rest or with increased frequency is typical of unstable angina.

The nurse teaches a patient who has asthma about peak flow meter use. Which action by the patient indicates that teaching was successful? a. The patient inhales rapidly through the peak flow meter mouthpiece. b. The patient takes montelukast (Singulair) for peak flows in the red zone. c. The patient calls the health care provider when the peak flow is in the green zone. d. The patient uses albuterol (Proventil) metered dose inhaler (MDI) for peak flows in the yellow zone.

d. The patient uses albuterol (Proventil) metered dose inhaler (MDI) for peak flows in the yellow zone. Readings in the yellow zone indicate a decrease in peak flow. The patient should use short-acting b2-adrenergic (SABA) medications. Readings in the green zone indicate good asthma control. The patient should exhale quickly and forcefully through the peak flow meter mouthpiece to obtain the readings. Readings in the red zone do not indicate good peak flow, and the patient should take a fast-acting bronchodilator and call the health care provider for further instructions. Singulair is not indicated for acute attacks but rather is used for maintenance therapy.

The nurse is obtaining a health history from a 24-year-old patient with hypertrophic cardiomyopathy (HC). Which information obtained by the nurse is most important? a. The patient has a history of a recent upper respiratory infection. b. The patient has a family history of coronary artery disease (CAD). c. The patient reports using cocaine a couple of times as a teenager. d. The patients 29-year-old brother died from a sudden cardiac arrest.

d. The patients 29-year-old brother died from a sudden cardiac arrest. About half of all cases of HC have a genetic basis, and it is the most common cause of sudden cardiac death in otherwise healthy young people. The information about the patients brother will be helpful in planning care (such as an automatic implantable cardioverter-defibrillator [AICD]) for the patient and in counseling other family members. The patient should be counseled against the use of stimulant drugs, but the limited past history indicates that the patient is not at current risk for cocaine use. Viral infections and CAD are risk factors for dilated cardiomyopathy, but not for HC.

A patient is receiving a 3% saline continuous IV infusion for hyponatremia. Which assessment data will require the most rapid response by the nurse? a. The patient's radial pulse is 105 beats/minute. b. There is sediment and blood in the patient's urine. c. The blood pressure increases from 120/80 to 142/94. d. There are crackles audible throughout both lung fields.

d. There are crackles audible throughout both lung fields. Crackles throughout both lungs suggest that the patient may be experiencing pulmonary edema, a life-threatening adverse effect of hypertonic solutions. The increased pulse rate and blood pressure and the appearance of the urine also should be reported, but they are not as dangerous as the presence of fluid in the alveoli.

What effect does valvular regurgitation have on a patient? a. It causes a pressure gradient difference across an open valve. b. A pericardial friction rub is heard on the right sternal border of the chest. c. It leads to decreased flow of blood and hypertrophy of the preceding chamber. d. There is a backward flow of blood and volume overload in the preceding chamber

d. There is a backward flow of blood and volume overload in the preceding chamber Valvular regurgitation causes a backward flow of blood and volume overload in the preceding chamber. Without treatment, eventually hypertrophy of that chamber occurs. Stenosis causes a pressure gradient difference and decreased blood flow and hypertrophy of the preceding chamber. A pericardial friction rub is not related to valvular regurgitation but would be heard at the lower left sternal border of the chest.

After defibrillation, the advanced cardiac life support (ACLS) nurse says that the patient has pulseless electrical activity (PEA). What is most important for the nurse to understand about this rhythm? a. The HR is 40 to 60 bpm. b. Hypoxemia and hypervolemia are common with PEA. c. There is dissociated activity of the ventricle and atrium. d. There is electrical activity with no mechanical response.

d. There is electrical activity with no mechanical response Pulseless electrical activity (PEA) occurs when there is electrical activity on the ECG but no mechanical activity on assessment and therefore no pulse. PEA is the most common dysrhythmia seen after defibrillation and may be caused by hypovolemia, hypoxia, metabolic acidosis, altered potassium level, hypoglycemia, hypothermia, toxins, cardiac tamponade, thrombosis, tension pneumothorax, and trauma. Dissociated atria and ventricles is a third-degree AV block.

A patient is admitted with active tuberculosis (TB). The nurse should question a health care providers order to discontinue airborne precautions unless which assessment finding is documented? a. Chest x-ray shows no upper lobe infiltrates. b. TB medications have been taken for 6 months. c. Mantoux testing shows an induration of 10 mm. d. Three sputum smears for acid-fast bacilli are negative.

d. Three sputum smears for acid-fast bacilli are negative. Negative sputum smears indicate that Mycobacterium tuberculosis is not present in the sputum, and the patient cannot transmit the bacteria by the airborne route. Chest x-rays are not used to determine whether treatment has been successful. Taking medications for 6 months is necessary, but the multidrug-resistant forms of the disease might not be eradicated after 6 months of therapy. Repeat Mantoux testing would not be done because the result will not change even with effective treatment.

When palpating the patient's popliteal pulse, the nurse feels a vibration at the site. How should the nurse record this finding? a. Thready, weak pulse b. Bruit at the artery site c. Bounding pulse volume d. Thrill of the popliteal artery

d. Thrill of the popliteal artery A palpable vibration of a blood vessel is called a thrill and usually indicates a narrowed or bulging vessel wall. A weak, thready pulse has little pressure and is difficult to palpate. A bruit is an abnormal buzzing or humming sound that may be auscultated over diseased blood vessels, and a bounding pulse is an extra full, hard pulse that may occur with atherosclerosis or hypervolemia.

Which guideline should the nurse include when teaching a patient how to use a metered-dose inhaler (MDI)? a. After activating the MDI, breathe in as quickly as you can. b. Estimate the amount of remaining medicine in the MDI by floating the canister in water. c. Disassemble the plastic canister from the inhaler and rinse both pieces under running water every week. d. To determine how long the canister will last, divide the total number of puffs in the canister by the puffs needed per day.

d. To determine how long the canister will last, divide the total number of puffs in the canister by the puffs needed per day.

The laboratory tests for 4 patients show the following results. Which patient should the nurse teach first about preventing CAD because the patient is at the greatest risk for CAD even without other risk factors? a. Total cholesterol: 152 mg/dL, triglycerides: 148 mg/dL, LDL: 148 mg/dL, HDL: 52 mg/dL b. Total cholesterol: 160 mg/dL, triglycerides: 102 mg/dL, LDL: 138 mg/dL, HDL: 56 mg/dL c. Total cholesterol: 200 mg/dL, triglycerides: 150 mg/dL, LDL: 160 mg/dL, HDL: 48 mg/dL d. Total cholesterol: 250 mg/dL, triglycerides: 164 mg/dL, LDL: 172 mg/dL, HDL: 32 mg/dL

d. Total cholesterol: 250 mg/dL, triglycerides: 164 mg/dL, LDL: 172 mg/dL, HDL: 32 mg/dL All the results are abnormal. The patient in option "c" is close to being at risk; if this patient is a woman, the HDL is too low and the other results are at or near the cut off for being normal. The other patients' results are at acceptable levels.

Which actions prevent the dislodgement of a tracheostomy tube in the first 3 days after its placement (select all that apply)? a. Provide tracheostomy care every 24 hours. b. Keep the patient in the semi-Fowler's position at all times. c. Keep a same-size or larger replacement tube at the bedside. d. Tracheostomy ties are not changed for 24 hours after tracheostomy procedure. e. Suction the tracheostomy tube when there is a moist cough or a decreased arterial oxygen saturation by pulse oximetry (SpO2 ). f. A physician performs the first tracheostomy tube change 2 days after the tracheostomy.

d. Tracheostomy ties are not changed for 24 hours after tracheostomy procedure. e. Suction the tracheostomy tube when there is a moist cough or a decreased arterial oxygen saturation by pulse oximetry (SpO2 ). Changing the tracheostomy tapes soon after placement of the tracheostomy will be irritating to the trachea and could contribute to dislodgement of the tracheostomy tube. Suctioning should be done when increased secretions are evident in the tube to prevent the patient from severe coughing, which could cause tube dislodgement. Tracheostomy care is done every 8 hours. Keeping the patient in a semi-Fowler's position will not prevent dislodgement. Keeping an extra tube at the bedside will speed reinsertion if the tracheostomy tube is dislodged, but it will not prevent dislodgement. The physician will not change the tracheostomy tube until the insertion site is healed, approximately 3 to 5 days after original insertion.

Which information will the nurse include in the asthma teaching plan for a patient being discharged? a. Use the inhaled corticosteroid when shortness of breath occurs. b. Inhale slowly and deeply when using the dry powder inhaler (DPI). c. Hold your breath for 5 seconds after using the bronchodilator inhaler. d. Tremors are an expected side effect of rapidly acting bronchodilators.

d. Tremors are an expected side effect of rapidly acting bronchodilators. Tremors are a common side effect of short-acting b2-adrenergic (SABA) medications and not a reason to avoid using the SABA inhaler. Inhaled corticosteroids do not act rapidly to reduce dyspnea. Rapid inhalation is needed when using a DPI. The patient should hold the breath for 10 seconds after using inhalers.

What should the nurse reading the monitor strip call a rhythm with a regular PR interval but a blocked QRS complex? a. Asystole b. Atrial fibrillation c. First-degree AV block d. Type II second-degree AV block

d. Type II second-degree AV block In type II second-degree AV block, a P wave is nonconducted without progressive PR interval lengthening. It is usually from a block in a bundle branch, occurs in a ratio of 2 P waves-to-1 QRS complex, 3:1, and so on. Asystole is absence of ventricular activity. Atrial fibrillation has a chaotic P wave. First-degree AV block is a prolonged AV conduction time, so the PR interval is prolonged.

The nurse supervises unlicensed assistive personnel (UAP) who are providing care for a patient with right lower lobe pneumonia. The nurse should intervene if which action by UAP is observed? a. UAP splint the patients chest during coughing. b. UAP assist the patient to ambulate to the bathroom. c. UAP help the patient to a bedside chair for meals. d. UAP lower the head of the patients bed to 15 degrees.

d. UAP lower the head of the patients bed to 15 degrees. Positioning the patient with the head of the bed lowered will decrease ventilation. The other actions are appropriate for a patient with pneumonia.

When discussing risk factor modification for a 63-year-old patient who has a 5-cm abdominal aortic aneurysm, the nurse will focus discharge teaching on which patient risk factor? a. Male gender b. Turner syndrome c. Abdominal trauma history d. Uncontrolled hypertension

d. Uncontrolled hypertension All of the factors contribute to the patients risk, but only hypertension can potentially be modified to decrease the patients risk for further expansion of the aneurysm.

A 39-yr-old woman with a history of smoking and oral contraceptive use is admitted with a venous thromboembolism (VTE) and prescribed unfractionated heparin. What laboratory test should the nurse review to evaluate the expected effect of the heparin? a Platelet count b Activated clotting time (ACT) c International normalized ratio (INR) d Activated partial thromboplastin time (APTT)

d. Unfractionated heparin can be given by continuous IV for VTE treatment. When given IV, heparin requires frequent laboratory monitoring of clotting status as measured by activated partial thromboplastin time (aPTT). Platelet counts can decrease as an adverse reaction to heparin.

Which action will be included in the plan of care when the nurse is caring for a patient who is receiving nicardipine (Cardene) to treat a hypertensive emergency? a. Keep the patient NPO to prevent aspiration caused by nausea and possible vomiting. b. Organize nursing activities so that the patient has undisturbed sleep for 6 to 8 hours at night. c. Assist the patient up in the chair for meals to avoid complications associated with immobility. d. Use an automated noninvasive blood pressure machine to obtain frequent blood pressure (BP) measurements.

d. Use an automated noninvasive blood pressure machine to obtain frequent blood pressure (BP) measurements. Frequent monitoring of BP is needed when the patient is receiving rapid-acting IV antihypertensive medications. This can be most easily accomplished with an automated BP machine or arterial line. The patient will require frequent assessments, so allowing 6 to 8 hours of undisturbed sleep is not appropriate. When patients are receiving IV vasodilators, bed rest is maintained to prevent decreased cerebral perfusion and fainting. There is no indication that this patient is nauseated or at risk for aspiration, so an NPO status is unnecessary.

A priority consideration in the management of the older adult with hypertension is to a. Prevent primary hypertension from converting to secondary hypertension. b. Recognize that the older adult is less likely to adhere to the drug therapy regimen than a younger adult. c. Ensure that the patient receives larger initial doses of antihypertensive drugs because of impaired absorption. d. Use precise technique in assessing the BP of the patient because of the possible presence of an auscultatory gap.

d. Use precise technique in assessing the BP of the patient because of the possible presence of an auscultatory gap.

The nurse needs to quickly estimate the heart rate for a patient with a regular heart rhythm. Which method will be best to use? a. Count the number of large squares in the R-R interval and divide by 300. b. Print a 1-minute electrocardiogram (ECG) strip and count the number of QRS complexes. c. Calculate the number of small squares between one QRS complex and the next and divide into 1500. d. Use the 3-second markers to count the number of QRS complexes in 6 seconds and multiply by 10.

d. Use the 3-second markers to count the number of QRS complexes in 6 seconds and multiply by 10. This is the quickest way to determine the ventricular rate for a patient with a regular rhythm. All the other methods are accurate, but take longer.

Which of the following descriptions matches the mechanism of "facilitated diffusion"? a. Force determined by osmolality of a fluid b. Pressure exerted by plasma proteins c. Force exerted by a fluid d. Uses a protein carrier molecule

d. Uses a protein carrier molecule

Which function test fits description "Maximum amount of air that can be exhaled after maximum inspiration"? a. FEV1 b. TLC c. Vt d. VC

d. VC Vital capacity

Risk Factor for or Response to Respiratory Problem: Noncompliance with treatment plan, conflict with values. Which Functional Health Pattern does it fall under? a. Role-relationship b. Sexuality-reproductive c. Coping- stress tolerance d. Value-belief

d. Value-belief

A 62-year old man with chronic anemia is experiencing increased fatigue and occasional palpitations at rest. The nurse would expect the patients laboratory findings to include a. a hematocrit (Hct) of 38%. b. an RBC count of 4,500,000/mL. c. normal red blood cell (RBC) indices. d. a hemoglobin (Hgb) of 8.6 g/dL (86 g/L).

d. a hemoglobin (Hgb) of 8.6 g/dL (86 g/L). The patients clinical manifestations indicate moderate anemia, which is consistent with a Hgb of 6 to 10 g/dL. The other values are all within the range of normal.

The patient asks the nurse about valsartan (Diovan), the new medication prescribed for blood pressure. What is the best explanation the nurse can use to explain the action of this medication? a. Prevents the conversion of angiotensin I to angiotensin II b. Acts directly on smooth muscle of arterioles to cause vasodilation c. Decreases extracellular fluid volume by increasing Na + and Cl − excretion with water d. Vasodilation, prevents the action of angiotensin II, and promotes increased salt and water excretion

d. Vasodilation, prevents the action of angiotensin II, and promotes increased salt and water excretion Valsartan (Diovan) is an angiotensin II receptor blocker (ARB). ARBs prevent the action of angiotensin II, produce vasodilation, and increase salt and water excretion. Angiotensin-converting enzyme (ACE) inhibitors prevent the conversion of angiotensin I to angiotensin II. Direct vasodilators act directly on smooth muscle of arterioles to cause vasodilation. Thiazide diuretics decrease extracellular fluid volume by increasing Na + and Cl − excretion with water

What is the most effective treatment for cystic fibrosis? a. Heart-lung transplant b. Administration of prophylactic antibiotics c. Administration of nebulized bronchodilators d. Vigorous and consistent airway clearance techniques

d. Vigorous and consistent airway clearance techniques The major goals of therapy in CF are to relieve airway obstruction and control infection. Airway clearance techniques (ACT) are the mainstay of treatment. Aerobic exercise is effective in clearing the airways, requiring increased nutrition and fluid, plus salt loss replacement. Antibiotics are used for early signs of infection, and long courses are necessary, but they are not used prophylactically. Bronchodilators have shown no long-term benefit. Although CF has become a leading indication for heart-lung transplant, this treatment option may not be available for many patients.

What should the nurse teach a patient with intermittent asthma about identifying specific triggers of asthma? a. Food and drug allergies do not cause respiratory symptoms. b. Exercise-induced asthma is seen only in persons with sensitivity to cold air. c. Asthma attacks are psychogenic in origin and can be controlled with relaxation techniques. d. Viral upper respiratory infections are a common precipitating factor in acute asthma attacks.

d. Viral upper respiratory infections are a common precipitating factor in acute asthma attacks. Respiratory infections are one of the most common precipitating factors of an acute asthma attack. Sensitivity to food and drugs may also precipitate attacks, and exercise-induced asthma occurs after exercise, especially in cold, dry air. Psychologic factors may interact with the asthmatic response to worsen the disease, but it is not a psychosomatic disease.

The nurse develops a teaching plan to help increase activity tolerance at home for an older adult with severe chronic obstructive pulmonary disease (COPD). Which instructions would be most appropriate for the nurse to include in the plan of care? a. Stop exercising when short of breath. b. Walk until pulse rate exceeds 130 beats/minute. c. Limit exercise to activities of daily living (ADLs). d. Walk 15 to 20 minutes daily at least 3 times/week.

d. Walk 15 to 20 minutes daily at least 3 times/week. Encourage the patient to walk 15 to 20 minutes a day at least three times a week with gradual increases. Shortness of breath is normal with exercise and not an indication that the patient should stop. Limiting exercise to ADLs will not improve the patients exercise tolerance. A 70-year-old patient should have a pulse rate of 120 or less with exercise (80% of the maximal heart rate of 150).

The nurse is caring for a patient who is being discharged after an emergency splenectomy following an automobile accident. Which instructions should the nurse include in the discharge teaching? a. Watch for excess bruising. b. Check for swollen lymph nodes. c. Take iron supplements to prevent anemia. d. Wash hands and avoid persons who are ill.

d. Wash hands and avoid persons who are ill. Splenectomy increases the risk for infection, especially with gram-positive bacteria. The risks for lymphedema, bleeding, and anemia are not increased after a person has a splenectomy

An elderly client is being monitored for evidence of congestive heart failure. To detect early signs of heart failure, the nurse would instruct the CNA to do which of the following during care of the patient? a. Observe ECG reading and report deviations to the nurse b. Assist the client with ambulation 3x during the shift c. Monitor vital q15 minutes and report each finding to the nurse d. Weight the patient and report I/Os

d. Weight the patient and report I/Os Due to fluid accumulation, an expanded blood volume can result when the heart fails. Body weight is a sensitive indicator of water and sodium retention, which will manifest itself with edema, dyspnea - especially nocturnal - and pedal edema. Patients also should be instructed about the need to perform daily weights upon discharge to monitor body water. It is not within the role of the CNA to monitor ECG readings, and ambulation is not an assessment. Vital signs every 15 minute are not necessary for this level of patient care.

A nurse reviews the laboratory data for an older patient. The nurse would be most concerned about which finding? a. Hematocrit of 35% b. Hemoglobin of 11.8 g/dL c. Platelet count of 400,000/L d. White blood cell (WBC) count of 2800/L

d. White blood cell (WBC) count of 2800/L Because the total WBC count is not usually affected by aging, the low WBC count in this patient would indicate that the patients immune function may be compromised and the underlying cause of the problem needs to be investigated. The platelet count is normal. The slight decrease in hemoglobin and hematocrit are not unusual for an older patient.

The nurse reviews the complete blood count (CBC) and white blood cell (WBC) differential of a patient admitted with abdominal pain. Which information will be most important for the nurse to communicate to the health care provider? a. Monocytes 4% b. Hemoglobin 13.6 g/dL c. Platelet count 168,000/L d. White blood cells (WBCs) 15,500/L

d. White blood cells (WBCs) 15,500/L The elevation in WBCs indicates that the patient has an inflammatory or infectious process ongoing, which may be the cause of the patients pain, and that further diagnostic testing is needed. The monocytes are at a normal level. The hemoglobin and platelet counts are normal.

A patient asks, "How does air get into my lungs?" The nurse bases her answer on knowledge that air moves into the lungs because of a. positive intrathoracic pressure. b. contraction of the accessory abdominal muscles. c. stimulation of the respiratory muscles by the chemoreceptors. d. a decrease in intrathoracic pressure from an increase in thoracic cavity size.

d. a decrease in intrathoracic pressure from an increase in thoracic cavity size.

Following an acute myocardial infarction, a previously healthy 63-year-old develops clinical manifestations of heart failure. The nurse anticipates discharge teaching will include information about a. digitalis preparations. b. b-adrenergic blockers. c. calcium channel blockers. d. angiotensin-converting enzyme (ACE) inhibitors.

d. angiotensin-converting enzyme (ACE) inhibitors. ACE inhibitor therapy is currently recommended to prevent the development of heart failure in patients who have had a myocardial infarction and as a first-line therapy for patients with chronic heart failure. Digoxin therapy for heart failure is no longer considered a first-line measure, and digoxin is added to the treatment protocol when therapy with other medications such as ACE-inhibitors, diuretics, and b-adrenergic blockers is insufficient. Calcium channel blockers are not generally used in the treatment of heart failure. The b-adrenergic blockers are not used as initial therapy for new onset heart failure.

A patient at the clinic says, I have always taken a walk after dinner, but lately my leg cramps and hurts after just a few minutes of starting. The pain goes away after I stop walking, though. The nurse should a. check for the presence of tortuous veins bilaterally on the legs. b. ask about any skin color changes that occur in response to cold. c. assess for unilateral swelling, redness, and tenderness of either leg. d. assess for the presence of the dorsalis pedis and posterior tibial pulses.

d. assess for the presence of the dorsalis pedis and posterior tibial pulses. The nurse should assess for other clinical manifestations of peripheral arterial disease in a patient who describes intermittent claudication. Changes in skin color that occur in response to cold are consistent with Raynauds phenomenon. Tortuous veins on the legs suggest venous insufficiency. Unilateral leg swelling, redness, and tenderness indicate venous thromboembolism (VTE).

During a physical examination of a 74-year-old patient, the nurse palpates the point of maximal impulse(PMI) in the sixth intercostal space lateral to the left midclavicular line. The most appropriate action for the nurse to take next will be to a. ask the patient about risk factors for atherosclerosis. b. document that the PMI is in the normal anatomic location. c. auscultate both the carotid arteries for the presence of a bruit. d. assess the patient for symptoms of left ventricular hypertrophy.

d. assess the patient for symptoms of left ventricular hypertrophy.

During a physical examination of a 74-year-old patient, the nurse palpates the point of maximal impulse (PMI) in the sixth intercostal space lateral to the left midclavicular line. The most appropriate action for the nurse to take next will be to a. ask the patient about risk factors for atherosclerosis. b. document that the PMI is in the normal anatomic location. c. auscultate both the carotid arteries for the presence of a bruit. d. assess the patient for symptoms of left ventricular hypertrophy.

d. assess the patient for symptoms of left ventricular hypertrophy. The PMI should be felt at the intersection of the fifth intercostal space and the left midclavicular line. A PMI located outside these landmarks indicates possible cardiac enlargement, such as with left ventricular hypertrophy. Cardiac enlargement is not necessarily associated with atherosclerosis or carotid artery disease

The ECG monitor of a patient in the cardiac care unit after an MI shows ventricular bigeminy with a rate of 50 beats/min. The nurse would a. perform defibrillation. b. administer IV amiodarone. c. prepare for temporary pacemaker insertion. d. assess the patient's response to the dysrhythmia.

d. assess the patient's response to the dysrhythmia.

The ECG monitor of a patient in the cardiac care unit after an MI shows ventricular bigeminy with a rate of 50 beats/min. The nurse would a. perform defibrillation. b. administer IV amiodarone. c. prepare for temporary pacemaker insertion. d. assess the patient's response to the dysrhythmia.

d. assess the patient's response to the dysrhythmia. Correct answer: d Rationale: A premature ventricular contraction (PVC) is a contraction originating in an ectopic focus in the ventricles. When every other beat is a PVC, the rhythm is called ventricular bigeminy. PVCs are usually a benign finding in patients with a normal heart. In patients with heart disease, PVCs may reduce the cardiac output and precipitate angina and heart failure, depending on the frequency. Because PVCs in coronary artery disease (CAD) or acute myocardial infarction indicate ventricular irritability, the patient's physiologic response to PVCs must be monitored. Assessing the patient's hemodynamic status is important for deciding the need for drug therapy.

While caring for a 23-year-old patient with mitral valve prolapse (MVP) without valvular regurgitation, the nurse determines that discharge teaching has been effective when the patient states that it will be necessary to a. take antibiotics before any dental appointments. b. limit physical activity to avoid stressing the heart. c. take an aspirin a day to prevent clots from forming on the valve. d. avoid use of over-the-counter (OTC) medications that contain stimulant drugs.

d. avoid use of over-the-counter (OTC) medications that contain stimulant drugs. Use of stimulant medications should be avoided by patients with MVP because these may exacerbate symptoms. Daily aspirin and restricted physical activity are not needed by patients with mild MVP. Antibiotic prophylaxis is needed for patients with MVP with regurgitation but will not be necessary for this patient.

Several hours after an open surgical repair of an abdominal aortic aneurysm, the UAP reports to the nurse that urinary output for the past 2 hours has been 40 mL. The nurse notifies the health care provider and anticipates an order for a(n) a. hemoglobin count. b. additional antibiotic. c. decrease in IV infusion rate. d. blood urea nitrogen (BUN) level.

d. blood urea nitrogen (BUN) level. The decreased urine output suggests decreased renal perfusion, and monitoring of renal function is needed. There is no indication that infection is a concern, so antibiotic therapy and a WBC count are not needed. The IV rate may be increased because hypovolemia may be contributing to the patients decreased urinary output.

The most common type of leukemia in adults in western countries is a. acute myelocytic leukemia. b. acute lymphocytic leukemia. c. chronic myelocytic leukemia. d. chronic lymphocytic leukemia.

d. chronic lymphocytic leukemia.

Which of these electrolyte imbalances is the cause of "metabolic alkalosis"? (select all that apply) a. hypernatremia b. hyponatremia c. hyperkalemia d. hypokalemia e. hypercalcemia f. hypocalcemia g. hyperphosphatemia h. hypophosphatemia i. hypermagnesemia j. hypomagnesemia

d. hypokalemia

When obtaining a health history from a 72-year-old man with peripheral arterial disease (PAD) of the lower extremities, the nurse asks about a history of related conditions, including a. venous thrombosis. b. venous stasis ulcers. c. pulmonary embolism. d. coronary artery disease (CAD).

d. coronary artery disease (CAD). Regardless of the location, atherosclerosis is responsible for peripheral arterial disease (PAD) and is related to other cardiovascular disease and its risk factors, such as coronary artery disease (CAD) and carotid artery disease. Venous thrombosis, venous stasis ulcers, and pulmonary embolism are diseases of the veins and are not related to atherosclerosis.

Cancer arising from granulocytic cells in the bone marrow will have the primary effect of causing a. risk for hemorrhage. b. altered oxygenation. c. decreased production of antibodies. d. decreased phagocytosis of bacteria.

d. decreased phagocytosis of bacteria

Because myelodysplastic syndrome arises from the pluripotent hematopoietic stem cell in the bone marrow, laboratory results the nurse would expect to find include a(n) a. excess of T cells. b. excess of platelets. c. deficiency of granulocytes. d. deficiency of all cellular blood components.

d. deficiency of all cellular blood components.

A 56-year-old patient who has no previous history of hypertension or other health problems suddenly develops a blood pressure (BP) of 198/110 mm Hg. After reconfirming the BP, it is appropriate for the nurse to tell the patient that a. a BP recheck should be scheduled in a few weeks. b. dietary sodium and fat content should be decreased. c. there is an immediate danger of a stroke and hospitalization will be required. d. diagnosis of a possible cause, treatment, and ongoing monitoring will be needed.

d. diagnosis of a possible cause, treatment, and ongoing monitoring will be needed. A sudden increase in BP in a patient over age 50 with no previous hypertension history or risk factors indicates that the hypertension may be secondary to some other problem. The BP will need treatment and ongoing monitoring. If the patient has no other risk factors, a stroke in the immediate future is unlikely. There is no indication that dietary salt or fat intake have contributed to this sudden increase in BP, and reducing intake of salt and fat alone will not be adequate to reduce this BP to an acceptable level.

A barrier to hospice referrals for patients with stage D heart failure is a. family member refusal. b. scarcity of hospice facilities. c. history of pacemaker placement. d. difficulty in estimating prognosis.

d. difficulty in estimating prognosis.

A patient in the emergency department complains of back pain and difficulty breathing 15 minutes after a transfusion of packed red blood cells is started. The nurses first action should be to a. administer oxygen therapy at a high flow rate. b. obtain a urine specimen to send to the laboratory. c. notify the health care provider about the symptoms. d. disconnect the transfusion and infuse normal saline.

d. disconnect the transfusion and infuse normal saline. The patients symptoms indicate a possible acute hemolytic reaction caused by the transfusion. The first action should be to disconnect the transfusion and infuse normal saline. The other actions also are needed but are not the highest priority.

A patient has a normal cardiac rhythm and a heart rate of 72 beats/minute. The nurse determines that the P-R interval is 0.24 seconds. The most appropriate intervention by the nurse would be to a. notify the health care provider immediately. b. give atropine per agency dysrhythmia protocol. c. prepare the patient for temporary pacemaker insertion. d. document the finding and continue to monitor the patient.

d. document the finding and continue to monitor the patient. First-degree atrioventricular (AV) block is asymptomatic and requires ongoing monitoring because it may progress to more serious forms of heart block. The rate is normal, so there is no indication that atropine is needed. Immediate notification of the health care provider about an asymptomatic rhythm is not necessary.

After noting a pulse deficit when assessing a 74-year-old patient who has just arrived in the emergency department, the nurse will anticipate that the patient may require a. emergent cardioversion. b. a cardiac catheterization. c. hourly blood pressure (BP) checks. d. electrocardiographic (ECG) monitoring.

d. electrocardiographic (ECG) monitoring. Pulse deficit is a difference between simultaneously obtained apical and radial pulses. It indicates that there may be a cardiac dysrhythmia that would best be detected with ECG monitoring. Frequent BP monitoring, cardiac catheterization, and emergent cardioversion are used for diagnosis and/or treatment of cardiovascular disorders but would not be as helpful in determining the immediate reason for the pulse deficit.

When developing a teaching plan for a 61-year-old man with the following risk factors for coronary artery disease (CAD), the nurse should focus on the a. family history of coronary artery disease. b. increased risk associated with the patients gender. c. increased risk of cardiovascular disease as people age. d. elevation of the patients low-density lipoprotein (LDL) level.

d. elevation of the patients low-density lipoprotein (LDL) level. Because family history, gender, and age are nonmodifiable risk factors, the nurse should focus on the patients LDL level. Decreases in LDL will help reduce the patients risk for developing CAD.

The nurse would expect that a patient with von Willebrand disease undergoing surgery would be treated with administration of vWF and a. thrombin. b. factor VI. c. factor VII. d. factor VIII.

d. factor VIII.

A client is admitted to the hospital with acid-base imbalances. ABGs results are are pH 7.35, PaCO2 52, HCO3 30. How should the nurse interpret findings? a. uncompensated metabolic acidosis b. partially compensated respiratory acidosis c. uncompensated respiratory alkalosis d. fully compensated respiratory acidosis

d. fully compensated respiratory acidosis

The nurse is unable to flush a central venous access device and suspects occlusion. The best nursing intervention would be to a. apply warm moist compresses to the insertion site. b. try to force 10 mL of normal saline into the device. c. place the patient on the left side with the head down. d. have the patient change positions, raise arm, and cough.

d. have the patient change positions, raise arm, and cough.

A patient is diagnosed with Lt. ventricular hypertrophy from hypertension, which potential disease process could develop a. cirrhosis b. neuropathy c. Addison's d. heart failure

d. heart failure

Tachycardia that is a response of the sympathetic nervous system to the pain of ischemia is detrimental because it increases oxygen demand and a. increases cardiac output. b. causes reflex hypotension. c. may lead to atrial dysrhythmias. d. impairs perfusion of the coronary arteries.

d. impairs perfusion of the coronary arteries. An increased heart rate (HR) decreases the time the heart spends in diastole, which is the time of greatest coronary blood flow. Unlike other arteries, coronary arteries are perfused when the myocardium relaxes and blood backflows from the aorta into the sinuses of Valsalva, which have openings to the right and left coronary arteries. Thus the heart has a decreased oxygen supply at a time when there is an increased oxygen demand. Tachycardia may also lead to ventricular dysrhythmia. The other options are incorrect.

A patient has metabolic acidosis secondary to type 1 diabetes. what physiologic response should the nurse expect to assess in the patient? a. vomiting b. increased urination c. decrease heart rate d. increased respiratory rate

d. increased respiratory rate

The nurse plans close monitoring for the patient during electrophysiologic study (EPS) because this study a. requires the use of dyes that irritate the myocardium. b. causes myocardial ischemia, resulting in dysrhythmias. c. involves the use of anticoagulants to prevent thrombus and embolism. d. induces dysrhythmias that may require cardioversion or defibrillation to correct.

d. induces dysrhythmias that may require cardioversion or defibrillation to correct. Electrophysiologic study (EPS) involves electrical stimulation to various areas of the atrium and ventricle to determine the inducibility of dysrhythmias and often induces ventricular tachycardia or ventricular fibrillation. The patient may have "near-death" experiences and requires emotional support if this occurs. Dye and anticoagulants are used for coronary angiograms.

A new nurse caring for a pt. with hypocalcemia needs further teaching about this condition if they: a. Vitamin D supplements b. test for hypocalcemia with Chvostek's sign c. IV calcium gluconate d. inducing hyperventilation

d. inducing hyperventilation

Which body compartment has the most amount of fluid in the body? a. extracellular b. blood vessels c. interstitial spaces d. intracellular

d. intracellular

The nurse teaches the patient being evaluated for rhythm disturbances with a Holter monitor to a. connect the recorder to a computer once daily. b. exercise more than usual while the monitor is in place. c. remove the electrodes when taking a shower or tub bath. d. keep a diary of daily activities while the monitor is worn.

d. keep a diary of daily activities while the monitor is worn. The patient is instructed to keep a diary describing daily activities while Holter monitoring is being accomplished to help correlate any rhythm disturbances with patient activities. Patients are taught that they should not take a shower or bath during Holter monitoring and that they should continue with their usual daily activities. The recorder stores the information about the patients rhythm until the end of the testing, when it is removed and the data are analyzed.

A 73-year-old patient with chronic atrial fibrillation develops sudden severe pain, pulselessness, pallor, and coolness in the right leg. The nurse should notify the health care provider and immediately a. apply a compression stocking to the leg. b. elevate the leg above the level of the heart. c. assist the patient in gently exercising the leg. d. keep the patient in bed in the supine position.

d. keep the patient in bed in the supine position. The patients history and clinical manifestations are consistent with acute arterial occlusion, and resting the leg will decrease the oxygen demand of the tissues and minimize ischemic damage until circulation can be restored. Elevating the leg or applying an elastic wrap will further compromise blood flow to the leg. Exercise will increase oxygen demand for the tissues of the leg.

In teaching a patient with hypertension about controlling the condition, the nurse recognizes that: a. all patients with elevated BP require medication b. obese persons must achieve a normal weight to lower BP c. It is not necessary to limit salt in the diet if taking a diuretic d. lifestyle modifications are indicated for all persons with elevated BP

d. lifestyle modifications are indicated for all persons with elevated BP

Which of the following acid-base imbalances is the common cause for diabetic ketosis? a. respiratory acidosis b. respiratory alkalosis c. metabolic alkalosis d. metabolic acidosis

d. metabolic acidosis

Which of the following acid-base imbalances is the common cause for renal failure? a. respiratory acidosis b. respiratory alkalosis c. metabolic alkalosis d. metabolic acidosis

d. metabolic acidosis

Which of the following acid-base imbalances is the common cause for severe shock? a. respiratory acidosis b. respiratory alkalosis c. metabolic alkalosis d. metabolic acidosis

d. metabolic acidosis

Which of the following acid-base imbalances matches the mechanism "decreased base bicarbonate (HCO3-)"? a. respiratory acidosis b. respiratory alkalosis c. metabolic alkalosis d. metabolic acidosis

d. metabolic acidosis

When teaching the patient with newly diagnosed heart failure about a 2000-mg sodium diet, the nurse explains that foods to be restricted include a. canned and frozen fruits. b. fresh or frozen vegetables. c. eggs and other high-protein foods. d. milk, yogurt, and other milk products.

d. milk, yogurt, and other milk products. Milk and yogurt naturally contain a significant amount of sodium, and intake of these should be limited for patients on a diet that limits sodium to 2000 mg daily. Other milk products, such as processed cheeses, have very high levels of sodium and are not appropriate for a 2000-mg sodium diet. The other foods listed have minimal levels of sodium and can be eaten without restriction.

A critical action by the nurse caring for a patient with an acute exacerbation of polycythemia vera is to a. place the patient on bed rest. b. administer iron supplements. c. avoid use of aspirin products. d. monitor fluid intake and output.

d. monitor fluid intake and output. Monitoring hydration status is important during an acute exacerbation because the patient is at risk for fluid overload or underhydration. Aspirin therapy is used to decrease risk for thrombosis. The patient should be encouraged to ambulate to prevent deep vein thrombosis (DVT). Iron is contraindicated in patients with polycythemia vera.

A nurse is reviewing the ABG results for four clients. Which of the following findings should the nurse identify as metabolic acidosis? a. pH 7.51, PaO2 94 mm Hg, PaCO2 38 mm Hg, HCO3- 29 mEq/L b. pH 7.48, PaO2 89 mm Hg, PaCO2 30 mm Hg, HCO3- 24 mEq/L c. pH 7.36, PaO2 77 mm Hg, PaCO2 52 mm Hg, HCO3- 26 mEq/L d. pH 7.26, PaO2 84 mm Hg, PaCO2 38 mm Hg, HCO3- 20 mEq/L

d. pH 7.26, Pa02 84mm hg, PaC02 38 mmhg, HCO3- 20 mEq/L When pH and HCO3- are both above or below the expected reference range, a metabolic imbalance is present. A pH of 7.26 indicates acidosis and a HCO3- of 20 mEq/L indicates the acidosis is due to a metabolic cause. Therefore, the nurse should identify these findings as metabolic acidosis.

The laboratory has just called with the arterial blood gas (ABG) results on four patients. Which result is most important for the nurse to report immediately to the health care provider? a. pH 7.34, PaO2 82 mm Hg, PaCO2 40 mm Hg, and O2 sat 97% b. pH 7.35, PaO2 85 mm Hg, PaCO2 45 mm Hg, and O2 sat 95% c. pH 7.46, PaO2 90 mm Hg, PaCO2 32 mm Hg, and O2 sat 98% d. pH 7.31, PaO2 91 mm Hg, PaCO2 50 mm Hg, and O2 sat 96%

d. pH 7.31, PaO2 91 mm Hg, PaCO2 50 mm Hg, and O2 sat 96% These ABGs indicate uncompensated respiratory acidosis and should be reported to the health care provider. The other values are normal or close to normal.

The nurse would interpret an induration of 5 mm resulting from tuberculin skin testing as a positive finding in which patient? a. patient with a history of illegal IV drug use b. patient with diabetes & end-stage kidney disease c. patient who immigrated from India 3 mths ago d. patient who has human immunodeficiency virus- infected

d. patient who has human immunodeficiency virus- infected Induration of 5 mm in an HIV-infected person is considered a positive reaction.

The nurse prepares a patient for synchronized cardioversion knowing that cardioversion differs from defibrillation in that a. defibrillation delivers a lower dose of electrical energy. b. cardioversion is a treatment for atrial bradydysrhythmias. c. defibrillation is synchronized to deliver a shock during the QRS complex. d. patients should be sedated if cardioversion is done on a nonemergency basis.

d. patients should be sedated if cardioversion is done on a nonemergency basis.

The nurse prepares a patient for synchronized cardioversion knowing that cardioversion differs from defibrillation in that a. defibrillation delivers a lower dose of electrical energy. b. cardioversion is a treatment for atrial bradydysrhythmias. c. defibrillation is synchronized to deliver a shock during the QRS complex. d. patients should be sedated if cardioversion is done on a nonemergency basis.

d. patients should be sedated if cardioversion is done on a nonemergency basis. Correct answer: d Rationale: Synchronized cardioversion is the therapy of choice for patients with hemodynamically unstable ventricular or supraventricular tachydysrhythmias. A synchronized circuit in the defibrillator delivers a countershock that is programmed to occur on the R wave of the QRS complex of the electrocardiogram. The synchronizer switch must be turned on when cardioversion is planned. The procedure for synchronized cardioversion is the same as for defibrillation with a few exceptions: If synchronized cardioversion is done on a nonemergency basis, the patient is sedated before the procedure, and the initial energy needed for synchronized cardioversion is less than the energy needed for defibrillation.

A nurse is teaching a client who has a new diagnosis of severe peripheral arterial disease. Which of the following instructions should the nurse include? a. wear tightly-fitted insulated socks with shoes when going outside b. elevate both legs above the heart when resting c. apply a heating pad to both legs for comfort d. place both legs in dependent position while sleeping

d. place both legs in dependent position while sleeping Instruct the client to place their legs in a dependent position, such as hanging off the edge of the bed while sleeping. This can alleviate swelling and discomfort of the legs.

The nurse has identified a nursing diagnosis of acute pain related to inflammatory process for a patient with acute pericarditis. The priority intervention by the nurse for this problem is to a. teach the patient to take deep, slow breaths to control the pain. b. force fluids to 3000 mL/day to decrease fever and inflammation. c. remind the patient to request opioid pain medication every 4 hours. d. place the patient in Fowlers position, leaning forward on the overbed table.

d. place the patient in Fowlers position, leaning forward on the overbed table. Sitting upright and leaning forward frequently will decrease the pain associated with pericarditis. Forcing fluids will not decrease the inflammation or pain. Taking deep breaths will tend to increase pericardial pain. Opioids are not very effective at controlling pain caused by acute inflammatory conditions and are usually ordered PRN. The patient would receive scheduled doses of a nonsteroidal antiinflammatory drug (NSAID).

A patient has the following arterial blood gas results: pH 7.52, PaCO2 30 mm Hg, HCO3− 24 mEq/L. The nurse determines that these results indicate a. metabolic acidosis. b. metabolic alkalosis. c. respiratory acidosis. d. respiratory alkalosis.

d. respiratory alkalosis.

During the postoperative care of a 76-year-old patient, the nurse monitors the patient's intake and output carefully, knowing that the patient is at risk for fluid and electrolyte imbalances primarily because a. older adults have an impaired thirst mechanism and need reminding to drink fluids. b. water accounts for a greater percentage of body weight in the older adult than in younger adults. c. older adults are more likely than younger adults to lose extracellular fluid during surgical procedures. d. small losses of fluid are significant because body fluids account for 45% to 50% of body weight in older adults.

d. small losses of fluid are significant because body fluids account for 45% to 50% of body weight in older adults.

An appropriate nursing intervention to assist a patient with pneumonia manage thick secretions and fatigue would be to a. perform postural drainage every hour. b. provide analgesics as ordered to promote patient comfort. c. administer O2 as prescribed to maintain optimal O2 levels. d. teach the patient how to cough effectively and expectorate secretions

d. teach the patient how to cough effectively and expectorate secretions

Nadolol (Corgard) is prescribed for a patient with chronic stable angina and left ventricular dysfunction. To determine whether the drug is effective, the nurse will monitor for a. decreased blood pressure and heart rate. b. fewer complaints of having cold hands and feet. c. improvement in the strength of the distal pulses. d. the ability to do daily activities without chest pain.

d. the ability to do daily activities without chest pain. Because the medication is ordered to improve the patients angina, effectiveness is indicated if the patient is able to accomplish daily activities without chest pain. Blood pressure and heart rate may decrease, but these data do not indicate that the goal of decreased angina has been met. The noncardioselective b-adrenergic blockers can cause peripheral vasoconstriction, so the nurse would not expect an improvement in distal pulse quality or skin temperature.

Multiple drugs are often used in combinations to treat leukemia and lymphoma because a. there are fewer toxic and side effects. b. the chance that one drug will be effective is increased. c. the drugs are more effective without causing side effects. d. the drugs work by different mechanisms to maximize killing of cancer cells.

d. the drugs work by different mechanisms to maximize killing of cancer cells.

Complications of transfusions that can be decreased by using leukocyte depletion or reduction of RBC transfusion are a. chills and hemolysis. b. leukostasis and neutrophilia. c. fluid overload and pulmonary edema. d. transmission of cytomegalovirus and fever.

d. transmission of cytomegalovirus and fever.

A major consideration in the management of the other adult with hypertension is to: a. prevent primary hypertension from converting to secondary hypertension b. recognize that the older adult is less likely to adhere to the drug therapy regimen than a younger adult c. ensure that the patient receives larger initial doses of antihypertensive drugs because of impaired absorption d. use careful technique in assessing the BP of the patient because of the possible presence of ab auscultatory gap

d. use careful technique in assessing the BP of the patient because of the possible presence of ab auscultatory gap

The nurse obtains a rhythm strip on a patient who has had a myocardial infarction and makes the following analysis: no visible P waves, P-R interval not measurable, ventricular rate 162, R-R interval regular, and QRS complex wide and distorted, QRS duration 0.18 second. The nurse interprets the patients cardiac rhythm as a. atrial flutter. b. sinus tachycardia. c. ventricular fibrillation. d. ventricular tachycardia.

d. ventricular tachycardia. The absence of P waves, wide QRS, rate >150 beats/minute, and the regularity of the rhythm indicate ventricular tachycardia. Atrial flutter is usually regular, has a narrow QRS configuration, and has flutter waves present representing atrial activity. Sinus tachycardia has P waves. Ventricular fibrillation is irregular and does not have a consistent QRS duration.

Which bleeding disorder affects both genders, is autosomal dominant, and will have laboratory results showing prolonged bleeding time? a. Hemophilia A b. Hemophilia B c. Thrombocytopenia d. von Willebrand disease

d. von Willebrand disease This description is characteristic of von Willebrand disease with prolonged bleeding time occurring because of defective platelets, which does not occur with either type of hemophilia. Although inherited thrombocytopenia is believed to be autosomal dominant, the number of platelets is decreased.

The nurse assesses the patient with chronic biventricular HF for paroxysmal nocturnal dyspnea (PND) by questioning the patient regarding a. the presence of restlessness and confusion. b. frequent awakening to void during the night. c. the presence of swelling in dependent body areas. d. waking in a panic with a feeling of suffocation.

d. waking in a panic with a feeling of suffocation. Paroxysmal nocturnal dyspnea (PND) is awakening from sleep with a feeling of suffocation and a need to sit up to be able to breathe. Patients learn that sleeping with the upper body elevated on several pillows helps prevent PND. Behavior changes are seen in late stages of HF. Nocturia occurs with HF as fluid moves back into the vascular system during recumbency, increasing renal blood flow. Dependent edema does not indicate PND.

Which obstructive pulmonary disease would a 30-year-old white female patient with a parent with the disease be most likely to be diagnosed with? a. COPD b. Asthma c. Cystic fibrosis d. α1 -Antitrypsin (AAT) deficiency

d. α1 -Antitrypsin (AAT) deficiency α1 -Antitrypsin (AAT) deficiency is an autosomal recessive disorder that is a genetic risk factor for COPD with symptoms (often by age 40 years) in people with no tobacco use and family history of COPD or liver disease. AAT occurs in about 3% of people diagnosed with COPD. Although CF occurs in 1 in 3000 white births, legislation requires babies to be screened at birth, so it would have been previously diagnosed. Asthma is a multifactorial genetic disorder.

What are the characteristics of a fenestrated tracheostomy tube (select all that apply)? a. The cuff passively fills with air. b. Cuff pressure monitoring is not required. c. It has two tubings with one opening just above the cuff. d. Patient can speak with an attached air source with the cuff inflated. e. Airway obstruction is likely if the exact steps are not followed to produce speech. f. Airflow around the tube and through the window allows speech when the cuff is deflated and the plug is inserted.

e. Airway obstruction is likely if the exact steps are not followed to produce speech. f. Airflow around the tube and through the window allows speech when the cuff is deflated and the plug is inserted. The fenestrated tracheostomy tube has openings on the outer cannula to allow air to pass over the vocal cords to allow speaking. If the steps of using the fenestrated tracheostomy tube are not completed in the correct order, severe respiratory distress may result. The cuff of the tracheostomy tube with a foam-filled cuff passively fills with air and does require pressure monitoring, although cuff integrity must be assessed daily. The speaking tracheostomy tube has 2 tubes attached. One tube allows air to pass over the vocal cords to enable the person to speak with the cuff inflated.

The nurse is preparing the patient for and will assist the health care provider with a thoracentesis in the patient's room. Which of the following is the fifth action? a. Verify breath sounds in all fields. b. Obtain the supplies that will be used. c. Send labeled specimen containers to the laboratory. d. Direct the family members to the waiting room. e. Observe for signs of hypoxia during the procedure. f. Instruct the patient not to talk during the procedure. g. Position the patient sitting upright with the elbows on an over-the bed table.

e. Observe for signs of hypoxia during the procedure. Observing for hypoxia is done to keep the HCP informed.

The nurse is preparing the patient for and will assist the health care provider with a thoracentesis in the patient's room. Which of the following is the fourth action? a. Verify breath sounds in all fields. b. Obtain the supplies that will be used. c. Send labeled specimen containers to the laboratory. d. Direct the family members to the waiting room. e. Observe for signs of hypoxia during the procedure. f. Instruct the patient not to talk during the procedure. g. Position the patient sitting upright with the elbows on an over-the bed table.

f. Instruct the patient not to talk during the procedure. The patient is instructed not to talk or cough to avoid damage to the lung.

Number the components of normal hemostasis in the order of occurrence, beginning with 1 for the first component and ending with 6 for the last component. a. _______ Adhesion b. _______ Activation c. _______ Aggregation d. _______ Platelet plug formation e. _______ Clot retraction and dissolution f. _______ Vascular injury and subendothelial exposure

f. ___1____ Vascular injury and subendothelial exposure a. ___2____ Adhesion b. ___3____ Activation c. ___4____ Aggregation d. ___5____ Platelet plug formation e. ___6____ Clot retraction and dissolution

Which of these electrolyte imbalances is the cause of "parathyroidectomy cause"? (select all that apply) a. hypernatremia b. hyponatremia c. hyperkalemia d. hypokalemia e. hypercalcemia f. hypocalcemia g. hyperphosphatemia h. hypophosphatemia i. hypermagnesemia j. hypomagnesemia

f. hypocalcemia

Which of these electrolyte imbalances is the cause of "Vitamin D deficiency"? (select all that apply) a. hypernatremia b. hyponatremia c. hyperkalemia d. hypokalemia e. hypercalcemia f. hypocalcemia g. hyperphosphatemia h. hypophosphatemia i. hypermagnesemia j. hypomagnesemia

f. hypocalcemia h. hypophosphatemia

The nurse is preparing the patient for and will assist the health care provider with a thoracentesis in the patient's room. Which of the following is the third action? a. Verify breath sounds in all fields. b. Obtain the supplies that will be used. c. Send labeled specimen containers to the laboratory. d. Direct the family members to the waiting room. e. Observe for signs of hypoxia during the procedure. f. Instruct the patient not to talk during the procedure. g. Position the patient sitting upright with the elbows on an over-the bed table.

g. Position the patient sitting upright with the elbows on an over-the bed table. The patient is positioned.

Number in sequence the events that occur in disseminated intravascular coagulation (DIC). Number the events from 1 through 7, starting with 1. a. ________ Activation of fibrinolytic system b. ________ Uncompensated hemorrhage c. ________ Widespread fibrin and platelet deposition in capillaries and arterioles d. ________ Release of fibrin-split products e. ________ Fibrinogen converted to fibrin f. ________ Inhibition of normal blood clotting g. ________ Production of intravascular thrombin

g. __1______ Production of intravascular thrombin e. ___2_____ Fibrinogen converted to fibrin c. ___3_____ Widespread fibrin and platelet deposition in capillaries and arterioles a. ___4_____ Activation of fibrinolytic system d. ___5_____ Release of fibrin-split products f. ___6_____ Inhibition of normal blood clotting b. ___7_____ Uncompensated hemorrhage

A woman has ham with gravy and green beans cooked with salt pork for dinner. What fluid regulation mechanisms are stimulated by the intake of these foods?

intake of these foods stimulates antidiuretic hormone (ADH) release from the posterior pituitary, which increases water reabsorption from the kidney, lowering the sodium concentration but increasing vascular volume and hydrostatic pressure, perhaps causing fluid shift into interstitial spaces


Conjuntos de estudio relacionados

Chapter 29 (Postpartum) & 30 (Postpartum Complications)

View Set

NETSUITE ERP Consultant exam - Sample

View Set

McKinney Maternal Test Bank Chap 1-30

View Set

ap european history semester 2 final

View Set

SAS Base Programing: Syntax Review

View Set